Sie sind auf Seite 1von 532

7. Calalang v. Williams GR 47800, December 02, 1940, Laurel, J.

(Police Power)
ISSUE:
Whether the prohibition of animal-drawn vehicles from passing along certain streets and locations
during certain times is a valid regulation and exercise of police power.

HELD: YES.
The rules and regulations complained of were promulgated in compliance with law to promote
safe transit upon and avoid obstructions on national roads in the interest and convenience of the
public. In enacting said law, therefore, the National Assembly was prompted by considerations of
public convenience and welfare. It was inspired by a desire to relieve congestion of traffic, which
is a menace to public safety. Thus, public welfare lies at the bottom of the enactment, and the state
in order to promote the general welfare may interfere with personal liberty, property,
businesses, and occupations. Persons and property may be subjected to all kinds of restraints and
burdens in order to secure the general comfort, health, and prosperity of the state. The individual
rights are subordinated to this fundamental governmental aim.

The scope of police power keeps expanding as civilization advances. "The right to exercise the
police power is a continuing one, and a business lawful today may in the future, because of the
changed situation, the growth of population or other causes, become a menace to the public health
and welfare, and be required to yield to the public good."

8. Ichong v. Hernandez, GR L-7995, May 31, 1957, Labrador, J. (Police Power)


ISSUE:
Whether a law (Republic Act 1180) nationalizing retail trade and prohibiting aliens or juridical
persons the capital of which are not wholly owned by Filipinos from engaging in retail trade, is a
valid exercise of police power.
HELD: YES.
Police power is inherent in the State. Constitutions do not grant it, but they set forth the limitations
thereof. The basic limitations on police power are due process and equal protection. The equal
protection clause does not demand absolute equality among residents. It merely requires that all
persons under like circumstances and conditions shall be treated alike.

The due process clause has to do with the reasonableness of legislation enacted in pursuance with
the police power. It inquires whether the law is reasonably necessary for the accomplishment of
the legislature’s purpose, whether the law is arbitrary or oppressive, or whether public welfare is
involved.

The police power legislation must be firmly grounded on public interest and welfare, and a
reasonable relation must exist between purposes and means. And if distinction and classification
has been made, there must be a reasonable basis for said distinction.

The legislature, which is the constitutional repository of police power and exercises the prerogative
of determining the policy of the State, is by force of circumstances primarily the judge of necessity,
adequacy or reasonableness and wisdom, of any law promulgated in the exercise of the police
power, or of the measures adopted to implement the public policy or to achieve public interest.
Courts have only interfered with this exercise of legislative prerogative where there has been a
clear, patent, or palpable arbitrary and unreasonable abuse thereof. Courts can never inquire into
the wisdom of the law.

The retailer is important in the life of the community. Thru him, the infinite variety of articles and
goods needed for daily life are placed within the easy reach of consumers. He is so much a part of
daily existence.

Statistics unmistakably point out to the dominance and control by the alien of the retail trade. The
alien invests more capital, buys and sells six to seven times more and gains much more than the
Filipino. We therefore find that alien domination and control to be a fact, a reality proved by
official statistics, and felt by all sections and groups that compose the Filipino community. It is
this domination and control that is the legislature’s target in enacting the disputed nationalization
law.

There is a prevailing feeling that such predominance may endanger national interest. With ample
capital, unity of purpose and action, and thorough organization, alien retailers and merchants can
act in complete unison and concert on vital matters like fixing of prices, the determination of the
amount of goods or articles to be made available in the market, and the choice of goods they would
or would not sell. Nationals, producers, and consumers can be placed completely at their mercy.
Grave abuses have characterized the exercise of retail trade by aliens. It is a fact within judicial
notice that there is a general feeling on the part of the public that alien participation in retail trade
has been attended by intolerable practices in unlawful restraint of commerce. The present
dominance of the alien retailer would be a potential source of danger on occasions of war of other
calamity. They owe no allegiance or loyalty to the State, and the State cannot rely upon them in
times of crisis or emergency.

Thus, we are satisfied that the law is not a product of racial hostility, prejudice, or discrimination,
but the expression of the legitimate desire and determination of the people through their
authorized representatives. The law is clearly in the interest of the public, of the national
security, and indisputably falls within the scope of police power.

9. Ynot v. Intermediate Appellate Court, GR 74457, March 20, 1987, Cruz, J. (Police Power;
Due process; notice and hearing, but not always necessary; exceptions, if there is immediacy
and urgency to solve the problem)
FACTS:
EO 626-A amends EO 626 to include carabeef in the ban in EO 626 of carabao from being
transported from one province to another. EO 626 also prohibits the slaughter of carabao except if
at least 7 years old if male and 11 if female.

Ynot transported 6 carabaos in a pump boat from Masbate to Iloilo when they were confiscated by
the police for violating EO626-A. Ynot sued for recovery with the RTC. The RTC sustained the
confiscation and declined to rule on the constitutionality of the EO citing lack of authority and its
presumed validity.

The IAC affirmed the RTC ruling. Hence this certiorari petition. Ynot claims that the EO is
unconstitutional as it authorizes the outright confiscation of the carabao or carabeef being
transported across provincial boundaries. He claims that the penalty is invalid as it is imposed
without according the owner the right to be heard as guaranteed by due process.
ISSUE:
1. Whether a law (EO 626) prohibiting the slaughter of carabao except if they are at least seven
years old if male and eleven years old if female is a valid exercise of police power.
2. Whether a law (EO 626-A) prohibiting the transportation of carabao, regardless of age, sex,
physical condition, or purpose, and carabeef from one province to another, and in violation thereof
imposes the penalty of confiscation and forfeiture by the government, is a valid exercise of police
power.
3. Whether EO 626-A violates due process.

HELD:
The due process clause was intentionally kept vague so it would remain resilient. Flexibility must
be the best virtue of guaranty. It was meant to adapt easily to every situation, enlarging or
constricting its protection as the changing times and circumstances may require.

The minimum requirements of due process are notice and hearing intended as a safeguard
against official arbitrariness. There are exceptions, however, like the conclusive presumption
barring admission of contrary evidence if the presumption is bsed on human experience and there
is a rational connection between fact proved and the fact ultimately presumed, or abatement of a
nuisance per se, like a mad dog on the loose which must be killed immediately.

1. YES
The protection of the general welfare is the particular function of the police power which both
restraints and is restrained by due process. The police power is simply defined as the power
inherent in the State to regulate liberty and property for the promotion of the general welfare.

Its reach is virtually limitless. It is a ubiquitous and often unwelcome intrusion. Even so, as long
as the activity or the property has some relevance to the public welfare, its regulation under the
police power is not only proper but necessary. And the justification is found in the venerable Latin
maxims, Salus populi est suprema lex and Sic utere tuo ut alienum non laedas, which call for the
subordination of individual interests to the benefit of the greater number.

The law was issued for the reason, as expressed in its whereas clauses, that “present conditions
demand that the carabaos and the buffaloes be conserved for the benefit of the small farmers who
rely on them for energy needs." We affirm at the outset the need for such a measure. In the face of
the worsening energy crisis and the increased dependence of our farms on these traditional beasts
of burden, the government would have been remiss, indeed, if it had not taken steps to protect and
preserve them.

"To justify the State in thus interposing its authority in behalf of the public, it must appear, first,
that the interests of the public generally, as distinguished from those of a particular class, require
such interference; and second, that the means are reasonably necessary for the accomplishment of
the purpose, and not unduly oppressive upon individuals. The carabao, as the poor man’s tractor,
has a direct relevance to the public welfare. The method is also reasonably necessary for the
purpose sought to be achieved and not unduly oppressive. There is no doubt that by banning the
slaughter of these animals except where they are at least seven years old if male and eleven years
old if female upon issuance of the com necessary permit, the executive order will be conserving
those still fit for farm work or breeding and preventing their improvident depletion.

2. NO.
While EO 626-A has the same lawful subject as EO 626, it does not comply with the second
requirement that there be a lawful method. The object of the prohibition escapes us. The reasonable
connection between the means employed and the purpose sought to be achieved is missing. We do
not see how the prohibition of interprovincial transport of carabaos can prevent their indiscriminate
slaughter considering that they can be killed anywhere.

3. YES.
But even if there was reasonable connection, the law violates due process as the accused is not
accorded opportunity to be heard and is immediately condemned and confers to administrative
authorities the power to adjudge the guilt of the accused. No trial is prescribed and the property
being transported is immediately impounded by police and declared, by the law itself, as forfeited
to the government. In this case, the carabaos were arbitrarily confiscated by the police. The law
defined the prohibition, convicted petitioner and immediately imposed punishment.

While notice and hearing may be validly dispensed with on occasion, and that summary action
may be taken in administrative proceedings as procedural due process is NOT necessarily
judicial only, in these cases there is a justification for the omission of the right to a previous
hearing: immediacy of the problem to be corrected and urgency of the need to correct it. There is
no such pressure of time in this case. There is no reason why the offense should not have been
proved first in a court of justice.

Thus, EO 626-A was declared unconstitutional.

10. Philippine Association of Service Exporters, Inc. v. Drilon, GR 81958, June 30, 1988,
Sarmiento, J. (Police Power; Equal Protection)
ISSUE:
Whether a department order from the Department of Labor and Employment (DOLE) suspending
the deployment of Filipino domestic and household workers abroad is a valid exercise of police
power.
HELD: YES.
It has been defined as the "state authority to enact legislation that may interfere with personal
liberty or property in order to promote the general welfare." As defined, it consists of (1) an
imposition of restraint upon liberty or property, (2) in order to foster the common good. It does
not originate from the Constitution, but it is inborn in the very fact of statehood and sovereignty.
It constitutes an implied limitation on the Bill of Rights.

However, it may not be exercised arbitrarily or unreasonably. When it is used to further the private
interests at the expense of the citizenry, there is a clear misuse of the power.

There is no satisfactory reason why the order should be nullified. While it applies only to “female
contract workers,” it does not make an undue discrimination between the sexes. It is well-settled
that "equality before the law" under the Constitution does not import a perfect identity of rights
among all men and women. It admits of classifications, provided that (1) such classifications rest
on substantial distinctions; (2) they are germane to the purposes of the law; (3) they are not
confined to existing conditions; and (4) they apply equally to all members of the same class. The
classification made, preference for female workers, rests on substantial distinctions. The Court is
aware of the plight of the female labor force abroad amid exploitative working conditions marked
by abuse. But this cannot be said of our male workers. There is no evidence except for isolated
instances that our men have been afflicted with an identical predicament. Distinctions are borne
by the evidence. Discrimination in this case is justified.

It is germane to the purpose of the measure to “enhance the protection for Filipino female overseas
workers.” It does not narrowly apply to existing conditions but is intended to apply indefinitely so
long as those conditions exist. It provides for the lifting of the suspension upon recommendation
of the POEA under certain conditions.

There is also no merit in the contention that DO 1 is an invalid exercise of legislative power. While
police power is the domain of the legislature, it does not mean that such an authority may not be
lawfully delegated. The Labor Code itself vests the DOLE with rule-making powers in the
enforcement thereof.

We do not find the order tainted with grave abuse of discretion to warrant the extraordinary relief
prayed for. The petition is dismissed.

11. Lozano v. Martinez, GR L-63419, December 18, 1986, Yap, J. (Police Power)
ISSUE:
Whether Batas Pambansa (BP) 22, a law punishing a person who makes or draws and issues any
check on account or for value, knowing at the time of issue that he does not have sufficient funds
in or credit with the drawee bank for the payment of said check in full upon presentment, which
check is subsequently dishonored by the drawee bank for insufficiency of funds or credit or would
have been dishonored for the same reason had not the drawer, without any valid reason, ordered
the bank to stop payment or a person who, having sufficient funds in or credit with the drawee
bank when he makes or draws and issues a check, shall fail to keep sufficient funds or to maintain
a credit to cover the full amount of the check if presented within a period of ninety days from the
date on the check which is dishonored by the drawee bank, is a valid exercise of police power.

HELD: YES.
BP22 is aimed at stopping the practice of issuing worthless checks. BP22 punishes the act of
making and issuing a worthless check or a check that is dishonored upon its presentation for
payment. It is not the non-payment of an obligation which the law punishes. It is within the
prerogative of the legislature to proscribe acts deemed pernicious to public welfare. Acts mala in
se are not the only acts which the law can punish. Because of the harm it inflicts on the community,
it can be criminally punished as malum prohibitum. The State can do this in its exercise of police
power.

The enactment of BP22 is a declaration by the legislature that, as a matter of public policy, the
making and issuance of a worthless check is deemed a public nuisance to be abated by the
imposition of penal sanctions. It is sufficient that a reasonable nexus exists between means and
end. It had been reported that the approximate value of bouncing checks per day was close to 200
million pesos, and when overdrafts were banned, between 50 to 80 million.

A check is a bill of exchange drawn on a bank and payable on demand. It partakes of a


representation that the drawer has funds on deposit against which the check is drawn sufficient to
ensure payment upon its presentation to the bank. There is an element of assurance that the
instrument will be paid upon presentation. Thus, checks have become widely accepted as a
medium of payment and although not legal tender, they have come to be perceived as convenient
substitutes for currency in commercial and financial transactions. The basis of such perception is
confidence. If such confidence is shaken, the usefulness of checks as currency substitutes would
be greatly diminished. Any practice therefore tending to destroy that confidence should be
deterred, for the proliferation of worthless checks can only create havoc in trade circles and
the banking community.

Statisitcs show that 1/3 of the country’s money supply consists of peso demand deposits. These
are the funds against, among others, which commercial papers like checks are drawn. The
magnitude of the amount involved justifies the concern of the state to preserve the integrity of the
banking system.

The effects of the issuance of a worthless check transcends the private interests of the parties
directly involved in the transaction and touches the interests of the community at large. The
mischief it creates is not only a wrong to the payee or holder, but also an injury to the public. The
harmful practice of putting valueless commercial papers in circulation, multiplied a thousandf-old,
can very well pollute the channels of trade and commerce, injure the banking system and
eventually hurt the welfare of society and the public interest.

Thus, BP22 is a valid exercise of police power and not repugnant to the constitutional inhibition
against imprisonment for debt.

12. Department of Education, Culture and Sports (DECS) v. San Diego, GR 89572,
December 21, 1989, Cruz, J. (Police Power)
ISSUE:
Whether a DECS rule disallowing a person who has thrice failed the National Medical Admission
Test (NMAT) from taking said test again and thus disallowing his entry into a medical school is a
valid exercise of police power.

HELD: YES.
Police power is validly exercised if (a) the interests of the public generally, as distinguished from
those of a particular class, require the interference of the State, and (b) the means employed are
reasonably necessary to the attainment of the object sought to be accomplished and not unduly
oppressive upon individuals. In other words, the proper exercise of the police power requires the
concurrence of a lawful subject and a lawful method.
The subject of the challenged regulation is certainly within the ambit of the police power. It is the
right and indeed the responsibility of the State to insure that the medical profession is not infiltrated
by incompetents to whom patients may unwarily entrust their lives and health.

The method employed by the challenged regulation is not irrelevant to the purpose of the law nor
is it arbitrary or oppressive. The three-flunk rule is intended to insulate the medical schools and
ultimately the medical profession from the intrusion of those not qualified to be doctors.

While every person is entitled to aspire to be a doctor, he does not have a constitutional right to be
a doctor. This is true of any other calling in which the public interest is involved; and the closer
the link, the longer the bridge to one's ambition. The State has the responsibility to harness its
human resources and to see to it that they are not dissipated or, no less worse, not used at all. These
resources must be applied in a manner that will best promote the common good while also giving
the individual a sense of satisfaction.

13. MMDA v. Garin, GR 130230, April 15, 2005, Chico-Nazario, J. (Police Power)
ISSUE:
Whether Section 5(f) of RA 7924, creating the MMDA, grants the MMDA the power to revoke or
suspend drivers’ licenses without another existing law granting it authority to do so.

HELD: NO.
A license to operate a motor vehicle is a privilege that the state may withhold in the exercise of its
police power. It is not a property right but a privilege that may be suspended or revoked by the
state in the exercise of its police power in the interest of public safety and welfare subject to
procedural due process requirements.

Police power, as an inherent attribute of sovereignty, is the power vested by the Constitution in the
legislature to make, ordain, and establish all manner of wholesome and reasonable laws, statutes,
and ordinances not repugnant to the Constitution as they shall judge to be for the good and welfare
of the commonwealth and its subjects. It cannot be exercised by any group or body of individuals
not possessing legislative power. However, the legislature may delegate this power. Thru the LGC,
Congress delegated police power to local government units (LGUs).

Metro Manila is a body composed of several LGUs. It was declared by RA 7924 as a “special
development and administrative region” and the administration of “metro-wide” basic services
placed under a “development authority,” the MMDA.

RA 7924 does not grant the MMDA police power nor legislative power, not even the Metro Manila
Council. It is a “development authority.” It is an agency created for the purpose of laying down
policies and coordinating with the various national government agencies, people's organizations,
non-governmental organizations and the private sector for the efficient and expeditious delivery
of basic services in the vast metropolitan area. All its functions are administrative in nature. The
MMDA is not a political unit of government. It is not granted authority to enact ordinances and
regulations for the general welfare of the inhabitants of Metro Manila. Thus, the power to
confiscate and suspend or revoke licenses, as understood by the lower court, without need of any
other legislative enactment is an unauthorized exercise of police power.
Section 5 (f) of RA 7924 states that the MMDA shall “install and administer a single ticketing
system,… in the enforcement of such traffic laws and regulations,…” and that it “shall enforce
all traffic laws and regulations in Metro Manila…”

Thus, where there is a traffic law or regulation validly enacted by the legislature or those agencies
to whom legislative powers have been delegated (the City of Manila in this case), the petitioner is
not precluded — and in fact is duty-bound — to confiscate and suspend or revoke drivers' licenses
in the exercise of its mandate of transport and traffic management, as well as the administration
and implementation of all traffic enforcement operations, traffic engineering services and traffic
education programs.

Section 5(f) grants the MMDA the duty to enforce existing traffic rules and regulations.

14. White Light Corporation v. City of Manila, GR 122846, January 20, 2009, Tinga, J.
(Police Power)
ISSUE:
Whether an ordinance issued by the City of Manila (Ordinance 7774) prohibiting short-time
admission, i.e. admittance and charging of room rate for less than 12 hours or renting out of rooms
more than twice a day, of hotels, motels, inns, lodging houses, pension houses, and similar
establishments in Manila, to regulate public morals and illicit activity in transient lodging
establishments is a valid exercise of police power.

HELD: NO.
The ban is sought to be rooted in the police power as conferred on LGUs by the Local Government
Code (LGC) through such implements as the general welfare clause.

The apparent goal of the Ordinance is to minimize if not eliminate the use of the covered
establishments for illicit sex, prostitution, drug use and alike. These goals, by themselves, are
unimpeachable and certainly fall within the ambit of the police power of the State. Yet the
desirability of these ends do not sanctify any and all means for their achievement. Those means
must align with the Constitution, and our emerging sophisticated analysis of its guarantees to the
people.

The standards of judicial review for the validity of an ordinance on substantive due process
grounds in US jurisprudence are strict scrutiny for laws dealing with freedom of the mind or
restricting the political process, the rational basis standard of review for economic legislation, and
the heightened or immediate scrutiny for evaluating classifications based on gender and
legitimacy.

Under rational basis examination, laws or ordinances are upheld if they rationally further a
legitimate governmental interest. Under intermediate review, governmental interest is
extensively examined and the availability of less restrictive measures is considered. Applying
strict scrutiny, the focus is on the presence of compelling, rather than substantial, governmental
interest and on the absence of less restrictive means for achieving that interest.
The rights at stake here fall within the fundamental rights to liberty. Liberty includes the “right to
exist and the right to be free from arbitrary restraint or servitude,” subject only to the restraint as
are necessary for the common welfare. Legitimate sexual behavior among consenting married or
consenting single adults is constitutionally protected. We cannot discount other legitimate
activities which the ordinance would impair. There are legitimate users of wash-rate or renting the
room for more than twice a day. Any person in need of comfortable private spaces for a span of a
few hours for purposes other than sex or using illegal drugs can legitimately look to staying in a
motel or hotel.

It must appear that the interests of the public generally, as distinguished from those of a particular
class, require an interference with private rights and the means must be reasonably necessary for
the accomplishment of the purpose and not unduly oppressive of private rights. It must also be
evident that no other alternative for the accomplishment of the purpose less intrusive of private
rights can work. More importantly, a reasonable relation must exist between the purposes of the
measure and the means employed for its accomplishment. The exercise of police power is subject
to judicial review when life, liberty, or property is affected.

The behavior which the ordinance seeks to curtail is already prohibited and could be diminished
simply by applying existing laws. Less intrusive measures such as curbing the proliferation of
prostitutes and drug dealers through active police work would be more effective in easing the
situation. So would the strict enforcement of existing laws and regulations penalizing prostitution
and drug use. These measures would have minimal intrusion on the businesses of the petitioners
and other legitimate merchants. The ordinance is in effect an arbitrary and whimsical intrusion into
the rights of the establishments as well as their patrons. It needlessly restrains the operation of
businesses and restricting the rights of their patrons without sufficient justification. It rashly
equates wash rates and renting out a room more than twice a day with immorality without
accommodating innocuous intentions.

Ordinance 7774 was declared unconstitutional.

15.City of Manila v. Laguio, GR 118127, April 12, 2005, Tinga, J. (Police Power)
ISSUE:
Whether an ordinance (Ordinance 7783) prohibiting persons, partnerships, corporations, or any
entity from engaging in any business in the Ermita-Malate area providing certain forms of
amusement, entertainment, services, and facilities where women are used as tools in entertainment
and which tend to disturb the community, annoy the inhabitants, ad adversely affect the social and
moral welfare of the community, such as massage parlors, karaoke bars, dance halls, motels, inns,
etc., and providing for padlocking and closure for its violation is a valid exercise of police power
by the City of Manila to protect the social and moral welfare of the community.

HELD: NO.
Ordinance 7783 was passed by the City Council in the exercise of its police power. LGUs are
endowed with police power delegated by Section 16 of the LGC, known as the general welfare
clause. This empowers the City Council or Sangguniang Panlungsod to enact ordinances for the
general welfare of the city and its inhabitants.
Due process is a limitation upon the exercise of police power. This guaranty protects against
arbitrary regulation. This imposes 2 limits on government, procedural and substantive due process.
Procedural due process refers to the procedures that the government must follow before it deprives
a person of life, liberty, or property. Substantive due process asks whether the government has an
adequate reason or sufficient justification for taking away such life, liberty, or property.

For a valid exercise of police power, it must appear that the interests of the public generally, as
distinguished from those of a particular class, require an interference with private rights, and the
means adopted must be reasonably necessary to accomplish the purpose and not unduly
oppressive. It must be evident that no other alternative for the accomplishment of the purpose less
intrusive of private rights can work.

The means employed for the accomplishment of the ordinance’s purpose, the promotion and
protection of the social and moral values of the community, were unreasonable and unduly
oppressive. This aim can be achieved through means less restrictive of private rights and by
reasonable restrictions rather than by an absolute prohibition. The prohibition of the enumerated
establishments will not per se protect and promote the social and moral welfare of the community;
it will not in itself eradicate the alluded social ills of prostitution, adultey, fornication nor will it
arrest the spread of sexual disease in Manila.

Even if the Ermita-Malate area allegedly teems with houses of ill-repute which the City Council
may prohibit, it is baseless to bring within that classification sauna parlors, massage parlors,
karaoke bars, etc., which are lawful pursuits and not per se offensive to the moral welfare of the
community. Sexual immorality may take place in the most innocent of places. The City Council
should instead regulate human conduct that occurs inside the establishments, but not to the
detriment of liberty and privacy. It may impose reasonable regulations such as daily inspections.

Liberty includes the right to exist and the right to be free from arbitrary restraint or servitude. It
includes the right to privacy, the right to be let alone. It includes personal decisions relating to
procreation and marriage, choices central to personal dignity and autonomy. Persons desirous of
owning the establishments prohibited by the ordinance may seek autonomy for these purposes.
Consensual sexual behavior does not contravene the Constitution. Adults have a right to choose to
forge such relationships.

The ordinance is unreasonable and oppressive as it also divests the respondent of beneficial use of
its property. The Ordinance in Section 1 thereof forbids the running of the enumerated businesses
in the Ermita-Malate area and in Section 3 instructs its owners/operators to wind up business
operations or to transfer outside the area or convert said businesses into allowed businesses. An
ordinance which permanently restricts the use of property that it can not be used for any reasonable
purpose goes beyond regulation and must be recognized as a taking of the property without just
compensation. It is intrusive and violative of the private property rights of individuals.

A regulation that permanently denies all economically beneficial or productive use of land is a
taking, if it leaves no reasonable economically viable use of property in a manner that interferes
with reasonable expectations for use. A restriction on use of property may also constitute a "taking"
if not reasonably necessary to the effectuation of a substantial public purpose. Such would require
compensation under the takings clause. The ordinance leave no reasonably economically viable
use of property in a manner that interferes with reasonable expectations for use. The directive to
wind up or transfer are confiscatory. It is a taking.

Ordinances placing restrictions upon the lawful use of property must, in order to be valid and
constitutional, specify the rules and conditions to be observed and conduct to avoid; and must not
admit of the exercise, or of an opportunity for the exercise, of unbridled discretion by the law
enforcers in carrying out its provisions. The Ordinance does not specify the standards to ascertain
which establishments "tend to disturb the community," "annoy the inhabitants," and "adversely
affect the social and moral welfare of the community."

Thus, City of Manila cannot order the closure of the establishments without infringing due process.
They may be regulated, but not prevented from carrying on their business.

16. Ermita-Malate Hotel and Motel Operators Association, Inc. v. City Mayor of Manila, GR
L-24693, July 31, 1967, Fernando, J. (Police Power)
ISSUE:
Whether Ordinance xxxx of the City of Manila requiring motels, hotels, etc. to require customers
to fill up a registration form open to public view at all times, increasing the annual license fees of
hotels and motels, and prohibiting them to lease or rent any room or portion thereof more than
twice every 24 hours, to improve public morals violates due process, and thus is an invalid exercise
of police power.

HELD: NO.
Due process furnishes a standard to which governmental action should conform so that deprivation
of life, liberty, or property be valid. This standard is responsiveness to the supremacy of reason,
obedience to the dictates of justice. Arbitrariness is ruled out and unfairness avoided. It is hostile
to any official action that is unreasonable.

The ordinance was enacted to minimize certain practices hurtful to public morals, such as
prostitution, adultery, and fornication in Manila traceable in great part to the existence of motels
as these provide the necessary atmosphere for clandestine entry, presence, and exit. Requiring the
transients to fill up a registration form open to public view is calculated to shatter the privacy that
characterizes the registration of transients. The increase in the license fees was intended to
discourage establishments of the kind from operating for illegal purposes and to increase the city’s
income. This manifestation of police power is aimed to safeguard public morals and is immune
from an imputation of nullity based on pure conjecture and unsupported by anything of substance.

The municipal board of the City of Manila felt the need for a remedial measure. It is presumed
valid. Thus, a strong case must be found in the records to nullify the ordinance, but there was no
evidence presented in this case.

Municipal license fees could be classified as those imposed for regulating occupations or regular
enterprises for revenue purposes only. Licenses for non-useful occupations are incidental to police
power and the right to exact a fee may be implied from the power to license and regulate. In fixing
the amount of license fees, municipal corporations are allowed a wide discretion as long as such
are not unreasonable, oppressive, or tyrannical. Taxation may be made to implement the state’s
police power. The broad taxing authority of conferred by the Local Autonomy Act of 1959 to cities
and municipalities covers a wide range of subjects with the only limitation that the tax levied be
for a public purpose, just, and uniform.

The prohibition of renting out a room more than twice every 24 hours is neither unreasonable nor
arbitrary. It was intended to curb the opportunity for the immoral or illegitimate use that the
premises could be devoted. There is a correspondence between an undesirable situation and the
legislative attempt at correction. While every regulation of conduct amounts to curtailment of
liberty, liberty is not absolute. The right of the individual is subject to reasonable restraint by
general law for the common good. The citizen’s liberty may be restrained in the interest of public
health, order, and safety.

If the liberty involved were freedom of the mind or the person, the standard for the validity of
governmental acts is much more rigorous and exacting. But where the liberty curtailed affects at
the most rights of property, the permissible scope of regulatory measures is wider.

17. Quezon City v. Hon. Ericta, GR L-34915, June 24, 1983 Gutierrez, Jr., J. (Police Power)
ISSUE:
Whether Section 9 of Ordinance 6118 of Quezon City, setting aside 6% of the total area of a private
memorial park cemetery for charity burial of the dead who are paupers is a valid exercise of police
power.

HELD: NO.
Section 9 cannot be justified under the police power granted by the Charter of Quezon City (RA
537) to regulate business, trades, and occupation practiced in the city. The power to regulate does
not include the power to prohibit or to confiscate. Section 9 not only confiscates but also prohibits
the operation of a memorial park cemetery because the ordinance punishes violation thereof.

Police power is the power to promote public welfare by regulating the use of liberty and property.
If the owner is deprived of his property outright, it is not taken for public use but rather to destroy
to promote the general welfare. The owner does not recover from the government the injury he
sustained. Police power is usually exercised in the form of regulation. It does not involve taking
or confiscation except if necessary to destroy the property to protect and promote general welfare.
Section 9 is a confiscation as it deprives a person of his private property without due process and
even compensation.

There is no reasonable relation between setting aside 6% of the total area of all private cemeteries
for charity burial grounds of deceased paupers and the promotion of the general welfare of the
people. The ordinance is a taking without compensation. Quezon City rely solely on the general
welfare clause in the LGC or on implied powers of the municipal corporation. While this clause is
interpreted broadly and liberally, we cannot stretch it to cover this particular taking.

18. The Office of the Solicitor General v. Ayala Land Incorporated, GR 177056, September
18, 2009, Chico-Nazario, J. (Police Power)
ISSUE:
Whether the State, pursuant to its aim of safeguarding life, health, property, and public welfare,
may validly mandate private malls to provide parking spaces free of charge in the exercise of the
State’s police power.

HELD: NO.
Respondents also incur expenses in the maintenance and operation of the mall parking facilities.
By totally prohibiting them from collecting parking fees from the public for the use of mall parking
facilities, the State would be acting beyond the bounds of police power. Assuming arguendo that
the DPWH secretary and local building officials do have regulatory powers over the collection of
parking fees for privately owned parking facilities, they cannot allow or prohibit such collection
arbitrarily. They must pass the test of reasonableness and propriety of the means in the promotion
of the end sought to be accomplished.

Police power is the power of promoting public welfare by restraining the use of liberty and
property. It is usually exerted to merely regulate the use and enjoyment of the property of the
owner. The power to regulate does not include the power to prohibit nor to confiscate. Police power
does not involve the taking of property except if necessary to destroy it to promote general welfare.

When there is taking for public use, the State is no longer exercising police power but eminent
domain. This enables the State to forcibly acquire private lands intended for public use upon
payment of just compensation. Usually, where title remains with the private owner, that inquiry
should be made whether the impairment of a property is merely regulated or amounts to
compensable taking. A regulation that deprives any person of the profitable use of his property
constitutes a taking and entitles him to compensation unless the invasion of rights is so slight to
permit the regulation to be justified under police power. Similarly, a police regulation that
unreasonably restricts the right to use business property for business purposes amounts to a taking
of private property, and the owner may recover therefor.

In this case, although title or possession of the parking facilities remain with respondents, the
prohibition against the collection of parking fees is tantamount to a taking. This is an excessive
intrusion into the property rights of respondents. Not only are they being deprived of the right to
use a portion of their properties as they wish, they are also prohibited from profiting from its use
or even just to recover expenses from its maintenance.

The total prohibition against the collection by respondents of parking fees from persons who use
the mall parking facilities has no basis in the National Building Code or its IRR. The State also
cannot impose the same prohibition by generally invoking police power, since said prohibition
amounts to a taking of respondents' property without payment of just compensation.

19. Association of Small Landowners in the Philippines, Inc. v. Secretary of Agrarian


Reform, GR 78742, July 14, 1989, Cruz, J. (Police Power)
ISSUE:
Whether a law that 1) prescribes retention limits for landowners and deprives them of whatever
lands they own in excess of the maximum area allowed for purposes of agrarian reform and 2)
providing for manners of payment other than money of just compensation for lands taken from
owners who own lands beyond the retention limit is a valid exercise of police power and the power
of eminent domain.

HELD: YES.
Property condemned under police power is noxious or intended for a noxious purpose which
should be destroyed in the interest of public welfare. In eminent domain, the property involved is
wholesome and intended for a public use. The general rule is that while property may be regulated
to a certain extent, if regulation goes too far it will be recognized as a taking. To the extent that the
law prescribes retention limits for landowners, there is an exercise of police power. Where it
deprives such owners of lands they own in excess of the maximum area allowed, there is a taking
under eminent domain for which payment of just compensation is imperative.

To be valid, the exercise of police power must have a lawful subject and lawful method. The
interests of the public generally as distinguished from those of a particular class require state
interference and the means employed are reasonably necessary to attain the purpose sought to be
achieved and not unduly oppressive. As the subject of agrarian reform have been laid down by the
Constitution itself, the first requirement has been satisfied.

The power of eminent domain is not absolute. The requirements for its proper exercise are 1)
public use and 2) just compensation. Just compensation is defined as the full and fair equivalent
of the property taken from its owner by the expropriator. The measure is not the taker’s gain but
the owner’s loss. The law is more than mere regulation and is an actual taking of private
agricultural lands depriving owners of all its beneficial use and enjoyment. This entitles them to
just compensation. As earlier observed, the requirement for public use has already been settled for
us by the Constitution itself. No less than the 1987 Charter calls for agrarian reform, which is the
reason why private agricultural lands are to be taken from their owners, subject to the prescribed
maximum retention limits.

There is compensable taking when the following conditions concur: (1) the expropriator must enter
a private property; (2) the entry must be for more than a momentary period; (3) the entry must be
under warrant or color of legal authority; (4) the property must be devoted to public use or
otherwise informally appropriated or injuriously affected; and (5) the utilization of the property
for public use must be in such a way as to oust the owner and deprive him of beneficial enjoyment
of the property. All these requisites are envisioned in the measures before us

Where the State is the expropriator, it is not necessary for it to make a deposit upon its taking
possession of the condemned property, as the compensation is a public charge, the good faith of
the public is pledged for its payment, and all the resources of taxation may be employed in raising
the amount.

The determination of just compensation is a function addressed to the courts of justice and may
not be usurped by any other branch or official of the government. The determination thereof by
the Department of Agrarian Reform (DAR) under the law is not final and conclusive. It is only
preliminary unless accepted by all parties concerned.
2. The traditional medium of payment of just compensation is money and no other. However, we
do not deal here with the traditional exercise of the power of eminent domain; there is here a
revolutionary kind of expropriation. The expropriation in this case affects all private agricultural
lands intended to benefit not only a particular community or small segment of the population but
of the entire Filipino nation. Such a program will have tremendous costs. We estimate that
hundreds of billions of pesos will be needed.

The framers are assumed to have been aware of the financial limitations of the government. We
may thus assume that their intention was to allow other manners of payment such as that provided
in the CARP Law, i.e. with other things of value. Nothing in the records of the Constitutional
Commission shows any categorical agreement regarding the meaning of just compensation as
applied to comprehensive agrarian reform, but there is nothing also that militates against the
assumptions we are making of the general sentiments and intention of the members on the manner
of payment to be made to the landowner.

20. Alejandro Manosca v. CA, GR 106440, January 29, 1996, Vitug, J. (Eminent Domain)
ISSUE:
Whether the expropriation of property found to be the birthsite of Felix Manalo, founder of Iglesia
Ni Cristo (INC), a religious entity, and thus declared to be a national historical landmark, to
commemorate Felix’s contribution to Philippine culture, is a valid exercise of the power of eminent
domain.

HELD: YES.
Eminent domain is an inherent power of sovereignty. It need not be clothed with any constitutional
gear to exist; instead, provisions in the Constitution are meant more to regulate rather than grant
the exercise of the power. Eminent domain is generally so described as "the highest and most exact
idea of property remaining in the government" that may be acquired for some public purpose
through a method in the nature of a forced purchase by the State. It is a right to take or reassert
dominion over property within the state for public use or to meet a public exigency. The only direct
constitutional qualification is that "private property shall not be taken for public use without just
compensation."

“Public use” must be considered in its general concept, not having been otherwise defined by the
constitution, of meeting a public need or exigency. “Public use” is one which confers some benefit
or advantage to the public. It is not confined to actual use by the public. As long as public has right
of use, whether exercised by one or many members of public, a 'public advantage' or 'public benefit'
accrues sufficient to constitute a public use. Each member of society need not be equally interested
in such use. The idea that “public use” is strictly limited to clear cases of “use by the public” has
long been discarded. The concept of public welfare is broad and inclusive. It represents spiritual,
physical, aesthetic, and monetary values. Whatever may be beneficially employed for the general
welfare satisfies the requirement of public use.

What is significant is the principal objective of and not the casual consequences that might follow
from the exercise of the power. The purpose in setting up the marker is to recognize the distinctive
contribution of Felix to the culture of the Philippines rather than to commemorate his founding
and leadership of the INC. While greater benefit may be derived by members of INC than by most
others, such peculiar advantage still remains to be merely incidental and secondary. That only a
few would actually benefit from the expropriation of property does not necessarily diminish the
essence and character of public use.

21. Lorenzo Sumulong v. Hon. Guerrero, GR L-48685, September 30, 1987, Cortes, J.
(Eminent Domain)
ISSUE:
1. Whether expropriation for the purpose of “socialized housing” under PD 1224, expanding the
Bagong Nayon Housing Project to provide housing facilities to low-salaried government
employees, is for public use and, therefore, a valid exercise of the power of eminent domain.
2. Whether the lower court judge in issuing a writ of possession in favor of NHA without notice
and hearing violated procedural due process.

HELD:
1. YES.
The exercise of the power of eminent domain is subject to limitations imposed by the constitution
that private property shall not be taken for public use without just compensation and that no person
shall be deprived of property without due process of law.

“Socialized housing” is defined as “the construction of dwelling units for the middle and lower
class members of our society, including the construction of the supporting infrastructure and other
facilities.” This includes slum clearance and resettlement of squatters, slum improvement, and
other activities the objective of which is to provide and maintain housing for the greatest number
of people.

The “public use” requirement for a valid exercise of the power of eminent domain is flexible and
evolving. Whatever may be beneficially employed for the general welfare satisfies the
requirement of public use. To the literal import of the term signifying strict use or employment by
the public has been added the broader notion of indirect public benefit or advantage.

Urban renewal or redevelopment and the construction of low-cost housing is recognized as a public
purpose not only because of the expanded concept of public use but also because of specific
provisions in the Constitution providing that the State shall free the people from poverty thorugh
policies providing for adequate social services (Art.II, Sec.9) and undertake a continuing program
of urban land reform and housing (Art.XIII, Sec.9). Housing is a basic human need. Its shortage is
a matter of state concern as it directly affects public health, safety, the environment, and the general
welfare. Thus, “socialized housing” falls within “public use.”

The State acting through the NHA is vested with broad discretion to designate the particular
property/properties to be taken for socialized housing purposes and how much thereof may be
expropriated. The property owner may not interpose objections merely because in their judgment
some other property would have been more suitable, or just as suitable, for the purpose. The right
to the use, enjoyment and disposal of private property is tempered by and has to yield to the
demands of the common good.

2. YES.
It is violative of due process to deny the owner the opportunity to prove that the valuation in the
tax documents is unfair or wrong. It is imperative that before a writ of possession is issued by the
Court in expropriation proceedings, the following requisites must be met: (1) There must be a
Complaint for expropriation sufficient inform and in substance; (2) A provisional determination
of just compensation for the properties sought to be expropriated must be made by the trial court
on the basis of judicial (not legislative or executive) discretion; and (3) The deposit requirement
under Section 2, Rule 67 must be complied with.

22. Republic v. PLDT company, GR L-18841, January 27, 1969, Reyes, J.B.L., J. (Eminent
Domain)
ISSUE:
Whether the government may compel the PLDT, a telephone company, through the power of
eminent domain, to permit interconnection of government telephone systems and that of PLDT
after the latter severed the government’s connection for serving the general public and private
persons which are competitors of PLDT.

HELD: YES.
While the Republic may not compel the PLDT to celebrate a contract with it, it may, in the exercise
of the sovereign power of eminent domain, require PLDT to allow interconnection of the
government telephone system with that of PLDT subject to payment of just compensation to be
determined by the court. Normally, the power of eminent domain results in the taking or
appropriation of title to, and possession of, the expropriated property; but no cogent reason appears
why such power may not be availed of to impose only a burden upon the owner of condemned
property without loss of title and possession. Private property is subjected to a burden for public
use and benefit. If under Section 6, Article XIII, of the Constitution, the State may, in the interest
of national welfare, transfer utilities to public ownership upon payment of just compensation, there
is no reason why the State may not require a public utility to render services in the general interest,
provided just compensation is paid therefor. Ultimately, the beneficiary of the interconnecting
service would be the users of both telephone systems, so that the condemnation would be for public
use.

The severance of telephone connections of the government telephone systems would isolate the
Philippines from other countries, the Bureau of Telecommunications would be prevented from
properly discharging its functions to the prejudice of the general public.

There is no unfair competition since the demand for telephone service is very much more than the
supposed competitors can supply. PLDT cannot meet the demands for service.

23. People v. Juan Fajardo, GR L-12172, August 29, 1958, Reyes, J.B.L., J. (Eminent
Domain)
ISSUE:
Whether an ordinance stating that “any person or persons who will construct or repair a building
should, before constructing or repairing, obtain a written permit from the municipal mayor”,
penalizing violation thereof and mandates destruction of buildings constructed without permit that
destroys the view of the public plaza or occupies any public property, and pursuant to such
ordinance the mayor denied permit to construct a building on the ground that it would “destroy the
view of the public plaza” is a valid exercise of the power of eminent domain.

HELD: NO.
The ordinance fails to state any policy or standard to guide or limit the mayor’s action. No purpose
to be attained by requiring the permit is expressed and no conditions for its grant or refusal are
enumerated. Standards are entirely lacking. It confers upon the mayor arbitrary and unrestricted
power to grant or deny the issuance of building permits. This is invalid. Municipal ordinances
placing restrictions upon lawful conduct or use of property must specify the rules and conditions
to be observed.

The ordinance permanently deprives appellants of the right to use their own property. It is
unreasonable and oppressive. It oversteps the bounds of police power and amounts to a taking of
appellant’s property without just compensation. While property may be regulated in the interest of
the general welfare and thus the state may prohibit structures offensive to the sight, the State may
not permanently divest owners of the beneficial use of their property and practically confiscate
them solely to preserve the aesthetic appearance of the community.

24. City of Manila v. Chinese Community of Manila et al., GR 14355, October 31, 1919,
Johnson, J. (Eminent Domain)
ISSUE:
Whether in expropriation proceedings, the courts may not inquire into the necessity of such
expropriation, and thus, the expropriation sought by the City of Manila of a cemetery to extend
Rizal Avenue of Manila may not be reviewed by the courts as their role is limited to appraising
the value of just compensation.

HELD: NO.
If the legislature under proper authority should grant the expropriation of a certain or particular
parcel of land for some specified public purpose, the courts would have NO jurisdiction to inquire
into the purpose of that legislation. However, if the legislature should grant general authority to
a municipal corporation to expropriate private land for public purposes, the courts have ample
authority to make inquiry and to hear proof concerning whether the lands were private and if the
purpose was in fact public.

The city of Manila has authority to expropriate private lands for public purposes. The right of
expropriation is not inherent in a municipal corporation. A law must exist conferring the power
upon it. When the courts come to determine the question, they must find not only 1) that a law or
authority exists for the exercise of the right of eminent domain, but 2) also that the right or authority
is being exercised in accordance with law. In this case, the authority conferred upon the City of
Manila has 2 conditions. First, the land must be private, and second, the purpose must be public.
If either condition fails, the exercise is not in accordance with law.

There is a wide distinction between a legislative declaration that a municipality is given authority
to exercise the right of eminent domain and a decision by the municipality that there exists a
necessity for the exercise of that right in a particular case. The first is a declaration simply that
there exist reasons why the right should be conferred upon municipal corporation, while the second
is the application of the right to a particular case. The legislative declaration relating to the
advisability of granting the power cannot be a declaration that a necessity exists for its exercise in
a particular case. Whether it was wise to confer upon a municipality the power to exercise the
right of eminent domain is a question with which the courts are not concerned. But when that right
is exercised for depriving citizens of their property, the courts may hear proof upon the necessity
in the particular case and not the general authority.

The general power to exercise the right of eminent domain must not be confused with the right to
exercise it in a particular case. The power of the legislature to confer, upon municipal corporations
and other entities within the State, general authority to exercise the right of eminent domain cannot
be questioned by the courts, but that general authority of municipalities or entities must not be
confused with the right to exercise it in particular instances. The moment the municipal corporation
or entity attempts to exercise the authority conferred, it must comply with the conditions
accompanying the authority. The necessity for conferring the authority upon a municipal
corporation to exercise the right of eminent domain is admittedly within the power of the
legislature. But whether or not the municipal corporation or entity is exercising the right in a
particular case under the conditions imposed by the general authority, is a question which the
courts have the right to inquire into.

The taking of private property for any use which is not required by the necessities or convenience
of the inhabitants of the state, is an unreasonable exercise of the right of eminent domain, and
beyond the power of the legislature to delegate. As long as there is a constitutional or statutory
provision denying the right to take land for any use other than a public use, the question whether
any particular use is a public one or not is ultimately, at least, a judicial question. The legislature
may declare certain uses to be public, and the courts will sustain such action unless it appears that
the particular use is clearly not of a public nature.

The exercise of the right of eminent domain, whether directly by the State, or by its authorized
agents, is necessarily in derogation of private rights, and the rule in that case is that the authority
must be strictly construed. Not only must the authority of municipal corporations to take property
be expressly conferred and the use for which it is taken specified, but the power, with all
constitutional limitation and directions for its exercise, must be strictly pursued.

The very foundation of the right to exercise eminent domain is a genuine necessity, and that
necessity must be of a public character. The ascertainment of the necessity must precede or
accompany, and not follow, the taking of the land.

In this case, even if there was necessity for opening the street in question, there is no proof of the
necessity of opening the same through the cemetery. Adjoining and adjacent lands have been
offered to the city free of charge which will answer every purpose of the City.

25. Municipality of Parañaque v. VM Realty Corporation, GR 127820, July 20, 1998,


Panganiban, J. (Eminent Domain)
ISSUES:
1. Whether expropriation by a local government unit may be done through a resolution authorizing
its chief executive to initiate expropriation proceedings.
2. Whether the power of eminent domain may be barred by a previous final judgment denying its
exercise by reason of lack of compliance with a legal requirement.

HELD:
1. NO.
The power of eminent domain is lodged in the legislative branch of government, which may
delegate the exercise thereof to LGUs, other public entities and public utilities. An LGU may
therefore exercise the power to expropriate private property only when authorized by Congress
and subject to the latter's control and restraints imposed "through the law conferring the power or
in other legislations." In this case, Section 19 of the LGC (RA 7160), which delegates to LGUs
the power of eminent domain, lays down the parameters for its exercise. It provides that “A local
government unit may, through its chief executive and acting pursuant to an ordinance, exercise
the power of eminent domain for public use…” Thus, the following requisites must concur before
an LGU can exercise the power of eminent domain:
1. An ordinance is enacted by the local legislative council authorizing the local chief
executive, in behalf of the LGU, to exercise the power of eminent domain or pursue
expropriation proceedings over a particular private property.
2. The power of eminent domain is exercised for public use, purpose or welfare, or for the
benefit of the poor and the landless.
3. There is payment of just compensation, as required under Section 9, Article III of the
Constitution, and other pertinent laws.
4. A valid and definite offer has been previously made to the owner of the property sought
to be expropriated, but said offer was not accepted.

In this case, there was no compliance with the first requisite that the mayor be authorized through
an ordinance. A municipal ordinance is different from a resolution. An ordinance is a law, but a
resolution is merely a declaration of the sentiment or opinion of a lawmaking body on a specific
matter. An ordinance possesses a general and permanent character, but a resolution is temporary
in nature. Additionally, the two are enacted differently — a third reading is necessary for an
ordinance, but not for a resolution, unless decided otherwise by a majority of all the Sanggunian
members.

The power of eminent domain involves a derogation of a private right. Thus, the change in the old
LGC (BP337) from “resolution” to “ordinance” under RA 7160 or the current LGC demands a
strict construction.

2. NO.
The principle of res judicata, which finds application in generally all cases and proceedings, 45
cannot bar the right of the State or its agent to expropriate private property. The very nature of
eminent domain, as an inherent power of the State, dictates that the right to exercise the power be
absolute and unfettered even by a prior judgment or res judicata. The scope of eminent domain is
plenary and, like police power, can "reach every form of property which the State might need for
public use." Thus, the State or its authorized agent cannot be forever barred from exercising said
right by reason alone of previous non-compliance with any legal requirement.
26. Philippine Press Institute, Inc. (PPI) v. COMELEC, GR 119694, May 22, 1995, Feliciano,
J. (Eminent Domain, necessity of taking not presumed, must be shown)
ISSUE:
Whether COMELEC Resolution 2772, mandating various publishers of newspapers (members of
PPI), one in each province or city, to provide free of charge a “COMELEC space” of not less than
½ page to the COMELEC to enable candidates for election to make known their qualifications,
stands on public issues, and platforms and programs of government and to disseminate vital
election information, is a valid exercise of the power of eminent domain.

HELD: NO.
To compel print media companies to donate “COMELEC space” of not less than ½ page amounts
to taking of private personal property for public use or purposes. The extent of the taking is not
insubstantial. The monetary value of the compulsory donation, measured by the advertising rates
ordinarily charged by newspaper publishers whether in cities or in non-urban areas, may be very
substantial indeed.

The requisites for a lawful taking are 1) necessity of taking, and 2) legal authority to effect the
taking. The necessity for the taking was not shown by COMELEC. It has not been shown that
members of PPI are unwilling to sell the print space to COMELEC. It has also not been suggested
that COMELEC has been granted the power of eminent domain either by the Constitution or by
legislative authority. A reasonable relationship between that power and the enforcement and
administration of election laws by Comelec must be shown; it is not casually to be assumed.
27. TELEBAP v. COMELEC, GR 132922, April 21, 1998, Mendoza, J. (Eminent Domain;
franchises may be regulated for public purpose; airwaves are not owned by broadcasting
companies, thus they may be compelled to provide airtime for free for the public)
*As distinguished with PLDT case and PLDT’s franchise, PLDT actually owns their
telecommunication systems, thus there must be just compensation
FACTS:
Petitioners Telecommunications and Broadcast Attorneys of the Philippines, Inc. and GMA
Network, Inc. challenge the validity of Section 92 of BP 881, the Omnibus Election Code on the
grounds that 1) it takes property without due process of law and without just compensation, 2) it
denies radio and television broadcast companies the equal protection of the laws, and 3) that it is
in excess of the power of COMELEC to supervise or regulate the operation of media of
communication or information during the period of election.

Sec. 92. Comelec time. — The commission shall procure radio and television time to be known as
"Comelec Time" which shall be allocated equally and impartially among the candidates within the
area of coverage of all radio and television stations. For this purpose, the franchise of all radio
broadcasting and television stations are hereby amended so as to provide radio or television time,
free of charge, during the period of the campaign.

ISSUE:
1. Whether Section 92 of BP 881, authorizing the COMELEC to procure radio and television time,
“COMELEC Time,” which shall be allocated impartially among electoral candidates and for this
purpose amends the franchise of all radio and television stations to provide radio or television time
free of charge during the campaign must entitle the radio and television stations affected to just
compensation for the air time used for “COMELEC Time.”
2. Whether said section violates equal protection for singling out radio and television stations
without including newspapers and magazines.

HELD:
1. NO.
It will be noted that while §90 of B.P. Blg. 881 requires the COMELEC to procure print space
which, as we have held, should be paid for, §92 states that air time shall be procured by the
COMELEC free of charge.

Petitioners contend that Section 92 violates due process and the eminent domain provision by
taking air time from radio and television broadcasting stations without payment of just
compensation. It claims that such would cost it millions of lost revenue.

All broadcasting, whether by radio or by television stations, is licensed by the government.


Airwave frequencies have to be allocated as there are more individuals who want to broadcast than
there are frequencies to assign. A franchise is thus a privilege subject, among other things, to
amended by Congress in accordance with the constitutional provision that "any such franchise or
right granted . . . shall be subject to amendment, alteration or repeal by the Congress when the
common good so requires."

Article XII, Section 11 of the Constitution authorizes amendment of franchises for “the common
good.” What better measure can be conceived for the common good than one for free air time for
the benefit not only of candidates but even more of the public, particularly the voters, so that they
will be fully informed of the issues in an election? "[I]t is the right of the viewers and listeners, not
the right of the broadcasters, which is paramount."

Radio and television broadcasting companies, which are given franchises, DO NOT OWN the
airwaves and frequencies through which they transmit broadcast signals and images. They are
merely given the temporary privilege of using them. The exercise of the franchise privilege may
reasonably be burdened with the performance of some form of public service.

The claim that petitioner would lose P52,380,000 in unrealized revenue is based on the assumption
that air time is “finished product” which become property of the company, like oil from refining
after undergoing a process for production. But air time is NOT OWNED by broadcast companies.
Licenses to broadcast do not confer ownership of designated frequencies, but only the
temporary privilege of using them. The claim that expenses will be incurred for technical
facilities like props, video tapes, etc. has no basis since COMELEC Resolution 2983, Section 6(d)
provides that the candidates would arrange the consideration to be paid therefor with the
radio/television station concerned.

2. NO.
There are important differences in the characteristics of the two media, however, which justify
their differential treatment for free speech purposes. Because of the physical limitations of the
broadcast spectrum, the government must, of necessity, allocate broadcast frequencies to those
wishing to use them. There is no similar justification for government allocation and regulation of
the print media.

In allocating limited resources, relevant conditions may validly be imposed on licensees because
the government spends public funds for the allocation and regulation of the broadcast industry,
which it does not do for print media. Unlike print media like books and newspapers which are
usually found only in metropolitan areas, the television and transistor radio is found everywhere.

28. Export Processing Zone Authority v. Hon. Dulay, GR L-59603, April 29, 1987, Gutierrez,
Jr., J. (Eminent Domain, only court determines just compensation)
ISSUE:
Whether PD 76, 464, 794, and 1533, setting the basis of just compensation in expropriation
proceedings as the fair and current market value declared by the owner of the property sought to
be expropriated or anyone having legal interest in the property or such market value as determined
by the assessor, whichever is lower, is valid and constitutional and thus superseded Rule 67,
Sections 5 to 8 of the Revised Rules of Court on the appointment of commissioners to determine
just compensation.

HELD: NO.
The method of ascertaining just compensation under the aforecited decrees constitutes
impermissible encroachment on judicial prerogatives. It tends to render this Court inutile in a
matter which under the Constitution is reserved to it for final determination. Although the court
technically would still have the power to determine just compensation following the decrees, its
task would be simply stating the lower value of the property as declared either by the owner or
assessor. The court cannot exercise its discretion or independence in determining what is just or
fair. The valuation in the decree may only serve as a guiding principle or one of the factors in
determining just compensation, but it may not substitute the court’s own judgment.

Just compensation means the value of the property at the time of the taking. It means a fair and
full equivalent for the loss sustained. All the facts as to the condition of the property and its
surroundings, its improvements and capabilities, should be considered.

The values given by provincial assessors are usually uniform for very wide areas. The value of
land is based on generalities as its possible cultivation for rice or other crops. Buildings are
described in terms of only two or three classes of building materials and estimates of areas are
more often inaccurate. Tax values can serve as guides but cannot be absolute substitutes for just
compensation. It is violative of due process to deny to the owner the opportunity to prove that the
valuation in the tax documents is unfair or wrong. And it is repulsive to basic concepts of justice
and fairness to allow the haphazard work of a minor bureaucrat or clerk to absolutely prevail over
the judgment of a court promulgated only after expert commissioners have actually viewed the
property, after evidence and arguments pro and con have been presented, and after all factors and
considerations essential to a fair and just determination have been judiciously evaluated.

29. Republic v. BPI, GR 203039, September 11, 2013, Carpio, J. (Eminent Domain, just
compensation and consequential damages)
ISSUE:
Whether in constructing the Zapote-Alabang Fly-Over necessitating the expropriation of portions
of BPI properties, just compensation must include the value of the building as consequential
damages although it is not to be included in the expropriation.

HELD: YES.
The State’s power of eminent domain is limited by the constitutional mandate that private property
shall not be taken for public use without just compensation. Just compensation is the full and fair
equivalent of the property sought to be expropriated. The general rule is that just compensation is
the market value, which is that sum of money which a person desirous but not compelled to buy,
and an owner willing but not compelled to sell, would agree on as a price to be paid by the
buyer and received by the seller. But this general rule is modified where only a part of a property
is expropriated. The owner is not restricted to compensation for the portion actually taken as he is
also entitled to recover consequential damage, if any, to the remaining part of the property.

No actual taking of the bulding is necessary to grant consequential damages. These are awarded if
as a result of expropriation, the remaining property of the owner suffers from an impairment or
decrease in value. (Rule 67, Section 6, RoC)

To determine just compensation, the trial court should first as certain the market value of the
property, to which should be added the consequential damages after deducting therefrom the
consequential benefits which may arise from the expropriation. If the consequential benefits
exceed the consequential damages, these items should bed is regarded altogether as the basic value
of the property should be paid in every case.

30. Republic v. Carmen Vda, GR L-20620, August 15, 1974, Zaldivar, J. (Eminent Domain)
ISSUE:
1. When was the “taking” of Castellvi’s property by the Republic commenced? 1) From 1947
when, by virtue of a lease contract renewable yearly between the two, the Republic, through the
AFP, possessed the property, 2) when, in 1956, Castellvi refused to renew the lease and demanded
the AFP vacate the property, 3) when the Republic filed expropriation proceedings against the
property on June 26, 1959, or 4) when the Republic, on August 10, 1959, was placed by the trial
court in possession of the property?
2. Whether the properties sought to be expropriated may be considered residential lands, and
therefore such fact may be considered in determining just compensation.
3. Whether the just compensation of P10 per square meter as recommended by the commissioners
assigned by the trial court is proper.
HELD:
1. When the Republic filed or commenced expropriation proceedings.
Taking is the entering upon private property for more than a momentary period and under warrant
or color of legal authority, devoting it to a public use or otherwise informally appropriating or
injuriously affecting it as to oust the owner and deprive him of all beneficial enjoyment thereof.
Entry is present when by virtue of the lease agreement between the Republic, through the AFP,
took possession of the property of Castellvi.

“Momentary” when applied to possession of real property should mean “a limited period”, not
permanent. The lease contract was for a period of one year, renewable from year to year, thus the
entry was temporary only, intended to last only a year, regardless of the permanent nature of the
installations constructed by the AFP.

The entry was under warrant of legal authority, the Republic entering the Castellvi property as
lessee. The property being devoted to public use is present as this was used by the AFP air force.

The utilization did not oust Castellvi as the latter remained as owner of the property. Neither was
she deprived of all beneficial use of the property as the Republic was bound to pay and had been
paying the rentals until it filed the complaint for eminent domain on June 26, 1959.

Thus, the taking could not be considered as taking place in 1947. It commenced with the filing of
the complaint for expropriation on June 26, 1959. Under Section 4, Rule 67 of the Rules of Court,
just compensation is to be determined as of the date of the FILING of the complaint. When the
taking coincides with the filing, or takes place subsequent to the filing, just compensation should
be determined as of the filing. The Republic was placed in possession of the Castellvi property by
court authority on August 10, 1959. Thus, the “taking” for purposes of determining just
compensation mustb e reckoned as of June 26, 1959.

2. YES.
In determining the value of land appropriated for public purposes, the same consideration are to
be regarded as in a sale of property between private parties. The inquiry, in such cases, must be
what is the property worth in the market, viewed not merely with reference to the uses to which it
is at the time applied, but with reference to the uses to which it is plainly adapted, that is to say,
What is it worth from its availability for valuable uses? Thus, the owner has a right to its value for
the use for which it would bring the most in the market. The owner may show every advantage
that his property possesses, present and prospective.

The Castellvi property has, since 1957, been classified as residential for tax purposes. Its location
justifies its suitability for a residential subdivision as it is near the entrance of the Basa Air Base
and bounded on two sides by roads. The municipal building, sugar mills, school, and chapel are
near. The same may be said for the lands of Toledo-Gozun that adjoin that of Castellvi. The owners
of these lands have the right to their value for the use for which they would bring the most in the
market at the time of taking.

3. NO.
A court of first instance or, on appeal, the Supreme Court, may change or modify the report of the
commissioners by increasing or reducing the amount of the award if the facts of the case so justify.
While great weight is attached to the report of the commissioners, yet a court may substitute
therefor its estimate of the value of the property as gathered from the record in certain cases, as,
where the commissioners have applied illegal principles to the evidence submitted to them, or
where they have disregarded a clear preponderance of evidence, or where the amount allowed is
either palpably inadequate or excessive. The report of the commissioners are merely advisory.

The commissioners made ocular inspections of the lands and considered the nature and similarities
of such lands in relation to the lands in other places in the province of Pampanga. The price of P10
per square meter is quite high. The price of P5 would be a fair valuation of the lands considering
the resolution of the Provincial Committee on Appraisal of Pampanga informing that in 1959, the
land of Castellvi could be sold from P3 to P4 per square meter, while Toledo-Gozun’s, from P2.5
tp P3. The Court has also taken judicial notice that the value of the Peso has gone down since 1959.

31. National Power Corporation v. Lucman Ibrahim, GR 168732, June 29, 2007, Azcuna, J.
(Eminent Domain, just compensation determined at filing EXCEPT if there is undue
increment from government use of the property)
FACTS:
Ibrahim filed an action for recovery of possession of land and damages before the RTC against
National Power Corporation (NAPOCOR). He claims that they were owners of several parcels of
land of 70,000 m2. NAPOCOR, allegedly through stealth and without their knowledge and consent,
took possession of the sub-terrain area of their lands and constructed underground tunnels in
1978. They discovered the tunnels in July 1992. NAPOCOR uses the tunnels in siphoning water
of Lake Lanao in operating NAPOCOR’s Agus projects.

They demanded NAPOCOR to pay damages and vacate the sub-terrain but the latter refused.
NAPOCOR claims that they were never in possession of the sub-terrain portion of the land. It also
claims that the tunnels are a government project for the benefit of all. The RTC denied the prayer,
but ordered NAPOCOR to pay the Fair Market Value of the lands at P1,000 per square meter,
rental, moral damages and attorney’s fees. The CA affirmed the judgment, but removing the moral
damages.

NAPOCOR appeals to the SC, claiming that Ibrahim and his co-heirs were not denied the
beneficial use of their properties to entitle them to just compensation by way of damages and that
their right to the subsoil does not extend beyond what is necessary to allow them to obtain all the
utility that the property can normally give. It claims that they were able to use the property even
with the tunnels. It also claims that there was no evidence as to the valuation of the property at the
time of its taking in 1978.

ISSUE:
1. Whether the payment of just compensation by NAPOCOR who constructed sub-terrain
underground tunnels 115 meters below the land of Ibrahim is proper
2. Whether the computation of just compensation should be computed from 1978, when the tunnels
were constructed by NAPOCOR under the property of respondents, or as of discovery of such
tunnels by the landowners Ibrahim on 1992.

HELD:
1. YES.
The sub-terrain portion of the property belongs to Ibrahim and his co-heirs by virtue of Article 437
of the NCC:
ART. 437. The owner of a parcel of land is the owner of its surface and of everything under
it, and he can construct thereon any works or make any plantations and excavations which
he may deem proper, without detriment to servitudes and subject to special laws and
ordinances. He cannot complain of the reasonable requirements of aerial navigation.
Thus, the ownership of land extends to the surface as well as to the subsoil under it. The rights
over land are indivisible.

The tunnels deprived respondents of the use of their lands as when they applied for permit to
construct a motorized deep well, they were refused because “the construction of the deep well”
under the land “will cause danger to lives and property.” The tunnel endangered the lives and
properties of respondents as Marawi lies in an area of local volcanic and tectonic activity. The
tunnels deprived them of the lawful use of the land and considerably reduced its value. When
respondents applied for a P2million loan with the Amanah Islamic Bank to be secured by the land,
they were denied because of the tunnel under the land.

Preponderantly, respondents have established the condemnation of their land without even the
benefit of expropriation proceedings or just compensation since 1978.

Landowners cannot be deprived of their right over their land until expropriation proceedings are
instituted in court. The court must see to it that the taking is for public use, that there is just
compensation, and that there is due process. Notwithstanding the fact that petitioner only occupies
the sub-terrain portion, it is liable to pay not merely an easement fee but rather the full
compensation for land. This is so because in this case, the nature of the easement practically
deprives the owners of its normal beneficial use. Respondents, as the owners of the property
thus expropriated, are entitled to a just compensation which should be neither more nor less,
whenever it is possible to make the assessment, than the money equivalent of said property.

2. As of discovery of the tunnels.


Just compensation is the just and complete equivalent of the loss ordinarily determined by referring
to the value of the land and its character at the time it was taken by the expropriating authority.
There is “taking” when the owners are actually deprived or dispossessed of their property,
where there is a practical destruction or a material impairment of the value of their property,
or when they are deprived of the ordinary use thereof. There is taking when the expropriator
enters the private property for a more permanent duration to devote the property to a public use
as to oust the owner and deprive him of all beneficial enjoyment thereof. The entry must be under
warrant or color of legal authority.

The entry is not under warrant or color of legal authority. NAPOCOR justified its nonpayment of
indemnity upon its mistaken belief that the property (subsoil) was public dominion.

The general rule in determining just compensation in eminent domain is the value of the property
as of the date of the filing of the complaint EXCEPT where this Court fixed the value of the
property as of the date it was taken and not as of the filing. The exception applies where the owner
would be given undue incremental advantages arising from the use to which the government
devotes the property expropriated. But in this case, there is no evidence that the valuation of P1,000
per square meter reached in 1992 was due to increments directly caused by petitioner NAPOCOR’s
use of the land. Since petitioner is claiming an exception to Rule 67, Section4, it has the burden to
prove its claim that its occupancy, and not ordinary inflation and increase in land values, was the
direct cause of the increase in valuation from 1978 to 1992.
"A number of circumstances must be present in "taking" of property for purposes of eminent
domain: (1) the expropriator must enter a private property; (2) the entrance into private property
must be for more than a momentary period; (3) the entry into the property should be under warrant
or color of legal authority; (4) the property must be devoted to a public use or otherwise informally
appropriated or injuriously affected; and (5) the utilization of the property for public use must be
in such a way to oust the owner and deprive him of all beneficial enjoyment of the property.

In this case, NAPOCOR’s entrance in 1978 was without intent to expropriate and not made under
color of legal authority as it believed that the land was public. Only in 1990 did it recognize
respondent’s ownership and tried to negotiate for the land’s voluntary purchase. This is not the
expropriation contemplated by law but only an attempt at voluntary purchase and sale. NAPOCOR
neglected to exercise eminent domain.

It was only in 1992 that respondent sued to recover possession and NAPOCOR filed its complaint
to expropriate. If we decree the FMV of the land to be as of 1978, we would be sanctioning a
deceptive scheme whereby NAPOCOR would occupy another’s property and when later pressed
for payment, first negotiate for a low price and then conveniently expropriate when the owner
refuses its offer claiming that the taking should be reckoned as of occupation and that the value of
the property to be computed as of the taking despite the increase in the meantime in the value of
the property.

To allow the valuation to be based on the date the tunnels were constructed would be grossly
unfair. NAPOCOR did not enter the land under warrant of legal authority or with intent to
expropriate. It did not even bother to notify the owners and wrongly assumed it had the right to
dig those tunnels. The tunnels in no way contributed to the increase in the value of the land. Thus,
the computation must be as of 1992, when respondents discovered the construction of the tunnels.

As for the valuation, there is no evidence that the court and the appointed commissioners abused
their authority in evaluating the evidence submitted to them nor misappreciate the clear
preponderance of evidence. The amount of P1000 per square meter is not grossly exorbitant.

32. Republic v. Heirs of Saturnino Borbon, GR 165354, January 12, 2015, Bersamin, J.
(Eminent Domain, dismissal of expropriation proceedings converts it to action for damages;
at any stage of the proceedings, when the public purpose disappears, expropriation case must
be dismissed, but damages for loss of use of property due to taking must be paid)
FACTS:
National Power Corporation (NAPOCOR) is a GOCC with authority under RA 6395 to undertake
the development of hydro-electric generation of power, etc and other works to develop hydraulic
power from any river, lake, creek etc. in PH and to supply power to its inhabitants.

In February 1993, NAPOCOR entered a property in Batangas to construct transmission lines. The
property, with a total area of 14,257 square meters, was owned by heirs of Saturnino Borbon. On
May 26, 1995, NAPOCOR filed a complaint for expropriation to acquire an easement right of way
over a portion of the property involving an area of only 6,326 square meters.
Respondents claim that the area being expropriated covered only the portion affected directly by
the transmission lines but the remaining property was also affected because the line passed through
the center of the land. They claim that the installation of the transmission lines destroyed some
fruit trees and plants without them being paid.

Respondents claimed P550/square meter because the land was industrial at the time of
NAPOCOR’s entry while NAPOCOR objected to the joint report, claiming that the property was
agricultural at the time of its taking in March 1993, stating that it should only pay 10% of the
assessed value in the tax declaration.

The RTC stated that there is no evidence of when entry was made, thus just compensation
should be as of the filing of the complaint on May 05, 1995. Thus, it ordered just compensation
of P550/square meter and legal interest from May 05, 1995 until full payment. The CA affirmed
this, but it applied the P550 just compensation only to the 6,326 square meters actually occupied.
NAPOCOR appealed.

During the pendency of this appeal, NAPOCOR moved to defer proceedings, stating that they were
negotiating to amicably settle. On January 3, 2014, NAPOCOR moved to discontinue
expropriation proceedings stating that the parties failed to reach an agreement, that the property
sought to be expropriated was no longer necessary for public purpose because of the intervening
retirement of the transmission lines installed on the respondents’ property. Thus they prayed that
the compensation to be awarded to respondents be reduced by the equivalent of the benefit they
received from the land during occupation.
ISSUE:
Whether the expropriation proceedings filed by NAPOCOR, a GOCC, may be discontinued on the
ground that the property to be expropriated is no longer necessary for any public purpose since the
transmission lines used on the property sought to be expropriated have been retired.

HELD: YES, BUT upon such terms as the court deems just and equitable.
The exercise of eminent domain is limited by the mandatory requirements 1) that is is for a public
purpose, and 2) that just compensation be paid to the property owner. These are implied conditions
that should be complied with. Public use, in common acceptation, means “use by the public.” But
it has been expanded to include utility, advantage, or productivity for the benefit of the public.
Public use has been held to be synonymous with “public interest,” “public benefit,” and “public
convenience.” It is essential that the element of public use be maintained throughout the
proceedings for expropriation. If not, the expropriator must return the property to its private
owner, if the latter desires to reacquire it.

Public use is the fundamental basis of an action for expropriation. The moment that it appears at
any stage of the proceedings that the expropriation is not for a public use, the action must be
dismissed because the action cannot be maintained except when the expropriation is for some
public use. This is true even during pendency of appeal or at any stage of the proceedings. Thus,
NAPOCOR’s motion to discontinue proceedings should be granted. The retirement of the
transmission lines stripped the expropriation proceedings of the element of public use.
Thus, the Court grants the motion and requires the return of the property to respondents. However,
NAPOCOR entered the property without the owner’s consent and without paying just
compensation. It did not deposit any amount as required before entry. Considering that NAPOCOR
destroyed some fruit trees and plants without payment and the transmission lines went through the
middle of the land as to divide the property into three lots, rendering the entire property inutile for
any future use, NAPOCOR is liable for the disturbance of respondent’s property rights from entry
until restoration. There is showing that NAPOCOR entered and took possession of respondent’s
property as early as March 1993 without filing a petition for eminent domain. Thus, March 1993
is the reckoning point of NAPOCOR’s taking instead of May 5, 1995, which is the time of filing.

Normally, where the filing precedes taking, just compensation is fixed as of the filing. The
assumption of possession by the expropriator is conditioned on its deposits of the provisional value
of the property. However, where taking precedes filing, just compensation should be determined
as of the time of taking, not of filing.

There is a "taking" when the owner is actually deprived or dispossessed of his property; when there
is a practical destruction or a material impairment of the value of his property or when he is
deprived of the ordinary use thereof. There is a "taking" in this sense when the expropriator enters
private property not only for a momentary period but for a more permanent duration, for the
purpose of devoting the property to a public use in such a manner as to oust the owner and deprive
him of all beneficial enjoyment thereof.

In view of the discontinuance of the proceedings, there is no need to pay just compensation
because their property would not be taken by NAPOCOR. Instead of full market value,
NAPOCOR should compensate the respondents for the disturbance of their property rights
from the time of entry in March 1993 until the time of restoration of possession by paying
ACTUAL or other COMPENSATORY DAMAGES.

If the public purpose of the taking is not initiated or not at all pursued or is peremptorily
abandoned, then the former owners, if they so desire, may seek reversion of the property
subject to return of just compensation received. Compensation must be based on what they
actually lost as a result or by reason of their dispossession of the property and of its use, including
the value of the fruit trees, plants and crops destroyed by NAPOCOR’s construction of the
transmission lines. The Court now treats the dismissal of expropriation proceedings as converting
the case into an action for damages. The Court remands the case to the court of origin for further
proceedings.

33. Secretary of DPWH v. Spouses Heracleo and Ramona Tecson, GR 179334, July 1, 2013,
Peralta, J. (Eminent Domain, belated claim for just compensation coupled with taking
without filing expropriation proceedings = just compensation as of taking no matter how
inequitable, cruel effects of belated claim)
FACTS:
Respondent spouses are co-owners of a parcel of land. This was among the properties taken by the
government sometime in 1940 without the owner’s consent and without expropriation proceedings
and used to construct the MacArthur Highway. Respondents on December 15, 1994 demanded
payment of the fair market value (FMV) of the land. The DPWH offered to pay P0.7 per square
meter, but the respondents demanded for just compensation at the land’s current FMV. This was
unheeded, thus they filed a complaint for recovery of possession with damages against DPWH.
They claim that the land was assessed at P2,543,800. Petitioners moved to dismiss.

The RTC granted the dismissal on the basis of state immunity from suit. The CA ruled that state
immunity is not applicable as the recovery of compensation is the only relief available to the
landowner, to deny which would cause injustice to him. It remanded the case to the trial court for
determination of just compensation. The RTC then held just compensation to be at P1,500 per
square meter. The CA affirmed this, adding 6% interest per annum from the filing of the action on
March 17, 1995 until full payment. The CA found that it would be unjust if the compensation be
based on the value on 1940 of P0.7/ sq. m, finding it necessary to deviate from the general rule.
Thus, it determined compensation based on the time of payment.

ISSUE:
Whether just compensation should be determined at the time of taking in 1940, thus only P0.7/sq
m should be rewarded, or at the time of filing of complaint on March 17, 1995 until full payment
with interest, thus P1,500/sq m must be awarded where the property owner failed for 50 years to
claim just compensation and where the taking was done in 1940 without the government filing
expropriation proceedings.

HELD: At the time of taking in 1940.


After more than 50 years, the owners seek recovery of possession of their property. Where private
property is taken by the Government for public use without first acquiring title thereto either
through expropriation or negotiated sale, the action to recover the land or its value does not
prescribe. For failure to question the lack of expropriation proceedings for a long period,
respondents are deemed to have waived and are estopped from assailing the power of the
government to expropriate or the public use for which the power was exercised. What is left to
respondents is the right of compensation.

Just compensation is "the fair value of the property as between one who receives, and one who
desires to sell, x x x fixed at the time of the actual taking by the government." This rule holds true
when the property is taken before the filing of an expropriation suit, and even if it is the property
owner who brings the action for compensation

Just compensation in this case should be fixed not as of the time of payment but at the time of
taking, in 1940. Where property is taken ahead of the filing of condemnation proceedings, the
value thereof may be enhanced by the public purpose for which it is taken. The entry may have
depreciated its value, or there may have been a natural increase in value from the taking to the
filing due to general economic conditions. The owner of the property should be compensated only
for what he actually loses. Compensation is not intended to extend beyond his loss or injury, and
what he loses is only the actual value of his property at the time of taking.

While the P0.7/sq m may appear inequitable, it is equally true that they too are remiss in guarding
against the cruel effects of belated claim. Just compensation does not imply fairness to the property
owner alone. Compensation must be just not only to the owner, but also to the public.
Petitioners had been occupying the property for more than 50 years without expropriation
proceedings. Thus, respondents are entitled to adequate compensatory or actual damages which
should be the legal interest of six percent per annum(6%) on the value of the land at the time of
taking until full payment. This is based on the principle that interest runs as a matter of law and
follows from the right of the landowner to be placed in as good position as money can accomplish,
as of the date of taking

33.1 April 21, 2015, Peralta, J., (Eminent Domain, Interest computation, not inequitable or
unconscionable since based on law and jurisprudence) (Motion for Reconsideration of
Secretary of DPWH v. Spouses Tecson)
FACTS:
Respondents insist that gross injustice will result if the amount awarded today will be based simply
on the value of the property at the time of the actual taking. They suggest a happy middle ground
by meeting the need for doctrinal precision and the thirst for substantial justice.

ISSUE:
1. Whether the just compensation must be based on the time of taking in 1940, or the time of filing
of expropriation proceedings on March 17, 1995 where the taking was done without filing of
expropriation proceedings and where the property owner did not claim just compensation for 50
years.
2. Whether the interest is 6% or 12% and whether such must be compounded.
HELD:
1. The reckoning date for property valuation in 1940 has been squarely addressed in the assailed
decision. The State is not obliged to pay premium to the property owner for appropriating his
property. It is only bound to make good the loss sustained by the landowner with due circumstances
availing at the time the property was taken. Compensation must be fair not only to the owner but
also to the public.

2. Notwithstanding, we recognize that the owner’s loss is not only his property but also its
income-generating potential. Thus, when property is taken, full compensation of its value must
immediately be paid to achieve a fair exchange for the property and the potential income lost. The
rationale of imposing interest is to compensate for the income they would have made had they
been properly compensated for their properties at the time of taking. Just compensation due
to landowners amounts to an effective forbearance on the part of the State.

On May 1, 1916, Act 2655 prescribed an interest of 6% or such rate as may be prescribed by the
Central Bank Monetary Board (CB-MB) for loans or forbearance of money in the absence of
express stipulation as to such rate of interest:
Section 1. The rate of interest for the loan or forbearance of any money goods, or credits
and the rate allowed in judgments, in the absence of express contract as to such rate of
interest, shall be six per centum per annum or such rate as may be prescribed by the
Monetary Board of the Central Bank of the Philippines for that purpose in accordance
with the authority hereby granted.
Sec. 1-a. The Monetary Board is hereby authorized to prescribe the maximum rate or rates
of interest for the loan or renewal thereof or the forbearance of any money, goods or credits,
and to change such rate or rates whenever warranted by prevailing economic and social
conditions.

On July 29, 1974, CB-MB, pursuant to the authority granted to it under Act 2655, issued
Resolution 1622 and Circular 416, implementing resolution 1622, increasing the rate of interest
for loans and forbearance of money to 12% per annum in the absence of express contract as to
such rate of interest. CB 905, which took effect on December 22, 1982, sustained the 12% interest
rate.

The BSP Monetary Board (BSP-MB), in Resolution 796 dated May 16, 2013, amended CB 905,
s1982 and issued CB 799 s2013, effective July 1, 2013, changing the prevailing interest rate for
loans and forbearance of money to 6% per annum in the absence of express contract as to such rate
of interest:
Section 1. The rate of interest for the loan or forbearance of any money, goods or
credits and the rate allowed in judgments, in the absence of an express contract as to
such rate of interest, shall be six percent (6%) per annum.

Pursuant to Article 2212 of the NCC, interest shall be compounded at the time of judicial
demand. The interest due shall itself earn legal interest from the time it is judicially demanded.

The exact date of taking cannot be determined, thus is deemed to be on January 1, 1940. The land
area is 7,268 square meters. Thus:
7,268*0.7 = P5,087.60
1. January 1, 1940 to July 28, 1974 = (5,087.60 *6% *34 years) + (5,087.60 * 6% * 209/365days)
= P10,553.49

2. July 29, 1974 to March 16, 1995 = (10553.49 * 12% * 20 years) + (10553.49 * 12% * 155/365)
+ (10553.49 * 12% * 75/365) = P26,126.31 (Judicial demand was March 17, 1995, thus
thereafter the interests will be compounded)

3. March 17, 1995 to June 30, 2013 = (26126.31 * (1+1%) *219.5 months) = P232,070.33, (The
rate of interest annually is 12%, divided by 12 to get the monthly interest of 1% multiplied
by the months. The equation to get the monthly compounded interest is P(1+m) t where
P=starting/average balance, m= monthly interest, t= months)

4. July 1, 2013 to September 30, 2014 = (232,070.33 * (1+0.5%) * 15months) = P250,098.19 (The
6% interest is divided by 12 months also)
5, Adding 1-4 = P518,848.32 is the market value at the time of taking including interest.

6. Adding from this: Exemplary Damages = P1million; Attorney’s Fees = P200,000

The Total amount of interest due to respondents as of September 30, 2014 is P1,718,848.16

Upon finality of this resolution until full payment, the total amount due shall earn a straight 6%
legal interest pursuant to CB799. This is in the nature of a judicial debt.

The award of interest on the value of the land at the time of taking in 1940 until full payment is
adequate compensation. It cannot be inequitable and unconscionable since it resulted from
application of law and jurisprudence. Thus, adding the interest computed to the market value of
the property at the time of taking signifies the real, substantial, full and ample value of the property.
Verily, the same constitutes due compliance with the constitutional mandate on eminent domain
and serves as a basic measure of fairness. In addition to the foregoing interest, additional
compensation shall be awarded to respondents-movants by way of exemplary damages and
attorney's fees in view of the government's taking without the benefit of expropriation proceedings.
An irregulatiry in an expropriation proceeding cannot ensue without consequence.

The award of interest renders unwarranted the grant of back rentals because if the condemnor is to
pay the compensation as of the time of taking, the payment of compensation is deemed to retroact
to the actual taking. Thus, there is no basis to claim rentals from the taking. The compensation
must not be in the form of rentals but by way of interest.

The failure of the government to initiate expropriation proceeding to the prejudice of the landowner
may be corrected with the awarding of exemplary damages, attorney’s fees, and costs of
litigation. Considering that respondents were deprived of beneficial ownership over their property
for more than 70 years without the benefit of a timely expropriation proceeding, and to deter the
State from failing to institute such proceeding, exemplary damages of P1,000,000 is fair. An award
for attorney’s fees of P200,000 is in order.

34. Republic v. Rural Bank of Kabacan, Inc., GR 185124, January 25, 2012, Sereno, J.
(Eminent Domain, excavated soil is part of value of land as property ownership is indivisible;
when ownership of property expropriated is uncertain, follow Section9 of Rule 67)
FACTS:
National Irrigation Administration (NIA) is a GOCC created under RA 3601. It is primarily
responsible for irrigation development and management in the country. Under PD 552, to carry
out its purpose, NIA was authorized to exercise the power of eminent domain.

NIA needed land to construct Malitubog-Marigadao Irrigation Project. On September 08, 1994, it
filed with the RTC a complaint for expropriation of a portion of 3 parcels of land of 14,497.91 sq
m. The lands affected were Lot 3080 of Rural Bank of Kabacan, Lot 455 of the Lao’s, and Lot
3039 of Littie Sarah Agdeppa. NIA prays that it be allowed to take possession after depositing
with the PNB P19,246.58, representing the provisional value thereof.
Respondents allege that NIA has no authority to expropriate and that it was not necessary to
expropriate their properties as there was an abandoned government property adjacent to theirs, that
Lot 3080 was no longer owned by Rural Bank of Kabacan, that NIA’s valuation is inaccurate, and
that NIA never negotiated before taking the properties, causing damages. Intervenors Margarita
Tabaoda and Portia Charisma Ruth Ortiz claim to be the new owners of Lot 3080.

The RTC issued a writ of possession after the committee of commissioners submitted its
Commissioners’ Report, stating that the area to be occupied is 18,930 sq m, fully planted with
gmelina trees, with 220 banana clumps, and 15 coco trees. But the committee could not agree on
the FMV, so a new committee was made.

The new committee found the FMV to be P65/sq m based on the zonal valuation of the BIR. For
the improvements, they recommended P200 for each gmelina tree more than 4 years old and P150
if more than 1 year old, P164 for each coco tree, and P270 for each banana clump. It included the
value of the earthfill excavated from portions of Lots 3080 and 3039. NIA objected to the
inclusion of excavated soil.

The RTC adopted the commissioners report, including compensation for earthfill. The CA ruled
that the commissioners properly performed their function, but it deleted the value of soil excavated
in the just compensation, stating that the owner was entitled to compensation only for the value of
the property at the time of the taking. Hence this appeal by NIA, claiming that the CA erred in
affirming the RTC of just compensation of the land and improvements thereon based on the
Commissioner’s Report.

ISSUE:
1. Whether the commissioner report, based on, for the valuation of the property, ocular inspections,
BIR data on zonal valuation, interview of adjacent property owners, distance from highway and
town center, and, for the valuation of the improvements, based on valuation of the provincial
assessor, the age of the trees, and their inputs and productivity, may be set aside.
2. Whether it was established that Lot 3080 was no longer owned by Rural Bank of Kabacan and
is now owned by Intervenors Margarita Tabaoda and Portia Charisma Ruth Ortiz, thus the just
compensation must then be paid to them instead.

HELD:
1. NO. Just compensation is defined as the full and fair equivalent of the property taken from its
owner by the expropriator. The measure is not the taker's gain, but the owner's loss. The word just
is used to intensify the meaning of the word compensation and to convey thereby the idea that the
equivalent to be rendered for the property to be taken shall be real, substantial, full and ample. It
is the sum equivalent of the market value of the property or the fair value of the property as between
one who receives and one who desires to sell it, fixed at the time of actual taking by the
government.

The commissioners properly determined the just compensation to be awarded. The RTC followed
the procedure under Rule 67 of the 1997 Rules of Civil Procedure when it formed a committee to
determine the just compensation. The first set of commissioners made an ocular inspection of the
properties, determined the exact areas affected and the improvements thereon. When they were
unable to agree on the valuation, the RTC selected another batch of disinterested members to
determine such value. The new committee also made a second ocular inspection of the property.
They obtained data from the BIR to determine the zonal valuation of the properties, interviewed
the adjacent property owners, and considered other factors such as distance from the highway and
the nearby town center. From these, the committee members endeavored a rigorous process to
determine just compensation.

The just-ness of the compensation may be attained only by using reliable and actual data. The
committee in this case based their recommendations on reliable data and considered various factors
that affected the value of land and the improvements.

We also uphold the deletion of the value of excavated soil in the payment of just compensation.
There is no legal basis to separate the value of excavated soil from that of expropriated properties.
In the context of expropriation proceedings, the soil has no value separate from that of
expropriated land. Just compensation ordinarily refers to the value of the land to compensate for
what the owner actually loses. Rights over lands are indivisible. The law does not limit use of
expropriated land to the surface only. To allow payment for the excavated soil is to allow the
landowners to recover more than the value of the land at the time of taking.

2. NO.
It was imprudent to rely only on the mere declaration of Rural Bank of Kabacan of non-ownership
and non-participation in the expropriation proceeding to validate the intervenors’ claim of
entitlement to payment. For conveyance of real property to be valid, it should be embodied in a
public instrument and registered in the office of the Register of Deeds where the property is
situated. There was no proof of conveyance presented. The trial court should have still required
the rural bank and intervenors to show proof of conveyance. The court cannot rely on mere
inference, considering that the payment of just compensation is intended to be awarded solely
owner based on the latters proof of ownership.

The trial court should have been guided by Rule 67, Section 9 of the 1997 Rules of Court:
SEC. 9. Uncertain ownership; conflicting claims. If the ownership of the property taken is
uncertain, or there are conflicting claims to any part thereof, the court may order any sum
or sums awarded as compensation for the property to be paid to the court for the benefit
of the person adjudged in the same proceeding to be entitled thereto. But the judgment shall
require the payment of the sum or sums awarded to either the defendant or the court before
the plaintiff can enter upon the property, or retain it for the public use or purpose if entry
has already been made.
There is doubt as to the real owner of Lot 3080. Despite the title to it still being registered under
its name, the Rural Bank manifested that it was no longer the owner but presented no proof of
conveyance. Thus we remand the case to the trial court for reception of evidence to establish the
present owner of Lot 3080 and who will be entitled to receive payment of just compensation.

35. Victoria Amigable v. Nicolas Cuenca, GR L-26400, February 29, 1972, Makalintal, J.
(Eminent Domain, government immunity from suit doesn’t apply when it takes property
without expropriation proceedings)
FACTS:
Amigable owns Lot 639. Without expropriation or negotiated sale, the government used a portion
of said lot with an area of 6,167 sq m for the construction of the Mango and Gorordo Avenues.
Said avenues already existed since 1921 although “they were in bad condition and very narrow,
unlike the wide and beautiful avenues that they are now.” The tracing of the roads begun in 1924,
and formal construction began in 1925.

On March 27, 1958, Amigable’s counsel wrote the president requesting payment of the portion of
the lot appropriated. The Auditor General disallowed it.

On February 06, 1959, Amigable filed a complaint against the Republic and Nicolas Cuenca as
Commissioner of Public Highways for recovery of ownership and possession of the 6,167 sq m
land and for compensatory damages, moral damages, attorney’s fees, and costs. The trial court
ruled that the government did not consent to be sued, thus it had no jurisdiction. Hence this appeal.

ISSUE:
Whether a private property owner whose property had been used by the government without
expropriation proceedings to construct roads may sue the government for compensation and
damages without the government’s consent to be sued.

HELD: YES.
Where the government takes away property from a private landowner for public use without the
legal process of expropriation or negotiated sale, the aggrieved party may properly maintain a suit
against the government without violating the doctrine of governmental immunity from suit without
its consent. This doctrine cannot be used to perpetrate injustice on a citizen. It is unthinkable that
because of failure to abide by what the law requires, the government would stand to benefit. When
the government takes any property for public use, which is conditioned upon payment of just
compensation to be judicially ascertained, it makes manifest that it submits to the jurisdiction
of a court.

Since there is no annotation on the certificate of title in favor of the government, Amigable remains
the owner. She could bring action to recover possession, but since such is neither convenient nor
feasible because the property is now used for road purposes, the only relief available is due
compensation to be determined as of the time of taking. As for damages, Amigable is entitled in
the form of legal interest from taking until payment. The government should also pay attorney’s
fees.

The case was remanded to the trial court to determine compensation and attorney’s fees. No costs.

36. National Power Corporation (NAPOCOR) v. Socorro Posada, GR 191945, March 11,
2015, Leonen, J. (Eminent Domain; RA 8974 governs expropriation for national government
infrastructure project; withdrawal of expropriation proceeding not automatic, must file
motion to withdraw with trial court to determine damages)
FACTS:
NAPOCOR filed expropriation proceedings for acquisition of right-of-way easement over land
owned by respondents Pasada et al. It was for the construction and maintenance of its Substation
Island Grid Project. NAPOCOR offered P500/sq m, while respondents wanted P2000/sq m.
The commissioners recommended a fair market value of P1,500/sqm. NAPOCOR objected. The
RTC issued a writ of possession. Respondents moved to grant them time to remove their houses
and improvements as well as for additional deposit for use in land acquisition and expenses for
such transfer of houses, which was granted. The RTC ordered NAPOCOR to make additional
deposit.

The RTC fixed just compensation at P2000/sqm. It also recalled the writ of possession for failure
of NAPOCOR to make the additional deposit. The CA denied NAPOCOR’s appeal. Thus, it filed
a petition for review on certiorari before the SC.

NAPOCOR informed its counsel on July 24, 2014 that it no longer needed the properties as it was
set to acquire an alternative site. Thus, NAPOCOR filed the present Motion to Withdraw Appeal,
praying for the withdrawal of its appeal before the Court.

ISSUE:
1. Whether the trial court, in issuing a writ of possession in favor of NAPOCOR upon the latter’s
deposit to a government authorized depositary of the provisional value of the infrastructure
property, acted properly.
2. Whether the recall of the writ of possession amounts to a prohibit injunction under Section 3 of
RA 8975 (not 8974).
3. Whether the withdrawal by NAPOCOR of its petition for review of a decision of the CA
affirming the decision of the trial court, which recalled the writ of possession issued in favor of
NAPOCOR, on the ground that NAPOCOR no longer needs the property to be expropriated for a
public use and it will purchase another lot for that purpose.
HELD:
1. NO.
Expropriation proceedings for national infrastructure projects are governed by Rule 67 of the Rules
of Court and Republic Act No. 8974.

Expropriation proceedings undergo 2 phases under Rule 67: 1) determining the propriety of the
action; 2) determining the just compensation. The first phase ends with an order, if not dismissal,
of condemnation declaring that the plaintiff has a lawful right to take the property for the public
use described in the complaint upon payment of just compensation. The second phase is done with
the assistance of 3 commissioners. The order fixing just compensation based on evidence and the
findings of commissioners would be final as it would dispose of the second phase.
The first phase commences with the filing and ends with the order to proceed with expropriation
and determination of just compensation.

RA 8974, enacted on November 7, 2000, also governs expropriation of private property for
national government infrastructure projects. It qualifies the manner by which the government
may enter and take possession of the property to be expropriated. Section 2 of Rule 67 states that
entry and possession of real property is allowed only upon deposit of the assessed value for tax
purposes. Section 4 of RA 8974 states that upon filing, payment must immediately be made
equivalent to 100% of the property value based on current relevant zonal valuation of the BIR
and the value of the improvements.
RA 8974 is more favorable to the owner than Rule 67. NAPOCOR must comply with RA 8974
before possession as projects relating to “power generation” are national infrastructure projects.
The trial court erred when it based the value of the improvements on the property on the
determination of commissioners, not by NAPOCOR as required by Section 7 of RA 8974 and
Section 10 of its IRR. According to the law, it is the implementing agency, not commissioners,
that determines the proffered value of the improvements and structures. A writ of possession may
be issued once there is confirmation by the trial court of the proffered value.

Another error is when it issued the writ of possession based on NAPOCOR’s deposit with the Land
Bank, not its actual payment to respondents. Section 4 of RA 8974 requires direct payment to the
property owner, not mere deposit with the authorized government depositary. It is only when
immediate payment cannot be made even if the implementing agency is willing to do so, such as
when the owner contests the power of such agency to exercise eminent domain or the proffered
value, may the agency deposit the proffered value with the trial court.

2. NO.
NAPOCOR argues that the project cannot be restrained by the recall of a previously issued writ of
possession because this would amount to an injunctive writ prohibited by Section 4 of RA 8975.

Section 3 states that no court except the SC may issue any preliminary injunction against the
government or its subdivisions to restrain acquisition, clearance, and development of right-of-way
of any national government project. The recall is not the same as issuance of an injunctive writ.
The first is an action by the trial court to correct an erroneous issuance while the second is an
ancillary remedy to preserve rights. The object of a preliminary injunction is to preserve the status
quo until final judgment. The recalling does not revert the parties to the status quo. It was
NAPOCOR’s own acts that prevented it from implementing its infrastructure project.

- On difference of just compensation and the provisional value under RA 8974


Just compensation required by the Constitution is different from the provisional value required by
RA 8974. The trial court does not exercise its discretion in determining the provisional value as
the legislature has fixed the amount under Section 4 of RA 8974. The provisional value is based
on the current zonal valuation while just compensation is based on the prevailing FMV. The
provisional value serves the purpose of prepayment if the property is expropriated and of
indemnity for damages if the proceedings are dismissed. It may be a factor considered in
determining just compensation.

Just compensation is the final determination of the FMV of the property. It is the just and complete
equivalent of the loss of the owner by reason of expropriation.

The statutory requirement to pay a provisional amount equivalent to the full BIR zonal valuation
does not substitute for the judicial determination of just compensation. No statute, decree, or
executive order can mandate that its own determination prevail over the court’s findings.
*Section 19 of LGC applies to LGUs, while RA 8974 to government instrumentalities? Or to
infrastructure projects regardless of which entity? – ONLY NATIONAL government
infrastructure projects.

3. YES.
The State has the burden of proving the public purpose of taking. If it fails to prove such, it must
return the property subject to damages incurred in the course of taking. Matters involving dismissal
of an expropriation case or the return of expropriated property must be determined on a case-to-
case basis. No expropriation proceeding can continue if the property to be expropriated will not be
for public use.

Since NAPOCOR is no longer using respondents’ properties to build the Substation Project, it may
discontinue the expropriation proceedings subject to court approval. However, the grant of the
motion to withdraw carries the consequence only of making the trial court’s condemnation order
final and executory.

The rule is that expropriation must be dismissed when it is determined that it is NOT for a public
purpose, EXCEPT when:
1. The trial court’s order already became final and executory;
2. The government already took possession of the property; and
3. The expropriation case already caused prejudice to the landowner.

The expropriation case is NOT automatically dismissed when the property ceases to be for public
use. The state must first file the appropriate Motion to Withdraw before the trial court having
jurisdiction over the proceedings. The grant or denial of any Motion to Withdraw in an
expropriation proceeding is always subject to judicial discretion.

In this case, respondents have not yet been deprived of their property since NAPOCOR was never
able to take possession. The damages cannot be determined. Thus, the case is remanded to the trial
court to determine whether respondents have already been prejudiced. The withdrawal of the
petition before this Court will have no practical effect othat than to make the trial court’s order of
condemnation final and executory. To prevent this absurdity, NAPOCOR should file the motion
to withdraw before the trial court and prove before it its reasons for discontinuing with the
expropriation. Respondents may also plead and prove damages incurred from the commencement
of the expropriation.

37. Republic v. Vicente Lim, GR 161656, June 29, 2005, Sandoval-Gutierrez, J. (Eminent
Domain; where government does not pay just compensation within 5 years from finality of
judgment of expropriation, owner may recover possession; special circumstances in this case,
57 years of non-payment of just compensation, justify recovery of possession by owner
[normally, may only recover just compensation])
*unlike in Sec. of DPWH v. Spouses Tecson, owners in this case were not remiss in claiming
just compensation
FACTS:
On September 5, 1938, the Republic filed a case of expropriation with the CFI involving Lots 932
and 939 of the Banilad Friar Land Estate to establish a military reservation for the Philippine Army.
Lot 932 was registered in the name of Gervasia Denzon with an area of 25,137 sqm while Lot 939
in the name of Eulalia Denzon with 13,164 sqm.

After depositing P9500 with the PNB, the Republic took possession. On May 14, 1940, the CFI
rendered decision ordering the Republic to pay the Denzons P4,062.10 as just compensation.

For failure of the Republic to pay for the lots, on September 20, 1961, the Denzons successors-in-
interest, Francisca valdehueza and Josefina Panerio, filed with the CFI an action for recovery of
possession with damages against the Republic and officers of the AFP in possession of the
property. The CFI ruled in favor of Francisca and Josefina, stating that for failure of Republic to
pay P4,062.10, they retained ownership. But they were ordered to execute a deed of sale in favor
of the Republic, adjusting the value to P16,248.40, to be paid with 6% interest per annum from
April 5, 1948, date of entry. They appealed to the SC.

The SC held on May 19, 1966 that they were the owners, but they could not recover possession
but may only demand payment of the property’s FMV.

In 1964, Valdehueza and Panerio mortgaged Lot 932 to Vicente Lim, respondent. He had the
mortgaged foreclosed in 1976. Thus, title was issued in his name. He filed on August 20, 1992 a
complaint for quieting of title with the RTC against the Republic and AFP officials, petitioners.

The RTC ruled that Lim is the absolute owner of Lot 932 including right of possession. The CA,
on appeal by petitioners, held that the claim of petitioners constitutes a cloud or uncertainty on the
title of Lim and can be removed by action to quiet title. Thus it affirmed the RTC ruling.

Petitioners appealed to the SC, claiming that they remained the owner of Lot 932 as held in
Valdehueza v. Republic. Although it was denied, it filed a motion for leave to file another motion
for reconsideration.

ISSUE:
Whether a private property owner Lim, whose property has been expropriated may, recover
possession of it after failure of the expropriator, for 57 years and despite twice being ordered by
competent court, to pay just compensation.

HELD: YES.
The final judgment in the expropriation proceedings was entered on April 5, 1948. More than 50
years has passed but the landowner, now respondent, has remained empty handed. The Court of
Appeals is correct in saying that Republic’s delay is contrary to the rules of fair play, as just
compensation embraces not only the correct determination of the amount to be paid to the
owners of the land, but also the payment for the land within a reasonable time from its
taking. Without prompt payment, compensation cannot be considered just.

Section 9, Article III of the Constitution should be strictly interpreted against the expropriator and
liberally in favor of the property owner.
In opposing respondent’s claim, the Republic is invoking Valdehueza v. Republic, a decision it
utterly defied (it required the Government to pay the owners P16,248.40 as just compensation, but
the government did not). How could the Republic acquire ownership over Lot 932 when it has not
paid its owner the just compensation, required by law, for more than 50 years? The recognized
rule is that title to the property expropriated shall pass from the owner to the expropriator
only upon full payment of the just compensation. Without full payment of just compensation,
there can be no transfer of title. The Republic’s acquisition of ownership is conditioned upon the
full payment of just compensation within a reasonable time.

Expropriation consists of two stages: 1) determination of authority to exercise eminent domain and
the propriety of its exercise, ending with an order of condemnation or dismissal, and 2)
determination of just compensation. It is only upon completion of the 2 stages that expropriation
is completed. The process is not completed until payment of just compensation. Thus, the failure
to pay for 57 years rendered the expropriation process incomplete.

The facts of this case do not justify the application of the doctrine that non-payment of just
compensation does not entitle recovery of possession of expropriated lots. The Republic was
ordered twice to pay just compensation. We cannot but construe the Republic’s failure as
deliberate refusal on its part. Under such circumstance, recovery of possession is in order. In
several jurisdictions, it is held that recovery of possession may be had where rightful entry is made
and the party condemning refuses to pay the compensation which has been assessed or agreed
upon.

The Republic’s assertion that State defense will be endangered if Lot 932 is ordered reversed to
respondent is an overstatement. Lot 932 had ceased to operate as an airport. What remains is only
the National Historical Institute marking that Lot 932 was formerly the location of Lahug Airport.
There are only 13 structures on Lot 932, 8 of which are residence apartments of military personnel.
Only two (2) buildings are actually used as training centers. Thus, practically speaking, the
reversion of Lot 932 to respondent will only affect a handful of military personnel. It will not result
to irreparable damage or damage beyond pecuniary estimation, as what the Republic vehemently
claims.

Thus, the special circumstances entitle respondent to recover possession of the expropriated lot
from the Republic.

While the prevailing doctrine is that non-payment of just compensation does not entitle the private
landowner to recover possession of the expropriated lots, however, in cases where the government
failed to pay just compensation WITHIN FIVE (5) YEARS from the FINALITY of the
judgment in the expropriation proceedings, the owners shall have the right to RECOVER
POSSESSION of their property. The government cannot keep the property and dishonor the
judgment.

POWER OF TAXATION
38. Walter Lutz v. J. Antonio Araneta, GR L-7859, December 22, 1955, Reyes, J.B.L., J.
(Taxation; taxation may be used to implement police power, thus same requirement as police
power of public purpose)
FACTS:
CA 567, Sugar Adjustment Act, to stabilize the sugar industry to prepare it for the loss of its
preferential position in the US market because of the eventual imposition of export taxes on sugar,
increases the tax on manufacture of sugar in Section 2, levies on owners or persons in control of
lands for sugar cane cultivation which is on lease or otherwise a tax equivalent to the difference
between the rental and 12% of the land’s assessed value in Section 3. Section 6 created a Sugar
Adjustment and Stabilization Fund from the collections under the law to protect the sugar industry
despite gradual loss of preferential position on PH sugar in the US market, etcetc.

Plaintiff Walter Lutz seeks to recover taxes paid under Section 3, alleging that such tax is
unconstitutional for being levied in aid of the sugar industry exclusively, which is not a public
purpose. The CFI dismissed the petition. He appealed to the SC.

ISSUE:
Whether a law imposing a tax on owners of land for sugar cane cultivation to create a fund to be
used to protect the sugar industry from its eventual loss of preferential position in the US market
due to the eventual imposition of export tax on Philippine sugar is a valid imposition of tax.

HELD: YES.
The tax is levied for a regulatory purpose, to provide means to rehabilitate and stabilize the
threatened sugar industry. The act is primarily an exercise of police power.

Sugar production is one of the great industries in the nation and occupies a leading position among
its export products. It gives employment to thousands and is a great source of the State’s wealth.
It is pivotal in the plans of a regime committed to a policy of currency stability. Thus, its promotion
redounds greatly to the general welfare.

The protection and promotion of the sugar industry is a matter of public concern. Thus, the
legislature may determine within reasonable bounds what is necessary for its protection and
expedient for its promotion. The legislative discretion must be allowed full play, subject only to
the test of reasonableness. If objective and methods are both constitutional, no reason is seen
why the state may not levy taxes to raise funds for their attainment. Taxation may be made the
implement of police power. It is rational that the tax be obtained precisely from those who are
to be benefited from the expenditure of the funds derived from it, the sugar producers.

It is inherent in the power to tax that a state be FREE TO SELECT THE SUBJECTS of
TAXATION, and the inequalities which result from singling out of a particular class for
taxation or exemption infringe no constitutional limitation. It may be that other industries need
similar protection, but the legislature is not required to adhere to a policy of “all or none.”
39. Romeo Gerochi v. DOE, GR 159796, July 17, 2007, Nachura, J. (Taxation; whether an
imposition is a tax or an exercise of police power depends on its primary PURPOSE- 1)
generation of revenue, or 2) regulation)
FACTS:
Petitioners challenge Section 34 of RA 9136, Electric Power Industry Reform Act of 2001
(EPIRA), imposing the Universal Charge (UC), and Rule 18 of its IRR seeking to implement it
as unconstitutional. Section 34 imposed a universal charge on all electricity end-users to pay for
the government’s stranded debts and stranded contract costs of National Power Corporation
(NPC), missionary electrification, and imposing an environmental charge of P0.0025/kWh for an
environmental fund for watershed rehabilitation and management to be managed by the NPC.

The NPC filed with the Energy Regulatory Commission (ERC) to avail from the universal charge
of its share for missionary electrification. It also asked for the environmental charge of
P0.0025/kWh, or P119,488,847.59, to be approved for withdrawal. The ERC authorized the NPC
to draw up to P70M for its 2003 Watershed Rehab Budget subject to availability of funds for the
Environmental Fund component of the Universal Charge. Thus, Panay Electric Company (PECO)
charged petitioner and all other end-users with the Universal Charge Hence this petition.

Petitioners claim that the imposition of the universal charge on all end-users is oppressive and
confiscatory and amounts to taxation without representation as the consumers were not given
a chance to be heard and represented. They claim that the UC has the characteristics of a tax and
is collected to fund NPC operations.

ISSUE:
Whether the UC imposed under Section 34 of RA 9136 or the Electric Power Industry Reform Act
of 2001, for the purpose of ensuring the viability of the country’s electric power industry, imposing
a universal charge to be determined by the Energy Regulatory Commission on all electricity end-
users, is a tax.

HELD: NO.
The power to tax is an incident of sovereignty and is unlimited in its range so that security against
its abuse is to be found only in the responsibility of the legislature which imposes the tax on the
constituency that pays for it. It is based on the principle that taxes are the lifeblood of the
government, and their prompt and certain availability is an imperious need. Thus, the theory behind
the exercise of the power to tax emanates from necessity.

On the other hand, police power is the power of the state to promote public welfare by restraining
and regulating the use of liberty and property. Its justification is found in the maxims salus populi
est suprema lex (the welfare of the people is the supreme law) and six utere tuo ut alienum non
laedas (so use your property as not to injure the property of others).

The pivotal distinction rests in the purpose for which the charge is made. If generation of revenue
is the primary purpose and regulation is merely incidental, it is a tax. But if regulation is the
primary purpose, the fact that revenue is incidentally raised does not make it an imposition of tax.

The state’s police power is invoked in imposing the UC. The purposes in Section 34 of the UC
are regulatory in character. From the purposes in Section 2, its declaration of policies, it can be
gleaned that the UC is not a tax, but an exaction in the exercise of police power. Public welfare
is promoted.

The taxing power may be used as an implement of police power. The purpose is to ensure the
viability of the country’s electric power industry.
*Not Consti2 related
Potestas delegate non delegari potest. What has been delegated cannot be delegated. This is based
on the principle that delegated power is not only a right but a duty to be performed by the delegate
through his own judgment and not through the intervening mind of another. But in the increasing
complexity of modern life, delegation of legislative power to administrative agencies is allowed
as an exception. All that is required to validly exercise this subordinate legislation is that the
regulation be germane to the objects and purposes of the law and that the regulation be not in
contradiction to, but in conformity with, the standards prescribed by the law. These are the
completeness test and the sufficient standard test.

Under the completeness test, the law must be in all its terms and conditions when it leaves the
legislature such that when it reaches the delegate, the only thing he will have to do is to enforce it.
The second test mandates adequate guidelines or limitations in the law to determine the boundaries
of the delegate's authority and prevent the delegation from running riot.

The Court finds that the EPIRA is complete in all its essential terms and conditions, and that it
contains sufficient standards.

Although Sec. 34 of the EPIRA merely provides that "within one (1) year from the effectivity
thereof, a Universal Charge to be determined, fixed and approved by the ERC, shall be imposed
on all electricity end-users," and therefore, does not state the specific amount to be paid as
Universal Charge, the amount nevertheless is made certain by the legislative parameters provided
in the law itself. One such parameter is in Sction 43 (b) (ii). Moreoever, the ERC does not enjoy a
wide discretion in determining the UC. Section 51 (d) and (e) provides that the stranded debts and
contract costs calculated by the NPC shall be the basis of ERC in determining the UC. Thus, the
law passes the first test.

As to the second test, EPIRA provisions such as “to ensure the total electriDcation of the country
and the quality, reliability, security and affordability of the supply of electric power" and
"watershed rehabilitation and management" meet the requirements for valid delegation, as they
provide the limitations on the ERC's power to formulate the IRR. These are sufficient standards.

40. Wenceslao Pascual v. Secretary of Public Works and Communications, et al. GR L-


10405, December 29, 1960, Concepcion, J. (Taxation; public funds may be appropriated only
for public use; it is the DIRECT OBJECT or main purpose of the expenditure that is looked
at, NOT the magnitude of public benefit; incidental public benefit does not justify
appropriation for the private purposes)
FACTS:
Pascual, as governor of Rizal, filed this action for declaratory relief with injunction on the ground
that RA 920, An Act Appropriating Funds for Public Works, contained in Section 1-C(a) an item
(43[h]) of P85k “for the construction, reconstruction, repair, extension and improvement" of "Pasig
feeder road terminals; that at the time of approval of the law, the said feeder roads were “nothing
but projected and planned subdivision roads, not yet constructed within the Antonio Subdivision;”
that the Antonio Subdivision was respondent Jose Zulueta’s who, at the time of the passage of the
act, was a senator; that since the projected feeder roads were private property at the time of
passage and approval of RA 920, the P85k appropriation made therein was void; that the
construction of the feeder roads would greatly enhance the value of his Antonio Subdivision, aside
from relieving Zulueta of the burden of construction his subdivision streets at his own expense.
Thus he states that unless restrained by the court, the construction of the feeder roads are being
undertaken by the Bureau of Public Highways to the prejudice of the Filipino nation.

Pascual prays that such item in RA 920 be declared void and the secretary of Public Works and
Communications etc. be issued a writ of injunction against to enjoin them from continuing the
feeder road project. The lower court dismissed the case, hence this appeal.

Zulueta claims that a law passed by Congress can never be illegal since Congress is the source of
all laws and that he is not aware of any law making it illegal to appropriate public funds for the
improvement of private property.

ISSUE:
Whether the construction of feeder roads on certain portions of the Antonio Subdivision, private
property of then Senator Zulueta, at the expense of public funds appropriated by RA 920 at P85k,
relieving Zulueta of the burden of constructing its subdivision streets or roads at his own expense
and, if constructed, greatly enhancing or increasing the value of the subdivision of Zulueta, is a
valid and constitutional appropriation of law despite the subsequent donation of Zulueta of such
property to the government.

HELD: NO.
The legislature is without power to appropriate public revenue for anything BUT a public
purpose. It is the essential character of the direct object of the expenditure which must determine
its validity as justifying a tax and NOT the magnitude of interests to be affected nor the degree
to which the general advantage to the public may ultimately be benefited by their promotion.
Incidental advantage to the public or the state from the promotion of private interests and
prosperity of private enterprises or business does NOT justify their aid by the use of public
money.

The taxing power must be exercised for public purposes only and not for the advantage of private
individuals. Under the express or implied provisions of the constitution, public funds may be used
for a public purpose. The right of the legislature to appropriate funds is correlative with its right to
tax, under constitutional provisions against taxation except for public purposes and prohibiting the
collection of a tax for one purpose and the devotion thereof to another purpose, no appropriation
of state funds can be made for other than a public purpose.

The test of constitutionality of a statute requiring the use of public funds is whether the statute is
designed to promote the public interests, as opposed to the furtherance of the advantage of
individuals although each advantage to individuals might incidentally serve the public.

The validity of a statute depends upon the powers of Congress at the time of its passage and not
upon events subsequent thereto. The legality of the P85k appropriation depended upon whether
the feeder roads were public or provate property when the bill, which became RA 920, was passed
and became effective. As the land belonged to Zulueta, the appropriation sought a private purpose,
and was thus void. Zulueta’s donation to the government five (5) months after the effectivity of
the Act did not cure its basic defect.

Although the validity of a statute may be contested only by one who will sustain a direct injury in
consequence of its enforcement, there are many decisions nullifying, at the instance of taxpayers,
laws providing for disbursement of public funds upon the theory that the expenditure of public
funds by an officer of the State to administer an unconstitutional act is a misapplication of
funds, which may be enjoined at the request of a taxpayer.

41. Silvestre Punsalan, et al. v. The Municipal Board of the City of Manila, et al., GR L-4817,
May 26, 1954, Reyes, J. (Taxation; the legislature may, in its discretion, select what
occupations to tax and what not to tax; nothing is inherently obnoxious with double taxation-
tax on the same occupation by both the national and local government)
FACTS:
A suit was filed in the CFI of Manila by 2 lawyers, medical practitioner, accountant, dental
surgeon, and pharmacist, “in their own behalf” and of “other professionals practicing in” Manila.
They sought to annul Ordinance 3398 of Manila and the provision of the Manila Charter
authorizing it and the refund of taxes collected under the ordinance paid under protest.

The ordinance imposes a municipal occupation tax on persons exercising various professions in
the city and penalizes non-payment thereof by a fine or imprisonment or both. It was enacted
pursuant to Section 18, par (1) of the Revised Charter of the City of Manila, empowering the
Municipal Board of Manila to impose a municipal occupation tax not to exceed P50 per year on
persons engaged in various professions. Plaintiffs paid the occupational tax under protest and filed
this suit.

The lower court upheld the provision in the Charter but declared the ordinance void stating that
the penalty for non-payment therein was not authorized. Hence both parties appealed.

ISSUE:
Whether the provision of the Revised Charter of the City of Manila authorizing it to impose a
municipal occupation tax on the exercise of various occupations not to exceed P50 per annum and
Ordinance 3398, enacted pursuant to such authority, imposing such occupation tax and penalizing
with fine and imprisonment non-payment thereof

HELD:
The Charter, on the last paragraph of Section 18, expressly empowers the municipal board to
penalize violation of ordinance for a single offense.

Petitioners contend that the ordinance and law authorizing it is class legislation, unjust, oppressive,
and authorize double taxation.

The legislature may, in its discretion, select what occupations to tax, and in the exercise of that
discretion it may tax all, or it may select certain classes to tax and leave others untaxed. As
to the contention that the law authorized only Manila and not other cities to impose the tax, it is
not for the courts to judge what cities or municipalities should be empowered to impose occupation
tax in addition to that imposed by the national government. That matter is within the domain of
the political departments.

The argument against double taxation may not be invoked where one tax is imposed by the state
and the other is imposed by the city, it being widely recognized that there is nothing inherently
obnoxious in the requirement that license fees or taxes be exacted with respect to the same
occupation, calling, or activity by both the state and political subdivisions thereof.

BILL OF RIGHTS
42. Philippine Blooming Mills Employees Organization v. Philippine Blooming Mills Co.,
Inc., GR L-31195, June 05, 1973, Makasiar, J. (Bill of Rights)
FACTS:
PBM Employees Organization (PBMEO) is a legitimate labor union composed of PBM
employees. Tolentino, Padrigano, Roxas, de Leon, Paciente, Vacuna, Pagcu, and Munsod are
officers of PBMEO. PBMEO claims that they decided to stage a mass demonstration at
Malacañang on March 04, 1969, to protest alleged abuses of the Pasig police to be participated by
workers in the first shift and second and third shifts. They informed PBM of such.

PBM appealed to the PBMEO to excuse first and regular shift workers from the demonstration and
use workers in the 2nd and 3rd shifts in order to not violate their CBA. Otherwise, those who do not
follow will be dismissed.

PBMEO and their 400 members proceeded with the demonstration, PBM thus filed a charge
against them for violating Section 4(a)-6 in relation to Sections 13 and 14 and 15 of RA 875, of
their CBA providing “No strike and no lockout.” PBMEO claims that they did not violate the CBA
as they gave prior notice, that it was a valid exercise of their constitutional freedom of speech, and
it was not a strike as it was not directed against PBM.

Judge Salvador of the Court of Industrial Relations (CIR) found PBMEO guilty of bargaining in
bad faith and petitioners Padrigano, Tolentino, etc. as responsible for the unfair labor practice and
considered them to have lose their employment with PBM.

PBMEO claim that they received the above order on September 23, 1969, and they filed their MR
on September 29 because September 28 fell on a Sunday. CIR dismissed the MR for being filed
beyond the reglementary period under CIR rules of 5 days from receipt. Thus PBMEO filed this
appeal with the SC.

ISSUE:
Whether PBMEO, in demonstrating not against their company PBM but against alleged abuses of
the Pasig Police despite the plea of PBM to excuse from such demonstration workers in the first
and regular shift to avoid unduly prejudicing the company, committed bad faith bargaining.

HELD:
Basic concepts and principles underlie the issues in this case.
1. In a democracy, the preservation and enhancement of the dignity and worth of the human
personality is the cardinal article and central core of faith of our civilization. The inviolable
character of man as an individual must be “protected to the largest possible extent in his thoughts
and beliefs as the citadel of his person.

2. The Bill of Rights is designed to preserve the ideals of liberty, equality and security “against
the assaults of opportunism, the expediency of the passing hour, the erosion of small
encroachments, and the scorn and derision of those who have no patience with general
principles.

Its purpose is to withdraw “certain subjects from the vicissitudes of political controversy, to place
them beyond the reach of majorities and officials, and to establish them as legal principles to be
applied by the courts. One's rights to life, liberty and property, to free speech, or free press,
freedom of worship and assembly, and other fundamental rights may not be submitted to a vote;
they depend on the outcome of no elections.” The happiness of the individual, not the well-being
of the State, is the criterion by which its behavior is to be judged.

3. The freedoms of expression and assembly and the right to petition are among the immunities
reserved by sovereign people not only to protect the minority who want to talk, but also to benefit
the majority who refuse to listen. The liberties of one are the liberties of all, and the liberties of
one are not safe unless the liberties of all are protected.

4. The rights of free expression, free assembly and petition are not only civil but also political
rights essential to man’s enjoyment of life, his happiness, and full fulfillment. Thru these
freedoms the citizens can participate not merely in the establishment of government thru suffrage
but also in the administration of public affairs and in discipline of abusive public officers. The
citizen is accorded these rights so that he can appeal to the appropriate governmental officers or
agencies for redress and protection.

5. While the Bill of Rights also protects property rights, the primacy of human rights over
property rights is recognized. Because these freedoms are "delicate and vulnerable, as well as
supremely precious in our society" and the "threat of sanctions may deter their exercise almost as
potently as the actual application of sanctions," they "need breathing space to survive," permitting
government regulation only "with narrow specificity."

Property rights can be lost thru prescription while human rights are imprescriptible. Otherwise,
the Bill of Rights is a useless attempt to limit the power of government. In the hierarchy of civil
liberties, free expression and assembly occupy a preferred position as they are essential to the
preservation of our civil and political institutions. Such priority gives these liberties the sanctity
not permitting dubious intrusions.

While a mere reasonable or rational relation between means employed and its purpose, that
the law is neither arbitrary, discriminatory, nor oppressive, would validate a law impairing
property rights, a valid infringement of human rights requires a more stringent criterion, the
existence of a GRAVE and IMMEDIATE danger of a substantive evil which the state has a
right to prevent. The improbable danger rule formulated by Judge Learned Hand tests whether
the gravity of the evil, discounted by its improbability, justifies such invasion of free expression
as is necessary to avoid the danger.
The CIR’s conclusion of bad faith bargaining in violation of the CBA, tested by the foregoing
principles governing a democratic society, cannot be sustained. The demonstration was not against
their employer PBM. It was purely and complete an exercise of their freedom of expression in
general and of their right of assembly and of petition for redress of grievances before the
appropriate governmental agency, the Chief Executive, against the Pasis police. They exercised
their civil and political rights for their mutual aid. It was the duty of PBM to protect PBMEO from
the harassment of local police officers. It was to the interest of PBM to rally to the defense of its
employees so that they can work without harassment.

PBMEO were fighting for their very survival, utilizing only weapons afforded by the Constitution-
the untrammeled enjoyment of their basic human rights. The pretension of PBM that it would
suffer damage for absence of its employees from 6am to 2pm (1st shift) is a plea for the
preservation merely of their PROPERTY rights. Such apprehended loss would not spell the
difference between life and death of PBM. The condition of the employees with the Pasig police
affected their right to individual existence and that of their families.

Material loss can be compensated, but the debasement of the human being can never be fully
evaluated in monetary terms.

The primacy of human rights over property rights has been sustained. To hold the demonstration
as evidence of bad faith collective bargaining stretches unduly the compass of the CBA, is a potent
means of inhibiting speech and thus inflicts a moral and mortal wound on the constitutional
guarantees of free expression, peaceful assembly, and of petition.

The CIR held that the CBA, fixing the working shifts, in effect imposes the duty to observe regular
working hours. This strained construction that such working shifts deny the workers the right to
stage a mass demonstration against police abuses during working hours is a virtual tyranny over
the mind and life of the workers. Renunciation of freedom should not be predicated on such a
slender ground.

The demonstration could not be enjoined legally by this court as such injunction would be
trenching upon the freedom of expression of the workers.
PBM’s claim that there was no need for all the employees to attend fails to appreciate the sine qua
non of an effective demonstration especially of a labor union, the complete unity of union members
to generate maximum sympathy for their cause and immediate action. Circulation is one of the
aspects of freedom of expression. The more participants, the more persons can be apprised of the
rally’s purpose. PBM was notified 2 days in advance of the demonstration; it could have made
arrangements to counteract losses it might sustain. There was a lack of human compassion on
PBM’s part. To regard as ground for dismissal the mass demonstration not against the company is
gross vindictiveness, which is as unchristian as it is unconstitutional.

It is PBM who is guilty of unfair labor practice as their refusal to permit its employees to join the
demonstration and the separation of 8 petitioners from the service is an unconstitutional restraint
on their freedom of expression, assembly, and to petition for redress of grievances. Such
unfair labor practice is defined in Section 4(a-1) in relation of Section 3 of RA 875, Industrial
Peace Act. Section 3 of Republic Act No. 875 guarantees to the employees the right "to engage in
concerted activities for . . . mutual aid or protection"; while Section 4(a-1) regards as an unfair
labor practice for an employer "to interfere with, restrain or coerce employees in the exercise of
their rights guaranteed in Section Three."

The purpose of the demonstration was the mutual aid and protection against alleged police abuses.
While CIR found that the demonstration paralyzed to a large extent the operations of PBM, it did
not make any finding of fact as to the loss actually sustained by PBM. This can only mean that
PBM did not sustain any loss. Contrarily, it saved a sizable amount in the form of wages, cost of
fuel, water, and electric consumption for that day. Such could have amply compensated for
unrealized profits or damages it might have sustained by reason of absence of its workers for only
one day.

PBMEO was also denied of social justice as assured under Section 5, Article II, imposing upon
the State the “promotion of social justice to insure the well-being and economic security of all of
the people,” which is emphasized by Section 6 of Article XIV that “the State shall afford protection
to labor.” CIR, as an agency of the State, is obliged to give meaning to these constitutional
guarantees.

Violation of a constitutional rights divests the court of jurisdiction. Both CIR and private firm
PBM failed to accord preference to such rights. Violating these basic human rights, the CIR ousted
itself of jurisdiction and its orders are a nullity. Protection of such freedoms are imperative on all
public officers including the courts and private citizens, the enjoyment of which must not be
nullified by a mere procedural rule of CIR exercising a purely delegated legislative power when
even Congress must yield to the enjoyment of these rights.

There is no time limit to the exercise of these freedoms. They are not exhausted by delivery of one
speech or etc. It is a continuing immunity.

CIR rules state that a MR of its order must be filed within 5 days from notice thereof. The MR was
filed on September 29, 7 days from September 22, date of notice, or 2 days late. It is claimed that
it could have been filed on September 28, but it was a Sunday. But to accord supremacy to the CIR
rules promulgated pursuant to legislative delegation over basic human rights is not only
incompatible with the basic tenet of constitutional government that the Constitution is superior
to any statute or subordinate rule, but also does violence to natural reason and logic. The CIR
rule as applied in this case does not reinforce the affected constitutional rights but constrict them.
The rule is unreasonable and thus beyond the authority granted by the Constitution and the law. 5
days to file a MR is too short, especially for the workers who usually do not have ready funds to
meet the necessary expenses therefor.

It is a procedural rule that generally all causes of action and defenses presently available must be
specifically raised in the complaint or answer; so that any cause of action or defense not raised in
such pleadings, is deemed waived. However, a constitutional issue can be raised any time, even
for the first time on appeal, if it appears that the determination of the constitutional issue is
necessary to a decision of the case, the very lis mota of the case without the resolution of which
no final and complete determination of the dispute can be made. Thus, a procedural rule of
Congress or the SC gives way to a constitutional right.

In this case, enforcement of the basic human freedoms is a most compelling reason to deny
application of a CIR rule impinging on such rights. The SC has the inherent power to suspend its
own rules or to except a particular case from its operation whenever the purposes of justice
require, as long as the parties are given their full day in court. Insisting on applying the CIR rule
would be an unreasoning adherence to “procedural niceties,” which denies justice to the workers.

Many a time, this Court deviated from procedural technicalities when they ceased to be instruments
of justice, for the attainment of which such rules have been devised. They should give way to the
realities of the situation. Even if the CIR rule were to be given effect, the dismissal from
employment is harsh for a one-day absence from work.

Thus, the SC nullied the CIR order and directed the reinstatement of the 8 petitioners who were
dismissed with back pay.

DUE PROCESS
43. Francisco Chavez v. Hon. Romulo, GR 157036, June 09, 2004, Sandoval-Gutierrez, J.
(Due Process; first inquiry is to determine if there is a life, liberty, or property right to
protect; license is not a property or property right, thus not protected by due process)
FACTS:
PGMA, in a speech, directed then PNP chief respondent Ebdane to suspend the issuance of permits
to carry firearms outside of residence (PTCFOR). The issuance of permits was limited to only
ownership and possession of guns and not to carrying them in public places. Only authorized law
enforcement officers and uniformed military men can carry firearms in public places, but civilian
owners may no longer do so. She also allowed use of guns for target practice from time to time
but they may not load the guns with bullets until in the premises of the firing range.

The PNP chief thus issued guidelines implementing such directive. It revoked all existing
PTCFOR, but stated that the holders thereof may re-apply under the new conditions in the
guidelines.

Petitioner Francisco Chavez, a licensed gun owner with a PTCFOR, requested the DILG to
reconsider the guidelines, but such was denied. Hence this petition. He claims, among others, that
the guidelines violate due process because they deprive petitioner of his only means to defend
himself and that the ownership of firearms are constitutionally protected property rights. Petitioner
also claims that even if the guidelines are an exercise of police power, it is unreasonable and
unnecessary and unduly oppressive.

ISSUE:
Whether the guidelines issued by the PNP Chief, revoking the permit to carry firearms outside of
residence (PTCFOR) of all previous holders and makes them apply for new permits under the new
conditions under the new guidelines, violates the citizens’ right to due process, right to bear arms,
right to property.
Whether such guidelines is a valid exercise of police power.

HELD:
1. Right to bear arms. NO.
Petitioner claims that in revoking all existing PTCFOR, PGMA and PNP Chief Ebdane arrogated
unto themselves legislative power.

Potestas delegate non delegari potest. Delegated power may not be delegated. The rule that forbids
delegation of legislative power is not absolute. An exception allows the legislative body to delegate
its licensing power, including the power to promulgate necessary rules and regulations.

PD 1866 states that the “Chief of Constabulary may, in meritorious cases as determined by him
and under such conditions as he may impose, authorize lawful holders of firearms to carry them
outside of residence.” These are pursuant to the general power granted by PD 1866 empowering
him to promulgate rules and regulations for the effective implementation of the decree. The PNP
chief succeeded the Chief of Constabulary and assumed the latter’s licensing authority. PD 1866
and RA 6975 authorize the PNP chief to issue the guidelines.

The right to bear arms, as found in the second amendment of the US Constitution, is founded on
the citizens’ “collective right” to take arms in defense of the State used in civilized warfare, not to
the citizens’ “individual right” to own and possess arms. Moreover, our constitution contains no
similar provision to the second amendment. Possession of firearms by PH citizens it the exception,
not the rule. The right to bear arms is a mere statutory privilege, not a constitutional right.

2. Rigth to property without due process – NO.


Petitioner claims that Section 1 Article 3 provides that “no person shall be deprived of life, liberty,
or property without due process of law.” He claims that the revocation of his PTCFOR deprived
him of his vested property right without due process.

In evaluating a due process claim, the first consideration must be whether life, liberty, or
property interest exists. A license authorizing a person to enjoy a certain privilege is not property
or a property right, it is only a permit to do what otherwise would be unlawful and not a contract
between the grantor and grantee. It does NOT create vested right.

A PTCFOR may be revoked at any time. It only confers a personal privilege to be exercised under
existing restrictions and as thereafter may be reasonably imposed. A licensee takes his license
subject to conditions as the legislature sees fit to impose, and one of the statutory conditions of
this license is that it might be revoked by the selectmen at their pleasure. The revocation does NOT
deprive property, immunity, or privilege within the meaning of these words in the
declaration of rights. The correlative power to revoke or recall a permission is a necessary
consequence of the main power.

3. Police power – NO.


Assuming that the PTCFOR is a property right protected by the constitution, it cannot be
considered absolute as to be beyond the reach of police power. All property is held subject to the
state’s general regulations necessary to the common good and general welfare. The test to
determine the validity of a police measure is:
(1) The interests of the public generally, as distinguished from those of a particular class,
require the exercise of the police power; and (2) The means employed are reasonably
necessary for the accomplishment of the purpose and not unduly oppressive upon
individuals.

This merely reiterates the essence of the constitutional guarantees of substantive due process,
equal protection, and non-impairment of property rights.

The basis for the guidelines was the need for peace and order in society owing ot the proliferation
of crimes especially those by the NPA. PGMA deemed it best to impose a nationwide gun ban.
The motivating factor is the interest of the public in general.

What they proscribe is merely the carrying of firearms outside of residence, and those who wish
to do so may re-apply for a new PTCFOR. This is a reasonable regulation. If the carrying of
firearms is regulated, crime incidents will be curtailed. It would be difficult for criminals to roam
around with their guns, and it would be easier for the PNP to apprehend them.

44. Legaspi v. City of Cebu, GR 159110, December 10, 2013, Bersamin, J. (Due Process; test
of valid ordinance; test of reasonableness; procedural due process means notice and hearing,
but not always necessary if there are valid reasons)
FACTS:
The Sangguniang Panlungsod of the City of Cebu enacted Ordinance 1664 authorizing traffic
enforcers to immobilize any motor vehicle violating parking restrictions and prohibitions. It
authorized the clamping of any vehicle found violating any existing ordinance of Cebu regulating
vehicle parking.

Atty. Jaban Sr. and Atty. Jaban Jr. brought suit in the RTC seeking the declaration of
unconstitutionality of Ordinance 1644 for violating due process and being contrary to law. They
alleged that Jaban Sr. properly parked his car in Manalili Street to get documents from his office
and upon his return less than 10 minutes later, his car was clamped with a notice. Thus he was
unable to meet an important client, and he was fined P4,200 for the release of the car. He was also
fined P1500 for another traffic violation “without court hearing and final judgment.” Atty. Jaban
Jr. also was fined P1400 without a court hearing and final judgment.

Valentino Legaspi also sued to demand delivery of personal property and the nullity of Ordinacne
1664. His car was also clamped.

The RTC declared the ordinance unconstitutional, but the CA overturned this decision. Hence
these petitions for certiorari.

ISSUE:
Whether Ordinance 1664, authorizing the immobilization thru clamping of motor vehicles found
violating parking regulations and fining the owners or drivers of those vehicles before its release
back to its owners, is a valid ordinance.
HELD: YES.
The tests of a valid ordinance are that it (1) must not contravene the Constitution or any statute;
(2) must not be unfair or oppressive; (3) must not be partial or discriminatory; (4) must not prohibit
but may regulate trade; (5) must be general and consistent with public policy; and (6) must not be
unreasonable.

The test is divided into the formal, whether the ordinance was enacted within the corporate powers
of the LGU and the procedure prescribed by law, and substantive, the conformity of the ordinance
with limitations of the constitution and statutes and the requirements of fairness and reason and its
consistency with public policy.

The ordinance complied with formal requirements. Congress, thru the LGC, delegated to LGUs
the powers of police power, eminent domain, and taxation. The legislative power to enact traffic
rules and regulations was expressly done thru Section 458 of the LGC and also by its General
Welfare Clause in Section 16. Section 458 allows LGUs to approve ordinance which regulate the
use of streets etc.

The first substantive requirement is adherence to the onstitutional guaranty of due process in
Section 1, Article III. The guaranty of due process is a constitutional safeguard against any
arbitrariness of the Government whether by the executive, legislature, or judiciary. If the law
unreasonably deprives a person of life, liberty, or property, or if the exercise of his right is
conditioned on an unreasonable requirement, he is denied due process.

This clause has been interpreted as imposing 2 separate limits on government, procedural and
substantive due process. Procedural due process refers to the procedures that the government
must follow before it deprives a person of life, liberty, or property. Substantive due process
asks whether the government has adequate reason to take away a person’s life, liberty, or
property. It looks to whether there is sufficient justification for governmental action. Whether
there is justification depends on the level of scrutiny used (rational basis, strict scrutiny)

Individual rights may be adversely affected only to the extent that may fairly be required by the
legitimate demands of public welfare. Due process requires the intrinsic validity of the law in
interfering with the rights of the person to life, liberty, or property.

The Jabans claim that leaving the immobilization to traffic enforcers instead of to officials
exercising judicial authority violates due process and that such immobilization should only be after
hearing. Legaspi claims that Ordinance 1664 is arbitrary and oppressive.

Even under strict scrutiny review, Ordinance 1664 met the substantive tests of validity and
constitutionality by its conformity with Constitutional or statutory limitations, the requirements of
fairness and reason, and by its consistency with public policy. Since traffic congestions were
retarding the growth and progress in the population and economic centers of the country, the aim
of ordinance 1664 was to advance general welfare in Cebu and to fulfill the compelling
government purpose of addressing the burgeoning traffic congestions cause by illegally parked
vehicles obstructing the streets of Cebu.
The petitioners also assert that drivers or owners affected were not accorded the opportunity to
protest the clamping, towing, and impounding of the vehicles, or even to be heard to explain
their side.

Ordinance 1664 is not oppressive or arbitrary. Any driver or vehicle owner whose vehicle was
clamped could protest such action. Section 3 permits the release of the immobilized vehicle
upon protest directly made to the chairman of CITOM etc. The release could be ordered even
without fine. That none of the petitioners resorted to such recourse did not diminish the
reasonableness of the escape clause. Also, the clamping or immobilization was not necessary if
the driver or owner was around at the time of apprehension. The enforcer would simply require
the driver or owner to move the vehicle or issue a traffic citation if he persists. The clamping would
only happen to prevent the use of the vehicle to escape the due sanctions. Lastly, the towing away
was not equivalent to summary impounding, but designed to prevent the immobilized vehicle
from obstructing traffic. The owner would not be deprived of property.

As for procedural due process, notice and hearing are the eseential requirements. But there are
many instances in which the absence of one or both is not necessarily denial of due process. The
immobilization was necessary because the transgressors were not around at the time of
apprehension. Notice and hearing would be superfluous under such circumstances. Giving the
transgressors the chance to reverse apprehensions thru a timely protest could satisfy the need for
hearing. The prior intervention of a court of law was not indispensable to ensure compliance
with due process. (*seems like as long as it is reasonable or with valid reasons)

45. Ichong v. Hernandez, GR L-7995, May 31, 1957, Lbrador, J. (Due Process; test of
reasonableness, not arbitrary or oppressive and with a reasonable relation to a proper
legislative purpose; Equal Protection; not violated if there is reasonable ground for
classification; does not require equality for all, only for all persons in a class must be treated
alike)
FACTS:
RA 1180, an Act to Regulate the Retail Business, nationalizes the retail trade business. It
1)prohibits aliens and corporations not wholly owned by Filipinos from engaging directly or
indirectly in retail trade except those aliens actually engaged in said business on May 15, 1954,
who are allowed to continue to engage therein, unless their license are forfeited, until their death
or voluntary retirement for natural persons and 10 years from approval of the act or expiration of
term in case of juridical persons; 2)prohibiting aliens actually engaged in the retail business from
opening additional stores or branches; 3) requiring aliens actually engaged to present for
registration a verified statement giving the nature of business, their assets and liabilities, and
offices, 4) allowing heirs of aliens now engaged in retail business who die, to continue such
business for 6 months for liquidation.

ISSUE:
Whether a law (Republic Act 1180) nationalizing retail trade and prohibiting aliens or juridical
persons the capital of which are not wholly owned by Filipinos from engaging in retail trade except
those already engaged at the time of enactment, prohibiting aliens actually engaged in the retail
business from opening additional stores or branches, allowing heirs of aliens now engaged in retail
business who die, to continue such business for 6 months for liquidation, is valid and reasonable.

HELD: YES.
The limitations on police power are due process and equal protection. Under Section 1 of Article
III of the Constitution, “No person shall be deprived of life, liberty, or property without due process
of law, nor shall any person be denied the equal protection of the laws.”

Due process has to do with the reasonableness of legislation enacted in pursuance of the police
power. Is public welfare involved? Is the act reasonable necessary to accomplish the legislature’s
purpose? Is it not unreasonable, arbitrary, or oppressive? Police power, due process, and equal
protection are supposed to coexist. The balancing is the essence or the indispensable means to
attain the legitimate aspirations of any democratic society. There can be no absolute power, for
that would be tyranny. Yet there can be no absolute liberty, for that would mean license and
anarchy. The test or standard is always reason.

The equal protection clause is against undue favor and individual or class privilege and hostile
discrimination or oppression of inequality. It is not intended to prohibit legislation. It does NOT
demand equality among residents. It merely requires that all persons shall be treated alike
under like circumstances and conditions as to privileges conferred and liabilities enforced. It is
not violated when the law applies only to persons within a class IF it applies alike to all persons in
such class and reasonable grounds exists for making a distinction between those in the class and
those who do not.

*check case 8 – importance of retail trade, alien retailer dominance, alien control and threat; law
is not a product of racial hostility, prejudice, or discrimination, but the expression of the legitimate
desire and determination of the people thru their authorized representatives. The law is in the
interest of the public and of national security, falling within the scope of police power.

1. Equal Protection
The mere fact of alienage is the root cause of the distinction between the alien and national as a
trader. The alien resident owes allegiance to his country of birth and his stay here is for personal
convenience and the lure of profit. While not illegitimate or immoral goals, he naturally lacks that
loyalty and enthusiasm for this country where he temporarily stays or that spirit of sympathy and
consideration for his Filipino customers as would prevent him from taking advantage of their
weakness. The alien never makes a genuine contribution to national wealth, only to general
distribution. The practices resorted to by aliens and their secret manipulations of stocks of
commodities and prices etc. show the real and actual, positive and fundamental differences
between an alien and a national which fully justify the legislative classification in the retail
trade measure.

The Court finds the classification actual, real and reasonable, and all persons of one class are
treated alike. It is not patently unreasonable or unfounded. A law violates equal protection only
when the classification is without reasonable basis.
Citizenship is a legal ground for classification. The difference in status between citizens and
aliens constitutes a basis for reasonable classification in police power. It is wise for the legislature
to limit the business of those who are supposed to have regard for the welfare of the community.
An alien cannot be sufficiently acquainted with our institutions and our life as to enable him to
appreciate the relation of this particular business to our entire social fabric.

Alien race and allegiance may bear in some instances such a relation to a legitimate object of
legislation as to be made the basis of permitted classification.

In US cases where the courts held distinction between aliens and citizens as invalid ground for
classification, the reasons were because the laws were found to be the product of racial antagonism
and hostility, and there was no question of public interest involved. This case is different. As
mentioned, aliens do not possess the sympathetic consideration and regard for their customers nor
the patriotic desire to bolster the nation’s economy except insofar as it enhances their profit, nor
the loyalty and allegiance which the national owes the land.

2. Due Process
Due process demands only that the law be not unreasonable, arbitrary, or capricious, and that
the means selected shall have a real and substantial relation to the subject sought to be attained.
If the law is with a reasonable relation to a proper legislative purpose and are neither arbitrary
nor discriminatory, the requirements of due process are satisfied.

The real question at issue is: Is the exclusion in the future of aliens from the retail trade
unreasonable, arbitrary and capricious, taking into account the illegitimate and pernicious form
and manner in which the aliens have heretofore engaged therein? The answer is clear. The law is
deemed necessary to bring about the desired legislative objective, to free national economy from
alien control and dominance.

The test of reasonableness of a law is the appropriateness or adequacy under all


circumstances of the means adopted to carry out its purpose into effect.

Freedom and liberty are not real and positive if the people are subject to the economic control and
domination of others, especially if not of their own race or country. Economic independence is a
legitimate aspiration of a people. A nationalistic tendency is manifested in the Constitution. The
preamble states a principal objective to conserve the patrimony of the nation and, corollarily, the
provision limiting to citizens of the Philippines the exploitation, development, and utilization of
its natural resources. The nationalization of the retail trade is only a continuance of the nationalistic
protective policy of the Constitution.

The law is made prospective and recognizes the right and privilege of those already engaged in
the occupation to continue therein during the rest of their lives. The right or privilege is denied
only to those upon conviction of certain offenses. The test of the validity of a law attacked as a
violation of due process, is not its reasonableness, but its unreasonableness, and we 9nd the
provisions are not unreasonable.
46. Philippine Communications Satellite Corporation (PHILCOMSAT) v. Jose Alcuaz, GR
84818, December 18, 1989, Regalado, J. (Due process)
FACTS:
PHILCOMSAT was granted a franchise thru RA 5514 to establish and operate stations and
facilities for international satellite communications. The petition seeks to annul the order of
respondent Commissioner Jose Luis Alcuaz of the National Telecommunications Commission
(NTC) directing the provisional reduction of the rates which may be charged by petitioner for
certain lines of its services by 15% with reservation to make further reductions. Petitioner contends
that such violates due process of law.

Since 1968, PHILCOMSAT has been leasing its satellite circuits to PLDT etc. The satellite
services of PHILCOMSAT enable said international carriers to serve the public with indispensable
communication services like live television, overseas telephone, facsimile, etc.

Under RA 5514, PHILCOMSAT was not under jurisdiction of NTC, but EO 196 on June 17, 1987
place PHILCOMSAT under NTC’s jurisdiction. On September 9, 1987, PHILCOMSAT filed with
NTC an application to continue operating and maintaining the same facilities it has been operating
since 1967 and to charge the current rates. It was granted provisional authority to continue
operating its existing facilities and to charge rates it was charging. Until September 16, 1988. The
NTC order assailed extended the provisional authority of PHILCOMSAT for 6 months from
September 16, but directed it to charge modified reduced rates of 15%.

PHILCOMSAT claims that the order violates procedural due process for having been issues
without prior notice and hearing, and the rate reduction it imposes is unjust and confiscatory,
violating substantive due process. It also claims that EO 546, NTC’s enabling act, empowering it
to fix rates for public service does not provide the necessary standards constitutionally required.

ISSUE:
1. Whether EO 546, the enabling law of NTC empowering it to fix rates for public service, and
EO 196, placing PHILCOMSAT under the jurisdiction of NTC, are invalid delegations of
legislative power for lacking standards.
2. Whether NTC’s order to PHILCOMSAT, provisionally authorizing it to continue charging rates
but at a reduced rate of 15% for 6 months, violates procedural due process for lack of hearing and
notice and substantive due process for being unjust and confiscatory.

HELD:
1. NO.
Delegation of legislative power is valid if some standard and the manner of its exercise is
prescribed. When the administrative agency concerned, NTC in this case, establishes a rate, it must
both be non-confiscatory and must be in the manner prescribed by the legislature. In case of
delegation of rate-fixing power, the only standard which the legislature is required to prescribe is
that the rate be reasonable and just.

Under Section 15(g) of EO 546, NTC should be guided by the requirements of public safety, public
interest, and reasonable feasibility of maintaining effective competition of private entities in
communications and broadcasting facilities. NTC is limited by requirements of public safety,
satisfying the requirement of a valid delegation.

2.
a. Procedural due process –violated.
Petitioner claims that the order was based merely on an initial unilateral evaluation and since rate
fixing-power is quasi-judicial and adjudicatory, not quasi-legislative, notice and hearing are
necessary.

The necessity of notice and hearing in an administrative proceeding depends on the character
of the proceeding and the circumstances involved. When the rates are meant to apply to all
enterprises of a given kind throughout the Philippines, it is legislative in character. As a general
rule, notice and hearing are NOT essential to the validity of administrative action where the
administrative body acts in the exercise of executive, administrative, or legislative functions.
But where a public administrative body acts in a judicial or quasi-judicial matter, and its acts
are particular and immediate rather than general and prospective, the person whose rights or
property may be affected by the action is entitled to notice and hearing.

The order of NTC by Alcuaz is quasi-judicial adjudication. The said order pertains exclusively to
petitioner PHILCOMSAT and to no other. It is premised on a finding of fact, although superficial,
that there is merit in reducing the rates charged based on an initial evaluation of
PHILCOMSAT’s financial statements without affording PHILCOMSAT the benefit of an
explanation as to what particular aspects of the financial statements warranted the rate
reduction. No rationalization was offered which prompted NTC to impose the 15% rate reduction.
PHILCOMSAT was not even afforded opportunity to cross-examine the inspector who issued the
report which NTC based its order.

NTC insists that despite the order being issued pursuant to its quasi-judicial functions, notice and
hearing is not needed since the order is merely incidental to the entire proceedings and temporary
in nature. While NTC may fix a temporary rate pending final determination of PHILCOMSAT’s
application, such temporary order is not exempt from statutory procedural requirements of notice
and hearing and the requirement of reasonableness. Assuming that such power is vested in NTC,
it may not exercise it in an arbitrary and confiscatory manner. The Public Service Act, Section
16(c), requires proper notice and hearing in fixing rates of any public service. Thus, NTC has no
authority to fix rates without first giving PHILCOMSAT a hearing, whether the order be
permanent or temporary.

b. Substantive due process – violated.


PHILCOMSAT contends that the rate reduction is confiscatory in that its implementation would
result in a cessation of its operations and closure. NTC claims that PHILCOMSAT is operating
thru a legislative franchise, thus it has no vested right therein. It merely has a privilege or license
which may be revoked at will by the State. PHILCOMSAT claims that such withdrawal should
nevertheless be not arbitrary, but fair and reasonable.
PHILCOMSAT is a mere grantee of legislative franchise subject to amendment, alteration, or
repeal by Congress when the common good so requires. Such grant cannot be unilaterally
revoked without a showing that the termination of said utility is required by the common good.

The power of the State to regulate the business of public utilities is limited in that it is not the
owner of the property of the utility since the private right of ownership remains and is not to be
destroyed by the regulatory power. Any regulation which operates as confiscation of private
property or is an arbitrary or unreasonable infringement of property rights is void as it violates
due process and equal protection.

The inherent power of the State or its agent to regulate rates charged by public utilities should be
subject to the requirement that the rates so fixed shall be reasonable and just. This requirement
of reasonableness comprehends such rates which must not be so low as to be confiscatory, or too
high as to be oppressive.

What is just and reasonable is based on sound business judgment based on evidence, a question
of fact calling for the exercise of discretion and independent judgment. In determining whether
a rate is confiscatory, the given situation, requirements, and opportunities of the utility must be
considered. A method in determining reasonableness is the fair return upon the value of the
property to the public utility. Competition is also a factor.

The rate reduction in this case is solely based on the initial evaluation made on the financial
statements of PHILCOMSAT. NTC also did not elaborate on how it arrived at the rates.
PHILCOMSAT is engaged in several projects, formulated on the premise of rates at their present
and reasonable levels. An undue reduction may lead to cessation of its business. While we concede
the primacy of public interest in an adequate service, the same is not necessarily to be equated with
reduced rates. Reasonableness in the rates assumes that it is fair to both public utility and the
consumer.

47. Ynot v. Intermediate Appellate Court, GR 74457, March 20, 1987, Cruz, J. (Police Power;
Due process; notice and hearing, but not always necessary; exceptions, if there is immediacy
and urgency to solve the problem) *see case 9 for police power ruling
FACTS:
EO 626-A amends EO 626 to include carabeef in the ban in EO 626 of carabao from being
transported from one province to another. EO 626 also prohibits the slaughter of carabao except if
at least 7 years old if male and 11 if female.

Ynot transported 6 carabaos in a pump boat from Masbate to Iloilo when they were confiscated by
the police for violating EO626-A. Ynot sued for recovery with the RTC. The RTC sustained the
confiscation and declined to rule on the constitutionality of the EO citing lack of authority and its
presumed validity.

The IAC affirmed the RTC ruling. Hence this certiorari petition. Ynot claims that the EO is
unconstitutional as it authorizes the outright confiscation of the carabao or carabeef being
transported across provincial boundaries. He claims that the penalty is invalid as it is imposed
without according the owner the right to be heard as guaranteed by due process.
ISSUE:
Whether EO 626-A, authorizing confiscation by police of carabao and carabeef transported
interprovincially in violation of its provisions, violates due process.

HELD: YES.
The due process clause was intentionally kept vague so it would remain resilient. Flexibility must
be the best virtue of guaranty. It was meant to adapt easily to every situation, enlarging or
constricting its protection as the changing times and circumstances may require.

The minimum requirements of due process are notice and hearing intended as a safeguard
against official arbitrariness. There are exceptions, however, like the conclusive presumption
barring admission of contrary evidence if the presumption is bsed on human experience and there
is a rational connection between fact proved and the fact ultimately presumed, or abatement of a
nuisance per se, like a mad dog on the loose which must be killed immediately.

But even if there was reasonable connection, the law violates due process as the accused is not
accorded opportunity to be heard and is immediately condemned and confers to administrative
authorities the power to adjudge the guilt of the accused. No trial is prescribed and the property
being transported is immediately impounded by police and declared, by the law itself, as forfeited
to the government. In this case, the carabaos were arbitrarily confiscated by the police. The law
defined the prohibition, convicted petitioner and immediately imposed punishment.

While notice and hearing may be validly dispensed with on occasion, and that summary action
may be taken in administrative proceedings as procedural due process is NOT necessarily
judicial only, in these cases there is a justification for the omission of the right to a previous
hearing: immediacy of the problem to be corrected and urgency of the need to correct it. There is
no such pressure of time in this case. There is no reason why the offense should not have been
proved first in a court of justice.

Thus, EO 626-A was declared unconstitutional.

48. Sangguniang Panlungsod ng Baguio City v. Jadewell Parking Systems Corporation, GR


160025, April 23, 2014, Sereno, CJ., (Due process; Notice and hearing required only in
judicial or quasi-judicial proceeding, not in legislative or administrative act; whether
legislative or judicial, depends on the purpose)
FACTS:
The two principal parties (cuz may 9 petitions, lots of petitioners) executed a MOA on June 26,
2000 where City of Baguio authorized Jadewell to regulate and collect parking fees for on-street
parking in the city and to implement installation of modern parking meters, privatizing on-street
parking in the City streets.

On August 31, 2000, the parties executed a supplemental MOA to include the Ganza/Burnham
parking space. This supplemental MOA was not confirmed or ratified by the Sanggunian. In
September, 2000, Jadewell began to take over the parking facilities of Ganza/Burnham Park.
Questions arose as to Jadewell’s compliance with the MOA such as obtaining DPWH
recommendation for installation of parking meters and the legality of parking fee coolection by its
parking attendants before the installation of parking meters.

Sanggunian passed Resolution 395 s2000, directing Jadewell to comply with the MOA and install
the necessary number of parking meters. On January 27, 2001, Jadewell wrote that it had
completed installation of parking meters. The City Treasurer demanded remittance of Baguio’s
share of the parking fees collected. Jadewell responded that it had complied with this obligation.

The legal disputes began when the Sangguniang Panlungsod of Baguio, in its resolution outlining
the violations of Jadewell of the MOA, revoked the MOA alleging substantial breach of Jadewell.
Mayor Vergara vetoed the resolution, stating that the MOA provides for a minimum of 5 years
before the right of rescission can be exercised and that the right of Jadewell to due process was
violated due to lack of opportunity to hear its side, but the Sanggunian overrode this thru another
resolution on April 17, 2002. – FIRST act of rescission of the MOA. Jadewell denied the breach
and filed with the RTC a suit questioning the revocation.

On May 27, 2002, Jadewell filed with the RTC a petition for certiorari assailing the validity of
Resolution 37 which rescinded the MOA. The RTC found the rescission unlawful. The Sanggunian
appealed with the CA. The CA affirmed the RTC decision. The Sanggunian filed a certiorari
petition with this Court.

The Sanggunian and mayor issued various resolutions and orders prohibiting Jadewell from
clamping and towing and collecting parking fees. Thus Jadewell filed also various petitions for
indirect contempt of the SC for issuing the said orders. The SC resolved to order Mayor Yaranon
to reopen the streets occupied by Jadewell and since he did not comply, he was cited in contempt
and arrested.

On September 22, 2006, the Sanggunian issued Resolution 204 s2006, rescinding the MOA. –
SECOND act of rescission.

ISSUE:
Whether the rescission of a Memorandum of Agreement between Jadewell, private corporation,
and the Sangguniang Panlungsod of the City of Baguio by the latter unilaterally violated due
process.

HELD: NO.
Jadewell argues that the rescission of the MOA was invalid as it violated due process and also
because there was no substantial breach on its part.

The Sanggunian, when it resolved to rescind the MOA, did not give Jadewell an opportunity to
present its side. When denial of due process is raised, it is directed primarily against the exercise
of governmental authority that “deprives life, liberty, and property” without due process. It is
not a relevant argument in contractual breach between two purely private entities nor is it available
against the government when the latter is not discharging a governmental function but merely
pursuing a purely commercial activity in its proprietary capacity.
The regulation of on-street and off-street parking is a governmental function by local
governments. Baguio’s objective in privatizing the administration of such parking and executing
a MOA with Jadewell for environmental and peace and safety reasons. The MOA involves
delegation of governmental functions in terms of regulating the designation and use of parking
spaces and fee collection for such use. Thus, the resolution or MOA has the character of a
franchise because what is being privatized is a government-monopolized function.

Is there a provision in applicable laws or the MOA or franhise that grants Baguio the right to
revoke the latter at will or upon certain conditions, such that ordinary due process protection can
be considered waived by the franchisee? There is only one provision allowing for unilateral
revocation in the MOA, granting a minimum of 5 years against rescission. Thus, before this 5 year
period lapses, the right to revoke the MOA arises only under Article 1191 of the NCC. Thus, before
the 5 year period lapses, Baguio could only 1) negotiate the termination of the MOA or 2) exercise
its option under Article 1191.

The negotiation is an inherent right of every party in a contract inferred from the freedom to
contract and modify previous covenants. The parties were not constrained to mutually modify the
restriction of the 5 year minimum guaranty against rescission. The second option is the exercise
of a unilateral right to rescind a bilateral contract on the part of a party who believes that it has
been injured by a breach substantial enough to warrant revocation.

Rescission under Article 1191 takes place 1) thru extrajudicial declaration of rescission or 2) upon
the grant of a judicial decree of rescission. If extrajudicial rescission is not opposed, it produces
legal effect. But if impugned by the other party, it is subject to judicial determination, and the court
may declare the rescission as having been properly or improperly made, or to give the debtor a
period to perform the obligation.

The MOA is a grant of franchise imbued with public interest and not merely an agreement between
two private parties. Even if there is a 5 year guaranty, Article 1191 overrides any stipulation to the
contrary, but the grounds to apply the article must be clearly established. In exercising its option
in Article 1191, it was NOT necessary for Baguio to provide Jadewell opportunity to air its side;
it was not denied procedural due process.

Previous notice and hearing as elements of due process are required for protection of life or vested
property rights and liberty when its limitation or loss takes place in consequence of judicial or
quasi-judicial proceeding generally dependent upon a past act or event which has to be established
or ascertained. It is not essential to the validity of general rules or regulations promulgated to
govern future conduct of a class or persons or enterprises, unless the law provides otherwise.

In this case, the act of the Sanggunian in rescinding the MOA, be it the first or second rescission,
was in the exercise of its legislative or administrative functions and NOT of a judicial or quasi-
judicial function. It does not possess such functions. The MOA supports this view as the reason
behind the MOA was to provide order, given Baguio’s parking problems in identified areas, and
to generate income. The objectives of the Sanggunian and its intention to rescind, as it deems it
to no longer serve the city’s interest, is an exercise of legislative or administrative function.
But as to whether the substantial breach was established as a ground to exercise its right to rescind
is another matter. Baguio claims that Jadewell only installed 14 parking meters. It claims that
Jadewell did not remit the collected parking fees. It also failed to have its parking attendants
deputized.

The MOA does not provide for the exact parking meters to be installed, but 100 parking spaces
were allotted. The MOA obligates Jadewell to have its parking attendants deputized so that they
shall have authority to enforce traffic rules. These are the two most important obligations of
Jadewell considering the nature and objective of the MOA. But the RTC had not properly received
evidence that would allow it to determine the extent of the claimed violations. Baguio was still
duty-bound to establish the alleged breach. There is no evidence to make such determination.

49. Southern Hemisphere Engagement Network, Inc. v. Anti-Terrorism Council, GR 178552,


October 5, 2010, Carpio-Morales, J. (Due Process; 1)Facial vs As applied + 2) Vague vs
Overbreadth; Overbreadth may be applied only to facial challenge on free speech; vagueness
challenge may be applied “as applied” to petitioners actually before the court)
FACTS:
6 petitions challenging the constitutionality of RA 9372, Human Security Act of 2007.

ISSUE:
Whether the challenge against RA 9372
HELD:
1. Locus Standi
Locus standi requires a personal stake in the outcome of the controversy as to assure that concrete
adverseness which sharpens the presentation of issues upon which the court so largely
depends for illumination of difficult constitutional questions. A party assailing the
constitutionality of a statute must have a direct and personal interest, that it sustained or is in
immediate danger of sustaining some direct injury as a result of its enforcement and not
merely that it suffers in some indefinite way. It must show that 1) it personally suffered some
actual or threatened injury as a result of the alleged illegal conduct of the government, 2) the
injury is fairly traceable to the challeneged action, and 3) the injury is likely to be redressed by a
favorable action.

Petitioner organizations assert standing on the basis of being suspected “communist fronts” by the
government while individual petitioners invoke transcendental importance and their status as
citizens and taxpayers. While transcendental public importance dispenses with the requirement of
direct and personal injury, cases involving penal legislation belong to a different genus of
constitutional litigation necessitating a closer judicial scrutiny of locus standi. None of petitioners
face any charge under RA 9372, thus they have no personal stake in the outcome of the
controversy. (*Most of petitioners only claim that they are being surveilled but did not establish
the link of such surveillance to RA 9372, also some petitioners do not claim linkage with the Abu
Sayyaf Group, having a more direct interest since they are being charged by the DOJ under RA
9372 as a terrorist organization.)

Neither can standing be conferred upon petitioners as taxpayers and citizens, a taxpayer suit being
proper when there is exercise of spending or taxing power by Congress, and citizen standing must
rest upon direct and personal interest in the proceeding. RA 9372 is a penal statute, and individual
petitioner-citizens has not alleged any direct and personal interest in the implementation of the
law.

2. Actual case or controversy


The controversy must be justiciable. The pleadings must show an active antagonistic assertion
of a legal right and a denial thereof. It must concern a real and not merely theoretical
question or issue. There must be an actual and substantial controversy as distinguished from
an opinion advising what the law would be upon a hypothetical state of facts.

Petitioners fail to show that RA 9372 forbid constitutionally protected conduct or activity that they
seek to do. Their obscure allegations of sporadic surveillance and supposedly being tagged as
communist fronts in no way approximate a credible threat of prosecution.

Without any justiciable controversy, the petitions have become please for declaratory relief which
the Court has no original jurisdiction over.

3. Overbreadth doctrine, facial invalidation


Petitioners claim RA 9372 is impermissibly broad in defining terrorism, leaving law enforcement
agencies with no standard to measure the prohibited acts.

A facial challenge is allowed against a vague statute and to one which is overbroad because of
possible “chilling effect” upon protected speech. This rationale does not apply to penal statutes.
Criminal statutes have general in terrorem effect from their very existence. If facial challenged is
allowed for this reason alone, the State may well be prevented from enacting laws against socially
harmful conduct.

The overbreadth and vagueness doctrines have special application only to free speech cases. They
are inapt for testing penal statutes.

Statutes found vague as a matter of due process are invalidated only “as applied” to a particular
defendant. "On its face" invalidation of statutes results in striking them down entirely on the
ground that they might be applied to parties not before the Court whose activities are
constitutionally protected. It is a departure from the case and controversy requirements.

The doctrine of vagueness and that of overbreadth do not operate on the same plane. A law
suffers from vagueness when it lacks comprehensible standards that men of common
intelligence must necessarily guess at its meaning and differ as to its application. It 1) violates
due process for failure to accord persons targeted by it fair notice of the conduct to avoid, and 2)
it leaves law enforcers unbridled discretion in carrying out its provisions and becomes an arbitrary
flexing of the government muscle.

The overbreadth doctrine decrees that a governmental purpose to control or prevent activities
constitutionally subject to state regulations may not be achieved by means which sweep
unnecessarily broadly and thereby invade the area of protected freedoms. Distinguished from the
vagueness doctrine, the overbreadth doctrine assumes that individuals will understand what a
statute prohibits and will thus refrain from that behavior even though some of it is protected.

A facial challenge is different from “as applied” challenge. Distinguished from as applied
challenge which considers only extant facts affecting real litigants, a facial invalidation is an
examination of the entire law pinpointing its flaws and defects not only on the basis of its actual
operation to the parties, but also on the assumption or prediction that its very existence may cause
others not before the court to refrain from constitutionally protected speech or activities.

The vagueness and overbreadth doctrines, as grounds for a facial challenge, are not applicable to
penal laws. A facial challenege against a criminal statute on vagueness or overbreadth grounds
cannot be done.

Only statutes on free speech, religious freedom, and other fundamental rights may be facially
challenged. Under no case may ordinary penal statutes be subjected to a facial challenge. Else,
prosecution of crimes may be hampered. On the other hand, the application of the overbreadth
doctrine is limited to a facial kind of challenge and, owing to the rationale of a facial challenge,
applicable only to free speech cases.

By its nature, the overbreadth doctrine has to necessarily apply a facial type of invalidation to plot
areas of protected speech almost always under situations not before the court. A statute cannot
be properly analyzed for being substantially overbroad if the court confines itself only to
facts as applied to the litigants. In overbreadth analysis, challengers are permitted to raise the
rights of third parties and the court invalidates the entire statute on its face, not merely “as applied
for.”

Since a penal statute may only be assailed for being vague AS APPLIED to petitioners, a limited
vagueness analysis of “terrorism” in RA 9372 is impermissible without actual or imminent
charge against them.

Vagueness challenges not involving free speech must be examined in light of specific facts (as
applied) and not with regard to the statute’s facial validity.

Under RA 9372, terrorism has the following elements: 1) any cited predicate crime is committed,
2) such creates a condition of widespread and extraordinary fear and panic among the populace,
and 3) the offender is actuated by the desire to coerce the government ot give in to an unlawful
demand. It is contended that “unlawful demand” must be transmitted through expression protected
by the free speech clause.

But what the law penalizes is conduct, not speech. Given the first element of commission of a
predicate crime, any attempt to highlight the communicative component of the prohibition cannot
recategorize the unprotected conduct into a protected speech. Almost every crime entails some
mincing of words. It has never been deemed an abridgement of freedom of speech or press to make
a course of conduct illegal merely because the conduct was, in part, initiated, evidenced, or carried
out by means of language, either spoken, written, or printed.
Since speech is not involved here, the Court cannot heed the call for a facial analysis.

50. Ang Tibay v. The Court if Industrial Relations (CIR), GR 46496, February 27, 1940,
Laurel, J. (Due Process; in administrative proceedings; decision of administrative tribunal
must be based on substantial evidence limited to those presented for its consideration;
tribunal has the duty to consider such pieces of evidence presented to it)
FACTS:
Toribio Teodoro claims that since there was shortage of leather soles in ANG TIBAY, it was
necessary to lay off members of the National Labor Union Inc. The SC ruled that there is nothing
in the evidence that the laborers of the union were laid off due to their affiliation or activity.
National Labor Union Inc. prays for vacation of this judgment and a remanding of the case to the
CIR while the OSG, representing CIR, moves for reconsideration. It is claimed that Toribio’s
reason of supposed lack of leather material was but a scheme to discharge the union members, and
this fact is desired to be proved with the records of the Bureau of Customs and books of accounts
of native dealers in leather. It is also claimed by National Labor Union that the documents are so
inaccessible that even with due diligence they could not be expected to have obtained and offered
them as evidence in the CIR.

ISSUE:
Whether a motion for new trial on the ground that there are new documents to be presented which,
even with due diligence, could not have been offered to the CIR, a lower court, may be granted.

HELD:
CIR is a special court which is more of an administrative board than part of the judicial system.
Unlike a court which is passive, CIR is more active. It not only exercises judicial or quasi-judicial
functions in disputes between employers and employees but it has jurisdiction over the entire
Philippines to consider, investigate, decide, and settle any question or controversy between
employers and employees, laborers, and landlords etc. subject to CA 103. The CIR is not narrowly
constrained by technical rules of procedure, and CA 103 requires it to act according to justice and
equity and substantial merits of the case, without regard to technicalities or legal forms and shall
not be bound by any technical rules of evidence but may inform its mind in such manner as it may
deem just and equitable.

However, the fact that CIR may be free from the rigidity of certain procedural requirements does
not mean that it can entirely ignore or disregard the fundamental and essential requirements of due
process in trials and investigations of an administrative character. It must observe cardinal
primary rights:
1) The right to a hearing – which includes the right of the party interested or affected to present
his own case and submit evidence in support thereof.
2) The tribunal must consider the evidence presented. The right to adduce evidence without the
duty to consider it is vain and futile.
3) While the duty to deliberate does not impose the obligation to decide right, there must be
something to support its decision. A decision without anything to support it is a nullity.
4) The evidence supporting the finding or conclusion must be substantial. Substantial evidence
means such relevant evidence as a reasonable mind might accept as adequate to support a
conclusion.
The law provides that “the rules of evidence prevailing in courts of law and equity shall not be
controlling.” The purpose is to free administrative boards from the compulsion of technical rules
so that admission of matter deemed incompetent in judicial proceedings would not invalidate the
administrative order. But this does not justify orders without a basis in evidence. Mere
uncorroborated hearsay or rumor does not constitute substantial evidence.

5) The decision must be rendered on the evidence presented at the hearing or at least contained
in the record and disclosed to the parties affected. Only by confining the administrative tribunal to
the evidence disclosed can the parties be protected in their right to know and meet the case against
them.
6) The CIR or its judges must act on its or his own independent consideration of the laws and facts
of the controversy and not simply accept the views of a subordinate in arriving at a decision.
7) The CIR should render its decision such that the parties can know the various issues involved
and the reasons for the decisions rendered.

In this case, the record is barren and there is no factual basis to predicate a conclusion of law. Thus,
the interest of justice would be better served if the movant is given opportunity to present the
documents referred to in his motion.

51. GSIS v. CA & Liwanag, GR 128523, September 28, 1998, Davide, Jr., J. (Due Process;
Administrative or quasi-judicial proceedings; must present “substantial evidence”; must be
an impartial tribunal; must respect findings of quasi-judicial agencies expert on their field)
FACTS:
Zenaida Liwanag is the surviving spouse of the late Jaime Liwanag, who had served in the police
force for 27 years. At the time of his death, he was a senior superintendent of the PNP. He died of
Upper GI Bleeding, Cirrhosis secondary to Hepatitist B. Liwanag filed a claim with the GSIS for
compensation benefits under PD 626, which was denied for not being an occupational disease
under the law. The Employees Compensation Commission (ECC) affirmed the GSIS ruling

The ECC ruled that Jaime’s ailments are not among those listed as occupational diseases nor was
there proof that the risk of contracting the said diseases were increased by Jaime’s working
conditions as PNP. It found that Jaime’s ailments are not inherent among policemen and everybody
is susceptible to the said diseases.

On appeal with the CA, Liwanag relied on 2 documents for proof of the causal relation of the
ailments to Jaime’s working conditions: 1) Investigation report by Police Chief Inspector and 2)
Report of proceedings of Line Of Duty (LOD) board.

The Investigation report stated that “it is highly believed that the late Jaime acquired his illness in
the course of his employment with the PNP considering that there are some personnel in his office
who are positive to Hepatitis B.” It recommended that the death benefits be granted.

The LOD report (transcript) mentioned that it is highly possible that Jaime got infected since there
were 5 personnel out of 45 reactive to Hepatitis B; that “I strongly believe that he got this while
working at headquarters since this is already endemic in the camp.” The board resolved that Jaime
died in the line of duty and moved that benefits should be granted to Jaime’s heirs.

The CA granted the benefits. Hence this appeal by GSIS. It claims that Hepatitis B is not acquired
by simple association with others who are infected. It is often transmitted by blood.

ISSUE:
Whether Jaime Liwanag, who at the time of death was a senior superintendent of the PNP and died
of Hepatitis B, may be granted compensation benefits under PD 626 requiring that the cause of
death be related to or risk of acquisition of disease be aggravated by working conditions where the
claimant surviving spouse bases her assertion of Hepatitis B being work-related upon a PNP
investigation report stating that Jaime’s illness were acquired from working conditions and a Line
of Duty (LOD) report finding that Jaime got infected from other policemen with Hepatitis B.

HELD: NO.
For the death to be compensable, the claimant must prove either 1) that the sickness was the result
of an occupational disease under Annex A of the Rules on Employees’ Compensation or 2) if not
listed therein, that the risk of contracting the disease was increased by working conditions.
Claimant must prove this causal relation by substantial evidence since the proceeding is taken
before the ECC, an administrative or quasi-judicial body. While strict rules of evidence are
not applicable to quasi-judicial proceedings, nevertheless, in adducing evidence, the basic rule
that mere allegation is not evidence applies.

Hepatitis B is not listed as an occupational disease under Annex A. Thus, Liwanag’s burden was
to prove by substantial evidence the causal relationship between Jaime’s illness and his working
conditions. She failed to do so.
Liwanag merely relied on PNP reports and nothing more. But these contained sweeping
statements and conclusions and treated the matter in a most perfunctory manner. The PNP
report stated that it is highly believable that Jaime’s illness was acquired in the course of
employment. But this is based on a layman’s point of view, not on medical findings, and is mere
hearsay. Mere uncorroborated hearsay or rumor does not constitute substantial evidence.

Substantial evidence means such relevant evidence as a reasonable mind might accept as adequate
to support a conclusion.

The PNP Board proceedings and that of ECC are separate and distinct, treating of 2 totally different
subjects. The PNP Board relied on the Ministry of National Defense Dept. Order 162 that a
member of the military who died while in active service is presumed to have died in the line of
duty, but this order concerns itself only as to whether one died in the line of duty, while PD 626
addresses the issue of whether a causal relation existed between a claimant’s ailment and his
working conditions. These are different issues calling for differing forms of proof or evidence.
There is a favorable presumption under the Defense Department order but not under PD 626 when
the disease is not listed in Annex A.

Also, Police chief inspector Gonzalodo, having prepared the Investigation report recommending
the grant of benefits, should have inhibited himself from the proceedings of the PNP Board.
Having prejudged the matter already by way of his recommendation, he could not be said to have
acted impartially as a member of the PNP Board. Ang Tibay failed to explicitly state was, on the
need to ensure due process in administrative proceedings, the requirement of an IMPARTIAL
tribunal.

The CA should have respected the findings of the ECC on the technical matter concerning the
nature of Jaime’s illness. The ECC’s rejection of Liwanag’s claim was not unfounded as it even
took pains to quote from a medical manual to substantiate its holding. Courts are loathe to interfere
with and should respect the findings of quasi-judicial agencies in fields where they are deemed to
be experts due to their special technical knowledge.

52. Senator Jinggoy Estrada v. Office of the Ombudsman, GR 212140-41, January 21, 2015,
Carpio, J. (Due Process; criminal vs administrative case; preliminary investigation requires
probable cause only, administrative cases require “substantive evidence”; cannot apply one
to the other; must exhaust all administrative remedies, else cannot claim due process
violation)
FACTS:
The Ombudsman served upon Sen. Estrada a copy of the complaint (OMB 0313) filed by NBI and
Atty. Baligod, praying for criminal proceedings for plunder as defined in RA 7080. Sen. Estrada
filed his counter-affidavit.

The Ombudsman served a copy of the complaint (OMB 0397) filed by the FIO of the Ombudsman,
praying for criminal proceedings for plunder and for violation of Section 3(e) of RA 3019. Estrada
filed his counter-affidavit.

Estrada requested for copies of the counter-affidavits of his co-respondents in the two complaints
“pursuant to the right of a respondent to examine the evidence submitted by the complainant” (sec
3(b), Rule 112) and “to have access to evidence on record.”(sec 4(c) Rule II of Rules of Procedure
of the Office of the Ombudsman)

The Ombudsman issued the assailed order in OMB 0313 on March 27, 2014 stating that the cited
provisions or rules do not entitle Estrada to be furnished all the filings of the respondents. The
respondents are only required to furnish counter-affidavits to the complainant and not to the other
respondents. Thus, it denied Estrada’s request.

It then resolved in both complaints OMB 0313 and 0397 finding probable cause to indict Estrada
and his co-respondents with plunder and 11 counts of violation of Section 3(e) of RA 3019. Estrada
moved for reconsideration.

But without moving for reconsideration of the March 27, 2014 order in OMB 0313, Estrada filed
this petition for certiorari. He claims that his right to due process was violated and thus the order
of March 27, 2014 is void.

On May 7, 2014, the Ombudsman jointly ordered in both complaints, furnishing Estrada with the
counter-affidavits of his co-respondents, directing him to comment thereon. Estrada filed for a
motion to suspend proceedings in OMB 0313 and 0397 because of this petition, which was denied.
Estrada did not file any comment thereon.

Estrada claims that even if he was given the counter-affidavits, he was still denied due process as
he was not given other counter-affivadits.

ISSUE:
Whether the ombudsman, in denying Estrada’s request to be furnished copies of his co-
respondents’ counter-affidavits to two complaints and in ruling that the respondents are only
required to furnish copies thereof to complainants and not to other respondents but subsequently
furnishing such copies, violated Estrada’s right to due process.

HELD: NO.
There is no law or rule which requires the Ombudsman to furnish a respondent with copies of the
counter-affidavits of his co-respondents. Estrada fails to cite a law or rule stating that it is a
compulsory requirement of due process in a preliminary investigation that the ombudsman furnish
a respondent with the counter-affidavits of his co-respondents. Neither Section 3(b) Rule 112 of
the Revised Rules of Criminal Procedure nor Section 4(c) Rule II of the Rules of Procedure of the
Office of the Ombudsman support his claim.

What the Ombudsman rules require is to furnish the respondent with a copy of the complaint and
supporting affidavits and documents at the time the order to submit the counter-affidavits is issued
to the respondent.

The provision in Section 4(c) of the Ombudsman Rule II that a respondent shall have “access to
the evidence on record” should be read in relation to Sections 4 (a,b) requiring the investigating
officer to furnish respondent with the “affidavits and other supporting documents submitted by the
“complainant or supporting witnesses.” Thus, Section 4(c) refers to the affidavits and supporting
documents of the complainant or supporting witnesses in Section 4(a).

Justice Velasco dissents, relying on the case of Ombudsman v. Reyes, where it was held that Reyes’
right to due process was violated when he was not furnished with copies of the counter-affidavits
of his co-respondent Peñaloza and the latter’s 3 witnesses where both Reyes and Peñaloza were
employees of LTO. In Reyes, the failure to furnish the counter-affidavits happened in the
administrative case on the merits, while Estrada’s denial of his request happened during the
preliminary investigation where the only issue is existence of probable cause to determine
whether an information should be filed. This does not prevent Estrada from requesting a copy of
the counter-affidavits of his co-respondents during the pre-trial or trial.

The quanta of proof and rules vary depending on the cases are criminal, civil, or administrative
in character. In criminal actions, proof beyond reasonable doubt is required; in civil actions,
preponderance of evidence; and in administrative cases, substantial evidence as basis for
adjudication. In criminal and civil actions, the RoC is strictly apply while in administrative
proceedings, technical rules of pleading and procedure and of evidence are NOT STRICTLY
adhere to. They only apply suppletorily.
A preliminary investigation is not part of the trial and merely determines probable guilt and should
be determined in a summary manner. It is only in a trial where an accused can demand the full
exercise of his rights, like the right to confront and cross-examine his accusers. The rights of
respondent in a preliminary investigation are limited to those granted by procedural law.

The quantum of evidence required in preliminary investigation is such as to “engender a well-


founded belief” as to the fact of the commission of a crime and the respondent’s probable guilt
thereof. It is not the occasion for the full and exhaustive display of he parties’ evidence. The
technical rules on evidence are not binding on the fiscal who has jurisdiction and control over the
conduct of a preliminary investigation.

Moreover, the rights of a party cannot be prejudiced by an act, declaration, or omission of another.
The admissions of Estrada’s co-respondents cannot prejudice him.

The Ang Tibay guidelines for administrative cases do not apply to preliminary investigations in
criminal cases. It enumerated constitutional requirements of due process, described as the
fundamental and essential requirements of due process in trials and inestigations of an
administrative character. (guidelines 1-7, see case 50) In GSIS v. CA, the Court further clarified
the requirement of an impartial tribunal. The investigating officer, which is the role of the
Ombudsman, will never be the impartial tribunal as required. Its purpose is to find probable
cause for filing an information, and not to make a final adjudication of the rights and obligations
of the parties under the law, which is the purpose of the guideleins in Ang Tibay. Thus, such
guidelines are inapplicable to preliminary investigations.

Moreover, a person under preliminary investigation as Estrada is not yet an accused person, hence
he cannot demand the full exercise of the rights of an accused person.
The rights conferred upon accused persons in preliminary investigations depend upon the
provisions of law by which such rights are specifically secured, rather than upon the phrase “due
process of law”. The right to a preliminary investigation is statutory, not constitutional. Such
investigation does not initiate a criminal action so as to trigger into operation Section 14(2) of
Article III. It is the filing of a complaint or information in court that initiates a criminal action. The
rights in Ang Tibay and GSIS are constitutional, thus they may not be taken away by legislation.

“Substantial evidence” is greater than probable cause under preliminary investigations. Probable
cause means “a reasonable ground for belief of guilt.” Even hearsay evidence may establish
probable cause as long as there is substantial basis for crediting the hearsay because such
investigation is merely preliminary. But in administrative cases, where rights and obligations are
finally adjudicated, “substantial evidence” is required, which cannot rest on even partially on
hearsay evidence. To apply Ang Tibay to preliminary investigations would change the quantum of
evidence required in determining probable cause.

This petition is also premature as Estrada did not file any pleading or even a MR to the March 27
order in OMB 0313 and immediately proceeded with this petition. A MR is required prior to a
certiorari.
In Ruivivar v. Ombudsman, Ruivivar was reprimanded. She filed a MR on the ground that she was
not furnished copies of the affidavits of the private respondent’s witnesses. The Ombudsman
furnished her such copies and ordered her to file such pleading she may deem fit under the
circumstances. But she simply filed a manifestation maintaining that her receipt of the affidavits
did not alter her deprivation of due process. She received the affidavits after the Ombudsman
decision against her. We ruled that the CA was correct in dismissing her certiorari petition as she
failed to exhaust all administrative remedies available to her before the Ombudsman. She was
given the opportunity to rebut the affidavits and had a speedy and adequate administrative remedy,
but she failed to avail thereof.

Exhaustion of administrative remedies and due process are related principles. The “exhaustion”
principle applies when the ruling court or tribunal is not given opportunity to re-examine its
conclusions because of an available opportunity that a party omitted to take. Under due process,
a violation occurs when a court or tribunal rules against a party without giving him the
opportunity to be heard. The exhaustion principle is considered from the point of view of the
ruling court or tribunal, while due process, from the litigating party against whom a ruling
was made. Ruivivar was given an opportunity to submit “such pleading” she may deem fit. A
party cannot feign denial of due process where he had the opportunity to present his side.

In this case, when the Ombudsman gave Estrada copies of the counter-affidavits and waited for
the lapse of the given period for his filing of comment, Estrada failed to avail of the opportunity
to be heard DUE TO HIS OWN FAULT. His failure cannot be construed as violation of due
process much less GADALEJ.

Lastly, his current petition amounts to forum shopping, having raised in his MR of the March 28
order the same issue of due process violation in the March 27 order as in this petition. His petition
was dismissed.

53. Diosdado Guzman v. National University (NU), GR L-68288, July 11, 1986 (Due Process;
for students in universities to be disciplined, must first be investigated)
FACTS:
Petitioners students of NU seek relief from what they describe as the school’s “continued and
persistent refusal to allow them to enroll.” They claim that the reason therefor is their participation
in peaceful mass actions within the premises of the university. They also claim that they were
expelled without cause or without being informed of such cause and without opportunity to defend
themselves.

NU claims that Guzman lead boycotts of classes and participated in activities within the university
premises without permit from the school that disturbed the classes therein. He is also facing a
charge for malicious mischief before the MTC for destruction of NU property. NU also claims that
petitioners have failures in their records and are not of good scholastic standing.

NU did not conduct any proceeding to determine whether petitioners-students had indeed
participated in activities within university premises etc..

ISSUE:
Whether NU, denying re-enrollment of petitioners who are students of NU on the ground that they
participated in mass actions disturbing the classes in the university and in destroying NU property
without conducting proceedings of any sort, violated the students’ right to due process.

HELD:
Apparent from the comments and memorandum is that NU never conducted proceedings of any
sort to determine whether the petitioners-students had indeed participated in activities within the
university that disturbed classes or perpetrated acts of vandalism, etc. The pendency of a civil case
for damages and a criminal case for malicious mischief against Guzman cannot, without more,
furnish sufficient warrant for his expulsion or debarment from re-enrollment. There is also no rule
cited by which students may be expelled or refused re-enrollment for poor scholastic standing.

Education Act of 1982 grants petitioners as students the right to freely choose their field of study
and to continue their course except for academic deficiency or violation of disciplinary regulations.
Petitioners were denied this right or being disciplined without due process, violating the Manual
of Regulations for Private Schools that no penalty shall be imposed only after due investigation.
Thus, NU’s act of imposing sanctions on students without due investigation is illegal.

The imposition of disciplinary sanctions requires procedural due process. Due process in
disciplinary cases of students does not entail proceedings and hearings like in courts of justice.
Such may be summary and cross-examination is not essential thereto. The minimum standards
which must be met to satisfy procedural due process are (1) the students must be informed in
writing of the nature and cause of any accusation against them; (2) they shall have the right to
answer the charges against them, with the assistance of counsel, if desired; (3) they shall be
informed of the evidence against them; (4) they shall have the right to adduce evidence in their
own behalf; and (5) the evidence must be duly considered by the investigating committee or
official designated by the school authorities to hear and decide the case

54. ADMU v. Hon. Capulong, GR 99327, May 27, 1993, Romero, J. (Due Process;
administrative disciplinary proceeding of a school do not need to meet the technical
procedures in judicial proceedings; Guzman [*case 53] guidelines is applicable in
disciplinary investigations in academic institutions)
FACTS:
Respondents-students seek re-admission to ADMU. They were found guilty of violating Rule 3 of
the Ateneo Law School Rules on Discipline prohibiting participation in hazing activities. One
neophyte died and serious physical injuries were inflicted on another.

As requisite to membership, the Aquila Legis, a fraternity organized in ALS, held its initiation
rites. Leonardo Villa died as a result. Bienvenido Marquez was also hospitalized.

Dean Castillo, in a notice, created a joint administration faculty-student investigating committee


to investigate and submit a report on the circumstances surrounding the death of Villa. Said notice
also required respondent students to submit their written statements. The students failed to file a
reply. They were preventively suspended. They requested for copies of the charges.
The investigating committee, in an order on February 14, 1991, found a prima facie case against
the students for violating Rule 3 of the Law School Catalogue entitled “Discipline.” The students
were required to file their written answers to the formal charge.

Respondents-students were directed to appear before the committee for clarificatory questions on
their answer. They were also informed that the proceedings will be summary in nature in
accordance with the rules in Guzman v. NU, they have no right to cross-examine the affiants-
neophytes, hazing shall be defined in accordance with SB 3815 proposed by Sen.Lina, and that the
board’s decision will be appealable to the university president, Joaquin Bernas.

The students were found guilty of violating Rule 3 of ALS rules on discipline. They were found
to have acted as “auxies” during the initiation and took part in the physical hazing. As to the
penalty, there was lack of unanimity among the members as to dismissal, so the Board left the
penalty to the university administration.

Petitioner Bernas, as president of ADMU, accepted the factual findings of the Board. He imposed
the penalty of dismissal on the respondent students.

Respondent students filed with the RTC a petition for certiorari, prohibition, and mandamus
alleging that they were currently enrolled as students and unless a TRO is issued, they would be
prevented from taking their exams. The petition centered on the alleged lack of due process in their
dismissal. The RTC, thru judge Madayag, issued a TRO.

When it expired, the students filed a supplemental petition of certiorari with prayer for TRO. Judge
Amin, as pairing judge of respondent Judge Capulong, granted the prayer.

Judge Capulong issued a preliminary injunction upon posting of bond of the students, ruling that
the students had been denied due process in the investigation. Hence this special civil action of
certiorari questioning the order of Judge Capulong reinstating respondent students.

ISSUE:
Whether ALS is within its rights and observed procedural due process in expelling students from
its academic community pursuant to its disciplinary rules and moral standards after investigation
and giving said expelled students opportunity to adduce evidence in their behalf but not to cross
examine the witnesses and documents against them where the students violated such rules when
they were found to have participated in initiation rites involving hazing which led to the death of
the neophyte Leonardo Villa and the hospitalization for serious physical injuries of Bienvenido
Marquez.

HELD: YES.
The students claim that the decision of Bernas to expel them was arrived at without affording them
their right to procedural due process. On the contrary, respondent students’ rights in a school
disciplinary proceeding have been respected by petitioners in the various investigative
proceedings held before they were expelled.
They claim that it is the Ang Tibay case and not the Guzman case which is applicable. However,
the Guzman case is more apropos in this case since it deals specifically with the minimum
standards to be satisfied in the imposition of disciplinary sanctions in academic institutions. (*see
case 53 for the due process requirements 1-5 in academic institutions)

Dean Castillo of the ALS notified and required respondents to submit their written statement on
the incident within 24 hours, but they instead requested copies of the charges. While some
submitted their written statements, the others failed to do so. The latter were granted an extension
to file their statements.

The nature and cause of the accusation were adequately spelled out in petitioners’ notices. They
were given ample opportunity to adduce evidence in their behalf. The requisite assistance of
counsel was met when the law firm of Gonzales Batiller and Bilog and Associates appeared and
filed pleadings in behalf of respondents.

Respondents cannot argue that since they did not see the written statements which became the
basis of petitioners’ February 14, 1991 order finding a prima facie case, they were denied
procedural due process. This would not detract from the observance of due process, for disciplinary
cases involving students need not necessarily include the right to cross examination. An
administrative proceeding to investigate students’ participation in a hazing activity need not be
clothed with the attributes of a judicial proceeding.

The respondents’ claim that the investigating committee failed to consider their evidence is far
from the truth as the February 14 order states that it was reached only after receiving the written
statements and testimonies of several witnesses.

Respondents claim that the word “hazing” was not explicitly defined. The charge filed before the
investigating committee and disciplinary Board is not a criminal case requiring proof beyond
reasonable doubt but is merely administrative in character. It is not subject to rigorous
requirements of criminal due process particularly with respect to the specification of the charge.
We deem the notice of the use of the definition of “hazing” in SB 3815 as sufficient for purposes
of the investigation under scrutiny.

Respondents claim that the petitioners cannot file this petition because they failed to file a MR
first with the RTC, bypassing it and the CA. An exception to the doctrine of exhaustion of remedies
is when the case involves a question of law, as in this case where the issue is whether the students
have been afforded procedural due process before their dismissal.

The essential freedoms under “academic freedom” includes 1) who may teach; 2) who mayb e
taught; 3) how it shall be taught; and 4) who may be admitted to study. As found in Article XIV,
Section 5 (2) states: "Academic freedom shall be enjoyed in all institutions of higher learning." It
was left to the courts to develop further the parameters of academic freedom. This freedom is also
enjoyed by the faculty and students.

55. Cudia v. The Superintendent of the PMA, GR 211362, February 24, 2015, Peralta, J. (Due
Process; procedural due process in administrative proceedings need not be like judicial
proceedings; PMA disciplinary proceedings, being an academic institution, follows uzman v.
NU guidelines for procedural due process; proof need only be substantial evidence; military
disciplinary proceedings- enough that accused is apprised of charge and given opportunity
for defense, no need to be represented by counsel like in other administrative proceedings
since proceeding is non-litigation)
FACTS:
6 days before March 16, 2014, graduation ceremonies of PMA, Cudia filed this petition for
certiorariHe was a member of Siklab Diwa class of 2014 and supposed to graduate as class
salutatorian.

They had a lesson examination (LE) on Operations Research (OR432) under Dr. Costales from
1:30-3:00pm. Cudia was late for his next class for 2 minutes as he was inquiring with Dr. Costales
after the LE. He explained that he was dismissed “a bit late” by Dr. Costales, but Costales denies
dismissing the class late. Maj. Hinand reported Cudia to the Honor Committee (HC) for violation
of the Honor Code for lying.

In his explanation with HC, he said he considers the duration of “class” as including not just the
lecture but every communication a teacher does with students, so it was only reasonable for him
to say that his class was dismissed a bit late. The HC found him first 8-1 guilty-not guilty, then
with a 9-0 guilty vote. His appeal with the HC chairman was denied. The Staff Judge Advocate
(SJA) found the report to be legally in order. The commandant of cadets affirmed the HC findings
and recommended Cudia’s dismissal. Vice Admiral Abogado approved the recommendation to
dismiss Cudia.

Cudia’s parents sent a letter to the new PMA superintendent Maj. Lopez asking to recognize the
8-1 voting instead. Lopez referred the matter to the Cadet Review and Appeals Board (CRAB).

Cudia requested for copies of the minutes of the HC proceedings, documents pertaining to the
case, and video recordings of HC hearings. This was denied.

The AFP Chief of Staff ordered a reinvestigation after the FB post of their plight of Annavee
Cudia, Cudia’s sister, went viral. Cudia’s parents also filed a complaint with the CHR alleging
violation of Cudia’s human rights, particularly his rights to due process, education, and privacy of
communication.

The CHR came out with its preliminary findings, recommending the 8-1 vote be upheld and thus
for the PMA to announce that Cudia was not guilty.

The Office of the President (OP) sustained the findings of the AFP Chief of Staff and CRAB.
Hence this appeal, claiming Cudia was denied his right to due process as his access to evidence to
prove his defense was denied, that he was vaguely informed of the HC decisions, and that he was
afforded a sham trial by the HC, CRAB, and PMA.

ISSUE:
Whether Cudia, who was dismissed from the PMA for alleged lying by claiming that his earlier
Operations Research class was dismissed “a bit late” as reason for his tardiness for his next class,
thereby being subjected to investigation and found guilty of lying in violation of the first tenet of
the Honor Code of PMA and who in the process of such investigation was denied access to
documents and pieces of evidence he requested for such as the Honor Committee deliberations
and relevant case documents allegedly for the purpose of proving his defense, was denied his right
to due process.

HELD: NO.
-On judicial interference in military affairs
While the constitutional permissibility of the military to set and enforce uncommonly high
standards of conduct and ethics is recognized, the courts have expanded the scope of judicial
review of military determinations. Section 1, Article VIII of the 1987 Constitution expanded the
scope of judicial power by mandating that the duty of the courts includes determining whether
there has been GADALEJ on the part of any branch or instrumentality of the government
even if the latter does not exercise judicial, quasi-judicial, or ministerial functions. Grave abuse
implies such capricious and whimsical exercise of judgment as is equivalent to lack of jurisdiction
or where the power is exercised in an arbitrary or despotic manner by reason of passion or personal
hostility which must be so patent and gross as to amount to an evasion of a positive duty or a
virtual refusal to perform the duty enjoined or to act at all in contemplation of law.

The proceedings of the HC can, for purposes of due process, be considered a governmental
activity. The relationship between the HC and the separation process at the Academy has been
sufficiently formalized and is sufficiently interdependent so as to bring that committee’s activities
within the definition of governmental activity. No one is above the law. Consistent with checks
and balances, the Court has been entrusted with the duty to determine in appropriate cases the
validity of any legislative or executive action.

-Respondent’s claim that the standard of rights of a cadet is not the same as a civilian since a
cadet’s rights have been recalibrated to best serve military purpose and necessity.
A student at a military academy must be prepared to subordinate his private interests for the
proper functioning of the educational institution he attends. The PMA Honor Code states that the
cadet is required to surrender basic rights and liberties for the good of the group. But a cadet facing
dismissal from the military academy for misconduct has constitutionally protected private
interests (life, liberty, property). Thus, disciplinary proceedings must be conducted within the
bounds of due process. The PMA is not immune from the strictures of due process. Where a
person’s good name, reputation, honor, or integrity is at stake, due process must be satisfied.

-PMA’s Academic Freedom.


The school-student relationship is contractual in nature. Such contract is imbued with public
interest due to the high priority given by the Constitution to education. It is also reciprocal, giving
rise to reciprocal rights and obligations. The four essential freedoms of a university are: To
determine for itself on academic grounds 1) who may teach, 2) what may be taught, 3) how it
shall be taught, and 4) who may be admitted to study. An educational institution has the power to
adopt such rules expedient for its government, this being incident to the very object of
incorporation and indispensable to the successful management of the college.
The school’s power to instill discipline is subsumed in academic freedom (“How it shall be taught)
and the establishment of rules pertaining to discipline is vital to its very survival. This includes the
power to impose disciplinary sanctions and to expel students who violate such rules. The right to
discipline is also evident in “who may be admitted to study.” The school may determine whom to
admit and whom to exclude or expel or impose lesser sanctions upon.

The PMA has the right to invoke academic freedom in enforcing its internal rules, the Honor Code
and Honor System particularly.

-Procedural safeguards in a student disciplinary case.


Respondents assert that Cudia was accorded due process using the case of Guzman v. NU while
petitioners argue that it is Ang Tibay v. CIR that applies.

Guzman is more apropos since it deals with the minimum standards to be satisfied in the imposition
of disciplinary sanctions in specifically academic institutions. It is the authority on procedural
rights of students in disciplinary cases. (*Case 53, Guzman v. NU, see 1-5 minimum standards)

Due process in student disciplinary cases do not entail proceedings like in courts. The
proceedings may be summary and cross examination is not an essential part of the investigation
or hearing. The required proof in student disciplinary action, an administrative case, is only
SUBSTANTIAL EVIDENCE or such relevant evidence as a reasonable mind might accept as
adequate to support a conclusion.

Notice and hearing is the bulwark of administrative due process. The essence of due process is an
opportunity to be heard or, as applied to administrative proceedings, an opportunity to explain
one’s side or to seek reconsideration of the action or ruling complained of. A formal trial-type
of hearing is not at all times essential. It is enough that the parties are given a fair opportunity to
explain their respective sides and to present supporting evidence on which a fair decision can be
based.

The PMA Honor Code recognizes that an administrative proceeding to investigate a cadet’s honor
violation need not be clothed with the attributes of a judicial proceeding. There is aversion towards
undue judicialization of an administrative hearing in the military academy. Over-proceduralizing
military determinations gives soldiers less time to prepare in fighting wars.

Our military has been patterned after the US Army and US military code produced a salutary effect
in the PH military justice system. Thus, US case laws are persuasive.

Due process is a flexible concept. The flexibility inherent in due process precludes the dogmatic
application of specific rules developed in one context to entirely distinct forms of government
action. To determine what procedures due process requires, the court must carefully determine
and balance the nature of the private interest affected and the government interest involved,
taking into account of history and the precise circumstances surrounding the case. While the
government must have a legitimate concern with the subject matter before it may validly affect
private interests, private interest must yield in vital areas of government concern like national
security and military affairs. Thus, says the US court, due process requires for the dismissal of a
cadet that he be given a fair hearing at which he is apprised of the charges against him and
permitted a defense. He must be given opportunity to present his defense both from the point of
view of time and the use of witnesses and other evidence. The hearing may be procedurally
informal and need not e adversarial.

Cudia’s investigation followed the prescribed procedure and existing practices in PMA. He was
notified of Maj. Hindang’s report and given opportunity to explain the report against him. He
was informed of his options and of the entire process of the case. A preliminary investigation
followed after he submitted his written explanation. The investigating team them submitted a
written report to the HC Chairman. The HC then reviewed the findings and recommendations.
When the honor case was submitted for formal investigation, a new team was assigned to conduct
the hearing, and during the formal investigation, he was informed of the charge against him and
given the right to enter a plea. He had the chance to explain his side, confront witnesses, and
present evidence. His guilty verdict was reviewed from the OIC of the HC, to the SJA, to the
commandant of cadets, and to the PMA superintendent. A review was also conducted by CRAB
upon directive of AFP-GHQ.

By reason of their special knowledge and expertise from handling specific matters falling under
their respective jurisdictions, the factual findings of administrative tribunals are ordinarily
accorded respect if not finality unless such findings are not supported by evidence or vitiated by
fraud, imposition, or collusion, etc. In this case, there is no reason to deviate from the general rule.

-As to right to be represented by counsel.


Petitioners claim that respondents must give Cudia the right to be represented by counsel not just
in assisting him in preparing for the investigative hearing before the HC and CRAB but in
participating fully in said hearings.

We disagree. There is nothing in the Constitution stating that a party in a non-litigation proceeding
is entitled to be represented by counsel. The assistance of a lawyer, while desirable, is not
indispensable. A party in an administrative inquiry may or may not be assisted by counsel and
no duty rests on such body to furnish counsel. Where the proceeding is non-criminal or
investigative and not adversarial and the government does not proceed through counsel, where
the individual concerned is mature and educated, where his knowledge of the events should
enable him to develop the facts adequately through available sources and where other aspects of
the hearing taken as a whole are fair, due process does not require counsel.

In general, US courts decline to recognize a right to representation by counsel in military


disciplinary proceedings. This is motivated by the policy of treading lightly on the military domain
with broad confines of constitutional due process and the courts’ views that disciplinary
proceedings are not judicial and should be kept informal, and that educated cadets should be able
to defend themselves.

Cudia was assisted by a PAO lawyer. Due process is satisfied as he was aided in drafting and filing
the appeal memorandum and the counsel even acted as observer who had no right to actively
participate in the proceedings. The offense of Cudia is not criminal. The hearings before HC and
CRAB are investigative and not adversarial. Cudia’s excellent academic standing puts him in
the best position to look after his own vested interest in the Academy.

-As to confidentiality of records in the proceedings.


Petitioners claim that Maj. Lopez denied Cudia’s request for documents and recordings of HC
hearings, the vital evidence negating the regularity of the HC trial. Respondents do not deny the
documents’ existence but refused to present them for the parties and this Court. Petitioners want
the Court to assume that the documents and footages withheld are favorable to Cudia and
respondents’ refusal to produce them is denial of procedural due process. They are wrong.

Petitioners have not particularly identified any documents, witness testimony, or oral or written
presentation of facts submitted at the hearing that would support Cudia. The Court may require
that an administrative record be supplemented, but only where there is a strong showing of bad
faith or improper behavior on the part of the agency, both of which are not present.

-As to late and vague decisions.


It is claimed that Cudia was kept in the dark of the charge against him and of the decisions of HC,
CRAB, and PMA. No written decision was furnished to him and the information was unjustly
belated and the justifications vague.

While there is a constitutional mandate that no decision shall be rendered by any court without
expressing therein clearly the facts and the law on which it is based, this does not apply to Cudia’s
case. The Honor Code and Honor System Handbook has no written rule on the matter. What counts
is that, albeit furnished late, Cudia was informed of how it was decided with an explanation of the
factual and legal reasons leading to the conclusions of the reviewing body.

-As to blind adoption of HC findings.


Petitioners claim that the PMA blindly followed the HC’s finding of guilt. Respondents assure that
there was a proper assessment of the procedural and legal correctness of the guilty verdict and
there was a separate investigation by HTG. We agree with respondents.

The HC’s task in case of Honor Code breach is entirely investigative, examining in the first
instance a suspected violation. The HC does not have authority to order the separation of a cadet.
The results of its proceedings are purely recommendatory and have no binding effect. But even if
the HC is covered by the due process clause, it is not required that procedural due process be
afforded at every stage of developing disciplinary action. What is required is that an adequate
hearing be held before the final act of dismissing a cadet. In Cudia’s case, the OIC of HC, SJA,
Commandant of Cadets, and PMA Superintendent reviewed the HC findings. A separate
investigation was also conducted by HTG.

-Nature of CHR findings.


The findings of fact and conclusions of law of the CHR are merely recommendatory and not
binding to this Court. The CHR’s constitutional mandate extends only to the investigation of all
forms of human rights violations involving civil and political rights. It is only a fact-finding body,
not a court of justice or quasi-judicial agency.
The SC dismissed the case.

EQUAL PROTECTION
56. People v. Cayat, GR 45987, May 05, 1939, Moran, J. (Equal Protection; classification of
non-Christian tribes for the purpose of facilitating their steady civilization by the
government is valid)
FACTS:
Cayat is charged for violation of Act 1639. His information states that “being a member of the
non-Christian tribes, xxx, have in his possession xxx, one bottle of A-1-1 gin, an intoxicating
liquor, other than the so-called native wines and liquors which the members of such tribes have
been accustomed themselves to make prior to the passage of Act 1693.”

Section 2 of Act 1639 states that it is unlawful for any member of a non-Christian tribe to but,
receive, possess, or drink any intoxicating liquor other than native wines which the tribes have
been accustomed themselves to make prior to the passage of Act 1639.

Cayat claims that the law is discriminatory and violates equal protection, due process, and is an
improper exercise of police power.

ISSUE:
Whether a law prohibiting members of non-Christian tribes from possessing, drinking, receiving,
etc. intoxicating liquors except those native wines which the tribes have been accustomed
themselves to make prior to the passage of the law is discriminatory and violates equal protection,
due process, or is an improper exercise of police power.

HELD: NO.
As early as 1551, the Spanish Government regarded it a sacred duty to conscience and humanity
to civilize these less fortunate people living in the “obscurity of ignorance.” This policy continues
during the American Period. Since then, the government has been vexed with the problem of
determining practicable means to bring about their advancement in civilization and material
prosperity. To complement this policy, the Legislature has passed Act 1639 to secure for them
the blessings of peace and harmony and to facilitate their steady march to civilization.

The guaranty of equal protection is not violated by legislation based on reasonable classification.
The classification, to be reasonable, e, (1) must rest on substantial distinctions; (2) must be
germane to the purposes of the law; (3) must not be limited to existing conditions only; and (4)
must apply equally to all members of the same class.

Act 1639 rests on substantial or real distinctions. It is based on the degree of civilization and
culture. The term “non-Christian tribes” refers, not to religious belief, but to the geographical area
and to natives of the PH with a low grade of civilization. This distinction is reasonable, for the Act
was intended to meet the peculiar conditions existing in non-christian tribes.

It is germane to the purpose of the law. The prohibition is designed to insure peace and order in
and among the non-Christian tribes. The experience of the past disclose that free use of
intoxicating liquors by the non-Christian tribes often resulted in lawlessness and crimes,
hampering efforts of the government to civilize them.
It is not limited to existing conditions only, but it is intended to apply for all times as long as
those conditions exist. The legislature understood that civilization of a people is a slow process
and such must have measures of protection and security.

It applies equally to all members of the same class evident from a perusal thereof. That it may
be unfair against some non-Christians by reason of their degree of culture is not an argument
against the equality of its application.

It is contended that the provision allowing any police officer or authorized agent to seize and
destroy prohibited liquors found in the possession of any member of the non-Christian tribe is
violative of due process. But to constitute due process, notice and hearing are not always
necessary. Due process of law means simply: (1) that there shall be a law prescribed in harmony
with the general powers of the legislative department of the government; (2) that it shall be
reasonable in its operation; (3) that it shall be enforced according to the regular methods of
procedure prescribed; and (4) that it shall be applicable alike to all citizens of the state or to all
of a class.

The Act is also a valid police power measure. Any measure intended to promote the health, etc. of
the people or ti increase state industry, develop its resources, etc. is a valid exercise of police power
unless shown to be capricious as to interfere with the rights of individuals. It is designed to
promote peace and order in the non-Christian tribes to remove obstacles to their moral and
intellectual growth and to hasten their equalization with the rest of their Christian brothers.
The purpose is to unify the Filipino people with a view to a greater PH.

57. Philippine Association of Service Exporters v. Drilon (See case #10)

58. Eleazar Quinto v. COMELEC, GR 189698, February 22, 2010, Puno, C.J. (Equal
Protection; distinction between appointive and elective officials with respect to automatic
resignation upon filing of certificate of candidacy does not violate equal protection)
FACTS:
The assailed decision granted the petition for certiorari filed by Quinto and Tolentino with the SC
and declared as unconstitutional the 2nd proviso in the 3rd paragraph of Section 13 of RA 9369,
Section 66 of the Omnibus Election Code (OEC), and Sec 4(a) of COMELEC Resolution 8678 on
the ground that they violate equal protection and suffer from overbreadth. Thus, the assailed
decision allowed public appointive officials to continue discharging their functions
notwithstanding their filing of certificates of candidacy for election.

This MR by COMELEC argues that the provisions do not violate equal protection when they
accord differential treatment between elective and appointive officials as such treatment rests on
substantial distinctions and is germane to the law’s purpose.

The assailed decision struck down the aforesaid decision for violating equal protection and for
being overbroad as they prohibit the candidacy of all civil servants holding appointive posts 1)
without distinction as to whether they occupy high/influential positions in the government and 2)
they limit these civil servants’ activity regardless of whether they be partisan or nonpartisan in
character, or whether on the national, municipal, or barangay level, and that Congress has not
shown any compelling state interest to restrict the fundamental right of these public appointive
officials.

ISSUE:
Whether provisions of law (section 13 of RA 9369, 66 of the OEC, and section 4(a) of COMELEC
Resolution 8678) deeming appointive officials as resigned from their office upon submission of
certificate of candidacy for public office but not so deeming elective officials who submit the same
violate the equal protection of the laws.

HELD: NO.
We reverse our December 01, 2009 decision

-Section 4(a) of COMELEC Resolution 8678 is compliant with law.


This section provides that under Section 13 of RA 9369, reiterating Section 66 of the OEC, any
person holding a public appointive office or position shall be considered resigned upon filing of
his certificate of candidacy. But for elective officials, they are not deemed resigned upon such
filing. These implement Section 2(4) of Article IX-B of the Constitution prohibiting civil service
officers from engaging in any electioneering or partisan political campaign.

But Section 2(4) of Article IX-B and the implementing statutes apply only to civil servants holding
apolitical offices. It does NOT cover elected officials. This is because elected public officials, by
the very NATURE OF THEIR OFFICE, engage in partisan political activities almost all year
round even outside the campaign period. Political partisanship is the inevitable essence of a
political office, elective positions included.

Notwithstanding, civil service officers are allowed to vote and express their political views or
mention the names of candidates for public office whom they support. This is clear from the
deliberations of the Constitutional Commission that there was no intention to disenfranchise any
government.

-The provisions do not violate equal protection.


Equal protection is not absolute, but is subject to reasonable classification. If the groupings are
characterized by substantial distinctions that make real differences, one class may be treated and
regulated differently from the other. It is against undue favor or class privilege and hostile
discrimination, but not intended to prohibit legislation limited either in its object or territory. It
merely requires that all persons under like circumstances shall be treated alike both as to privileges
conferred and liabilities enforced.

Substantial distinctions exist between elective and appointive officials. Elective officials occupy
their office by virtue of the mandate of the electorate and may be removed only upon stringent
conditions. Appointive officials hold their office by virtue of their designation by an appointing
authority. Appointive officials are also prohibited under the Administrative Code from engaging
in any partisan political activity or taking part in any election except to vote while elective officials
are allowed.
-Germane to purposes of the law.
The test developed to determine a valid classification is that of reasonableness, which has four
requisites: 1) The classification rests on substantial distinctions; 2) It is germane to the purposes
of the law; 3) It is not limited to existing conditions only; and 4) It applies equally to all members
of the same class.

The assailed decision stated that the measure is not germane to the law’s purpose. But the fact that
a legislative classification is underinclusive will not render it unconstitutionally arbitrary. There
is no requirement that regulation must reach each and every class or that the legislature must
regulate all or none.

An equal protection challenge must show a “palpably arbitrary or capricious” classification. He


must refute ALL possible rational bases for the differing treatment whether cited or not by the
legislature. The law will be upheld even if the reasonableness of the classification is “fairly
debatable.” The petitioners failed to do this.

Considering that elected officials are put in office by their constituents for a definite term, it may
justifiably be said that they were excluded from the ambit of the “deemed resigned” provisions.
Deference is accorded to the will of the electorate. This is not applicable to appointive officials.
Thus, the distinction is germane to the purposes of the law. The assailed decision would equalize
the playing field, but it would favor a situation in which the evils exist at the behest of both
appointive and elected officials over another where a portion thereof is contained. That elected
officials have greater political clout is a matter for the legislature to consider as it is within its
power to make the deemed resigned provisions apply to elected officials should it later decide that
the evils sought to be prevented are of such magnitude as to tilt the balance in favor of expanding
the class.

The US SC, faced with the issue of whether a law prohibiting federal and state employees from
taking an active part in political campaigns as unconstitutional as to warrant facial invalidation,
upheld the law stating that 1) in regulating the speech of its employees, the state as employer has
interests that differ significantly from those it possesses in regulating the speech of the citizenry
in general; 2) the court must balance the legitimate interest of employee free expression against
the interests of the employer in promoting efficiency of public services; 3) if employees’
expression interferes with the maintenance of efficient and regularly functioning services, the
limitation on speech is not unconstitutional; and 4) the Legislature is given some flexibility in
ascertaining which positions are to be covered by any statutory restrictions.

Where conduct and not merely speech is involved, the overbreadth of a statute must not only be
real, but also substantial. The assailed decision’s submission that the right to run for public office
is “inextricably linked” with two freedoms- expression and association- lies on barren ground.
American case law never recognized a fundamental right to express one’s political views
through candidacy. One’s interest in seeking office, by itself, is not entitled to constitutional
protection. This is also not under freedom of association absent any allegation that, by running for
an elective position, one is advancing the political ideas of a particular set of voters.
Thus, it is clear that the provisions do not violate equal protection. They substantially serve
government interests: 1) efficient civil service faithful to the government and the people rather
than to party; 2) avoidance of the appearance of “political justice” as to policy; 3) avoidance of the
danger of a powerful political machine; and 4) ensuring that employees achieve advancement on
their merits and that they be free from coercion and the prospect of favor from political activity.

-The provisions do not suffer from overbreadth.


The assailed decision states that the provisions limit the candidacy of all civil servants without due
regard for the type of position being held and without regard for the type of office being sought,
whether partisan or nonpartisan in character; thus they are overbroad.

The assailed decision assumes that the evils sought to be prevented exist only when appointive
officials hold an influential post. It fails to consider an equally plausible threat: the danger of
systematic abuse perpetuated by a powerful political machine that has amassed the scattered
powers of government workers so as to give itself and its incumbent workers an unbreakable grasp
on the reins of power. Thus, the limitation regardless of the incumbent appointive official’s
position is valid.

It also states that the restrictions, insofar as it precludes even candidates for non-partisan elective
offices, are overbroad. But Resolution 8678 refers to the filing of certificates of candidacy and
nomination of official candidates of registered political parties in connection with the May 10,
2010 national and local elections. The provisions were issued particularly for the May 10, 2010
elections which are partisan in character. The restrictions thus apply only to candidacies of
appointive officials vying for partisan elective posts. But even if the rules would apply to non-
partisan public offices, the overbreadth challenge would still be futile. The government has an
interest in regulating the conduct and speech of its employees that differes significantly from
those it possesses in connection with regulating speech of the citizenry.

The question is a matter of degree. Measuring the substantiality of a statute’s overbreadth would
entail a rough balancing of the number of valid applications with invalid applications. Assuming
that the partisan-non-partisan distinction is necessary such that a statute failing to make this
distinction is susceptible to overbreadth attack, the challenge must demonstrate the number of
potentially invalid elections.

In this case, the probable harm to society in permitting incumbent appointive officials to remain
in office as they pursue elective posts far outweight the less likely evil of having arguably protected
candidacies blocked by the possible inhibitory effect of a potentially overly broad statute.

59. Louis Biraogo v. The Philippine Truth Commission of 2010, GR 192935, December 07,
2010, Mendoza, J. (Equal Protection)
FACTS:
During Pres. Aquino’s campaign, his battlecry was “kung walang corrupt, walang mahirap.” He
was voted into the presidency. To transform his campaign slogan into reality, Pres. Aquino issued
EO 1, creating the Philippine Truth Commission of 2010.
EO1 granted the powers of an investigative body to the Commission. It was tasked to conduct a
fact-finding investigation of reported cases of corruption during the previous administration
and to submit thereafter its recommendations.

Cases were filed assailing EO1 as violative of Congress’ legislative power, usurping its authority
to create a public office and to appropriate funds therefor, and for violating the equal protection
clause for selectively targeting the past (GMA) administration.

ISSUE:
Whether EO1, creating the Philippine Truth Commission (PTC) of 2010 which was tasked to
conduct a fact-finding investigation of reported graft and corruption during the administration of
then President GMA, is a valid executive issuance.

HELD: NO.
-Nature of the Truth Commission
The PTC is a mere ad hoc body formed under the OP. It is an entity within the OP Proper and
subject to the president’s control. It constitutes a public office, as an ad hoc body is one. It is not
a quasi-judicial body as it cannot adjudicate. It can only gather evidence and make
recommendations.

Truth commissions have been described as bodies that share the following characteristics: (1) they
examine only past events; (2) they investigate patterns of abuse committed over a period of time,
as opposed to a particular event; (3) they are temporary bodies that finish their work with the
submission of a report containing conclusions and recommendations; and (4) they are officially
sanctioned, authorized or empowered by the State. Their main goals range from retribution to
reconciliation. The marching order of PTC is the identification and punishment of perpetrators.

-Legal standing
Petitioner-legislators have legal standing as they claim usurpation of the power of Congress as a
body to which they belong as members. Legislators have legal standing to see to it that the
prerogative, powers, and privileges vested by the Constitution in their office remain inviolate.

Petitioner-taxpayer Biraogo, while not in danger of sustaining any injury, the Court leans on the
doctrine that the rule on standing may be relaxed when the public interest so requires, such as when
the matter is of transcendental importance of overreaching significance to society.

-President’s power to create PTC.


It is argued that the creation of public office lies within the province of Congress and not with the
executive branch of government. They claim that the president’s delegated authority to reorganize
under the Revised Administrative Code (RAC) does not permit him to create a public office and
is limited to the reorganization of the administrative structure of the OP.

Does the creation of PTC fall within the power to reorganize under Section 31 of the RAC? Section
31 contemplates “reorganization” as limited by the following: 1) restructuring the internal
organization by abolishing or merging units or transferring functions from one unit to another; or
2) transferring any function or agency from OP to any department and vice versa. The provision
thus refers to reduction of personnel, consolidation of offices, or abolition thereof for
redundancy of functions. The creation of an office is not mentioned. Thus, the answer to the
question is no.

The PTC creation is not justified by the power of control, which is the power to alter or modify or
set aside what a subordinate officer had done in the performance of his duties and to substitute the
judgment of the former for that of the latter. This is entirely different from the power to create
public offices. Control is inherent in the executive, while creation of office finds basis from either
a valid delegation from Congress or his inherent duty to faithfully execute the laws.

The creation of PTC, however, finds justification under Section 17 of Artcile VII of the
Constitution, imposing upon the president to ensure that the laws are faithfully executed. The
allocation of power in the 3 branches of government is a grant of all powers inherent in them. The
president’s power to investigate to aid in ensuring the faithful execution of laws (public
accountability in this case) is inherent in the president’s power as Chief Executive. He is given
much leeway in ensuring that laws are faithfully executed and not limited to those specific powers
under the Constitution. One of these recognized powers is the power to create ad hoc committees
flowing from the need to ascertain facts and determine if laws have been faithfully executed.

As for the funds, there will be no appropriation but only an allocation of existing funds already
allocated to the OP to the PTC.

-Power of PTC to investigate.


Fact-finding is not adjudication. The function of receiving evidence and ascertaining therefrom the
facts of a controversy is not a judicial function. Thus, the PTC will, contrary to petitioner’s
apprehensions, not supplant the Ombudsman or DOJ or erode their powers; if at all, the PTC will
complement them. The recommendation to prosecute is but a consequence of the overall task of
conducting a fact-finding investigation, but it is not PTC itself that would prosecute the offenders.

-Violation of the Equal Protection Clause


Petitioners claim that the singling out of the “previous administration” as its sole object makes
the PTC an “adventure in partisan hostility.” It must cover all administrations previous to that of
GMA to be valid.

Respondents claim that EO1 identifies the previous administration as the initial subject of the
investigation. The PTC will not confine itself to cases solely in the said administration, following
Section 17 of EO1. They claim that the Arroyo administration is distinguished from past
administrations 1) due to widespread reports of large scale graft and corruption therein, 2) unlike
administrations long gone, the current administration will most likely bear the immediate
consequence of the policies of the previous administration, 3) evidence of possible criminal
activity are more easily established in the immediately preceding administration, and 4) many
administrations subject the transactions of their predecessors to investigations to provide closure
to issues pivotal to national life.

The equal protection of the laws is embraced in the concept of due process, as every unfair
discrimination offends the requirements of justice and fair play. It merely requires that persons
similarly situated should be treated alike as to rights conferred and responsibilities imposed. It
permits classification that is reasonable, that is: (1) The classification rests on substantial
distinctions; (2) It is germane to the purpose of the law; (3) It is not limited to existing conditions
only; and (4) It applies equally to all members of the same class. Superficial differences do not
make for a valid classification.

The classification must include or embrace all persons who naturally belong to the class. It is
not necessary that the classification be made with absolute symmetry in the sense that the members
of the class should possess the same characteristics. SUBSTANTIAL SIMILARITY will suffice.
The mere fact that an individual belonging to a class differs from other members, as long as that
class is substantially distinguishable from all others, does not justify the non-application of the
law to him.

It must not be based on existing circumstances only, or so constituted as to preclude addition


to the number included in the class. It must embrace all those who may thereafter be in similar
circumstances. It must not leave out or underinclude those that should otherwise fall into a certain
classification.

EO1 thus violates the equal protection clause. It singles out the previous administration. The
Arroyo administration is but JUST A MEMBER OF A CLASS, that is, a class of PAST
ADMINISTRATIONS. It is not a class of its own. Not to include past administrations similarly
situated constitutes arbitrariness. The distinctions enumerated by respondents OSG are not
substantial enough to merit restriction of the investigation to the previous administration. The
reports of widespread corruption in the Arroyo administration are not inherent and do not inure
solely thereto.

The reason that “to include other past administrations” may overburden the commission and lead
it to lose effectiveness is specious. It is irrelevant to the legitimate objective of PTC to end
corruption.

The probability that there would be difficulty in unearthing evidence or that the earlier reports of
earlier administrations were already inquired into is beside the point. Deceased presidents and
cases which have already prescribed can no longer be the subjects of PTC’s inquiry. Neither is
PTC expected to conduct simultaneous investigations of previous administrations given its limited
time and resources. The law does not require the impossible.

However, the fact remains that EO1 suffers from arbitrary classification. To be true to its mandate,
the PTC must not exclude other past administrations. It must at least have authority to investigate
all past administrations. While reasonable prioritization is permitted, it should not be arbitrary.

While it may be argued that the PTC, as an ad hoc body, has a limited scope, equal protection
cannot be circumvented. While the thrust of PTC is specific (investigation of corruption), EO1
must be read together with the Constitution. A number of such classifications have been held or
assumed to be arbitrary; those include: race, national origin, gender, political activity or
membership in a political party, union activity or membership in a labor union.
Mere “underinclusiveness” is not fatal to the validity of the law. Legislation is not unconstitutional
merely because it is not all-embracing. It is not unconstitutional simply because it is incomplete.
A legislature does not risk losing the entire remedial scheme simply because it fails, thru
inadvertence, to cover every evil that might conceivably have been attacked. However, in EO1,
there is NO inadvertence. The previous administration was deliberately picked out as it was
underscored at least 3 times in the EO. EO1 does not even mention any particular act or report to
be focused on. The equal protection clause is violated by purposeful and intentional
discrimination.

-Section 17 of EO1
OSG argues that PTC does not only confine itself to the previous administration as Section 17
allows the president to expand the mandate of PTC to include prior administrations if there is such
need to expand in his judgment.

But although Section 17 allows such, it does not guarantee that they would be covered in the
future. Such would still depend on the whim and caprice of the president. If he decides not to
include them, the section would be meaningless.

EO1 was declared unconstitutional. Perhaps a revision of the executive issuance to include the
earlier past administrations would allow it to pass the test of reasonableness.

59.1 Sereno Dissenting Opinion


The decision upsets the long line of precedents on equal protection and displays self-contradiction.
It is a court of unelected people proposing to supplant the will of the more than 15 million voters
who voted for Pres. Aquino by imposing unreasonable restrictions on the impossible, unknowable
standards for presidential action. It prevents the fulfillment of the political contract between the
president and the Filipino people. It not only violates separation of powers, it usurps the sovereign
power of the people to determine the priorities of government.

The decision first affirms the president’s power to conduct investigation. But then it impermissibly
restricts this power by denying him the right to choose the priority, in this case the Arroyo
administration, in his graft-busting campaign. The decision is telling the president to proceed with
his anti-corruption program on the condition that, when making a fact-finding commission, he
must include all past administrations without exception except those dead presidents or whose
crimes have prescribed.

The president already made his prioritization, and such is not arbitrary. The OSG already explained
why investigation of the Arroyo administration is priority. The Court seems to have set an
inordinately high standard for reasonableness that is impossible to satisfy, primarily because it is
unknowable and unpredictable.

The majority suggests that EO1 be “tweaked” to become acceptable. But a reading of the decision
already indicates that the moment the prioritization hints at focusing on the Arroyo
administration, the majority is ready to once again strike it down. The test that should have been
applied by the Court is whether EO1 is unconstitutional for prioritizing fact-finding on the reported
corruption of the Arroyo administration without foreclosing, but not guaranteeing, future
investigation into other administrations.

-Unwarranted creation of “class of all political administrations.”


EO1 did not create two classes- that of Pres. Arroyo and of other past administrations. It just
prioritized fact-finding on the administration of Arroyo while saying that the president could later
expand EO1’s coverage. PRIORITIZATION per se is NOT CLASSIFICATION.

The majority lumps into a single class all past administrations spanning 111 years for testing valid
legislation. This is unwarranted. There is inherent illogic in the premise of the decision that
administrations from the time of Aguinaldo to Arroyo belong to one class.

Assuming arguendo that all political administrations fall under one class, the test of reasonableness
has been met by EO1.

The decision concedes that classification per se is not forbidden. In legislation or regulation, a
step-by-step process resulting in a classification of those that are immediately included therein
versus those that have yet to be included in the future is constitutional. Lastly, the decision also
concedes that underinclusiveness is not unconstitutional especially when the purpose can be
attained thru inclusive future legislation. It also observes that valid underinclusiveness can result
from either inadvertence or delierateness. Regardless of these discussions supporting EO1, the
decision strikes it down.

The decision creates an argument for the invalidity of EO1 by quoting from general principles of
case law and ignoring specific applications of constitutional tests for valid classification. The
reasonableness of the classification in EO1 was amply demonstrated by the OSG, but the majority
simply responds dismissively that such are superficial, specious, and irrelevant, without clearly
explaining why they are so.

-OSG reasons for valid classification of the Arroyo Administration.


1. Past administrations have already been investigated, hence, there is constitutional basis not to
include them in the immediate focus of the investigation. Luna v. Sarmiento supports this. It
classified within the class of taxpayers obligated to pay taxes from 1941 to 1945 into those who
did not pay and those who did. Thus, within the class of political administrations, a good basis
for distinction is an administration that has not yet been investigated and one already
investigated. There is valid basis to distinguish the Marcos, Ramos, and Estrada administrations,
which have already been subject of fact-finding commissions, and the Arroyo administration.

2. It would be unduly oppressive to require PTC to investigate all administrations. Case law holds
that administrative constraints are a valid basis for classification. The president believed that the
most effective way of jump-starting his administration’s fight against corruption was to focus on
the latest past administration. There is no violation based on PTC’s investigation being limited to
what can be feasibly investigated, a classification based on the Executive’s practical
administrative constraints.
3. Unlike with administrations long gone, the current administration will most likely bear the
immediate consequence of the policies of the previous administration. Evidence of possible
criminal activity are more easily established in the immediately preceding regime.

To ignore the limitation of time by assuming that a public official has all the time in the world to
accomplish an investigative goal, and to force the subject of his scrutiny to comprise all past
administrations is the height of legal unreasonableness.

-The majority decision omitted the analytical process required in equal protection claims.
1st test: Is there substantial distinction? The OSG already gave several reasons. The distinction
does not lie in any claim that corruption is the sole hallmark of the Arroyo administration. It lies
in reason- administrative constraints, availability of evidence, immediate past acts, non-
prescription of causes of actions, all of which are not whimsical or superficial.

2nd test: The classification is germane to the purpose of the law- to get a HEADSTART on the
campaign against corruption. Investigation into the root of corruption must start somewhere,
and the best place to start is the immediate past administration.

3rd test: Not relevant, not prospective but a fact-finding into past acts.

4th test: This asks whether the law applies equally to all members of the segregated class. The
question should have been whether within the SUBCLASS of third level public officials of the
Arroyo administration- the subject f EO1- there is unequal treatment, not whether there is
equality of treatment between all political administrations. The answer is no. The majority applied
this test backwards by asking whether there is equality among all political administrations even
before it could answer whether the classification between the Arroyo administration and other past
administrations was reasonable.

-Selective prosecution.
The government has broad discretion over decisions to initiate criminal prosecutions and whom to
prosecute. But such discretion is not without limit. To violate equal protection, selective
enforcement must be deliberately based on unjustifiable or arbitrary classification. The mere
failure to prosecute all offenders is NO ground for denial of equal protection. To support a
claim of selective prosecution, a defendant must show that the prosecution 1) had a discriminatory
effect and 2) was motivated by a discriminatory purpose. He must prove that his selection was
based on impermissible considerations such as race, religion, or the desire to prevent the
exercise of constitutional rights.

The prosecution of one guilty person, while others equally guilty are not prosecuted, however,
is not, by itself, a denial of equal protection. In this case, the fact that other administrations are
not subject of PTC’s investigation is not a case of selective prosecution that violates equal
protection.

-Presumption of good faith.


The presumption of good faith must be observed especially when the action is pursuant to a
constitutionally enshrined state policy for taking measures against corruption. If a law neither
burdens a fundamental right nor targets a suspect class, the Court must uphold the classification.

-EO1 activities
EO1 activities are at most initiatory investigations. The PTC is tasked to collect, receive, review,
and evaluate evidence related to corruption, tasks that constitute nothing more than a general
inquiry into such reported cases. To strike down the PTC’s mandate simply because other
administrations are not immediately included is tantamount to saying that a police investigation of
a recent murder case violates equal protection because there are other prior yet equally heinous
murders that remain uninvestigated and unsolved by the police.

The challenge is also being raised at the inception stage for determination of possible criminal
liability, where threat to liberty is most absent. Even if PTC recommends that corruption and
acts of graft were committed by the Arroyo administration, there is still a long way to go before
the recommendation would ripen to criminal prosecution. The Ombudsman must accept the
referral and conduct its own preliminary investigation etc.

-Public’s right to know.


Justice Brion’s concurring opinion speaks of the fear that PTC would be a mind-conditioning
commission such that if the Ombudsman, Sandiganbayan, or the SC were to reject its findings, the
would incur the people’s ire. He objects to PTC’s appropriation of the word “truth” and assumes
that all conclusions contrary to PTC’s would be labeled as “untruth.”

The public does not need sheltering from the potentially prejudicial effects of truth-telling. It does
not follow that the judiciary and Ombudsman must inevitably accede to public clamor. To declare
that the Filipino public is undeserving of the truth on grounds of its supposed lack of capacity to
deal with the truth and its alleged susceptibility to “priming” effects of the PTC’s findings, while
ignoring the public’s need to know the truth and to seek redress for wrongs, is to deny the
public the means to move towards social justice. The public’s right to know and the public policy
of full public disclosure support the fact-finding mandate of PTC.

-The “Least Dangerous” branch


If the Court is to avoid illegitimacy in its actions, it must ensure that its discharge of the duty to
prevent abuse of executive power does not translate to striking down even a legitimate exercise
thereof, especially when the exercise is in keeping with the will of the people. The 1987
Constitution expanded the parameters of judicial power, but that did not create an imperial
judiciary, composed of the unelected, whose sole constituency is the blindfolded lady without the
right to vote, is counter-majoritarian, hence inherently inimical to the central ideal of democracy.
In a government whose cornerstone rests on separation of powers, we cannot be the repository
of all remedies.

60. Ormoc Sugar Company v. The Treasurer of Ormoc City, GR L-23794, February 17,
1968, Bengzon, J.P., J. (Equal Protection)
FACTS:
The municipal board of Ormoc City passed Ordinance 4 s.1964, imposing on all productions of
centrifugal sugar milled at the Ormoc Sugar Company, Inc. in Ormoc City a municipal tax of
1% per export sale to the US and other foreign countries. Payments therefor were made under
protest. Petitioner then filed with the CFI a complaint against Ormoc City etc. alleging that the
ordinance violates equal protection and the rule on uniformity of taxation.

The CFI upheld the validity of the ordinance. Hence this appeal.

ISSUE:
Whether a City ordinance by Ormoc city, Ordinance 4 s.1964, imposing a municipal tax of 1% per
export sale to foreign countries only on Ormoc Sugar Company, Inc. violates equal protection.

HELD:
The equal protection clause applies only to persons or things identically situated and does not bar
a reasonable classification, and a classification is reasonable where (1) it is based on substantial
distinctions which make real differences; (2) these are germane to the purpose of the law; (3) the
classification applies not only to present conditions but also to future conditions which are
substantially identical to those of the present; (4) the classification applies only to those who
belong to the same class.

The ordinance does not meet these requisites for it taxes only centrifugal sugar produced and
exported by Ormoc Sugar Company, Inc. and none other. At the time of the ordinance’s
enactment, it is true that petitioner was the only sugar central in Ormoc. Still, for classification to
be reasonable, it should apply to future conditions as well. The taxing ordinance should not be
singular and exclusive as to exclude any subsequently established sugar central of the same
class as petitioner. As it is now, even if a later similar company is set up, it cannot be subject to
the tax because the ordinance expressly points to petitioner only.

61. Armando Yrasuegui v. Philippine Airlines, GR 168081, October 17, 2008, Reyes, R.T., J.
(Equal Protection; equal protection may be invoked only against governmental acts)
FACTS:
Petitioner Yraseugui was an international flight steward of PAL. He stands 5’8” tall with a large
body frame. The proper weight for a man of his height and body is from 147 to 166 pounds, the
ideal being 166 lb as mandated by the Cabin and Crew Administration Manual of PAL.

PAL advised Yraseugui in 1984 to go on extended vacation leave to address his weight concerns,
but he failed to meet the company’s standards, prompting another leave. He then met the required
weight, was allowed to work, then when his weight problem recurred, he went on leave again from
October 1988 to February 1989. On April 26, 1989, he weighed 209 pounds. He was removed
from flight duty in line with company policy and requested to trim down to his ideal weight.

On February 25, 1989, he was found to weigh 215lb, gaining instead of losing weight. He was
directed to report for weight checks, but he failed to comply. He was warned that a repeated refusal
to report for weight check would be dealt with accordingly. He was given another set of weight
check dates, but he failed to comply again.
On November 1992, he was served with a notice of administrative charge for violation of company
standards on weight requirements. He answered that his violation had already been condoned since
“no action has been taken by the company” regarding his case “since 1988.” He also claims that
PAL discriminated against him because the company was not fair in treating cabin crew members
who are similarly situated.

On June 15, 1993, his services were terminated. Yrasegui filed a complaint for illegal dismissal.
The LA ruled that he was illegally dismissed. The NLRC affirmed the LA decision. The CA set
aside the NLRC decision. Hence this appeal by Yrasegui.

ISSUE:
Whether Yrasegui, an international flight steward, may be validly dismissed for failing to meet the
required standard of weight of Philippine Airlines, Inc. for stewards of his height despite
repeatedly being required by PAL to reduce his weight to the ideal weight and for refusing to report
to weight checks repeatedly.

HELD: YES.
1. The obesity of Yrasegui is a ground for dismissal under Article 282 of the Labor Code. It is a
continuing qualification of an employee to keep the job.
2. Bona fide occupational qualification is a valid defense. The weight standards of PAL are
reasonable. A common carrier, from the nature of its business and for reasons of public policy, is
bound to observe extraordinary diligence for the safety of its passengers. It is bound to carry its
passengers safely as far as human care and foresight can provide, using the utmost diligence of
very cautious persons with due regard for all circumstances.

The primary objective of PAL in imposing weight standards is flight safety. The task of a cabin
crew is not limited to serving meals or attending to passengers. The most important activity is to
care for the safety of passengers and the evacuation of the aircraft when an emergency occurs.
Passenger safety is the core of the job of a cabin attendant. Airlines need cabin attendants who
have the necessary strength to open emergency doors and the stamina to withstand grueling flight
schedules. Being overweight impedes mobility.

3. Yrasegui failed to substantiate his claim of discrimination. The element of discrimination only
camt to play in order for Yraegui to escape the consequence of dismissal that being overweight
entailed. He cannot establish discrimination by simply naming supposed cabin attendants who are
allegedly similarly situated with him. Substantial proof must be shown as to how and why they
are similarly situated and the differential treatment petitioner got from PAL despite the similarity
of his situation with other employees.

Other than naming the other attendants, he failed to indicate their respective ideal weights, weights
over their ideal weights, the periods they were allowed to fly despite their being overweight, the
particular flights assigned to them, the discriminating treatment they got from PAL, and other
relevant date.

While findings of fact of administrative agencies are accorded respect, even finality, this only
applies if the findings are duly supported by substantial evidence. Findings of fact of
administrative agencies must be set aside when they fail the test of arbitrariness. The LA and
NLRC misappreciated evidence.

Yrasegui invokes equal protection. But in the absence of GOVERNMENTAL interference, the
liberties in the Constitution cannot be invoked. The Bill of Rights is NOT meant to be invoked
against acts of PRIVATE individuals. Private actions, no matter how egregious, cannot violate
the equal protection guarantee.

62. Central Bank Employees Association, Inc. v. BSP, GR 148208, December 15, 2004, Puno,
J. (Equal Protection; relative constitutionality- a statute valid at one time may become
unconstitutional at another because of changed conditions that make the practical operation
of such a statute arbitrary; GFIs are one distinct class, arbitrary discrimination as to their
employees would violate equal protection)
FACTS:
RA 7953 (New Central Bank Act) took effect on July 3, 1993, abolishing the old Central Bank of
the Philippines and created the Banko Sentral ng Pilipinas (BSP). On June 08, 2001, petitioner
filed a petition for prohibition against BSP and the Executive Secretary to restrain them from
implementing the last proviso in Section 15(c), article II of RA 7653 on the ground that it is
unconstitutional. This provides:

Section 15.Exercise of Authority. — In the exercise of its authority, the Monetary Board
shall: xxx xxx xxx
(c)establish a human resource management system which shall govern the selection, hiring,
appointment, transfer, promotion, or dismissal of all personnel. Such system shall aim to
establish professionalism and excellence at all levels of the Bangko Sentral in accordance
with sound principles of management.
A compensation structure, based on job evaluation studies and wage surveys and subject
to the Board's approval, shall be instituted as an integral component of the Bangko Sentral's
human resource development program: Provided, That the Monetary Board shall make its
own system conform as closely as possible with the principles provided for under Republic
Act No. 6758 [Salary Standardization Act]. Provided, however, That compensation and
wage structure of employees whose positions fall under salary grade 19 and below shall be
in accordance with the rates prescribed under Republic Act No. 6758.

Petitioner claims that the above last proviso makes an unconstitutional cut between two classes of
employees in the BSP: 1) BSP officers or exempted from the Salary Standardization Law (SSL)
(exempt class), and 2) the rank-and-file (SG19 and below) or those not exempt from SSL (non-
exempt class). It is claimed that the classification is arbitrary and violates equal protection.

ISSUE:
Whether RA 7953, in subjecting BSP employees whose positions fall under salary grade 19 and
below only to the provisions of RA 6758 or the Salary Standardization Act, violate the equal
protection clause.

HELD:
1. Under present standards of equal protection- valid.
Equal protection does not prevent the legislature from establishing classes of individuals or objects
upon which different rules shall operate as long as such is not unreasonable. The exemption of
officers SG20 and above from SSL was intended to address the BSP’s lack of competitiveness
in attracting competent officers. It was not intended to discriminate against the rank-and-
file. this distinction has a rational basis and is not arbitrary.

2. But the enactment of subsequent laws exempting all other rank-and-file employees of GFIs from
the SSL renders the challenged application a violation of equal protection.
-Relative constitutionality.
The constitutionality of a statute cannot always be determined by mere comparison of its
provisions with the Constitution since it may be valid as applied to one set of facts and invalid as
applied to another. A statute valid at one time may become void at another time because of altered
circumstances. Thus, even if affirmed by former adjudication, a statute is open to inquiry in light
of changed conditions.

Courts are not confined to the language of the statute under challenge in determining whether that
statute has any discriminatory effect. A statute nondiscriminatory on its face may be
discriminatory in its operation. There is no difference between a law which denies equal
protection and a law which permits such denial.

-Enactment of RA nos. 7907, 8282, 8289, 8291, 8523, 8763, and 9302 = consequential
unconstitutionality of challenged proviso.
Petitioner claims that the proviso violates equal protection since after it was enacted, the charters
of the GSIS, Land Bank of the Philippines (LBP), Development Bank of the Philippines (DBP),
and SSS were amended, but the personnel of the latter GFIs were all exempted from SSL. Thus,
within the rank-and-file personnel of Governmental Financial Institutions (GFI), the BSP rank-
and-file are discriminated.

After the BSP charter was enacted in 1993, Congress also amended the charters of the following
GFIs:
1.R.A. No. 7907 (1995) for LBP; 2.R.A. No. 8282 (1997) for SSS; 3.R.A. No. 8289 (1997) for
Small Business Guarantee and Finance Corporation, (SBGFC); 4.R.A. No. 8291 (1997) for GSIS;
5.R.A. No. 8523 (1998) for DBP; 6.R.A. No. 8763 (2000) for Home Guaranty Corporation (HGC);
and 7.R.A. No. 9302 (2004) for Philippine Deposit Insurance Corporation (PDIC).

The subsequent charters of the seven other GFIs share this common proviso: a blanket
exemption of all their employees from the coverage of the SSL, expressly or impliedly. Thus,
11 years after the amendment of the BSP charter, the rank-and-file of seven other GFIs were
granted the exemption that was specifically denied to the rank-and-file of the BSP.

The previous view on constitutionality of RA 7653 was confined to its classification between
rank-and-file and BSP officers. This was found reasonable as there is substantial distinction
between the two classes. But the subsequent enactments constitute significant changes in
circumstance that alter the reasonability of the continued operation of the last proviso in Section
15(c) of Article II of RA 7653, exposing it to more serious scrutiny.
This time, the scrutiny relates to the constitutionality of the classification, albeit indirectly as a
consequence of the passage of the 8 other laws, between the rank-and-file of BSP and the seven
other GFIs. The classification must not only be reasonable but must apply equally to all
members of the class.

Is Congress' power to classify so unbridled as to sanction unequal and discriminatory treatment,


simply because the inequity manifested itself, not instantly through a single overt act, but gradually
and progressively, through seven separate acts of Congress?

The inequality of treatment cannot be justified on the mere assertion that each exemption rests on
a policy determination by the legislature. All legislative enactments necessarily rest on a policy
determination, even those unconstitutional ones. If this were enough to sustain a law’s validity, no
challenge would ever prosper. What is at issue in this 2nd level of scrutiny is not the declared policy
of each law per se, but the oppressive results of Congress’ inconsistent and unequal policy
towards the BSP rank-and-file and those of seven other GFIs. The equal protection clause
includes the prohibition against enacting laws that allow discrimination, directly or indirectly.

There exists no substantial distinctions exist to differentiate BSP rank-and-file from the other rank-
and-file of the seven GFIs as regards exemption from SSL coverage. Legal history shows that
GFIs have long been recognized as comprising one distinct class, separate from other
governmental entities.

It is argued that the need for the scope of exemption necessarily varies with the particular
circumstances of each institution, and the corresponding variance in the benefits received by the
employees is merely incidental. However, the BSP is the central monetary authority and the banker
of the government and all its political subdivisions. It has authority to supervise banks and regulate
finance companies etc. including the exempted GFIs. Hence the argument that the rank-and-file of
the seven GFIs were exempted because of the importance of their institution’s mandate cannot
stand.

It is misleading to say that the need and scope of exemption varies with the institution’s
circumstance. The legislative deliberations show that while each GFI has a mandate different from
that of another, the raison d’etre of the SSL-exemption was inextricably linked to and based on
factors COMMON to the eight GFIs: (1) the pivotal role they play in the economy; (2) the
necessity of hiring and retaining qualified and effective personnel to carry out the GFI's mandate;
and (3) the recognition that the compensation package of these GFIs is not competitive, and fall
substantially below industry standards.

Considering also that 1) the BSP was the first GFI granted SSL exemption and 2) the subsequent
exemptions of other GFIs did not distinguish between officers and rank-and-file, the classification
made between the BSP rank-and-file and those of other seven GFIs was inadvertent and not
intended. It was not based on any substantial distinction vis-à-vis the particular circumstances of
each GFI.

There are no characteristics peculiar only to the seven GFIs or their rank-and-file as to justify the
exemption which BSP rank-and-file were denied. The distinction is superficial and arbitrary.
The exemption from the SSL is a "privilege" fully within the legislative prerogative to give or
deny. However, its subsequent grant to the rank-and-file of the seven other GFIs and continued
denial to the BSP rank-and-file employees breached the latter's right to equal protection. While the
granting of a privilege per se is a matter of policy exclusively within the domain and prerogative
of Congress, the validity or legality of the exercise of this prerogative is subject to judicial review.
Equal protection does not demand absolute equality, but it requires that all persons shall be
treated alike, under like circumstances and conditions both as to privileges conferred and
liabilities enforced.

3. There are concerns as to a ruling voiding the challenged provision claiming that the remedy is
not with this Court but with Congress. Under most circumstances, the Court will exercise judicial
restraint in deciding questions of constitutionality. Judicial scrutiny would be based on the
“rational basis” test and legislative discretion would be given deferential treatment. But if the
challenge to the statute is premised on the denial of a fundamental right, or the perpetuation of
prejudice against persons favored by the Constitution with special protection, judicial scrutiny
ought to be stricter.

SEARCHES AND SEIZURES


63. Harry Stonehill v. Hon. Diokno, GR L-19550, June 19, 1967, Concepcion, C.J. (Searches
and Seizures; search warrants shall issue only upon probable cause determined by the judge
personally; the warrant must particularly describe the things to be seized; else, searches in
violation of person’s right against unreasonable searches and seizures are inadmissible in
evidence against him- exclusionary rule)
FACTS:
42 search warrants were issued against petitioners and corporations of which they are officers to
search the persons named therein and the premises of their offices for "Books of accounts, financial
records, vouchers, correspondence, receipts, ledgers, journals, portfolios, credit journals,
typewriters, and other documents and/or papers showing all business transactions including
disbursements receipts, balance sheets and profit and loss statements and Bobbins (cigarette
wrappers)." as the subject matter of the offense and its proceeds or fruits or used or intended to be
used as a means of committing the offense which are described as “violation of Central Bank Laws,
Tariff and Customs Laws, Internal Revenue (Code) and the RPC.”

Petitioners filed with the SC a petition for certiorari, prohibition etc. alleging that the search
warrants are void as 1) they do not describe with particularity the things to be seized, 2) cash
money, not mentioned in the warrants, were actually seized, 3) the warrants were issued to fish
evidence, 4) the searches and seizures were illegally made, and 5) the documents, papers, cash
seized were not delivered to the courts that issued the warrants. They pray that an injunction be
issued restraining respondent-prosecutors from using the seized items in the deportation cases
against them and to have the search warrants declared void.

The things seized may be grouped into 2: 1) those found and seized in the offices of the
corporations and 2) those found seized in the residences of petitioners.
ISSUE:
Whether a search warrant for for "Books of accounts, financial records, vouchers, correspondence,
receipts, ledgers, journals, portfolios, credit journals, typewriters, and other documents and/or
papers showing all business transactions including disbursements receipts, balance sheets and
profit and loss statements and Bobbins (cigarette wrappers)." as the subject matter of the offense,
which is described as “violation of Central Bank Laws, Tariff and Customs Laws, Internal
Revenue and the RPC.” is valid and constitutional.

HELD: NO.
1. First group of things seized in the offices of corporations.
Petitioners have no cause of action to assail the legality of the warrants and seizures for the reason
that corporations have their respective personalities separate and distinct from the personality of
petitioners regardless of the amount of shares or interest they have in the corporations and whatever
office they hold therein. The legality of a seizure can be contested only by the party whose rights
have been impaired thereby and that the objection to an unlawful search and seizure is purely
personal and cannot be availed by third parties. Petitioners cannot object to the use in evidence
against them of those things seized from the offices of the corporations.

2. Second group of things seized in the residence of petitioners.


Petitioners claim that the search warrants are in the nature of general warrants and thus void.

Under the constitutional mandate (Art.III, S2), two points are stressed:
1) that no warrant shall issue except upon PROBABLE CAUSE, to be determined by the judge
in the manner set forth therein, and 2) that the warrant shall PARTICULARLY describe the things
to be seized.

None of these has been complied with. The same were issued upon applications stating that the
persons therein had committed a “violation of Central Bank Laws, xxx.” In other words, no
specific offense had been alleged in the applications. The averments were abstract. As a
consequence, it was impossible for the judges to have found probable cause, for the same
presupposed the introduction of competent proof that the party against whom it is sought has
performed particular acts, specific omissions, violating a given provision of our criminal laws.

To uphold the validity of the warrants would uphold the evil sought to be remedied by the
constitutional provision above- to outlaw the so-called general warrants. The seriousness of the
irregularities of the warrants made this Court amend its rules providing that “a search warrant shall
not issue upon probable cause in connection with one specific offense. We added a paragraph “that
no search warrant shall issue for more than one offense.”

The warrants also authorized the search of records pertaining to all business transactions of
petitioners regardless of whether they were legal or illegal (“Books of accounts, xxx”). They
sanctioned the seizure of all records of petitioners and the corporations whatever their nature,
contravening the command that the things to be seized be PARTICULARLY described.

3. Admissibility.
Respondents-prosecutors claim that even if the searches and seizures were unconstitutional, the
documents, papers, and things seized are admissible in evidence against petitioners, relying on
Moncado v. People’s Court.

However, We rule that the Moncado case be abandoned. This was based on the theory that the
criminal must not be allowed to go free merely because the “constable has blundered” and that the
constitutional prohibition against unreasonable searches is protected by other means such as the
common-law action for damages, criminal punishment against those who procured the warrant,
etc.

Most common law jurisdictions have given up this approach and adopted the exclusionary rule,
realizing that this is the only practical means of enforcing the constitutional injunction against
unreasonable searches and seizures. “Only in case the prosecution which itself controls the seizing
officials knows that it cannot profit by their wrong, will that wrong be repressed. – Judge
Learned Hand.”

If the applicant for a search warrant has competent evidence to establish probable cause of the
commission of a crime by the party against whom the warrant is intended, then there is no reason
why the applicant should not comply with the requirements of the fundamental law. If he has no
such evidence, then it is not possible for the judge to find probable cause, hence no justification
for issuance of the warrant. The only possible explanation for its issuance is the necessity of fishing
evidence of the commission of a crime, which is indicative of absence of evidence to establish
probable cause.

The theory that criminal prosecution is sufficient protection against those who secure an illegal
search warrant overlooks the fact that violations thereof are, in general, committed by agents of
the party in power. Even then, if they are convicted, they may be pardoned thru the pardoning
power of the party for whose benefit the illegality had been committed.

64. Esteban Morano v. Hon. Vivo, GR L-22196, June 30, 1967, Sanchez, J. (Searches and
Seizures; deportation proceeding and criminal proceeding are separate and distinct;
guarantee against unreasonable search and seizure that probable cause be determined by a
judge does not apply to deportation proceedings)
FACTS:
Chan Sau Wah, a Chinese citizen born in Fukien, China, arrived in the PH on November 23, 1961.
With her was her minor son Fu Yan Fun. They were permitted entry in PH under a temporary
visitor’s visa for 2 months.

On January 24, 1962, Chan married Esteban Morano, a Filipino. Born of this union was Esteban
Morano Jr. Chan and Yan Fun obtained several extensions. The last extension expired on
September 10, 1962.

They were warned by the Commissioner of Immigration that if they do not leave the country on
or before September 10, 1962, he will issue a warrant for their arrest. But on September 10, Chan
and Esteban and Yan Fun petitioned the CFI for mandamus to compel the commissioner to cancel
their alien certificates of registration and to prohibit the commissioner from issuing warrants of
arrest pending resolution of this case. The CFI declared Chan a citizen of PH upon her marriage
with Esteban, but not Yan Fun.

Hence this appeal.

ISSUE:
Whether the Commissioner of Immigration, as authorized by the Immigration Act of 1940. may
validly issue a warrant of arrest againt Chan Sau Wah and Fu Yan Fun for their deportation after
the expiration of their visa without violating the constitution.

HELD: YES.
Marriage of an alien woman to a Filipino does not ipso facto make her Filipino. She must show
that she has all the qualifications and none of the disqualifications of the Naturalization Law. Chan
did not become Filipino.

Also in issue is the constitutionality of Section 37(a) of the Immigration Act of 1940:
"Sec. 37. (a) The following aliens shall be arrested upon the warrant of the Commissioner
of Immigration or of any other officer designated by him for the purpose and deported
upon the warrant of the Commission of Immigration after a determination by the Board of
Commissioners of the existence of the ground for deportation as charged against the alien:
xxx xxx xxx
(7) Any alien who remains in the Philippines in violation of any limitation or condition
under which he was admitted as a nonimmigrant."

Petitioners argue that this violates the constitution which limits to judges (“determined by the
judge…”) the authority to issue warrants of arrest. Thus, the legislative delegation of such power
to the commissioner is unconstitutional.

Section 1(3) Art.III (*now section 2, Art.III) does not require judicial intervention in the
execution of a final order of deportation issued in accordance with law. It contemplates an order
of arrest in the exercise of judicial power as a step preliminary or incidental to prosecution or
proceedings for a given offense or administrative action, not as a measure indispensable to carry
out a valid decision by a competent official, like a legal order of deportation, issued by the
Commisisoner of Immigration, in pursuance of valid legislation.

Congress has power to order the deportation of aliens whose presence in the country it deems
hurtful. The determination of propriety of deportation is not a prosecution for, or conviction of,
crime, nor is deportation a punishment. The proceeding is in effect simply a refusal by the
government to harbor persons whom it does not want. A criminal prosecution and a proceeding
for deportation are separate and independent.

Thus, the constitutional guarantee requiring the issue of probable cause be determined by a judge
does NOT extend to deportation proceedings.

The power to deport aliens is an attribute of sovereignty. Also, a temporary visitor is subject to
certain contractual stipulation as contained in the cash bond put up by him upon entry, among
others, that in case of breach the Commissioner may require the recommitment of the person in
whose favor the bond has been filed.

65. In the matter of petition for habeas corpus of Andrew Harvey v. Hon. Miriam Santiago,
GR 82544, June 28, 1988, Melencio-Herrera, J. (Searches and Seizure; guarantee against
search and seizure contemplates criminal prosecution; deportation proceedings are
administrative; arrest for deportation is a preliminary step toward possible deportation, thus
constitutional)
FACTS:
Petitioners Andrew Harvey and John Sherman are Americans residing in Laguna while Adriaan
Van Den Elshout is Dutch also residing in Laguna. They were apprehended from their residences
by agents of the Commission on Immigration and Deportation (CID) by virtue of Mission Orders
by respondent Commissioner Miriam Santiago of CID.

Seized during their apprehension were photo negatives and photos of the suspected child
prostitutes shown in salacious poses and boys and girls engaged in the sex at. There were also
posters and other literature advertising child prostitutes. They were with children when
apprehended.

Deportation proceedings were instituted against petitioners for being undesirable aliens. Warrants
of arrest were issued against petitioners by respondent for violation of the Immigration Act and
Revised Administrative Code.

They filed this petition for a writ of habeas corpus, alleging that there is nothing in the PH
Immigration Act nor Revised Administrative Code which legally clothes the Commissioner with
authority to arrest and detain petitioners pending determination of probable cause leading to an
administrative investigation, that respondents violated Section 2 of Article III of the Constitution
prohibiting unreasonable searches and seizures since the CID agents were not clothed with valid
warrants of arrest, search and seizure as required by the provision, and that mere confidential
information made to CID agents and their suspicion of petitioners’ activities that they are
pedophiles are not valid legal grounds for their arrest and detention.

ISSUE:
Whether a Commissioner of the Commission on Immigration and Deportation may order the arrest
of suspected alien pedophiles on mere suspicion of their activities without violating the
constitutional guarantee against unreasonable searches and seizures.

HELD: YES.
One of the constitutional requirements of a valid search warrant or warrant of arrest is that it must
be based upon probable cause. Probable cause has been defined as referring to “such facts and
circumstances antecedent to the issuance of the warrant that in themselves are sufficient to
induce a cautious man to rely on them and act in pursuance thereof.” The 1985 Rules on
Criminal Procedure also provide that an arrest without warrant may be effected by a peace officer
or even a private person 1) when a person has committed, actually committing, or is attempting to
commit an offense in his presence , and 2) when an offense has, in fact, been committed and he
has personal knowledge of facts indicating that the person to be arrested has committed it.
The arrest of petitioners was based on probable cause after close surveillance for three(3)
months during which their activities were monitored. This justified the arrest and seizure of the
photo negatives, photographs, and posters without warrant. Those articles were seized as an
incident to a lawful arrest and are admissible in evidence.

Even if their arrest was not valid at inception, formal deportation charges against them were filed.
The restraint against their persons became legal. A writ of habeas corpus will not be granted when
the confinement has become legal although illegal at the beginning.

The contraints both in the 1935 and 1987 Constitutions contemplate prosecutions essentially
CRIMINAL in nature. Deportation proceedings are administrative in character.

Respondent Commissioner’s warrant of arrest was issued specifically for violations of the
Immigration Act and Revised Administrative Code. Before that, deportation proceedings had been
commenced against them as undesirable aliens and the arrest was a step preliminary to their
possible deportation. Section 37 of the Immigration Law empowering the Commissioner of
Immigration to issue warrants of arrest of overstaying aliens is constitutional. The arrest is a step
preliminary to the deportation of aliens who had violated the condition of their stay in this country.

What is essential is that there should be a specific charge against the alien intended to be arrested
and deported, that a fair hearing be conducted with the assistance of counsel, if desired, and that
the charge be substantiated by competent evidence.

66. Mayor Abdula v. Hon. Guiani, GR 118821, February 18, 2000, Gonzaga-Reyes, J.
(Searches and Seizure; judge should determine probable cause personally- personally
examine fiscal’s supporting documents or require submission of supporting affidavits of
witnesses; determination of probable cause by a judge has a different objective than such
determination by the fiscal- thus, judge must decide independently)
FACTS:
A complaint for murder was filed against petitioners and 6 others in connection with the death of
a certain Abdul Dimalen, former COMELEC Registrar of Kabuntalan, Maguindanao. Prosecutor
Panda in a resolution dismissed the charges of murder against petitioners finding no prima facie
case. But Panda recommended filing an information for murder against a certain Kasan Mama. An
information for murder was filed against Mama before respondent Judge Guiani.

Guiani ordered that the case be returned to the provincial prosecutor for further investigation.
Judge Guiani noted that although there were 8 respondents, the information charged only one of
the eight respondents without the needed resolution required under Section 4, Rule 112 of the RoC
to show how the prosecutor arrived at such a conclusion.

Prosecutor Dimaraw was assigned to further investigate. Two new affidavits of witnesses were
submitted to support the charge of murder against petitioners and others. After evaluation of the
evidence, Dimaraw found a prima facie case for murder against petitioners. He recommended
filing of charges against petitioners.
An information for murder was filed against petitioners with the RTC before Judge Guiani. The
following day, judge Guiani issued a warrant of arrest against petitioners. Petitioners filed a motion
for setting aside the warrant, arguing that the information was prematurely filed and petitioners
intended to file a petition for review with the DOJ. A petition for review was filed with DOJ.
Despite this, judge Guiani did not act upon petitioners motion to set aside the warrant.

ISSUE:
Whether a warrant of arrest issued one day after the filing of the information in court despite the
petitioners subject of such warrant’s manifestation that they are yet to file a petition for review
with the DOJ of the information is a valid warrant.

HELD:
Petitioners argue that the warrant of arrest was issued without Guiani personally examining the
evidence or calling the complainant and his witnesses. The information was filed at 4pm of January
2, 1995 and the order of arrest was immediately issued on January 03, 1995. Respondent claims
that there was no reason to doubt the validity of the certification made by the assistant prosecutor
that a preliminary investigation was conducted and probable cause exists, thus he issued the
warrant of arrest.

"Section 2 [Article III]. The right of the people to be secure in their persons, houses, papers,
and effects against unreasonable searches and seizures of whatever nature and for any
purpose shall be inviolable, and no search warrant or warrant of arrest shall issue except
upon probable cause to be determined personally by the judge after examination under oath
or affirmation of the complainant and the witnesses he may produce and particularly
describing the place to be searched and the persons or things to be seized."

The Constitution requires the judge to determine probable cause “PERSONALLY.” The judge is
not required to personally examine the complainant and his witnesses. He shall: 1) personally
evaluate the report and supporting documents submitted by the fiscal regarding the existence of
probable cause and, on the basis thereof, issue a warrant of arrest, or 2) if on such basis he finds
no probable cause, he may disregard the fiscal’s report and require the submission of supporting
affidavits of witnesses to aid him in arriving at a conclusion as to the existence of probable cause.

The determination of probable cause by the prosecutor is for a different purpose than that which
is made by the judge. The prosecutor passes upon whether there is reasonable ground to believe
that the accused is guilty of the offense charged and should be held for trial. The judge determines
whether a warrant of arrest should be issued, whether there is a necessity for placing him
under immediate custody to not frustrate the ends of justice.

Since their objectives are different, the judge cannot rely solely on the report of the prosecutor
in finding probable cause to justify issuing a warrant of arrest. The judge must decide
independently. He must thus have supporting evidence OTHER THAN the prosecutor’s bare
report.

It is not required that the entire records of the case during preliminary investigation be submitted
to and examined by the judge. What is required is that the judge must have sufficient supporting
documents upon which to make his independent judgment. A judge cannot rely solely on the
prosecutor’s recommendation.

Judge Guiani relied solely on the certification of the fiscal. Although the fiscal enjoys the
presumption of regularity in the performance of his official duties, the Constitution commands the
judge to personally determine probable cause. A judge failes this duty if he merely relies on the
certification or report of the investigating officer. The inordinate haste that attended the issuance
of the warrant of arrest and Guiani’s own admission belie any pretense of fulfillment of his duty.

The warrant of arrest was declared null and void.

67. Sen. Jinggoy Estrada v. Office of the Ombudsman, GR 212140-41, January 21, 2015,
Carpio, J. (Searches and Seizures; Probable cause only requires “probability of guilt,” rests
on belief, and may even rest partially or entirely on hearsay if there is substantial basis to
credit it) *See case 52
HELD:
The quantum of evidence needed in Ang Tibay is greater than that in a preliminary investigation
to establish probable cause, or to establish the existence of a prima facie case that would warrant
the prosecution of a case. Ang Tibay refers to “substantial evidence” while the establishment of
probable cause needs “only more than bare suspicion or “less than evidence which would
justify conviction.”

There are four instances in the Revised Rules of Criminal Procedure where probable cause needs
to be established:
(1) In Sections 1 and 3 of Rule 112: By the investigating officer, to determine whether there is
sufficient ground to engender a well-founded belief that a crime has been committed and the
respondent is probably guilty thereof, and should be held for trial. A preliminary investigation is
required before the filing of a complaint or information for an offense where the penalty prescribed
by law is at least four years, two months and one day without regard to the fine;
(2) In Sections 6 and 9 of Rule 112: By the judge, to determine whether a warrant of arrest or a
commitment order, if the accused has already been arrested, shall be issued and that there is a
necessity of placing the respondent under immediate custody in order not to frustrate the ends of
justice;
(3) In Section 5 (b) of Rule 113: By a peace officer or a private person making a warrantless arrest
when an offense has just been committed, and he has probable cause to believe based on personal
knowledge of facts or circumstances that the person to be arrested has committed it; and (4) In
Section 4 of Rule 126: By the judge, to determine whether a search warrant shall be issued, and
only upon probable cause in connection with one speci-c offense to be determined personally by
the judge after examination under oath or af-rmation of the complainant and the witnesses he may
produce, and particularly describing the place to be searched and the things to be seized which
may be anywhere in the Philippines.

In all these cases, the evidence necessary to establish probable cause is based only on the
likelihood or probability of guilt. The determination of probable cause needs only to rest on
evidence showing that, more likely than not, a crime has been committed and there is enough
reason to believe that it was committed by the accused. What is merely required is probability
of guilt.

The determination of probable cause does not depend on the validity or merits of a party’s
accusation or defense on the admissibility or veracity of testimonies presented. These matters are
better ventilated in trial. Probable cause has been defined as the existence of such facts and
circumstances as would excite the belief in a reasonable mind, acting on facts within the
knowledge of the prosecutor that the person charged was guilty of the crime for which he
was prosecuted. It is merely based on opinion and reasonable belief. A finding of probable cause
does not require an inquiry on whether there is sufficient evidence to procure a conviction, but that
it is BELIEVED that the act or omission complained of constitutes the offense charged.

The determination of probable cause can rest partially or even entirely on hearsay evidence as
long as the person making the hearsay statement is credible. Hearsay may be the basis for issuance
of the warrant as long as there is substantial basis for crediting the hearsay.

Hearsay is admissible in a preliminary investigation because such is merely preliminary and does
not adjudicate finally rights and obligations of the parties. But in administrative cases where such
are finally adjudicated, what is required is “substantial evidence” which cannot rest on hearsay.
Thus, Ang Tibay cannot apply to preliminary investigations as this will change the quantum of
evidence required in determining probable cause to substantial evidence of guilt.

68. Nicomedes Silva v. Presiding judge, RTC of Negros Oriental, GR 81756, October 21,
1991, Fernan, C.J. (Searches and Seizures; must comply with 1) constitutional and 2)
statutory requirements of determining probable cause personally and through searching
questions and answers)
FACTS:
Sgt. Ranulfo Villamor, chief of PC Narcom Detachment, filed an “application for search warrant”
with the RTC against petitioners Nicomedes and Marlon Silva. This was accompanied with a
“Deposition of Witness.” On the same day, the RTC judge issued search warrant No. 1, directing
police officers to search the room of Marlon in the residence of Nicomedes for violation of RA
6425, Dangerous Drugs Act of 1972. The search warrant was served.

Petitioners moved to quash Search Warrant No. 1 on the ground that it was issued solely based on
a mimeographed “Application for Search Warrant “and “Deposition of Witness,” which were
accomplished by merely filling in the blanks and 2) the judge failed to personally examine the
complainant and witnesses by searching questions and answers in violation of Section 3, Rule 126
of the RoC. The RTC denied the motion. Hence this petition for certiorari.

ISSUE:
Whether a search warrant issued based on an “Application for Search Warrant” and a “Deposition
of Witnesses” containing questions and answers answerable by merely filling in the blanks with
yes or no is valid.

HELD: NO.
Rule 126 of the RoC provide for the requisites for issuance of a search warrant:
"SECTION 3. Requisite for issuing search warrant. — A search warrant shall not issue but
upon probable cause in connection with one specific offense to be determined
personally by the judge after examination under oath or affirmation of the complainant
and the witnesses he may produce, and particularly describing the place to be searched and
the things to be seized.
"SECTION 4. Examination of complainant; record. — The judge must, before issuing the
warrant, personally examine in the form of searching questions and answers, in writing
and under oath the complainant and any witnesses he may produce on facts personally
known to them and attach to the record their sworn statements together with any affidavits
submitted."

The judge must, before issuing a search warrant, determine whether there is probable cause by
examining the complainant and witnesses through searching questions and answers.

"The 'probable cause' for a valid search warrant, has been defined 'as such facts and circumstances
which would lead a reasonably discreet and prudent man to believe that an offense has been
committed, and that objects sought in connection with the offense are in the place sought to be
searched'. This probable cause must be shown to be within the personal knowledge of the
complainant or the witnesses he may produce and not based on mere hearsay."

In this case, the RTC judge failed to comply with the requirement that he must examine the
applicant and his witnesses in the form of searching questions and answers to determine probable
cause. The joint “Deposition of Witness” contained suggestive questions answerable by merely
placing “yes” or “no” in the blanks.
Q Do you personally know M/Sgt. Ranulfo Villamor, Jr. the applicant for a search
warrant?" A Yes, sir. "Q Do you have personal knowledge that the said premises subject
of the offense stated above, and other proceeds of fruit of the offense, used or obtain (sic)
or intended to be used as means of committing an offense?" A Yes, sir. "Q Do you know
personally who is/are the person who has have the property in his/their possession and
control?" A Yes, sir. "Q How did you know all this (sic) things?" A Through discreet
surveillance."

The deposition not only contained leading questions but was also very broad. The questions were
not probing but merely routinary. It was already mimeographed and all the witnesses had to do
was fill in their answers on the blanks provided.

In issuing a search warrant, the judge must strictly comply with the constitutional and statutory
requirement that he must determine probable cause by personally examining the applicant and
witnesses in the form of searching questions and answers. Failure to comply constitutes abuse
of discretion.

The officers also abused their authority when, in implementing the search, seized the money of
Antonieta Silva, who is not even named as respondent. The warrant did not indicate seizure of
money but only of marijuana leaves, cigarettes, and joints.

Search Warrant No.1 was declared null and void.


69. People v. Normando Del Rosario, GR 109633, July 20, 1994, Melo, J. (Searches and
Seizures)
FACTS:
Del Rosario was charged with illegal possession of firearm and ammunitions in Criminal case 236-
91 and illegal sale of regulated drugs in 237-91. He pleaded not guilty. The RTC convicted him in
both cases.
Prosecution’s version
Upon application of SPO3 Untiveros, RTC Judge Guia issued a search warrant authorizing search
and seizure of an “undetermined quantity” of shabu and its parephernalias in Del Rosario’s house.
A raiding team was organized. It was agreed that PO1 Luna will buy shabu from Del Rosario and
after his return from Del Rosario’s house, the team will implement the warrant. During the alleged
buy-bust, Del Rosario handed over to Luna shabu. Luna paid him marked money then returned to
the police station and informed the raiding team that he had already bought shabu.

PO1 Luna with a companion went to Del Rosario’s house to implement the warrant. The search
was witnessed by the Brgy. Captain, Del Rosario, and the latter’s sister Norma. They found a black
canister containing shabu, an aluminum foil, plastic .22 caliber, three used ammunition and three
wallets, one containing the marked money. There were also napkins and a burner. The seized items
were photographed thereat. Receipts were issued for the seized items. The items were taped and
initialed, forwarded to the NBI Forensic Chemist, and found positive for shabu.

ISSUE:
Whether Del Rosario may be convicted of illegal possession of firearm and ammunition due to the
.22 caliber and used ammunition seized in his home by virtue of a search warrant authorizing the
search and seizure of “shabu and its paraphernalia.”

HELD: NO.
The prosecution failed to call to the witness stand PO1 Luna, the poseur-buyer. There is thus
absence of evidence to establish the purported sale of shabu. The testimonies of the rest of the buy-
bust operation are hearsay since the poseur-buyer was never presented at the trial. This casts
serious doubts that an illegal sale actually took place.

The version of the prosecution, that Del Rosario handed over to PO1 Luna shabu and he paid Del
Rosario the marked money, that Luna then returned to the police station to inform the raiding team
that he had already bought the shabu, and that the team then went to Del Rosario’s house to
implement the search warrant is highly incredible. The record is devoid of any reason why the
officers did not make any attempt to arrest Del Rosario at the time he allegedly sold the shabu.
This was the most opportune moment to arrest him. The prosecution’s version is contrary to human
experience. The usual procedure in buy-busts is for the officers to arrest the pusher at the very
moment he hands over the dangerous drug to the poseur-buyer.

We thus doubt that the shabu in the small canister was actually seized at the residence of Del
Rosario. By calling witnesses after the police had already entered the house, the policemen had
more than ample time to plant the shabu. The constitutional precept that, in all criminal
prosecutions, the accused shall be presumed innocent until the contrary is proved requires that to
convict an accused, the circumstances of the case must exclude all and every hypothesis
consistent with his innocence. The facts in this case do not rule out the hypothesis that Del
Rosario is innocent.

The accused cannot be convicted of illegal possession of firearm and ammunition. The search
warrant implemented by the raiding party authorized only the search and seizure of “the
described quantity of Methamphetamine Hydrochloride commonly known as shabu and its
paraphernalia.” The raiding team was authorized to seize only shabu and paraphernalia for its
use and no other. A search warrant is NOT a sweeping authority empowering a raiding party
to undertake a fishing expedition to seize all kinds of evidence or articles relating to a crime.
The constitution mandates that the search warrant must particularly describe the things to be
seized. Thus, the warrant was no authority for the police officers to seize the firearm which was
not mentioned nor described with particularity in the search warrant. It cannot be maintained
that the gun was seized in the course of an arrest for the accused’s arrest was far from regular and
legal. The firearm, being illegally seized, is not admissible in evidence.

Del Rosario was acquitted.

70. Ariel Vallejo v. CA, GR 156413, April 14, 2004, Callejo, SR., J. (Searches and Seizures;
no need for technical precision in describing things to be seized, only REASONABLE
particularity; this requirement makes general warrants impossible; search warrant must be
for one specific offense)
FACTS:
Vallejo is a lawyer in the register of deeds of Isabela. On February 16, 2000, NBI agent Javier
filed a sworn application for search warrant before the RTC on the Office of the Register of Deeds
of Isabela to seize “1) fake land titles, official receipts in the cashier’s office, primary entry book,
and other pertinent documents related therewith, 2) blank forms of land titles, 3) land transfer
transactions without payment of documentary stamps and capital gains tax, all of which are used
in falsification of land titles and being hidden in said office.”

On the same date, Judge Alban issued Search Warrant 2000-03 against Vallejo. It stated that “ It
appearing to the satisfaction of the undersigned after examining under oath NBI Head Agent
Franklin M. Javier and his witness that there are reasonable grounds to believe that Falsification
of Land Titles xxx has been committed or is about to be committed and that there are good and
sufficient reasons to believe that the Registry of Deeds, xxx, Isabela, has in its possession and
control” 1), 2), and 3) above.

Vallejo moved to quash the warrant, which the trial court denied. He filed a MR claiming that the
warrant was in the form of a general warrant. This was denied. The CA dismissed his appeal for
being the wrong remedy and subsequently his petition for certiorari for being filed out of time.
Hence this petition.

ISSUE:
Whether a search warrant ordering the seizure of: “1 . Undetermined number of Fake Land Titles,
Official Receipts in the Cashier's Office, Judicial Form No. 39 known as Primary Entry Book
under No. 496 and other pertinent documents related therewith; 2 . Blank Forms of Land Titles
kept inside the drawers of every table of employees of the Registry (sic) of Deeds; 3 .
Undetermined number of Land Transfer transactions without the corresponding payment of
Capital Gains Tax and payment of Documentary Stamps.” is a valid search warrant.

HELD: NO.
The issue in this case is no less than the legality of the issuance of a warrant of arrest. It behooved
the CA to look past of technicality and to resolve the case on its merits considering Vallejo was
invoking a constitutional right. In dismissing Vallejo’s appeal and motion to admit petition for
certiorari, the CA gravely abused its discretion.

Section 2 of Article III of the 1987 Constitution guarantees the right to be free from unreasonable
searches and seizures. Rule 126 of the Revised Rules of Criminal Procedure provides for the
requisites of issuing a search warrant. Failure to comply with the constitutional and statutory
requirements constitutes grave abuse of discretion.

The things to be seized must be described with particularity. Technical precision of description
is not required, only that there be REASONABLE particularity and certainty as to the identity
of the property to be searched for and seized. The requirement that search warrants particularly
describe the things to be seized makes general searches impossible and prevents the seizure of
one thing under a warrant describing another. The property to be searched for must be so
particularly described to preclude the possibility of seizing any other property.

The search warrant in this case failed to pass this test of particularity. The terms are too all-
embracing with the intent of subjecting all the records pertaining to all transactions of Vallejo’s
office to search and seizure. The executing officer’s function is to apply the description to its
subject matter, which function may involve limited discretion in identifying the property
described. A description of such generality, however, as to lodge in the executing officer virtually
unlimited discretion as to what property shall be seized is repugnant to the constitution.

The search warrant must be issued for one specific offense. The questioned warrant is a scatter-
shot warrant for having been issued for more than one offense- falsification of land titles under
the Articles 171 and 213 of the RPC and violation of RA 3019. A warrant must be issued upon
probable cause in connection with one specific offense. The application for the warrant also shows
that the applicant did not allege any specific act performed by Vallejo constituting the mentioned
offenses.

Thus, they search warrant was struck down and the NBI ordered to return to Vallejo all items it
seized from the subject premises.

71. People v. Roberto Salanguit, GR 133254, April 19, 2001, Mendoza, J. (Searches and
Seizures; warrant to search an object without probable cause does not invalidate the whole
warrant if other object has probable cause; description of place is sufficient if the officer with
the warrant can, with reasonable effort, ascertain and identify the place intended to be
searched; once search warrant has been executed, plain view doctrine no longer allows
admission of items found subsequently)
FACTS:
Insp. Aguilar applied for a warrant with the RTC to search the residence of Robert Salanguit. He
presented as witness SPO1 Badua, who testified that as poseur-buyer, he was able to purchase 2.12
grams of shabu from Salanguit in Salanguit’s room. The search warrant was granted by Judge
Español. A group of about 10 policemen searved the warrant.

They found 12 transparent plastic bags with a white crystalline substance, a paper clip box with
the same substance, and 2 bricks of dried leaves which appeared to be marijuana weighing 1255
grams. The police then took Salanguit to the police station with the seized items. The white
crystalline substance was found to be shabu and the dried leaves found to be marijuana upon
laboratory examination.

Salanguit was charged with possession of 11.14 grams of shabu and another charge of possession
of 1254 grams of marijuana. The RTC convicted him. Hence this appeal.

ISSUE:
Whether a search warrant authorizing seizure of “undetermined quantity of shabu and drug
paraphernalia” despite there being evidence of probable cause only as to the shabu and describing
the place to be searched as only “Binhagan St., San Jose, Quezon City” is valid.

HELD: YES.
Rule 126, Sec.4 of the Revised Rules on Criminal Procedure provides that a search warrant shall
not issue except upon probable cause in connection with one specific offense to be determined
personally by the judge after examination under oath or affirmation of the complainant and the
witnesses he may produce, and particularly describing the place to be searched and the things to
be seized which may be anywhere in the PH. Nothing can justify the issuance of the search warrant
unless all the legal requisites are fulfilled.

Salanguit assails the validity of the warrant: 1) no probable cause; 2) it was issued for more than
one specific offense; and 3) the place to be searched was not described with sufficient particularity.

1. Probable Cause - present


The warrant authorized seizure of “undetermined quantity of shabu and drug paraphernalia.”
It is contended that there is no evidence of the existence of the drugs. This has no merit.

SPO1 Bauda, the poseur-buyer, did not testify on anything about drug paraphernalia. But the fact
that there was no probable cause to support the application for the seizure of drug paraphernalia
does not make the warrant void. This would be material only if drug paraphernalia was in fact
seized by the police. None was taken. If at all, the warrant would be void inly insofar as it
authorized the seizure of drug paraphernalia, but valid as to the seizure of shabu as to which
evidence was presented showing probable cause as to its existence.

It would be drastic if a warrant issued on probable cause particularly describing some items to be
seized is to be invalidated in toto because the judge erred in authorizing a search for other items
not supported by evidence. Thus, the first part of the warrant, authorizing the search for an
undetermined quantity of shabu, is valid, even though the second part, with respect to drug
paraphernalia, is not.

2. Particularity of the Place - present


While the address stated is merely “Binhagan St., San Jose, Quezon City, the trial court took note
that the records contained several documents which identified the premises to be searched: 1) the
application for warrant stated that the premises was located between No. 7 and 11 at Binhagan St.;
2) the witness described the premises as a house without a number located at Binhagan St., and 3)
the pencil sketch of the location.

The rule is that a description of the place to be searched is sufficient if the officer with the warrant
can, with reasonable effort, ascertain and identify the place intended to be searched.

Thus, the warrant was properly issued, being founded on probable cause personally determined by
the judge under oath or affirmation of the deposing witness and particularly describing the place
to be searched and the things to be seized.

3. Plain view doctrine on the marijuana - invalid


While the warrant authorized seizure of shabu but not marijuana, the seizure of the latter is sought
to be justified on the ground that the drug was seized within “plain view” of the searching party.
Under this doctrine, unlawful objects within the “plain view” of an officer who has the right to
be in the position to have that view are subject to seizure and may be presented in evidence.
There must be (a) prior justification; (b) inadvertent discovery of the evidence; and (c)
immediate apparent illegality of the evidence before the police.

a. No prior justification
Because the location of the shabu was indicated in the warrant and known to the police, it is
reasonable to assume that the police found the packets of the shabu first. Once the valid portion
of the warrant has been executed, the “plain view” doctrine can no longer provide any basis
for admitting the other items SUBSEQUENTLY found. The doctrine supplements the prior
justification – whether it be a warrant for another object, hot pursuit, search incident to lawful
arrest, or other legitimate reason for being present – and permits the warrantless seizure. The
doctrine may not be used to extend a general exploratory search from one object to another until
something incriminating at last emerges.

The other possible justification for intrusion is search pursuant to Salanguit’s lawful arrest for
possession of shabu. But this is limited to the PERSON of the one arrested and the PREMISES
within his immediate control. The rationale is to prevent the person arrested from obtaining a
weapon or to reach for and destroy incriminatory evidence. The police failed to allege in this case
the time when the marijuana was found, whether before or contemporaneous with the shabu subject
of the warrant, or within the person or immediate area of Salanguit. Its recovery therefore
presumable after the shabu had been recovered was invalid.

b. No apparent illegality
The marijuana bricks were wrapped in newsprint. There was no apparent illegality to justify
their seizure. No presumption of regularity may be invoked by an officer in aid of the process
when he undertakes to justify an encroachment of rights secured by the Constitution. The
marijuana was found in newsprint, not in a transparent container, thus these could not be readily
discernible as marijuana. Thus, the marijuana is inadmissible, but the confiscation is upheld.

4. Unnecessary force - justified


Rule 126, Sec.7 of the Revised Rules on Criminal Procedure provides that if an officer is refused
admittance to the place of directed search after giving notice of his purpose and authority, he may
break open any outer or inner door or window of a house or any part therin to execute the warrant.

The claim of the police that they had to use some force to gain entry cannot be doubted. The
occupants of the house, especially Salanguit, refused to open the door despite the searching party
having knocked several times. The agents also saw suspicious movements of the people inside the
house. These circumstances justified the party’s forcible entry into the house, founded on the
apprehension that the execution of their mission would be frustrated unless they do so.

72. People v. Tiu Won Chua, GR 149878, July 1, 2003, Puno, J. (Searches and Seizures;
mistake in name of person to be searched does not invalidate search warrant)
FACTS:
Tiu Won Chua and Qui Yaling y Chua were charged with illegal possession of a regulated drug,
shabu.

Police, acting on an information that drug-related activities were going on at the HCL Building,
Masangkay St., Binondo Manila, surveyed the place. They conducted a test-buy operation. They
were able to buy P2000 worth of shabu from appellants. They did not immediately arrest the
suspects but applied for a warrant to search Unit 4-B of HCL Building. Their application to search
the unit supposedly owned by “Timothy Tiu” was granted by the RTC. They went to the place
armed with the warrant. They conducted the search with the building coordinator and his wife as
witnesses.

They searched the sala and 3 bedrooms of Unit 4-B. they found 1 big pack of 234.5 grams of shabu
and 16 small packs of 20.3673 grams of shabu, etc.

The defense denied that Timothy Tiu and Tiu Won Chua are one and the same person as stated in
the search warrant. He claimed that he did not live in the apartment subject of the warrant but
admitted that Qui Yaling is his mistress. Qui Yaling admitted being the occupant of the apartment.

The RTC convicted Tiu Won and Qui Yaling. Hence this appeal. It is contended that the search
warrant was issued in the name of Timothy Tiu and did not include Qui Yaling and because of this
defect, the search and arrest are illegal. Being fruits of an illegal search, the evidence cannot serve
as basis for their conviction.

ISSUE:
Whether a search warrant to search a certain “Timothy Tiu” when the real name of the subject of
the search is Tiu Won Chua may be executed and is valid.

HELD: YES.
There are only four requisites for a valid warrant: (1) it must be issued upon "probable cause";
(2) probable cause must be determined personally by the judge; (3) such judge must examine
under oath or affirmation the complainant and the witnesses he may produce; and (4) the warrant
must particularly describe the place to be searched and the persons or things to be seized. A
mistake in the name of the person to be searched DOES NOT invalidate the warrant especially
in this case since the authorities had personal knowledge of the drug-related activities of the
accused. a “John Doe” warrant satisfies the requirements as long as it contains a description
personae as will enable the officer to identify the accused. A mistake in the identification of the
owner of the place does not invalidate the warrant provided the place to be searched is properly
described.

The place to be searched was described properly, thus the warrant cannot be invalidated even if
the name is incorrect. They can be presumed to have personal knowledge of the identity of the
persons and place to be searched since they conducted a surveillance and test-buy operation before
obtaining the warrant.

But the search conducted on the car was illegal since it was not part of the description of the
place to be searched mentioned in the warrant. It is mandatory that the search be directed at the
place particularly described in the warrant. This cannot be a search incidental to a lawful arrest
since such must be limited to that point within the reach of the person arrested. The car was parked
a few meters away from the building and appellants were arrested inside the building.

73. Ruben del Castillo v. People, GR 185128, January 30, 2012, Peralta, J. (Searches and
Seizures)
FACTS:
Pursuant to confidential information that Castillo was selling shabu, police, after conducting
surveillance and test-buy operation at Castillo’s house, secured a search warrant from the RTC and
served it.

Upon arrival, somebody shouted “raid” which prompted them immediately to disembark from the
jeep and go directly to Castillo’s house and cordoned it. Castillo’s house is a two-storey house and
he was staying in the 2nd floor. They saw Castillo run toward a nipa hut in front of his house. SPO3
Masnayon chased him but to no avail as they were unfamiliar with the entrances and exits of the
place.

In the presence of the barangay tanod Gonzalado and Castillo’s elder sister Dolly, the police
searched Castillo’s house including the nipa hut where he ran for cover. The men who searched
the residence of Castillo found nothing. But one of the barangay tanods was able to confiscate
from the nipa hut 4 plastic packs of shabu.

Thus an information for violation of RA 6425 was filed against Castillo for possession of 0.31
grams of shabu.

He was found guilty by the RTC. The CA affirmed this decision. His MR was denied. hence this
petition for certiorari. Castillo claims 1)that there was no probable cause to issue the search
warrant since the officer who applied for it had no personal knowledge of the alleged illegal sale
of drugs during a test-buy operation before such application, 2) the nipa hut is 20 meters away
from his house and is no longer within the “permissible area” that may be searched by the police
due to the distance and the warrant did not include the nipa hut as a place to be searched, and 3)
he could not be presumed in possession of shabu just because they were found in the nipa hut.

ISSUE:
Whether a prohibited drug, shabu, found by a barangay tanod in a nipa hut 20 meters away from
the residence of Castillo, accused, may be admitted in evidence for prosecution of possession of
regulated drug.

HELD: NO.
The requisites for the issuance of a search warrant are: (1) probable cause is present; (2) such
probable cause must be determined personally by the judge; (3) the judge must examine, in
writing and under oath or affirmation, the complainant and the witnesses he or she may
produce; (4) the applicant and the witnesses testify on the facts personally known to them; and
(5) the warrant specifically describes the place to be searched and the things to be seized.

1. Probable cause is defined as such facts and circumstances as would lead a reasonably discreet
and prudent man to believe that an offense has been committed and that the objects sought in
connection with the offense are in the place sought to be searched. It only needs to rest on evidence
showing that, more likely than not, a crime has been committed and it was committed by the
accused. A magistrate’s determination of probable cause is paid great deference by a reviewing
court as long as there was substantial basis for that determination. Substantial basis means that
the questions of the examining judge brought out such facts and circumstances as would lead a
reasonably discreet and prudent man to believe that an offense has been committed, and the objects
in connection with the offense sought to be seized are in the place sought to be searched. There is
substantial basis in this case.

2. The warrant must particularly describe the place to be searched and persons or things to be
seized for it to be valid. A designation or description that points out the place to be searched to
the exclusion of all others, and on inquiry unerringly leads the peace officers to it satisfies the
constitutional requirement of definiteness. In this case, the warrant specifically designates the
residence of Castillo. The items were seized by a barangay tanod in a nipa hut 20 meters from
his residence. These items, being found in a place other than the one described in the search
warrant, can be considered fruits of an invalid warrantless search. The contention that since it
was a barangay tanod, a private individual, who found the items, they must still be admissible in
evidence as the constitutional guaranty against unreasonable searches applies only against
government authorities is devoid of merit.

The police asked for the assistance of the barangay tanods. Thus, the tanods acted as agents of
persons in authority as defined under Article 152 of the RPC. The LGC in Section 388 also
describes the function of a barangay tanod as an agent of persons in authority. Thus, the search
was unreasonable and the confiscated items are inadmissible in evidence.

But even if the tanod were considered a private individual, making the items admissible in
evidence, the prosecution failed to establish constructive possession of the drugs.
3. The elements of illegal possession of shabu are 1) the accused is found in possession of a
regulated drug, 2) he is not authorized by law or duly constituted authorities, and 3) he has
knowledge that said drug is a regulated drug.

Constructive possession exists when the drug is under the dominion and control of the accused or
when he has the right to exercise control and dominion over the place where it is found. The records
do not show that Castillo owns the nipa hut nor was it established that he used it as a shop. The
prosecution failed to prove that the nipa hut was under Castillo’s control and dominion.

74. People v. Benny Go, GR 144639, September 12, 2003, Carpio-Morales, J. (Searches and
Seizures)
FACTS:
Police conducted a test-buy operation at the residence of Go where they purchased P1500 worth
of shabu. They did not immediately arrest Go but applied first for a search warrant for Go’s
residence. A raiding team went to Go’s house armed with the warrant commanding them to “make
an immediate search anytime of the day or night” of Go’s residence and to seize “shabu, weighing
scale, and other drug paraphernalia and proceeds of the above crime.”

Jack Go, Benny’s son, was the only one present at the time the policemen went to the residence.
The police introduced themselves, informed him that they had a warrant, and handcuffed him to a
chair. They got barangay kagawads to be witnesses to the search and to sign afterwards the
inventory receipt and affidavit of orderly search. They instructed the handcuffed Jack Go to
witness the search, but he refused saying that there is a barangay official anyway.

The police seized a plastic bag of shabu, a plastic bag of yellowish substance, a weighing scale,
documents, passports, bank books, checks, a typewriter, a check writer, dry seals, stamp pads,
Chinese and PH currency, and Go’s Toyota car. The items, except the car, were brought to the
dining table on the ground floor of Go’s house for inventory.

Go’s wife and his friends arrived one after the other. Go himself arrived when the search was
almost through. The police prepared a handwritten inventory receipt and an “affidavit of orderly
search” which he read to Benny Go, who signed them as did the kagawads and Jack Go.

The police brought Go, his wife, son, and friends, and the items, to Camp Bagong Diwa Taguig,
Metro Manila for investigation. Benny was detained while the others were released. The arresting
officers prepared an affidavit of arrest.

The prosecution presented the Yamato weighing scale recovered on top of Go’s frige although it
was not among those listed in the inventory receipt, affidavit of arrest, or return of search
warrant.

The kagawad witness claimed that the inventory receipt presented in court had been replaced since
their signatures were gone and “Chinese Medicine” was replaced with “undetermined quantity of
white crystalline granules” and that the shabu was not among the items seized and inventoried.
The defense witnesses/version claims that Jack Go signed the affidavit of orderly search and
receipt without having read them and only because he was hit by the police. Benny Go was also
made to sign without having read its contents and Jack Go was prevented from explaining its
contents. On the way to Bicutan, the police told Benny that they wanted P10M from him or he
would be charged of possession of illegal drugs, which amount was reduced later to P5M to P2M
then to P500k. Benny refused the demands since he did not commit any crime.

The RTC convicted him of illegal possession. Hence this appeal.

ISSUE:

HELD:
What constitutes reasonable or unreasonable search or seizure is a purely judicial question
determinable from considering the circumstances, purpose of the search, presence of probable
cause, the manner of serach and seizure, the place or thing searched, and character of the articles
procured.

Since no presumption of regularity may be invoked to justify an encroachment of rights, courts


must cautiously weigh the evidence before them. The warrant must conform strictly to
constitutional and statutory requirements.

In this case, an examination of the testimonies of the police show several irregularities in the
manner by which the search was conducted.

By PO2 Abulencia’s own account, to enter the residence, they deliberately side-swiped Go’s car
parked alongside the road instead of following the regular “knock and announce” procedure under
Section 7 of Rule 126 of RoC. Since the police had not yet notified the occupant of their
intention and authority to conduct a search and without a reasonable cause to believe that prior
notice of service of warrant would endanger its successful implementation, this sideswiping was
unreasonable and unjustified.

He also claims to have immediately handcuffed Jack Go upon entry. He justifies that this was
standard operating procedure and that he was unfamiliar with Jack Go and unsure of how he would
react. But there is no showing of any action of provocation by Jack. Considering the degree of
intimidation, alarm, and fear produced in one suddenly confronted under similar circumstances,
the restraint of Jack was all the more unjustified even after the kagawads arrived.

While the search warrant authorized seizure of “shabu, wighing scale, other drug
paraphernalias and proceeds of the above crime,” the policemen seized numerous other items
clearly unrelated to illegal drugs or drug paraphernalia. While an inventory was prepared, it
did not contain a detailed list of all items seized, particularly the voluminous documents. We
have stressed the necessity for a detailed receipt of the items seized to safeguard the right of the
person (nature and kind of documents).

PO2 Abulencia presented to Benny for his signature the inventory (Section 10, Rule 126) without
showing that Benny was informed of his right not to sign it and to assistance of counsel. Neither
was he warned that the same could be used as evidence against him. We have held that an
admission previously prepared by police is inadmissible for having been obtained in violation of
rights as a person under custodial investigation for an offense. This practice of inducing suspects
to sign receipts for property allegedly confiscated from their possession is unusual and violative
of the constitutional right to remain silent. It is the police who confiscated the items that should
have signed such receipts. This is in the nature of an extra-judicial confession for an offense. The
inventory receipt signed by Go is thus not only inadmissible for violating his custodial right to
remain silent, but it is also an indicum of the irregularity of the manner the raiding team conducted
the search.

The return of search warrant was also not veried under oath as required under Section 12(a) of
Rule 126. The delivery of the items seized to the court which issued the warrant together with a
true and accurate inventory duly verified under oath is mandatory to preclude substitution of
said items. Section 12 of Rule 126 mandates the judge to ensure compliance with the requirements
for 1) the issuance of a detailed receipt, 2) delivery of the seized property to court, together with
3) a verified true inventory of the items seized. Any violation of these constitutes contempt of
court.

Thus, considering these irregularities, the reliance of the trial court on the disputable presumption
that the police regularly performed their duty was misplaced.

The affidavit of orderly search is not under oath, thus only a prepared form filled up after the search
who instructed Benny, Jack and the kagawads to sign as witnesses. Moreover, the affidavit
purports to be executed by Benny, but he was not present when the search took place, having
arrived only when it was “almost through.” Thus, this affidavit prepared without aid of counsel
and by the very police who searched his residence provides no proof of the regularity of the search.

In fact, from the polices’ accounts, their search failed to comply with the provisions of Section 8
of Rule 126:
SEC. 8. Search of house, room, or premises, to be made in presence of two witnesses. —
No search of a house, room, or any other premise shall be made except in the presence of
the lawful occupant thereof or any member of his family or in the absence of the latter,
two witnesses of sufficient age and discretion residing in the same locality.

The police admitted that the search of the upper floor where the shabu was allegedly recovered did
not take place in the presence of Benny or Jack. This requirement is mandatory to ensure
regularity in the execution of the search warrant. That the raiding team summoned two
kagawads to witness the search in the 2nd floor is of no moment. The RoC establishes a hierarchy
among the witnesses. The search should be witnessed by “two witnesses of sufficient age…”
only in the absence of either the lawful occupant or member of his family. Thus, the search
should have been witnessed by Benny’s son Jack who was present. Jack did not waive his right
when he refused to witness since a waiver must be made voluntarily, knowingly, and intelligently.
He was handcuffed to a chair, alone and confronted by five police who deprived him of liberty.
He cannot be considered to have voluntarily, knowingly, and intelligently waived his right to
witness the search. Consent under such intimidating circumstances is no consent.
The numerous irregularities and departure from the procedure mandated by Section 8, Rule 126,
tainted the search with unreasonableness, compelling this Court to apply the exclusionary rule
and declare the seized items inadmissible in evidence. Only in case the prosecution which itself
controls the seizing officials knows that it cannot profit by their wrong, will that wrong be
repressed.

With the exclusion of the shabu, the very corpus delicti, the conviction must be reversed.

The testimonies also fail to provide the moral certainty needed to sustain conviction. The two
kagawads, witnesses to the search chosen by the police in substitution of Jack, both testified that
no shabu was recovered from the residence. The police claim that the kagawads deliberately
falsified their testimonies, but no evidence was presented. The police did not also actively pursue
their complaint for obstruction of justice against the kagawads with the DOJ.

As to the motion for return of personal documents, vehicle, and paraphernalia, the general
rule is that only personal properties particularly described in the search warrant may be seized by
authorities. There are exceptions. Thus, evidence obtained through a warrantless search may be
admissible under: (1) search incident to a lawful arrest; (2) search of a moving motor vehicle;
(3) search in violation of customs laws; (4) seizure of evidence in plain view; and (5) when the
accused himself waives his right against unreasonable searches and seizures.

The police sought to justify seizure of the P52,760 and Chinese Yuan 25,000 as either “proceeds
of the offense” or “means of committing an offense” under the warrant. But neither the money or
car was particularly described in the warrant. In seizing the items, the police were exercising their
own DISCRETION and determining for themselves which items they believed were proceeds
or means of committing the offense. This is absolutely impermissible. The purpose of the
requirement that the articles to be seized be particularly described in the warrant is to leave the
officers of the law with no discretion regarding what articles they should seize.

They also claim that the seizure of passports, bankbooks, typewriter, etc. were “seizure of evidence
in plain view.” Under the plain view doctrine, objects within the “plain view” of an officer who
has a right to be in the position to have that view are subject to seizure:
(a) the law enforcement officer in search of the evidence has a prior justification for an intrusion
or is in a position from which he can view a particular area; (b) the discovery of the evidence in
plain view is inadvertent; (c) it is immediately apparent to the officer that the item he observes
may be evidence of a crime, contraband or otherwise subject to seizure

The police testimonies do not establish how they became aware of the seized items allegedly within
their “plain view.” (“sa mesa lahat iyan”, “atop the dining table located also at the sala”)

The illegal character of the dry seals and stamp pads cannot be said to have been immediately
apparent. SPO1 Fernandez had to first make an impression of the dry seal on paper before he
could determine that it purported to be the seal of the Bureau of Immigration and Deportation. The
counterfeit nature of the seals and stamps was in fact not established until after they had been
turned over to the Chinese embassy and BID for verification.
Nonetheless, since the dry seals and stamps have been certified as counterfeit by BID, they may
not be returned and are confiscated in favor of the State. The bankbooks and passports, not
belonging to Benny, may not be returned. Its seizure may be contested only by the party whose
rights have been impaired thereby and objection to an unlawful search and seizure is purely
personal and cannot be availed of by third parties.

WARRANTLESS ARRESTS
75. Joey Pestilos v. Moreno Generoso, GR 182601, November 10, 2014, Brion, J.
(Warrantless Arrests; requisites of Section 5(b), Rule 113 of RoC, warrantless arrest)
FACTS:
An altercation ensued between Pestilos et al. and Atty. Moreno Generoso at Kasiyahan St. where
they reside. Atty. Generoso called the Batasan Hills Police Station to report the incident. Police
arrived at the scene of the crime and saw Atty. Generoso badly beaten. He pointed to petitioners
as those who mauled him. The police “invited” petitioners to Batasan Hills Police Station for
investigation. The petitioners went with the police. At the inquest proceeding, the city prosecutor
found that petitioners stabbed Atty. Generoso with a bladed weapon. He survived.

The petitioners, in an information, were indicted for attempted murder. Petitioners moved for
regular preliminary investigation on the ground that they had not been lawfully arrested. They
claim that they were just “invited” to the police station. Thus, an inquest proceeding was improper.
The RTC denied this. The CA also denied this. Hence this petition.

ISSUE:
Whether police officers who indicted petitioners in an information for attempted murder after
“inviting” them to the Batasan Hills Police Station for investigation after arriving at the scene of
the crime to a badly beaten Atty. Generoso who pointed at petitioners as those who mauled him
validly arrested petitioners without a warrant.

HELD: YES.
Petitioners claim that no valid warrantless arrest took place as the incident happened two hours
before the officers arrived at the crime scene. The police could not have undertaken a valid
warrantless arrest as they had no personal knowledge that petitioners were the authors of the crime.

Arrest falls under the term “seizure.” The requirements of a warrantless arrest are now summarized
in Rule 113, Section 5of the RoC:
(a) When, in his presence, the person to be arrested has committed, is actually committing,
or is attempting to commit an offense;
(b) When an offense has just been committed, and he has probable cause to believe based
on personal knowledge of facts or circumstances that the person to be arrested has
committed it; and
(c) When the person to be arrested is a prisoner who has escaped from a penal establishment
or place where he is serving final judgment or is temporarily confined while his case is
pending, or has escaped while being transferred from one confinement to another.
Section 5(a) contemplates “in flagrante delicto” warrantless arrests while that in 5(b), a “hot
pursuit” arrest.

We focus on Section 5(b), which is applicable in this case.

Prior to 1940 and the establishment of the 1940 RoC, the gauge for a valid warrantless arrest was
the arresting officer’s reasonable suspicion (probable cause) that a crime was committed and the
person sought to be arrested participated in its commission. The arresting officer’s determination
of probable cause/reasonable suspicion applied both as to 1) whether a crime was committed and
2) whether the person to be arrested committed it. But the 1940 and 1964 RoC required that there
should be actual commission of an offense. The determination of probable cause was limited only
to whether the person to be arrested has committed the offense. These changes were adopted to
minimize arrests based on mere suspicion or hearsay. The 1985 RoC added a qualification that the
offense not only should have been “committed” but “just committed.” This limited the officer’s
time frame. Section 5(b) was further amended to incorporate the word “probable cause” as the
basis of the arresting officer’s determination of whether the person has committed the crime. Hence
it is presently worded.

Thus, 1) the offense was qualified by “just”, connoting immediacy and 2) the warrantless arrest
should be based on probable cause based on the officer’s personal knowledge of facts and
circumstances that the person to be arrested has committed it.

Presently, the elements of Section 5(b) of Rule 113 are: 1) an offense has just been committed;
and 2) the arresting officer has probable cause to believe based on personal knowledge of facts
or circumstances that the person to be arrested has committed it.

1. 1st element - Probable Cause


Probable cause is synonymous to “reasonable cause” and “reasonable grounds.” The standards for
evaluating the factual basis supporting a probable cause assessment are not less stringent in a
warrantless arrest than in a case where a warrant is sought from a judicial officer. The probable
cause in warrantless arrest is based on information that the arresting officer possesses at the time
of arrest and not on information acquired after.

In evaluating probable cause, probability and not certainty is the determinant of reasonableness.
It is a pragmatic question to be determined in each case in light of particular circumstances and
the particular offense involved.

In determining probable cause, the arresting officer may rely on all information in his possession,
his fair inferences therefrom and his observations. Mere suspicion is not probable cause if it is a
mere general suspicion. Probable cause may rest on reasonably trustworthy information and
personal knowledge. Thus, he may rely on information from a witness or victim of a crime and
need not verify such information. Probable cause under Sec.5(b) means an actual belief or
reasonable grounds of suspicion. The grounds of suspicion are reasonable when it is based on
actual facts supported by circumstances sufficiently strong in themselves to create the
probable cause of guilt.
2. Probable cause in Sec.5(b), Rule 113 vs. in preliminary investigations and judicial proceeding
for issuance of warrant of arrest.

The purpose of a preliminary investigation is to determine whether a crime has been committed
and whether there is probable cause to believe that the accused is guilty of the crime and should
be held for trial. It is the existence of facts and circumstances as would excite the belief in a
reasonable mind, acting on facts within the knowledge of the prosecutor, that the person charged
was guilty of the crime for which he was prosecuted. The basis is the submitted documents of the
complainant, respondent, and his witnesses.

Probable cause in judicial proceedings for issuance of a warrant of arrest is defined as the
existence of such facts and circumstances that would lead a reasonably discreet and prudent person
to believe that an offense has been committed by the person sought to be arrested. The judge must
be satisfied that based on submitted evidence, there is sufficient proof that a crime has been
committed and that the person to be arrested is probably guilty thereof. The judge is not yet tasked
to review in detail the evidence in the preliminary investigation. It is sufficient that he personally
evaluates the evidence in determining probable cause to issue a warrant of arrest.

In contrast, the arresting officer’s determination of probable cause under Section 5(b) Rule
113 is based on his personal knowledge of facts or circumstances that the person sought to be
arrested has committed the crime. These facts pertain to actual facts or raw evidence i.e.
supported by circumstances sufficiently strong in themselves to create the probable cause of guilt
of the person to be arrested. A reasonable suspicion must be founded on probable cause coupled
with good faith on the arresting officer.

Thus, the standard for determining “probable cause” is the existence of such facts and
circumstances that would lead a reasonably discreet and prudent person to believe that an
offense has been committed by the person sought to be arrested or held for trial, as the case
may be.

Although these officers, the arresting officer, public prosecutor, and judge, use the same standard
of a reasonable man, they possess dissimilar quantity of facts or circumstances as set by the rules
upon which they must determine probable cause. The arresting officer operates on more limited
facts, evidence, or available information that he must personally gather within a limited time frame.

3. 2nd/3rd elements- Crime has just been committed/Personal knowledge of facts or circumstances
that the person to be arrested has committed it.
In Posadas v. Ombudsman, the killing happened on December 08, 1994, but it was only on
December 11, 1994, 3 days after the crime, on the basis of 2 supposed witnesses, that the NBI
attempted to arrest the suspected perpetrators. It was held that the officers cannot be said to have
personal knowledge.

In People v. Burgos, Cesar surrendered to authorities stating that Ruben forcibly recruited him to
the NPA. A joint team arrested Ruben. The arrest was invalid since the only information of the
police was from a third person. There was no certainty of the commission of a crime.
The requirement that an offense has just been committed means there must be a large measure of
immediacy between the commission of the offense and the arrest. If there is appreciable lapse of
time, a warrant of arrest must be secured. It has been held that arrest one day and six days after the
crime was invalid. Arrests held on the same day and about three hours after the crime were held to
be valid. The Court’s appreciation of the 2nd and 3rd elements depended on the particular
circumstances of the cases.

“Personal knowledge of facts or circumstances” covers facts, or in the alternative, circumstances,


According to Black’s law dictionary, “circumstances are attendant or accompanying facts, events,
or conditions.” Circumstances may pertain to events or actions within actual perception, personal
evaluation or observation of the police officer at the scene of the crime. Thus, even if a police
has not seen someone actually fleeing, he may arrest if based on his personal evaluation of the
circumstances, he could determine the existence of probable cause that the person sought to be
arrested has committed the crime. But the determination of probable cause and the gathering of
facts or circumstances must be made immediately after the crime.

The clincher element in “personal knowledge of facts or circumstances” is the required element
of immediacy within which these facts should be gathered. This guarantees that the police
would have no time to base their probable cause finding on facts or circumstances obtained after
an exhaustive investigation. As the time gap from commission to arrest widens, the pieces of
information gathered are prone to become contaminated and subjected to external factors,
interpretations, and hearsay.

The requirements of a valid warrantless arrest are thus: 1) the crime should have just been
committed, and 2) the arresting officer's exercise of discretion is limited by the standard of
probable cause to be determined from the facts and circumstances within his personal
knowledge. The requirement of probable cause objectifies the reasonableness of the warrantless
arrest for purposes of compliance with the mandate against unreasonable searches.

Thus, in this case, the questions to be resolved are: 1) ) has the crime just been committed when
they were arrested? 2) did the arresting officer have personal knowledge of facts and
circumstances that the petitioners committed the crime? and 3) based on these facts and
circumstances that the arresting officer possessed at the time of the petitioners' arrest, would a
reasonably discreet and prudent person believe that the attempted murder of Atty. Generoso
was committed by the petitioners? Yes.

4. Application in this case


The police blotter stated that the alleged crime was committed at 3:15am on Feb.20, 2005. The
entry of the complaint therein at 4:15am, with Atty. Generoso and petitioners already in the police
station, connotes that the arrest took place less than one hour from occurrence of the crime.

The officers’ personal observation of Atty. Generoso’s bruises when they arrived at the scene
is corroborated by petitioners’ submissions that Atty. Generoso suffered blows from them but that
they did it in self-defense only. The bruises were also corroborated by a medico-legal certificate
issued on the same day of the alleged mauling.
The alleged crime transpired in a community where Atty. Generoso and petitioners reside. Atty.
Generoso positively identified petitioners as those responsible for his mauling and they lived
almost in the same neighborhood. When petitioners were confronted, they did not deny their
participation in the incident with Atty. Generoso.

With these facts and circumstances the police gathered and personally observed less than one hour
from when they arrived until arrest, the police had personal knowledge of facts or circumstances
justifying the warrantless arrests. These circumstances were well within the police’s observation,
perception, and evaluation at the time of arrest.

In determining the reasonableness of warrantless arrests, courts must consider the requirements
under Section 5(b): immediacy, personal knowledge of facts or circumstances, and propriety of
determination of probable cause that the person sought to be arrested committed the crime.

Personal knowledge does not require actual presence at the scene while a crime was being
committed. It is enough that evidence of the recent commission is patent and the police has
probable cause.

Considering the circumstances of the stabbing, the locality where it took place, its occasion, the
personal circumstances of the parties, and immediate on-the-spot investigation that took place, the
warrantless arrests were proper. The inquest proceeding that the city prosecutor conducted was
appropriate.

“Invite” is construed to mean authoritative command. The police could not but have the intention
of arresting petitioners following Atty. Generoso’s account. The police did not need apply violent
physical restraint when a simple directive to petitioners to follow him to the station would produce
a similar effect.

76. Rolito Tambunting v. CA, GR 101837, February 11, 1992, Feliciano, J. (Warrantless
Arrests; no arrest if person himself put himself at police’s disposal)
FACTS:
Eldon Maguan was driving his car along Wilson St., San Juan heading towards P. Guevarra St. He
Tambunting entered Wilson St., which is one-way, and started travelling in the opposite or wrong
direction. Tambunting and Maguan’s cars nearly bumped each other. Tambunting alighted, walked
over, and shot Maguan inside his car. Tambunting then left the scene. A security guard nearby got
his plate number. Police acquired from the scene an empty shell and one round of live ammunition.

The police returned to the scene the day after. They were informed that Tambunting had dined at
Cravings Bake Shop shortly before the shooting. The police obtained a facsimile or impression of
the credit card used by Tambunting from the cashier of the bake shop and the security guard
positively identified him as the person who shot Maguan. Thus, the police launched a manhunt.

Tambunting presented himself before the San Juan Police Station to verify that he was being
hunted by the police accompanied by 2 lawyers. The police detained him. He was positively
identified by an eyewitness. The police that same day filed a complaint for frustrated homicide
against him.
While the complaint was still with the prosecutor and before an information could be filed, Maguan
died of his gunshot wounds. Thus, instead of filing an information for frustrated murder, the
prosecutor filed an information for murder with the RTC. No preliminary investigation was
conducted since accused did not execute a waiver of Article 125 of the RPC.

Tambunting filed with the prosecutor a motion for immediate release and proper preliminary
investigation, alleging that the warrantless arrest was unlawful and no preliminary investigation
was conducted before filing of information. The prosecutor interposed no objection to the grant of
provisional liberty on a cash bond of P100k. the judge approved the cash bond. Tambunting was
released.

The prosecutor moved for leave to conduct preliminary investigation, which was granted. But the
judge issued motu proprio an order on July 17, 1991, stating that the bail was recalled and giving
Tambunting 48 hrs to surrender himself and the grant to conduct preliminary investigation was
recalled.

Tambunting filed a petition for certiorari before the SC assailing the July 17 order. It was
remanded to the CA. The RTC judge ordered his arraignment. During arraignment, he refused to
enter a plea, so it was entered as not guilty.

He filed with the CA a habeas corpus petition. Trial with the RTC commenced. The CA dismissed
the petitions stating that the warrantless arrest was valid and the petition for HC could not be
granted since there was a valid information for murder. Hence this petition for certiorari.

ISSUE:
Whether Tambunting, who went to the police station to verify if he was really being hunted for
being identified by eyewitnesses and information to have been the one who shot Maguan after the
latter almost bumped his car, was lawfully arrested without warrant.

HELD: NO. Not “arrested.”


Tambunting argues that he was not lawfully arrested without warrant because he went to the police
station 6 days after the shooting. Thus, the crime had not been “just committed” at the time of
arrest. None of the arresting police had been an eyewitness to the shooting, thus there was no
“personal knowledge.”

The arrest took place 6 days after the shooting. The “arresting” officers were not present at the
time of the shooting. Neither could the “arrest” effected 6 days after the shooting be regard as
effected when the shooting had “just been committed” within Section 5(b) (Rule 113). None of the
officers had “personal knowledge” that Tambunting was the gunman who shot Maguan. The
information the police acted upon were derived from statements by alleged eyewitnesses to the
shooting. These do not constitute “personal knowledge.” Thus, there was no lawful warrantless
arrest within the meaning of Section 5 of Rule 113. Rule 112, Section 7 is also not applicable.

Tambunting was not arrested at all. When he walked into the San Juan Police Station, he placed
himself within the disposal of the police authorities. He did not state that he was surrendering
himself to avoid the implication that he was admitting that he had slain Maguan. When the police
filed a complaint for frustrated homicide with the prosecutor, the latter should have immediately
scheduled a preliminary investigation to determine if there was probable cause for charging
Tambunting in court. But the prosecutor proceeded under the erroneous supposition that Rule 112
Sec. 7 was applicable and required Tambunting to waive Article 125 of the RPC as a condition to
carry out such investigation. This was wrong, for Tambunting was entitled to a preliminary
investivation and that right should have been accorded without any conditions. Also, since
Tambunting was not arrested, with or without warrant, he was entitled to release forthwith
subject only to his appearing at the preliminary investigation.

Tambunting did not waive his right to preliminary investigation since he had from the very
beginning demanded that it be conducted. While that right is statutory, it is a component part of
due process in criminal justice. It is not a mere formal or technical, but a substantive right.

77. People v. Edison Sucro, GR 93239, March 18, 1991, Gutierrez, Jr., J. (Warrantless
Arrest)
FACTS:
Edison Sucro was charged with and convicted of violation of the Dangerous Drugs Act (possession
of 19 pieces of marijuana cigarette sticks and 4 tea bags of dried marijuana leaves). He appealed
from the RTC’s conviction.

Fulgencio, member of INP, Kalibo Aklan, was instructed to monitor Sucro because of information
that he was selling marijuana. Fulgencio saw Sucro enter a chapel, taking marijuana from a cart
found inside the chapel and hand it to a buyer. Sucro went back to the chapel, came out with
marijuana again, and gave it to a group of persons. Macabante, buyer of marijuana, admitted that
he bought it from Sucro. Sucro was arrested.

ISSUE:
Whether Sucro, monitored and seen to have taken marijuana from a chapel and selling the same to
multiple persons and thereafter arrested, was lawfully arrested without warrant.

HELD: YES.
Sucro claims that there was enough time for the police to apply for search and arrest warrant.

Rule 113 Sec. 5 provides for instances where arrest without warrant is lawful: (a) When in his
presence, the person to be arrested has committed, is actually committing, or is attempting to
commit an offense; (b) When an offense has in fact just been committed, and he has personal
knowledge of facts indicating that the person to be arrested has committed it;"

An offense is committed in the presence or within the view of an officer when the officer SEES
the offense, although at a distance, or HEARS the disturbances created thereby and proceeds
at once to the scene thereof.

Fulgencio within a distance of two meters saw Sucro conduct his nefarious activity, talk to persons,
go inside a chapel, and return to them and exchange some things. He did this 3 times.
Also, the fact that Macabante, when intercepted by police, was caught throwing the marijuana stick
and admitted that he bought the same from Sucro indicates that Sucro had just sold it and thus,
committed an illegal act of which the police officers had personal knowledge, having monitored
Sucro. Officers have personal knowledge of the actual commission of the crime when it had earlier
conducted surveillance activities of the accused.

The failure of the officers to secure a warrant stems from the fact that their knowledge from the
surveillance was insufficient to fulfill the requirements for issuance of a search warrant, which is
probable cause. Among the exceptions from the rule that searches and seizures must be supported
by a valid warrant is a search incidental to a lawful arrest under Sec.12, Rule 126.

Since there is nothing unlawful about the arrest, the fruits are admissible.

WARRANTLESS SEARCHES
78. Romeo Posadas v. CA, GR 89139, August 2, 1990, Gancayco, J. (Warrantless Searches)
FACTS:
2 members of the Integrated National Police (INP) were conducting surveillance along Magallanes
St., Davao City. While within Rizal Memorial Colleges, they spotted Posadas carrying a “buri”
bag and noticed him acting suspiciously. They approached him and identified themselves as
members of INP. Posadas tried to flee but was thwarted. They then checked the “buri” bag where
they found a revolver, 4 live ammunitions for different caliber guns, and a smoke tear gas grenade.
They brought Posadas to the police station for further investigation. Posadas failed to produce
license for authority to possess firearms. Posadas was prosecuted for illegal possession of firearms
and ammunitions in the RTC. The RTC found him guilty. The CA affirmed the conviction. Hence
this petition for review, alleging illegal search and seizure and arrest, thus the articles are
inadmissible against him.

ISSUE:
Whether police who noticed Posadas carrying a “buri” bag acting suspiciously, approached him
and identified themselves as police officers, and where Posadas thereafter attempted to flee but
was thwarted, with the police recovering from the “buri” bag a revolver, live ammunitions, and a
smoke grenade, validly searched the bag and arrested Posadas.

HELD:
Rule 113, Sec.5 of the 1985 Rules on Criminal Procedure provides for lawful warrantless arrests.
The OSG argues that when the police approached Posadas, he was actually committing illegal
possession of firearms and ammunitions in the presence of the police.

When the police identified themselves and apprehended Posadas when he attempted to flee, they
did not know that he had committed or was actually committing illegal possession. They just
suspected that he was hiding something in the buri bag but did not know what it contained. The
said circumstances did not justify arrest without warrant.

But there are instances where a warrant and seizure can be effected without being preceded by an
arrest, like the “stop and search” without a search warrant at military or police checkpoints,
which is constitutionally valid. Not all searches and seizures are prohibited. Those which are
reasonable are not forbidden. A reasonable search is not determined by any fixed formula but is
resolved according to the facts of each case. Between the inherent right of the state to protect
its existence and promote public welfare and an individual’s right against a warrantless search
which is reasonably conducted, the former should prevail.

The search in this case is reasonable as it was effected on the basis of PROBABLE CAUSE. The
probable cause is that when Posadas acted suspiciously and attempted to flee, thus there was
probable cause that he was concealing something illegal in the bag and it was the right and duty
of the officers to inspect it. It would be too much to require the officers to secure a warrant in this
case as it may prove to be useless, futile, and too late.

79. Mario Veridiano v. People. GR 200370, June 07, 2017, Leonen, J. (Warrantless Searches)
FACTS:
Veridiano was charged with illegal possession of dangerous drugs under Article II, Section 11 of
RA 9165.

A concerned citizen called the police station, informing him that a certain “Baho”, later identified
as Veridiano, was on the way to San Pablo City to obtain illegal drugs. Police set up a checkpoint
at Brgy. Taytay, Laguna. The police officers at the checkpoint personally knew Veridiano. They
chaned upon Veridiano inside a passenger jeepney coming from San Pablo, Laguna. They flagged
down the jeep and asked the passengers to disembark. The officers recovered from Veridiano a tea
bag containing marijuana.

PO1 Cabello marked it with his initials. Veridiano was arrested and apprised of his rights, then
brought to the police station. It was turned over to PO1 Solano who also placed his initials. He
made a lab exam request which he personally brought with the tea bag to the PNP Crime
Laboratory.

The RTC convicted Veridiano. He appealed, saying that he was illegally arrested, and that the tea
bag is inadmissible for being the “fruit of a poisonous tree.” The CA affirmed his guilt. Hence this
petition on certiorari.

Veridiano argues that the tea bag containing marijuana was seized in violation of his right against
unreasonable searches and seizures and that the arrest was illegal. Veridiano was merely seated
inside the jeepney at the time of apprehension. He did not act in any manner that would give the
officers reasonable ground to believe that he had just committed or is committing a crime. Reliable
information is insufficient to constitute probable cause to support a valid warrantless arrest. Since
the arrest was illegal, the search was also illegal.

ISSUE:
Whether Veridiano, who was searched then arrested in a checkpoint when found in possession of
marijuana while in a jeepney, where he was searched due to a tip received by the police that he
was on the way to San Pablo city to obtain illegal drugs, was validly searched and arrested.

HELD: NO.
The invalidity of an arrest leads to several consequences among which are: (a) the failure to acquire
jurisdiction over the person of an accused; (b) criminal liability of law enforcers for illegal arrest;
and (c) any search incident to the arrest becomes invalid thus rendering the evidence acquired
as constitutionally inadmissible.

Lack of jurisdiction from invalid arrest must be raised through a motion to quash before an accused
enters his plea. Otherwise, the objection is deemed waived. The voluntary submission of an
accused to the jurisdiction of the court and his active participation during trial cures any defect or
irregularity that may have attended an arrest. The legality of an arrest affects only the jurisdiction
of the court over the person of the accused.

Nevertheless, this does not preclude questioning the admissibility of evidence seized. Jurisdiction
over an accused and constitutional inadmissibility of evidence are separate consequences of an
illegal arrest.

The constitutional guarantee does not prohibit all forms of searches and seizures, only those that
are unreasonable. The language of the constitution implies that searches and seizures are normally
unreasonable unless authorized by a validly issued warrant. But there are exceptional
circumstances when searches are reasonable even when warrantless: (1) a "warrantless search
incidental to a lawful arrest," (2) search of "evidence in 'plain view,'" (3) "search of a moving
vehicle," (4) "consented warrantless search[es]," (5) "customs search," (6) "stop and frisk," and
(7) "exigent and emergency circumstances." What is reasonable in any given situation is purely
a judicial question depending upon the unique and distinct factual circumstances.

1. Warrantless arrest was unlawful.


A search incidental to a lawful arrest requires that there must be a lawful arrest before a search.
For a lawful arrest, either there must be a valid warrant or it is justified under Rule 113, Section 5
of the Rules of Criminal Procedure.

The first kind (“when, in his presence…) is in flagrante delicto arrest. The validity of this
warrantless arrest requires compliance with the OVERT ACT test. Two elements must concur:
(1) the person to be arrested must execute an overt act indicating that he [or she] has just
committed, is actually committing, or is attempting to commit a crime; and (2) such overt act is
done in the presence or within the view of the arresting officer."

In People v. Racho, the warrantless search was considered unlawful. Police received information
that a man was in possession of illegal drugs on board a bus going to Baler and wearing a red and
white striped shirt. Police waited for the bus along the highway. When it arrived, Racho
disembarked and waited for a tricycle. The officers approached him and invited him to the station.
As Racho pulled out his hands from his pocket, an envelope with shabu fell. Racho was not
committing a crime in the presence of the officers and it was based only on a tip.

Section 5(b), Rule 113 of RoC pertains to a hot pursuit arrest. It requires that an offense has just
been committed. It connotes immediacy in point of time. Officers need not personally witness
the crime, but must have personal knowledge of facts and circumstances indicating that the person
sought to be arrested committed it.
In People v. Gerente, it was held that the officers had personal knowledge of the crime as they
saw the victim dead in the hospital and when they inspected the scene of the crime, they found the
instruments that the killers had used to bludgeon him to death. The eye-witness also pinpointed
her neighbor, Gerente, as the killers.

Veridiano’s arrest could not be justified as in flagrante delicto under Section5(a) as he was not
committing a crime at the checkpoint. He was merely a passenger who did not exhibit any
unusual conduct to incite suspicion. The tip received as the sole basis of the warrantless arrest is
insufficient without any overt act that a crime has been, was being, or is about to be committed.

It cannot also be justified under Section5(b) as the officers had no personal knowledge that
petitioner just committed an offense. A hearsay tip by itself does not justify warrantless arrests.
Officers must have personal knowledge of facts, based on their observation, that the person has
just committed a crime. This is what gives rise to probable cause justifying warrantless search
under Section 5(b).

3. Search not justified under “stop and frisk” searches.


A stop and frisk search is defined as “the act of a police officer to stop a citizen on the street
and interrogate him, and pat him for weapons or contraband.” The allowable scope of such
is limited to a “protective search of outer clothing for weapons.”

Officers do not have unbridled discretion in conducting stop and frisk searches. While probable
cause is not required, it cannot be validated on mere suspicion or hunch. Officers must have a
genuine reason to believe, based on their experience and the particular circumstances of each
case, that criminal activity may be afoot. Reliance on one or even no suspicious activity cannot
produce a reasonable search.

In people v. Solayao, the police were conduting an intelligence patrol to verify reports of armed
persons in Caibiran. They met a group of drunk men. When the officers approached, the accused’s
companions fled, leaving him behind. He was searched and from the accused was found a firearm.
It was held that there was justifiable reason to stop and frisk accused when his companions fled
upon seeing the government agents.

Veridiano was a mere passenger in a jeepney who did not exhibit any act that would give police
officers reasonable suspicion to believe that he had drugs in his possession. Reasonable
persons will act in a nervous manner in any check point. There was no evidence that the officers
had basis or personal knowledge that would reasonable allow them to infer anything suspicious.

His silence or lack of resistance cannot be consent. Although the right against unreasonable
searches and seizures may be surrendered through a valid waiver, the prosecution must prove that
the waiver was executed with clear and convincing evidence. Consent to warrantless search and
seizure must be unequivocal, specific, intelligently given, and unattended by duress or
coercion. Mere passive conformity or silence is only an implied acquiescence, which is no consent.
This is a natural reaction to a coercive environment brought about by the officers’ excessive
intrusion into his private space.
3. Search of moving vehicle.
Officers cannot be expected to appear before a judge and apply for a search warrant when time is
of the essence considering the efficiency of vehicles in facilitating transactions involving
contraband. But the inherent mobility of vehicles cannot justify all kinds of searches. Officers must
act based on probable cause.

A checkpoint search is a variant of a search of a moving vehicle. Checkpoints are allowed in


exceptional circumstances to protect lives and ensure safety and when government survival is in
danger. Since these intrude on a motorist’s right to free passage to a certain extent, they must be
conducted in a way least intrusive to motorists. The extent of routine inspections must be limited
to a visual search. A search where the officer merely draws aside the curtain of a vacant vehicle,
looks into it, or flashes a light therein is not unreasonable.

But an extensive search may be conducted on a vehicle at a checkpoint when law enforcers have
probable cause to believe that the vehicle’s passengers committed a crime or if it contains
instruments of an offense. An extensive search is not justified without probable cause while
routinary searches of moving vehicles are allowed if limited to a visual search.

In this case, the extensive search was done without probable cause to believe that Veridiano
violated any law except for the tip they received. They did not observe any peculiar activity from
him that would arouse their suspicion or verify the tip. Also, the search was flawed at inception as
the checkpoint was set up to target the arrest of the accused.

Thus, the marijuana is inadmissible. Veridiano was acquitted.

80. Ricardo Valmonte v. Gen. Renato de Villa, GR 83988, September 29, 1989, Padilla, J.
(Warrantless Searches)
FACTS:
The National Capital Region District Command (NCRDC) was activated pursuant to Letter of
Instruction 02/87 of PH General Headquarters, AFP, with the mission of conducting security
operations within its area of responsibility for the purpose of establishing effective territorial
defense, maintaining peace and order etc. in the NCR. As part of its duty, the NCRDC installed
checkpoints in various parts of Valenzuela, Metro Manila.

Petitioners aver that because of the checkpoints, the residents of Valenzuela are worried of being
harassed and of their safety being placed at the arbitrary disposition of the military manning the
checkpoints considering that their cars and vehicles are being subjected to regular searches
especially at night or dawn, without search warrant or court order. Their fear increased when
Benjamin Parpon was gunned down allegedly by NCRDC members manning the checkpoint when
he refused to submit himself to the checkpoint and for continuing to speed off despite warning
shots fired in the air. Valmonte also claims that his car was searched on several occasions in the
checkpoints. There are also instances where citizens are harassed.

ISSUE:
Whether a general claim of fear and apprehension by Integrated Bar of the Philippines members
over military checkpoints around Valenzuela that allegedly conducts unreasonable searches of
vehicles passing through such checkpoints may be entertained.

HELD: NO.
Petitioners’ concern for their safety and apprehension at being harassed at the checkpoints are not
sufficient grounds to declare the checkpoints per se illegal. No proof was presented to show that
the military committed specific violations of petitioners’ right against unlawful search and seizure
or other rights. Individual petitioners who do not allege that any of their rights were violated are
not qualified to bring the action as real parties in interest.

The constitutional right against unreasonable searches is a personal right invocable only by those
whose rights have been infringed or threatened to be infringed. Valmonte’s general allegation,
without more, i.e. without stating the details of the incidents which violate his rights, is not
sufficient to enable the Court to determine if there was a violation of his rights. Not all searches
and seizures are prohibited. Those reasonable are allowed as determined according to the facts of
each case.

The checkpoints may be considered as a security measure to enable the NCRDC to pursue its
mission. Between the inherent right of the state to protect its existence and promote public welfare
and an individual’s right against a warrantless search which is reasonably conducted, the former
should prevail.

While the manning of checkpoints is susceptible of abuse in the same manner that all governmental
power is susceptible, at the cost of occasional discomfort or irritation to the citizen, the checkpoints
during these abnormal times, when reasonably conducted, are part of the price we pay for an
orderly society and a peaceful community.

81. Erwin dela Cruz v. People, GR 209387, January 11, 2016, Leonen, J. (Warrantless
Searches; Warrantless searches in ports)
FACTS:
Dela Cruz was an OJT trainee of an inter-island vessel. He frequently traveled back and forth
taking a vessel. He was at a pier of the Cebu Domestic Port to go home to Iloilo. While buying a
ticket, he left his bag on the floor with a porter. It took him 15 minutes to purchase a ticket. He
then proceeded to the terminal entrance and placed his bag on the x-ray scanning machine for
inspection. The operator of the x-ray, Flores, saw firearms inside his bag. Flores called the attention
of port personnel Igot, baggage inspector. Igot asked Dela Cruz if he owned the bag. Dela Cruz
said yes and consented to Igot’s manual inspection.

3 revolvers, NBI clearance, seaman’s book, other personal items, 4 live ammunitions were found
in the bag. When asked if he had the proper documents therefor, he said no. Dela Cruz was then
arrested and informed of his violation of a crime punishable by law and of his rights.

He was charged in two informations for violation of RA 8294 for illegal possession of firearms
and COMELEC resolution 7764 in relation to Sec.261 of BP881.
The RTC convicted him of violating the gun ban under COMELEC resolution 7764. It did not
give credence to Dela Cruz’s claim that the firearms were planted inside his bag while buying a
ticket. It dismissed the case for violation of RA 8294. The CA affirmed the RTC’s judgment.
Hence this petition.

Dela Cruz claims that as OJT trainee of an inter-island vessel, he was well-acquainted with the
inspection scheme at the ports and he would not have placed prohibited items like unlicensed
firearms inside his luggage knowing the consequences of such action. He claims that since he
believed that no incriminating evidence would be found in the bag out of innocent confidence,
there was no voluntary waiver against a warrantless search.

ISSUES:
1. Whether Dela Cruz, in believing that there would be nothing illegal found in his bag and thus
consented to its search by port authorities, may be deemed to have waived his right against
unreasonable searches and seizures.
2. Whether there was a valid search of a bag, assuming there was no waiver.
HELD:
There are three (3) points of alleged intrusion into the right to privacy of petitioner: 1) when he
gave his bag for x-ray, 2) when the baggage inspector opened his bag and called port authority
police, and 3) when the officer opened the bag to search and seize the firearms and ammunition.

A. The first point was at the x-ray. With regard to searches and seizures, the standard imposed on
private persons is different from that imposed on state agents. In People v. Marti, the private
shipping company, following standard procedure, opened packages sent by the accused for
shipment to Switzerland and detected a peculiar odor. They found dried marijuana leaves and
reported the matter to the NBI. It was held that there was no unreasonable search or seizure as the
evidence was procured not by the state acting through its agencies or officers. The Bill of Rights
does not govern relationships between individuals and cannot be invoked against acts of
private individuals. Thus, items seized pursuant to a reasonable search by private persons are not
covered by the exclusionary rule.

We must revisit the history and organizational structure of the PH Ports Authority (PPA) to
determine if it is a private or public person. The PPA’s revised charter allows it to make rules and
regulation for management etc. of any port for maintenance of good order. EO 513 gave it police
authority to regulate entry and exit and movement within ports of vehicles and watercraft. Control
of ports was transferred to Cebu Port Authority, which adopted security measures imposed by the
Office for Transportation Security. It is clothed with authority by the state to oversee the security
of persons and vehicles within its ports. While there is distinction between port personnel and
port police officers considering that port personnel are not necessarily law enforcers, both should
be considered agents of government under Article III of the Constitution. The actions of port
personnel during routine security checks at ports have the color of a state related function.

Nonetheless, searches pursuant to port security are not unreasonable per se. In People v. Suzuki,
the accused entered the pre-departure area of an airport terminal carrying a small box. The metal
detector sounded when he passed through. When he was asked to open the box, he answered “open,
open.” Marijuana were found therein. It was held that the search was a valid exception to the
prohibition against warrantless searches as it was pursuant to a routine airport security procedure.
The reason is because there is a reasonable reduced expectation of privacy when coming into
airports or ports of travel. Persons may lose the protection of the search and seizure clause by
exposure of their persons or property to the public in a manner reflecting lack of subjective
expectation of privacy, which expectation society is prepared to recognize as reasonable. This
is akin to routine security procedures in domestic ports.

Port authorites were acting within their duties and functions when it used x-ray scanning machines
for inspection of passengers’ bags.

B. Second point of intrusion.


The port personnel’s actions proceed from the authority and policy to ensure the safety of travelers
within the port. Petitioner already submitted himself voluntarily for inspection by placing his
bag in the x-ray machine. He had the choice not to travel if he did not want his bag inspected. It is
not too burdensome to be an affront to a person’s right to travel if weighed against the safety of all
passengers.

C. Third point of intrusion- submission to port security measures.


A warrantless search is presumed unreasonable. But the exceptions where warrantless searches
are deemed legitimate are (1) warrantless search incidental to a lawful arrest; (2) seizure in "plain
view"; (3) search of a moving vehicle; (4) consented warrantless search; (5) customs search; (6)
stop and frisk; and (7) exigent and emergency circumstances.

In case of consented searches or waiver, to constitute a waiver, it must appear that 1) the right
exists; 2) the person had knowledge, either actual or constructive, of the existence of such
right; and 3) the person had actual intention to relinquish the right.

1. YES. (Waived)
He claims that he did not validly consent to the search and did not have intention to relinquish his
right against a warrantless search.

There was probable cause that petitioner was committing a crime leading to the search of his
personal effects. When the search revealed firearms and ammunitions, he is deemed caught in
flagrante delicto, justifying his arrest under Section 5(a), Rule 113. Having permitted authorities
to search his person and premises, petitioner is precluded from claiming an invalid warrantless
search when he voluntarily submitted to the search on his person. His consent was not given
under intimidating or coercive circumstances.

His belief that no incriminating evidence would be found does not automatically negate valid
consent to the search when incriminating items are found. His belief must be measured against the
totality of the circumstances. He voluntarily submitted himself and was familiar with the security
measures in ports.
2. YES. The search conducted on petitioner’s bag is valid under a valid consented search and
during routine port security procedures.

D. Not a customs search.


Customs searches are allowed when “persons exercising police authority under the customs law
effect search and seizure in the enforcement of customs laws.” The Tariff and Customs Code
provides authority therefor (Sec.2203). To be a valid customs search: (1) the person/s conducting
the search was/were exercising police authority under customs law; (2) the search was for the
enforcement of customs law; and (3) the place searched is not a dwelling place or house. In this
case, the search was part of routine port security measures. It was not conducted by persons
authorized under customs law nor motivated by provisions of the Tariff and Customs Code or other
customs laws. Although customs searches usually occur within ports or terminals, the search must
be to enforce customs laws.

82. Ricardo Papa v. Remedios Mago, GR L-27360, February 28, 1968, Zaldivar, J.
(Warrantless Searches; Customs search without a search warrant)
FACTS:
Petitioner Martin Alagao, head of the counter-intelligence unit of the Manila Police Department,
acting upon reliable information to the effect that a certain shipment of personal effects, allegedly
misdeclared and undervalued, would be released the following day from the customs zone of the
port of Manila, and upon orders of Chief of Police Ricardo Papa, conducted surveillance at gate 1
of customs zone. When the trucks left gate 1, elements of the counter-intelligence unit went after
them and intercepted them. The loads of the two trucks, 9 bales of goods, and the trucks were
seized. Upon investigation, a person claimed ownership and showed to the policemen a statement
of receipts of duties collected on informal entry issued by the Bureau of Customs (BoC).

Claiming to have been prejudiced by the seizure, Mago and Valentin Lanopa filed with the CFI a
petition for mandamus alleging that Mago owned the goods seized and that she hired trucks owned
by Lanopa to transport the goods to her residence; that the goods were seized by police without a
search warrant; that the bales had not yet been opened although Papa had arranged with the
Commissioner of Customs regarding their disposition, and unless restrained, their rights would be
violated and they would suffer irreparable injury. They thus prayed for a restraining order.

Judge Jarencio issued an order restraining the officers from opening the bales. But when the order
was received by petitioners, some bales had already been opened and by the examiners of the BoC.
The articles were inventoried. Mago moved for their release since the inventory did not show
articles of prohibited importation. This was opposed as the articles mostly were undeclared and
thus subject to forfeiture. Respondent judge ordered the release of the goods upon bond of P40,000.

Papa filed an MR and, without waiting for the CFI’s action with the MR alleging no plain, speedy,
adequate remedy, petitioners filed this petition for prohibition and certiorari. They claim that the
CFI had acted beyond its jurisdiction.

ISSUE:
Whether the search and seizure of a truck pursuant to authority given to the police officers by the
Commissioner of Customs under the Tariff and Customs Code to search any vehicle suspected of
carrying any prohibited or dutiable article imported contrary to law may be validly done without a
search warrant.
HELD: YES.
The goods were imported from Hongkong. As long as the importation has not been terminated,
the imported goods remain under jurisdiction of the BoC. Importation is deemed terminated only
upon payment of duties, taxes, and other charges upon the articles at the port of entry and the legal
permit for withdrawal shall have been granted.

The record shows that the duties and taxes had not yet been paid in full. They were also
underdeclared to presumably avoid payment of duties. Thus, they were subject to forfeiture.
Merchandise, the importation of which is effected contrary to law, is subject to forfeiture.

It is also claimed that Alagao could not seize the goods without a search warrant. The Chief of the
Manila Police, Papa, having been deputized by the Commissioner of Customs, could, for purposes
of the enforcement of customs and tariff laws, effect searches, seizures, and arrests, and it was
his duty to make seizure of any article when subject to forfeiture under customs and tariff laws.
He could lawfully examine any box or container if he had reasonable cause to suspect the
presence therein of dutiable articles introduced into the PH contrary to law and to stop and
search any vehicle, beast, or person reasonably suspected of holding or conveying such article.

Alagao and his companion policemen had authority to effect the seizure without warrant. The
Tariff and Customs Code does not require said warrant in this case. The Code authorizes persons
having police authority under Sec.2203 to enter, pass through, or search any land, inclosure,
warehouse, etc., not being a dwelling house, and any vessel or aircraft and trunk, package, or any
person etc. suspected of holding or conveying any dutiable or prohibited article introduced into
the PH contrary to law without mentioning the need of a search warrant. Only in the search of a
dwelling house is a search warrant required.

83. People v. Victor Cogaed, GR 200334, July 30, 2014, Leonen, J. (Warrantless Searches)
FACTS:
Police Senior Inspector Bayan received a text message from an unidentified civilian informer than
one Marvin Buya would be transporting marijuana. PSI Bayan organized checkpoints in order to
“intercept the suspect.” A passenger jeep arrived at SPO1 Taracatac’s checkpoint. The driver
disembarked and signaled to Taracatac, indicating the two male passengers who were carrying
marijuana. Taracatac approached the two male passengers, Cogaed and Santiago Dayao. Cogaed
was carrying a blue bag and a sack while Dayao, a yellow bag.

Taracatac asked Cogaed and Dayao about the contents of their bags. They said that they did not
know since they were transporting their bag as a favor for their barriomate Marvin. Cogaed opened
the bag, revealing three bricks of marijuana. Taracatac arrested Cogaed and Dayao. They were
charged under RA 9165 for illegal possession of dangerous drugs. The case against Dayao was
dismissed as he was only 14 years old at that time.

The RTC convicted Cogaed. The CA affirmed the conviction, finding that he waived his right
against warrantless searches when he “without any prompting from SPO1 Taracatac, he voluntarily
opened his bag.” Hence this appeal.

ISSUE:
HELD:
The right to privacy has many dimensions, one of which is its prohibition of unreasonable searches
and seizures under Section 2, Article III.

There are instances when searches are reasonable even without warrants such as searches
incidental to lawful arrests etc.

1. Stop and Frisk (Terry searches)


Stop and frisk searches are conducted to prevent the occurrence of a crime. The search in this case
was initially a stop and frisk search, but it did not comply with all the requirements of reasonability.

These searches are necessary for law enforcement. They must be given the legal arsenal to prevent
commission of offenses. But this should be balanced with need to protect the privacy of citizens.
The balance lies in “suspiciousness” present in the situation. This may be based on the experience
of the police officer. Experienced officers have personal experience dealing with criminals, thus
they should have the ability to discern whether an individual is acting in a suspicious manner.

Cogaed was simply a passenger carrying a bag and travelling in a jeepney. There was nothing
suspicious about riding a jeepney or carrying a bag. The assessment of suspicion was made
not by the officer but by the jeepney driver. It was the driver who signaled to police that
Cogaed was “suspicious.” The driver had to point to Cogaed or he would not have been identified
by the officers.

It is the officer who should observe facts that would lead to a reasonable degree of suspicion of
a person; he should not adopt the suspicion initiated by another person.

2. Probable cause
For warrantless searches, probable cause was defined as “a reasonable ground of suspicion
supported by circumstances sufficiently strong in themselves to warrant a cautious man to
believe that the person accused is guilty of the offense with which he is charged. It does not
have to be probable cause, but it cannot be mere suspicion. It has to be a “genuine reason” in
light of the officer’s experience and surrounding conditions to warrant the belief that the person
has weapons concealed about him. There must be presence of more than one seemingly innocent
activity, which, taken together, warranted a reasonable inference of criminal activity, and not only
of one single suspicious circumstance.

There was not a single suspicious circumstance in this case. The person searched was not even
the person mentioned by the informant. While Cogaed mentioned that he was transporting the bag
to Marvin Buya, this is only one circumstance, not enough reason to search Cogaed.

A stop and frisk search serves two-fold interests: 1) effective crime prevention and detection and
2) safety and self-preservation. This search was originally limited to outer clothing and for the
purpose of detecting dangerous weapons. Jurisprudence also allows stop and frisk for cases
involving dangerous drugs.
3. None of the other exceptions to warrantless searches exist to allow the evidence to be admissible.
None of the instances enumerated in Rule 113, Section 5 of the RoC were present. This is also not
a search incidental to a lawful arrest under Rule 126, Section 13. For in flagrante delicto
warrantless arrest, there must be 1) overt act indicating that he has just committed, is actually, or
attempting to commit a crime, and 2) the overt act is done in the presence or view of the arresting
officer. Both were missing when Cogaed was arrested. There were no overt acts. He was also not
an escapee prisoner.

4. There is no valid waiver of Cogaed’s constitutional rights even if he did not object when the
police asked him to open his bags. His silence or lack of aggressive objection was a natural
reaction to a coercive environment brought about by the officer’s excessive intrusion into his
private space.

For a valid waiver of constitutional right, it is not sufficient that the officer introduce himself. He
must inform the person to be searched that any inaction on his part will amount to a waiver
of his objections that the circumstances do not amount to a reasonable search. The officer
must communicate this clearly and in a language known to the person about to waive his rights.

Thus, the marijuana bags are inadmissible. Cogaed is acquitted.

84. Terry v. Ohio, 392 U.S. 1, June 10, 1968, Mr. Chief Justice Warren (Warrantless
Searches; Stop and Frisk; reasonable- facts, circumstances, experience of officer, suspicious,
outer clothing)
FACTS:
Petitioner Terry was convicted of carrying a concealed weapon. The prosecution presented 2
revolvers and some bullets seized from Terry and a co-defendant, Chilton, by Cleveland Police
McFadden.

While McFadden was patrolling in plain clothes, his attention was attracted by two men, Chilton
and Terry. He was unable to say precisely what first drew his eye to them. But he testified that he
had been a policeman for 39 years and a detective for 35, and he had been assigned to patrol the
area of downtown Cleveland for shoplifters and pickpockets for 30 years. He had developed
routine habits of observation over the years. He said that “they didn’t look right to me at the time.”
Thus, he observed them from a store 300-400 feet away.

He saw one of the men leave the other and walk past some stores. The man paused for a moment
and looked in a store window, then walked a short distance, turned around and walked back
toward the corner, pausing again to look in the same store window. He rejoined his companion
and the two conferred briefly. The second man went through the same motions and looking through
the same window. The two repeated this ritual alternatively 5 to 6 times each, around a dozen trips
in all. A third man approached them and talked to them briefly, then left. Chilton and Terry
resumed their measured pacing, peering, and conferring. After 10-12 minutes, the two men walked
of together following the path earlier of the third man.

Officer McFadden had become suspicious. He suspected the two men of casing a job, a “stick-up.”
He followed the men and saw them talk to the third man. McFadden approached the three men,
identified himself as a police and asked for their names. When the men mumbled something in
response, McFadden grabbed Terry, spun him around, and patted down the outside of his clothing.
McFadden felt a pistol. He ordered all three to enter a Zucker’s store where he removed Terry’s
overcoat, a revolver from the pocket, and ordered the men to face the wall with their hands raised.
He patted Chilton and the third man, Katz, and found another revolver from Chilton, but not Katz.
He patted the outer garments and did not put his hands beneath the garments until he felt the guns.

The three were charged for carrying concealed weapons. The trial court ruled that McFadden had
reasonable cause to believe that defendants were conducting themselves suspiciously based on his
experience. Thus, he had the right to pat down the outer clothing of these men who he had
reasonable cause to believe might be armed. Their motion to suppress evidence, the guns, were
thus denied.

The CA affirmed their conviction. The SC of Ohio dismissed their appeal.

ISSUE:
Whether the search conducted by officer McFadden on Chilton and Terry, who he found acted
suspiciously by going back and forth and looking through a store window several times and
following a third man with whom they conversed with briefly earlier, where the officer found the
three men thereafter conversing, by patting down their outer garments and feeling a revolver which
he confiscated where the officer has been a policeman for 39 years and detective for 35 and had
been assigned in the same area for shoplifters for 30 years is valid.

HELD: YES.
The Fourth Amendment provides that "the right of the people to be secure in their persons, houses,
papers, and effects, against unreasonable searches and seizures, shall not be violated. . . ." What
the constitution forbids is not all searches and seizures, but unreasonable searches and seizures.
The issue is not the abstract propriety of the police conduct, but the admissibility against petitioner
of the evidence uncovered by the search and seizure.

The exclusionary rule has its limitations. It cannot be invoked to exclude the products of legitimate
police investigative techniques. Proper adjudication of cases in which this rule is invoked demands
awareness that the harassment by certain elements of police of which minority groups, particularly
negroes, frequently complain, will not be stopped by the exclusion of any evidence from criminal
trial. But, a rigid and unthinking application of this rule may also exact a toll in efforts to prevent
crime. No judicial opinion can comprehend the variety of the street encounter, and we can only
judge the facts of the case before us.

1. At what point in this encounter does the fourth amendment become relevant? When did Officer
McFadden “seized” Terry, and when did he conduct a “search?”
McFadden “seized” petitioner and subjected him to a “search” when he took hold of him and
patted down the outer surfaces of his clothing. We must decide whether at that point, it was
reasonable for McFadden to interfere with petitioner’s personal security. And in determining
whether the seizure and search were “unreasonable,” we ask 1) whether the officer’s action was
justified at its inception, and 2) whether it was reasonably related in scope to the
circumstances which justified the interference in the first place.
2. To assess the reasonableness of McFadden’s conduct, it is necessary to first focus upon the
governmental interest which allegedly justifies official intrusion upon constitutionally protected
interests of the private citizen. In justifying the particular intrusion, the officer must point to
specific facts, which, taken together with rational inferences from those facts, reasonably
warrant that intrusion. Would the facts available to the officer at the moment of the seizure or the
search "warrant a man of reasonable caution in the belief" that the action taken was
appropriate?

One general governmental interest is that of effective crime prevention and detection. But the
crux of the case is not the propriety of McFadden’s taking steps to investigate petitioner’s
suspicious behavior, but whether there was justification for his invasion of Terry’s personal
security in the course of such investigation. We are now concerned with more than the interest of
investigating crime. There is the more immediate interest of the officer to take steps to assure
himself that the person with whom he is dealing is not armed with a weapon that could
unexpectedly and fatally be used against him. Many law enforcement officers are killed in the
line of duty. Thus, there is a need for law enforcement officers to protect themselves and other
prospective victims of violence in situations where they may lack probable cause for an arrest.

But we must still consider the nature and quality of the intrusion on individual rights. Petitioner
claims that it is unreasonable for an officer to investigate suspicious circumstances until there is
probable cause to make an arrest. But this argument fails to take account of traditional limits upon
the scope of searches and recognizes no distinction in purpose, character, and extent between a
search incident to an arrest and a limited search for weapons. A search incident to an arrest can
involve a relatively extensive exploration of the person since it is justified not only by the necessity
to protect the arresting officer from assault, but also on other grounds. But a search for weapons
without probable cause to arrest must be strictly circumscribed by the exigencies which justify its
initiation. It must thus be limited to what is necessary for discovery of weapons which might be
used to harm the officer or others nearby. It is less than a “full” search.

An arrest is a different kind of intrusion from a limited search for weapons, and the interests each
serves are different. Arrest is an initial stage of criminal prosecution. Search for weapons
constitutes a brief intrusion upon the sanctity of the person. A reasonable apprehension of danger
may arise even if the officer has not yet adequate information to justify arresting a person.

The officer need not be absolutely certain that the individual is armed. The issue is whether a
reasonably prudent man, in the circumstances, would be warranted in the belief that his
safety or that of others was in danger. In determining if an officer acted reasonably, weight must
be given not to his unparticularized suspicion or “hunch, but to the specific reasonable inferences
which he is entitled to draw from the facts in light of his experience.

3. Reasonableness of McFadden’s search and seizure.


From the facts and circumstances, a reasonably prudent man would have been warranted in
believing petitioner was armed, and thus was a threat to the officer’s safety while he was
investigating his suspicious behavior. The actions of Terry and Chilton showed that they were
contemplating a daylight robbery which, it is reasonable to assume, would likely involve
weapons. When McFadden approached the three men, he had observed enough to make it
reasonable to fear that they were armed.

4. Manner of search and seizure.


The sole justification of the search in the present situation is the protection of the officer and
others nearby; it must thus be confined in scope to an intrusion reasonably designed to
discover guns, knives, clubs, or other hidden instruments for the assault of the officer.

McFadden patted down the outer clothing of the three men. He did not place his hands in their
pockets or under the outer surface of their garments until he felt weapons, and he merely reached
for and removed the guns. He confined his search to what was minimally necessary to learn if the
men were armed.

5. Thus, the revolver was properly admitted in evidence. Each case of this sort will have to be
decided on its own facts. We merely hold that where a police officer observes unusual conduct
which leads him reasonably to conclude in light of his experience that criminal activity may
be afoot and that the persons with whom he is dealing may be armed and presently
dangerous, and when he identifies himself as policeman and makes reasonable inquiries, nothing
in the initial stages of the encounter dispels his reasonable fear for his or others’ safety, he is
entitled for his protection and of others as to conduct a carefully limited search of the
OUTER CLOTHING of such persons to discover weapons which might be used to assault him.

85. People v. Noel Tudtud, GR 144037, September 26, 2003, Tinga, J. (Warrantless Searches)
FACTS:
The Toril Polcie Station, Davao City, received a report from a civilian asset, Solier, about a certain
Noel Tudtud. Solier said that his neighbors have been complaining about Tudtud, who was
allegedly responsible for the proliferation of marijuana in their area. In reaction, officers conducted
surveillance in Solier’s neighborhood. They gathered information and learned that Tudtud was
involved in illegal drugs. According to his neighbors, Tudtud was engaged in selling marijuana.

On August 1, 1999, Solier informed the police that Tudtud headed to Cotabato and would be back
later that day with new stocks of marijuana. He described Tudtud as big-bodied, short, and usually
wore a hat. Officers wearing civilian clothes awaited Tudtud’s arrival at a corner of Saipon and
McArthur Highway.

Later that evening, two men disembarked from a bus and helped each other carry a carton marked
“King Flakes.” One of the men fit Tudtud’s description. He also had a plastic bag. The officers
approached the suspects and identified themselves as police officers. Tudtud denied that he was
carrying drugs. PO1 Desierto asked him if he could see the contents of the box. Tudtud obliged,
saying “it was alright.” Tudtud opened the box himself as his companion looked on. The box
yielded dried fish, under which were two wrapped bundles. PO1 Desierto asked Tudtud to unwrap
the packages. They contained marijuana leaves. The officers thus arrested Tudtud and his
companion, informed them of their rights, and brought them to the police station. They did not
resist.
Tudtud and his companion Bulong were charged for illegal possession of prohibited drugs. They
cried frame-up. The RTC convicted them. On appeal, they claim that the marijuana leaves were
seized in violation of their right against unreasonable searches and seizures.

ISSUE:

HELD:
The rule is that a search and seizure must be carried out through judicial warrant, else it is
“unreasonable” within the meaning of Article III, Section 2. But this covers only “unreasonable”
searches. There are those deemed not “unreasonable” such as warrantless search incidental to
lawful arrest, search of evidence in “plain view,” etc.

The RTC justified the search as one incidental to a lawful arrest. Rule 126, Sec.12 sanctions this:
SEC. 12. Search incident to lawful arrest. — A person lawfully arrested may be searched
for dangerous weapons or anything which may be used as proof of the commission of an
offense, without a search warrant.

Section 5(a) of Rule 113 allows warrantless arrests.

The search in this case preceded the arrest. The arrest must precede the search. The process
cannot be reversed. But a search can precede the arrest if the officer has probable cause to make
the arrest at the outset of the search. The question is thus whether the officers had probable cause
to arrest appellants. Probable cause has been defined as “an actual belief or reasonable grounds of
suspicion. The grounds of suspicion are reasonable when, in the absence of actual belief of the
arresting officers, the suspicion that the person to be arrested is probably guilty of committing the
offense, is based on actual facts, i.e., supported by circumstances sufficiently strong in themselves
to create the probable cause of guilt of the person to be arrested.”

“Reliable information” alone is not sufficient to justify a warrantless arrest under Section 5(a).
there must be some overt act to indicate that he has committed, is actually committing, or is
attempting to commit an offense. The arresting officer must have personal knowledge of this fact.
The offense must be committed in his presence or within his view.

In this case, appellants were not performing any overt act or acting in a suspicious manner that
would hint that a crime has been, was being, or was about to be committed. Although Tudtud
appeared “afraid and perspiring, pale, and trembling,” this was only after he was asked to open the
said box.

The knowledge of the arresting officers that Tudtud was in possession of marijuana was not
personal, having learned it only from their informant Solier. Solier testified that he obtained his
information only from his neighbors and friends of Tudtud. His information is hearsay. He did
not even elaborate how his neighbors or Tudtud’s friends acquired their information.

With such a dubious informant, the police perhaps felt it necessary to conduct their own
“surveillance.” This surveillance, it turns out, did not consist of staking out Tudtud to catch him in
the act of plying his illegal trade, but only of a mere “gathering of information from the assets
there.” Neither were the arresting officers impelled by any urgency that would allow them to do
away with the requisite warrant. The officers received Solier’s information around 9:00am. Tudtud
was expected to arrive around 6:00pm. There was sufficient time to procure a warrant.

Lastly, there is an effective waiver of rights against unreasonable searches and seizures if the
following are present: 1) the right exists, 2) the person had knowledge, actual or constructive, of
the existence of such right, and 3) the person had an actual intention to relinquish the right.

The prosecution failed to establish the 2nd and 3rd requisites. When the police introduced
themselves and requested to see the contents of the box, Tudtud said “it was alright.” He did not
resist and opened the box himself. But fundamental law and jurisprudence requires more than the
presence of these circumstances to constitute a valid waiver. Courts indulge every reasonable
presumption against waiver of fundamental constitutional rights. The fact that a person failed
to object to a search is not permission thereto. Peaceful submission is not consent but is merely a
demonstration of regard for the supremacy of the law.

Since warrantless search is in derogation of a constitutional right, officers cannot invoke regularity
in the performance of official functions and shift the burden of proof to the accused that the search
was unconsented.

Appellants were acquitted as the marijuana were held inadmissible in evidence.

**People v. Valeriano Amestuzo, GR 104383, July 12, 2001, Kapunan, J. (Custodial


Investigations)
FACTS:
8 armed men wearing masks entered the house of Perlita Lacsamana and robbed it of valuables
worth P728k. 2 members of the gang raped Maria Catanyag and Estrella Rolago, niece and
employee of Perlita. Amestuzo and Bagas, among others, was charged with robbery in band with
double rape.

The RTC convicted Amestuzo and Bagas. He appealed to the SC, alleging 1) deprivation of his
constitutional right to be represented by counsel during his identification, 2) the trial court’s error
in giving due weight to the open court identification of him which was based on a suggestive and
irregular out-of-court identification, and 3) the trial court’s improper rejection of his alibi. He
narrates the circumstances of his arrest.

Four days after the alleged crime, a group of policemen, together with accused Ampatin who was
a suspect, went to the factory in Pasay where Bagas was working. They were looking for a certain
“Mario” and searched the first and second floors of the building. Failing to find Mario, the police
hit Ampatin and uttered, “Niloloko lang yata tayo ng taong ito” and “Magturo ka ng kahit sino.”
Thus, Ampatin pointed to Bagas as he was the first person Ampatin chanced to look upon. Bagas
was arrested. Ampatin, aboard the police vehicle, told Bagas that he (Ampatin) committed an error
in pointing him out to police. They were placed under detention.

When the complainants arrived, Bagas was brought out, instructed to turn to the left and right and
to talk. Complainant Lacsamana asked him if he knew accused Amestuzo and Viñas, he said no.
the police told the complainants that Bagas was one of the suspects, which incited complainants
to an emotional frenzy, kicking and hitting him.

ISSUE:
Whether the identification of Bagas by complainants Lacsamana et al. who were victims of robbery
in band with double rape after the police took Bagas out of his detention cell and presented him
alone to complainants who asked him if he knew the other accused, to which he replied that he did
not, but where the complainants were induced into an emotional frenzy and started kicking and
hitting him after the police told complainants that he was one of the suspects may be admissible in
evidence.
HELD: NO.
1. Right to counsel.
Bagas claims that he was deprived of his right to counsel during his investigation. His single
presentation to complainants for identification without counsel is a violation of his right to counsel.
His identification was a critical stage of prosecution at which he was entitled to counsel.

The guarantees of Section 12(1), Art.III of the Constition, so-called Miranda rights, may be
invoked only by a person while he is under custodial investigation. This starts when the police
investigation is no longer a general inquiry into an unsolved crime but has begun to focus on
a particular suspect taken into custody by the police who starts the interrogation of the
person to elicit incriminating statements. Police line-up is not part of custodial investigation.
Hence, the right to counsel cannot be invoked at this stage. This is because during a police line-
up, the process has not yet shifted from the investigatory to the accusatory and it is usually the
witness or complainant who is interrogated and who gives a statement in the course of the line-up.

There was also no showing that during his identification, the police sought to elicit any admission
or confession from him. Alleged infringement of the rights of accused under custodial
investigation is relevant only in cases in which an extrajudicial admission or confession
extracted from the accused becomes the basis of his conviction.

2. Identification irregularity
Bagas claims that the manner he was presented to complainants for identification was irregular as
he was not placed in a police line-up but made to stand before complainants alone.

There is no law requiring a police line-up as essential to proper identification. The fact that he was
brought out of the detention cell alone before the complainants does not detract from the validity
of the identification process.

But the out-of-court identification of Bagas was seriously flawed as to preclude its admissibility.
In admissibility and reliability of out-of-court identifications, we apply the totality of
circumstances test which lists the following factors:
(1) the witness' opportunity to view the criminal at the time of the crime; (2) the witness'
degree of attention at that time; (3) the accuracy of any prior description given by the
witness; (4) the level of certainty demonstrated by the witness at the identification; (5)
the length of time between the crime and the identification; and (6) the suggestiveness
of the identification process.
The out-of-court identification of Bagas by complainants is improperly suggestive. Even before
complainants had opportunity to view Bagas’ face, the police announced that he was one of the
suspects and he was pointed to by Ampatin. The identification was practically suggested by the
police. The fact that this information came to the knowledge of complainants prior to their
identification based on their own recall of the incident detracts from the spontaneity of their
subsequent identification and thus its objectivity.

3. Defense of ablibi.
The trial court erroneously rejected Bagas’ alibi. Bagas clearly and positively testified that at the
time of the crime, he was working as a shell cutted in a factory in Pasay. Four days later, he was
arrested when Ampatin randomly pointed him out to police. This testimony of Bagas was
materially corroborated by two of his co-employees who were with him on the night of the crime
and his employer. The defense of alibi assumes strength when it is amply corroborated by a
credible witness. To be given weight, accused must prove not only that he was somewhere else
during the crime, but it was physically impossible for him to be present at the crime or its vicinity
during commission.

As impartial credible witnesses, Bagas’ co-employees and employer cannot be doubted without
showing of undue bias or prejudice. It was also testified that Bagas worked in the factory until
10pm and went to sleep after while the crime was committed 930pm. There was only one door in
the factory as the only means of entrance and exit and this was locked by the employer after 10pm.
There was no possible exit through the windows of the building. The crime took place in Kalookan
city, while Bagsa’ place of work was in Pasay.

Ampatin’s declaration in court that he does not know Bagas and merely pointed to him out of fear
of the police is corroborated. Thus, Bagas was acquitted.

**People v. Anthony Escordial, GR 138934-35, January 16, 2002, Mendoza, J. (Custodial


Investigations)
FACTS:
Erma was awakened by the presence of a man who had his head covered with a t-shirt and carried
a knife. He warned Erma not to shout or he would kill her. He got the money of Erma and Michelle.

The man gave a t-shirt to Erma to blindfold Teresa and another to Michelle to blindfold Erma. He
blindfolded Michelle himself and began touching her in different parts of her body. He raped
Michelle.

PO3 Tancinco, one of the policemen who responded to the report after the crime, questioned
certain people about Escordial and found out that he was a helper of Hinolan, owner of Coffee
Break Corner. Hinolan said that Escordial went home on December 27, 1996, day of the crime, to
Brgy. Miranda, Pontevedra. Tancinco found Escordial at the basketball court and “invited” him to
go to the police station for questioning.

Michelle remained at the police station. When Escordial was brought there, he saw Michelle and
blushed. She recognized him as the man who robbed and raped her.
Erma, Teresa, Joniega, and Esmeral were asked to identify Escordial from four people inside a jail
cell. They picked Escordial.

As to the circumstances of arrest, police, led by PO3 Tancinco, went to Pontevedra police to ask
for help in locating Escordial, wanted for robbery with rape. Although Tancinco and company had
a mission order, they did not have a warrant of arrest. They found Escordial at the basketball court
watching a game. After informing him that he was a suspect, the group invited him to go with them
to the police headquarters.

Brgy. Captain Dojilo testified that he was at the police station and followed Tancinco and Escordial
to the investigating room. Michelle said that she could identify the attacker if she could see a lump
on his back. Michelle, after accused took off his shirt, said she was not sure because the attacker
was wearing a mask. Tancinco requested to be allowed to bring accused to Bacolod as they still
had some witnesses there who could identify the suspect. Accused was allowed to go with them,
but the Pontevedra police asked the Bacolod police Tancinco not to harm him.

In Bacolod, two of the complainants arrived and the police asked them to identify him, but they
just kept looking and they even asked the police if he was the suspect.

The RTC convicted Escordial. Hence this appeal.


ISSUE:
Whether Escordial may be convicted of robbery with rape based on the testimonies of witnesses
who have identified him in a show-up and line-up but claim 1) that at the time of rape, she was
blindfolded and could only feel keloids on the back of the assailant but Escordial has no such
keloid and 2) that she saw the appellant at the time of rape through her blindfold but without having
immediately reported his identity immediately and doing so only a week after the crime.
HELD: NO.
1. Warrantless arrest.
Escordial questions the legality of his arrest without warrant. The cases at bar do not fall under
Rule 113, Section 5(a) or (c). Escordial was watching a game when arrested in Brgy. Miranda. He
was not committing or attempting to commit a crime when arrested nor was he an escaped
prisoner.

Under Sec.5(b), “personal knowledge” must be based upon probable cause, which means “an
actual belief or reasonable grounds of suspicion.” The grounds of suspicion are reasonable
when, in the absence of actual belief of the arresting officers, the suspicion that the person to be
arrested is probably guilty is based on actual facts.

The crime took place on December 27, 1996, but Escordial was arrested only on January 3, 1997.
As the officers were not present when the crime was committed, they could not have “personal
knowledge.” The officers had no reason for not securing a warrant.

But Escordial pleaded not guilty to the crimes without questioning his arrest. Thus, he waived
objection to the legality of his arrest. Any defect in the arrest is deemed cured when he voluntarily
submitted to the jurisdiction of the court, for the legality of an arrest only affects jurisdiction of
the court over the person of the accused.

2. Escordial invokes Article II, Sec. 12(1): [a]ny person under investigation for the commission of
an offense shall have the right to be informed of his right to remain silent and to have competent
and independent counsel preferably of his own choice. If the person cannot afford the services of
counsel, he must be provided with one. These rights cannot be waived except in writing and in the
presence of counsel." He claims that he was subjected to custodial interrogation without being
informed of his right to remain silent and have independent counsel.

While it cannot be denied that he was deprived of his right to be informed of his rights to remain
silent and to have competent and independent counsel, he has not shown that, as a result of his
custodial interrogation, the police obtained any statement from him, whether inculpatory or
exculpatory, which was used in evidence against him. No uncounseled statement was obtained
from him which should have been excluded as evidence against him.

3. Right to counsel.
An accused is not entitled to assistance of counsel in a police line-up since it is not part of custodial
inquest. But Escordial had already been under custodial investigation when the out-of-court
identifications were conducted by the police.

An out-of-court identification can be made in various ways.in a show-up, the accused alone is
brought face to face with the witness for identification. In a police line-up, the suspect is identified
by a witness from a group of persons. During custodial investigation, these identifications are
“critical confrontations of the accused by the prosecution” which necessitate presence of
counsel for the accused since the results of these pre-trial proceedings might settle the accused’s
fate and reduce trial to a mere formality. Any identification of an uncounseled accused made in a
police line-up or show-up after the start of custodial investigation is inadmissible in evidence.

Escordial was identified by Michelle in a show-up on January 3 1997 and by Erma, Teresa,
Joniega, and Esmeralda in a police line-up on various dates. Since accused had no counsel, these
out-of-court identifications are inadmissible for being the direct result of the illegal line-up.

But the inadmissibility of these out-of-court identifications does not render in-court identification
of accused inadmissible.

4. Credibility of prosecution witness.


Joniega and Esmeralda pointed to Escordial as the man they saw on the night of December 27 and
the person they identified inside a jail cell. Erma testified that she saw through her blindfold
Escordial raping Michelle and identified him in court.

The test is whether the prosecution was able to establish by clear and convincing evidence that
the in-court identifications were based upon observations of the suspect other than the line
up identification. This test considers various factors like prior opportunity to observe the alleged
criminal act, existence of any pre-line-up description and defendant’s actual description, any
identification prior to lineup of another person, identification by picture prior to lineup, failure to
identify the defendant on a prior occasion, and lapse of time between the alleged act and lineup
identification.

A. A show-up, as undertaken herein by police in identifying Escordial by Michelle, has been held
to be an underhanded mode of identification for “being pointedly suggestive, generating
confidence where there was none, activating visual imagination, and subverting their reliability as
an eyewitness.” Michelle knew she was going to identify a suspect. Upon seeing Escordial escorted
by police, she knew that he was the suspect she was supposed to identify. When Escordial was
shown to her, there was thus no doubt of what was expected of her. By arrainging a lineup, police
evidence their belief that they have caught the criminal. Witnesses, realizing this, will feel foolish
if they cannot identify anyone and may thus choose someone despite residual uncertainty.

The failure of Michelle to see the face of the assailant and the apparent suggestiveness of the show-
up places in doubt her credibility. The possibility that her identification of Escordial was merely
planted in her mind cannot be disregarded. Michelle’s affidavit also indicated that she felt keloids
on the back of the assailant but testified that she did not see keloids on Escordial although she said
that his skin was rough.

B. Erma testified that she saw through her blindfold the assailant raping Michelle and identified
Escordial in open court. But the police blotter dated December 28, 1996 referred to an “unknown
suspect.” Also, when the police went to Pontevedra for Escordial, it was Michelle, who admitted
that she did not see the assailant’s face, and not Erma, who said she recognized Escordial, that was
taken along. Why did they take Michelle instead of Erma?

The affidavit of Erma was prepared on January 4, 1997, a day after the arrest of Escordial. This
delay belies her claim that she saw the assailant through her blindfold, for the normal reaction of
one who actually witnessed a crime and recognized the offender is to reveal it to authorities at the
earliest opportunity.

C. Escordial’s testimony that he was in Brgy. Miranda is corroborated by 3 people. Considering


the uncertainties of the testimonies of the prosecution witnesses, the defense of alibi by Escordial
deserves credence.

86. People v. Idel Aminnudin, GR 74869, July 06, 1988, Cruz, J. (Warrantless Searches)
FACTS:
PC officers received a tip from one of their informers that Aminnudin was on board a vessel bound
for Iloilo carrying marijuana. He was identified by name. Two days later, the officers waited for
him.

Idel Aminnudin was arrested shortly after disembarking from the MV Wilcon in Iloilo City. The
officers approached him after the informer pointed to him. The officers who were waiting for him
simply accosted him, inspected his bag, and finding marijuana, took him to headquarters for
investigation. An information for violation of the Dangerous Drugs Act was filed against him.

Aminnudin disclaimed the marijuana, claiming that all he had was clothing. He alleged that he was
arbitrarily arrested and immediately handcuffed and his bag confiscated without search warrant.
At headquarters, he was manhandled to force him to admit he was carrying marijuana. He did not
know what marijuana looked like and his business was selling watches and cigarettes.

The RTC convicted him as he was only carrying two watches at the time even as he claimed to be
selling watches. The watches were not discovered when he was bodily searched nor damaged as a
result of his manhandling. Aminnudin said he sold one and gave the other watch away to a friend
whose full name he did not even know.

ISSUE:
Whether Aminnudin’s arrest and the search of his bag, conducted by officers while he was
disembarking from the MV Wilcon vessel after the informant, who informed the police two days
before that Aminnudin will arrive carrying marijuana, pointed to him, was valid.
HELD: NO.
The officers admit that they did not have a warrant when they arrested Aminnudin and seized his
bag. Their only justification was the tip they had earlier received from a regular and reliable
informer.

Aminnudin was not caught in flagrante nor was a crime about to be or has just been committed
to justify the warrantless arrest allowed under Rule 113 of the RoC. Even expediency could not be
invoked to dispense with the obtention of the warrant. Vessels and aircrafts are subject to
warrantless searches and seizures only for violation of customs law as the vehicles may be quickly
moved out of the locality or jurisdiction before the warrant can be secured.

There is no such urgency in this case. The officers had at least 2 days to obtain a warrant to arrest
and search Aminnudin. His name was known. The vehicle was identified. The date of arrival was
certain. And the information they received could have persuaded a judge that there was probable
cause to justify issuance of warrant.

Aminnudin was descending the gangplank of the MV Wilcon and there was no outward indication
calling for his arrest. He was like any other passenger innocently disembarking from the vessel.
It was only when the informer pointed to him that he suddenly became suspect. The identification
of the informer was the probable cause as determined by the officers, not a judge, that authorized
them to arrest Aminnudin.

Thus, the marijuana is inadmissible for being seized illegally. It is the fruit of the poisonous tree.
Hence, Aminnudin was acquitted, there being no more evidence.

87. People v. Rogelio Mengote, GR 87059, June 22, 1992, Cruz, J. (Warrantless Searches)
FACTS:
The Western Police District received a telephone call from an informer that there were 3
suspicious-looking persons in Juan Luna in Tondo. A surveillance team of plainclothesmen was
dispatched. They saw two men looking from side to side, one of whom was holding his
abdomen. They approached the persons and identified themselves as policemen. The two tried to
run away but was unable to escape. They were searched. One of them, Mengote, was found with
a revolver with six live bullets. They were turned over to police headquarters for investigation.
Mengote was charged with possession of firearm without license. One prosecution witness testified
that the weapon was among the articles stolen from him during the robbery in his house in
Malabon. Mengote claimed that it was planted on him at the time of arrest.

The RTC convicted Mengote for violation of PD 1866. In his appeal, he claims that the revolver
should be inadmissible because of its illegal seizure, no warrant therefor having been previously
obtained. It was not seized incidental to a lawful arrest as the arrest was unlawful also as it was
without warrant.

ISSUE:
Whether the search and arrest of Mengote after he alighted from a jeepney, based on a call by an
informant of “suspicious-looking persons” about to attempt robbery and the officer’s suspicion
aroused by Mengote’s “looking from side to side” and “holding his abdomen,” was valid.
HELD: NO.
It is claimed that the arrest and search of Mengote were lawful under Rule 113, Section 5 of the
RoC. Section 5(c) is inapplicable. Section 5(a) requires that the person be arrested 1) after he has
committed or while he is actually committing or is at least attempting to commit an offense, 2) in
the presence of the arresting officer.

These requirements have not been established. Mengote was merely “looking from side to side”
and “holding his abdomen.” There was no offense that had just been committed or was being
actually committed or being attempted by Mengote in their presence.

These are not sinister acts and the setting made them less so if at all. It might have been different
if Mengote was arrested at an ungodly hour in a place where he had no reason to be. But he was
arrested 11:30 in the morning in a crowded street shortly after alighting from a jeep. He was not
skulking in the shadows but walking in the clear light of day. There was nothing clandestine about
his being on that street at that busy hour.

There could have been many innocent reasons why his eyes were darting from side to side and for
him to hold his abdomen. If this excited suspicion in the officers, it was not shown what the
suspicion was all about. The officers were dispatched only because of a call from an informer
that there were “suspicious-looking” persons in that vicinity about to commit robbery. From the
acts, it could not be inferred that an offense had just been, was actually, or being attempted to be
committed.
Section 5(b) is also not applicable because its requirements have not been satisfied. It was not
shown that an offense had in fact just been committed and that the officers had personal
knowledge of facts indicating that Mengote committed it. All they had was hearsay information
from the caller.

They did not know then what offense, if at all, had been committed. It was only later after
Danganan appeared that they learned of the robbery in his house and of Mengote’s supposed
involvedment therein. The policemen discovered the illegal possession only after he had been
searched and the investigation revealed that he was not its owner nor was he licensed to
possess it. Before these, the officers did not know Mengote nor had any suspicion that he was
unlawfully carrying a firearm or that he was involved in the robbery.
In section 5(b), there must in fact been a crime committed first. It is not enough to suspect that a
crime may have been committed.

The revolver was held inadmissible. Thus, without evidence, Mengote was acquitted.

88. People v. Mikael Malmstedt, GR 91107, June 19, 1991, Padilla, J. (Warrantless Searches)
FACTS:
Malmstedt, Swedish, entered the PH as a tourist. He left for Baguio City. Upon his arrival, he took
a bus to Sagada and stayed for two days. He went to Nangonogan bus stop in Sagada to catch the
first trip to Baguio on 7am, May 11, 1989. From Baguio, he planned to take a late afternoon trip
to Angeles, then proceed to Manila to catch his flight out of the country scheduled on May 13,
1989.

At 8am on May 11, Captain Vasco, commanding officer of the First Regional Command
(NARCOM), ordered his men to set up a temporary checkpoint to check all vehicles coming
from the Cordillera region. This order was prompted by persistent reports that vehicles coming
from Sagada were transporting marijuana and drugs. Information was also received by Vasco that
that same morning, a Caucasian coming from Sagada has in his possession prohibited drugs.

At 1:30pm, the bus Malmstedt was riding was stopped. NARCOM members inspected the bus
from the front going towards the rear. Malmstedt was the sole foreigner riding the bus at the rear.
During the inspection, CIC Galutan noticed a bulge on Malmstedt’s waist. Suspecting it to be a
gun, the officer asked for Malmstedt’s passport and identification papers. When he failed to
comply, the officer required him to bring out whatever was bulging from his waist. The bulging
object turned out to be hashish, a derivative of marijuana.

He invited out of the bus for questioning. He got two travelling bags with teddy bears, the officers
found hashish in the teddy bears. Thus, an information was filed against him for violation of the
Dangerous Drugs Act.

Malmstedt claims illegal search and that the hashish was planted by the NARCOM officers. The
RTC convicted Malmstedt. Hence this appeal.

ISSUE:
Whether the search of Malmstedt after the NARCOM officer’s suspicion was aroused by a bulge
on Malmstedt’s waist, which bulge turned out to be a pouch containing hashish, a derivative of
marijuana, was a valid search.
HELD: YES.
Where the search is made pursuant to a lawful arrest, there is no need to obtain a search warrant
(Section 5, Rule 113 of RoC). Malmstedt was searched and arrested while transporting
prohibited drugs. A crime was actually being committed and he was caught in flagrante delicto.
Thus, this falls under Section 5(a).

Under the circumstances, there was sufficient probable cause for the officers to believe that
Malmstedt was committing a crime. Probable cause is defined as as such facts and circumstances
which could lead a reasonable, discreet and prudent man to believe that an offense has been
committed, and that the objects sought in connection with the offense are in the place sought to be
searched. The required probable cause is not determined by a fixed formula but is resolved
according to the facts of each case.

Aside from the persistent reports received by the NARCOM that vehicles from Sagada were
transporting drugs, their Commanding Officer also received information that a Caucasian coming
from Sagada on that particular day had drugs in his possession. When NARCOM received the
information a few hours before the arrest of Malmstedt, there was no time to obtain a search
warrant.

At first, the NARCOM officers merely conducted a routine check of the bus where Malmstedt was
riding and no extensive search was made. It was only when the officer noticed a bugle on
accused’s waist that he was required to present his passport. His failure to present identification
papers only aroused the suspicion of the officer that the accused was trying to hide his identity.
For it is regular norm for an innocent man, with nothing to hide, to readily present his identification
papers when required to do so.

The information and this suspicious failure, taken together as a whole, led the NARCOM officers
to reasonably believe that accused was trying to hide something illegal from the authorities.
These circumstances gave rise to probable cause which justified the warrantless search.

89. Rodolfo Espano v. CA, GR 120431, April 01, 1998, Romero, J. (Warrantless Searches;
Search incidental to a lawful arrest- limited to immediate surroundings, body search for
dangerous weapons and anything which may be proof of commission of an offense)
FACTS:
Police testified that they went to Zamora and Pandacan Streets, Manila, to confirm reports of drug
pushing in the area. They saw Espano selling “something” to another person. After the buyer left,
they approached Espano, identified themselves as policemen, and frisked him. They found 2 bags
of marijuana. When asked if he had more, he said that there were more in his house. The police
found 10 more bags of marijuana in his house. He was charged with possession of prohibited drugs.

As defense, Espano claims that he was sleeping in his house when awakened and handcuffed by
the police. The RTC convicted him and found the prosecution more credible. The CA affirmed
this decision. Hence this petition.

Espano claims that 1) the pieces of evidence were inadmissible, 2) his right to be presumed
innocent should be superior over the presumption of regularity (of performance of duty*), 3) he
was denied the right to confrontation and compulsory process, and 4) his conviction was based on
irrelevant and not properly identified evidence.

ISSUE:
Whether the marijuana 1) found after the officers saw Espano sell “something” to a buyer and who
thereafter frisked him and 2) those found in Espano’s house after he told the officers that he had
more marijuana in his house, without a warrant, are admissible in evidence.
HELD:
The trial court findings on witness credibility deserve a high degree of respect. Espano failed to
show that Pat. Pagilagan, in testifying against him, was motivated by reasons other than his duty
to curb drug abuse and had intent to falsely impute to him such crime. Doctrine of presumption
of regularity in the performance of official duty: “In the absence of proof of any intent on the
part of the police authorities to falsely impute” the crime, “the presumption of regularity in the
performance of official duty must prevail over the self-serving and uncorroborated claim of
appellant…”

Espano’s defense of alibi cannot be considered. Alibi is the weakest of all defenses, and the
accused must prove that he was not at the scene of the crime at the time of its commission and that
it was physically impossible for him to be there. No clear and convincing evidence was presented
to prove this defense.

Espano’s claim that the failure to present the alleged informant warrants his acquittal. But the
failure to present the informant is of no moment since he is not even the best witness to establish
the fact that a buy-bust operation had been conducted. Pat. Pagilagan, one of the officers who
apprehended Espano, testified on the incident and identified him as the one they caught in
possession of prohibited drugs. Thus, the prosecution proved that Espano committed the crime.

1. YES.
Also, Espano’s arrest falls under Rule 113, Section 5(a) of the RoC. He was caught in flagranti as
a result of a buy-bust operation by the police on the basis of information received regarding the
illegal trade of drugs within that area. The officers saw Espano handing something to an alleged
buyer and he was thus searched. Marijuana was found. The arrest was thus lawful and the two bags
of marijuana are admissible being fruits of the crime.

2. NO.
But the 10 bags of marijuana found in Espano’s residence are inadmissible in evidence.

An exception to the rule against unreasonable searches is warrantless search incidental to a lawful
arrest for dangerous weapons or anything which may be used as proof of the commission of
an offense. It may extend beyond the person of the one arrested to include the premises or
surroundings under his immediate control. It is limited to a body search and to that point
within reach or control of the person arrested. The marijuana were seized in Espano’s house.
They do not fall under this exception. The house was beyond the reach and control of Espano.

90. Ongcoma Homar v. People, GR 182534, September 02, 2015, Brion, J. (Warrantless
Search; There must be an intention to arrest to be considered an arrest; Valid warrantless
arrest must precede search incidental to a lawful arrest; No arrest in this case since no intent
to arrest)
FACTS:
PO1 Tan, lone witness of the prosecution, testified that he was ordered with civilian agent Tangcoy
to go to Roxas Boulevard. While proceeding to the area, they saw Homar crossing a no jaywalking
portion of Roxas Boulevard. They immediately accosted him and told him to cross the pedestrian
crossing area. Homar picked up something from the ground, prompting Tangcoy to frisk him
resulting in the recovery of a knife. Thereafter, Tangcoy conducted a thorough search of Homar’s
body and found shabu.

The RTC convicted him, ruling that PO1 Tan and Tangcoy are presumed to have performed their
duties regularly. It believed PO1 Tan’s testimony. The CA affirmed the RTC ruling, stating that
the arrest was incidental to a lawful arrest, being caught in flagranti possessing shabu.

ISSUE:
Whether the search of Homar’s person, done by civilian agent Tangcoy who was with PO1 Tan
when they saw Homar allegedly jaywalking and accosted him then pointed him to the right place
to cross, but when Homar picked something up from the ground he was frisked which yielded a
kitchen knife and thereafter extensively searched which yielded shabu, was valid.
HELD: NO.
1. No lawful warrantless arrest preceded the search.
A valid warrantless arrest must PRECEDE the search. The process cannot be reversed. Section 5,
Rule 113 provides the only occasions when a person may be lawfully arrested without warrant.

To be a valid in flagrante delicto arrest, 1) the person must execute an overt act indicating that he
has just committed, is actually committing, or is attempting to commit a crime and 2) such overt
act is done in the presence of or within the view of the arresting officer. The prosecution must
prove the legality of the warrantless arrest for without this, the warrantless search is a violation of
his constitutional right.

Aside from the bare testimony of PO1 Tan, there was no other proof that the requirements of an in
flangrante arrest were met. The prosecution, particularly, failed to prove that petitioner was
committing a crime. It failed to specifically identify the area where Homar allegedly crossed.
Tan merely stated that Homar was crossing a no jaywalking area, failing to prove that the portion
of Roxas Boulevard where Homar crossed was indeed such area. Homar was also not charged of
jaywalking. We clarify that the filing of a criminal charge is not a condition precedent to prove a
valid warrantless arrest as the prosecution is not relieved of the burden to prove the valid
warrantless arrest even if such were filed.

The presumption of regularity of performance of official duty cannot overcome the presumption
of innocence or be proof of guilt beyond reasonable doubt.

Also, there is doubt as to whether Tan and Tangcoy INTENDED TO ARREST Homar for
jaywalking. Arrest is the taking of a person into custody in order that he or she may be bound to
answer for the commission of an offense. It is effected by actual restraint of the person to be
arrested or by that person’s voluntary submission to the custody of the one making the arrest.
Neither the application of actual force or physical restraint nor a formal declaration of arrest is
required. It is enough that there be an 1) intention of one party to arrest the other, and that
there be an 2) intent of the other to submit, under the belief and impression that submission
is necessary.

When Tan and Tangcoy saw Homar jaywalking, they did not arrest him but accosted him and
pointed him to the right place for crossing. They did not intend to arrest him. In Luz v. People, it
was held that since the officer had no intent to arrest, the search was unlawful even if the accused
was caught in flagrante for violating an ordinance and could have been lawfully arrested.

2. The waiver of an illegal warrantless arrest does not mean a waiver of the inadmissibility of
evidence seized during an illegal warrantless arrest. While Homar did not timely object to the
irregularity of his arrest before arraignment and is thus deemed submitted to the jurisdiction of the
trial court, this only affects the jurisdiction of the court over his person.

91. Rodel Luz v. People, GR 197788, February 29, 2012, Sereno, J. (Warrantless Search)
FACTS:
PO2 Alteza, assigned as traffic enforcer, testified that he saw Luz driving a motorcycle without a
helmet. He flagged down Luz for violating a municipal ordinance requiring motorcycle drivers to
wear helmets while driving said motor vehicle. He invited Luz to come inside their sub-station
since the place where he flagged him down was almost in front of the sub-station. The officers
noticed that while they were issuing a citation ticket, the accused was uneasy and kept getting
something from his jacket. Alteza was alerted, so he told Luz to take out the contents of his jacket
as it may have a weapon. Luz obliged and took out the contents of his jacket, one of which was a
nickel-like tin container 2-3 inches in size. Alteza asked Luz to open it. He noticed a cartoon cover
and something beneath it. Upon his instruction, Luz spilled out the contents of the container which
turned out to be 2 empty sachets and 2 sachets of shabu. He was charged of possession of
dangerous drugs.

The RTC convicted Luz, finding the prosecution evidence sufficient to show that he had been
lawfully arrested for a traffic violation and then subjected to a valid search. The CA affirmed.
Hence this petition for certiorari.

ISSUE:
Whether Luz, who was driving a motorcycle without a helmet in violation of an ordinance and
thus flagged down to be issued a citation and, for convenience, invited into the police station, was
actually arrested and thus, the drugs seized from him may be admissible in evidence.
HELD: NO.
1. There was no valid arrest.
Arrest is the taking of a person into custody so that he may be bound to answer for the commission
of an offense. It is effected by actual restraint of the person to be arrested or by that person’s
voluntary submission to the custody of the one making the arrest. Neither the application of actual
force, manual touching of the body, or physical restraint, nor a formal declaration of arrest, is
required. It is enough that there be an intention on the part of one party to arrest the other and
there be an intent of the other to submit under the belief and impression that submission is
necessary.

Under the Land Transportation and Traffic Code, RA 4136, the procedure for dealing with a traffic
violation is not arrest, but confiscation of the driver’s license of the latter. This is also provided in
the PNP Operations Manual.

When Luz was waiting for PO3 Alteza to write his citation ticket, he could not be said to be
“under arrest.” There was no intention of Alteza to arrest him, deprive him of his liberty, or take
him into custody. Before the issuance of the ticket, the period during which Luz was at the station
may be characterized as mere waiting time.

The US SC held that a roadside questioning of a motorist detained pursuant to a routine traffic stop
cannot be considered custodial interrogation nor formal arrest due to the nature of the questioning,
expectations of the motorist and officer, and the length of time the procedure is conducted:
1) Detention of a motorist pursuant to a traffic stop is presumptively temporary and brief. A
motorist’s expectations when flagged is that he will be obliged to spend a short period of time
answering questions and waiting for the officer to check his license and registration etc.
2) The circumstances associated with the typical traffic stop are not such that the motorist feels
completely at the mercy of the police.
In both these respects, the usual traffic stop is more analogous to a “Terry stop” (Terry v. Ohio)
than to a formal arrest.

It ruled that since the motorist was only subjected to modest questions while at the scene of the
traffic stop, he was not under custody, and neither can this be treated as a formal arrest. Similarly,
Luz cannot be considered “under arrest.”

While a city ordinance was violated, this is penalized by fine only. Under RoC, a warrant of arrest
need not be issued if the charge is for an offense penalized by fine only. It may be stated as
corollary that neither can a warrantless arrest be made for such an offense. But this does not imply
that there can be no arrest for a traffic violation. But in this case, the officer’s issuance or intent to
issue a traffic citation negates the possibility of an arrest for the same violation.

2. Even if Luz was deemed arrested upon being flagged down, the requirements for a valid arrest
were not complied with.
It is the duty of the arresting officer to inform the arrested person of the reason for the arrest and
must show a warrant of arrest if any. Persons shall be informed of their constitutional rights to
remain silent and to counsel, and that any statement they make could be used against them.
These were complied with only after Luz was arrested for illegal possession of drugs. If Luz is
deemed arrested already when flagged down, there would have been no need to arrest him a second
time when the drugs were discovered.

3. There being no valid arrest, the warrantless search that resulted from it was illegal.
None of the instances for a valid warrantless search (incidental to a lawful arrest, plain view, etc.)
apply to this case.

The evidence seized was not in plain view as it was concealed inside a metal container. It was not
immediately apparent. There was no consented warrantless search. The consent must be
unequivocal, specific, intelligently given, and uncontaminated by duress or coercion. The alleged
accession of Luz to Alteza’s instruction does not suffice to prove valid and intelligent consent.

Whether consent to the search was in fact voluntary is a question of fact to be determined from
the totality of all the circumstances. Relevant to this determination are the following
characteristics of the person giving consent and the environment in which consent is given: (1) the
age of the defendant; (2) whether the defendant was in a public or a secluded location; (3)
whether the defendant objected to the search or passively looked on; (4) the education and
intelligence of the defendant; (5) the presence of coercive police procedures; (6) the defendant's
belief that no incriminating evidence would be found; (7) the nature of the police questioning;
(8) the environment in which the questioning took place; and (9) the possibly vulnerable
subjective state of the person consenting. All that was alleged was that Luz was alone at the
police station at 3am accompanied by several police officers. These weigh heavily against valid
consent.

The “stop and frisk” rule also does not apply. This normally applies when an officer observes
suspicious or unusual conduct which may lead him to believe that a crime may be afoot. It is
limited to protective search of outer clothing for weapons.

The US SC has held that when a police officer stops a person for speeding and issues a citation
instead of arresting him, this procedure does not authorize the officer to conduct a full search of
the car. Instead, officers may only conduct minimal intrusions, like ordering the motorist to alight
from the car or doing a patdown. While the concern for officer safety is still present in a routine
traffic stop, this concern in this context may justify “minimal” intrusion of ordering the driver
and passengers out of the car. It does not justify a greater intrusion. The officers may conduct a
“patdown” of the driver or a “Terry patdown” of the passenger compartment of a vehicle upon
reasonable suspicion that an occupant is dangerous and may gain immediate control of a weapon.

Thus, the drugs are inadmissible. Luz was acquitted.

92. People v. Huang Zhen Hua, GR 139301, September 29, 2004, Callejo, Sr., J. (Warrantless
Searches)
FACTS:
Police operatives of the Public Assistance and Reaction Against Crime (PARAC) under the DILG
received word from their informant that Lao, Chan and appellants Jogy Lee and Huang Zhen Hua
were engaged in illegal drug trafficking.

PARAC police secured a search warrant for violation of PD 1866, Illegal possession of firearms
and explosives, and for violation of Sections 12, 14, and 16 of RA 6425 on the Cityland condo and
Pacific Grand Villa. They went to Cityland first. No persons were found inside, and the policemen
found two kilos of shabu, paraphernalia for its production, and machines and tools used for the
production of fake credit cards.

They acquired information that Lao and Chan would deliver shabu at a restaurant in Manila, and
the policemen rushed there. When one of the police approached and introduced himself, Chan and
Lao fired shots. A shoot-out ensued, and Chan and Lao were killed. The police found one kilo of
shabu in Lao’s car.

The police then went to Pacific Grand Villa to enforce search warrant. They were with a
Cantonese interpreter to communicate with Lee, who let them enter the unit. They searched the
masters bedroom and found a kilo of shabu on a small cabinet, a transparent plastic baby bottle
containing suspected shabu, and a small plastic canister containing suspected shabu. Thus, they
brought Lee and Hua to PARAC headquarters and therein detained them.
Appellants were charged with violation of Section 16 of RA 6425. They were arraigned and
pleaded not guilty.

The RTC convicted appellants. Hence this appeal.

Huang Zhen Hua claims that he arrived as a tourist in the Philippines upon the invitation of Lee
and at the time of his arrest, he had been in the Philippines for merely 4 days. He claims to be just
a guest at the townhouse where Jogy Lee was staying and had no control over the townhouse. The
search of his room and his person was negative for drugs. He was not shown to be together with
Lau and Chan on any occasion. Thus, the people, thru the OSG, moves for his acquittal.

ISSUE:
Whether 1) Huang Zhen Hua and 2) Jogy Lee are guilty of the crime charged.

HELD:
1. On Huang Zhen Hua. NO. *Not consti related
The essential elements of the crime of possession of regulated drugs are the following: (a) the
accused is found in possession of a regulated drug; (b) the person is not authorized by law or by
duly constituted authorities; and, (c) the accused has knowledge that the said drug is a regulated
drug. This crime is mala prohibita, and, as such, criminal intent is not an essential element.
However, the prosecution must prove that the accused had the intent to possess (animus posidendi)
the drugs. Possession, under the law, includes not only actual possession, but also constructive
possession. Actual possession exists when the drug is in the immediate physical possession or
control of the accused. On the other hand, constructive possession exits when the drug is under the
dominion and control of the accused or when he has the right to exercise dominion and control
over the place where it is found. However, the prosecution must prove that the accused had
knowledge of the existence and presence of the drug in the place under his control and dominion
and the character of the drug.

In this case, there is no proof that Hua had actual or constructive possession of the regulated drug
found in the masters bedroom where Lee was sleeping or that Hua had accessed the room at any
time or that he had knowledge of the shabu in Lee’s bedroom. There is no evidence that Hua was
aware of the alleged illegal drug activities of Lao, Chan, and Lee. The evidence also falls short of
proving conspiracy of Hua with Lee, Chan, or Lao. The mere fact that on 2 or 3 occasions from
Hua’s arrival that he was seen with Lao, Chan, and Lee having dinner or lunch does not constitute
sufficient proof of conspiracy with them. Mere association with the principals or mere knowledge
of the conspiracy, without more, does not suffice.

2. On Jogy Lee. YES.


Lee avers that there are irregularities in the issuance and implementation of the search warrant; 1)
The policemen failed to show to her the warrant, inform her of their authority and explain their
presence in the condominium unit; 2) the policemen gained entry into the condominium unit by
force while she was sleeping; and 3) articles and personal effects she and Lao owns were taken
although not specified in the search warrant. Thus, she avers that the articles procured are
inadmissible in evidence.
The constitutional proscription against unreasonable search and seizure applies to Filipino citizens,
as well as to aliens temporarily residing in the country. The rule against unreasonable search
and seizure forbids every search that is unreasonable.

Section 7, Rule 126 of the Revised Rules of Criminal Procedure provides:


SEC. 7. Right to break door or window to effect search. The officer, if refused admittance
to the place of directed search after giving notice of his purpose and authority, may
break open any outer or inner door or window of a house or any part of a house or anything
therein to execute the warrant or liberate himself or any person lawfully aiding him when
unlawfully detained therein.

The police officers were obliged to give the appellant notice, show to her their authority, and
demand that they be allowed entry. They may only break open any outer or inner door or window
of a house to execute the search warrant if, after such notice and demand, such officers are
refused entry to the place of directed search. This is known as the knock and announce principle.
The method of entry of an officer into a dwelling and the presence or absence of such notice are
as important considerations in assessing whether subsequent entry to search and/or arrest is
constitutionally reasonable. A search would violate the Constitution if entry were illegal,
whether accomplished by force, illegal threat, or mere show of force.

Generally, officers implementing a search warrant must announce their presence, identify
themselves to the accused and to the persons who rightfully have possession of the premises to be
searched, and show to them the search warrant to be implemented by them and explain to them
said warrant in a language or dialect known to and understood by them. The requirement is not
a mere procedural formality but is of the essence of the substantial provision which safeguards
individual liberty. It is sufficient that the accused has notice of the officers, their authority and
purpose of the search and the object to be seized. A lawful entry is the indispensable predicate
of a reasonable search.

Unannounced intrusion into the premises is permissible when (a) a party whose premises or is
entitled to the possession thereof refuses, upon demand, to open it; (b) when such person in the
premises already knew of the identity of the officers and of their authority and persons; (c) when
the officers are justified in the honest belief that there is an imminent peril to life or limb; and
(d) when those in the premises, aware of the presence of someone outside (because, for example,
there has been a knock at the door), are then engaged in activity which justifies the officers to
believe that an escape or the destruction of evidence is being attempted. Suspects have no
constitutional right to destroy evidence or dispose of evidence. But these are NOT exclusive as
law enforcement interest may establish the reasonableness of an unannounced entry. Each case is
to be decided on its own facts and circumstances. In determining the lawfulness of an unallowed
entry and existence of probable cause, the courts are concerned only with what the officers had
reason to believe and the time of entry.

What constitutes breaking includes the lifting of a latch, turning a door knob, unlocking a chain or
hasp, removing a prop to or pushing open a closed door of entrance to the house, even a closed
screen door. As to how long an officer must wait before entering, each case must be decided on a
case-to-case basis.

The policemen in this case complied with Section 7 of Rule 126 of the Revised Rules of Criminal
Procedure before entering the condominium unit. Lee admitted that the police were accompanied
by a Cantonese interpreter who informed her that his companions were police officers who had
a search warrant and explained to her that the police were going to search the condominium unit.
Thus, Lee was sufficiently aware of the authority of the policemen.

Lee failed to prove that the police broke open the door. She could have asked for an ocular
inspection to show the door allegedly broken or show evidence in pictures, but she failed to do so.

While Lee was not one of the accused named in the search warrants, this did not proscribe the
policemen from arresting her. There was probable cause for her warrantless arrest independent
of that found by the judge who issued the search warrants against Lao and Chan. Probable cause
exists for the warrantless detention and arrest of one at the premises being searched when the facts
and circumstances within their knowledge and of which they had reliable and trustworthy
information are sufficient to themselves warrant a reasonable belief of a cautious person that an
offense has been or is being committed. In this case, the policemen had information that Lee and
Lao were living together as husband and wife and that Lee handled the accounting of the proceeds
of the illegal activities of Lao. They found that Lee and Lao shared a bedroom. Thus, Lee had joint
control and possession of the bedroom and the shabu found therein. There is probable cause to
believe that Lee had joint possession of the shabu.

93. People v. Andre Marti, GR 81561, January 18, 1991, Bidin, J. (Warrantless Searches;
search by private individual/establishment)
FACTS:
Marti and his common-law wife, Shirley Reyes, went to the booth of the “Manila Packing and
Export Forwarders” in the Pistang Pilipino Complex, Ermita, carrying 4 gift wrapped packages.
Anita, proprietress, attended to them. Marti informed Anita that he was sending the packages to a
friend in Switzerland and filled up the contract. Anita asked to inspect the packages, but Marti
refused, assuring her that it contained books, cigars, and gloves and were gifts to his friend. Anita
no longer insisted on inspecting. The 4 packages were placed in a brown box.

Before delivery of Marti’s box to the BoC, Job, proprietor and husband of Anita, following SOP,
opened the boxes for final inspection when he smelled a peculiar odor. He squeezed the bundles
and felt dried leaves. He took several grams of the contents and reported the shipment to NBI to
request for a lab exam. The NBI went to Job’s office in Ermita, brought out the box, opened it, and
found dried marijuana leaves. The NBI took charge of the box.

Marti, while claiming mail at the Central Post Office, was invited by the NBI to shed light on the
attempted shipment of the marijuana. Marti was charged with violation of RA 6425. The RTC
convicted him. Hence this appeal.

ISSUE:
Whether 4 packages containing marijuana given to Manila Packing and Export Forwarders, a
private establishment, for shipment to Switzerland which was opened by Job, the proprietor, after
smelling something peculiar from the packages without participation of police authorities and
thereafter delivered to the custody of the NBI are admissible in evidence.
HELD: YES.
1. Marti argues that the evidence was obtained in violation of his constitutional rights against
unreasonable search and seizure and privacy of communication, Sections 2 and 3, Art. III of the
Constitution and thus, they are inadmissible (Sec.3(2), Art.III).

The constitutional provision against unreasonable search originated in the 1935 charter which was
in turn derived almost verbatim from the US Constitution Fourth Amendment. Following the
exclusionary rule in Mapp v. Ohio, the Court in Stonhill v. Diokno applied the exclusionary rule,
abandoning Moncado v. People’s Court. The 1973 charter constitutionalized the Stonehill ruling
and is carried over to the 1987 Constitution.

In a number of cases, the Court strictly adhered to the exclusionary rule. But in all those cases, the
evidence was obtained by the State acting through the medium of its law enforcers or other
authorized government agencies. In this case, the evidence was discovered and obtained by a
PRIVATE person acting in a private capacity without participation of State authorities. Under
the circumstances, the accused cannot validly claim that his constitutional right was violated. An
act of a private individual allegedly in violation of Marti’s constitutional rights CANNOT be
invoked against the STATE. Without governmental interference, the liberties guaranteed in the
Constitution cannot be invoked against the State.

The right against unreasonable search refers to immunity of a person, whether citizen or alien,
from interference by government.

In State v. Bryan, where a parking attendant who searched a car to ascertain its owner found
marijuana instead without participation of police, the marijuana was declared admissible. The
fourth amendment also has been ruled to proscribe only governmental action.

Since the marijuana in this case came into government possession without violating his right
against unreasonable search, it is admissible. It was Job who searched/inspected the packages,
which was reasonable and SOP of Job as a precautionary measure before delivery of packages
to the BOC. He took samples to the NBI, summoned NBI agents to his office, and opened the
packages. The NBI agents made no search and seizure.

The mere presence of the NBI agents did not convert the reasonable search of Job into a warrantless
search. Merely to observe and look at that which is in plain sight is not a search. Where the
contraband are identified without trespass of the arresting officer, there is no search.

The Bill of Rights is not meant to be invoked against acts of private individuals. This is supported
by the framer’s deliberations. It is protection against the state. The Bill governs the relation of
the individual with the state.
If the search is made at the initiative of a private establishment for its own private purposes without
intervention of police authorities, the right against unreasonable search cannot be invoked. This
reasoning applies to Marti’s alleged violation of his rights to privacy and communication.

94. Social Justice Society (SJS) v. Dangerous Drugs Board, GR 157870, November 03, 2008,
Velasco, Jr., J. (Warrantless Searches; mandatory drug testing)
FACTS:
Section 36 of RA 9165, Comprehensive Dangerous Drugs Act of 2002, requiring mandatory drug
testing of candidates for public office, students of secondary and tertiary schools, officers and
employees of public and private offices, and persons charged before the prosecutor’s office with
certain offenses, among others, is assailed as unconstitutional. Pertinently, it provides:
SEC. 36. Authorized Drug Testing.—Authorized drug testing shall be done by any
government forensic laboratories or by any of the drug testing laboratories accredited and
monitored by the DOH to safeguard the quality of the test results. . . . The drug testing shall
employ, among others, two (2) testing methods, the screening test which will determine
the positive result as well as the type of drug used and the confirmatory test which will
confirm a positive screening test. . . . The following shall be subjected to undergo drug
testing:
(c) Students of secondary and tertiary schools. — Students of secondary and tertiary
schools shall, pursuant to the related rules and regulations as contained in the school's
student handbook and with notice to the parents, undergo a random drug testing . . .;
(d) Officers and employees of public and private offices. — Officers and employees of
public and private offices, whether domestic or overseas, shall be subjected to undergo a
random drug test as contained in the company's work rules and regulations, . . . for purposes
of reducing the risk in the workplace. (found positive= ground for suspension or
termination under Art. 282 of Labor Code)
(f) All persons charged before the prosecutor's office with a criminal offense having an
imposable penalty of imprisonment of not less than six (6) years and one (1) day shall
undergo a mandatory drug test;
(g) All candidates for public office whether appointed or elected both in the national or
local government shall undergo a mandatory drug test.

The COMELEC issued Resolution 6486, the rules and regulations for mandatory drug testing of
candidates for public office for the May 10, 2004 national and local elections. Section 1 thereof
required all candidates for public office in the May 10, 2004 synchronized national and local
elections to undergo mandatory drug test. Section 5 prohibited elected officers to enter upon the
duties of his office until he has undergone the mandatory drug test.

1. Petitioner Acquilino Pimentel, Jr., senator and candidate for re-election in the May 10, 2004
elections, filed a petition for certiorari and prohibition, seeking to nullify Section 36(g) and
Resolution 6486 for imposing a qualification for candidates for senators in addition to those
already provided in the 1987 Constitution.

2. SJS, a registered political party, petitioned to prohibit the Dangerous Drugs Board (DDB) and
PDEA from enforcing Section 36, (c) (d) (f) and (g) as they are unconstitutional as undue
delegation of legislative power, giving unbridled discretion to schools and employers to determine
the manner of drug testing. It also violates equal protection as they can be used to harass a student
or employee. The people’s right against unreasonable searches is also violated.

3. Petitioner Atty. Laserna, citizen and taxpayer, seeks that Sec.36 (c) (d) (f) and (g) be struck
down for infringing the right to privacy, against unreasonable search, against self-incrimination,
due process, and equal protection.

ISSUE:
Whether RA 9165, insofar as it requires public and private employees, candidates for public office,
students of secondary and tertiary schools, and persons accused of crimes punishable by not less
than 6 years and 1 day imprisonment, is constitutional.
HELD:
1. Standing.
Pimentel, as senator and candidate, has substantial interests in the subject matter of the petition.
As for SJS and Laserna, the rule on standing is relaxed owing to the transcendental importance
and paramount public interest involved in enforcing Sec. 36 of RA 9165.

2. Candidates for public office- Additional qualification, thus unconstitutional.


If a law or administrative rule violates the constitution, it is void and of no effect. The powers of
government are confined within the four walls of the constitution, and each department can
exercise only powers as are necessarily implied from the given powers. Thus, legislative power is
subject to substantive and constitutional limitations which circumscribe the exercise of power and
the allowable subjects of legislation. The substantive constitutional limitations are chiefly found
in the Bill of Rights and other provisions like Sec. 3, Art. VI, prescribing qualifications of
candidates for senators.

COMELEC also cannot impose qualifications on senatorial candidates as Congress cannot even
do so.

Sec. 36(g) effectively enlarges the qualification requirements under Sec.3 Art. VI. Sec.36(g)
requires a candidate for senator to be certified illegal-drug clean as pre-condition to the validity of
a certificate of candidacy (COC) for senator or to be voted upon and proclaimed as senator-elect.

The argument in defense that the provision does not expressly state that non-compliance with the
drug test is a disqualifying factor or would nullify a COC would be plausible if the drug test was
optional. But the law, without exception, made drug-testing on those covered mandatory.

3. Secondary and tertiary level students and public and private employees. Valid.
The drug test under Sec. 36 (c) (d) and (f), while mandatory, aims to stamp out illegal drug and
safeguard the well-being of the citizenry, particularly the youth, from the harmful effects of
dangerous drugs. This purpose can be achieved thru an intensive campaign against the trafficking
and use of dangerous drugs.

The right to privacy is a facet of the right protected by the guarantee against unreasonable search
and seizure under Sec.2, Art.III of the Constitution. Since this case is one of first impression, we
turn to US cases which are persuasive.
From the cases of Veronia School District v. Acton and Board of Education v. Earls, et al., we can
deduce and apply in this jurisdiction the following: (1) schools and their administrators stand in
loco parentis with respect to their students; (2) minor students have contextually fewer rights
than an adult, and are subject to the custody and supervision of their parents, guardians, and
schools; (3) schools, acting in loco parentis, have a duty to safeguard the health and well-being
of their students and may adopt such measures as may reasonably be necessary to discharge
such duty; and (4) schools have the right to impose conditions on applicants for admission
that are fair, just, and non-discriminatory.

Thus, the requirement of mandatory, random, and suspicionless drug testing of students is
constitutional. It is within the prerogative of educational institutions to require, as condition for
admission, compliance with reasonable school rules and regulations. The right to enroll is not
absolute but subject to fair, reasonable, and equitable requirements. Judicial notice of the
proliferation of drugs in the country threatening the well-being of the people is taken. The testing
may be necessary for the safety of the students, a legitimate government concern.

The mandatory drug test for public and private office employees is also justifiable. In saying that
the subjecting to drug testing without probable cause is unreasonable, petitioners failed to show
how it would violate the right to privacy and constitute unlawful or unconsented search under
Art.III Secs. 1 and 2.

The essence of privacy is the right to be left alone. The right to privacy means the right to be
free from unwarranted exploitation of one’s person or from intrusion into one’s private
activities in such a way as to cause humiliation to a person’s ordinary sensibilities. The right
to privacy yields to certain paramount rights of the public and defers to police power.

Under Sec.2, Art.III, reasonableness is the touchstone of the validity of a government search or
intrusion. Whether a search is reasonable is judged by balancing government-mandated
intrusion on the individual’s privacy interest against the promotion of some compelling state
interest. Since the drug testing for employees and students under RA 9165 is in the nature of
administrative search needing “swift and informal disciplinary procedures, the probable cause
standard is not required or even practicable. The review should nevertheless focus on the
reasonableness of the administrative search in question.

The first factor to consider in the matter of reasonableness is the nature of the privacy interest
which the drug testing, which is a search within Sec.2, Art.III, intrudes upon. The employee’s
privacy interest in an office is largely circumscribed by the company’s work policies, the CBA (if
any), and the inherent right of the employer to maintain discipline and efficiency in the workplace.
Their privacy expectation in a regulated office environment is reduced.

The search is “narrowly drawn” or “narrowly focused” as formulated in Ople v. Torres. Sec.36
and its IRR contain provisions specifically directed towards preventing embarrassment of the
employees. Nobody is singled out in advance for drug testing. The goal is to discourage drug use
by not telling in advance when and who is to be tested. The drug testing shall also be undertaken
under conditions calculated to protect as much as possible the employee’s privacy and dignity.
The procedure shall employ two testing methods- screening and confirmatory test- to ensure
trustworthiness of the results. It shall be conducted by trained professionals in labs monitored by
DOH. The test result shall be kept confidential. RA 9165 does not oblige the employer to report
any information relating to violation of RA 9165 received as a result of drug testing. Thus, the
intrusion into the employee’s privacy under RA 9165 has proper safeguards against embarrassing
leakages of test results and is relatively minimal.

The need for drug testing to minimize illegal drug use is substantial enough to override the
individual’s privacy interest under the premises. Considering the 1) reduced expectation of
privacy of employees, 2) compelling state concern, and 3) the well-defined limits in the law to
guide authorities in the conduct of drug testing, the drug test of employees is constitutional. Public
employees labor under supervision and restrictions imposed by Civil Service laws and other laws
on public officers like private employees.

4. Sec.36 is not undue delegation since it expressly provides how drug testing for students and
employees shall be conducted, enumerates the persons who shall undergo testing, taking into
account school rules or company work rules. The random procedure shall be observed. Safeguards
against compromising the confidentiality of the test results are established.

5. But the mandatory drug testing for persons accused of crimes, under Sec.36 (f) is
unconstitutional. The operative concepts in mandatory drug testing are “randomness” and
“suspicionless.” For persons charged with a crime, a mandatory drug testing can never be random
or suspicionless. They are not randomly picked nor beyond suspicion. Persons suspected of
committing a crime are impleaded against their will. Thus, by the fact of being hauled before the
prosecutor’s office and peaceably submitting themselves to drug testing, the accused do not
necessarily consent to the procedure nor waive their right to privacy. This would be a blatant
attempt to harness a medical test as a tool for criminal prosecution. This would violate right to
privacy under Sec. 2, Art. III. The accused are also forced to incriminate themselves.

95. Briccio Pollo v. Chairperson Constantino-David, GR 181881, October 18, 2011,


Villarama, Jr., J. (Warrantless Searches; search of computer of government employee;
noninvestigatory work-related serach and work-related misconduct search must be
reasonable at inception and scope; legitimate expectation of privacy; subjective expectation
and society is prepared to accept as reasonable;)
FACTS:
Pollo is a former supervising personnel specialist of the CSC Regional Office IV and the officer-
in-charge of the Public Assistance and Liaison Division (PALD). An unsigned letter-complaint
addressed to CSC Chairperson Karina Constantino-David, marked “confidential” and sent thorugh
LBC courier from a certain “Alan San Pascual” was received by the Integrated Records
Management Office (IRMO) at CSC Central Office. The letter said that an employee of the CSC,
chief of the “Mamamayan muna hindi mamaya na” division, Pollo, was acting as lawyer of accused
government employees having pending cases in the CSC. David formed a team of four with
background in IT directing them to investigate and back up all files in the computers found in
the Mamamayan Muna PALD and Legal divisions. The backing of all files in the hard disk of
computers at PALD was witnessed by several employees. Pollo was informed of the ongoing
copying thru text message.
Diskettes containing the back-up files from the hard disk of PALD computers were given to David.
The diskettes were examined and the 17 diskettes containing the files from Pollo’s computer were
found to have 40-42 draft pleadings or letters in connection with administrative cases in the CSC
and other tribunals. David thus required Pollo to explain.

Pollo denied that he is the one referred to in the anonymous complaint as he is not a lawyer nor
lawyering for people with cases in the CSC. He accused CSC officials of conducting a “fishing
expedition” when they unlawfully copied his personal files in the computer violating his right to
privacy, against self-incrimination, and warrantless search and seizure. The CSC charged Pollo
with dishonesty, grave misconduct, conduct prejudicial to the best interest of the service and
violation of RA 6713, Code of Conduct and Ethical Standards for Public Officials and Employees.
The CSC set the hearing of the investigation. Pollo filed for TRO and preliminary injunction with
the CA. He did not attend the CSC hearing since he wanted to suspend it until the CA issues its
decision. Thus, the CSC found him guilty of the charges and dismissed from the service.

The CA dismissed his petition for certiorari assailing the show-cause order to explain and the CSC
charges against him. Hence this appeal.

ISSUE:
Whether the search of a government-issued computer used by Pollo, consisting of the copying of
files therefrom, after an anonymous complaint against Pollo that he was “lawyering” for people
who have cases with the CSC office in which he was working, where CSC memorandum policy
states that the computers are subject to monitoring and its use is subject to waiver of expectation
of privacy, violates Pollo’s right to privacy and against unreasonable searches and seizures.
HELD: NO.
Pollo alleges that the search of his office computer and copying of personal files without his
knowledge and consent violates his right to privacy.

The right to privacy is a facet of the guarantee against unreasonable search and seizure under Sec.2,
Art.III of the 1987 Constitution. In Katz v. US, the US SC held that the electronic recording of a
conversation in an enclosed public telephone booth by the FBI violates right to privacy and
constituted a search and seizure since petitioner had a reasonable expectation of privacy therein. It
was noted that the existence of privacy right involved 2 requirements: 1) a person has exhibited an
actual (subjective) expectation of privacy, and 2) that the expectation be one that society is
prepared to recognize as reasonable (objective).

In O’Connor v. Ortega, the US SC stated that individuals do not lose fourth amendment rights
merely because they work for the government instead of a private employer. The correct analysis
has 2 steps: 1) because some government offices may be so open to fellow employees or the public
that no expectation of privacy is reasonable, a court must consider the operational realities of the
workplace to determine if an employee’s fourth amendment rights are implicated, and 2) where
an employee has a legitimate privacy expectation, an employer’s intrusion on that expectation
for noninvestigatory, work-related purposes and investigations of work-related misconduct should
be judged by the standard of reasonableness under all circumstances. It was held that fourth
amendment rights are implicated only if the conduct infringed an expectation of privacy that
society is prepared to consider as reasonable.

It was held that since Dr. Ortega kept personal correspondence and private items in his office while
work-related files were stored outside his office, without regulation discouraging employees from
storing personal papers in their desks, Dr. Ortega has a reasonable expectation of privacy in his
desk.

In Searches by a public employer, we must balance the invasion of the employee’s legitimate
expectations of privacy against the government’s need for supervision, control, and the
efficient operation of the workplace. Requiring a warrant whenever the employer wants to enter
an employee’s desk or cabinets for a work-related purpose would seriously disrupt the conduct of
business. The governmental interest justifying work-related intrusions by public employers is the
efficient and proper operation of the workplace. A probable cause requirement for searches to
correct employee misconduct rather than reasonable suspicion would translate to irreparable
damage to the agency’s work and to the public interest. The special needs beyond the normal need
for law enforcement make the probable cause requirement impracticable for legitimate, work-
related noninvestigatory intrusions and investigations of work-related misconduct.

Thus, the US SC held that public employer intrusions on privacy interests of government
employees for noninvestigatory work-related purposes and investigations of work-related
misconduct should be judged by the standard of REASONABLENESS under all circumstances.
Under this, both the INCEPTION and SCOPE of the intrusion must be reasonable.

A search of an employee’s office by a supervisor will be justified at inception when there are
reasonable grounds for suspecting that the search will turn up evidence that the employee is
guilty of work-related misconduct or that the search is necessary for a noninvestigatory work-
related purpose like to retrieve a needed file. The scope is permissible when the measures adopted
are reasonably related to the objectives of the search and not excessively intrusive in light of
the nature of the misconduct.

In this US case, a balancing test was laid down where government interests are weighed against
the employee’s reasonable expectation of privacy. This reasonableness test implicates neither
probable cause nor the warrant requirement, which are related to law enforcement.

In US v. Mark Simons, it was held that Simons did not have a legitimate expectation of privacy
with regard to the record or fruits of his internet use (downloaded porn) as the CIA’s internet policy
stated clearly that there would be audits, inspections and monitoring of the employees’ internet
use. Upon initial discovery of prohibited internet activity from his computer, they examined and
confirmed that he downloaded porn. The warrantless entry into his office was reasonable since at
the inception of the search, the employer had “reasonable grounds for suspecting” that the hard
drive would yield evidence of misconduct.

1. Did Pollo have a reasonable expectation of privacy in his office and computer files? NO.
The relevant circumstances to consider include: 1) employee’s relationship to the item seized, 2)
whether the item was in the immediate control of the employee when seized, and 3) whether the
employee took actions to maintain his privacy in the item.” These are relevant to both the
subjective and objective prongs of the reasonableness inquiry. Where the employee used a
password on his computer, did not share his office and locked it, he had a legitimate expectation
of privacy and any search therein must comply with the fourth amendment.

Pollo failed to prove that he had an actual (subjective) expectation of privacy either in his office
or computer. He did not allege that he had a separate enclosed office which he did not share with
anyone or that his office was always locked, or that he used passwords or means to prevent others
from accessing his computer files. Contrarily, he admits that being in the public assistance office
of CSC-ROIV, he normally would have visitors in his office whom he allowed to use his
computer. Under this scenario, Pollo could not have such expectation of privacy that society
would recognize as reasonable.

Even if he did have at least a subjective expectation of privacy in his computer, this is negated by
a policy regulating the use of office computers as in Simons. Office Memorandum 10 S.2002
provides that computer resources are to be used only for legitimate business purposes and that
“Users expressly waive any right to privacy in anything they create, store, send, or receive on the
computer through the internet.” “Users understand that the CSC may use human or automated
means to monitor the use of its computer resources. It also provides that a computer resource is
not personal property and may be shared or operated by other users, and that “passwords do not
imply that users have an expectation of privacy in the material they create or receive on the
computer system.”

The CSC had implemented a policy that put its employees on notice that they have no
expectation of privacy in anything they create etc. on office computers and that the CSC may
monitor the use of computers. This implies that on-the-spot inspections may be done to ensure
that the computer is used only for legitimate business purposes.

2. Was the search authorized by CSC Chair David reasonable in its inception and scope? YES.
The search of Pollo’s files was in connection with investigation of work-related misconduct
prompted by an anonymous letter-complaint. A search by a government employer of an
employee’s office is justified at inception when there are reasonable grounds for suspecting that
it will turn up evidence that the employee is guilty of work-related misconduct.

The nature of the imputation, that a division chief in CSCRO IV was “lawyering” for parties with
cases in said office, was serious and grievously disturbing. Considering the nature of the
accusation, the CSC had to act fast. The computers would be a likely starting point in finding
incriminating evidence. The ephemeral nature of computer files, that they could be easily
destroyed at one click, necessitated immediate action.

The case of Anonymous Letter-Complaint against Atty. Morales, Clerk of Court, where a spot-on
investigation of Atty. Morales anonymously alleged to be consuming his working hours attending
to personal cases and where it was held that the documents retrieved from his computer is
inadmissible for violating his right against unreasonable searches is to be distinguished from this
case. The computer therein involved a personal computer of a court employee where Pollo’s is a
government-issued computer which the CSC has absolute right to regulate and monitor. The
relationship of Pollo with the seized item, computer, and other factors and the CSC policy failed
to establish that he had a reasonable expectation of privacy in the office computer assigned to him.

As to the validity of Pollo’s dismissal, the fact that the documents were retrieved from his computer
raises the presumption that he was the author thereof as he had control of said computer. Thus, the
dismissal was valid.

96. Disini v. SOJ, GR 203335, February 18, 2014, Abad, J. (Warrantless Searches)
FACTS:
RA 10175, the Cybercrime Prevention Act of 2012, is assailed as unconstitutional.

ISSUE:
1. Whether RA 10175, in penalizing computer-related identity theft and defining it as the
“intentional acquisition, use, misuse, transfer, possession, alteration, or deletion of identifying
information” of another without right, violates the right to privacy.
2. Whether Section 12 of RA 10175 is constitutional.
3. Whether Section 19 of RA 10175 is constitutional.
HELD:
1. NO.
Section 4(b)(3) provides that the following is a cybercrime and computer-related offense:
(3) Computer-related Identity Theft. — The intentional acquisition, use, misuse, transfer,
possession, alteration, or deletion of identifying information belonging to another,
whether natural or juridical, without right: Provided: that if no damage has yet been caused,
the penalty imposable shall be one (1) degree lower.

This is assailed as violative of due process, privacy of correspondence, and freedom of the press.

The right to privacy or the right to be let alone is a facet of the right against unreasonable searches
and seizures. In Morfe v. Mutuc, the Court ruled that the right to privacy exists independently of
its identification with liberty. It is in itself fully deserving of constitutional protection.

“Zones of privacy” are protected. Within these zones, any form of intrusion is impermissible
unless excused by law and in accordance with customary legal process. The meticulous regard
of these zones arises not only from the conviction that the right to privacy is the right most valued
by civilized men, but also from our adherence to the Universal Declaration of Human Rights
(UDHR), mandating that "no one shall be subjected to arbitrary interference with his privacy" and
"everyone has the right to the protection of the law against such interference or attacks."

Two constitutional guarantees create these zones: 1) right against unreasonable searches, and 2)
right to privacy of communication and correspondence. In assessing the challenge that the state
intruded into these zones, a court must determine if a person has exhibited a reasonable
expectation of privacy and, if so, whether that expectation was violated by unreasonable
government intrusion.

The usual identifying information regarding a person includes his name, his citizenship, his
residence address, his contact number, his place and date of birth, the name of his spouse if any,
his occupation, and similar data. The law punishes those who acquire or use such identifying
information without right, implicitly to cause damage. Petitioners fail to show how government
effort to curb computer-related identity theft violates the right to privacy and correspondence and
due process.

2. NO.
Section 12 of RA 10175 provides:
Sec. 12. Real-Time Collection of Traffic Data. — Law enforcement authorities, with due
cause, shall be authorized to collect or record by technical or electronic means traffic data
in real-time associated with specified communications transmitted by means of a computer
system.
Traffic data refer only to the communication's origin, destination, route, time, date, size,
duration, or type of underlying service, but not content, nor identities.
All other data to be collected or seized or disclosed will require a court warrant.

This is assailed as violative of the right to privacy and to be protected from government snooping
into the messages or information that they send to one another.

A law may require disclosure of matters normally considered private but only upon showing that
such requirement has a rational relation to the purpose of the law, that there is compelling State
interest behind the law, and that the provision is narrowly drawn. In assessing regulations
affecting privacy rights, courts should balance the legitimate concerns of the State against
constitutional guarantees. The State has a compelling state interest. There is a need to put order
to the tremendous activities in cyberspace for public good. The government should be able to
monitor traffic data to enhance its ability to combat cybercirmes.

Yesterday’s traffic data is not adequate for fighting cybercrimes and real-time data is not thus
superfluous as those who commit the cybercrimes could easily evade detection and prosecution by
simply moving the physical location of their computers from day to day. Cybercrimes can be
committed anywhere.

Petitioners point out that Section 12 does not provide ample safeguards against invading right to
privacy. But this right is not unqualified. The US SC, in Whalen v. Roe, classified privacy into two
categories: 1) decisional privacy and 2) informational privacy. Decisional privacy involves the
right to independence in making certain important decisions, while informational privacy refers to
the interest in avoiding disclosure of personal matters. It is informational privacy that is sought
to be protected from collection of real-time data.

Informational privacy has two aspects: 1) the right not to have private information disclosed,
and 2) the right to live freely without surveillance and intrusion. In determining if a matter is
entitled to the right to privacy, this Court has laid down a two-fold test: 1) subjective test- one
claiming the right must have an actual or legitimate expectation of privacy over a certain
matter, and 2) objective test- where his expectation of privacy must be one society is prepared
to accept as objectively reasonable. The challenge is in relation to all users of information and
communications technology (ICT). The expectation of privacy must thus be measured from the
general public’s point of view.
Transmitting communications is akin to putting a letter in an envelope properly addressed, sealing
it closed, and sending it through postal service. Those sending letters have no expectations that no
one will read the information outside the envelope. When data is sent from any one source, the
content is broken into packets and around the packets is a header. The header contains traffic data:
where the packet originated, what kind of data is in the packet (SMS, voice call, video, email, etc.),
where the packet is going, etc. Data sent through the internet across the ocean only show the
recipient’s internet protocol (IP) addresses and not their names or addresses. Section 12 does not
let authorities look into the contents of the messages and uncover the identities of the sender
and recipient.

ICT users must know that they cannot exchange data or communicate over cyberspace except
through some service providers to whom they must submit certain traffic data needed for
successful cyberspace communication. The conveyance of the data removes them from the private
sphere, making the expectation to privacy regarding them an expectation society is not
prepared to recognize as reasonable.

BUT when seemingly random bits of traffic data are gathered in bulk and analyzed, they reveal
patterns of activities which can be used to create profiles of the persons under surveillance.
With enough traffic data, analysts may be able to determine a person’s close associations,
religious views, political affiliations, and sexual preferences. Such is beyond what the public
might expect to be disclosed and falls within matters protected by the right to privacy.

Section 12 fails to hint at the meaning it intends for the phrase “due cause.” It just says, “with due
cause,” justifying a general gathering of data. It is akin to a general warrant.

Due cause also does not describe the purpose for which the data collection will be used. Will it be
for identifying the perpetrator? For case build-up? Etc. The authority Sec.12 gives to law
enforcement agencies is too sweeping and lacks restraint. The restraint of not disclosing
identities or content data is but an illusion. Nothing can prevent them from looking into the identity
of the sender and receiver and the data’s contents. The grant of power to track cyberspace
communications in real-time and determine their sources and destinations must be narrowly
drawn to prevent abuses.

3. NO.
Section 19 provides:
Sec. 19. Restricting or Blocking Access to Computer Data. — When a computer data is
prima facie found to be in violation of the provisions of this Act, the DOJ shall issue an
order to restrict or block access to such computer data.

Petitioner claims that this violates freedom of expression and the right against unreasonable
searches and seizures.

Computer data may refer to entire programs or lines of code. Computer data produced or created
by authors may constitute personal property. Thus, they are protected from unreasonable
searches and seizures, whether stored in their personal computers or in the service provider’s
systems. The computer data’s content may also constitute speech, thus Sec.19 may operate as a
restriction on freedom of expression over cyberspace. Sec.19 requires only prima facie in violation
of the cyber crime law. Thus, it is unconstitutional for violating freedom of expression and the
right against unreasonable searches and seizures.

WRIT OF HABEAS DATA


97. Marynette Gamboa v. Marlou Chan, GR 193636, July 24, 2012, Sereno, J. (Writ of
Habeas Data)
FACTS:
Marynette Gamboa was the Mayor of Dingras, Ilocos Norte at the time this petition was filed.
Respondent Police Senior Superintendent Chan was the officer-in-charge and respondent William
Fang was the chief of the Provincial Investigation and Detective Management Brach.

PGMA issued AO 275, creating an independent commission to address private armies in the
country. This body, the Zeñarosa Commission, was formed to investigate the existence of private
army groups (PAGs) in the country with a view to eliminate them before the May 10, 2012
elections. Upon the conclusion of its investigation, the Zeñarosa commission submitted to the OP
a confidential report.

Gamboa claims that the PNP in Ilocos Norte conducted surveillance operations against her and her
aides and classified her as someone who keeps a PAG. The PNP forwarded this information to
the Zeñarosa Commission causing her inclusion in the Report’s enumeration of individuals
maintaining PAGs. ABS-CBN also broadcasted on its evening news the portion of the report
naming her as one of the politicians maintaining a PAG. Her association with a PAG also appeared
on print media. Thus, she was publicly tagged as someone who maintains a PAG based on the
information the PNP forwarded to the Zeñarosa commission. and police surveillance operations.

Since her right to privacy was violated and her reputation maligned, she filed a petition for writ
of habeas data with the RTC against respondents officials of PNP-Ilocos Norte. She sought the
destruction of the unverified reports from the PNP database, withdrawal of all info forwarded to
higher PNP officials, rectification of the damage to her honor, and ordering respondents to stop
forwarding unverified reports against her and from making baseless reports.

The RTC instructed respondents to submit all information and reports forwarded to the Zeñarosa
commission as basis to include her in the list of persons with PAGs and to cease from forwarding
any information gathered against Gamboa to any government entity. The respondents claim that
their information is based on two criminal cases in which she was implicated.

The RTC ruled that while Gamboa’s right to privacy was violated, she failed to prove thru
substantial evidence that the information originated from respondents and that they forwarded the
information without prior verification. Thus, this petition by Certiorari.

ISSUE:
Whether Gamboa’s right to privacy is violated when the PNP collected private information about
her without her knowledge related to her being suspected of maintaining PAGs and forwarding
said information to the Zeñarosa commission established by AO275 established for the purpose of
investigating PAGs in the country and ultimately to dismantle them permanently, such that her
writ of habeas data may be granted.
HELD: NO.
1. Right to Privacy.
The right to privacy has long been recognized as a constitutional right. (Morfe v. Mutuc) Liberty,
in the constitutional sense means more than freedom from unlawful government restraint. It
includes privacy as well if it is to be a repository of freedom. The right to be let alone is the
beginning of all freedom. This is the most comprehensive of rights and the right most valued by
civilized men. The right to privacy is accorded recognition independently of its identification with
liberty. In itself, it is fully deserving of constitutional protection. The concept of limited
government has always included the idea that government powers stop short of certain
intrusions into the personal life of the citizen. This is one distinction between absolute and
limited government. Ultimate and pervasive control of the individual in all aspects of his life is
the hallmark of an absolute state. A limited government system safeguards a private sector, which
belongs to the individual, distinguishing it from the public sector, which the state can control.

The right of privacy is recognized in several Constitutional provisions: Sections 3(1), 1, 2, 6, 8,


and 17. The right to privacy is a fundamental right guaranteed by the Constitution.

But this right is NOT absolute. It has been held that while Article VI, Sec.21 guarantees respect
for the rights of persons affected by legislative investigation, not every invocation of the right to
privacy should be allowed to thwart a legitimate congressional inquiry. The right of the people to
access information on matters of public concern generally prevails over the right to privacy of
ordinary financial transactions. It is not absolute where there is an overriding COMPELLING
state interest. Thus, when this right competes with a state objective, the courts must weigh both
notions.

2. Writ of Habeas Data.


The writ of habeas data is an independent and summary remedy designed to protect the image,
privacy, honor, information, and freedom of information of an individual, and to provide a forum
to enforce one’s right to the truth and to informational privacy. It protects a person’s right to
control information regarding oneself, particularly when such information is collected through
unlawful means to achieve unlawful ends. To be granted, there must exist a nexus between the
right to privacy and the right to life, liberty, or security.

Section 1 of the Rule on the writ of habeas data reads:


Habeas data. — The writ of habeas data is a remedy available to any person whose right to
privacy in life, liberty or security is violated or threatened by an unlawful act or omission
of a public official or employee, or of a private individual or entity engaged in the
gathering, collecting or storing of data information regarding the person, family, home and
correspondence of the aggrieved party.

In Leander v. Sweden of the European Court of Human Rights (ECHR), Leander, a Swedish
citizen, was refused employment in a Naval Museum adjacent to a restricted military security
zone when the requisite personnel control resulted in an unfavorable outcome based on
information in the secret police register to which he was prevented access. He claimed that this
procedure of security control violated his right to privacy as nothing in his background would
warrant his classification in the register as a security risk. The ECHR ruled that the storage in the
register of information relating to his private life plus the refusal to allow him opportunity to refute
it amount to an interference in his right to respect for private life. But the interference was
justified on the ground, among others, that the personnel control system had a legitimate aim, to
protect national security.

The ECHR said that the notion of necessity implies that the interference corresponds to a social
need, and that it is proportionate to the legitimate aim pursued. The interest of the state to protect
national security must be balanced against the seriousness of the interference with Leander’s right
to respect for his private life. The state has a wide margin of appreciation in assessing the social
need and in choosing the means to achieve a legitimate aim. The fact that the information released
to the military was not communicated to Leander cannot by itself warrant the conclusion that the
interference was not necessary for national security for it is the absence of communication which
ensures the efficacy of the personnel control procedure. Thus, the state concluded that national
security interest prevails over the individual interests of Leander.

Similarly, in this case, determining whether the privilege of the writ of habeas data may be granted
entails a delicate balancing of the alleged intrusion into the private life of Gamboa and the relevant
state interest involved.

3. Collection and Forwarding of information by the PNP vs. state interest to dismantle PAGs.
The constitution mandates dismantling of private armies and armed groups not recognized by duly
constituted authority. It provides for one police force national in scope and civilian in character
and controlled by a national police commission. Thus, the issuance of AO 275 has a legitimate
aim- investigate PAGs with the aim of dismantling them permanently.

AO275 authorized the Zeñarosa commission to deputize the AFP, NBI, DOJ, PNP, and other law
enforcement agency to assist it in performing its functions. Thus, pursuant to the state interest of
dismantling PAGs and the commission’s powers, the PNP collected information on individuals
suspected of maintaining PAGs.

The forwarding of information by the PNP to the Commission was thus NOT an unlawful act
that violated or threatened Gamboa’s right to privacy in life, liberty, or security. The PNP was
expected to share intelligence regarding PAGs with the body created for investigating them.

The fact that the PNP released information to the Zeñarosa Commission without prior
communication to Gamboa and without affording her opportunity to refute the same is not a
violation or threat to her right to privacy since that act is an inherent and crucial component
of intelligence-gathering and investigation. Also, the PNP had a validation system to update
information on individuals to ensure that the data mirrored the situation on the field. Thus, there
were safeguards to make sure that the information remained accurate.

Pending a law on data protection, the Court declines to determine the propriety of sharing
information during specific stages of intelligence gathering. To do otherwise would supplant the
discretion of investigative bodies resulting in undue encroachment on their competence. But the
Court cautions these investigating entities that intelligence gathered must be released exclusively
to the authorities empowered to receive the relevant information.

While the respondents admitted the existence of the Report, they emphasized its confidential
nature. That it was leaked to others and the media was regrettable, but Gamboa failed to establish
that respondents were responsible for this unintended disclosure. There are other reliefs to address
the purported damage to her reputation. Resorting to the extraordinary remedy of the writ of habeas
data is unnecessary and improper. The state interest of dismantling PAGs outweighs the alleged
intrusion on Gamboa’s private life, especially when the collection and forwarding by the PNP of
information against her was pursuant to lawful mandate.

98. Rhonda Vivares v. St. Theresa’s College, GR 202666, September 29, 2014 (Writ of
Habeas Data; Facebook photo; privacy setting)
FACTS:
Nenita Julia Daluz and Julienne Suzara, minors, were graduating high school students at St.
Theresa’s College (STC), Cebu. In January 2012, while changing into their swimsuits for a beach
party, they took digital pictures of themselves clad only in their undergarments. These were
uploaded by Angela Tan on her Facebook profile. Mylene Escudero, a computer teacher at STC,
learned from her students that Julia, Julienne, and Chloe Taboada uploaded online pictures dressed
only in brassieres.

Escudero’s students logged in to their personal Facebook accounts and showed Escudero the
photos: 1) Julia and Julienne drinking liquor and smoking in a bar and 2) them along the streets of
Cebu wearing clothes that show their brassieres. There were times when the photos were viewable
by any Facebook user. Escudero reported the matter and showed it to Kristine Tigol, STC’s
discipline-in-charge. After investigation, STC found that the students deported themselves in a
manner prohibited by the student handbook.

The students in the pictures reported to the office of Celeste Purisima, the high school principal.
They claimed that they were castigated and verbally abused by STC officials in the meeting.
Purisima told their parents that as penalty, they were barred from joining the commencement
exercises.

Angela’s mother, Dr. Armenia Tan, filed a petition for injunction with the RTC against STC,
praying that STC be enjoined from implementing the penalty. Petitioner Rhonda Vivares, mother
of Julia, joined as intervenor.

Petitioners filed a petition for writ of habeas data, stating that the privacy of their children’s
Facebook accounts was set to “Friends only.” Thus, they have a reasonable expectation of privacy
which must be respected. Escudero violated their rights by saving digital copies of the photos and
showing them to STC officials. The RTC issued the writ and ordered respondents to file their
return with supporting affidavits. STC complied and filed their return arguing for the denial of the
petition.
The RTC dismissed the petition, stating that petitioners failed to prove violation of the minors’
right to privacy and that the photos, being uploaded to FB without restrictions as to who may view
them, lost their privacy. Hence this petition.

ISSUE:
Whether a writ of habeas data may issue where students of STC uploaded photos to Facebook of
themselves wearing only brassieres and drinking liquor and where an STC teacher, from the FB
account of her student, found and saved these photos and showed them to STC officials which
resulted in the students being barred from attending their graduation ceremony, and where the
privacy settings of the photos was sometimes set to public where every FB user could view them.
HELD: NO.
Given the nature of an online social network (OSN), 1) that it facilitates and promotes real-time
interaction among millions or billions of users sans the spatial barriers and 2) that any information
uploaded in OSNs leaves an indelible trace in the provider’s databases outside the control of end-
users, is there a right to informational privacy in OSN activities of its users?

1. The writ of habeas data is not only confined to cases of extralegal killings and enforced
disappearances, contrary to respondents’ submission. Section 2 of the rule on the writ provides:
Sec. 2. Who May File. — Any aggrieved party may file a petition for the writ of habeas
data. However, in cases of extralegal killings and enforced disappearances, the petition
may be filed by:

Had the intention been to narrow the operation of the writ to only extralegal killings, the above
underscored portion reflecting a variance of habeas data situations would not have been made.
Habeas data was designed to safeguard individual freedom from abuse in the information age..
It can be availed as an independent remedy to enforce one’s right to informational privacy.

2. “Engaged in the gathering, collecting, and storing” of data or information- meaning.


STC’s contention that it is not an entity engaged in such and thus, habeas data may not issue against
it, while valid to a point, is erroneous. Nothing in the rule suggests that habeas data is available
only against abuses of a person or entity engaged in the business of gathering, etc.:
Section 1. Habeas Data. — The writ of habeas data is a remedy available to any person
whose right to privacy in life, liberty or security is violated or threatened by an unlawful
act or omission of a public official or employee, or of a private individual or entity
engaged in the gathering, collecting or storing of data or information regarding the
person, family, home and correspondence of the aggrieved party.
When taken in its proper context, the above conveys that habeas data is a protection against
unlawful acts or omissions of public officials or private individuals engaged in such. They do NOT
need to be engaged in the BUSINESS of collecting or storing data. To “engage” in something
is different from undertaking a business. “Engage” means “to do or take part in something.” It
does not necessarily mean that the activity is done in pursuit of business. Whether the gathering
etc. is done in pursuit of business or of a personal endeavor, or for any other reason or no reason
at all, is immaterial.

3. Right to informational privacy on FB


The three strands of the right to privacy are: 1) locational or situational privacy, 2) informational
privacy, and 3) decisional privacy. What is relevant in this case is the right to informational
privacy, usually defined as the right of individuals to CONTROL information about
themselves.

Having an expectation of informational privacy is not necessarily incompatible with engaging in


cyberspace activities. But up to what extent is the right to privacy protected in OSNs?

4. FB’s privacy tools.


The purpose of OSN is to give users the ability to interact and stay connected to other members of
the same platform through sharing of statuses, photos, videos, etc. By “friending” another, the user
invites another to connect their accounts so that they can view any and all “public” and “friends
only” posts of the other. FB was armed with different privacy tools to regulate accessibility of a
user’s profile and information uploaded by the user. By selecting his/her privacy setting, users can
regulate the visibility and accessibility of photos etc.

Thus, it is through the availability of these privacy tools that OSN users may have a subjective
expectation that only those to whom they grant access to their profile will view the information
they post or upload thereto. But before one can have such expectation of privacy in his OSN
activity, they must first manifest the INTENTION to keep certain posts private through
employment of measures to prevent access thereto or to limit its visibility. The utilization of
the privacy tools is such manifestation of the user’s invocation of right to informational privacy.

If a photo uploaded with a privacy of “only me” were to be deemed still public, this position would
strip the privacy tools of their function and disregard the intention of the user to keep the
information within his private space. Did the minors limit disclosure of the photos such that the
images were kept within their zones of privacy?

Petitioners’ position is that since the photos were set to “only friends” safeguarded with a
password, people who are not FB friends, including STC, are barred from accessing the post
without their knowledge and consent.

Escudero said that her students, the minors’ FB “friends,” showed her the photos through their
own FB accounts. Thus, there was no special means resorted to by Escudero’s students to view the
photos. It is then reasonable to assume that the photos were viewable either by their FB friends or
the public at large. Since the default FB setting is “public,” it can be surmised that the photos
were viewable to everyone on FB without proof that the minors limited disclosure thereof. They
thus cannot invoke the protection attached to the right to informational privacy. A person who
places a photograph on the internet precisely intends to renounce all privacy rights especially
where there is no employment of protective measures to control access thereto. The more open
the method of transmission, the less privacy one can reasonable expect.

Even if the photos were viewable by “friends only,” this still remains outside their zones of privacy
since a lot of FB users “Friend” strangers and in view of the sheer number of “friends” one usually
has. A user’s friend can “share” the user’s post or tag others not friends with the user. Thus, setting
privacy to “friends” is no assurance that it can no longer be viewed by another user not FB friends
with the user.

Moreover, even if the photos were visible only to the students’ FB friends, STC cannot be taken
to task as it was merely a recipient of what were posted and they did not resort to unlawful
means of gathering information as it was voluntarily given to them by persons who had
LEGITIMATE access to said posts. The fault lies with the minors’ friends.

Self-regulation on the part of OSN users is the best means of avoiding privacy rights violations.

SEC.3 Privacy of Communication and


Correspondence
99. In the matter of petition for habeas corpus of Capt. Gary Alejano et al. v. Gen Pedro
Cabuay, GR 160792, August 25, 2005, Carpio, J. (Privacy of communication and
correspondence)
FACTS:
Some 321 armed soldiers, led by the now detained junior officers, entered and took control of the
Oakwood Premier Luxury Apartments in Makati. They disarmed the security officers and planted
explosives. They publicly renounced their support for PGMA and called for her resignation. 7pm
of the same day, July 27, 2003, they voluntarily surrendered after negotiations and returned to their
barracks. On July 31, Gen. Abaya, as Chief of Staff of AFP, directed all major service commanders
to turn over custody of ten junior officers to the Intelligence Service of the AFP (ISAFP) detention
center. The soldiers were charged with coup d’etat. The RTC issued the commitment orders, giving
custody of junior officers Trillanes and Capt. Gambala to the ISAFP.

Abaya issued directed all major service commanders to take into custody the military personnel
under their command who took part in the Oakwood incident. Petitioners filed a petition for habeas
corpus with the SC. The SC issued the writ and directed respondents to make a return of the writ
and to produce the persons of the detainees before the CA for hearing. The CA dismissed the
petition. It said that while the reading of Trillanes’ letter was a violation of his right to privacy of
communication, this does not justify habeas corpus. Hence this petition.

ISSUE:
Whether the opening and reading of the letters of Trillanes and Maetrecampo that they gave to
their lawyers while the former were in the ISAFP detention center, which letters were not related
to their lawyer-client relationship, violated the detainees’ right to privacy.
HELD: NO.
In a habeas corpus petition, the order to present an individual before the court is a preliminary step
in the hearing of the petition. The respondent must produce the person and explain the cause of his
detention. But this order is not a ruling on the propriety of the remedy or on the substantive matters
covered by the remedy. The SC’s order to the CA to conduct a factual hearing was not an
affirmation of the propriety of the habeas corpus. If the court finds the cause of detention unlawful,
then it should issue the writ and release the detainees. The CA, after hearing, found habeas corpus
inapplicable. The purpose of the writ is to determine whether a person is being illegally deprived
of his liberty. If the detention is proven lawful, the proceedings terminate.

Petitioners do not question the legality of the detention but bewail the regulation of Gen. Cabuay
in the ISAFP detention center preventing petitioners as lawyers from seeing the detainees, their
clients. They also claim that the ISAFP officials violated the detainees right to privacy of
communication when the ISAFP officials opened and read the personal letters of Trillanes and
Capt. Milo Maestrecampo.

Section 4(b) of RA 7438 reads:


b) Any person who obstructs, prevents or prohibits any lawyer, any member of the
immediate family of a person arrested, detained or under custodial investigation, or any
medical doctor or priest or religious minister or by his counsel, from visiting and conferring
privately chosen by him or by any member of his immediate family with him, or from
examining and treating him, or from ministering to his spiritual needs, at any hour of the
day or, in urgent cases, of the night shall suffer the penalty of imprisonment of not less
than four (4) years nor more than six (6) years, and a fine of four thousand pesos
(P4,000.00).
The provisions of the above Section notwithstanding, any security officer with custodial
responsibility over any detainee or prisoner may undertake such reasonable measures as
may be necessary to secure his safety and prevent his escape.

RA 7438, Sec.4(b) makes it an offense to prohibit a lawyer from visiting a detainee client “at any
hour of the day or, in urgent cases, of the night.” But the last paragraph qualifies that
“notwithstanding,” the detention officer may undertake reasonable measures to secure his safety
or prevent escape. The regulations governing a detainee’s confinement must thus be reasonable
measures. The regulation must be reasonably connected to the government’s objective to secure
the safety and prevent escape of the detainee.

Petitioners contend that there was prohibition of the detainees’ right to effective representation
when their visits were limited by the schedule of visiting hours. But the ISAFP officials did not
deny, but merely regulated, the detainees’ right to counsel. The purpose of the regulation is not
to render ineffective the right to counsel, but to secure the safety and security of the detainees on
pre-trial confinement. The visiting hours accorded to the lawyers of the detainees are reasonably
connected to the legitimate purpose of securing the safety and preventing the escape of all
detainees. While petitioners may not visit any time they want, they still can on a daily basis, thus
there is no impairment of the detainees’ right to counsel. They could visit from 8am-5pm, which
are regular business hours, the same time lawyers usually meet clients in their law office. Thus,
these hours pass the standard of reasonableness

It is argued that the bars separating the detainees from their visitors and the boarding of the iron
grills with plywood in the cells are unusual and excessive punishment. While a detainee may not
be punished before adjudication of guilt, detention inevitably interferes with a detainee’s desire to
live comfortably. It is only when the restrictions are arbitrary and purposeless that courts will infer
intent to punish. The fact that the restrictions inherent in detention intrude into the detainee’s desire
to live comfortably does not convert those into punishment.
An action constitutes punishment when 1) it causes the inmate to suffer some harm or disability
and 2) the purpose of the action is to punish the inmate. The harm must be significantly greater
than or be independent of the inherent discomforts of confinement. Moreover, courts should
exercise judicial self-restraint over prison matters in deference to administrative expertise.

In this case, the separation of the detainees from visitors by iron bars is not punishment but merely
a limitation on contact visits. It only prevents direct physical contact but still allows visual,
verbal, non-verbal, and limited physical contact with visitors. This is a reasonable, non-punitive
response to valid security concerns. This measure intends to fortify the individual cells to prevent
detainees from passing contraband and weapons from one cell to another. the boarded grills ensure
security and prevent disorder. The diminished illumination and ventilation are but discomforts
inherent in the fact of detention.

-On Right to Privacy.


It is contended that the detainees’ right to privacy was violated when the ISAFP officials opened
and read the letters handed by detainees Trillanes and Maestrecampo to one of petitioners
for mailing. The letters were not sealed in envelope but only folded as there were no envelopes in
the ISAFP detention center.

US courts generally permit prison officials to open and read incoming and outgoing mail of
convicted prisoners to prevent smuggling of contraband and to avert coordinated escapes. Inmates
were deemed to have no right to correspond confidentially with anyone. The only restriction
was that the right of inspection should not be used to delay unreasonably the communications
between the inmate and lawyer. Eventually, the inmates’ outgoing mail to attorneys, courts, and
court officials received respect, the infringement of which was held to violate a prisoner’s right
to consult with his attorney in absolute privacy, which right is not abrogated by the prison
authorities’ interests in the administration of the institution. There is small risk that attorneys would
conspire in plots that threaten prison security.

The US SC also held that an inmate has no reasonable expectation of privacy in his cell and for
incoming mail. He knows that if he sends non-privileged mail, he exposes his letter to possible
inspection by jail officials. But incoming mail from LAWYERS of inmates enjoys limited
protection such that prison officials can open and inspect the mail for contraband but could
not read the contents without violating the inmates’ right to correspond with his lawyer. The
inspection of privileged mail is limited to physical contraband and not verbal contraband.

The letters in this case were not sealed in an envelope. The inspection of the folded letter is a
valid measure as it serves the same purpose of opening sealed letters for inspection of
contraband. The letters alleged to have been read by the ISAFP authorities were not confidential
letters between the detainees and their lawyers. Petitioners were merely acting as the detainees’
personal courier and not as counsel when he received the letters for mailing. Since the letters were
NOT confidential communication between the detainees and their lawyers, the ISAFP
officials could read them. If the letters are MARKED confidential communication, the detention
officials should not read but only inspect it in the presence of the detainees. (US court said that for
any communication to be given special treatment, it must be specially marked as originating from
an attorney as not to make prison officials check in every case if a communication was from an
attorney before inspecting it, which would be an impossible task of administration.)

That a law is required before an executive officer could intrude on a citizen's privacy rights is a
guarantee that is available only to the public at large but not to persons who are detained or
imprisoned. The right to privacy of those detained is subject to Section 4 of RA7438, as well as to
the limitations inherent in lawful detention or imprisonment. By the very fact of their detention,
pre-trial detainees and convicted prisoners have a diminished expectation of privacy rights.

In assessing the regulations in detention and prison facilities alleged to violate the rights of
prisoners, US courts balance the guarantees of the constitution with the legitimate concerns of
prison administrators. This deferential review stems from the principle that strict scrutiny would
seriously hamper the daily judgment of prison officials and their ability to adopt innovative
solutions to prison administration.

The junior officers are detained with other high-risk persons from the Abu Sayyaf and NPA. Thus,
the military custodian must be given a wider range of deference in implementing ISAFP
regulations. He is in a better position to know the security risks.

100. Cecilia Zulueta v. CA, GR 107383, February 20, 1996, Mendoza, J. (Privacy of
communications and correspondence)
FACTS:
Zulueta is the wife of private respondent Alfredo Martin. She entered the clinic of Alfredo, a
doctor, and in the presence of her mother, a driver, and Alfredo’s secretary, forcibly opened the
drawers and cabinet in her husband’s clinic and took 157 documents consisting of private
correspondence between Dr. Martin and his alleged paramours, greeting cards, diaries, passport,
photos, etc. they were used in evidence in a case for legal separation and disqualification from the
practice of medicine which Zulueta filed against Alfredo.

Alfredo brought this action for recovery of the documents and for damages against Zulueta with
the RTC. The RTC ruled for Alfredo, declaring him the owner of the properties and ordering
Zulueta to return the properties and to pay damages. They were enjoined from using the papers as
evidence. The CA affirmed.

ISSUE:
Whether Zulueta, wife of Dr. Alfredo, who went to the clinic of Alfredo and forcibly took papers
and documents from Alfredo’s drawers and cabinets may use these papers as evidence in a legal
separation and disqualification from practice of medicine proceeding.
HELD: NO.
It is contended that in the disbarment case filed against Zulueta’s lawyer, Atty. Felix, the SC stated
that the evidence acquired from Alfredo were admissible in evidence and their use by Felix is not
malpractice, thus in this case, it should be admissible. But that case was for disbarment. Felix’s
acquittal therein is no more than a declaration that his use of the papers to secure Dr. Alfredo’s
admission as to their authenticity did not constitute a violation of the injunctive order of the RTC.
The decision in that case did not establish the admissibility of the papers. He was acquitted only
because at the time he used the coduments, the order of the RTC prohibiting their use was
temporarily restrained by this Court.

The papers are inadmissible. The constitutional injunction that “the privacy of communication
and correspondence” to be inviolable is no less applicable simply because it is the wife who is
the party against whom the constitutional provision is to be enforced. The only exception is if
there is a lawful order from a court or when public safety or order requires otherwise, as
prescribed by law. Any violation of this provision renders evidence obtained inadmissible “for
any purpose in any proceeding.” The intimacies between husband and wife do not justify them
breaking the cabinets of the other and ransacking them for evidence of marital infidelity. A person,
by contracting marriage, does not shed his integrity or right to privacy as an individual and the
constitutional protection is available to him.

101. Kilusang Mayo Uno (KMU) v. Director-General of NEDA, GR 167798, April 19, 2006,
Carpio, J. (Privacy of communication and correspondence)
FACTS:
PGMA issued EO 420, requiring all government agencies to harmonize their identification (ID)
systems and authorizing the director-general of NEDA to implement it. It required data to be
collected and recorded, limited to the following: name, home address, sex, picture, signature,
birthday, birthplace, marital status, parents’ names, height, weight, two index fingers and two
thumbmarks, prominent distinguishing features, and TIN. EO 420 requires all government
agencies and GOCCs to adopt a uniform data collection and format for their existing ID systems.

It is alleged that EO 420 usurps legislative functions and infringes the right to privacy.

ISSUE:
Whether an executive order, EO 420, by PGMA directing all government agencies and GOCCs to
harmonize their identification systems, requiring data to be collected and recorded on the name,
home address, sex, picture, signature, birthday, birthplace, marital status, parents’ names, height,
weight, two index fingers and two thumbmarks, prominent distinguishing features, and TIN of
persons, and requiring such government agencies to adopt a uniform data collection and format
for their existing ID systems is a valid EO.
HELD:
1. On alleged usurpation of legislative power. – no usurpation
EO420 applies only to government entities that issue ID cards as part of their functions under
existing laws. They have already been issuing ID cards even before EO 420, like SSS, GSIS,
Philhealth, LTO, etc. All government entities that issue IDs as part of their functions under existing
laws are required to adopt a uniform data collection and format for their IDs. Its purpose is to
reduce costs, achieve efficiency and reliability, insure compatibility, and provide convenience to
people served by them. EO 420 limits the data to be collected only to 14 specific items. These are
the usual data required for personal identification by government entities and even by the private
sector. Currently, entities like LTO require more data from applicants. EO 420 would reduce the
data required to be collected in the ID databases. The data currently collected by various
government entities are disparate and the IDs they issue are dissimilar. Making the ID formats
uniform will achieve substantial benefits like savings, compatibility, etc.
The president may direct government entities under the Executive department to adopt a uniform
ID format as she has “control of all executive departments” under Section 17, Art. VII of the
Constitution. Thus, the president, under the power of control, can direct all government entities, in
the exercise of their functions under existing laws, to adopt a uniform ID format to achieve savings
etc. But EO 420 only applies to the executive department, not to the judiciary or COMELEC, so it
does not establish a single ID system for all branches of the government as legislation is needed
for this.

The Constitution also mandates the president to ensure that laws are faithfully executed. There are
laws mandating government entities to reduce costs, increase efficiency and improve public
services. Thus, EO 420 only ensures that such laws are faithfully executed. Thus, EO 420 is simply
an executive issuance, not legislation.

EO 420 also does not require any appropriation because the existing ID card systems have the
proper appropriation already. It is not compulsory on all branches nor on all citizens and requires
only a narrow and focused collection of personal data while safeguarding its confidentiality.

2. Right to privacy.
Since petitioners do not claim that the ID systems of government entities prior to EO 420 violate
their right to privacy, they have even less basis to complain against the unified ID system. The
data collected under EO 420 is limited to only 14 specific data, and the ID card itself will show
only 8 specific data. The ID card system under EO 420 will require even less data than under the
disparate systems prior to EO 420.

It even establishes safeguards on the collection, recording, and disclosure of personal ID to protect
right to privacy which before it there were none. It mandates that the data collected be strictly
confidential and limited for purposes of establishing the identity of the person, etc.

The right to privacy does not bar the adoption of reasonable ID systems. Except the 8 data shown
on the ID card, the personal data collected are treated as strictly confidential under EO 420. These
are not only strictly confidential but also personal matters. Sec.7 of Art.III grants the “right of
the people to information on matters of public concern.” Personal matters are exempt from outside
coverage of the people’s right to information on public matters. The data treated as strictly
confidential under EO 420 being private matters, they cannot be released.

In Whalen v. Roe, the US SC upheld the validity of a NY law requiring doctors to give the
government reports on patients who received prescription drugs with potential for abuse. The
government had a central computerized database with the names and addresses of the patients and
identity of the prescribing doctors. It was held that disclosures of private medical information to
public health agencies are essential to modern medical practice although it may reflect unfavorably
on the patient.

Compared to the data in Whalen, the 14 specific data under EO 420 are far less sensitive and
personal and are far benign. These data are just routine data for ID systems. Whalen thus has
persuasive force for upholding EO 420 as non-violative of the right to privacy. (the case was cited
by the dissenters)
102. Blas Ople v. Ruben Torres, GR 127685, July 23, 1998, Puno, J. (Privacy of
communication and correspondence)
FACTS:
AO308 was issued by President Ramos for the adoption of a national computerized identification
reference system. Ople filed this petition against ES Torres. He claims that AO308 is a usurpation
of legislative powers, requires appropriation of public funds, and violates rights to privacy.

ISSUE:
Whether a law which provides for a national computerized identification reference system and for
the collection of biometrics data for such purpose violates the right to privacy.
HELD:
1. Usurpation of legislative power. YES
Administrative power is concerned with applying policies and enforcing orders as determined by
proper governmental organs. It allows the president to fix a uniform standard of administrative
efficiency and check the conduct of his agents. He can, to this end, issue AOs, rules and
regulations.

AO 308 involves a subject not appropriate to be covered by an AO. It establishes for the first time
a national computerized identification reference system. It is not correct to argue that AO 308 is
not a law as it confers no right, imposes no duty, affords no protection, and creates no office. under
AO308, a citizen cannot transact business with government agencies delivering basic services to
the people without the ID. Without the ID, a citizen cannot exercise his rights and enjoy his
privileges. An AO must be in harmony with the law and for the purpose of implementing it and
carrying out legislative policy.

2. Right to privacy. – Violated.


Facially, AO 308 violates the right to privacy. Its essence is the right to be let alone. It has a
constitutional foundation. (Secs. 1, 2, 3(1), 6, 8, and 17, of Art.III) Zones of privacy are also
protected in our laws. The NCC provides that "[e]very person shall respect the dignity, personality,
privacy and peace of mind of his neighbors and other persons" and punishes as torts the meddling
into the privacy of another. It also makes a public officer or private individual liable for damages
for violation of the rights of another person and recognizes the privacy of letters and private
communications.

We prescind from the premise that the right to privacy is a fundamental right guaranteed by the
Constitution. Thus, it is the burden of government to show that AO308 is justified by some
compelling state interest and that it is narrowly drawn. AO308 aims to 1) provide citizens and
foreigners with the facility to conveniently transact business and 2) the need to reduce fraudulent
transactions and misrepresentations by persons seeking basic services. It is debatable whether these
are compelling enough to warrant AO308. But what is not arguable is the broadness, the
vagueness, the overbreadth of AO 308 which if implemented will put our people’s right to
privacy in clear and present danger.

The heart of AO308 lies in sec.44 which provides for a Population Reference Number (PRN) as a
“common reference number to establish a linkage among concerned agencies through biometrics
technology and computer application designs. Biometrics is the science of the application of
statistical methods to biological facts; a mathematical analysis of biological data. It provides a
precise confirmation of an individual’s identity thru the individual’s own physiological and
behavioral characteristic like a fingerprint, retinal scan, hand geometry, or facial features. A
behavioral characteristic is influenced by the individual’s personality and includes voice print,
signature, and keystroke. Most biometric ID systems use a card or personal identification number
(PIN) for initial identification. It is used to verify that the individual holding the card or the PIN is
the legitimate owner thereof.

A most common form of biological encoding is finger-scanning where a fingertip is scanned and
turns the unique pattern therein into an individual number called a biocrypt. It is stored in computer
data banks and becomes a means of identifying an individual using a service. A retinal scan maps
the capillary pattern of the retina and produces a unique print. Another method, “artificial nose”,
analyzes the unique combination of substances excreted from the skin. A thermogram uses
infrared cameras to see a unique heat distribution of the face, different bone , skin, fat, blood vessel
densities that contribute to the person’s heat signature.

AO308 does not state what specific biological characteristics and what particular biometrics
technology shall be used to identify people who seek its coverage. It also does not state whether
the encoding of data is limited to biological information alone for identification purposes. The
indefiniteness of AO308 can give the government roving authority to store and retrieve
information for a purpose other than the identification of the individual through his PRN. The
POTENTIAL FOR MISUSE of the data gathered cannot be underplayed. Everytime a person
uses his PRN to deal with a government agency, it will be recorded. His transactions for loan
availments, income tax returns, salns, etc. may be included. The more frequent use of the PRN,
the better chance of building a huge information base which may be used for gainful government
purposes, but this may also be an invitation for misuse, a temptation that may be too great for
some authorities to resist. There are no controls to guard against leakage of information.

Thus, AO 308 does not assure that personal information will be gathered only for unequivocally
specified purposes. The lack of proper safeguards may interfere with the individual’s liberty of
abode and travel by enabling authorities to track down his movement. The possibilities of
abuse and misuse threaten the very abuses that the Bill of Rights seeks to prevent.

The dissenters dismiss these dangers as speculative and hypothetical. But the Court will not be
true to its role as the guardian of the people’s liberty if it would not smother the sparks that
endanger their rights but wait for the fire to consume them.

We reject the argument that an individual has a reasonable expectation of privacy with regard to
the national ID and biometrics technology. Factors like customs, physical surroundings, and
practices may create or diminish this expectation. The use of biometrics and computer technology
in AO308 does not assure the individual of a reasonable expectation of privacy. As technology
advances, the level of reasonably expected privacy decreases. AO 308 is so widely drawn that a
minimum standard for a reasonable expectation of privacy regardless of technology used cannot
be inferred from it.
The OSG argues using the rational relationship test, saying that the purposes of AO 308 to speed
up government services, eradicate fraud by avoiding duplication of services, and generate
population data for development planning justify the incursions to the right of privacy for the
means are rationally related to the end. In Morfe v. Mutuc, we upheld RA 3019 as a valid police
power measure not violating right to privacy requiring public officers to annually disclose their
saln to promote morality in public administration and minimize opportunities for official
corruption.

But the same do not obtain here. RA 3019 is a statute, not an AO. RA 3019 is sufficiently detailed.
The law is clear on what practices were prohibited and it was narrowly drawn to avoid abuses.
While AO 308 has a worthy purpose, it is not narrowly drawn. When the integrity of a
fundamental right is at stake, this court will give the challeneged law, rule, or AO a stricter
scrutiny. It is not enough to invoke presumption of regularity or to prove that the act is not
irrational for a basic right to be diminished.

The right to privacy does not bar all incursions into individual privacy. It is not intended to stifle
scientific and technological advancements that enhance public service and the common good. It
merely requires that the law be narrowly focused and a compelling interest justify such
intrusions.

103. Socorro Ramirez v. CA, GR 93833, September 28, 1995, Kapunan, J. (Privacy of
communication and correspondence)
FACTS:
A civil case for damages was filed by Socorro Ramirez with the RTC alleging that private
respondent, Ester Garcia, in a confrontation in Ester’s office, allegedly vexed, insulted, and
humiliated her in a hostile and furious mood and in a manner offensive to Ramirez’ dignity and
personality contrary to morals, good customs, and public policy. Ramirez, in support of the claim,
produced a verbatim transcript of the event. The transcript was culled from a tape recording of
the confrontation made by Ramirez. Alleging that the act of secretly taping the confrontation was
illegal, Ester filed a criminal case with the RTC for violation of RA 4200, an act to prohibit and
penalize wire tapping etc. An information charged Ramirez of violation of said act. Petitioner
moved to quash the information. The RTC quashed the information, stating that RA 4200 refers to
taping of communication by a person other than a participant to the communication.

The CA held that the charge sufficiently constitute an offense in RA 4200. Hence this petition by
Ramirez. She asserts that RA 4200 does not apply to the taping of a private conversation by one
of the parties to the conversation.

ISSUE:
Whether an information charging a violation of RA 4200, Section 1, by stating that “Ramirez, not
being authorized by Ester S. Garcia to record the latter’s conversation with said accused, xxx, with
the use of a tape recorder secretly record the said conversation xxx” charges an offense.
HELD: YES.
Legislative intent is determined from the language of a law. Where it is clear, it is applied
according to its express terms. RA 4200 provides:
SECTION 1. It shall be unlawful for any person, not being authorized by all the parties to
any private communication or spoken word, to tap any wire or cable, or by using any other
device or arrangement, to secretly overhear, intercept, or record such communication or
spoken word by using a device commonly known as a dictaphone or dictagraph or
detectaphone or walkie-talkie or tape recorder, or however otherwise described.

The above clearly makes it illegal for any person not authorized by all parties to any private
communication to secretly record such communication by means of a tape recorder. The law
makes no distinction if the party is involved in the private communication or not. The intent to
punish all persons unauthorized to make such recording is found in the qualifier “any.” Thus, even
a person privy to a communication who records his private conversation with another without
knowledge of the latter qualifies as violator of RA 4200. The Senate Congressional Records
supports this conclusion.
“That is why when we take statements of persons, we say: "Please be informed that
whatever you say here may be used against you." That is fairness and that is what we
demand. Now, in spite of that warning, he makes damaging statements against his own
interest, well, he cannot complain any more. But if you are going to take a recording of the
observations and remarks of a person without him knowing that it is being taped or
recorded, without him knowing that what is being recorded may be used against him, I
think it is unfair.”
“If a party secretly records a public speech, he would be penalized under Sec.1? the speech
is public, but the recording is done secretly.”
“That particular aspect is not contemplated by the bill. It is the communication between
one person and another person – not between a speaker and a public.”

Thus, where the law makes no distinctions, one does not distinguish. The nature of the conversation
is immaterial. What RA 4200 penalizes are the acts of secretly overhearing, intercepting, or
recording private communications thru the enumerated devices therein. The mere allegation that
one made a secret recording of private communication thru tape recorder suffices to constitute an
offense under Sec.1.

The contention that private “communication” in Sec.1 does not include private “conversations”
narrows the meaning of “communication” to a point of absurdity. In its ordinary signification,
communication connotes the act of sharing or imparting, as in a conversation. Any doubts as to
the legislative body’s meaning of the phrase “private communication” are put to rest by the fact
that “conversation” and “communication” were interchangeably used by Senator Tañada in his
explanatory note to the bill.

104. Edgardo Gaanan v. IAC, GR L-69809, October 16, 1986, Gutierrez, Jr., J. (Privacy of
communication and correspondence)
FACTS:
Atty. Pintor and his client Montebon were in the living room of Montebon’s residence discussing
for the terms of the withdrawal of the complaint for direct assault they filed against Laconico.
After they decided on the proposed conditions, Atty. Pintor made a telephone call to Laconico.
Laconico also telephoned Gaanan, a lawyer, to come to his office to advise him on the settlement
of the direct assault case as his regular lawyer was on a business trip. Atty. Gaanan went to
Laconico’s office.

When Atty. Pintor called Laconico, Laconico requested Atty. Gaanan to secretly listen to the
telephone conversation through a telephone extension as to hear personally the proposed
conditions for the settlement. Atty. Pintor called 20 minutes later to ask if Laconico was agreeable
to the conditions. Laconico said yes. Pintor told him to give the money to his wife. Laconico, who
earlier alerted the criminal investigation service of the police insisted that Montebon personally
receive the money. When Pintor accepted it, he was arrested by police agents.

Gaanan executed an affidavit the next day stating that he heard Pintor demand P8k for the
withdrawal of the case for direct assault. Laconico attached the affidavit of Gaanan to his complaint
for robbery/extortion which he filed against Montebon. Since Gaanan listened to the telephone
conversation without Pintor’s consent, he charged Gaanan and Laconico of violating RA 4200,
Anti-Wiretapping Act.

The RTC found both guilty. The Intermediate Appellate Court (IAC) affirmed. Hence this petition
for certiorari.

ISSUE:
Whether Atty. Gaanan, listening in to the telephone conversation of Laconico and Atty. Pintor
without Pintor’s permission through a telephone extension, violated RA 4200, Anti-Wiretapping
Act.
HELD: NO.
RA 4200 provides:
"Section 1. It shall be unlawful for any person, not being authorized by all the parties to
any private communication or spoken word, to tap any wire or cable or by using any other
device or arrangement, to secretly overhear, intercept, or record such communication or
spoken word by using a device commonly known as a dictaphone or dictagraph or
detectaphone or walkie-talkie or taperecorder, or however otherwise described;

The issue is not the admissibility of evidence but whether there is violation of RA 4200. The
conversation between Atty. Pintor and Atty. Laconico was “private” in the sense that the words
uttered were made between one person and another as distinguished from words between a
speaker and a public. Only Laconico gave Gaanan authority to listen thru extension telephone line.
But affirmance of the criminal conviction would mean that a caller, by merely using a telephone
line, can force the listener to secrecy no matter how obscene, criminal, or annoying the call may
be.

Telephone users would often encounter “crossed lines” where an unwary citizen who happens to
pick up his telephone might overhear the details of a crime. He might hesitate to inform the police
if he might be accused under RA 4200. Surely the law never intended such mischievous results.

Is an extension of a telephone unit such a device or arrangement as would subject the user to
imprisonment? Is this included in the phrase “any other device or arrangement?” The law refers to
a "tap" of a wire or cable or the use of a "device or arrangement" for the purpose of secretly
overhearing, intercepting, or recording the communication. There must be either a physical
interruption through a wiretap or the deliberate installation of a device or arrangement in order to
overhear, intercept, or record the spoken words.

An extension telephone is not in the same category as a Dictaphone, dictagraph, or other devices
in Sec.1 as the use thereof cannot be considered as “tapping” the wire or cable of a telephone
line. The telephone extension in this case was not installed for that PURPOSE. It just happened
to be there for office use. It is a rule in STATCON that to determine the true intent of the
legislature, the clauses and phrases of the statute should not be taken as detached and isolated
expressions, but the whole and every part thereof must be considered.

The phrase “device or arrangement” in Sec.1 should be construed to comprehend instruments of


the same or similar nature, that is, instruments the use of which would be tantamount to tapping
the main line of a telephone. It refers to instruments whose installation cannot be presumed by the
parties being overhead since they are not of common usage and their purpose is precisely for
tapping. An extension telephone is very common. A person should safely presume that the party
he is calling probably has an extension telephone and he runs the risk of a third party listening.

Penal statutes must be construed strictly in favor of the accused. Thus, in case of doubt on whether
an extension telephone is included in “device or arrangement,” it must be construed as not
including such.

105. Felipe Navarro v. CA, GR 121087, August 26, 1999, Mendoza, J. (Privacy of
communication and correspondence)
FACTS:
Stanley Jalbuena and Enrique Lingan, reporters of the radio station DWTI in Lucena city, went to
Entertainment City following reports that it was showing nude dancers. After they seated
themselves at a table and ordered beer, a scantily clad dancer appeared on stage and began to
perform a strip act. As she removed her brassieres, Jalbuena took a picture. The floor manager,
Dante Liquin, with a security guard, Alex Sioco, approached Jalbuena and demanded to know why
he took a picture. Jalbuena replied: “Wala kang pakialam, because this is my job.” Alex pushed
Jalbuena towards the table as he warned him that he would kill him. Jalbuena saw that Alex was
about to pull out a gun, so he ran out joined with his companions.

Jalbuena and his companions went to the police station to report the matter. Three police, including
Navarro, were having drinks in front of the police station and asked Jalbuena and his companions
to join them. Jalbuena declined and went to desk officer Sgt. Añonuevo to report the incident.
Dante and Alex arrived. The latter talked with Navarro for 15 minutes. After, Navarro turned to
Jalbuena, pushed him to the wall, and said to him: “putaing ina, kinakalaban mo si Kabo Liquin,
anak yan ni Kabo Liquin, hindi mob a kilala?” Navarro pulled out his firearm, cocked it, and
pressed it on the face of Jalbuena and said: “Ano, uutasin na kita?”

Lingan intervened and said: “Huwag naming ganyan, pumarito kami para magpa-blotter, I am
here to mediate.” Navarro replied: “Walang press, press, mag-sampu pa kayo.” Navarro told
Añonuevo and told him to make a record of the behavior of Jalbuena and Linga. Lingan, angered,
said “O, di ilagay mo diyan.”
Lingan said “alisin mo yang baril at magsuntukan na lang tayo.” Navarro replied: “Ah ganoon?”
As Lingan turned away, Navarro hit him with the handle of his pistol above the left eyebrow.
Lingan fell. He tried to get up but Navarro punched him on the forehead. Jalbuena was made to
sign a blotter where it was put that Lingan “ang naghamon.” But he was able to only write his
printed name as his hand was trembling. Lingan died.

Unknown to Navarro, Jalbuena was able to record on tape the exchange between Navarro and
Lingan. Navarro was charged with homicide. The RTC convicted him. The CA affirmed. Hence
this appeal.

ISSUE:
Whether a voice recording by Jalbuena of the altercation between Lingan and Navarro where it
was heard that Lingan had provoked Navarro into a fist fight and where thereafter a scuffle ensued,
which as testified in court was actually Navarro hitting Lingan on the head with the butt of his gun
causing his death, is admissible in evidence against Navarro under RA 4200 in a prosecution of
the latter for homicide.
HELD: YES.
Navarro questions the credibility of the testimony of Jalbuena on the ground that he was a biased
witness who had a grudge against him. But this alone does not make a witness unreliable. Trial
courts are competent to determine whether his testimony should be given credence.

Jalbuena’s testimony is confirmed by the voice recording he made. It may be asked whether the
tape is admissible in view of RA 4200 which prohibits tapping. It is admissible. RA 4200 reads:
SECTION 4. Any communication or spoken word, or the existence, contents, substance,
purport, effect, or meaning of the same or any part thereof, or any information therein
contained obtained or secured by any person in violation of the preceding sections of this
Act shall not be admissible in evidence in any judicial, quasi-judicial, legislative or
administrative hearing or investigation.

The law prohibits the overhearing, intercepting, or recording of private communications. Since
the exchange between Navarro and Lingan was NOT PRIATE, its tape recording is not
prohibited. It is also duly authenticated by witness (Jalbuena’s) testimony since 1) he personally
recorded the convo, 2) the tape played in court was the one he recorded, and 3) the voices on the
tape are those of the persons such are claimed to belong.

The recording established that there was a heated exchange and some form of violence occurred
between Navarro and Lingan, with the latter getting the worst of it.

FREEDOM OF EXPRESSION
106. US v. Felipe Bustos et al., GR L-12592, March 08, 1918, Malcolm, J. (Freedom of
Expression; privileged communication- made to whom, malice or GF, publicity; qualified
and absolute privilege; no qualified privilege if malicious)
FACTS:
In 1915, citizens of Pampanga Province assembled and signed a petition to the Executive Secretary
(ES) thru the law office of Crossfield & O’Brien, and 5 people signed the affidavits, charging
Roman Punsalan, justice of the peace, with malfeasance in office and asking for his removal. The
law office Crossfield submitted this petition and affidavits with a complaint to the ES. The petition
was signed by 34 citizens. They charge Roman Punsalan with:
1) Francisca, wanting to file a complaint against Mariano, visited Roman, who told her that he
would make the complaint for P5. After, he said he would take P3 which she paid. He also kept
her in the house for 4 days as servant and took from her 2 chickens and 12 gandus.
2) Valentin, interested in a case, went to see Roman who told him that if he wanted to win, he must
give Roman P50. Valentin did not have this, thus he lost the case. Roman told him that he could
still win if he gave P50.
3) Leoncio, who filed a complaint for assault against 4 persons, was called by Roman to his house
where he gave Leoncio P30. The complaint was then shelved.

The ES referred the complaint to the judge of first instance for investigation and proper action.
Roman denied the charges. The judge found the first count not proved but counts 2 and 3
established. The judge recommended to the governor general that Roman be removed from his
position as justice of the peace.

Roman moved for new trial. The judge granted and reopened the hearing. Documents were
introduced, including a letter by the municipal president asserting that Roman was the victim of
prosecution and that the charges were for personal reasons. The judge ordered suppression of the
charges and acquitted Roman. Complainants appealed to the governor general.

Thus, criminal action against defendants was instituted. It stated that with malicious intent to
prejudice and defame Roman Punsalan, wrote and published a writing which was false, malicious,
defamatory, and libelous against Roman. The defendants, except some, were found guilty by the
trial court.

ISSUE:
Whether a petition against justice of the peace Roman Punsalan with malfeasance in office and
asking for his removal containing charges and claims of acts of corruption and solicitation of
bribery, which were dismissed where Roman was acquitted, is libelous and beyond the protection
of freedom of speech, expression, and petition where the petitioners had probable cause to believe
that their charges were true.
HELD: NO.
The administrative proceedings were the basis of the accusation, the information, the evidence,
and the judgment. There is nothing more unjust than to pick out certain words which standing by
themselves and unexplained are libelous and then by shutting off all knowledge of facts which
would justify these words, to convict the accused.

Freedom of speech was unknown in PH before 1900. A primary cause for revolt was then made.
Jose Rizal, in “The PH a Century Hence,” stated that the minister who wants his reforms to be
reforms must begin by declaring the press in PH free. The Malolos Constitution by Revolutionary
Congress, in its bill of rights, guarded freedom of speech and press and assembly and petition.
President McKinley in the Magna Carta of PH Liberty, the Instruction to the Second PH
Commission of April 7, 1900, laying down the inviolable rule that “no law shall be passed
abridging the freedom of speech or of the press or of the rights of the people to peaceably assemble
and petition the government for a redress of grievances.”

The PH Bill, Act of Congress of July 1, 1902, the Jones Law, Act of Congress of August 29, 1916,
as organic acts of PH, continued this guaranty. The words are the counterpart of the first
amendment to the US Constitution. The guaranty is part and parcel of the PH Constitution. These
words in the PH Bill of Rights are not mere verbiage. It carries with it all applicable jurisprudence
of English and American Constitutional cases.

The interest of society and maintenance of good government demand a full discussion of public
affairs. Liberty to comment on the conduct of public men is a scalpel in free speech. While men in
public life may suffer under an unjust accusation, the wound can be assuaged with the balm of a
clear conscience. A public officer must not be too thin-skinned with comments upon his official
acts. Of course, criticism does not authorize defamation. Nonetheless, as the individual is less than
the State, so must expected criticism be born for the common good. Public opinion should be the
constant source of liberty and democracy.

The guarantees of a free speech and free press include the right to criticize judicial conduct. The
administration of law is a matter of vital public concern. If people cannot criticize a judge like
other public officers, public opinion will be muzzled. It is a duty which everyone owes to society
or to the State to assist in the investigation of any alleged misconduct. "The people are not obliged
to speak of the conduct of their officials in whispers or with bated breath in a free government, but
only in a despotism." – Justice Gayner

The right to assemble and petition is a necessary consequence of republican institution and the
complement of the right to free speech. Assembly means a right to meet peaceably for consultation
about public affairs. Petition means any person or group of persons can apply, without fear of
penalty, to the appropriate branch or office of the government for a redress of grievances. But the
persons assembling and petitioning must assume responsibility for the charges made.

The doctrine of privileged communications looks to the free and unfettered administration of
justice though, as an incidental result, may afford immunity to the evil-disposed and malignant
slanderer. Privilege is either absolute or qualified. We are not concerned with the first. As to
qualified privilege, it is as the words suggest a prima facie privilege which may be lost by
proof of malice. "A communication made bona fide upon any subject-matter in which the party
communicating has an interest, or in reference to which he has a duty, is privileged, if made to a
person having a corresponding interest or duty, although it contained criminatory matter which
without this privilege would be slanderous and actionable."

An application of this qualified privilege is a complaint in GF without malice regarding the


character or conduct of a public official when addressed to an officer or board having an interest
or duty in the matter. Even if the statements are false, if there is probable cause for belief in their
truthfulness and the charge is in GF, the mantle of privilege may still cover the mistake of
the individual. But the statements must be made under an honest sense of duty. Personal injury
is not necessary. All persons have an interest in the administration of justice and public affairs.
The duty in which a party is privileged is sufficient if it is social or moral in nature and the person
in GF believes that he is acting in pursuance thereof although he is mistaken. The privilege is not
defeated by the mere fact that the communication is intemperate.

A further element concerns the person to whom the complaint should be made. if a party applies
to the wrong person through some honest mistake, such will not take the case out of the privilege.

Malice can be presumed from defamatory words. Privilege destroy that presumption. The
onus of proving malice then lies on plaintiff. Falsehood and the absence of probable cause will
amount to proof of malice. A privileged communication should NOT be subjected to
microscopic examination to discover malice or falsity. Excessive scrutiny would defeat the
protection over privileged communications. The ultimate test is that of bona fides.

Having emphasized that our Libel Law as a statute must be construed with reference to the
guarantees of our Organic Law and having sketched the doctrine of privilege, we apply them to
the facts.

If the words said in the information were of a private person, it might be libelous per se. The
charges under certain conditions might conviction one of libel of a government official. Generally,
words imputing to a judge corruption in his office are actionable. But this case is not simply of a
direct and vicious accusation published in the press, but of charges predicated on affidavits made
to the proper official and thus qualifiedly privileged. Malice has not been proved. Although
the charges are probably not true, they were believed to be true by petitioners. They acted in GF.
There is probable cause for them to think that malfeasance in office is apparent. The motives, to
secure removal from office of one thought to be venal, were justifiable.

No undue publicity was given to the petition. The manner of commenting on the conduct of
Romano was proper. The charges were submitted thru lawyers to the proper functionary, the ES.

In US v. Julio Bustos, there was more publicity. There, the statements were wild with no basis in
fact made against members of the judiciary, to persons who could not furnish protection.
Malicious and untrue communications are not privileged.

107. Ayer Productions Pty. Ltd. V. Hon. Capulong, GR 82380, April 29, 1988 (Freedom of
Expression; public figure has less privacy rights- 3 reasons; film about EDSA is not about
Enrile’s life but about a significant historical event, and to exclude him would be grossly
unhistorical; movie must be fairly truthful- equilibrium between privacy and expression)
FACTS:
Petitioner Hal McElroy, Australian film maker, and his movie production company Ayer
Productions, envisioned in 1987 the filming for commercial viewing for PH and international
release the historic and peaceful struggle of the Filipinos at EDSA. Petitioners discussed the project
with local movie producer Lope Juban, who suggested they consult with the proper agencies and
with Gen. Fidel Ramos and Sen. Juan Ponce Enrile, who had major roles in EDSA.

The proposed movie, “The Four Day Revolution,” was endorsed by the Movie Television Review
and Classification Board and other agencies. Fidel Ramos also approved it. Hal McElroy informed
private respondent Enrile about the projected movie enclosing a synopsis of it. The movie would
essentially be a reenactment of EDSA.

Enrile replied that he would not approve the use, appropriation, reproduction, or exhibition of his
name, picture, or any member of his family in any cinema production, film, or other medium of
commercial exploitation. He advised petitioners that in the movie, there should be no reference
about him. Petitioners acceded and deleted from the movie script the name of Enrile.

On Feb 23, 1988, Enrile filed a complaint with TRO with the RTC, seeking to enjoin petitioners
from producing the movie, alleging that its production without Enrile’s consent and over his
objection constitutes a violation of his privacy. The RTC issued the TRO and set the case for
hearing. Hal McElroy and Ayer moved to dismiss contending that the movie would not involve
Enrile’s private life nor of his family and that an injunction would be a prior restraint on their right
of free expression. The RTC issued the preliminary injunction. Hence these certiorari petition.

ISSUE:
Whether the portrayal of Juan Ponce Enrile’s participation in the EDSA Revolution against his
will violates his right to privacy vis-à-vis the movie producers’ right to freedom of expression and
speech.
HELD:NO.
It is claimed that petitioners’ production of the movie is in exercise of freedom of speech and
expression. Enrile asserts his right to privacy.

The freedom of speech and expression includes the freedom to film and produce motion
pictures and to exhibit such in theaters or to diffuse them through television. Along with the
press, radio and television, motion pictures is a principal medium of mass communication for
information, education, and entertainment. Motion pictures are important both as a medium for
communication of ideas and expression of artistic impulse.

This freedom is available both to locally-owned and to foreign-owned motion picture


companies. That the production of movies is a commercial activity is not a disqualification for
availing freedom of speech and expression.

The counter-balancing claim of Enrile is to a right of privacy. Constitutional and statutory law
does include such right. It is left to case law to mark out the precise scope and content of this right
in different situations. The right to privacy, like of free expression, is not an absolute right. A
limited intrusion into a person’s privacy is permissible where that person is a public figure and
the information sought to be elicited from him or to be published about him are matters of a public
character. Thus, this right cannot be invoked to resist publication of matters of public interest.
The right sought to be protected by the right to privacy Is the right to be free from unwarranted
publicity, from wrongful publicizing of the private affairs and activities of a person outside
the realm of legitimate public concern.

Enrile relies on Lagunzad v. Vda. De Gonzales. It involved a suit for enforcement of a licensing
agreement between a movie producer as licensee and the widow and family of the late Moises
Padilla as licensors. It gave the licensee the right to produce a movie on Moises’ life, a mayoralty
candidate killed by Governor Lacson. The SC enforced the licensing agreement against the
licensee who produced and exhibited the movie but refused to pay the royalties. The averment
there that the respondents did not have property right over the life of Moises as the latter was a
public figure was rejected. It was said that being a public figure does not automatically destroy in
toto a person’s right to privacy. The right to invade privacy to disseminate public information does
not extend to a fictional representation of a person no matter how public a figure such may be.
While it exerted efforts to present the true life story of Moises, the producer had included romance
in the film as without it, the story would be drab. While the right of free expression occupies a
preferred position in the hierarchy of civil liberties, the doctrine of clear and present danger is a
limit. Another criterion is the balancing-of-interests test. This requires the court to take a detailed
consideration of the interplay of interests observable in a given situation. Taking into account the
interplay of the interests of privacy and expression, under the circumstances therein, the validity
of the agreement as not infringing freedom of expression was upheld as the limits of the freedom
are reached when expression touches upon matters of essentially private concern.

But here, a different conclusion must be reached as the movie does not violate Enrile’s right to
privacy.

What is involved here is a prior and direct restraint of respondent judge on the speech and
expression of petitioners. Hon. Capulong restrained them from filming the movie. In Lagunzad,
there was no prior restraint. Because of the preferred character of the right to freedom of speech
and expression, a weighty presumption of invalidity vitiates measures of prior restraint upon the
exercise of such freedoms. Capulong should have stayed his hand instead of issuing a TRO 1 day
after the filing of a complaint and issuing an injunction 20 days later, for the movie was not yet
completed and not exhibited to any audience. Neither Enrile nor Capulong knew what the
completed film would look like. Thus, there was NO clear and present danger of any violation
of privacy that Enrile could assert.

The subject matter of the movie, EDSA, is one of public interest and concern. It is of
international interest. It relates to a highly critical stage in the history of PH and must be regarded
as having passed into the public domain and an appropriate subject for speech and expression and
coverage by any form of mass media. The subject matter does not relate to the individual life
and certainly not to the private life of Enrile. Unlike in Lagunzad, which concerned the life
story of Moises Padilla necessarily including his immediate family, EDSA is not a film biography
of Enrile. The movie is not principally about nor focused upon the man Enrile, but it is
compelled, if it is to be historical, to refer to the role played by Enrile in EDSA.

The extent of the intrusion upon Enrile’s life would thus be LIMITED in character. Such extent
may be generally described as such intrusion as is reasonably necessary to keep that film a
truthful HISTORICAL account. Enrile does not claim that petitioners threatened to depict in the
movie any part of Enrile’s private life or that of his family.

At all relevant times, clearly of public concern, that petitioners propose to film were taking place,
Enrile is a PUBLIC FIGURE. A public figure is a person who, by his accomplishments, fame, or
mode of living, or by adopting a profession or calling which gives the public a legitimate interest
in his doings, his affairs, and his character, has become a public personage. He is a celebrity.
Included are those who have achieved some degree of reputation by appearing before the
public, like an actor, professional baseball player, a pugilist, or other entertainer, war heroes,
famous inventors and explorers, etc. It includes anyone who has arrived at a position where public
attention is focused upon him as a person.

Such public figures were held to have lost, to some extent at least, their right to privacy. The
reasons are: 1) They had sought publicity and consented to it, and so could not complain when
they received it; 2) their personalities and affairs had already become public, and could no longer
be regarded as their own private business; 3) the press had a privilege, under the Constitution,
to inform the public about those who have become legitimate matters of public interest.

The privilege of giving publicity to news and matters of public interest arise out of the desire and
right of the public to know what is going on in the world, and the freedom of the press and
other agencies of information to tell it. “News” includes all events and items of information which
are out of the ordinary humdrum routine, and which have 'that indefinable quality of information
which arouses public attention.'

The privilege of enlightening the public was not, however, limited to the dissemination of news in
the sense of current events. It extended also to information or education, or even entertainment and
amusement, by books, articles, pictures, films and broadcasts concerning interesting phases of
human activity in general, as well as the reproduction of the public scene in newsreels and
travelogues

Enrile is a public figure because of his participation as a principal actor in the events of change
of government in EDSA. Because of his major participation, a film reenactment of the peaceful
revolution that fails to make reference to his role would be grossly unhistorical. The right of
privacy of a public figure is narrower than an ordinary citizen. Enrile has not retired into the
seclusion of simple private citizenship. He continues to be a public figure. He sits in a very public
place, in the Senate.

The equilibrium here between the freedom of speech and expression and the right to privacy may
be marked out in terms of a requirement that the proposed movie must be fairely TRUTHFUL
and HISTORICAL in its presentation of events. There must be no reckless disregard of truth.
There must also be no presentation of the private life of Enrile and a revelation of intimate or
embarrassing personal facts. The movie must not enter into “matters of essentially private
concern.”

108. Manuel Lagunzad v. Maria Soto Vda. De Gonzales, GR L-32066, August 06, 1979,
Melencio-Herrera, J. (Freedom of Expression; the limits of freedom of expression are
reached when expression touches upon matters of essentially private concern; public figure
does not shed privacy merely due to being public; right to invade privacy to disseminate
matters of public concern does not extend to fictional representations)
FACTS:
Lagunzad, a newspaperman, began the production of a movie entitled “The Moises Padilla Story”
under his own business outfit, “MML Productions.” It was based on an unpublished book of Atty.
Rodriguez the rights to which Lagunzad purchased for P2k. The book narrates the events which
culminated in the murder of Moises Padilla, who was then a mayoralty candidate of the
Nacionalista Party, minority party, during the November 1951 elections. Governor Lacson,
member of the LP then in power and his men were convicted for that murder. In the book, Moises
is portrayed as a martyr.

Although the emphasis in the movie was the public life of Moises, there were portions which dealt
with his private and family life, including in some scenes of his mother, Maria Soto, and of Auring,
his girlfriend. The movie was scheduled for showing at the latest before the November 1961
elections.

Petitioner received a telephone call from Nelly, half-sister of Moises, objecting to the movie and
the “exploitation” of his life. Nelly and her mother, Maria Soto, demanded for changes,
corrections, and deletions in the movie. Lagunzad says he acceded as he had already invested
heavily in the movie. After some bargaining as to the amount to be paid (P50k then P20k),
Lagunzad and Maria, represented by her daughters and Atty. Rodriguez, executed a “Licensing
Agreement.” The licensor is Maria. It granted authority thru licensor to Lagunzad, licensee, to use
the life story of Moises to produce the movie for the consideration of P20k and royalty of 2.5% of
gross income derived from the movie. There was a condition that licensee shall change, delete, or
correct portions of the movie as the licensor may require.

Lagunzad claims that he was pressured into signing the agreement as he was threatened that Nelly
would call a press conference declaring the whole picture a fake or hoax. Lagunzad paid P5k but
claims he did so not pursuant to the agreement but to placate Maria Soto.

On October 14, 1961, the movie was completed. On October 16, it was shown in different theaters
all over the country. As Lagunzad refused to pay any additional amounts pursuant to the
agreement, Maria instituted this suit against him praying for the payments.

Lagunzad claims that the episodes in the life of Moises in the movie were matters of public
knowledge and occurred when Moises was a public figure and that Maria has no property right
over those incidents. He signed the agreement without valid cause or consideration and thru threat.
He prayed that the agreement be declared void.

The RTC ordered Lagunzad to pay the amounts in the agreement and 20% atty’s fees. The CA
affirmed. Hence this petition for certiorari. Lagunzad argued that the movie production was in
exercise of his constitutional right to freedom of expression and the licensing agreement restrains
freedom of speech and of the press.

ISSUE:
Whether Lagunzad, a movie producer, can produce a movie about the life-story of a deceased
person with certain fictional insertions and including depictions of the deceased Moises Padilla’s
family and heirs disregarding the agreement between him and the heirs for the payment of royalties
and P20k for the license to produce such movie on the ground that the heirs do not have the
property right to the life of Moises as such life, since Moises was a mayoralty candidate who was
murdered, is of public knowledge.
HELD: NO.
While Lagunzad purchased the rights to the book entitled “The Moises Padilla Story,” that did not
dispense with the need for prior consent and authority from the deceased heirs to portray publicly
episodes in said deceased’s life and of his mother and family. A privilege may be given the
surviving relatives of a deceased person to protect his memory, but the privilege exists for the
benefit of the living, to protect their feelings and to prevent a violation of their own rights in the
character and memory of the deceased.

The contention that Maria did not have property right over the life of Moises since Moises was a
public figure is not well taken. Being a public figure ipso facto does not automatically destroy
in toto a person's right to privacy. The right to invade a person's privacy to disseminate public
information does not extend to a fictional or novelized representation of a person, no matter
how public a figure he or she may be. Here, while it is true that petitioner exerted efforts to present
a true-to-life story of Moises Padilla, petitioner admits that he included a little romance in the
film because without it, it would be a drab story of torture and brutality.

The contention that the Licensing Agreement infringes on freedom of speech and of the press is
without merit. Freedom of expression occupies a preferred position in the hierarchy of civil
liberties. But it is not without limitations. While “no law may be passed abridging the freedom of
speech and of the press,” the realities of life in a complex society preclude a literal
interpretation. Freedom of expression is NOT absolute. There are other societal values that press
for recognition.

The clear and present danger rule is such a limitation. Another criterion for permissible limitation
on freedom of speech and of the press, including such vehicles of mass media as radio, television,
and movies, is the balancing-of-interests test. This principle requires a court to take conscious
and detailed consideration of the interplay of interests observable in a given situation or type
of situation.

Here, the interests are the right to privacy and freedom of expression. Taking into account the
interplay of those interests, under the particular circumstances, and considering the obligations
assumed in the Licensing Agreement, the validity of such agreement will have to be upheld
because the LIMITS of freedom of expression are reached when expression touches upon
MATTERS OF ESSENTIALLY PRIVATE CONCERN.

109. Erwin Tulfo v. People, GR 161032, September 16, 2008, Velasco, Jr., J. (Freedom of
Expression)
FACTS:
On the complaint of Atty. Carlos So of the BoC, 4 separate informations were filed against Tulfo,
as author, Susan Cambri, as managing editor, Rey Salao, as national editor, Jocelyn Barlizo, as
city editor, and Philip Pichay, as president of the Carlo Publishing House, Inc., of the daily tabloid
“Remate,” with the crime of libel in connection with the publication of the articles in the column
“Direct Hit.” It stated: “Ito palang si Atty. So ng BoC ang pinakamayaman na yata na government
official sa buong bansa sa pangungurakot lamang diyan sa south harbor. Hindi matibag ang gagong
attorney dahil malakas daw ito sa Iglesia ni Kristo.”- May 11, 1999 issue.
“Daan-daang libong piso ang kinikita ng masiba at matakaw na si So sa mga importer na ayaw
ideklara ang totoong laman ng mga container para makaiwas sa pagbayad ng malaking customs
duties at taxes.” – May 12, 1000 issue.

“Tulad ni Atty. So ng BoC, saksakan din ng lakas itong si Daniel Aquin ng Presidential Anti-
Smuggling Unit na nakatalaga sa South Harbor. – May 19,1999 issue.

“Nag file ng P10M na libel suit itong si Atty. So laban sa inyong lingkod at ilang opisyales ng
Remate sa Pasay City Court. Nagalit itong tarantadong si Atty. So dahil binanatan ko siya at
inexpose ang kagaguhan niya sa BoC. -June 25, 1999 issue.

The following were admitted by petitioners: (1) that during the four dates of the publication of the
questioned articles, the complaining witness was not assigned at South Harbor; (2) that the accused
and complaining witness did not know each other during all the time of the four dates of
publication; (3) that Remate is a newspaper/tabloid of general circulation in the Philippines; (4)
the existence and genuineness of the Remate newspaper; (5) the column therein and its authorship
and the alleged libelous statement as well as the editorial post containing the designated positions
of the other accused; and (6) the prosecution's qualified admission that it is the duty of media
persons to expose corruption.

Tulfo claims that he did not write the articles with malice. He did not know or meet Atty. So before
their publication. He testified that his criticism of a certain Atty. So of the south harbor was not
directed against complainant, but against one Atty. “Ding” So there. He claims that it was the
practice of some people to use other people’s names to advance their corrupt practices. He did not
do research on Atty. So before the articles because as columnist, he had to rely on his source, and
he had several sources in the BoC.

The other officials testified that Tulfo had autonomy in writing the column. He could write what
he wanted to, and the vice-president for editorials is the one who would decide what articles are to
be published and what are not.

The RTC convicted them of libel. The CA affirmed the conviction. Hence this appeal.

ISSUE:
Whether articles published by Tulfo in Remate Tabloid imputing corruption to Atty. So of the
Bureau of Customs in South Harbor by saying that he gets payment from those who wish to
underdeclare their importations to avoid customs taxes, stating that Atty. So is an embarrassment
to his religion for being so corrupt, and, after the libel case was filed, stating that Atty. So “nagalit
ang tarantadong ito dahil binanatan ko siya at inexpose ang kagaguhan niya sa BoC,” are libelous.
HELD: YES.
Tulfo claims that the articles fall under “qualifiedly privileged communication” under Borjal v.
CA and the presumption of malice in Art.354 of the RPC does not apply. But Borjal was a civil
action for damages on libel, not criminal. There was no sufficient identification of the complainant
there. The subject in Borjal was a private citizen. The articles of Art Borjal were “fair
commentaries” on matters of public interest.
1. Freedom of the press v. Responsibility of the press
Freedom of the press is not absolute. Its exercise comes with an equal burden of responsible
exercise of that right. The Journalist’s Code of Ethics adopted by the National Union of
Journalists of the PH shows that the press recognized that it has standards to follow in the exercise
of press freedom. It states that journalists “recognize the duty to air the other side and the duty to
correct substantive errors promptly.” Journalists “shall presume persons accused of a crime as
innocent until otherwise proven.”

Tulfo did not follow this Code and exercised his journalistic freedom responsibly. He stated that
Atty. So was involved in criminal activities and called him an embarrassment to his INC religion
(Ikaw na yata ang pinakagago at magnanakaw sa miyembro nito.). When Atty. So filed the libel
case, he challeneged him (Nagalit itong tarantadong…). Tulfo admitted that he did not personally
know Atty. So nor had he met him before the publication and that he did not make a more in-depth
research of his allegations. He claims that someone else may have been using the name of Atty.
So, and it is this person who was his target. But this weakens his case, for even with knowledge
that he may be in error, he made no effort to verify the info given by his source.

The RTC found the accusations false. Tulfo argues that falsity does not affect the articles’
privileged character. While falsity does not prove malice, it cannot be said that a false article
accusing a public figure would always be qualifiedly privileged. In Borjal, the exercise of press
freedom must be done “consistent with good faith and reasonable care.” This was abandoned
by Tulfo. This is no case of honest mistake but of a journalist abdicating his responsibility to verify
his story and instead misinforming the public. Journalists may be allowed an adequate margin of
error in the exercise of their profession, but this margin does not expand to cover every defamatory
statement or a total abandonment of responsibility. While Borjal expanded qualifiedly privileged
communications beyond that given in Art. 354 of the RPC, this does not cover Tulfo.

This expansion speaks of fair commentaries on matters of public interest. While Borjal placed
this within qualified privileged communication, it still allows a discreditable imputation to be
actionable if it is a false allegation of fact or comment based on a false supposition. If the comment
is only opinion based on established facts, then it is immaterial if it happens to be wrong. The
RTC found the accusations false and that Tulfo did not very his info. His articles had no details or
acts committed by Atty. So to prove that he was indeed a corrupt public official. These were
unsubstantiated attacks and cannot be countenanced simply because the target was a public
official. It is not demanded of Tulfo to reveal his source as confidentiality thereof is respected.
What cannot be accepted are journalists making no efforts to very info given by a source and using
these to throw wild accusations and besmirch the name of possibly an innocent person. Journalists
have a responsibility to tell the truth. They must have foundation for their reports. Their reports
must be warranted by facts.

In Art. 354 of the RPC, a qualified privileged communication is exempt from the presumption of
malice if these elements are present: (a) That it is a fair and true report of a judicial, legislative,
or other official proceedings which are not of confidential nature, or of a statement, report or
speech delivered in said proceedings, or of any other act performed by a public officer in the
exercise of his functions; (b) That it is made in good faith; and (c) That it is without any
comments or remarks.
The articles are not fair and true reports. They are baseless accusations backed up by the word of
one unnamed source. Good faith is lacking since Tulfo did not verify his story. He attacked the
character of Atty. So calling him a disgrace to his religion and legal profession. None of the
elements of Art. 354 are present, thus the articles are not qualified privileged communication in
the RPC.

“Fair” is defined as “having the qualities of impartiality and honesty.” “True” is “as conformable
to fact; correct; exact; genuine; honest.

2. Even if arguendo the articles were covered by qualified privileged communication, this would
still not protect Tulfo.
He claims that the presumption of malice under Art.354 is not present. The prosecution must thus
prove malice in fact.

Actual malice is the knowledge that a defamatory statement was false or with reckless disregard
of whether it was false or not. The prosecution proved actual malice; Tulfo did not know Atty. So
and he did not verify his info. There was no evidence to show that the accusations were true. The
fact that Tulfo published another article after the filing of the libel suit is further evidence of a
malicious design to injure the victim. This is a clear indication of his intent to malign Atty. So,
no matter the cost, and is proof of malice.

3. The prosecution presented evidence that there is only one Atty. So in the BoC. And most
damning is Tulfo’s last column referring to the libel suit against him by Atty. So. It is an admission
that Atty. So was in fact the target of his attacks. He cannot now point to a putative “Atty. Ding
So” or someone else using such name as the real subject of his attacks when he did not investigate
the existence or non-existence of an Atty. So at south harbor nor his alleged corrupt acts.

4. Cambri, Salao, Barlizo, and Pichay claim they had no participation in writing the articles and
are not liable. Art. 360 of the RPC lists the persons liable for libel.

Art. 360. Persons responsible. — Any person who shall publish, exhibit, or cause the
publication or exhibition of any defamation in writing or by similar means, shall be
responsible for the same. The author or editor of a book or pamphlet, or the editor or
business manager of a daily newspaper, magazine or serial publication, shall be responsible
for the defamations contained therein to the same extent as if he were the author thereof
This does not provide absence of participation as a defense. It is not a matter of whether they
conspired in publishing the articles, but the law simply states that they are liable as if they were
the author.

An editor or manager of a newspaper who has active charge and control of its management,
conduct, and policy, is general held to be equally liable with the owner of the publication of a
libelous article. It is their duty to know and control the contents of the paper. It is immaterial
whether they knew the contents of the publication. They cannot say they are not liable because
they did not fulfill their responsibilities as editors and publishers.
While petitioners are guilty, the punishment must be tempered with justice. They are to be
punished for libel for the first time. Freedom of expression and of the press may not be
unrestrained, but neither must it be reined in too harshly. Thus, considering the necessity of a free
press balanced with the necessity of a responsible press, the penalty of fine of P6k for each count
of libel with subsidiary imprisonment should suffice. They were ordered to pay jointly and
severally P1M as moral damages to Atty. So.

110. Arturo Borjal v. CA, GR 126466, January 14, 1999, Bellosillo, J. (Freedom of
Expression; in libel, third person must be able to identify the person defamed; fair
commentaries on matters of public interest are also qualifiedly privileged; margin of error
allowed, else self-censorship)
FACTS:
Borjal and Maximo Soliven are among the incorporators of Philippines Today, Inc. (PTI), not
PhilSTAR Daily, Inc., owner of The Philippine Star, a daily newspaper. Borjal was its President
while Soliven is the Publisher and Chairman of its Editorial Board. Borjal is among the regular
writers who run the column “Jaywalker.”

Private respondent Francisco Wenceslao is a civil engineer, businessman, business consultant, and
journalist by profession. He served in 1988 as technical adviser of Cong. Fabian Sison, then
Chairman of the HoR Sub-Committee on Industrial Policy.

During the congressional hearings on the transport crisis in September 1988 by said subcommittee,
those who attended agreed to organize the First National Conference on Land Transportation
(FNCLT) to be participated in by the private sector in the transport industry and government
agencies concerned to find ways to solve the transport crisis. The aim of FNCLT was to draft an
omnibus bill that would embody a long-term land transport policy for presentation to Congress.
The FNCLT, estimated to cost P1.815M, would be funded through solicitations from various
sponsors like government ageincies, private organizations, transport firms, etc.

On February 29, 1989, at a meeting of FNCLT, Wenceslao was elected Executive Director. He
thus wrote numerous solicitation letters to the business community to support the FNCLT. From
May to July 1989, various articles written by Borjal were published in his column Jaywalker,
dealing with the alleged anomalous activities of an “organizer of a conference” without naming
or identifying Wenceslao nor the FNCLT.

Another self-proclaimed 'hero' of the EDSA Revolution goes around organizing 'seminars and
conferences' for a huge fee. This is a simple ploy coated in jazzy letterheads and slick prose. The
'hero' has the gall to solicit fees from anybody with bucks to spare. Transport Sec. Ray Reyes asked
that his name be stricken off from the letterheads the “hero has been using to implement one of his
pet seminars. -May 31, 1989 issue.

Another questionable portion of the so-called conference is its unauthorized use of the names of
President Aquino and Secretary Ray Reyes. The conference program being circulated claims that
President Aquino and Reyes will be main speakers in the conference. Yet, the word is that Cory
and Reyes have not accepted the invitation to appear in this confab. Ray Reyes even says that the
conference should be unmasked as a moneymaking gimmick. -June 9, 1989 issue.
Similar issues were published on June 19 and 21 and July 3.

The scheming 'organizer' we have been writing about seems to have been spreading his wings too
far. A congressional source has informed the Jaywalker that the schemer once worked for a
congressman from the North as some sort of a consultant on economic affairs. -June 22, 1989.

Wenceslao sent a letter to the Philippine Star insisting that he was the organizer alluded to. He
refuted the matters contained in Borjal’s columns and openly challenged him that he is prepared
to relinquish his position if he is proven to have misappropriated even 1 peso of FNCLT money.
Thereafter, he filed a complaint with the National Press Club (NPC) against Borjal for unethical
conduct, accusing Borjal of using his column as leverage to get contracts for his public relations
firm, AA Borjal Associates. Wenceslao also filed a criminal case for libel against Borjal and
Soliven, which was dismissed by the prosecutor, DOJ, and OP.

Wenceslao then filed a civil case for damages on libel in this case. The RTC decided in favor of
Wenceslao. The CA affirmed, but just reduced damages. Hence this appeal by petitioners Borjal.

ISSUE:
Whether Borjal, publishing articles about the “organizer” of the FNCLT regarding such person’s
corruption and scheming without explicitly identifying the person, and where the
victim,Wenceslao, who felt that he was the one being alluded to by the articles filed a libel charge
against Borjal is a private person assigned as executive director of the FNCLT the functions of
which are to address the transport crisis in the country, is liable for libel.
HELD: NO.
1. Not identifiable.
To maintain a libel suit, the victim must be identifiable although not necessary that he be named.
It is not sufficient that the victim recognized himself as the person attacked, but that a third person
could identify him as the object of the libelous publication. These have not been met.

The articles of Borjal do not identify Wenceslao as the organizer of the conference. There were
millions of “heroes” of the EDSA revolution and anyone of them could be “self-proclaimed” or an
“organizer of seminars and conferences.” The FNCLT letterheads did not disclose the identity of
the conference organizer since these contained only an enumeration of names where Wenceslao
was described as Executive Director and Spokesman and not as a conference organizer. The
FNCLT had several organizers and he was only a part of the organization. He even asked Borjal
if he (Wenceslao) was the one referred to in the articles.

Identification is grossly inadequate when even the alleged offended party is himself unsure that he
was the object of the verbal attack. It was actually Wenceslao’s letter to the editor that revealed
his identity.

2. We now assume arguendo that Wenceslao was sufficiently identified. The RTC ruled that the
writings did not fall under any of the exceptions in Art. 354 of the RPC as it was neither “private
communications” nor “fair and true report without any comments or remarks.” But this is incorrect.
A privileged communication may either be absolutely or qualifiedly privileged. Absolutely
privileged communications are those which are not actionable even if the author acted in bad faith.
An example is Sec.11, Art. VI of the Constitution exempting members of Congress from liability
for any speech or debate in the Congress. Qualifiedly privileged communications containing
defamatory imputations are not actionable unless found to have been made without good
intention or justifiable motive. To this belong “private communications” and “fair and true report
without any comments or remarks.”

Borjal’s communications are not within the exceptions in Art. 354. But this does not mean they
are not privileged. The enumeration in Art.354 is not an exclusive list of qualifiedly privileged
communications since fair commentaries on matters of public interest are likewise
privileged. The rule on privileged communications had its genesis not in the nation’s penal code
but in the Bill of Rights of the Constitution guaranteeing freedom of speech and the press.

The doctrine of fair comment means that while in general every discreditable imputation publicly
made is deemed false because every man is presumed innocent until his guilt is judicially proved,
and every false imputation is deemed malicious, nevertheless, when the discreditable imputation
is directed against a public person in his public capacity, it is not necessarily actionable. To be
actionable, the discreditable imputation to a public official must either be a false allegation of fact
or a comment based on a false supposition. If the comment is an expression of opinion based
on established facts, then it is immaterial if it happens to be mistaken as long as it might be
reasonable inferred from the facts.

The articles dealt with matters of public interest. The objective of the FNCLT and how it was to
be funded lend its activities to be imbued with public interest. Thus, it cannot dissociate itself from
the public character of its mission. It cannot but invite close scrutiny by the media obliged to
inform the public of the legitimacy of the purpose of the activity and of the qualifications and
integrity of the personalities behind it.

To require critics of official conduct to guarantee the truth of all their factual assertions on pain of
libel judgments would lead to self-censorship, since critics would be deterred from voicing out
their criticisms even if such were believed to be true or were in fact true because of doubt of
whether it could be proved or of the expense of having to prove it. A defamatory falsehood relating
to official conduct is not actionable unless it is proven that it was made with actual malice-
knowledge that it was false or with reckless disregard of whether it was false or not.
Wenceslao is thus a public figure as defined in Ayers v. Capulong. The FNCLT was an
undertaking infused with public interest.

3. Even if Wenceslao not qualify as a public figure, it does not necessarily follow that he could not
validly be the subject of public comment for he could be as long as he was INVOLVED in a
public issue. If a matter is a subject of public interest, it is not less so merely because a private
individual is involved. The public’s primary interest is in the event and conduct of the participant,
content, effect, and significance of the conduct, not the participant’s prior anonymity.

The articles dealt with matters of public interest about which the public has the right to be
informed, taking into account the public character of FNCLT.
While Borjal may have gone overboard in his intemperate language, the privilege is not rendered
inutile for debate on public issues should be uninhibited, robust, and wide open and may include
caustic and sometimes unpleasantly sharp attacks on public officials.

While generally malice can be presumed from defamatory words, the privileged character of
a communication destroys the presumption of malice. Plaintiff must then prove actual malice.
Malice connotes ill will or spite and speaks not in response to duty but merely to injure the
reputation of the person defamed, and implies an intention to do ulterior and unjustifiable harm.
It is the essence of libel.

Wenceslao failed to prove that Borjal was animated by a desire to inflict unjustifiable harm on his
reputation or that the articles were published without good motives or justifiable ends. Borjal thus
is found to have acted in GF in exposing and denouncing what he perceived to be public deception.
There is also no proof of actual malice. “Reckless disregard of what is false or not” means that the
defendant entertains serious doubt as to the truth of the publication or that he possesses a high
degree of awareness of their probable falsity.

Borjal’s articles are at least based on reasonable grounds after he conducted several personal
interviews and considering the varied documentary evidence provided by his sources. Even if
the contents of the articles are false, mere error, inaccuracy, or even falsity alone does not
prove actual malice. Consistent with good faith and reasonable care, the press should not be
held to account, to a point of suppression, for honest mistakes or imperfections in the choice
of language. There must be some room for misstatement of fact and misjudgment. Only by giving
them much leeway can they effectively and courageously function as critical agencies in our
democracy. A public official must not be too thin-skinned to comments upon his official acts.

111. Eliseo Soriano v. Consoliza Laguardia, GR 164785, April 29, 2009 (Freedom of
Expression)
FACTS:
Petitioner, as host of the program “Ang Dating Daan” aired on UNTV 37, made the following
remarks on August 10, 2004: Lehitimong anak ng demonyo; sinungaling; Gago ka talaga Michael,
masahol ka pa sa putang babae o di ba. Yung putang babae ang gumagana lang doon yung ibaba,
[dito] kay Michael ang gumagana ang itaas, o di ba! O, masahol pa sa putang babae yan. Sabi ng
lola ko masahol pa sa putang babae yan. Sobra ang kasinungalingan ng mga demonyong ito.

Two almost identical affidavit-complaints were filed by Jessie Galapon and seven other private
respondence, all INC members, against petitioner Soriano before the MTRCB. Respondent
Michael Sandoval, who felt directly alluded to, was then a minister of INC and a regular host of
the TV program “Ang Tamang Daan.” The MTRCB preventively suspended the showing of Ang
Dating Daan for 20 days after a preliminary conference pursuant to Sec.3(d) of PD 1986, creating
the MTRCB and its IRR. The next day, Soriano sought reconsideration, praying that Chairperson
Consoliza Laguardia and two other members of the adjudication board recuse themselves from
hearing the case. Two days later, he sought to withdraw his MR as he filed with this Court a
certiorari petition to nullify the preventive suspension.
The MTRCB imposed 3 months suspension from Ang Dating Daan program Soriano. He filed
certiorari.

ISSUE:
Whether the MTRCB, in suspending Soriano from appearing in his program Ang Dating Daan for
3 months following his utterance of obscene speech in a G rated or “suitable for all ages” program
in a timeslot where children are the most likely audience, exceeded its authority and violated his
right to freedom of expression.
HELD: NO.
Soriano claims that his preventive suspension and the IRR authorizing it are invalid as PD 1986
does not expressly authorize MTRCB to issue preventive suspension. But MTRCB’s basic
mandate under PD 1986 reveals that it has authority, albeit impliedly, to issue the order of
preventive suspension stemming from and is necessary for the exercise of its power of regulation
and supervision. Soriano’s restrictive reading of PD 1986, limiting the MTRCB to functions within
the literal confines of the law, would give the agency little leeway to operate, rendering it inutile.
It is given powers “necessary or incidental to the attainment of the purposes” of the act.

He claims that his statement like “putang babae” were said in exercise of his religious belief. The
Court is at a loss as to how such comes within the pale of Sec. 5, Art.III on religious freedom.
There is nothing in the statement expressing any particular religious belief or furthering his
evangelical mission. The fact that his statements were made in a televised bible exposition program
does not automatically accord them the character of a religious discourse. Insults directed at
another person cannot be elevated to religious speech.

1. Freedom of speech
Soriano urges the striking down of the decision suspending him from hosting Ang Dating Daan
for three months as it violates his freedom of speech and expression guaranteed under Sec.4, Art.III
of the Constitution. He also urges that Sec.3(c) of PD 1986 be declared unconstitutional.

Expressions through newspapers, radio, television, and movies are protected by the free speech
and expression clause. Each method though, because of its dissimilar presence in the lives of
people and accessibility to children, tends to present its own problems. Restrictions, such as in the
form of prior restraint (judicial injunction against publication or threat of cancellation of
license/franchise or subsequent liability whether in libel or damage suits) are anathema to freedom
of expression. Prior restraint means official government restrictions on the press or other forms
of expression in advance of actual publication or dissemination. Freedom of expression may be
regulated to some extent to serve important public interests. The limits of freedom of expression
are reached when it touches upon matters of essentially private concern. Any sentiments must
be expressed within the proper forum and with proper regard for the rights of others.

There are certain well-defined and narrowly limited classes of speech that are HARMFUL, the
prevention and punishment of which is allowed. A speech would be unprotected if it involves “no
essential part of any exposition of ideas and are of such slight social value as a step of truth
that any benefit that may be derived from them is clearly outweight by the social interest in
order and morality. Being of no or little value, in regulating them, there is no need to apply the
clear and present danger rule or the balancing-of-interest test, they being modes of weighing
competing values and determining which of clashing interests should be advanced.

Petitioner’s speech is not protected. Unprotected speech or low-value expression refers to


libelous statements, obscenity or pornography, false or misleading advertisement, insulting
or “fighting words,” i.e. those by their utterance inflict injury or tend to incite an immediate
breach of peace and expression endangering national security.

Soriano’s statements are obscene, at least with respect to the average child. In that context, it is
unprotected. There is no perfect definition of “obscenity,” the basic guidelines are (a) whether to
the average person, applying contemporary standards would find the work, taken as a whole,
appeals to the prurient interest; (b) whether the work depicts or describes, in a patently
offensive way, sexual conduct specifically defined by the applicable state law; and (c) whether
the work, taken as a whole, lacks serious literary, artistic, political, or scientific value. What is
obscene is a proper issue for judicial determination and should be treated on a case to case basis
on the judge’s sound discretion.

While a cursory examination of Soriano’s utterances reveal that to an average adult they may only
be indecent utterance not obscene viewed only as figures of speech or a play on words, they were
uttered in a TV program that is rated G or for general viewerhip and in a timeslot likely to
reach CHILDREN. The unbridled use of such language of Soriano could corrupt impressionable
young minds. “Putang babae” means “female prostitute”, a term inappropriate for children who
could look it up in a dictionary and just get the literal meaning, missing the context within which
it was used. Children could ask what he meant when he said “gumagana lang doon yung ibaba”
(vagina). The average child may not have the adult’s grasp of figures of speech and may lack the
understanding that language may be colorful, and words may convey more than the literal
meaning. Thus, the speech is suggestive of the vagina and its function and is obscene and
unprotected.

Content-based restraint is aimed at the contents or idea of the expression, while a content-
neutral restraint intends to regulate the time, place, and manner of the expression under well-
defined standards tailored to serve a compelling state interest, without restraint on the message.
Content-based restraint is subject to strict scrutiny.

The MTRCB suspension is permissible. 1) the indecent speech was made via television, a
pervasive medium easily reaching every home where children are avid viewers; 2) the broadcast
was aired at the time of the day where there was risk that children might be the audience, and 3)
he uttered his speech on a G or general patronage rated program, suitable for all ages.

2. Clear and present danger vs balancing of interest test.


Soriano asserts that his utterances must present a clear and present danger of bringing about a
substantive evil that the state has a right and duty to prevent and such danger must be grave and
imminent.

The application of said test is uncalled for under the premises. This doctrine, first formulated by
Justice Holmes, accords protection for utterances so that the printed or spoken words may not be
subject to prior restraint or subsequent punishment unless its expression creates a clear and present
danger of bringing about a substantial evil which the government has the power to prohibit. This
doctrine evolved in the context of prosecutions for rebellion and other crimes involving the
overthrow of government. It was originally designed to determine the latitude which should be
given to speech that espouses anti-government action or to have serious deleterious consequences
on the security and public order of the community. But this is ntot a magic incantation to wipe out
all analysis in testing legitimacy of claims to free speech.

Generally, this applies to cases involving overthrow of the government and other evils which ddo
not clearly undermine national security. Not all evils can be measured in terms of “proximity
and degree.” Thus, this Court in several cases applied the balancing of interests test. Where the
legislation under constitutional attack interferes with the freedom of speech and assembly in a
more generalized way and where the effect of the speech and assembly in terms of the probability
of realization of a specific danger is not susceptible even of impressionistic calculation", then
the "balancing of interests" test can be applied.

When particular conduct is regulated in the interest of public order, and the regulation results in
an indirect, conditional, partial abridgment of speech, the duty of courts is to determine which of
two conflicting interests demands greater protection under the particular circumstances presented.
This test rests on the theory that it is the court’s function in a case before it when it finds public
interests served by legislation on one hand and the free expression clause affected on the other to
balance one against the other and arrive at a judgment where the greater weight shall be placed.
The balancing of interest test is more proper.

Here, Soriano’s freedom of speech is ranged against the duty of the government to protect and
promote the welfare of the youth. The government’s interest to protect the children’s welfare is
sufficient to restrain soriano’s prayer to continue as program host. The State has a compelling
interest in extending social protection to minors against all forms of neglect, exploitation, and
immorality which may pollute innocent minds. The government has the duty to act as parens
patriae to protect the children.

3. Prior restraint.
Soriano theorizes that the 3 month suspension is either prior restraint or subsequent punishment
that includes prior restraint, albeit indirectly. But what the MTRCB imposed is an administrative
sanction or subsequent punishment for his obscene language.

Statutes imposing prior restraints on speech are generally illegal and presumed
unconstitutional breaches of freedom of speech. The exceptions are movies, television, and radio
broadcast censorship in view of its access to numerous people like the young who must be
insulated from the prejudicial effects of unprotected speech. MTRCB has the power to regulate
and impose some prior restraint on radio and television shows, even religious programs. The power
of review and prior approval of MTRCB extends to all television programs and is valid despite
freedom of speech. All broadcast networks are regulated by MTRCB and are required to get a
permit before they air their programs. Their right to enjoy freedom of speech is subject to that
requirement.
The MTRCB became a “necessary evil” with the government taking the role of assigning
bandwidth to individual broadcasters. The stations agreed to this regulatory scheme. Otherwise,
chaos would result in television broadcast industry as competing broadcasters will interfere
with each other’s signals. In this scheme, station owners in effect waived their right to the full
enjoyment of their freedom of speech in radio and television programs and impliedly agreed that
said right may be subject to prior restraint- denial of permit or subsequent punishment, like
suspension or cancellation of permit, among others.

The 3 month suspension is not prior restraint as Soriano already had a permit. It is permissible
administrative sanction or subsequent punishment for the obscene remarks which MTRCB may
validly impose. It is imposed not on the exercise of freedom of speech, but for his indecent
utterances in a G rated program. He is deemed to have yielded his right to full enjoyment of
freedom of speech to regulation under PD 1986. The suspension is merely part of MTRCB’s duty
to enforce PD 1986.

The freedom of TV and radio broadcasting is lesser in scope than the freedom accorded to
newspaper and print media.

112. Leo Pita v. CA, GR 80806, October 05, 1989, Sarmiento, J. (Freedom of Expression;
what is obscene depends on judicial discretion on a case-to-case basis; judge must determine
what is obscene through a search warrant, cannot just be confiscated)
FACTS:
Pita is the publisher of Pinoy Playboy, a “men’s magazine.” Pursuing an Anti-Smut Campaign by
the Mayor of Manila, police officers of Manila seized and confiscated from dealers, peddlers etc.
along Manila sidewalks magazines and other publications believed to be obscene, pornographic,
and indecent and later burned the materials at Ubelt Recto. Among the magazines was Playboy.
Pita filed an injunction against Mayor Bagatsing to enjoin them from confiscating his magazines
or from preventing its sale, claiming that it is a decent, artistic, and educational magazine not per
se obscene. The RTC denied the motion. The CA dismissed the appeal.

ISSUE:
Whether the seizure of playboy magazines by police officers pursuant to the mayor’s anti-smut
campaign is a valid seizure and police power measure.
HELD: NO.
The test for obscenity is whether the tendency of the matter charged obscene, is to deprave or
corrupt those whose minds are open to such immoral influences and into whose hands a publication
or other article charged as being obscene may fall." Another test is that which shocks the ordinary
and common sense of men as an indecency. But whether a picture is obscene must depend upon
the circumstances of the case. But when does a publication have a corrupting tendency or be
offensive to human sensibilities?

People v. Padan y Alova introduced the “redeeming” element that should accompany the work to
save it from valid prosecution. An actual exhibition of the sexual act preceded by acts of
lasciviousness can have no redeeming features. One can see nothing in it but unmitigated obscenity
causing nothing but lust. But this leaves too much latitude for judicial arbitrament.
In Gonzales v. Kalaw Katigbak, the Court adopted the test: "Whether to the average person,
applying contemporary standards, the dominant theme of the material taken as a whole appeals to
prurient interest. It measured obscenity in terms of the dominant theme of the work than its isolated
passages.

It was in Miller v. California which established basic guidelines: "(a) whether 'the average person,
applying contemporary standards' would find the work, taken as a whole, appeals to the prurient
interest . . .; (b) whether the work depicts or describes, in a patently offensive way, sexual conduct
specifically defined by the applicable state law; and (c) whether the work, taken as a whole, lacks
serious literary, artistic, political, or scientific value.

The lack of uniformity in American jurisprudence as to what is obscenity is attributed to reluctance


of courts to recognize the constitutional dimension of the problem. The courts have assumed that
obscenity is not included in the guaranty of free speech which has allowed a climate of opinions
among magistrates predicated upon the vague theories as to what is acceptable to society.

There is no challenge in this case to the right of the state, in the exercise of police power, to
suppress smut- provided it is smut. Smut is not smut, simply because one insists it is smut.
Individual tastes develop with the advance of civilization. What shocked our forebears 5 decades
ago is not necessarily repulsive to the present generation.

It is a difficult question. But we have this much to say.

Immoral lore or literature comes within free expression, although not its protection. The
burden to show grave and imminent danger to justify action lies on the authorities. There must be
objective and convincing, not subjective or conjectural, proof of the existence of such clear
and present danger. It is essential that the authority does not rely solely on his own appraisal of
what the public welfare may require.

The PRESUMPTION is that speech may be validly said. The burden is on the state to
demonstrate a danger that is 1)clear and 2) present, to justify state action to stop the speech. If it
acts notwithstanding the absence of evidence clear and present danger, it must be accountable for
due process.

Petitioner also asserts due process and illegal search and seizure. The required proof to justify a
ban and confiscation of the literature was not shown. They were not possessed of a lawful court
order 1) finding the materials to be pornography, and 2) authorizing them to carry out a
search and seizure, by way of a search warrant.

While freedom of the press is not without restraint, as the state can protect society from
pornographic literature offensive to public morals, were the literature confiscated “pornographic”?
That the Mayor’s act was sanctioned by police power is no license to seize property in disregard
of due process.
Searches and seizures may be done only through a judicial warrant, else, they become unreasonable
and subject to challenge. There is no search incidental to a lawful arrest as no party has been
charged nor are such charges being readied against any party under Art.201 of the RPC.

Thus, 1) the authorities must apply for search warrant if, in their opinion, an obscenity rap is in
order. 2) They must convince the court that the materials sought to be seized are obscene and
pose a clear and present danger of an evil substantive enough to warrant state action. 3) The
judge must determine whether they are indeed obscene resolved on a case-to-case basis based on
his honor’s sound discretion. 4) If there is probable cause in the court’s opinion, it may issue the
search warrant. 5) The proper suit is then brought in the court under Art.201 of the RPC. 6) Any
conviction is subject to appeal. The appellate court may assess whether the properties seized are
indeed obscene. These do not foreclose defenses under the Constitution or applicable statutes.

The Court reversed the RTC decision. But since the magazines subject of the search and seizure
have been destroyed, the Court declines to grant affirmative relief. To that extent, the case is
moot and academic.

113. Francisco Chavez v. Raul Gonzales, GR 168338, February 15, 2008, Puno, C.J.
(Freedom of Expression)
FACTS:
On June 5, 2005, Press Sec. Ignacio Bunye told reporters that the opposition was planning to
destabilize the administration by releasing an audiotape of am obile phone conversation between
PGMA and a high-ranking COMELEC official. The convo was audiotaped through wire-tapping.
In a Malacanang press briefing, he produced two versions of the tape, one the complete, and the
other a spliced, “doctored” version, which would suggest that PGMA instructed the COMELEC
official to manipulate election results.

On June 7, former counsel of deposed President Erap, Atty. Alan Paguia, released an alleged
authentic tape recording of the wiretap. This included convos of PGMA, Jose Arroyo (husband),
COMELEC Commissioner Garcillano, and late Sen.Barbers. On June 8, DOJ Sec. Raul Gonzales
warned reporters that those who had copies of the CD could be liable under the Anti-Wiretapping
Act, RA 4200. Persons airing said tapes were committing a continuing offense.

On June 9, Sec. Gonzales ordered the NBI to go after media organizations found to have caused
the spread, the playing and printing of the contents of a tape of an alleged wiretapped convo
of PGMA about fixing votes. He said he would start with Inq7.net, joint venture between PH Daily
Inquirer (PDI) and GMA7.

On June 11, the NTC issued a press release, warning that the broadcast of such false info shall
be cause for suspension, revocation, or cancellation of the licenses issued to media companies.

Petitioner Chavez filed this petition under Rule 65 against Sec. Gonzales and NTC, praying for
prohibition to prevent the unlawful and oppressive exercise of authority by the respondents.

ISSUE:
HELD:
1. Legal standing- transcendental importance.
2. Freedom of speech, expression, press.
While all forms of communication are entitled to the broad protection of freedom of expression
clause, the freedom of film, television and radio broadcasting is somewhat lesser in scope than
the freedom accorded to newspapers and other print media, as will be subsequently discussed.

A. Tests.
All speech are not treated the same. Some types may be regulated by police power to not injure
the equal rights of others. The difference in treatment is because interests in one type of speech,
political, may vary from another, obscene. The techniques of reviewing alleged restrictions on
speech (overbreadth, vagueness, etc.) have been applied differently to each category. Generally,
restraints on freedom of speech are evaluated by either or a combination of 3 tests: (a) the
dangerous tendency doctrine which permits limitations on speech once a rational connection
has been established between the speech restrained and the danger contemplated; (b) the balancing
of interests tests, used as a standard when courts need to balance conflicting social values and
individual interests, and requires a conscious and detailed consideration of the interplay of
interests observable in a given situation or type of situation; and (c) the clear and present danger
rule which rests on the premise that speech may be restrained because there is substantial danger
that the speech will likely lead to an evil the government has a right to prevent. We have generally
adhered to the clear and present danger test.

The contribution to public weal makes freedom of the press deserving of extra protection.

B. Restrictions- prior restraint; content-neutral and content-based regulations.


The four aspects of freedom of the press are (1) freedom from prior restraint; (2) freedom from
punishment subsequent to publication; (3) freedom of access to information; and (4) freedom
of circulation.

It is argued that respondents’ press statements are prior restraints. Thus, a scrutiny of this and its
subspecie of content-based and content-neutral regulations. The parameters of impermissible
restraint have been etched on a case-to-case basis, tested by scrutinizing the governmental issuance
or act against the circumstances in which they operate, and then determining the appropriate
test with which to evaluate.

Prior restraint refers to official governmental restrictions on the press or other forms of expression
in advance of actual publication or dissemination. Freedom from prior restraint is freedom from
government censorship. Certain prior restraints are permitted, but determined upon careful
evaluation of the challenged act against the appropriate test by which it should be measured
against.

It is not enough to determine whether the act restrains speech. A distinction must be made whether
the restraint is 1) content-neutral, merely concerned with the incidents of the speech, controlling
time, place, or manner under well-defined standards, or 2) content-based, based on the subject
matter of speech. The cast of the restriction determines the test by which the challenged act is
assayed with.
C. Content-neutral- intermediate approach.
In content-neutral, only a substantial government interest is required for its validity as this does
not suppress any particular message and thus, are not subject to the strictest form of judicial
scrutiny but to an intermediate approach- somewhere between the mere rationality required
of any other law and the compelling interest standard applied to content-based restrictions.
It is called “intermediate” as the Court will not merely rubberstamp the validity of a law but also
require that the restrictions be narrowly-tailored to promote an important governmental interest
unrelated to the suppression of expression.

The intermediate approach is formulated thus: A governmental regulation is sufficiently justified


if it is within the constitutional power of the Government, if it furthers an important or
substantial governmental interest; if the governmental interest is unrelated to the suppression
of free expression; and if the incident restriction on alleged [freedom of speech & expression] is
no greater than is essential to the furtherance of that interest.

D. Content-based- clear and present danger.


When the restraint is based on content, it is given the strictest scrutiny in light of its inherent
invasive impact. It has to overcome the clear and present danger rule. The government must
show the type of harm the speech would bring about, especially the gravity and imminence
of the threatened harm. Otherwise, prior restraint will be invalid. Prior restraint cannot be
justified by hypothetical fears but only by showing a substantive and imminent evil. The question
is whether the words used are used in such circumstances and are of such a nature as to
create a clear and present danger that they will bring about the substantive evils that
Congress has a right to prevent. It is a question of PROXIMITY and DEGREE. The
regulation must also serve an important governmental interest unrelated to the suppression of free
expression. It must be no greater than what is essential also. It must thus be narrowly drawn with
the least restrictive means.

Here, the restriction is content-based- the alleged taped conversations between PGMA and
Garcillano.

3. Print v. Broadcast media.


Respondents argue that broadcast media enjoys less free speech rights than that of print media.
The regimes in place for each type of media differ from one another.

According to US Courts, the three major reasons why broadcast media stands apart from print
media are 1) scarcity of frequencies by which the medium operates, 2) its pervasiveness as a
medium, and 3) its unique accessibility to children. But despite this difference, PH case law
declares that the test for all forms of media is the clear and present danger rule.

PH case law is different from US case law on which test to apply to broadcast media content-based
prior restraints. The difference in treatment is in the regulatory scheme applied to broadcast media
that is not imposed on traditional print media, and narrowly confined to unprotected speech or
based on compelling government interest. The Court has held that the clear and present danger
test applies to content-based restrictions on media, without distinguishing as to print or
broadcast media.

When the Court declared that the freedom of broadcast media was somewhat “lesser in scope”
than that accorded to print, it was not as to what test should be applied, but the context by which
requirements of licensing allocation of airwaves. But the clear and present danger rule only
applies to broadcast media if the restraint is content-based.

4. Applied in the case at bar.


Since the restraint involved is content based, it is given the strictest scrutiny, with the government
having the burden of overcoming the presumed unconstitutionality by the clear and present danger
rule. This applies equally to all kinds of media, including broadcast.

This outlines the procedural map to follow in cases like the one at bar as it spells out the
following: (a) the test; (b) the presumption; (c) the burden of proof; (d) the party to discharge the
burden; and (e) the quantum of evidence necessary. Respondents failed to hurdle the clear and
present danger test. The great evil which the government wants to prevent is the airing of a tape
recording in alleged violation of the anti-wiretapping law. But the records are confusing, and
respondents’ evidence falls short of satisfying the c&p rule.

1) The various statements of the press secretary obfuscate the identity of the voices in the tape
recording. 2) The integrity of the taped conversation is suspect as he showed to the public two
versions. 3) Respondents’ evidence on the who’s and how’s of the wiretapping act is ambivalent,
especially considering the tape’s different versions. The identity of the wire-tappers, the manner
of its commission, etc. are unknown. 4) It is arguable whether its airing would violate the anti-
wiretapping law.

Not every violation of a law will justify restraining freedom of speech and the press. Our laws
are different, and some provide norms of conduct which if violated would only adversely affect a
person’s private comfort but does not endanger national security. Violation of such laws, by
itself, cannot support suppression of speech. It is merely a factor to be weighed in judging restraint.
The TOTALITY of the injurious effects of the violation to private and public interest must be
calibrated. There is no showing that the feared violation of the anti-wiretapping law clearly
endangers national security.

Do the mere press statements of the SOJ and NTC constitute a form of content-based prior
restraint? It is not decisive that the press statements were not reduced in or followed up with formal
orders or circulars. It is sufficient that these were made by respondents while in the exercise of
their official functions. Any act done, like a speech uttered, on behalf of the government in an
official capacity is covered by the rule on prior restraint. An “act” does not limit itself to those
converted to a formal order or circular. Otherwise, the non-formalization of an act will result in
easy circumvention of the prohibition on prior restraint.

There is enough evidence of chilling effect of the complained acts on record. The warnings given
to media came from no less the NTC, a regulatory agency that can cancel the Certificate of
Authority of the radio and broadcast media. They also came from the Secretary of Justice, the alter
ego of the Executive, who wields the awesome power to prosecute those perceived to be violating
the laws of the land. After the warnings, the KBP inexplicably joined the NTC in issuing an
ambivalent Joint Press Statement. After the warnings, petitioner Chavez was left alone to fight this
battle for freedom of speech and of the press. This silence on the sidelines on the part of some
media practitioners is too deafening to be the subject of misinterpretation.

114. Disini v. SOJ, GR 203335, February 11, 2014 (Freedom of Expression)


FACTS:
Anti-Cybercrime law is assailed as unconstitutional.

HELD:
1. Sec.4(c)(3)- penalizing the transmission of unsolicited commercial communications or “spam.”
To prohibit the transmission of unsolicited ads would deny a person the right to read his emails,
even unsolicited commercial ads addressed to him. Commercial speech is a separate category of
speech which is not accorded the same level of protection as that given to other constitutionally
guaranteed forms of expression but is nonetheless entitled to protection. The State cannot rob him
of this right without violating the constitutionally guaranteed freedom of expression. Unsolicited
advertisements are legitimate forms of expression.

2. Sec.4(c)(4) of the Cybercrime law incorporates the RPC provisions on libel:


(4) Libel. — The unlawful or prohibited acts of libel as defined in Article 355 of the
Revised Penal Code, as amended, committed through a computer system or any other
similar means which may be devised in the future.
The UNHRC did not actually enjoin the Philippines, as petitioners urge, to decriminalize libel. It
simply suggested that defamation laws be crafted with care to ensure that they do not stifle freedom
of expression. Indeed, the ICCPR states that although everyone should enjoy freedom of
expression, its exercise carries with it special duties and responsibilities. Free speech is not
absolute. It is subject to certain restrictions, as may be necessary and as may be provided by law.

Libel is not a constitutionally protected speech. The government has an obligation to protect
private individuals from defamation. Cyberlibel is not a new crime since Art.353 in relation to
Art.355 of the RPC already punishes it. It merely affirms that online defamation is “similar means”
for committing libel. But it is valid only insofar as the cybercrime law penalizes the author of the
libelous statement or article.

3. Sec. 5. Other Offenses. — The following acts shall also constitute an offense:
(a) Aiding or Abetting in the Commission of Cybercrime. — Any person who willfully abets or
aids in the commission of any of the offenses enumerated in this Act shall be held liable.
(b) Attempt in the Commission of Cybercrime. — Any person who willfully attempts to commit
any of the offenses enumerated in this Act shall be held liable.
This is assailed as suffering from overbreadth. Aiding or abetting has a well-defined meaning in
existing laws as lending themselves to the test of common sense and human experience. But the
idea of abetting or aiding in cybercrimes is not as clear.

Are online postings such as “liking” an openly defamatory statement or “commenting” or


“sharing” it “aiding and abetting?” Except for the original author of the assailed statement, the rest
(those who liked, commented, and shared) are knee-jerk sentiments of readers who may think
haphazardly of their response to the original post. Will they be liable? And considering the inherent
impossibility of joining thousands of responding “followers” in the criminal charge to be filed,
who will choose who should go to jail?

The old parameters for enforcing the traditional form of libel would be a square peg in a round
hole when applied to cyberspace libel. Unless the legislature crafts a cyberlibel law that takes into
account its unique circumstances and culture, such law will tent to create a chilling effect on
the millions that use this new medium of communication violating their freedom of expression.

The sanctions may well cause speakers to remain silent rather than communicate even arguably
unlawful words. it would silence some speakers whose messages are entitled to protection. While
libel in cyberspace can spread and travel fast with just one click of the mouse, a governmental
purpose to regulate the use of cyberspace communication technology to protect a person’s
reputation and peace of mind cannot adopt means that will unnecessarily and broadly sweep,
invading the area of protected freedoms. Penal laws should provide reasonably clear guidelines
for law enforcement officials and triers of facts to prevent arbitrary and discriminatory
enforcement.

When a penal statute encroaches upon the freedom of speech, a facial challenge grounded on the
void-for-vagueness doctrine is acceptable. In an “as applied” challenge, a petitioner can raise any
constitutional ground, but only when he asserts a violation of his own rights. This is the rule
against third-party standing. The exception is a “facial” challenge even without violation of his
own rights where it involves free speech on grounds of overbreadth or vagueness. The rationale is
to counter the chilling effect on protected speech. A person who does not know whether his
speech is a crime under an overbroad or vague law may simply restrain himself from
speaking to avoid being charged. The overbroad or vague law thus chills him into silence.
Netizens are not given “fair notice” or warning as to what is criminal conduct and what is lawful
conduct.

Of course, if the comment does not merely react to the original post but creates an altogether new
defamatory statement, then it should be considered an original posting published on the internet.
But the penal code and cybercrime law clearly punish authors.

As to aiding or abetting child pornography, when Google procures or stores child pornography and
facilitates completion of transactions involving dissemination thereof, does this make it and its
users abettors of child pornography crimes? The Cybercrime law lacks exemption of a provider or
plain user of interactive computer service from civil liability for child pornography. When a
person replies to a tweet containing child pornography, he republishes it whether wittingly or
unwittingly. Is he an accomplice to its distribution?

Thus, Sec.5 with respect to Sec.4(c)(4) is unconstitutional. Its vagueness has a chilling effect as
the crime of aiding or abetting ensnares all actors in the cyberspace front in a fuzzy way. Libel is
a formal crime punishable only when consummated. In the absence of a law tracing the interaction
of netizens and their level of responsibility like in other countries, Sec.5 in relation to
Sec.4(c)(4), Sec.4(c)(3) on unsolicited commercial communications, and Sec.(4)(c)(2) on child
pornography cannot stand scrutiny.

4. Sec. 19. Restricting or Blocking Access to Computer Data. — When a computer data is prima
facie found to be in violation of the provisions of this Act, the DOJ shall issue an order to restrict
or block access to such computer data.

Computer data may be personal property. Thus, it is protected from unreasonable searches and
seizures whether in personal computers or in the service provider’s systems. The DOJ would seize
and place computer data under its control without warrant. The DOJ order cannot substitute for
judicial search warrant.

The content of the computer data can also constitute speech. Thus, Sec.19 restricts freedom of
expression over cyberspace. Not all forms of speech are protected. The legislature amy declare
some kinds of expression illegal. But for an executive officer to seize content alleged to be
unprotected without any judicial warrant, it is not enough for him to be of the opinion that
such content violates some law, for to do so would make him judge, jury, and executioner all
rolled into one.

Sec.19 requires only that the data blocked be found prima facie in violation of any provision of
the cybercrime law. It disregards jurisprudential guidelines to determine validity on speech
restriction like the dangerous tendency doctrine, balancing of interest test, and the clear and present
danger rule. It does not take into consideration any of the tests. Thus, it is unconstitutional.

115. Social Wether Stations(SWS) Inc. v. COMELEC, GR 147571, May 05, 2001 (Freedom
of Expression)
FACTS:
SWS is a private research institution conducting surveys in various fields including economics,
politics, etc. and thereafter processing, analyzing, and publicly reporting the results. Petitioner
Kamahalan Publishing Corp publishes the Manila Standard, a newspaper, featuring newsworthy
items of info like election surveys. They bring this action for prohibition to enjoin COMELEC
from enforcing S5.4 of RA 9006 (Fair Election Act):
Surveys affecting national candidates shall not be published fifteen (15) days before an
election and surveys affecting local candidates shall not be published seven (7) days before
an election.
Election surveys are defined as those measurements of opinions and perceptions of the voters
regarding a candidate’s popularity, qualifications, etc. The COMELEC implemented S5.4 thru its
Resolution 3636, S24(h).

Both petitioners wish to publish election survey results. They argue that the restriction is prior
restraint without any clear and present danger to justify it. There is no historical evidence to support
the conclusion that there is danger to the voting process posed by election surveys. COMELEC
justifies the restriction as necessary to prevent manipulation and corruption of the electoral
process by unscrupulous and erroneous surveys just before the election.

ISSUE:
Whether RA 9006, in prohibiting the publication of election surveys affecting candidates for
election 15 days prior to an election for national candidates and 7 days prior for local candidates,
violates freedom of expression.

HELD: YES.
S5.4 is a prior restraint on freedom of speech, expression, and of the press. The government has
a heavy burden of showing justification for such restraint. There is a reversal of the normal
presumption of validity.

Justice Kapunan dissents. He rejects as inappropriate the test of clear and present danger for
determining validity of S5.4. This test was originally for criminal law and only later applied to free
speech cases. The test is concerned with gravity and imminence of danger as basis for curtailing
free speech, which is not the case for S5.4 and similar regulations. Kapunan purports to balance
the circumstances to determine if the public interest in free, orderly, etc. elections is served by the
regulation. The purpose is to prevent last-minute pressure on voters, bandwagon effect to favor
candidates, misinformation, etc. Thus, he concludes that the restriction is not unduly repressive or
unreasonable. It is a mere restriction, not absolute prohibition. It is limited in duration, applying
only during when voters are presumably contemplating whom they should elect.

But this does not show why, on balance, these considerations should outweigh the value of freedom
of expression. It relies instead on Art. IX-C, S4. It cannot be argued that because of Art.IX-C, S4
of the Constitution, giving the COMELEC supervisory power to regulate enjoyment or utilization
of franchise for operation of media of communication, no presumption of invalidity attaches to
S5.4. this grant is limited to ensuring equal opportunity, time, space, and the right to reply and
reasonable rates of charges for the use of such media facilities for “public information campaigns
and forums among candidates.”

RA 9006 has lifted the ban and now allows candidates to advertise their candidacies in media. To
sustain the ban of publication of survey results would censor all speaking by candidates in an
election on the ground that the usual bombasts and hyperbolic claims made during campaigns can
confuse voters and thus debase the electoral process.

The dissent only engaged a balancing at the margin. This ad hoc balancing predictably results in
sustaining the challeneged law and leaves freedom of speech with little protection. Anyone can
bring a plausible justification to show a rational connection between a law and governmental
purpose.

The ban cannot be justified on the ground that other countries impose similar restrictions. This is
inconclusive. What test then should be employed? The US SC, thru CJ Warren, in US v. Obrien
provides: [A] government regulation is suKciently justified [1] if it is within the constitutional
power of the Government; [2] if it furthers an important or substantial governmental interest; [3]
if the governmental interest is unrelated to the suppression of free expression; and [4] if the
incidental restriction on alleged First Amendment freedoms [of speech, expression and press] is
no greater than is essential to the furtherance of that interest.
This is the most influential test for distinguishing content-based from content-neutral
regulations. Under this test, even if a law furthers a substantial government interest, it is invalid
if such interest is not unrelated to the suppression of free expression. It should also not be greater
than is necessary.

1. 3rd criterion is not met.


The casual connection of expression to the asserted governmental interest makes such interest
not unrelated to the suppression of free expression. By prohibiting publication of election survey
results as it might undermine the integrity of election, S5.4 suppresses a whole class of expression,
while allowing the expression of opinion concerning the same subject matter by newspaper
columnists, radio, and TV commentators etc. Thus, S5.4 shows bias for a particular subject
matter or viewpoint by preferring personal opinion to statistical results. Freedom of
expression means that the government cannot restrict expression because of its message or content.
The inhibition should be upheld only if the speech is unprotected. (fighting words, lewd and
obscene, etc.)

Thus, the prohibition cannot be justified on the ground that it is only for a limited period only and
is incidental. While it is for a limited time, the curtailment of the right of expression is direct,
absolute, and substantial. It is a total suppression of a category of speech and is not made less
so just because it is for a limited period.

2. Even if governmental interest sought to be promoted is unrelated to suppression of speech and


the restriction is only incidental, S5.4 still fails to meet the 4th criterion of the O’Brien test. S5.4
aims to prevent last-minute pressure on voters etc. These cannot be attained at the sacrifice of the
fundamental right of expression when such aim can be more narrowly pursued by punishing
unlawful acts, rather than speech. The Administrative Code gives the COMELEC the power to
stop unlawful or misleading, false election propaganda. This is a LESS RESTRICTIVE means
than S5.4. COMELEC can thus confiscate bogus survey results calculated to mislead voters.
Candidates can have their own surveys conducted. No right of reply can be invoked by others. No
principle of equality is involved. It is a free market where each candidate brings his ideas.

The purpose of preventing bandwagon effect is doubtful. Can this be achieved by suppressing
survey results? Voters want to be identified with winners and this is a natural-enough tendency of
some voters.

To summarize then, we hold that §5.4 is invalid because (1) it imposes a prior restraint on the
freedom of expression, (2) it is a direct and total suppression of a category of expression even
though such suppression is only for a limited period, and (3) the governmental interest sought to
be promoted can be achieved by means other than the suppression of freedom of expression.

116. US v. O’Brien, 391 US 367 (1968), May 27, 1968 (Freedom of Expression; O’Brien test;
noncommunicative conduct itself is harmful, not its communicative aspect)
On the morning on March 31, 1966, David Paul O’ Brien and 3 companions burned their Selective
Service registration certifications in front of a sizable crowd on the steps of the South Boston
Courthouse. The crowd had several FBI agents who saw the event. The crowd immediately
attacked O’Brien and his companions. An FBI agent ushered him to safety in the courthouse. He
told the FBI agents that he burned his certificate because of his beliefs, knowing that he was
violating the law. He was thus tried and convicted. He argued that he burned the certificate publicly
to influence others to adopt his anti-war beliefs.

He was charged with “willfully and knowingly” mutilating, destroying, and changing by burning
the certificate, violating S462(b) of the App. US Code. The law was amended in 1965 which added
the bold part in “who forges, alters, knowingly destroys, knowingly mutilates, or in any other
manner changes any such certificate.” He argues that the 1965 amendment is unconstitutional,
abridging free speech, as it served no legislative purpose.

He was convicted by the District Court. But the CA held the 1965 amendment unconstitutional,
but it also convicted O’Brien of a lesser offense. Hence, the government petitioned for certiorari.

ISSUE:
Whether a regulation penalizing the burning of a Selective Service Certificate which contains
details of a person and his Selective Service number to facilitate easier communication between
the registrant and local boards, to serve as easy proof that one is not delinquent in his selective
service obligations and for other purposes abridges the freedom of speech on the ground that the
purpose of one punished under such regulation was to communicate a “symbolic speech” of his
disapproval of war and made before a group of people.
HELD:NO.
When a male reaches 18 yo, he is required to register with a local draft board and assigned a
selective service number. Within 5 days, he is issued a registration certificate. He answers a
questionnaire and, based thereon, is issued a notice of classification. Both registration and
classification certificates are small white cards, 2x3 inches.

Congress was concerned that certificates issued by the Selective Service System (SSS) might be
abused. The 1948 Act made it unlawful to commit acts in relation to the certificate, like “to forge,
alter, or in any manner change a certificate”. The 1965 amendment added “knowingly destroys or
knowingly mutilates.” On its face, the 1965 amendment does not abridge free speech. It deals
with conduct having no connection with speech. It prohibits the knowing destruction of
certificates issued by SSS, and there is nothing necessarily expression about such conduct.

But O’Brien argues that 1965, as applied to him, is unconstitutional as the “purpose” of Congress
was to suppress free speech.

1. He argues that as applied to him, the 1965 amendment is unconstitutional as his act of burning
his certificate was protected symbolic speech. He argues that freedom of expression includes all
modes of communication of ideas by conduct.

We cannot accept the view that a limitless variety of conduct can be labeled speech whenever the
person engaging therein intends to express an idea. But even if the communicative element in his
conduct would bring it into protection of the First Amendment, it does not follow that the
destruction of a registration certificate is constitutionally protected activity. When speech and
nonspeech elements are combined in the same course of conduct, a sufficiently important
governmental interest in regulating the nonspeech element can justify incidental limitations of
free speech. To characterize the quality of governmental interest which must be present, the Court
has employed a variety of descriptive terms: compelling, substantial, subordinating, paramount,
cogent, strong. But we think that a government regulation is sufficiently justified if 1) it is
within the constitutional power of the government; 2) if it furthers an important or substantial
governmental interest; 3) if the governmental interest is unrelated to the suppression of free
expression; and 4) if the incidental restriction on free speech is no greater than is essential to
the furtherance of that interest. The 1965 amendment meets all these requirements.

A. The constitutional power of Congress to raise and support armies and to make all laws necessary
to that end is broad and sweeping. Pursuant to this power, Congress may establish a system of
registration for individuals liable for training and service. The issuance of certificates indicating
the registration and eligibility classification of individuals is a legitimate and substantial
administrative aid in the functioning of this system.

The certificate is proof that the individual described thereon has registered for the draft. The
information therein facilitates communication between the registrant and local boards, simplifying
the system. It bears each registrant’s selective service number. The destruction or mutilation of
certificates increases the difficulty of detecting and tracing abuses of deceptive misuse of
certificates, alteration, or forgery. A mutilated certificate might be used for deceptive purposes.
Thus, Congress has a legitimate and substantial interest in preventing their wanton destruction.
The continuing availability to each registrant of his selective service certificates furthers the
smooth and proper functioning of the system Congress established to raise armies. The nation has
a vital interest in having a system for raising armies that is capable of quickly responding to
continually changing circumstances. Thus, the government has a substantial interest in assuring
the continuing availability of selective service certificates.

The 1965 amendment specifically protects this substantial governmental interest. We perceive no
alternative means that would more precisely assure continuing availability of such certificates than
a law preventing their willful mutilation or destruction. The 1965 amendment prohibits such
conduct and does nothing more. Both the governmental interest and the operation of the 1965
amendment are limited to the noncommunicative aspect of O’Brien’s conduct. When he
deliberately rendered unavailable his registration certificate, he willfully frustrated this
governmental interest and for this noncommunicative impact of his conduct, and for nothing else,
he was convicted.

B. This case is not where the alleged governmental interest in regulating conduct arises because
the communication integral to the conduct is itself thought to be harmful. (Unrelated) For example,
this Court struck down the prohibition of displaying any flag, badge, banner, or device to express
opposition to organized government since this was aimed at suppressing communication and
could not be sustained as a regulation of noncommunicative conduct.

The government has substantial interest. The measure is appropriately narrow means of protecting
this interest and condemns only the independent noncommunicative impact of conduct. The
noncumminicative impact of O’Brien’s act of burning his certificate frustrated the
Government’s interest.
2. O’Brien argues that the 1965 amendment is unconstitutional as the purpose of Congress behind
it was to suppress freedom of speech. But the purpose of Congress is not a basis for declaring
legislation unconstitutional. The Court will not strike down an otherwise constitutional statute
based on an alleged illicit legislative motive. What motivates one legislature to make a speech
about a statute is not necessarily what motivates scores of others to enact it.

117. The Diocese of Bacolod v. COMELEC, GR 205728, January 21, 2015, Leonen, J(
Freedom of Expression)
FACTS:
On February 21, 2013, petitioners posted two tarpaulins within a private compound housing the
San Sebastian Cathedral of Bacolod. Each tarp was approximately 6x10 feet. They were posted on
the front walls of the cathedral within public view. The first tarp contains “IBASURA RH Law.”
The second tarp, subject of this case, contains the heading “Conscience Vote” and lists of
candidates as either “(Anti-RH) Team Buhay” with a check mark or “(Pro-RH) Team Patay” with
an “X” mark. The candidates were classified on their vote for passing the RH Law.

The tarps were not sponsored or paid for by any candidate. They contain names of candidates for
the 2013 elections only.

Respondent Atty. Majarucon as Election Officer in Bacolod issued a notice to remove campaign
materials to petitioner bishop Navarra. COMELEC Resolution 9615 provides for the size
requirement of 2x3feet only. Otherwise, an election offense will be filed against petitioners.
Petitioners filed this case for certiorari and prohibition. They question the notice to remove.

ISSUE:
Whether COMELEC, in ordering the removal of The Diocese of Bacolod’s tarp showing election
candidates who voted favorably for the passage of the RH Bill as “Team Patay” and those who
voted against its passage as “Team Buhay” as the tarp was 6x10 feet in size larger than
COMELEC’s campaign materia maximum size of 2x3 feet, violated The Diocese of Bacolod’s
freedom of expression.
HELD: YES.
1. COMELEC had no legal basis to regulate expressions made by private citizens. Petitioners are
not candidates or belong to any political party.

COMELEC cites Art.IX-C, Sec.4 “The Commission may, during the election period, supervise or
regulate the enjoyment or utilization of all franchises or permits… Such supervision or regulation
shall aim to ensure equal opportunity, time, and space, and the right to reply, including reasonable,
equal rates therefor, for public information campaigns and forums among candidates…” The evil
sought to be prevented is the possibility that a franchise holder may favor a candidate in terms of
advertising space. Petitioners are neither franchise holders nor candidates.

Article IX-C, Sec.2(7) is also cited, giving the COMELEC power to recommend to Congress
measures to prevent and penalize election “frauds, offenses, malpractices, and nuisance
candidates.” It may be inferred that this provision only affects candidates. Sec.9 of the Fair Election
Act, regulating posting of campaign materuals, mentions only political “parties” and “candidates.”
The election propaganda refers to matter done by or on behalf of and in coordination with
candidates and political parties. Some level of coordination with candidates and political parties
for whom the propaganda are released would make it fall within the authorized expenses limitation.
But the tarp was not paid for by any candidate or political party.

2. The violation of the right to Freedom of speech and expression.


Petitioners contend violation of freedom of expression. Respondents maintain that the tarp is
election propaganda subject ot their regulation pursuant to their mandate under Art.IX-C, Sec.4.

Article III, Sec.4 of the Constitution is relevant: Section 4. No law shall be passed abridging the
freedom of speech, of expression, or of the press, or the right of the people peaceably to assemble
and petition the government for redress of grievances.

A. “No law…”
The Court has applied S4 to governmental acts even if they are not laws, like the opinion by the
COMELEC Law Department who issued the second notice to remove. This was applied to
requiring a mayor to issue a permit to public assembly and a COMELEC Resoution.

B. “…shall be passed abridging…”


The prohibition against abridgment of speech does not mean absolute prohibition against
regulation. The primary and incidental burden on speech must be weighed against a compelling
state interest.

C. “…of expression…”
“Expression” was added in the 1987 Constitution by Commissioner Brocka for having a wider
scope.

D. Communication is an essential outcome of protected speech.


Communication exists when 1) a speaker, seeking to signal others, uses conventional actions
because he reasonably believes that such actions will be taken by the audience in the manner
intended; and (2) the audience so takes the actions. Speech is not limited to vocal communication.
Conduct is a form of speech sometimes referred to as symbolic speech, such that when speech
and nonspeech elements are combined in the same course of conduct, the communicative
element of the conduct may bring into play freedom of expression. Thus, this right applies to the
entire continuum of speech from utterances made to conduct enacted and even to inaction itself
as a symbolic manner of communication. The guarantee to utter what is in his mind includes the
liberty not to utter what is not in his mind.

E. Size of the tarps.


Form of expression is as important as the information conveyed that it forms part of the
expression. Size matters: 1) It enhances efficiency in communication as larger tarps allow larger
fonts, making it easier to view from greater distances. It also makes it easier for passengers
inside moving vehicles, with lesser time to view the tarp than pedestrians, to read it. 2) The size
may underscore the importance of the message to the reader. From an ordinary person’s
perspective, those who post their messages in larger fonts care more about their message.
Effectivity of communication sometimes relies on emphasis. 3) Larger spaces allow for more
messages.
These become more salient when it is the electorate, not the candidates or political parties, that
speaks. Large tarps, thus, are not analogous to time and place. They are fundamentally part of
expression protected by S4, AIII of the Constitution.

F. Theories on need to protect freedom of expression.


1) Right of the people to participate in public affairs, including right to criticize government
actions. Deliberative democracy anchors on the principle that the cornerstone of every democracy
is that sovereignty resides in the people. 2) Free speech should be encouraged under the concept
of a market place of ideas. The exposure to the ideas of others allows one to consider and develop
their own conclusions. 3) Free speech involves self-expression that enhances human dignity. 4)
Expression is a marker for group identity. 5) The bill of rights is supposed to protect individuals
and minorities against majoritarian abuses perpetrated through the framework of democratic
governance. The citizenry, majorities, may be oppressed by dominant factions of the electorate
that capture the government for their own selfish ends. 6) Free speech must be protected under the
safety valve theory. This provides that nonviolent manifestations of dissent reduce likelihood of
violence.

G. Every citizen’s expression with political consequences enjoys a high degree of protection. It
is at the core of freedom of expression.
COMELEC argues that the tarp is election propaganda subject to its regulation. Petitioners invoke
their right to communicate their opinions and beliefs about issues and candidates.

Free speech and other intellectual freedoms are highly ranked in our scheme of constitutional
values. Property rights may be lost thru prescription, but human rights are imprescriptible.

Not all speech are treated the same. We distinguish between political and commercial speech.
Political speech refers to speech both intended and received as a contribution to public deliberation
about some issue, fostering informed and civic-minded deliberation. Commercial speech is speech
that does no more than propose a commercial transaction. The tarps are definitely political speech.

While the tarp may influence the success or failure of the named candidates, this does not
necessarily mean it is election propaganda. It was not paid for or posted “in return for
consideration” by any candidate or political party. Resolution 9615 of the COMELEC, RA 9006’s
IRR, defines political advertising as including matters, not falling within the scope of personal
opinion, that appear on any internet website etc., in return for consideration xxx. Personal opinions
are not included while sponsored messages are covered.

H. Content-based regulation: 1) size regulation; 2) applies only to those which “may affect the
elections”
COMELEC contends that the order to remove the tarp is content-neutral regulation as petitioners
failed to comply with the maximum size limitation for lawful election propaganda. Petitioners
argue that it is content-based.

The regulation may reasonably be considered as either content-neutral or content-based, but the
disposition of the case will be the same. Generally, compared with other forms of speech, the
proposed speech is content based. The interpretation of COMELEC in its order applies only to
posters and tarps that MAY AFFECT THE ELECTIONS because they deliver opinions that
shape both their choices. It does not cover, for instance, commercial speech.

There is also no distinction as to what will categorize speech as election-related or not.


COMELEC’s discretion to limit speech in this case is fundamentally unbridled. SIZE limitations
during elections hit at a core part of expression. The content of the tarp is not easily divorced
from the size of its medium.

Content-based regulation bears a heavy presumption of invalidity and is measured against the clear
and present danger rule. “The evil consequences sought to be prevented must be substantive,
'extremely serious and the degree of imminence extremely high.” Even with this test, COMELEC
fails to justify the regulation. There is no compelling and substantial state interest endangered
by the posting of the tarp. There is no reason to minimize the right of non-candidate petitioners
to post the tarp in their private property.

Content-based restrictions refer to those based on the subject matter of the speech. Content-neutral
regulation controls merely the incidents of the speech such as time, place, or manner of speech.

I. The dissenters argue that the size regulation is content-neutral. But even if the test for content-
neutral regulation was applied, the acts of COMELEC will not pass the requirements. The
intermediate approach has 4 requisites: [1] if it is within the constitutional power of the
Government; [2] if it furthers an important or substantial governmental interest; [3] if the
governmental interest is unrelated to the suppression of free expression; and [4] if the incident
restriction on alleged [freedom of speech & expression] is no greater than is essential to the
furtherance of that interest.

1) It is not within the constitutional powers of COMELEC to regulate the tarp as this is protected
speech of non-candidate petitioners. 2) Governmental interest must not only be important or
substantial, but also compelling. This includes constitutionally declared principles (welfare of
children and state as parens patriae).

Justice Brion dissents, opining that the size limit is proper to ensure equality of public information
among candidates and their supporters, else candidates with more money and supporters with
deep-pockets would have an undue advantage. But this interest has been held to be not as
important as the right of a private citizen to freely express his choice and exercise his right
to free speech.

3) The size regulation is not unrelated to the suppression of speech. Limiting the maximum size of
the tarp would render ineffective petitioners’ message and violate their freedom of expression.
The restriction must not be narrowly tailored to achieve the purpose and must allow
alternative avenues for the actor to make speech.

COMELEC’s acts is a strong deterrent to further speech by the electorate. There are indicators that
this will cause a chilling effect on robust discussion during elections given the stature of petitioners
and their message. The form of expression is just as important as the message. The medium is
the message.

3. Freedom of Expression and Equality


A. Possibility of abuse.
Freedom of expression of individuals without any relationship to any political candidate should
not be held hostage by the possibility of abuse by those seeking to be elected. Labelling all
expressions of private parties that tend to have an effect on the debate in elections as election
paraphernalia would be too broad a remedy that can stifle genuine speech. The better enforcement
will be the least restrictive means.

The message of petitioners are not what candidates would carry in their ads. Taken as a whole,
their message is an advocacy of a social issue it deeply believes in. It primarily advocates a stand
on a social issue and only secondarily, even almost incidentally, will cause the election or non-
election of a candidate.

The tarps exaggerate. “Team Patay” does not refer to a list of dead individuals or was it intended
to mean that the entire plan of the candidates in the list was to cause death intentionally. The tarp
parodies the intention of those in the list. “Team Patay” further emphasizes the theme of its author:
Reproductive health is an important marker for the church of petitioners to endorce. These are
different from the usual messages of candidates, which do not contain any sophisticated literary
allusion to any social objective. They simply exhort the public to vote for a person with a brief
description of the attributes of the candidate (Vote for x, sipag at tiyaga).

B. Some Considerations.- two paradigms of free speech- equality vis-à-vis liberty.


In an equality-based approach, politically disadvantaged speech prevails over regulation, but
regulation promoting political equality prevails over speech. This allows the government to
equalize “speaking power, such as protecting unpopular voices. This view acknowledges that there
are dominant political actors who can drown out the messages of others. This Court, in our
jurisprudence, acknowledged the Constitution’s guarantee for more substantive expressive
freedoms that take equality of opportunities into consideration during elections.

The other view is that considerations of equality of opportunity or equality in the ability of citizens
as speakers should not have a bearing in free speech doctrine. Under this view, the public is
trusted to make their own individual evaluations of speech, and government cannot intervene for
redistributive reasons. Ideas are best left to a freely competitive ideological market.

C. When private speech amounts to election paraphernalia.


Social inequality does have its effect on the exercise of free speech. Skewed distribution of
resources and cultural hegemony of the majority may drown out the speech of the minority. Those
who espouse the more popular ideas will have better reception than the dissenters of society. The
traditional view has been to tolerate the viewpoint of the speaker and content of his expression.
This view restricts laws or regulation allowing public officials to make judgments of the value
of such viewpoint or message content. This should be the principal approach.
But the requirements of the Constitution on equality in opportunity must provide limits to some
expression during electoral campaigns. Regulation of speech in electoral campaigns made by
candidates or members of their political parties or the party itself may be regulated as to
time, place, and manner. Regulation of speech in electoral campaigns made by persons who are
not candidates or, taken as a whole, principally advocates a social issue that the public must
consider during elections is unconstitutional. This is inconsistent with the guarantee of
according the fullest possible range of opinions from the electorate.

But speech by persons not candidates or speaking not as members of a political party may be
election paraphernalia, and thus regulation thereof would be valid, IF what is regulated is
declarative speech that, taken as a WHOLE, has for its PRINCIPAL OBJECT the
ENDORCEMENT OF A CANDIDATE only. The regulation should be 1) provided by law, 2)
reasonable, 3) narrowly tailored to meet the objective of enhancing the opportunity of all
candidates to be heard and considering the primacy of the guarantee of free expression, and (d)
demonstrably the least restrictive means to achieve that object. The regulation must only be as to
time, place, and manner (**content-neutral). The speech cannot be censored based on content.
For this purpose, it would not matter whether the speech is made on private property. But this is
not the situation here.

4. The tarp remains the private property of petitioners, even if readily seen by the public.
COMELEC ordered petitioners, private citizens, to remove the tarp from their own private
property. The absurdity of the situation is in itself an indication of unconstitutionality.

5. The tarp is not religious speech. There are two aspects of Art.III, S5: non-establishment cause
and free exercise and enjoyment of religious profession and worship. The second aspect is in issue
here.

Not all acts done by priests, bishops, or another religious make it immune from secular regulation.
The religious also have a secular existence and exist within a society regulated by law. The bishop
of Bacolod caused the posting of the tarp. Every act can be motivated by religious or moral
considerations. The characterizations of the religious of their acts are not conclusive on this court.
Thus, even if the Bishop said that the views of petitioners on the RH bill is connected to its Catholic
dogma, the Court’s powers of adjudication cannot be blinded.

The tarp on its face does not convey any religious doctrine of the Catholic church. The position of
the Catholic religion in the PH as to the RH law does not suffice to qualify the tarp as religious
speech. The enumeration of candidates on the face of the tarp precludes any doubt as to its nature
as speech with political consequences and not religious speech.

118. SWS v. COMELEC, GR 208062, April 07, 2015, Leonen, J.(Freedom of Expression)
FACTS:
SWS and Pulse Asia are social research and public polling firms who, among others, conduct pre-
election surveys.

SWS and Pulse Asia conducted a pre-election survey on voters’ preferences for senatorial
candidates. It published its findings. They asked “If the elections were held today, whom would
you most probably vote for as senators?” Representative Tiangco, Secretary General of the UN
Alliance (UNA), wrote Atty. Ladra, Director of COMELEC’s Law Department, and asked
COMELEC to compel SWS to either comply with the directive in the Fair Election Act and give
the names of subscribers who paid for the pre-election survey or be liable for an election offense.
Tiangco had asked SWS for the persons who paid for the pre-election survey conducted on
February 15-17, 2013 and those who subscribed to it, but SWS only gave him survey particulars
without disclosing the identity of persons who commissioned or subscribed thereto.

COMELEC Res. 9674 directed SWS and Pulse Asia, Inc. and other survey firms to submit to
COMELEC the names of all commissioners and payors of all surveys published from February
12, 2013 to April 23, 2013, including their subscribers.

COMELEC issued a subpoena, notifying SWS and Pulse Asia that a complaint for violation of the
Omnibus Election Code in relation to RA 9006, Fair Election Act, was filed against them.
Petitioners thus filed the present petition, assailing Res.9674 as being ultra vires.

ISSUE:
Whether COMELEC resolution 9674, implementing the Fair Election Act, requiring the election
surveys published by SWS and Pulse Asia to include the names of those who sponsored it and
subscribed to it, violates freedom of expression.
HELD: NO.
1. Not ultra vires.
Sec.5.2(a) of RA 9006, read in a manner consistent not only with its text but also its purpose,
sustains Res. 9674. RA 9006 was adopted to ensure equal opportunity for public service. It
provides means to realize the policy in Art.II, Sec.26 to guarantee equal access to opportunities for
public service. It also complies with Art.XIII, Sec.1, stating that Congress must give highest
priority to measures that reduce political inequalities by equally diffusing wealth and political
power for the common good. RA 9006 permits lawful election propaganda etc. It also governs
published surveys during elections.

Sec.5.2 enumerates the info that a person publishing an election survey during election period must
publish along with the survey itself: a. The name of the person, candidate, party, or organization
who commissioned or paid for the survey. Etc.

There are several possible effects of surveys on voters. 1) Bandwagon effect- electors rally to
support the candidate leading in the polls. 2) Underdog effect- electors rally to support the
candidate trailing in the polls. 3) Motivating effect- individuals who had not intended to vote are
persuaded to do so. 4) Demotivating effect- voters abstain from voting out of certainty that their
candidate will win. 5) Strategic voting- voting is influenced by the chances of winning. 6) Free-
will effect- voters cast their ballots to prove the polls wrong. Of these, the bandwagon effect has a
particular resonance and has been of concern.

Conformity pressures can suppress minority opinion. Surveys, far from being passive snapshots
of many viewpoints at a given time, can warp existing public opinion and mould it. This is the
reality that our policymakers, the Constitution’s framers, have seen fit to address. The inclusion of
election surveys in a law regulating election propaganda shows the recognition of lawmakers that
such surveys may influence voter preferences. The Fair Election act is a mechanism for ensuring
equality.

What is involved is petitioners’ freedom of speech to publish their findings. More specifically,
their political speech, referring to speech both intended and received as a contribution to public
deliberation about some issue, fostering informed and civic-minded deliberation. The nature of the
speech involved and RA 9006’s purpose calls into operation the equality-based approach to
weighing liberty to express vis-à-vis equality of opportunities. In this approach, politically
disadvantaged speech prevails over regulation, but regulation promoting political equality
prevails over speech.

Appraisal of speech in electoral campaigns principally designed to endorse a candidate, both by


candidates on one hand and private citizens on the other, is different. For those made by
candidates or political party members, it may be regulated as to time, place, and manner.
Regulation of speech in such context by those who are not candidates or who do not speak as
members of a political party which, taken as a whole, principally advocates a social issue that
the public must consider during elections is unconstitutional. Still, if those who are not
candidates make declarative speech that, taken as a whole, has for its principal object the
endorsement of a candidate only, it may be regulated. The regulation should be 1)provided by
law 2) reasonable 3) narrowly tailored to meet the objective of enhancing the opportunity of all
candidates to be heard, and 4) demonstrably the least restrictive means to achieve that object. The
regulation must only be as to time, place, and manner.

2. Election surveys is not election propaganda per se. on their face, they do not allude to preferred
candidates. But when published, the tendency is to shape voter preferences. In this respect,
published election surveys partake of the nature of election propaganda. It is thus declarative
speech in the context of an electoral campaign properly subject to regulation. Hence, S5.2’s
regulation of published surveys.

3. The requirement of Res.9674 of disclosing names of subscribers in light of requisites for valid
regulation of declarative speech by private entities in the context of election campaign.
Res.9674 finds basis in S5.2 of the Fair Election Act. S5.2 requires the disclosure of those who
sponsored the survey and those who paid for the survey. There is not only an important or
substantial state interest, but even a compelling one. Res.9674 thus passes scrutiny whether an
intermediate or strict standard is used. Constitutionally declared principles are a compelling
state interest. (Art.II, S26, Art.XIII,S26, Art.IX-C,S4, equal access to opportunities for public
service)

Res.9674 addresses the reality that election surveys are formative as it is descriptive. It partakes
of the nature of election propaganda. Res.9674 is narrowly tailored to enhance equal opportunity.
It is the least restrictive means to achieve this. While it does regulate expression or publication
of the surveys, it does not suppress it. There is no censorship aimed at election surveys and it can
still be published. Res.9674 only regulates the MANNER of publication.

4. Not prior restraint.


Prior restraint refers to official governmental restrictions on the press or other forms of expression
in advance of actual publication or dissemination. Res.9674 does not prohibit election surveys. It
only regulates the manner of publication. The disclosure requirement kicks in only upon, not prior
to, publication. The requirement to disclose subscribers is not unduly burdensome. Petitioners’
free speech rights must be weighed in relation to the Fair Election Act’s purpose of political
equality.

5. Res.9674 does not impair contracts in violation of Art.III, S10. The law is deemed written into
the contract between parties. This is a postulate of police power. Res.9674 is deemed written into
petitioners’ existing contracts.

6. However, the requirement in Res.9674 to “submit within 3 days from receipt of this Resolution
the names…” did not run. Petitioners were never actually served copies of Res.9674. It claims
that this is remedied by subsequent notice reproducing Res.9674’s dispositive portion. But
dismembering an official issuance by producing only a portion of it is not the same as serving on
the parties a copy of the official issuance itself. Petitioners may have been informed of what the
dispositive portion stated, but they were never notified and served copies of the resolution itself.
Res.9674 states compliance was expected “within 3 days from receipt of this resolution”, not its
partial, dismembered reproduction. Thus, there was no violation.

It also failed to give petitioners copies of the criminal complaint. Thus, it violated their right to
due process.

119. 1-United Transport Koalisyon (1-UTAK) v. COMELEC, GR 206020, April 14, 2015,
Reyes, J. (Freedom of Expression)
FACTS:
COMELEC Res.9615 was promulgated, the IRR of RA 9006 (Fair Election Act), in connection
with the May 13, 2013 elections and subsequent elections. Section 7 thereof prohibits certain forms
of election propaganda.
(f) To post, display or exhibit any election campaign or propaganda material outside of
authorized common poster areas, in public places, or in private properties without the
consent of the owner thereof.
(g) Public places referred to in the previous subsection (f) include any of the following:
xxx xxx xxx 5. Public utility vehicles such as buses, jeepneys, trains, taxi cabs, ferries,
pedicabs and tricycles, whether motorized or not; 6. Within the premises of public transport
terminals, such as bus terminals, airports, seaports, docks, piers, train stations, and the like.
Petitioner, thru its president Vargas, sought clarification from COMELEC regarding applicability
of Res.9615, Sec.7(g) (5)(6) vis-à-vis privately owned PUVs and transport terminals. He explained
that it impedes free speech of private owners of PUVs and transport terminals. He requested
COMELEC to reconsider implementing it.

COMELEC en banc issued Minute Resolution 13-0214, denying his request to reconsider
implementation. It said that in compliance with the O’Brien standards, RA 9006 furthers
equalizing opportunity, time, and space for all candidates and putting a stop to excessive campaign
spending. The purpose of placing political adverts on PUV or transport terminals is because it is
public and can be seen by all. Thus, it is logical that these be considered “public places.”
Hence this petition for certiorari.

ISSUE:
Whether Section 7 (g) items (5) and (6), in relation to Section 7 (f) of Resolution No. 9615, which
prohibits the posting of any election campaign or propaganda material, inter alia, in PUVs and
public transport terminals are valid regulations.

HELD:
1. The said sections are prior restraints on speech.
The prohibition affects the freedom of individuals, owners of PUVs and private transport
terminals, to express their preference through the posting of election campaign material in their
property, and convince others to agree with them. Posting election campaign material during
election period in PUVs and transport terminals has the penalty of revocation of public utility
franchise and the owner will be liable for an election offense. PUV owners and transport terminals
are forcefully inhibited from expressing their preferences.

2. Invalid content-neutral regulation.


Section 7 (g)(5)(6) are content-neutral regulations since they merely control the place where
election campaign materials may be posted. But it fails to satisfy all the requisites for a valid
content-neutral regulation (O’Brien). While Res.9615 furthers an important and substantial
governmental interest, ensuring equal opportunity, time, and space among candidates to have free,
orderly, etc. elections, and while the governmental interest in the prohibition is unrelated to the
suppression of free expression, this is not within the constitutionally delegated power of
COMELEC under Sec.4, Art.IX-C of the Constitution. There is also no necessity to restrict the
right to free speech of PUV owners and transport terminals.

3. COMELEC may only regulate the franchise or permit to operate and not the ownership per se
of PUVs and transport terminals. The constitutional grant in Art.IX-C,S4 of supervisory and
regulatory powers to COMELEC has its limits. While COMELEC has supervisory power vis-à-
vis the conduct and manner of elections, such power does not extend to the very freedom of an
individual to express his preference of candidates in an election by (Adiong case) placing election
campaign stickers on his vehicle. There is a difference between the franchise to operate
transportation and the ownership of vehicles used for public transport, operation vs ownership.
The right to operate a public utility may exist independently from ownership of facilities thereof.
The franchise to operate transpo utilities is a privilege granted to persons to engage in the business
of transporting people and goods. Ownership is a relation in private law by virtue of which a thing
pertaining to one person is completely subjected to his will in everything not prohibited by public
law or the concurrence with the rights of another. A restriction on franchise is a limitation on
ownership, but a limitation of ownership over a PUV is not necessarily a regulation on the
franchise.

The expression of opinion of an owner of a PUV, through posting election material on the vehicle,
does not affect considerations pertinent to the operation of the PUV. Regulating expression of
opinion in a PUV is not regulation of the franchise to operate but of the ownership of the vehicle
per se. In contrast, for example, a prohibition on posting of commercial adverts on windows of
buses as it hinders police from seeing whether the passengers inside are safe is a regulation on the
franchise or permit to operate. This has a direct relation to the operation of the vehicle as a PUV,
i.e. safety of passengers.

COMELEC does not have the constitutional power to regulate public transport terminals owned
by private persons. Thus, section 7 (g)(5)(6) is not within the constitutionally delegated power of
COMELEC to supervise/regulate the franchise or permit to operate of transportation utilities.

4. The restriction on free speech of owners of PUVs and transport terminals is not necessary to
further the stated governmental interest. The fourth requisite is missing in a valid content-neutral
regulation (no greater). The aim of equality of time, space, and opportunity of candidates may be
achieved without intrusion on right to expression.

RA 9006 is implemented by Res.9615, but RA 9006 did not provide prohibition on posting of
election campaign materials on PUVs and transport terminals. There are also more than sufficient
provisions in our present election laws to ensure equal time, space, and opportunity to candidates
in elections (S6, 9, 13 RA 9006). A strict implementation of these provisions would suffice to
achieve the governmental interest of equality for candidates. There is no necessity to curtail
the free speech of PUV owners and transport terminals.

5. COMELEC tries to justify by saying that PUVS and terminals hold a captive audience-
commuters who have no choice but be subjected to the blare of political propaganda. While
private PUV and terminals have a right to express their views to those who wish to listen, they
have no right to force their message upon an audience incapable of declining to receive it.

This is untenable. The captive-audience doctrine states that when a listener cannot, as a
practical matter, escape from intrusive speech, the speech can be restricted. This recognizes
that a listener has a right not to be exposed to an unwanted message in circumstances in which the
communication cannot be avoided. A regulation based on this doctrine is in the guise of censorship
which selectively shields the public from some kinds of speech on the ground that they are more
offensive than others. Such selective restrictions may be upheld only when the speaker intrudes
on the privacy of the home or the degree of captivity makes it either impossible or impractical
for the unwilling viewer or auditor to avoid exposure.

The US SC struck down an order prohibiting public utility companies from including in their
monthly bills discussing controversial issues of public policy as the customers could escape
exposure by simply throwing the bill. It also invalidated an ordinance prohibiting a drive-in movie
theater to exhibit films with nudity when the screen is visible from a public street or place as the
degree of captivity is not so great as to make it impracticable for an unwilling viewer to avoid
exposure as he can readily avert his eyes.

Thus, a regulation based on the captive-audience doctrine may not be justified if the supposed
captive audience may avoid exposure to intrusive speech. The commuters are not forced or
compelled to read the election campaign materials on PUVs and terminals. They can also
DECLINE to receive the messages in the election materials since they may simply AVERT
their eyes if they find the same unbearable intrusive.
6. Violation of equal protection.
The classification under Sec.7(g)(5)(6) is not limited to existing conditions and applies equally to
all members of the same class. But it is not based on substantial distinction and is not germane
to the purpose of the law. There is distinction between PUVs and terminals and private vehicles
and other properties in that the former, to be considered as such, needs to secure a franchise or
permit. The prohibition as mentioned regulates ownership per se, not the franchise.

There is no substantial distinction between PUV and terminal owners and owners of private
vehicles and other properties. The ownership of PUVs and terminals, though available for public
use, remains private. The distinction between them in terms of ownership is only superficial.

The fact that PUVs and terminals are used by the public is not substantial justification to set them
apart from private vehicles and other properties. Any election campaign material posted on PUVs
and terminals would be seen by many people but such materials posted on private vehicles and
other places frequented by the public (commercial establishments) would be also seen by many
people.

The classification also has no relation to the purpose of providing equality for candidates to
elections. Thus, Res.9615, Sec7(g)(5)(6) in relation to Sec7(f) are void.

ASSEMBLY AND PETITION


120.Jose Reyes v. Ramon Bagatsing, GRL-65366, November 09, 1983, Fernando, C.J.
(Assembly and Petition)
FACTS:
Petitioner retired Justice JBL Reyes, on behalf of the Anti-Bases Coalition, sought a permit from
Manila City to hold a peaceful march and rally on October 26, 1983 starting from Luneta to the
gates of the US embassy, 2 blocks away. Once there and in an open space of public property, a
short program would be held. After 2 brief speeches, a petition based on the resolution adopted on
the last day by the International Conference for General Disarmament, World Peace and the
Removal of All Foreign Military Bases held in Manila, would be given to a representative of the
embassy or its personnel to be delivered to the US Ambassador. There was assurance that the
march would be peaceful.

As of October 20, 1983, there was no action on the request. Hence, this suit for mandamus with
prayer for injunction. On October 25, the answer of respondent Mayor Bagatsing was filed and it
turned out that the permit was denied on October 19. Petitioner was unaware of such denial as it
was sent through mail. The reason for the denial was due to police intelligence reports which
strongly militate against the advisability of issuing the permit at this time and at the place
applied for. There was persistent intelligence reports of plans of criminal elements to infiltrate or
disrupt any assembly where a large number of people is expected to attend. Mayor Bagatsing
suggested that a permit may be issued if the rally is to be held at the Rizal Coliseum or other
enclosed area instead where the safety of the participants and the public may be ensured.
The Court issued a minute resolution on October 25, granting the injunction on the ground that
there was no showing of clear and present danger of a substantive evil to justify denial of a permit.
The resolution was without prejudice to a more extended opinion, hence this detailed exposition.

ISSUE:
Whether the Mayor of Manila, denying the permit to rally of members of the Anti-Bases Coalition
despite assurance that it would be peaceful, where the rally is to be held starting from Luneta then
towards the US Embassy 2 blocks away where a short program will be held culminating in giving
a petition to a representative of said embassy for delivery to the US Ambassador, on the ground
that there might be subversives who would take the opportunity of a large gathering of people to
commit crimes thus prompting the mayor to suggest another place for the rally, acted properly.
HELD: NO.
If the peaceful means of communication cannot be availed of, resort to non-peaceful means may
be the only alternative. Freedom of expression and peaceable assembly provides for a safety valve,
allowing parties the opportunity to give vent to their views, even if contrary to the prevailing
climate of opinion. Its value may lie in that there may be something worth hearing from the
dissenter. This is to ensure a true ferment of ideas. But there are limits. What is guaranteed is
peaceable assembly. One may not advocate disorder in the name of protest.

Without a clear and present danger of a substantive evil, there can be no legal objection on the
choice of Luneta as the starting place of the peace rally. Streets and parks have been immemorially
held in trust for the use of the public and for purposes of assembly, communicating thoughts
between citizens, and discussing public questions. The privilege of a citizen to use the streets and
parks for communication of views on national questions may be regulated in the interest of all.
It is not absolute, but relative, and must be exercised in subordination to the general comfort and
peace and order. But it must not, in the guise of regulation, be denied.

There is also no valid objection to the use of streets to the gates of the US embassy, 2 blocks
away from Roxas Blvd. A law requiring persons using public streets for a parade or procession
to procure a special license therefor from the local authorities is not an unconstitutional
abridgment of the rights of assembly or of free speech and press where the licensing authorities
are strictly limited to a consideration of time, place, and manner of the parade, in issuing the
license, with a view to conserving public convenience and of affording an opportunity to
provide proper policing, and are not invested with arbitrary discretion to issue or refuse the
license. Civil liberties as guaranteed by the Constitution imply the existence of an organized
society without which liberty itself would be lost in the excesses of unrestricted abuses. The
authority of a municipality to impose regulations to assure safety of the people in the use of public
highways is consistent with civil liberties and is a means of safeguarding the good order upon
which they ultimately depend. Where a restriction of the use of highways is desired to promote
public convenience in the interest of all, it cannot be disregarded by the attempted exercise of some
civil right.

There is a novel aspect to this case. The PH signed and ratified the Vienna Convention on
Diplomatic Relations, which requires it to “protect the premises of the mission against intrusion
or damage and to prevent any disturbance of the peace of the mission.” If there were a clear and
present danger of any intrusion, damage, or disturbance of the peace of the mission, there would
be justification to deny the permit.

There was no clear and present danger. There is freedom of access, as mentioned, to public places
especially to parks and streets. The question if to preserve free speech and peaceable assembly is
not as to the auspices under which the meeting is held but as to its purpose; not as to the relations
of the speakers, but whether their utterances transcend the bounds of free speech. There could be
danger to public peace if the gathering were marked by turbulence. It would then not be peaceful.
It is true that Mayor Bagatsing, the licensing official, has discretion in determining if a permit
would be granted. But this is not an unfettered discretion. While prudence requires a realistic
appraisal not of what may possibly occur but of what may probably occur, given all relevant
circumstances, still, the assumption, especially where the assembly is scheduled in a specific
public place, is that the permit must be for the assembly being held there. The exercise of such
right is not to be abridged on the plea that it may be exercised in some other place.

The mere assertion that subversives may infiltrate the ranks of the demonstrators does not suffice.
There was even assurance from the police superintendent that the police force is in a position to
cope with such emergency if it arises. Petitioner even assured that the assembly would be peaceful.

Applicants for a permit to hold assembly should inform the licensing authority of the date, the
public place where and the time when it will take place. If it were a private place, only the
consent of the owner is required. The application should be filed ahead of time to let the official
appraise valid objections. An indispensable condition for its refusal is that the clear and present
danger test be the standard used. If he is of the view that there is such danger, the applicants must
be HEARD on the matter. Thereafter, his decision, favorable or adverse, should be transmitted
to them at the earliest opportunity. Thus, if they want, they can have recourse to the proper
judicial authority.

As for the applicability of Ordinance 7295, prohibiting demonstrations within a radius of 500 feet
from any foreign mission, it is admitted that it finds support in Art.22 of the Vienna Convention
on Diplomatic Relations. But there was no showing that the distance between the chancery and
embassy gate is less than 500 feet. Even if this was shown, it does not follow that Bagatsing could
legally act the way he did. The validity of his denial could still be challenged. The unconstitutional
application of the ordinance to the right of peaceable assembly could be argued. Since there was
no proof, the need to pass on such issue was obviated.

121. Bayan Karapatan v. Eduardo Ermita, GR 169838, April 25, 2006, Azcuna, J. (Assembly
and petition)
FACTS:
There are 3 petitioners. Bayan et al. are citizens and taxpayers claiming their rights as an
organization and individual were violated when their rally on October 6, 2005 was violently
dispersed by policemen implementing BP880. Jess del Prado, et al., 26 individuals, allege they
were injured, arrested, and detained when a peaceful mass action they held on September 26, 2005
was preempted and violently dispersed by police. On October 5, they participated with a group
and marched to Malacañang to protest Palace issuances, but the protest was likewise violently
dispersed. Kilusang Mayo Uno (KMU) et al., claim that they conduct peaceful mass actions and
their rights as organizations and as individual members, specifically the right to peaceful assembly,
are affected by BP 880 and the policy of calibrated preemptive response (CPR) being followed
to implement it. KMU et al. had a rally on October 4 at MEndiola bridge but police blocked and
forcibly dispersed them.

Petitioners seek to stop the violent dispersals of rallies under the “no permit, no rally” policy and
the CPR policy recently announced. BP880 is “The Public Assembly Act of 1985”- “AN ACT
ENSURING THE FREE EXERCISE BY THE PEOPLE OF THEIR RIGHT PEACEABLY TO
ASSEMBLE AND PETITION THE GOVERNMENT [AND] FOR OTHER PURPOSES”.

Sec.4 requires a written permit to organize or hold a public assembly in a public place, except if
to be made in a freedom park established by law or ordinance or in a private property, in which
case only the consent of the owner is required, or in the campus of a government operated
educational institution, which shall be subject to the rules of said institution. Sec10 requires law
enforcement agencies, when their assistance is requested by the organizers, to perform their duties
to protect the right peaceably to assemble. It gives them guidelines to always be in complete
uniform and observe “maximum tolerance”, they shall not carry firearms, only baton or shields
etc., and tear gas or water cannons etc. shall not be used unless the assembly is attended by
violence, deliberate destruction of property, or serious threats of violence.

Sec.11 allows dispersal of an assembly with permit if it becomes violent. Sec.12 allows dispersal
of public assembly without permit where a permit is required, but peacefully.

On September 21, 2005, ES Eduardo Ermita released a statement, saying that due to reports of
plans of anti-government groups to inflame the political situation, the PNP and LGUs were
instructed to strictly enforce a “no permit, no rally” policy and to disperse persons violating this
and to arrest persons violating laws and ordinance on the proper conduct of mass actions. The rule
of calibrated preemptive response (CPR) is “now in force” in lieu of maximum tolerance.

Petitioners argue that BP 880 and the CPR is unconstitutional.

ISSUE:
Whether a law, BP880, requiring a permit from the mayor before a rally may be held and allowing
the mayor to deny such permit only if there is clear and present danger of a substantive evil that
Congress has a right to prevent, is valid.
HELD:YES.
Petitioners have standing as they have engaged in public assemblies without the required permits
to press their claim that no such permit can be required without violating the constitutional
guarantee.

BP 880, S4-6, practically codify the ruling in Reyes v. Bagatsing. (inform licensing authority of
date, where, and when; if private, only owner’s consent is required; application filed ahead of time
etc.) Thus, BP 880 is not an absolute ban of public assemblies but a restriction that simply
regulates the time, place, and manner of the assemblies. This is a “content-neutral” regulation
of holding public assemblies. Bp 880 applies to all kinds of public assemblies that would use
public places. Maximum tolerance is for the protection and benefit of all rallyists and is
independent of the content of the expressions in the rally.

The permit can only be denied on the ground of clear and present danger to public order, safety,
convenience, morals, or health. This is a recognized exception to the exercise of the right even
under the UDHR and the ICCPR.

“Public” does not have to be defined in the law as its ordinary meaning is well-known. It is “an
organized body of people; a group of people distinguished by common interests or characteristics”
– Webster’s dictionary.

Not every expression of opinion is a public assembly. The law refers to "rally, demonstration,
march, parade, procession or any other form of mass or concerted action held in a public place."
So it does not cover any and all kinds of gatherings. Neither is it overbroad as it regulates the right
to peaceful assembly and petition only to the extent needed to avoid a clear and present danger
of substantive evils Congress has the right to prevent. There is also no prior restraint since
the content of speech is not relevant to the regulation.

-Freedom Parks
As to the delegation of powers to the Mayor, the law has a sufficient standard – the clear and
present danger test in S6(a). S 15 provides an alternative forum for those who cannot wait for
permits through the creation of freedom parks where no prior permit is needed. It requires
every municipality and city to establish at least one suitable “Freedom park” or mall.

But as to compliance with the freedom park requirement, the degree of observance that a freedom
park be made within 6 months from effectivity of BP880 in 1985, 20 years ago, is pathetic and
regrettable. (Only Cebu has a freedom park as of the decision, and manila had one until it was
converted into a golf course.)

Since the existence of such freedom parks is essential to the law’s system of regulation of the
people’s exercise of their right, the Court rules that after 30 days from finality of this decision,
no prior permit may be required for the exercise of such right in any public park or plaza of
a city or municipality until it complies with S15 of the law. For without such alternative forum,
to deny the permit would be to deny the right.

-CPR standard.
The SOLGEN conceded that the use of this term should be discontinued as it does not mean
anything other than the maximum tolerance policy in BP880. When it was said that CPR is
enforced “in lieu of maximum tolerance”, this was not referring to the legal definition but to the
distorted and much abused definition that it acquired in the minds of the public since. SOLGEN
wanted to disabuse the minds of the public from the notion that law enforcers would shirt from
their responsibility of keeping peace even when confronted with dangerous behavior. The message
is that the police would no longer be lax in enforcing the law but would then follow it to the letter.

CPR serves no purpose if it means the same as maximum tolerance. If it means something else, it
would be illegal. What should be followed is that mandated by law, maximum tolerance.
-To address the situation where mayors do not act on applications for a permit and when police
demand it, and the rallyists could not produce one, the rally is immediately dispersed, as part of
maximum tolerance, rallyists who can show the police an application duly filed on a given date
can, after TWO DAYS from said date, rally in accordance with their application without
need to show a permit, the GRANT of the permit being then PRESUMED under the law. It
will be the burden of the authorities to show denial of the application, in which case the rally may
be dispersed peacefully.

122. IBP v. Mayor Lito Atienza, GR 175241, February 24, 2010, Carpio-Morales, J.
(Assembly and Petition)
FACTS:
On June 15, 2006, the IBP filed with the Manila Mayor a letter application for permit to rally at
the foot of Mendiola Bridge on June 22, 2006. On June 16, Mayor Atienza issued the permit,
allowing the rally on the date applied for but indicated the venue as Plaza Miranda. Thus,
petitioners filed with a CA a petition for certiorari. Since it was unresolved within 24 hours,
petitioners filed with this court a petition for certiorari, assailing the CA’s inaction within the
period provided under the Public Assembly Act of 1985.

The SC, by resolutions, denied the petition for being moot and denied the relief that the petition
be heard on its merits in view of the pendency of the CA case. The rally pushed through at
Mendiola Bridge after Cadiz (IBP president) discussed with P/Supt. Paglinawan whose contingent
from the MPD earlier blocked petitioners from going thereto and the participants voluntarily
dispersed after the peaceful program.

The Manila Police District (MPD) instituted a criminal action against Cadiz for violating the Public
Assembly Act by staging a rally at a venue not indicated in the permit. Meanwhile, the CA ruled
that the petition therein had become moot and lacked merit. Hence this petition for certiorari.

ISSUE:
Whether Mayor Atienza of Manila, in modifying the venue in the IBP’s application for permit to
rally in Mendiola Bridge to Plaza Miranda without specifying his reasons for such modification,
committed grave abuse of discretion.
HELD: YES.
1. Mootness.
The petition filed with the CA On June 21 had become moot upon the passing of the date of the
rally on June 22, 2006. A Moot case is one that ceases to present a justiciable controversy by virtue
of supervening events, so that a declaration thereon would be of no practical use or value. But
Courts still resolve moot issues if it is capable of repetition yet evading review. Here, the question
of the legality of a modification of a permit to rally will arise each time the terms of an
intended rally are altered, yet it evades review owing to the limited time in processing the
application where the shortest allowable period is five days prior to the assembly.

2. The CA found no grave abuse on the Mayor’s part since BP880 does not categorically require
him to specify in writing the imminent and grave danger which warrants denial or modification of
the permit. But Sec.6 of the Public Assembly Act reads:
(c) If the mayor is of the view that there is imminent and grave danger of a substantive evil
warranting the denial or modification of the permit, he shall immediately inform the
applicant who must be heard on the matter.
(d) The action on the permit shall be in writing and served on the application [sic] within
twenty-four hours.
(e) If the mayor or any official acting in his behalf denies the application or modifies the
terms thereof in his permit, the applicant may contest the decision in an appropriate court
of law.
(f) In case suit is brought before the Metropolitan Trial Court, the Municipal Trial Court,
the Municipal Circuit Trial Court, the Regional Trial Court, or the Intermediate Appellate
Court, its decisions may be appealed to the appropriate court within forty-eight (48) hours
after receipt of the same. No appeal bond and record on appeal shall be required. A decision
granting such permit or modifying it in terms satisfactory to the applicant shall, be
immediately executory.

The sole justification for a limitation on the exercise of the right to peaceably assemble is a grave
and imminent danger of a serious evil to public safety etc. It is an indispensable condition to such
refusal or modification that the clear and present danger test be the standard for the decision
reached. If he is of the view that there is such danger, the applicants must be heard on the matter.

In modifying the permit outright, Mayor Atienza gravely abused his discretion when he did not
immediately inform the IBP who should have been heard first on the matter of his perceived
grave imminent danger that may warrant the changing of the venue. The opportunity to be
heard precedes the action on the permit, since the applicant may directly go to court after an
unfavorable action on the permit.
Respondent failed to indicate how he arrived at modifying the terms of the permit against the said
standard, which is an indispensable condition to modification (or refusal). Nothing in the permit
adverts to an imminent grave danger, which “blank” denial or modification would, when
granted imprimatur as the CA did, render illusory any judicial scrutiny thereof.

The exercise of such right is not to be abridged on the plea that it may be exercised in some other
place. While respondent has discretion in determining if the permit should be granted, it is not
unfettered discretion. The assumption is that the permit must be for the assembly being held there.

123. Crispin Malabanan v. Hon. Anastacio Ramento, GR 62270, May 21, 1984, Fernando,
C.J. (Assembly and Petition)
FACTS:
Petitioners are students of the Gregorio Araneta University Foundation while respondent Ramento
is the Director of the NCR Ministry of Education, Culture and Sports and the Gregorio Araneta
University Foundation.

Petitioners were officers of the Supreme Student Council of respondent university. They sought
and were granted by the school authorities a permit to hold a meeting from 8am-12nn on August
27, 1982. Pursuant thereto, they held a general assembly at the Veterinary Medicine and Animal
Science (VMAS) basketball court, the place in the permit, not in the basketball court as therein
stated but at the second floor lobby. They manifested in vehement and vigorous language their
opposition to the proposed merger of the Institute of Animal Science with that of Agriculture. At
10:30am, they marched toward the Life Science Building and continued their rally. This was
outside the area covered by the permit. They uttered language severely critical of university
authorities and used megaphones. There was thus disturbance of classes. The non-academic
employees stopped their work because of the noise. They were asked to explain why they should
not be held liable for illegal assembly.

Petitioners were preventively suspended for failure to explain holding an illegal assembly in front
of the Life Science Bldg. The validity thereof was challenged by petitioners before the CFI in a
petition for mandamus and before the Ministry of Education. Ramento found petitioners guilty of
the charge of violating par.146(c) of the Manual for Private Schools or illegal assembly. The
penalty was suspension for one academic year. Hence this petition.

The SC issued a TRO that allowed petitioners to finish their studies.

ISSUE:
Whether students in schools have a right to peaceable assembly and petition inside private school
premises.
HELD: YES.
There is a need to pass upon the issue even if it may be moot since the issue involves freedom of
peaceable assembly and free speech.

The penalty of Ramento was unduly severe. While petitioners held the rally at a place other than
that specified in the permit, rallied for a longer period than allowed, and disturbed classes and
work, such undesirable consequences could have been avoided if they rallied in the basketball
court as indicated in the permit. But suspending them for 1 year is out of proportion to their
misdeed. A much lesser penalty is appropriate.

The invocation of the right to peaceable assembly carries with it the implication that the right to
free speech was likewise disregarded. The case here is different from Reyes v. Bagatsing. The
assembly was to be held not in a public place but in private premises of respondent university.
Petitioners sought the consent of the university. (If the place were private, only the consent of the
owner is required.) It was granted.

Petitioners are entitled to invoke their rights to peaceable assembly and free speech like the rest of
the citizens to express their views. They do not shed their constitutional rights to freedom of
speech or expression at the schoolhouse gate. While the authority of educational institutions
over students is recognized, it cannot go so far as to be violative of constitutional safeguards. The
principal use of schools is to accommodate students during the prescribed hours for certain types
of activities. Among those activities is personal intercommunication among students. A student’s
rights do not embrace merely classroom hours. When he is in the cafeteria, playing field, or on
campus during authorized hours, he may express his opinions even on controversial subjects, if
he does so without substantially interfering with the requirements of appropriate discipline
in the operation of the school and without colliding with the rights of others. But the conduct
of. The student, in or out of class, for any reason, which materially disrupts classwork or involves
substantial disorder or invasion of others’ rights, is not immunized by freedom of speech.
The students believed that the merger of the institute of animal science with the institute of
agriculture would be a serious problem and would result in increased tuition fees, an additional
headache for their parents. With an audience goading them on in the course of such demonstration,
it is understandable if extremely critical or even vitriolic utterances were let loose. Student leaders
are hardly timid. They are likely to be assertive. They would be ineffective if during a rally they
speak in the guarded language of the academe. With the activity taking place in school premises
and during daytime, there is no clear and present danger of public disorder. But this is without
prejudice to taking disciplinary action for conduct which disrupts classwork or invasion of others’
rights.

In US v. Apurado, 500 residents barged into a municipal council chamber, demanding that the
municipal treasurer etc. be dismissed. They were unarmed except some had canes. They were fairly
orderly and well-behaved. They were charged with sedition. The SC said that if the prosecution be
permitted to seize upon every instance of disorderly conduct as an excuse to characterize the
assembly as seditious, then the right to assemble and petition would become a delusion and the
attempt to exercise it on the most righteous occasion and in the most peaceable manner would
expose those who took part therein to the severest punishment. Utmost discretion must be
exercised in drawing the line between disorderly and seditious conduct and between an
essentially peaceable assembly and tumultuous uprising.

But petitioners cannot be totally absolved. They violated the terms of the permit. The rally was in
a place other than designated. It continued for a longer period than allowed.(It went on til 5:30pm.)
But on those facts, an admonition, even a censure, but certainly not suspension, could be
appropriate penalty. If proportionality between the offense and the sanction is not followed, there
would be arbitrariness. This would give rise to a due process question. We thus hold that a one-
week suspension is enough.

Guidelines, like in Reyes v. Bagatsing, is appropriate. The rights to peaceable assembly and free
speech are guaranteed students of educational institutions. Their exercise must not be subjected
to previous restraint or subsequent punishment unless there is clear and presentdanger. The
utmost leeway and scope is accorded to the content of placards displayed and the utterances
made. But the peaceable character of an assembly could be lost by an advocacy of disorder under
the name of dissent, whatever grievances might be aired being susceptible to correction through
the ways of law. If the assembly is to be held in school premises, permit must be sought from
school authorities, who cannot deny it arbitrarily or unreasonably. In granting the permit,
there may be conditions as to time and place to avoid disruption of classes or stoppage of work
of non-academic personnel. But even if there be violations of its terms, the penalty should not be
disproportionate to the offense.

124. Tinker v. Des Moines Sch. Dist., 393 US 503, February 24, 1969, Justice Fortas
(Assembly and Petition)
FACTS:
John Tinker, 15yo, and petitioner Christopher Eckhardt, 16 yo, attended high schools in Des
Moines, Iowa. Petitioner Mary Tinker, John’s sister, was a 13yo student in junior high school. In
December 1965, a group of adults and students in Des Moines held a meeting at the Eckhardt
home. They determined to publicize their objections to the hostilities in Vietnam and their support
for a truce by wearing black armbands during the holiday season and by fasting on December 16
and New Year’s Eve. Petitioners and their parents previously engaged in similar activities, and
they decided to participate also.

The principals of Des Moines became aware of this plan. They met and adopted a policy that any
student wearing an armband to school would be asked to remove it and if he refused he would be
suspended until he returned without the armband. Petitioners were aware of the new regulation.

Mary and Christopher wore black armbands to their schools. John Tinker wore his the next day.
They were all sent home and suspended until they came back without the armbands. They did not
return to school until after the planned period for wearing the armbands expired- until after new
year’s day.

Petitioners filed this complaint thru their fathers and prayed for an injunction to retrain the
respondent school officials and members of the board from disciplining petitioners. The district
court and CA ruled against petitioners.

ISSUE:
Whether a school may validly suspend its students who wear black armbands in protest of a
controversial national issue to prevent controversy in the school.
HELD: NO.
The wearing of armbands is not potentially disruptive conduct. It was akin to “pure speech” which
is entitled to comprehensive protection under the First Amendment. First Amendment rights are
available to teachers and students. They do not shed their rights to freedom of speech or expression
at the schoolhouse gate. The Court has also affirmed the comprehensive authority of the States and
school officials to prescribe and control conduct in schools. The problem is in the area where
students in the exercise of First Amendment rights collide with the rules of school authorities.

The school sought to punish petitioners for a silent, passive expression of opinion,
unaccompanied by any disorder or disturbance. There is no evidence of interference with the
school’s work or of collision with the rights of other students. This case does not concern speech
or action that intrudes upon the school’s work or the rights of other students. Only a few of the
18000 students wore black armbands. Only 5 students were suspended therefor.

The District Court held that the action of the school was reasonable because of fear of disturbance.
But undifferentiated apprehension of disturbance is not enough to overcome freedom of
expression. Any variation from the majority’s opinion may inspire fear. Any spoken word in class
or on campus that deviates from the views of another may start an argument or cause disturbance.
But our Constitution says we must take this risk.

In order for the State in the person of school officials to justify prohibition of an expression of
opinion, it must show that its action was caused by something more than a mere desire to avoid
the discomfort and unpleasantness that accompany an unpopular viewpoint. Where there is
no showing that engaging in the forbidden conduct would materially and substantially interfere
with the requirements of appropriate discipline in the operation of the school, the prohibition
cannot be sustained. There is no such showing here.

The action of the school authorities appears to have been based upon an urgent wish to avoid
controversy which might result from the expression of opposition to this nation’s part in the
conflagration in Vietnam. The order did not extend to other symbols of controversial significance
like to those students in some schools who wore buttons relating to national political campaigns
and the iron cross, symbol for Nazism. Instead, a particular symbol, black armbands, worn to
exhibit opposition to the nation’s involvement in Vietnam, was singled out. The prohibition of
expression of one particular opinion, at least without evidence that it is necessary to avoid
material interference with schoolwork, is not constitutionally permissible.

School officials do not possess absolute authority over their students. Students in and out of
school are “persons” under our constitution. They still possess fundamental rights. A students’
rights do not embrace merely classroom hours. The principal use to which schools are dedicated
is to accommodate students during prescribed hours for the purpose of certain types of activities.
Wherever he is in school during authorized hours, he may express his opinions, even on
controversial subjects like the conflict in Vietnam, if he does so without “materially and
substantially interfering with the requirements of appropriate discipline in the operation of the
school” and without colliding with the rights of others. Otherwise, conduct which materially
disrupts classwork etc. is not immunized by freedom of speech.
125. Ariel Non v. Hon. Sancho Dames, GR 89317, May 20, 1990, Cortes, J. (Assembly and
Petition)
FACTS:
Petitioners are students in private respondent Mabini Colleges, Inc. in Daet, Camarines Norte.
They were not allowed to re-enroll for the academic year 1988-1989 for participating in student
mass actions against the school. Petitioners filed a petition seeking readmission with the trial court,
but it was dismissed. It followed the ruling in Alcuaz et al. v. PSBA, et al. where it was held a
student, once enrolled, is enrolled only for one semester and that the end of the semester is also
the termination of the contract between student and school. Thus, the school may deny the student
readmission.

Hence this petition for certiorari.

ISSUE:
Whether a school may deny re-enrollment of students for participating and leading mass actions
against the school.
HELD: NO.
The Alcuaz ruling is at the heart of the controversy. After one semester, the contract between
student and school is terminated. The school cannot be compelled to enter into another contract
with the students and teachers. The courts have no power to make contracts for the parties. The
petition of students therein barred from re-enrolling after leading mass assemblies and putting up
barricades were dismissed. But in light of compassionate equity, students who, in view of absence
of academic deficiencies, were scheduled to graduate during the school year when the petition was
filed, should be allowed to re-enroll and to graduate in due time.
This case is not a simple refusal of re-enrollment. Petitioners led and participated in student
protests. This is a case that focuses on the right to speech and assembly as exercised by students
vis-à-vis the right of school officials to discipline them.

1. The student does not shed his rights at the schoolgate.


The ruling in Malabanan v. Ramento has been applied in three other decisions of the Court. The
Court reiterated in Villar v. TIP that the exercise of freedom of assembly could not be a basis
for barring students from enrolling. It enjoined the school and its officials from acts of
surveillance, blacklisting, suspension, and refusal to re-enroll. But the Court allowed non-
enrollment of students who incurred academic deficiency on the ground of academic freedom to
set academic standards to determine what failing grades suffice for expulsion of students. In
Guzman v. NU, the school was directed to allow the students to re-enroll without prejudice to
disciplinary proceedings that may be conducted due to their participation in protests that led to
stoppage of classes.

2. Permissible limitations on student exercise of constitutional rights within the school.


While high regard is afforded to exercise of rights to free speech and assembly, school authorities
are not powerless to discipline students (materially or substantially disrupt classwork= not
immunized by free speech). But the imposition of disciplinary sanctions requires due process as
stated in Guzman. The penalty must also be proportionate to the offense.

3. Since the ruling in Malabanan in 1984, schools circumvent the doctrine by not suspending or
expelling students who participate in student mass actions but instead deny them re-enrollment.

4. Nature of contract between school and student.


The contract between school and student is not an ordinary contract. It is imbued with public
interest, considering the high priority of the Constitution to education abd the grant to the
State of supervisory and regulatory powers over all educational institutions.

The school cannot justify its actions based on par.137 of the Manual of Regulations for Private
Schools, which provides that “when a student registers in a school, he is enrolling for the entire
semester.” The Court in Alcuaz construed this as authority for schools to refuse enrollment to a
student on the ground that his contract, which has a term of one semester, has already expired.

But this “termination of contract” theory does not even find support in the Manial. Par.137 merely
clarifies that a college student enrolls for the entire semester. It protects schools where tuition fees
are collected on installment basis. Even if a student does not complete the semester, but has stayed
on for more than 2 weeks, he may be required to pay his tuition for the whole semester before he
is given his credentials for transfer. This is the import of par.137, not the interpretation in Alcuaz.

Contrarily, the Manual recognizes in par.107 the right of the student to be enrolled in his course
for the entire period he is expected to complete it. This has been translated into a right in BP 232,
Education Act of 1982:
2.The right to freely choose their field of study subject to existing curricula and to continue
their course therein up to graduation, except in cases of academic deficiency, or violation
of disciplinary regulations.
5. Academic freedom is not a ground for denying the students’ rights. The right of an institution
to set academic standards cannot be utilized to discriminate against students who exercise their
constitutional rights to speech and assembly, for there will be violation of their right to equal
protection.

Of 13 petitioners, 5 do not have academic deficiencies. Thus, these 5 were refused re-enrollment
without just cause and should be allowed to re-enroll. Also, petitioners were not afforded due
process in the manner expressed in Guzman before they were refused re-enrollment. The decision
to refuse re-enrollment due to failing grades seems to be mere afterthought. Also, some of the 8
students with failing grades only have one or two failures. Their failures cannot be considered
marked academic deficiency.

As to those with several failures, it is unclear if the failures were incurred in only one semester or
through the course of several semesters. Neither were the academic standards of respondent school
alleged. Thus, there is insufficient information.

However, this is without prejudice to disciplinary action for breach of discipline if the facts so
warranted. In line with Malabanan, petitioners could be disciplined. But the penalty should be
commensurate to the offense and, as set forth in Guzman, there must be procedural due process.
But this disciplinary proceeding are already moot as petitioners have already been excluded from
the school for 4 semesters, which is more than sufficient penalty for any breach of discipline. To
still subject them to disciplinary proceedings would serve no useful purpose.

FREEDOM OF RELIGION
126. Re: Letter of Tony Q. Valenciano, Holding of Religious Rituals at the Hall of Justice
Building in Quezon City, AM 10-4-19-SC, March 07, 2017 (Freedom of Religion)
FACTS:
This controversy arose from letters of Tony Valenciano on January 6, 2009 addressed to then CJ
Reynato Puno. In his first letter, Valenciano reported that the basement of the Hall of Justice (HoJ)
in QC had been converted into a Roman Catholic Chapel, complete with offertory table, images
of Cahtolic religious icons, a canopy, electric organ, and projector. He belived that such practice
violated the constitutional provision of separation of Church and State and against appropriation
of public money or property to benefit a sect, church, denomination, or other system of religion.

He claimed that holding masses at such basement showed that it tended to favor Catholic litigants.
The rehearsals of the choir caused great disturbance to other employees. The public could no longer
use the basement as resting place. The employees and litigants could not attend to personal
necessities like going to the lavatories as they could not traverse the basement between 12nn to
1:15pm. The employees became hostile to each other as they vied for the right to read the epistle.
The water supply in the building was cut off during mass as the generator was turned off to ensure
silence.

CJ Puno referred the letter to then Deputy Court Administrator (DCA) and OIC of the Office on
Halls of Justice, Antonio Dujua. DCA Dujua refererd it to Executive Judge Bay and Maceren of
the MeTC for their comments. Judge Maceren clarified that the basement was known as the prayer
corner. He opined that its use for masses did not violate the constitutional prohibition on use of
public property for religious purposes as the religious character of such use was merely incidental
to a temporary use.

Valenciano wrote another letter on May 13, 2009 complaining that masses continued and called
for appropriate action. On March 23, 2010, he wrote another letter praying that rules be
promulgated by the Court to put a stop to the masses or other religious rituals. The Court referred
the matter to OCA.

Judge Sagun informed the Court on Sept. 9, 2010 that the complaints were addressed. The masses
were shortened to 30 minutes and only held during special holy days. the path to lavatories was
opened to use. There was no instance of employees who fought vying to read the epistle.

The OCA recommended on August 7, 2014 that the inconveniences cited by Valenciano were
unfounded. It recommended that his letter-complaints be dismissed.

ISSUE:
Whether the holding of masses at the basement of the QC hall of justice violates separation of
church and state and the constitutional prohibition against appropriation of public money or
property for the benefit of any sect, church, denomination, sectarian institution, or system of
religion.

HELD:
1. Holding religious rituals in the HoJ is not union of church and state.
Art.II, Sec.6 of the Constitution provides: The separation of Church and State shall be inviolable.

The union of church and state is prejudicial to both, for there may be occasions when the state will
use the church and the church the state as a weapon to further their respective ends and aims. The
rationale is “Strong fences make good neighbords.” It calls on the entities to “render therefore unto
Caesar the things that are Caesar’s and unto God the things that are God’s.” nonetheless, the State
recognizes the inherent right of the people to have some form of belief system. The Filipino people
in (preamble) “imploring the aid of Almighty God” manifested their spirituality innate in our
nature as a people, shaped by tradition and historical experience. As this is embodied in the
preamble, it means that the State recognizes with respect the influence of religion insofar as it
instills into the mind the purest principles of morality. Moreover, the 1935, 1973, and 1987
Constitutions contain benevolent and accommodating provisions towards religions like tax
exemption of church property, salary of religious officers in government institutions, and optional
religious instructions in public schools.

The right to believe or not to believe has been enshrined in Sec.5, Art.III of the 1987 Constitution:
SECTION 5. No law shall be made respecting an establishment of religion, or
prohibiting the free exercise thereof. The free exercise and enjoyment of
religious profession and worship, without discrimination or preference, shall
forever be allowed. No religious test shall be required for the exercise of civil or
political rights.
- Free exercise clause
The right to religious profession and worship has a two-fold aspect- freedom to believe and
freedom to act on one’s beliefs. The first is absolute as long as the belief is confined within the
realm of thought. The second is subject to regulation where the belief is translated into external
acts that affect public welfare.

The individual is free to believe or disbelieve. However absurd his beliefs may be to others, even
if hostile and heretical to the majority, he has full freedom to believe as he pleases. He may not be
required to prove his beliefs nor punished for his inability to do so. But when he externalizes his
beliefs in acts or omissions that affect the public, his freedom to do so becomes subject to the
authority of the State. Religious freedom, like other rights, can only be enjoyed with a proper
regard for the rights of others. Police power can prevent religious practices inimical to society even
if pursued out of sincere religious conviction. Religious freedom terminated disabilities, not
created new privileges. Its essence is freedom from conformity to religious dogma, not to law
because of religious dogma.

Allowing religion to flourish is not contrary to separation of church and state and in fact are in
perfet harmony with each other. While the State is aware of religious movements that believe in
the divinity of Jose Rizal, it does not implement measures to suppress such sects. This inaction or
indifference gives meaning to separation of church and state and recognizes the religious freedom
of these sects to worship their own supreme being.

The Roman Catholics express worship through holy mass and to stop these would be to repress
the right to free exercise of religion. Allowing citizens to practice their religion is not equivalent
to a fusion of church and state.

2. No Compelling state interest- no prejudice to public welfare/service.


Religious freedom is not absolute. It cannot have its way if there is a compelling state interest. It
must be demonstrated that the masses in the QC HoJ unduly disrupt the delivery of public
services or affect the judges and employees in the performance of their official functions.
This test is proper where conduct is involved for human conduct has different effects on the state’s
interests. Only the gravest abuses endangering paramount interests can limit this
fundamental right. A mere balancing of interests is not appropriate. A lesser test would let the
state batter religion, especially less powerful ones, until destroyed.

The masses were conducted only during noon breaks and were not disruptive of public
services. Court proceedings were not distracted or interrupted. No civil service rules were being
violated. Since there is no detrimental effect on public service or prejudice to the State, there
is no compelling state interest.

3. Accommodation, not establishment of religion.


The state adopts the policy of accommodation to give life to freedom of religion. Accommodation
is a recognition of the reality that some governmental measures may not be imposed on a certain
portion of the population for the reason that these measures are contrary to their religious beliefs.
As long as the exercise does not impair public welfare, regulation or prohibition of such right
would be unconstitutional.
Benevolent neutrality allows accommodation of religion under certain circumstances.
Accommodations are government policies that take religion specifically into account not to
promote the government’s favored form of religion, but to allow individuals and groups to exercise
their religion without hindrance. The purpose is to create without state involvement an
atmosphere in which voluntary religious exercise may flourish. Several laws have been enacted
to accommodate religion. (Revised Admin Code- Maundy Thursday, Good Friday, Xmas as
regular holidays; RA 9177- first day of Shawwal, 10th month of Islamic calendar as national
holiday for Eidul Adha; PD 1083, Code of Muslim Personal Laws, allowing muslims to have more
than 1 wife and divorce.)

4. Non-Establishment Clause- merely Accommodation.


“No law shall be made respecting an establishment of religion.” There is a thin line between
accommodation and establishment. This clause reinforces the separation of church and state. It
means that the State cannot set up a church nor pass laws which aid one or all religion, or
prefer one religion over another nor force nor influence a person to go to or remain away
from church against his will, or force him to profess a belief or disbelief in any religion. The
state cannot punish a person for entertaining or professing religious beliefs or disbeliefs, for church
attendance or nonattendance; that no tax in any amount, large or small, can be levied to support
any religious activity or institution whatever they may be called or whatever form they may adopt
or teach or practice religion; that the state cannot openly or secretly participate in the affairs of any
religious organization or group and vice versa. Its minimal sense is that the state cannot establish
or sponsor an official religion.

It also limits what religious sects can or cannot do. They can neither cause the government to adopt
their particular doctrines as policy for everyone nor to restrict other groups. To do so would cause
the State to adhere to a particular religion and establish a state religion.

"In effect, what non-establishment calls for is government neutrality in religious matters. Such
government neutrality may be summarized in four general propositions: (1) Government must not
prefer one religion over another or religion over irreligion because such preference would violate
voluntarism and breed dissension; (2) Government funds must not be applied to religious
purposes because this too would violate voluntarism and breed interfaith dissension; (3)
Government action must not aid religion because this too can violate voluntarism and breed
interfaith dissension; [and] (4) Government action must not result in excessive entanglement
with religion because this too can violate voluntarism and breed interfaith dissension."
-Fr. Bernas.

Establishment entails positive action while accommodation passive. In establishment, the state
becomes involved through the use of state resources to set up a state religion. In accommodation,
without being entangled, the state merely gives consideration to its citizens who want to freely
exercise their religion.

Thus, the masses in the HoJ basement is not establishment but merely accommodation. 1) There
is no law, ordinance or circular mandating that judiciary employees attend the masses. 2) The
employees attend at their own initiative, free will, and volition. 3) No government funds are spent
as the lighting and airconditioning continue to be operational even if no religious rituals are held.
4) The basement has not been converted into a chapel nor permanently appropriated for the
exclusive use of its faithful. 5) The masses has not prejudiced other religions.

5. No appropriation of public money or property for the benefit of any church.


Sec.29(2) Art. VI of the Constitution provides: "No public money or property shall be
appropriated, applied, paid, or employed, directly or indirectly, for the use, benefit, or support
of any sect, church, denomination, sectarian institution, or system of religion, or of any priest,
preacher, minister, or other religious teacher, or dignitary as such, except when such priest,
preacher, minister, or dignitary is assigned to the armed forces, or to any penal institution, or
government orphanage or leprosarium."

“Apply” means to “use or employ for a particular purpose.” “Appropriate” means “to prescribe
a particular use for particular moneys or to designate or destine a fund or property for a distinct
use, or for the payment of a particular demand.”

Under noscitur a sociis, where a word is ambiguous in itself, its correct construction may be made
clear by considering the company of words in which it is found or with which it is associated.
Thus, “pay” and “employ” should mean that what is prohibited is the use of public money or
property for the sole purpose of benefiting or supporting any church. The prohibition
contemplates where the appropriation is primarily intended to further a particular church.

It does not inhibit use of public property for religious purposes when the religious character of the
use is merely incidental to a temporary use which is available indiscriminately to the public
in general. Thus, a public street may be used for a religious procession. “Directly or indirectly”
refers to the manner of appropriation, not as to whether a particular act involves a direct or mere
incidental benefit to any church. Otherwise, the framers would have placed it before “use benefit,
or support.”

Ut magis aleat quam pereat. The Constitution is to be interpreted as a whole. Thus, the
interpretation finds support in the establishment clause that what is prohibited is the passage of
any law which tends to establish a religion, not merely to accommodate free exercise thereof.

The QC HoJ is not appropriated, applied, or employed for the SOLE purpose of supporting
the Roman Catholics. It has not been converted into a chapel for exclusive use of its faithful. It
is also being used as a public waiting area for most of the day and a meeting place. The holding of
masses is limited to lunch break from 12nn-1pm. There is thus no undue religious bias.

Directing the Executive judges of the RTC and MeTC to regulate and monitor the masses and
religious practices within courts does not promote collaboration between courts and religions, but
this is necessary to ensure that there would be no excessive entanglement. To disallow the masses
would make strict separation the norm, making sharia courts and the Muslim Code nugatory.

But the SC ruled that no part of a public building may be a permanent place for religious worship
nor may there be permanent display of religious icons in all halls of justice.
LEONEN, J. Dissenting.
Allowing court personnel to celebrate daily masses within public HoJ violations the prohibition
against establishment of religion. To provide that all faiths and denominations may likewise avail
of the same public space is a painful illusion. The majority invites judges to excessively entangle
themselves with religious institutions and worship. By holding daily catholic masses or religious
ritual within court premises, courts unnecessarily shed their impartiality.

According to the majority, we adopt the policy of accommodation. Despite separation of church
and state, the State may take into account religion in forming government policies not to favor
religion but only to allow its free exercise. The majority sees no violation of the establishment
clause as court personnel are not coerced to attend masses and no government funds are spent. The
use of the basement was not permanent and other religions are not prejudiced.

1. Allowing the exercise of religious rituals within government buildings violate both Sec.5, Art.III
and Sec.29(2), Art.VI of the Constitution. This provision articulates two duties of the state: 1)
Respect free exercise of religious faith 2) Not to establish, endorse, or favor any religion.

The holy eucharist is not simply a ritual among Roman Catholics. It requires a priest and, while
ongoing, prayers and incantations will be heard beyond the vicinity of its participants. The
offensiveness of this ritual cannot be obvious to those who belong to this dominant majority
religion. The inability to see how this practice will not square with those who believe otherwise
will be bcause religion is a matter of faith. The stronger one’s faith, the more tenacious the belief
in the correctness of his fundamental teachings. It will take great strides in both humility and
sensitivity to understand that religious practices within government buildings are offensive to
those who do not believe in any denomination of Christianity. Those who believe in a god but
do not practice any ritual for worship will also find the allowance of the full Catholic sacrament
of the Holy Eucharist demeaning.

The sponsorship of these rituals within the HoJ will not be acceptable to atheists. Courts are not
simply venues for resolution of conflict. Our HoJs should symbolize adherence to impartiality of
the rule of law. Unnecessary sponsorship of religious rituals undermines the primacy of secular
law and its impartiality. There is no urgency that manifestations of a specific belief be done in
HoJs when it can best be done in private churches etc.

2. Not allowable accommodation.


The jurisprudence cited is inappropriate. Jurisprudence providing exceptions to State regulation is
different from doctrinal support for endorsing a specific religion without a separate overarching
compelling lawful and separate state interest.

Escritor formulated a two-part test in resolving cases involving freedom to worship. 1)benevolent
neutrality or accommodation and 2) compelling state interest. It involved a state policy that was
neutral and the question as to whether its consistent application given facts specific to a religion
would violate the adherent’s freedom to worship. This is not the situation here. We are instead
asked to create a policy to sponsor religious rituals. There is no neutral state policy we are
asked to interpret.
Escritor involved accommodation or exceptions to a state policy. But here, we create a policy that
benefits a group of religions that have rituals. It will not benefit believers who do not have public
rituals or a deity. It will not benefit all beliefs like atheists or agnostics. In that case, 5 justices
dissented and 2 wrote separate concurring opinions. That benevolent neutrality is even doctrine is
thus suspect. More importantly, benevolent neutrality in reality may turn out to be a means for
those who believe in a majority decision to maintain their dominance in the guise of neutral
tolerance of all religions. Thus, this doctrine, in practice, favors the already dominant.

3. There is no urgency in holding masses within the HoJ. The Catholic Church owns many places
of worship and there are churches accessible to court personnel in QC HoJ during their lunch
hour a walking distance from their offices. Thus, allowing masses in HoJs has no other purpose
except to allow a religious denomination to express its beliefs. The primary purpose of the
policy favored by the majority is not secular but religious, contrary to constitutional law.

Sec.5, Art.III does not allow endorsement by the State of any religion except if such incidental
endorsement is in the context of a governmental act that satisfies the following three-part test:
1) It has a secular legislative purpose, 2) its primary effect is that which neither advances nor
inhibits religion, 3) it must not foster an excessive entanglement with religion. The majority
cites Fr. Bernas, but he is not the SC and he is a Catholic priest. His opinions on the impact of law
on religion should be taken with a lot of advisement.

Directive the executive judges to regulate and closely monitor the masses and religious practices
within courts promotes excessive entanglements between courts and religions. This will result in
unnecessary interaction between church and state. It will take time from our executive judges
who instead could perform their secular functions of reducing court dockets.

4. Sec.29(2), Art.VI specifically prohibits public property from being employed for the benefit or
support of any sect, church, denomination etc. This provision allows no qualification. Allowing
masses within HoJs employs public property for the “benefit or support of the Catholic.
Religion. Catholicism is a “church”, “denomination”, and a “system of religion.”

The majority interpreted this using noscitur a sociis in saying that it contemplates a scenario where
the appropriation is primarily intended for the furtherance of a particular church. But this implies
that the religious use of public property is allowable as long as other religious groups may
use or employ the property. Sec.29(2) is straightforward and needs no STATCON. Religious use
of public property is proscribed totally. It applies to any religion. This holistic interpretation is
more sensitive to those who disbelieve and to the concept that the state remains neutral in matters
pertaining to faith.

The Constitution guarantees liberty for those who believe in a god, but it does not sanction
insensibilities to those who believe otherwise. It is utter callousness to say that the mass using
powerful sound systems in a space in the center of a HoJ where the rituals resonate that it will
offend no one. Obviously it will not offend Catholics in the majority. It offends those who believe
that the State endeavor to be neutral and impartial.
Benevolent neutrality to render state regulation impotent in a situation where a religion dominates
becomes a painful illusion to those at the margins of our society. The Court rewards the dominant.
It reifies the hegemony of those who have power.

127. Andres Garces v. Hon. Numeriano Estenzo, GRL-53487, May 25, 1981, Laurel, J.
(Freedom of Religion)
FACTS:
The Brgy. Council of Valencia, Ormoc City, adopted Res.5 “reviving the traditional socio-
religious celebration” every April 5 of the feast day of San Vicente Ferrer, patron saint of Valencia.
It provided for acquisition of the image of San Vicente Ferrer and the construction of a waiting
shed as the barangay’s projects. The funds for these would be obtained through the selling of
tickets and cash donations.

The council passed Res.6, specifying that the chairman of hermano mayor of the fiesta would be
the caretaker of the image and it would remain in his residence for one year until the election of
his successor the next feast day. The image would also be available to the Catholic parish church
during celebration of the feast day. Funds were raised thru solicitations and cash donations of the
barangay residents and of neighboring places. Thus, the waiting shed and wooden image was
constructed and acquired.

During the fiesta on April 5, 1976, the image was temporarily placed in the altar of the church of
brgy. Valencia. After the mass, parish priest Fr. Osmeña, refused to return that image on the
pretext that it was property of the church as church funds were used for its acquisition. Thus, the
council adopted Res.10, authorizing hiring of a lawyer to file a replevin case against Fr. Osmeña
to recover the image. Osmeña assailed the constitutionality of the resolutions.

It was also contended by 3 other persons that the brgy. Council was not duly constituted as the
chairman of the kabataang barangay was not allowed to participate in the sessions.

ISSUE:
Whether the purchase of the image of Saint Ferrer authorized by the barangay council for a
traditional fiesta in the barrio through the raising of funds by selling tickets and cash donations
violates non-establishment of religion and the prohibition against using public money for the
benefit of a religious sect.
HELD: NO.
The absence of the barangay youth chairman despite being notified of the sessions did not render
the resolutions void as there was a quorum.

It is contended that the resolutions violate the constitutional provisions that “no law shall be made
respecting an establishment of religion” and that “no public money or property shall ever be
appropriated, applied, paid, or used, xxx for the benefit, or support of any sect, church, xxx. This
is devoid of merit.

The resolutions do not establish any religion nor abridge religious liberty nor appropriate public
money or property for the benefit of any sect, prist, or clergyman. The image was purchased with
PRIVATE funds, not tax money. The construction of a waiting shed is a secular matter.
The argument that the council favored the Catholic religion is preposterous. The image was
purchased in connection with the celebration of the fiesta honoring the saint and not for the
purpose of favoring any religion nor interfering with religious matters or beliefs of the barrio
residents. One of the highlights of the fiesta is the mass, and the image had to be placed in the
church when the mass was celebrated. If there is nothing illegal in holding a fiesta and having a
patron saint for the barrio, then any activity intended to facilitate the worship of the patron saint
(like acquiring his image) is not illegal also.

As noted in the first resolution, the feista is a socio-religious affair. Its celebration is ingrained
tradition in rural communities and relieves the monotony and drudgery of the lives of the masses.
The council designated a layman as custodian of the image to forestall any suspicion that it is
favoring the Catholic church.

The issues of separation of church and state, freedom of religion, and use of public money to favor
any sect is not even involved here. The image belongs to the barangay council. If it changes its
mind and gives the image to the Catholic church, it would not violate the constitution as the image
was acquired with PRIVATE funds and is its PRIVATE property. It had the right to take
measures to recover possession of the image thru Res. 10.

128. Gregorio Aglipay v. Juan Ruiz, GR 45459, March 13, 1937 (Freedom of Religion)
FACTS:
Petitioner Mons. Gregorio Aglipay, head of the PH Independent Church, seeks a writ of prohibition
to prevent respondent director of posts from issuing and selling postage stamps commemorative
of the 33rd international Eucharistic congress.

Director of posts announced in the newspapers in manila that he would issue postage stamps
commemorating the 33rd Eucharistic congress of the Catholic Church. In the center is a chalice,
with grape vine and stalks of wheat as border design. Some stamps were sold. Petitioner seeks to
prevent further sale thereof. He alleges that it violates the prohibition in the Constitution that no
public money shall be appropriated etc. for the use, benefit, or support of any sect, church, etc.

ISSUE:
Whether the issuance of postage stamps to celebrate the 33rd International Eucharistic Celebration
where the stamps contain an image of the PH and the location of Manila to attract more tourists to
PH violates separation of church and state.
HELD: NO.
The union of church and state is prejudicial to both. The Malolos Constitution recognized
separation of church and state. It was inserted in the Treaty of Paris between US and Spain of
Dec.10, 1898, reiterated in Pres.McKinley’s instructions to the PH Commission, reaffirmed in the
PH Bill of 1902 and the Autonomy Act of Aug.29, 1916, and finally embodied in the Constitution.
The Constitution guarantees religious liberty, not mere religious toleration.

Insofar as religion instills into the minds the purest principles of morality, its influence is felt and
appreciated. By imploring the “aid of Divine Providence” in the Constitution preamble, the people
manifested their intense religious nature.
Here, the Director of Posts issued the postage stamps under the provisions of Act 4052,
appropriating P60k for printing postage stamps with new designs. It authorizes the director to
dispose of the amount in the manner indicated and “as often as may be deemed advantageous to
the Government.” The postage stamps here has been approved by the president.

Act 4052 contemplates no religious purpose. It gives the director discretionary power to
determine when the issuance of special stamps would be “advantageous” to the government. While
this phrase (“advantages to..) does not authorize violation of the Constitution nor authorize
appropriation for the benefit of a sect or church, the stamps here were NOT inspired by any
sectarian feeling to favor a particular church. The stamps were not sold to benefit the Catholic
Church nor money derived therefrom given to that church. The only purpose in issuing and selling
the stamps was “to advertise the PH and attract more tourists to this country. The officials
merely took advantage of an event considered of international importance to give publicity to the
PH and its people. The stamps, instead of the original plan of showing a chalice, instead shows a
map of the PH and location of Manila. What is emphasized is not the Eucharistic congress itself
but Manila as the seat of that congress. The government should not be embarrassed in its activities
simply because of INCIDENTAL results more or less religious in character, if the purpose in
view is one which could legitimately be undertaken by appropriate legislation. The MAIN
purpose should not be frustrated by its subordination to mere incidental results not
contemplated.

129. INC v. CA, GR 119673, July 26, 1996,Puno, J. (Freedom of Religion)


FACTS:
Iglesia ni Cristo has a TV program entitled “Ang Iglesia ni Cristo” aired on Channel 2 every
Satuday and Channel 13 on Sunday. It propagates and presents INC’s religious beliefs, doctrines,
and practices often times in comparative studies with other religions. INC submitted to the
respondent Board of Review for Motion Pictures and TV the VTR tapes of its TV program. The
Board classified the series as “X” of not for public viewing on the ground that they “offend and
constitute an attack against other religions which is expressly prohibited by law.” It denied a permit
to show the tapes.

It appealed to the OP. the OP reversed the Board’s decision. INC also filed against the Board a
case with the RTC, claiming that the Board acted without jurisdiction or with grave abuse of
discretion in requiring INC to submit its tapes and in x-rating them.

The RTC ordered the Board (BRMPT) (Now MTRCB) to grant INC the permit for its program.
But INC was directed to refrain from offending other existing religion. INC prayed in its MR for
the deletion of this second part of the decision. It also prohibited BRMPT from requiring INC to
submit its tapes to it. But the CA reversed the RTC, ruling that BRMPT had jurisdiction to review
the program and BRMPT did not act with grave abuse of discretion.

ISSUE:
Whether the BRMPT gravely abused its discretion when it gave the TV program of INC an X-
rating or not for public viewing, thereby not granting it permit to show the program, on the ground
that it attacked other religions.
HELD: YES.
PD 1986 gives BRMPT the power to review and examine all television programs and to approve
or prohibit its broadcast. It is contended that “TV program” should not include religious programs
like INC’s, else, Sec.5 Art.III would be violated.

A person has full freedom to believe as he pleases. But where he externalizes his beliefs in acts or
omissions that affect the public, his freedom to do so becomes subject to state authority. The
essence of religious liberty is freedom from conformity to religious dogma, not freedom from
conformity to law because of religious dogma. Thus, we reject the contention that INC’s religious
program is beyond review by BRMPT. The exercise of religious freedom can be regulated when
it will bring about the clear and present danger of some substantive evil which the state is duty
bound to prevent.

The CA and BRMPT held that the attacks against other religions are indecent, contrary to law and
good customs. But we reverse these rulings.

Any act that restrains speech (Prior restraint) is presumed invalid. It is the burden of the BRMPT
to overthrow this presumption. It failed here. The so-called “attacks” are mere criticisms of deeply
held dogmas of other religions. The CA did not view the tapes as they were not presented in
evidence, yet it ruled that they were indecent. This suppresses INC’s freedom of speech and free
exercise of religion.

BRMPT may disagree with the criticisms, but that gives it no excuse to interdict such. It is not the
task of the state to favor any religion by protecting it against attack by another religion.
Religious dogmas are often at war and to preserve peace among their followers, the establishment
clause prohibits the state from leaning towards any religion. The state must be neutral. BRMPT
cannot squelch the speech of INC simply because it attacks other religions even if said religion
happens to be the most numerous church in the country. The bedrock of freedom of religion is
freedom of thought and it is best served by encouraging the marketplace of dueling ideas.

Also, “attacks against another religion” is not among the grounds in Sec.3 of PD 1986 justifying
an order prohibiting the broadcast of INC’s program. This ground was merely added by BRMPT
in its rules, which is void as it expands the law.

BRMPT invokes Art201(2)(b)(3) of the RPC to justify the “attack against any religion” ground as
the article punishes shows which offend any race or religion. But “attack” is not synonymous with
“offend”. Art.201 should also be invoked to justify subsequent punishment of a show offending
any religion. It cannot be used to justify prior censorship of speech.

In x-rating the program, BRMPT failed to apply the clear and present danger rule. It is only
where it is unavoidably necessary to prevent an immediate and grave danger to the security and
welfare of the community that infringement of religious freedom may be justified, and only to the
smallest extent necessary to avoid the danger. The findings of BRMPT is bereft of findings of
fact to justify the conclusion that the program is an impermissible attack against another religion.
There is no showing of the type of harm and gravity and imminence thereof that the tapes will
bring about. Prior restraint on speech, including religious speech, cannot be justified by
hypothetical fears but only by showing of a substantive and imminent evil which has taken the
life of a reality already on ground.

Originally, this rule was designed to apply to anti-government action. Its umbrella was used to
protect speech other than subversive speech in the 1940s. In the 1950s, the test was diminished
when the US SC used the formulation of “in each case, courts must ask whether the gravity of the
evil, discounted by its improbability, justifies such invasion of free speech as is necessary to avoid
the danger.” But in 1969, the strength of the test was reinstated. The test is presently applied in the
US to 4 types of speech 1) advocates dangerous ideas, 2) provokes hostile audience reaction, 3)
out of court contempt, and 4) release of information that endangers a fair trial. Thus, even
following the drift of American jurisprudence, there is reason to apply the clear and present danger
test to this case concerning speech that attacks other religions and could readily provoke hostile
audience reaction.

While the thesis cited by separate opinions that in US jurisprudence, only a judicial determination
of the question as to whether the speech falls within or outside protected speech is allowed and
may not be arrogated to administrative bodies has a lot to commend itself, we are not ready to hold
that it is invalid for Congress to grant an administrative body quasi-judicial power to preview and
classify TV programs and enforce its decision subject to review by our courts. The issue involves
policy considerations better addressed by our legislators.

Thus, the MTRCB/BRMPT has jurisdiction, but it cannot x-rate INC’s TV program on the grounds
it cited without clear and present danger.

130. American Bible Society v. City of Manila, GR L-9637, April 30, 1957, Felix, J. (Freedom
of Religion)
FACTS:
ABS is a foreign religious missionary corporation doing business in PH thru its PH agency. ABS
has been selling and distributing bibles or gospel portions thereof throughout the PH and
translating it into several PH dialects. On May 29, 1953, the treasurer of Manila informed ABS
that it needed a Mayor’s permit and to pay license fees. To prevent closure, it paid under protest
the fees, giving notice to the city treasurer that suit would filed in court to question the legality of
the ordinances under which the fees were being collected. It filed said complaint, praying that
Municipal Ordinance 3000, 2529, 3028, and 3364 be declared unconstitutional and that its
payment be refunded.

ABS claims that it never made profit from its sale of bibles which it disposes for a third of the cost.
To main its operating costs, it obtains remittances from its NY office and voluntary contributions
from churches in the US and PH. The trial court dismissed the case. The CA certified the case to
SC as it involved only questions of law.

ISSUE:
Whether ordinances requiring license fees and Mayor’s permit for a person to engage in selling
bibles and other religious pamphlets violates the right to free exercise of religious profession.
HELD: NO.
Sec.1, (7), Art.III of the 1935 Constitution provides: "(7) No law shall be made respecting an
establishment of religion, or prohibiting the free exercise thereof, and the free exercise and
enjoyment of religious profession and worship, without discrimination or preference, shall forever
be allowed. No religion test shall be required for the exercise of civil or political rights."

ABS claims that the ordinances are unconstitutional as it restrains and censors free exercise of its
religious profession: sale of religious literature.

"SEC. 1. FEES. — Subject to the provisions of section 578 of the Revised Ordinances of
the City of Manila, as amended, there shall be paid to the City Treasurer for engaging in
any of the businesses or occupations below enumerated, quarterly, license fees based on
gross sales or receipts realized during the preceding quarter in accordance with the rates
herein prescribed: PROVIDED, HOWEVER, That a person engaged in any business or
occupation for the first time shall pay the initial license fee based on the probable gross
sales or receipts for the first quarter beginning from the date of the opening of the business
as indicated herein for the corresponding business or occupation. xxx xxx xxx GROUP 2.
— Retail dealers in new (not yet used) merchandise, which dealers are not yet subject to
the payment of any municipal tax, such as (1) retail dealers in general merchandise; (2)
retail dealers exclusively engaged in the sale of . . . books, including stationery.

Thus, the license fees in Sec.1 of Ord.2529 are not imposed directly upon any religious
institution but upon those engaged in any of the businesses or occupations therein
enumerated, like retail dealers in general merchandise which allegedly covers selling of bibles,
books etc.

Any restraint on right to freedom of religious profession and worship can only be justified like
other restraints of freedom of expression- clear and present danger of any substantive evil which
the state has the right to prevent. Here, the license fee involved is imposed for ABS’s sale of bibles
and religious literature. It is one thing to impose a tax on the income or property of a preacher and
another to tax him for the privilege of delivering a sermon. The power to tax the exercise of a
privilege is the power to control or suppress its enjoyment.

While the price asked for the bibles and religious pamphlets was in some instances a little bit
higher than its actual cost, this cannot mean that ABS was engaged in selling said “merchandise”
for profit. Thus, Ord.2529 does not apply, for in doing so would impair ABS’s free exercise of
religious profession and worship and rights of dissemination of religious beliefs.

As to Ord.3000 requiring a Mayor’s permit, we do not find that it imposes any charge upon the
enjoyment of a constitutional right nor tax religious practices. It does not deprive ABS of its right
even though it prohibits it from introducing and carrying out a scheme or purpose which he
sees fit to claim as part of his religious system.

Thus, Ord.3000 is valid even as applied to ABS. But Ord.2529 is not applicable to ABS and Manila
is powerless to license ABS’s business for it would impair its right. But Ord.3000, as amended, is
also inapplicable to the business of ABS. Thus, we rule that the fees collected be returned. (*Altho
inapplicable, even if appied to ABS, valid.)
131. Arturo Tolentino v. Secretary of Finance, GR 115455, October 30, 1995, Mendoza, J.
(Freedom of Religion)
FACTS:
MR seeking reconsideration of the SC decision dismissing the petitions to declare RA 7716,
Expanded VAT law, as unconstitutional.

ISSUE:
Whether the imposition of VAT on the press and on the sale of Bibles is unconstitutional.
HELD: NO.
Generally, the press is not exempt from the taxing power of the State and what is prohibited are
laws which single out the press or target a group belonging to the press for special treatment
or which discriminate against the press. RA 7716 is none of these.

It is contended that by removing the exemption of the press from VAT while maintaining those
granted to others, the law discriminates against the press. At any rate, even nondiscriminatory
taxation of constitutionally guaranteed freedom is unconstitutional. As to the first, since the law
granted the press a privilege, it could take back the privilege anytime. By granting exemptions,
the State does not forever waive the exercise of its sovereign prerogative. In withdrawing the
exemption, the law merely subjects the press to the same tax burden that other businesses have
long ago been subject.

As to the contention that even nondiscriminatory tax on constitutionally guaranteed freedom is


unconstitutional, the case of Murdock v. Pennsylvania is cited. The Court in that case spoke of
license tax, which unlike an ordinary tax, is mainly for regulation. Its imposition on the press is
unconstitutional as it lays prior restraint on the exercise of its right. Its application to others
selling goods is valid, but to the press or religious groups in connection with the latter’s sale of
religious books is unconstitutional. It is one thing to impose tax on income of a preacher and
another to tax him for delivering a sermon.

In American Bible Society v. Manila, it was held that a city ordinance requiring license fee on those
engaged in sale of general merchandise cannot be imposed on sale of bibles without restraining
the free exercise of its right to propagate.

But VAT is different. It is not a license tax. It is not a tax on the exercise of a privilege much less
a constitutional right. It is imposed on the sale, barter, etc. of properties for purely revenue
purposes. To subject the press to its payment is not to burden its exercise of right any more than
to make the press pay income tax or general regulation is not to violate its freedom in the
Constitution.

PH Bible Society claims that although it sells bibles, the proceeds are used to subsidize the cost of
printing copies given free to those who cannot afford to pay so that to ta the sales would be to
increase the price while reducing volume of sale. Even if this be the case, the resulting burden on
religious freedom is so INCIDENTAL as to make it difficult to differentiate it from any other
economic imposition that might make the right to disseminate religious doctrines costly.
As to the registration fee of P1000, this is just to pay for the expenses of registration and
enforcement of provisions like those relating to accounting. That PBS distributes free bibles and
is thus not liable to pay VAT does not excuse it from payment of this fee because it also sells some
copies.

132. Roel Ebralinag v. The Division of Superintendent of Schools of Cebu, GR 95770, March
01, 1993 (Freeodm of Religion)
FACTS:

In 1989, DECS Regional Office in Cebu received complaints about teachers and pupils belonging
to the JW and enrolled in various public and private schools who refused to sing the national
anthem, salute the PH flag, and recite the patriotic pledge. Division Superintendent of Schools
(DSS) Susana Cabahug of Cebu Division of DECS, recalling the Gerona decision, issued Division
Memorandum 108 directing District Supervisors, principals, and heads of private schools to
consider removed from service school employees who choose not to participate in the daily flag
ceremony after due process. As to the students, citing Gerona, if they do not obey the flag salute,
they merely lose the benefit of public education being maintained at the expense of their fellow
citizens. (They could take it or leave it!) Having elected not to comply with the regulation on flag
salute, they forfeited their right to attend public schools.

In GR 95770, Ebralinag v. Division, petitioners are 43 high school and elementary school students
in the towns of Daan Bantayan, Cebu province. All minors, they are assisted by their parents who
belong to Jehovah’s Witnesses.

In GR 95887, May Amolo et al. v. Division, petitioners are 25 hs and grade school students
enrolled in public schools in Asturias, Cebu, whose parents are Jehovah’s Witnesses.

All petitioners were expelled from their classes by the public school authorities in Cebu for
refusing to salute the flag, sing the national anthem, and recite the patriotic pledge as required by
RA 1265 and Department Order (DO) 8 of the DECS, making the flag ceremony compulsory in
all educational institutions. DO 8 is RA 1265’s IRR. Thus, they filed these special civil actions for
mandamus, certiorari, and prohibition, alleging grave abuse of discretion ordering their expulsion
without due process and violating their right to free public education, free speech, religion, and
worship.

JWs teach their children not to salute the flag, sing the national anthem, recite the patriotic pledge
(*panatang makabayan) for they believe that those are “acts of worship” or “religious devotion”
which they cannot conscientiously give to anyone or anything except God. They feel bound by the
bible’s command to “guard ourselves from idols.” They consider the flag as an idol representing
the state. They claim that the action of the local authorities in compelling the flag salute and pledge
violates constitutional limits on the state’s powers and invades the sphere of intellect and spirit
which the Constitution protects against official control.

ISSUE:
Whether the state may compel members of Jehovah’s Witnesses to salute the national Philippine
flag and recite the patriotic pledge against their religious belief that doing so is akin to an “act of
worship” to idols which acts should, in their religious belief, only be given towards God.

HELD: NO.
In Gerona, et al. v. Secretary of Education, et al.(1959), this Court upheld the expulsion of the
students. It held there that the flag is not an image but a symbol of the Republic of the PH. Under
separation of church and state, the flag is utterly devoid of religious significance. Saluting it does
not involve religious ceremony. The requirement to salute is not imposing a religious belief, but
merely enforcement of a non-discriminatory school regulation applicable to all alike, whether
Christian, Muslim, Protestant, or JW. To exempt JW children would disrupt school discipline and
demoralize the rest of the school population which constitutes the great majority. Freedom of
religious belief does not mean exemption from reasonable and non-discriminatory laws. This was
reiterated in Balbuna et al. v. Secretary of Education.

These rulings have been incorporated in Sec.28, Title VI, Chapter 9 of the Administrative Code of
1987, providing that any teacher or pupil who refuses to participate in the flag ceremony may be
dismissed after due investigation. Petitioners here challenge not the Admin Code, only RA 1255
and the DECS IRR, DO8.

After the Gerona ruling had received legislative cachet by its incorporation in the Admin Code of
1987, this Court believes that it is time to reexamine it. The idea that one may be compelled to
salute the flag, sing the anthem etc. on pain of being dismissed or expelled is alien to the conscience
of the present generation of Filipinos who cut their teeth on the Bill of Rights which guarantees
their rights to free speech (flag salute, singing the national anthem, reciting the patriotic pledge are
all forms of utterances) and free exercise of religious profession and worship.

The right to religious profession and worship has a two-fold aspect: 1) freedom to believe, which
is absolute, and 2) freedom to act on one’s belief, which is subject to regulation. Petitioners stress
that while they do not take part in the compulsory flag ceremony, they do not engage in “external
acts” or behavior that may offend their countrymen who believe expressing love of country
through the flag ceremony. They quietly stand at attention during the ceremony to show their
respect.

The only justification for prior restraint on religious freedom is the existence of a grave and
present danger of a character both grave and imminent, of a serious evil to public safety,
morals, health, and other legitimate public interest that the state has a right and duty to prevent.
Without such threat, the expulsion of petitioners is not justified.

The situation feared in Gerona that “the flag ceremony will become a thing of the past if conducted
with very few participants, and the time will come when we would have citizens uninculcated in
and not imbued with reverence for the flag and love of country and patriotism xxx” has not come
to pass. We are not persuaded that by exempting the JW from saluting the flag, etc., which
admittedly comprises a small portion of the school population, will suddenly produce a nation
uninculcated xxx. After all, what petitioners seek is only exemption from the flag ceremony, not
exclusion from public schools where they may study the Constitution, democratic form of
government, PH history and culture, and training for a profession and be taught patriotism
etc. as part of the curricula. Forcing a small religious group to participate in a ceremony that
violates their religious belief will not be conducive to love of country or respect for duly constituted
authorities.

The expulsion of JW members from schools where they are enrolled will also violate their right as
PH citizens under the 1987 constitution to receive free education for it is the duty of the state to
“protect and promote the right of all citizens to quality education and to make such education
accessible to all.” (Sec.1, Art.XIV)

In Victoriano v. Elizalde Rope Worker’s Union, we upheld exemption of INC members from the
coverage of a closed shop agreement between employer and union as it would violate the teaching
of their church not to join any labor group. Similar exemptions may be accorded to JWs as to the
observance of the ceremony out of respect for their religious beliefs.

Nevertheless, their right not to participate in the ceremony does not give them a right to disrupt
such patriotic exercises. While the highest regard is afforded to free exercise of religion, school
authorities are not powerless to discipline students if they commit breaches of peace by actions
that offend the sensibilities of other persons. If they quietly stand at attention during the
ceremony, such do not disturb the peace or pose a grave and present danger.

133. Imbong v. Ochoa, GR 204819, April 08, 2014 (Freedom of Religion)


FACTS:
RH Bill is assailed as unconstitutional.

ISSUE:
1. Whether state-sponsored procurement of contraceptives violates religious freedom.
2. Whether the imposition of the duty to refer a patient seeking reproductive health services to
another practitioner able to provide for the patient’s needs violates a conscientious objector’s
religious freedom.
3. Whether the requirement of attending family planning and responsible parenthood seminars as
a prerequisite to acquiring marriage license violates religious freedom.
HELD:
CLAIMS:
1. Petitioners assail the state-sponsored procurement of contraceptives, arguing that the use of
their tax on contraceptives violates religious freedom as contraceptives violate their religious
beliefs.
2. The law imposes upon conscientious objectors the duty to refer the patient seeking reproductive
health services to another medical practitioner who would be able to provide for the patient’s
needs. This would make the objector cooperate with the very thing he refuses to do.
3. The law requires would-be couples to attend family planning and responsible parenthood
seminars to obtain a certificate of compliance, forcing individuals to participate in the
implementation of RH law even if it contravenes their religious beliefs.
RULING:
Sec.6, Art.II; Sec.5, Art.III; Sec.29(2), Art.VI.
The establishment clause "principally prohibits the State from sponsoring any religion or favoring
any religion as against other religions. It mandates a strict neutrality in affairs among religious
groups." Essentially, it prohibits the establishment of a state religion and the use of public resources
for the support or prohibition of a religion. On the other hand, the basis of the free exercise clause
is the respect for the inviolability of the human conscience. Under this part of religious freedom
guarantee, the State is prohibited from unduly interfering with the outside manifestations of one's
belief and faith.

The freedom of exercise of religion has two parts: freedom to believe, which is absolute; and
freedom to act on one’s belief, which is subject to state regulation.

In case of conflict between the free exercise clause and the state, the Court adheres to the doctrine
of benevolent neutrality. The benevolent neutrality theory believes that with respect to these
governmental actions, accommodation of religion may be allowed, not to promote the
government's favored form of religion, but to allow individuals and groups to exercise their
religion without hindrance

In ascertaining the limits of exercise of religious freedom, the compelling state interest test is
proper. Underlying this test is the notion that free exercise is a fundamental right and that laws
burdening it should be subject to strict scrutiny.

Here, it is not within the province of the Court to determine if the use of contraceptives or one’s
participation in the support of modern reproductive health measures is moral FROM a religious
standpoint. Matters dealing with faith, practice, ecclesiastical law, custom, and rule of a church
are unquestionably ecclesiastical matters outside the province of the civil courts. The jurisdiction
of the Court extends only to public and secular morality. While the Court has no authority to
rule on ecclesiastical matters, it has authority to determine whether the RH Law violates religious
freedom.

- Establishment clause and contraceptives.


The establishment clause restricts what the government can do with religion and what religious
sects can or cannot do with the government. They can neither cause the government to adopt
their particular doctrines as policy for everyone, nor can they not cause the government to
restrict other groups. To do so would make the state adhere to a particular religion and establish a
state religion.

(answer to 1.)
Consequently, the petitioners are misguided in their supposition that the State cannot enhance its
population control program through the RH Law simply because the promotion of contraceptive
use is contrary to their religious beliefs. Indeed, the State is not precluded to pursue its legitimate
secular objectives without being dictated upon by the policies of any one religion. One cannot
refuse to pay his taxes simply because it will cloud his conscience. The demarcation line between
Church and State demands that one render unto Caesar the things that are Caesar's and unto God
the things that are God's.

- Free exercise and duty to refer (answer to 2.)


Where free exercise of religion is allegedly burdened by government legislation or practice, the
compelling state interest test in line with the doctrine of benevolent neutrality finds application.
Here, the conscientious objector’s claim to religious freedom warrants an exemption from
obligations under the RH Law unless the government shows a compelling state interest in
accomplishing an important secular objective. The plea of conscientious objectors deserves strict
scrutiny.

The first inquiry is whether a conscientious objector’s right to religious freedom has been
burdened. There is no doubt that an intense tug-of-war plagues an objector- obedience to the law
and abandonment of religious beliefs or a clean conscience yet under pain of penalty. The
obligation to refer imposed by the RH Law violates religious belief. Once the medical practitioner
against his will refers a patient seeking health products, services, procedures, and methods, his
conscience is immediately burdened as he has been compelled to perform an act against his
beliefs. While the referral is said to be an opt-out clause, it is a false compromise as it makes pro-
life health providers complicit in performing an act they find morally repugnant. They cannot, in
conscience, do indirectly what they cannot do directly. One may not be the principal, but he is
equally guilty if he abets the offensive act by indirect participation.

Also, religious freedom is intertwined with the right to free speech, it being an externalization
of one’s thought and conscience. This includes the right to be silent. While the RH Law seeks
to provide freedom of choice through informed consent, freedom of choice guarantees the liberty
of the religious conscience and prohibits any degree of compulsion or burden, whether direct or
indirect, in the practice of one's religion.

In the conflict between the religious beliefs and moral convictions of individuals and the interest
of the state to provide access and information on reproductive health products, services,
procedures, and methods to enable the people to determine the timing, number, and spacing of the
birth of their children, the religious freedom of health providers, public or private, should be
accorded primacy. Thus, a conscientious objector should be exempt from compliance with the RH
Law.

The same is true with respect to non-maternity specialty hospitals and hospitals owned and
operated by a religious group and health care service providers. Since Sec.24 penalizes them if
they fail or refuse to comply with their duty to refer in Sec.7 and 23(a)(3), it must be struck down
for violating freedom of religion. The same applies to Sec.23(a)(1) and (2) in relation to Sec.24
considering that in the dissemination of information regarding programs and services in the
performance of reproductive health procedures, religious freedom of health care service providers
should be respected.

The RH Law IRR provides that skilled health professionals like provincial, city, or municipal
health officers, nurses and midwives cannot be considered conscientious objectors. This violates
equal protection. The conscientious objection clause should be equally protective of the religious
belief of public health officers. There is no distinction why they should not be exempt from the
mandates of the law. The protection of conscientious objectors should apply to all medical
practitioners without distinction whether they belong to the private or public sector. This is void
as there is no such provision in the RH Law and it violates equal protection.
- Compelling state interest?
The OSG was silent in the establishment of a more compelling state interest to rationalize the
curbing of a conscientious objector’s right not to adhere to an action contrary to his religious
convictions. Thus, there is no such interest. If the government fails to show the seriousness and
immediacy of the threat, state intrusion is constitutionally unacceptable. There is no immediate
danger to the life or health of an individual in the perceived scenario of the provisions. A couple
who plans the timing, number, and spacing of the birth of their children refers to a future event
contingent on whether the mother decides to adopt or use the information or method given to her
or if she even decides to be pregnant at all. On the other hand, the burden placed on those who
object to contraceptive use is immediate and occurs the moment a patient seeks consultation on
reproductive health matters.

Even if there was a compelling state interest, the state failed to show the “gravest abuses,
endangering paramount interests” which could limit or override a person’s fundamental right to
religious freedom. There is also no effort to show that the means is the least intrusive means. The
health concerns of women may still be addressed by other practitioners who may perform
reproductive health-related procedures with open willingness and motivation. At any rate,
there are other secular steps already taken by the legislature to ensure that the right to health is
protected like RA 4729, Contraceptive Act, RA 6365, The Population Act of the PH, and RA 9710,
The Magna Carta of Women.

From 1990-2008, the Filipino maternal mortality rate dropped to 48 percent even if there was still
no RH Law at the time.

An exception is in life threatening cases. While generally healthcare service providers cannot be
forced to render reproductive health care procedures if doing it would contravene their religious
beliefs, an exception must be made in life-threatening cases requiring emergency procedures. In
these situations, the right to life of the mother should be given preference, considering that
referral by a medical practitioner would be denial of service, placing the life of a mother in grave
danger. Principle of Double-Effect: in a conflict situation between the life of the mother and
child, the doctor is morally obliged to try to save both lives. If impossible, the resulting death to
one should not be deliberate. Intentional harm to either the mother or child is never justified to
bring about a “good” effect. He can act in favor of one (not necessarily the mother) when it is
impossible to save both, provided there is no direct harm intended on the other.

3. Family Planning Seminars.


As to the requirement of seminars in Sec.15 for the issuance of a marriage license, this is a
reasonable exercise of police power. Religious freedom is not violated. All the law requires is for
would-be spouses to attend a seminar on parenthood, family planning, breastfeeding, and infant
nutrition. It does not mandate the type of family planning methods to be included in the seminar.
Those who receive any information in the required seminars are not compelled to accept the
information and are completely free to reject the information they find unacceptable, and
retain the freedom to decide on matters of family life without intervention of the State.
134. Alejandro Estrada v. Soledad Escritor, AM P-02-1651, June 22, 2006, Puno, J. (Freedom
of Religion)
FACTS:
Complainant Estrada requested Judge Caoibes of the RTC Las Piñas City for an investigation of
Escritor, court interpreter in said court, for living with a man not her husband, and having borne
a child within this live-in arrangement. Estrada believes that Escritor is committing an immoral
act that tarnishes the image of the court, thus she should be dismissed as it might appear that the
court condones her act. Escritor was then charged for disgraceful and immoral conduct under the
Revised Admin Code.

Escritor testified that when she entered the judiciary in 1999, she was already a widow as her
husband died in 1998. She started living with Luciano Quilapio without marriange more than 20
years ago when her husband was still alive but living with another woman. She and Quilapio have
a son. As member of Jehovah’s Witnesses and the Watch Tower and Bible Tract Society, Escritor
asserted that their conjugal arrangement is in conformity with their religious beliefs and has the
approval of her congregation. 10 years after living together, she executed a “Declaration of
Pledging Faithfulness.”

For JWs, this declaration lets congregation members abandoned by their spouses to enter marital
relations. It thus makes the union moral and binding within the congregation all over the world
except in countries where divorce is allowed. It is required that at the time the declarations are
executed, the couple cannot secure the civil authorities’ approval of the marital relationship
because of legal impediments. Only couples baptized and in good standing may execute the
declaration, which requires approval of the elders of the congregation. The marital status of
declarants are investigated before the declarations are executed. Once all legal impediments are
lifted, the validity of the declarations ceases and the couples should legalize their union. In
Escritor’s case, although she was widowed in 1998 thus lifting the legal impediment to marry on
her part, her mate Luciano was still not capacitated to remarry. Thus, their declarations remain
valid. Insofar as JW is concerned, there is nothing immoral about their arrangement.

Escritor invokes her religious belief in asserting that her arrangement is not disgraceful and
immoral conduct.

ISSUE:
Whether a court interpreter can be held administratively liable for disgraceful and immoral conduct
for living with a man not her husband where she invokes her religious beliefs in which she executed
a Declaration of Pledging Faithfulness in accordance with the Jehovah’s Witnesses religion which
does not view her arrangement as immoral.
HELD: NO.
In a previous decision before this case was remanded to the OSG to let it present evidence, we held
that in resolving claims of religious freedom, 1) benevolent neutrality or accommodation is the
spirit underlying the religion clauses in our Constitution and 2) it is the compelling state interest
test, the strictest test, which must be applied in deciding Escritor’s plea of exemption.

US history has two different, even opposing, strains of jurisprudence on the religion clauses. 1)
Separation, which may be a) strict separation or b) the tamer version of strict neutrality or
separation or what Justice Carpio refers to as the theory of governmental neutrality. 2)
Benevolent neutrality or accommodation is based on the view that the wall of separation is meant
to protect the church from the state.

1. Strict Separation and Strict Neutrality/Separation


The strict separationist believes that the establishment clause was meant to protect the state from
the church, and the state’s hostility towards religion allows no interaction between the two.
Only the complete separation of religion from politics would eliminate the formal influence of
religious institutions and provide a free choice among political views, thus a strict “wall of
separation” is necessary.

Strict neutrality or separationist view (or governmental neutrality theory) believes that the “wall
of separation” does not require the state to be their adversary. The state must be neutral in its
relations with groups of religious believers and non-believers. State power is not used to handicap
or favor religion. It is not hostile to religion but strict in holding that religion may not be used as
basis for classification for purposes of governmental action. Only secular criteria may be the
basis of government action.

The problem with this approach is if applied in interpreting the establishment clause as it could
lead to a de facto voiding of religious expression in the free exercise clause. It could lead to a
pervasive devotion to the secular and a passive or even active hostility to the religious, which is
prohibited.

The dilemma of both separationist approaches is that, in real life, church and state cannot be
totally separate.

2. Benevolent Neutrality/Accommodation
This theory is premised on a different view of the “wall of separation.” Unlike the Jeffersonian
wall meant to protect state from church, this is meant to protect church from state. Benevolent
neutrality recognizes that religion plays an important role in the public life of the United States as
shown by many traditional government practices which, to strict neutrality, pose Establishment
Clause questions.

3. A legislative act that purportedly aids or inhibits religion will be challenged as unconstitutional
either as violative of the free exercise clause or establish clause or both. This is true whether one
subscribes to the separationist or benevolent neutrality approach. The more difficult religion cases
involve legislative acts which have a secular purpose and general applicability, but may
incidentally aid or burden religious exercise.

The benevolent neutrality theory believes that with respect to these government acts,
accommodation of religion may be allowed not to promote the government’s favored form of
religion but to allow individuals and groups to exercise their religion without hindrance. The
purpose is to remove a burden on or facilitate the exercise of a person’s religion. The
"government [may] take religion into account . . . to exempt, when possible, from generally
applicable governmental regulation individuals whose religious beliefs and practices would
otherwise thereby be infringed, or to create without state involvement an atmosphere in which
voluntary religious exercise may flourish." Thus, what is sought under this theory of
accommodation is not a declaration of unconstitutionality of a facially neutral law, but an
EXEMPTION from its application or “burdensome effect.”

4. US Jurisprudence on free exercise.


In Sherbert v. Verner, it was said that when a law of general application infringes religious
exercise, albeit incidentally, the state interest sought to be promoted must be so paramount and
compelling as to override the free exercise claim. It is not sufficient to show mere rational
relationship of the substantial infringement to religious right and a colorable state interest. Even
if there were serious detrimental effects, the state must also show that no alternative means of
regulations would address the detrimental effects without infringing religious liberty. This case
established the exemption doctrine: when general laws conflict with scruples of conscience,
exemptions ought to be granted unless some “compelling state interest” intervenes.

Thus, it was held that when government action burdens, even inadvertently, a sincerely held
religious belief or practice, the state must justify the burden by showing that the law embodies a
compelling state interest, that no less restrictive alternative exists, and that a religious
exemption would impair the state’s ability to effectuate its compelling interest.

In Wisconsin v. Yoder, the US Court exempted convicted Amish parents from a law of general
neutral application with a criminal penalty requiring compulsory school-attendance. This case and
Sherbert laid down the following: 1) free exercise clause claims were subject to heightened
scrutiny or compelling interest test if government substantially burdened the exercise of religion;
2) the test governed cases where the burden was direct- the religious exercise triggered a criminal
or civil penalty, and to cases where the burden was indirect- the exercise resulted to forfeiture of
government benefit. 3) The court could carve out accommodations or exemptions from a
facially neutral law of general application, whether general or criminal.

1) Action was protected, conduct beyond speech, press, or worship was included.2) Indirect
impositions on religious conduct were prohibited. 3) The protection granted was extensive. 4) The
government was required to provide proof of the important interest at stake and the dangers to that
interest presented by the religious conduct. 5) In determining the injury to government interest, a
court was required to focus on the effect that exempting religious claimants from the regulation
would have rather than on the value of the regulation in general. The fourth and fifth require that
facts, rather than speculation, had to be presented.

US Court members usually disagreed over which governmental interests should be considered
compelling, thus producing dissenting and separate opinions in religious conduct cases. This
general test established a strong presumption in favor of free exercise of religion.

In Employment Division, Oregon Department of Human Resources v. Smith, Native Americans


challenged an Oregon law prohibiting the religious use of peyote, a hallucinogenic substance,
which resulted in their dismissal from employment for misconduct and disqualification from
unemployment benefits. The claim for free religion exemption was rejected. It was held that an
individual’s religious beliefs does not excuse compliance with an otherwise valid law prohibiting
conduct that the state is free to regulate. The right to free exercise does not excuse compliance
with a valid and neutral law of general applicability on the ground that the law proscribes (or
prescribes) conduct that his religion prescribes (or proscribes). The decision examined previous
ones that upheld free exercise challenges and said the these cases involved another constitutional
right like free speech and had a “hybrid situation,” unlike Smith.

Smith changed the test for the free exercise clause to only rational basis test where the law
burdening religion is neutral or of general applicability no matter how much they burden
religion. There were dissenting justices and the decision was criticized intensely and widely. It is
criticized as allowing minority faiths to suffer at the hands of the majority faith. The Religious
Freedom Restoration Act (RFRA) of 1993 was adopted to negate the Smith test and to restore the
compelling interest test in Sherbert and Yoder. But this was declared unconstitutional, which
decision was also widely criticized.

Thus, Smith is dangerous precedent as it subordinates fundamental rights of religious belief and
practice to all neutral, general legislation. It effectively left religious freedom in the hands of the
political process, exactly where it would be if the religion clauses did not exist.

5. Accommodation under the religion clauses.


A free exercise claim could result to 3 kinds of accommodation: 1) those found constitutionally
compelled (required by the free exercise clause), 2) those discretionary or legislative (not
required by the free exercise clause but permitted by the establishment clause, and 3) those which
the religion clauses prohibit.

Mandatory accommodation results when the Court finds that it is required by the Free exercise
clause- when the Court itself carves out an exemption. This occurs when all 3 conditions of the
compelling interest test are met: 1) a statute or government action has burdened free exercise of
religion and there is no doubt as to the sincerity of the religious belief, 2) the state has failed to
demonstrate an important or compelling governmental goal in preventing an exemption, and
3) the state failed to demonstrate that it used the least restrictive means. Thus, if the objective
can be served as well or almost as well by granting an exemption to those burdened, the Court
must grant the exemption.

In permissive accommodation, the Court finds that the state may, but is not required to,
accommodate religious interests. The limits of permissible state accommodation is not co-
extensive with noninterference mandated by the free exercise clause. In Smith, it was effectively
ruled that this is the only accommodation allowed.

There is prohibited accommodation when the Court finds no basis for a mandatory
accommodation or it determines that legislative accommodation violates the establishment or free
exercise clause. Establishment concerns prevail over potential accommodation interests.

To determine which action to take, it is the strict scrutiny-compelling state interest test which
is most in line with the benevolent neutrality-accommodation approach. Usually now, free
exercise arguments contemplate religious exemptions from otherwise general laws as it is rare
now that laws be enacted specifically to disable religious belief.
The compelling state interest test follows a 3 step process. If the plaintiff can show that a
government practice inhibits free exercise, the burden shifts to government to show that it is
necessary to accomplish some compelling secular objective and that it is the least restrictive means
of achieving that objective. Plaintiff’s beliefs must also be sincere.

6. Some permissible accommodations in US jurisprudence are mandatory accommodations in ours,


like the constitutional provision on tax exemption of church property, salary of religious officers
in government institutions, and optional religious instruction. These provisions are wholly ours
and have no counterpart in the US constitution. PH jurisprudence allowed exemptions from a law
of general application. In effect, it interpreted our religion clauses to cover both mandatory and
permissive accommodations.

American Bible Society v. City of Manila is an example where plaintiff was exempted from a law
of general application based on the free exercise clause. Another case with mandatory
accommodation is Ebralinag v. The Division Superintendent of Schools. Permissive
accommodation is supported by the case of Victoriano v. Elizalde Rope Workers Union where
religious exemption is granted by a legislative act. The law, RA 3350, exempted employees from
the coverage of closed shop agreement based on religious objections (*INC peeps). It was held
that “government is not precluded from pursuing valid objectives secular in character even if the
incidental result would be favorable to a religion or sect. The secular purpose upheld was
actually the “advancement of the constitutional right to free exercise of religion.”

As to the test, jurisprudence mentions several. In American Bible Society, the “clear and present
danger” test was mentioned but not applied. Ebralinag employed the “Grave and immediate
danger test” and overruled Gerona, which pronounced that the test of permissibility of religious
freedom is if it violates established institutions of society and law. The use of the “compelling state
interest” test in Victoriano was inappropriate to the facts therein. Gerona is not congruent with the
benevolent neutrality approach. The “compelling state interest” test is proper where conduct is
involved for the whole gamut of human conduct has different effects on the state’s interests:
some effects may be immediate and short-term, while others delayed and far-reaching.

7. Compelling state interest test.


As mentioned, this test involves a 3 step process. 1)Has the statute or government act burdened
free exercise of religion? 2) Is there a sufficiently compelling state interest to justify this?, 3) Has
the state used the least intrusive means possible? This could result in the three situations of
accommodation mentioned- mandatory, permissive, or prohibited.

Jurisprudence covers both mandatory and permissive accommodations. Thus, not only a legislative
exemption but this court can make accommodation. While there is yet no case where there was
accommodation from general penal laws, permissive accommodation based on religious freedom
has been granted with respect to bigamy. (*muslims)

Justice Carpio argues that, based on Smith, the test in Sherbert is not applicable when the law is a
general criminal law. The question of mandatory accommodation in relation to criminal statutes
is uncertain, for PH law and jurisprudence have only yet allowed legislative accommodation.
Thus, the question is whether the Court can make exemptions like in Ebralinag and American
Bible Society, in cases involving CRIMINAL laws of GENERAL application. We hold that the
Constitution itself mandates the Court to do so for the following reasons.

1) The Benevolent neutrality-accommodation approach in PH jurisdiction is more pronounced than


in the US. 2) The purpose of the accommodation theory was to address the “inadvertent
burdensome effect” that an otherwise facially neutral law would have on religious exercise. Just
because the law is criminal should not bring it out of the ambit of the free exercise clause. 3) There
is wisdom in accommodation made by the Court as this is the recourse of minority religions who
are also protected by said clause. Mandatory accommodations are particularly necessary to protect
minority religions from the inevitable effects of majoritarianism. 4) Exemption from penal laws
on account of religion is not an entirely alien concept like in Muslim bigamy/polygamy.

5) The language of the religion clauses vis-à-vis the other fundamental rights in the BoR must be
considered. Unlike other rights, the religion clauses are stated in ABSOLUTE terms, unqualified
by the requirement of “due process,” “unreasonableness,” or “lawful order.” Only the right
to free speech is comparable in its absolute grant.

8. The current proceedings.


On the sincerity of Escritor’s religious belief, the OSG concedes this. Thus, having previously
established the preliminary conditions required by the compelling state interest (law/act inhibiting
free exercise of religion), the burden shifted to the government to demonstrate that the law justifies
a compelling secular objective and that it is the least restrictive means. The OSG’s evidence
fails to demonstrate the “gravest abuses endangering paramount interests” which could override
Escritor’s right to religious freedom. It also did not exert effort to show that the means are the least
intrusive.

The OSG claims that there is compelling interest to override religious belief to protect marriage
and the family as basic social institutions. The arrangement of Escritor and in the Declaration
should not be recognized as it is destructive of the institutions of marriage and the family. It is
claimed that this circumvents the RPC.

There is no question that the state has an interest in protecting marriage and family. But free
exercise of religion is specifically articulated as one of the fundamental rights in the Constitution.
Thus, it is not enough to contend that the state’s interest is important. The state must articulate in
specific terms the state interest involved preventing exemption, which must be compelling. It is
not the state’s broad interest in protecting such institutions of marriage and family that must be
weighed against Escritor’s claim, but its narrow interest in refusing to make an exception for
the cohabitation which Escritor’s faith finds moral. The government must do more than assert
the objectives at risk. It must precisely show how and to what extent those objectives will be
undermined if exemptions are granted.

The state’s interest cannot be merely abstract or symbolic.

The OSG also argues that the conjugal arrangement of Escritor should not be condoned as
adulterous relationships are frowned upon by society and the state laws on marriage, which are
moral in nature, take precedence over her religious beliefs or practices of any church. He argues
based on morality.

Public morality expressed in the law is necessarily SECULAR for the religion clauses prohibits
establishing a religion, including the morality it sanctions. Thus, when the law speaks of
“immorality”, the distinction between public and secular morality on one hand and religious
morality on the other should be kept in mind. Benevolent neutrality could allow for
accommodation of religious morality. The jurisdiction of the court extends only to public and
secular morality.

The government’s conduct may appear innocent, but it is in effect oppressive to the minority. Even
assuming the OSG proved a compelling state interest, it has to further demonstrate that the means
is the least intrusive so that free exercise is not infringed more than necessary. He utterly failed to
prove this element. Thus, the SC found in “this particular case and under these distinct
circumstances” that Escritor’s conjugal arrangement cannot be penalized as she made out a case
for exemption from the law based on her fundamental right to religious freedom.

LIBERTY OF ABODE AND OF TRAVEL


135. Ricardo Silverio v. CA, GR 94284, April 8, 1991, Melencio-Herrera, J. (Liberty of Abode
and of Travel)
FACTS:
Silverio was charged with violating Sec.20(4) of the Revised Securities Act in a criminal case in
the RTC. He posted bail for provisional liberty. More than 2 years after filing of the information,
respondent people of the PH filed an urgent motion to cancel the passport of and to issue a hold-
departure order against Silverio since he had gone abroad several times without the necessary
court approval resulting in postponements of his arraignment and hearings. The bail bond he
posted had been cancelled and warrants of arrest had been issued against him due to his failure to
appear at scheduled arraignments.

The RTC ordered the DFA to cancel Silverio’s passport or to deny his application therefor, and
the commissioner of immigration to prevent Silverio from leaving the country. This is based on
the fact that since the filing of the information, Silverio has not yet been arraigned as he never
appeared in court on the scheduled arraignment and there is evidence that Silverio left the country
without knowledge and permission of the court.

The CA denied his appeal. Hence this petition for review.

ISSUE:
Whether the RTC may validly prohibit Silverio from leaving the country on the ground that he left
the country on several occasions without permission and knowledge of the court, thus not
appearing on his scheduled arraignments and hearings since the filing of the information more than
2 years earlier.

HELD: YES.
Silverio claims that the right to travel can be impaired upon lawful order of the court only on
grounds of “interest of national security, public safety, or public health.”
Warrants of arrest having been issued against him for violation of the conditions of his bail bond,
he should be taken into custody. Bail is a security given for the release of a person in custody of
the law, furnished by him or a bondsman, conditioned upon his appearance before any court when
so required by the court or rules. This condition is a valid restriction of his right to travel. A
person facing criminal charges may be restrained by the Court from leaving the country or, if
abroad, compelled to return. “An accused released on bail may be re-arrested without a warrant if
he attempts to depart from the PH without prior permission of the court where the case is pending.”

In the 1935 constitution, liberty of abode and of travel were treated under one provision: "The
liberty of abode and of changing the same within the limits prescribed by law shall not be
impaired." The 1973 constitution altered this by explicitly including the liberty of travel: "The
liberty of abode and of travel shall not be impaired except upon lawful order of the court or when
necessary in the interest of national security, public safety, or public health"

The 1987 constitution has split the 2 freedoms into two distinct sentences and treats them
differently: "Sec. 6. The liberty of abode and of changing the same within the limits prescribed
by law shall not be impaired except upon lawful order of the court. Neither shall the right to travel
be impaired except in the interest of national security, public safety, or public health, as may be
provided by law."

Silverio’s claim that courts can impair the right to travel only on the grounds of “national security”
etc. is not well taken. Sec.6 should be interpreted to mean that while the liberty of travel may be
impaired even without court order, the executive or administrative authorities are not armed with
arbitrary discretion to impose limitations. They can impose limits only on the basis of “national
security, public safety, or public health” and “as may be provided by law, a limitive phrase not in
the 1973 text. The phraseology in the 1987 Constitution was a reaction to the ban on international
travel by the Marcos regime when there was a Travel Processing Center, which issued certificates
of eligibility to travel upon application of an interested party.

Sec.6 should not be construed as delimiting the inherent power of the Courts to use all means
necessary to carry their orders into effect in criminal cases pending before them. When by
law jurisdiction is conferred on a court or judicial officer, all auxiliary writs, process, and other
means necessary to carry it into effect may be employed by such court or officer. Holding an
accused in a criminal case within the reach of courts by preventing his departure from PH should
be considered a valid restriction on his right to travel so that he may be dealt with in accordance
with law.

136. Ferdinand E. Marcos v. Hon. Raul Manglapus, GR 88211, September 15, 1989, Cortes,
J. (Liberty of Abode and Travel)
FACTS:
Marcos was deposed from the presidency thru the people-power in February 1986 and forced into
exile. Cory was declared president in his stead under a revolutionary government. The failed
Manila Hotel coup in 1986 led by political leaders of Marcos, the takeover of TV station channel
7 by rebels, and the unsuccessful plot of the Marcos spouses to surreptitiously return from Hawaii
with mercenaries aboard an aircraft chartered by a Lebanese arms dealer awakened the nation to
the capacity of the Marcoses to stir trouble even from afar and to the blind loyalty of their
followers in the country.

The ratification of the 1987 Constitution enshrined the victory of “people power”. But this did not
stop bloody challenges to the government. On August 28, 1987, Col. Gregorio Honasan led a failed
coup that left many people, combatants and civilians, dead. There were other lesser armed sorties,
but their messages were the same- the threat that civilian government could be at the mercy of a
fractious military.

There were also the communist insurgency and secessionist movement in Mindanao which gained
ground during the rule of Marcos. Now, Marcos, in his deathbed, signifies his wish to return to PH
to die. But Cory, considering the dire consequences to the nation of his return, has stood firmly on
the decision to bar his return.

This case is unique. It should not create a precedent. For the case of a dictator forced out of office
and into exile after causing 20 years of political, economic, and social havoc in the country and
who within the short space of 3 years seeks to return, is in a class by itself. This petition for
mandamus asks the Court to order respondents to issue travel documents to Marcos and his family
and to enjoin implementation of Cory’s decision to bar their return to PH.

ISSUE:
Whether, in the exercise of the powers granted by the Constitution, Cory may prohibit the
Marcoses from returning to the Philippines on the ground that such return may cause dire
consequences to the nation which is still recovering economically and socially.

HELD: YES.
Petitioners argue that the right to return to the PH is guaranteed under Sections 1 and 6 of the Bill
of Rights. The president has no power to impair the liberty of abode because only a court may do
so “within the limits prescribed by law.” Cory is not authorized to impair this right as no law has
authorized her to do so. Before the right to travel may be impaired by any authority or
government agency, there must be legislation to that effect.

Section 6. The liberty of abode and of changing the same within the limits prescribed by
law shall not be impaired except upon lawful order of the court. Neither shall the right to
travel be impaired except in the interest of national security, public safety, or public health,
as may be provided by law.

Marcos also asserts that under international law, his right to return to PH is guaranteed. The UDHR
provides:
Article 13. (1) Everyone has the right to freedom of movement and residence within the
borders of each state. (2) Everyone has the right to leave any country, including his own,
and to return to his country.

The ICCPR, ratified by PH, provides: Article 12 xxx 4) No one shall be arbitrarily deprived of the
right to enter his own country.
Respondents argue that the issue here is a political question.

1. International Law
It would not do to view the case within the confines of the right to travel. The right involved is not
the right to travel from the PH to other countries or within the PH. These are what the right to
travel normally connote. The right involved is the right to return to one’s country, a totally
DISTINCT right under international law, independent from although related to the right to
travel. The UDHR and ICCPR treat the right to freedom of movement and abode within the
territory of a state, the right to leave a country, and the right to enter one’s country as separate
and distinct rights. The UDHR in Art.13(1) speaks of the right to free movement within a country,
separately from the right to leave and to return in his country in Art.13(2). The ICCPR guarantees
the right to liberty of movement and freedom to choose residence (12[1]), to be free to leave any
country including his own (12[2]), which rights may be restricted by laws necessary to protect
national security, public order, public health or morals or the separate rights and freedoms of others
(12[3]) as distinguished from the “right to enter his own country” of which one cannot be
“arbitrarily deprived.” (12[4]). It is thus inappropriate to construe the limits to the right to return
in the same context as those pertaining to the liberty of abode and the right to travel.

The right to return to one’s country is not among the rights guaranteed in the Bill of Rights,
which treats only of liberty of abode and right to travel, although it may be considered as a
generally accepted principle of international law, which, under our constitution, is part of the
law of the land (Art.II, S2). But it enjoys a different protection under the ICCPR- against being
“arbitrarily deprived.”

2. Executive Power.
The 1987 Constitution has fully restored the separation of powers. It also confers plenary
legislative, executive, and judicial powers subject only to limitations provided in the Constitution.
Art.VII S1 provides that “Executive power” is vested in the President. But it does not define
“executive power”, but it touches in some sections of certain powers of the president, i.e. power
of control, etc. The question is: by enumerating certain powers of the president, did the
framers intend that the president shall exercise those specific powers and no other?
Petitioners assert inclusion unius est exclusion alterious.

The 1935 Constitution created a strong president with explicitly broader powers than the US
president. The 1973 attempted to modify the system of government into the parliamentary type
with the president as mere figurehead, but through numerous amendments, the president became
even more powerful to the point that he was also the de facto legislature. The 1987 Constitution
brought back the presidential system of government and restored separation of powers and with
provisions for checks and balances.

It is inaccurate to say that executive power is the power to enforce the laws, for the president is
head of state and of government, and whatever powers inhere in such position pertains to the
office unless the constitution withholds it. Although the 1987 Constitution imposes limits on the
exercise of specific powers of the president, it maintains intact what is traditionally considered
as within the scope of “Executive power.” Thus, the president’s powers is not limited only to the
specific powers enumerated. It is said that whatever power inherent in the government that is
neither legislative nor judicial has to be executive.

The Constitution declares that the “prime duty of the government is to serve and protect the people”
and that the maintenance of peace and order etc. are essential for the enjoyment of the blessings of
democracy (art.II, Secs. 4 & 5). In the exercise of presidential functions, in drawing a plan of
government, in directing implementing action for these plans, and in making decisions as
president, the president must consider these principles and adhere to them.

The president considered these basic principles in arriving at a decision of whether to allow the
Marcoses to return. The president has the obligation under the constitution to protect the people,
promote their welfare and advance national interest. The Constitution is a social contract where
the people have surrendered their sovereign powers to the state for the common good. “Sovereignty
resides in the people and all government authority emanates from them.” (Art.II, S1)

To the president, the problem is one of balancing general welfare against the rights of certain
individuals. The power involved is the president’s residual power to protect the general welfare
of the people. The request or demand of the Marcoses to be allowed to return must be considered
in light of those residual unstated powers of the president which are implicit in and correlative to
the paramount duty residing in that office to safeguard and protect general welfare.

3. Political Question? - NO
The present Constitution limits the resort to the political question doctrine and broadens the scope
of judicial inquiry, but nonetheless there remain issues beyond the Court’s jurisdiction the
determination of which is exclusively for the president, congress, or the people themselves thru
plebiscite or refendum. When political questions are involved, the Constitution limits
determination to whether there has been GADALEJ. This is found in Art.VIII, Sec.1, which
incorporates in the fundamental law the ruling in Lansang v. Garcia.

Thus, the question is whether there exist factual bases for the president to conclude that it was
in the national interest to bar the return of the Marcoses into PH. If there is, then Cory did not
act arbitrarily or gravely abused her discretion. We find that from the oral arguments and the facts
revealed during the briefing in the chambers by the Chief of Staff of the AFP and the National
Security Adviser there exist factual bases for the president’s decision.

The country is besieged from within by a communist insurgency and rightist conspiracies to grab
power, urban terrorism, etc. The documented history of the efforts of the Marcoses and their
followers to destabilize the country bolsters the conclusion that the return of the Marcoses at this
time would only exacerbate and intensify the violence against the state and instigate more
chaos. With these before her, the president cannot be said to have acted arbitrarily.

Gutierrez, Jr., DISSENTING:


"The Constitution . . . is a law for rulers and people, equally in war and in peace, and covers with
the shield of its protection all classes of men, at all times, and under all circumstances. No doctrine
involving more pernicious consequences was ever invented by the wit of man than that any of its
provisions can be suspended during any of the great exigencies of government." -Ex Parte Miligan.
The Court has permitted a basic freedom to be taken away by government. I am disturbed by the
majority ruling which declares that it should not be a precedent. We are interpreting the
Constitution for only one person and constituting him into a class by himself. The constitution
is a law for all classes of men at all times. To have a person as one class by himself smacks of
unequal protection of the laws.

I believe the issue is one of rights and not of power. Marcos is insensate and would not live without
the machines for his kidneys and organs. To treat him as one with full panoply of power against
whom government forces should be marshalled is unrealistic. The government has the power to
arrest and punish him. But does it have the power to deny him his right to come home and die
among familiar surroundings? Hence this dissent.

Sec.6 of the BoR provides: xxx. The OSG invokes national security and public safety which is
hauntingly familiar as it was pleaded so often by Marcos to justify his acts under martial law. There
is no showing of existence of a law prescribing the limits of the power to impair and the occasions
for its exercise (“…as may be provided by law.”). And except citing breaches of law and order,
the more serious of which were totally unrelated to Marcos and which the military was able to
quell, the OSG has not pointed to any grave exigency which allows indefinite suspension of the
right to travel.

- Political question
For a political question to exist, there must be in the constitution a power vested exclusively in the
president or congress, the exercise of which the court should not examine or prohibit. The OSG
has not pointed to any provision in the constitution which commits or vests the determination of
the question raised to us solely in the president. The authority implied in Sec.6 of the BoR itself
does not exist because NO LAW has been enacted specifying the circumstances when the
right may be impaired in the interest of national security or public safety.

The closest resort to a textually demonstrable constitutional commitment of power may be found
in the commander-in-chief clause allowing the president to call out the armed forces in case of
lawless violence, invasion or rebellion etc. but there is no showing, not even a claim that the
followers of Marcos are engaging in rebellion or that he is in a position to lead them. The
respondents themselves are hardpressed to state who or what constitutes a Marcos “loyalist. Their
insinuations that the loyalist group is heavily funded by Marcos and his cronies and that they have
to be paid allowances to rally are strong indications that these “loyalists” who would follow
Marcos right or wrong are so few in number that they could not possibly destabilize the
government.

The Court should view the return of Marcos and his family solely in the light of the guarantee
of liberty of abode and travel as against the contention that national security and public safety
would be endangered by a grant of the petition.

Sec.6 provides that the liberty of abode and of changing the same within the limits prescribed by
law may be impaired ONLY upon a lawful order of a court. Not by an executive officer or the
president. It also says that the right to travel, obviously including the right to travel out of or
back into the PH, cannot be impaired except in the interest of national security, public safety, or
public health, as may be PROVIDED BY LAW.

There is no law setting the limits on a citizen’s right to move from one part of the country to
another or from the PH to a foreign country or from a foreign country to PH. There would also be
no disrespect of presidential determination if we grant the petition. We would simply be applying
the constitution.

The framers believed that the use of the political question doctrine allowed the Court during the
Marcos years to fall back on prudence etc. when it refused to examine and strike down an exercise
of authoritarian power. The Constitution has accordingly been amended and We are now precluded
by its mandate from refusing to invalidate a political use of power through a convenient resort to
the political question doctrine. The constitution requires the Court to determine GADALEJ. How
do we determine grave abuse of discretion?

The tested procedure is to require the parties to present evidence. Since vital information is usually
highly classified, the Court granted the OSG’s offer that the military give us a closed door factual
briefing with a lawyer for petitioners and a lawyer for respondents. The other method is to
avail of judicial notice. Here, judicial notice would be the only basis for determining the clear
and present danger to national security and public safety. The majority took judicial notice of
the communist rebellion, separatist movement, rightist conspiracies, and urban terrorism.
But is it fair to blame the present day Marcos for these incidents? All these problems are totally
UNRELATED to the Marcos of today and are in fact led by the people who have always opposed
him.

But we do not need to look into the factual bases of the ban Marcos policy nor are we forced to
fall back upon judicial notice to determine grave abuse. In the first place, the president never
pronounced that a clear and present danger to national security will arise if Marcos and his
family are allowed to return. It was only after this petition was filed that the alleged danger to
national security and public safety conveniently surfaced in the OSG’s pleadings. Cory limits
herself to the following reasons for banning Marcos’ return: 1) national welfare and interest,
and 2) continuing need to preserve the gains achieved in terms of recovery and stability. Both
do not satisfy the criteria of national security and public safety. We cannot validate her stance
simply because it is a popular one. SC decisions do not have to be popular as long as they follow
the constitution and the law.

There is rebellion not by Marcos followers but by the NPA. The fear that these communist rebels,
bangsamoro secessionists, Honasan ex-soldiers, loyalists, and other dissatisfied elements would
suddenly unite to overthrow the PH should a dying Marcos come home is too speculative and
unsubstantial to deny a constitutional right. It is not shown how extremists from the right and left
who loathe each other could find a rallying point in the coming of Marcos.

- Right to travel
I disagree with the dictum on the right to travel. We should not differentiate the right to return
home from the right to go abroad or to move around in the PH. If at all, the right to come home
must be more preferred than any other aspect of the right to travel. It was precisely the banning
of Marcos of the right to travel of Sen. Aquino, Jovito Salonga, and other undesirables during that
unfortunate period which led the framers not only to re-enact but to strengthen the declaration of
this right.

Cruz, J. DISSENTING:
Marcos, as a citizen of PH, is entitled to return to and live, and die, in his own country. The question
is whether respondents acted with grave abuse of discretion in barring Marcos from his own
country. My reluctant conclusion is that they have, absent proof that they said they could offer, but
could not, that Marcos’ return would prejudice the security of the state.

In the two hours of briefing (closed-door hearing), the government failed dismally to show that
the return of Marcos dead or alive would pose a threat to national security. The fears were based
on mere conjectures of political and economic destabilization without a single piece of
concrete evidence to back up their apprehensions. Amazingly, the majority concluded that there
are factual bases. This is not my recollection of the impressions of the court after that hearing.

The holding that the president has residual powers in addition to the specific powers granted by
the constitution does not square with the policy of the framers, which was precisely to limit rather
than expand presidential powers as a reaction to the excesses of the past dictatorship. When I sit
in judgment as member of this Court, I must cast all personal feelings aside.

Paras, J. DISSENTING:
The issue whether Marcos should be allowed to return to PH may be resolved by answering two
simple questions: 1) does he have the right to return to his own country?; and 2) should national
safety and security deny him this right? Marcos is still a PH citizen and both under the UDHR
and 1987 Constitution, he has the right to return to his own country except only if prevented by
the demands of national safety and security. Our armed forces failed to prove this danger.
They can only rely on sheer speculation. There is some danger but no showing as to the extent.

Thus, for me, Marcos should be allowed to reutn under condition that he and his family be under
house arrest in Ilocos Norte and should any of them die, the body should not be taken out of the
municipality of confinement and should be buried within 10 days.

Padilla, J. DISSENTING:
The core issue here is which right will prevail between the right of a Filipino, Marcos, to return to
PH and the right of the PH government to bar such return in the interest of national security. The
right to travel comprises the right to travel within the country, to travel out of the country and
to return to the country. Short of all such components, the right to travel is meaningless.
With or without restricting legislation, the interest of national security, public safety, or public
health can justify and even require restrictions on the right to travel. “As may be provided by law”
merely declares a constitutional permission for congress to enact laws that may restrict the right
to travel in the interest of national security etc. The power to restrict travel finds abundant support
in the police power of the state.

Yet, the power of the state, even if founded on police power, cannot be absolute under all
circumstances, much less can it be arbitrary and irrational. Marcos comes before the court as a
Filipino invoking a constitutional right. Have respondents presented sufficient evidence to offset
this exercise of right by Marcos? Did respondents show sufficient factual bases and data? I
assessed the “briefing” but searched in vain for convincing evidence that would defeat the right of
Marcos as Filipino to return. Respondents’ apprehensions do not escalate to proportions of
national security or public safety.

As member of the UN, PH has obligations under its charter. By adopting the generally accepted
principles of international law as part of the law of the land, PH cannot just pay lip service to
Art.13, par.2 of the UDHR. The guarantee to return to his country is reiterated in Art.12, par.2 of
the ICCPR (no one shall be arbitrarily deprived of the right to enter his own country”).
“Arbitrarily” was chosen by the drafters of ICCPR to protect an individual against excessive
encroachment on his rights by the state based on national traditions or a particular sense of justice
falling short of international law standards.

The drama today is the same drama in 1983 (Sen.Benigno Aquino). I asked the OSG how one
could validly defend the right of former Sen. Aquino to return to PH in 1983 and at the same time
credibly deny the right of Marcos, also Filipino, to return in 1989. I have not found a satisfactory
answer.

Sarmiento, J. DISSENTING:
The only issue is whether in the exercise of the powers granted by the Constitution, the President
may prohibit the Marcoses from returning to PH. I fear that my brethren have overstepped the
bounds of judicial restraint or, even worse, convicted them without trial. I also find quite strained
what the majority says are the “real issues”: right to return pitted against the right to travel and
freedom of abode, and their supposed distinctions under international law, as if such
distinctions under international law in truth and in fact exist. There is only one right involved
here, whether under municipal or international law: the right to travel whether within one’s own
country or to another and the right to return thereto. The Constitution makes no distinctions; thus,
let no one make a distinction. Ubi lex non distinguit, nec nos distinguere debemus.

While the president has powers not found expressly in the charter, but has them by constitutional
implication, the latter must yield to the paramountcy of the bill of rights. It does not suffice to
say that the president’s powers or by constitutional implication prevail over express constitutional
commands. This argument rests not upon the test of the constitution but upon a mere inference
therefrom. For if it were indeed the intent of the charter to create an exception by presidential
action to the right to travel or liberty of abode and changing the ame other than what it explicitly
says already (limits prescribed by law, upon lawful order of the court), the charter could have
specifically declared so. The lone deterrents to the right in question are: 1) decree of statute, or
2) lawful judicial mandate. Had the Constitution intended a third exception by presidential
initiative, it could have so averred. There is no law banning Marcos from the country nor decree
banishing him from PH.

Under the 1973 Constitution (Sec. 5. The liberty of abode and of travel shall not be impaired except
upon lawful order of the court, or when necessary in the interest of national security, public safety,
or public health), the right may be abated 1) upon lawful court order or 2) when necessary in the
interest of national security, etc. This enabled Marcos to moderate movement of citizens which,
Bernas says, justified “hamletting”, forced relocations, or establishment of free-fire zones.

But the new constitution has divested the executive’s implied power. The right may be impaired
ONLY within the limits provided by law. The president is out of the picture.

Assuming that the president may legally act, the question arises as to whether it has been proved
that Marcos or his return will pose a threat to national security, public safety, or public health.
There are vehement insistences that Marcos poses a threat to the national good, yet there are
persistent claims by the military during the closed-door hearing that this “government will not fall”
should Marcos step on PH soil. Which is which? At any rate, this determination cannot be left
solely to the president. The Court itself must be content that the threat is clear and present.

The president indeed has the obligation to protect the people. But from whom? If we say “from
Marcos”, we unravel chinks in our political armor and it flies in the face of claims that the
government “will not fall.”

The majority started this inquiry on the question of power. I hold that the president, under the
present constitution and existing laws, does not have it. Mandamus thus lies.

CRUZ, J. DISSENTING IN MReconsideration, October 27, 1989:


The death of Marcos has been met only with passing interest or if not indifference. This only shows
that if he was at all a threat to the national security when he was already moribund, that feeble
threat has died with him. His cadaver is not even regarded as a symbol of this or that or whatever
except by his fanatical followers. It is only a dead body waiting to be interred in this country. We
have more important things to do than debating over a corpse. I say let it be brought home and
buried deep and let us be done with it forever.

PARAS, J. DISSENTING:
I find no reason to deviate from my dissenting opinion already expressed. It is not correct to say
that a dead man has ceased to have rights. For instance, the RPC prohibits libel against a deceased
individual. And what about the human rights of his widow and other family members?

Up to now, the alleged threats to national security have remained unproved and unpersuasive. Our
armed forces can easily control and possible uprising or destabilization. In fact, the converse
appears to be nearer to the truth, that is, if we do not allow the remains to come, more trouble may
be expected.

Reconciliation can proceed faster if the petition for the return is granted. Refusing the request can
mean hardening of resistance against the administration. To grant it may soften the hearts of the
oppositionists, paving the way for a united citizenry.

PADILLA, J. DISSENTING:
Respondents succeeded in denying Marcos the right to return to and die in this country. The
remaining right of this Filipino that cries out for vindication is the right to be buried in this
country. Will respondents be allowed to complete the circle of denying the human right of Marcos
to travel? They should not be allowed if the constitution is still to prevail.

Those who deny this Filipino the right to be buried in this country say that this right applies to any
Filipino except Marcos because he was a dictator and he plundered the country. This is the most
irrelevant argument. Our democracy is built on the fundamental assumption that the constitution
applies to all Filipinos, whether dictator or pauper, learned or ignorant, religious or agnostic, as
long as he is Filipino.

It is said that letting this Filipino be buried here would pose a serious threat to national security.
What threat? As said in my previous dissent, respondents have not presented any hard evidence
of such threat. All we have are general conclusions of national security and public safety in
avoidance of a demandable constitutional and basic human right to return.

Marcos’ supporters pose a greater threat to peace and order with Marcos deprived of his right
to burial here. If Marcos be allowed burial here, his supporters would be deprived of an otherwise
potent argument so conducive to mass protests and violence that their idol has been cruelly denied
the right to be buried in his homeland.

It is also argued that Marcos, in cadaver form, has no constitutional or human rights to speak
of. One cannot overlook that the right of Marcos as Filipino to be buried in this country is asserted
not for the first time after his death as it was asserted before he died. More importantly, the right
of every Filipino to be buried in his country is part of a continuing right that starts from
birth and ends only the day he is finally laid to rest in his country. This opinion recognizes
that the PH government can lay down conditions for Marcos’ burial, but I submit that these
conditions must recognize the right of the man as a Filipino to be buried in this country NOW.

The majority ignored the constitutional dimension of the problem. What predictably follows will
be a continuing strife among our people of unending hatred and retaliations.

SARMIENTO, J. DISSENTING:
I do not agree that the president has unstated residual powers implied from the grant of executive
power necessary for her to comply with her duties under the Constitution, “this notwithstanding
the intent of the framers to limit the powers of the president.” It is a nice word game, but nothing
else.

The constitution says that the right to travel may only be restricted by 1) a court order, or 2) fiat
of law. Had it intended presidential imprimatur, it would have said so. It would also have
completed the symmetry: judicial, congressional, and executive restraints on the right. No amount
of presumed residual executive power can amend the charter.

The BoR primarily limits not only legislative encroachments on individual liberties, but more so
against presidential intrusions. That the threats to government to which the return of the marcoses
has been viewed to provide catalytic effect have not been shown to have ceased is mere conjecture,
speculation, and imagination. The military has shown no hard evidence that the Marcoses’ return
would pose a threat to national security.
RIGHT TO INFORMATION
137. Valentin Legaspi v. CSC, GR 72119, May 29, 1987 (Right to Information)
FACTS:
CSC denied Legaspi’s request for information on the civil service eligibilities of certain persons
employed as sanitarians in the Health Department of Cebu City. These government employees,
Julian and Mariano, had allegedly represented themselves as civil service eligible who passed the
civil service exams for sanitarians. Claiming that his right to be informed of their eligibilities is
guaranteed by the constitution, and he has no other plain speedy and adequate remedy to acquire
the info, Legaspi prays for issuance of mandamus to compel CSC to disclose the info.

ISSUE:
Whether a citizen may inquire upon the civil service eligibility of certain government employees
representing themselves as civil service eligible who have passed the civil service exams for
sanitarians.
HELD: YES.
The right to information on matters of public concern first gained recognition in the BoR in the
1973 Constitution. This is retained and amplified by Art.III, S7 of the 1987 Constitution with the
addition of the phrase” as well as to government research data used as basis for policy
development.”
The right of the people to information on matters of public concern shall be recognized.
Access to official records, and to documents, and papers pertaining to official acts,
transactions, or decisions, as well as to government research data used as basis for policy
development, shall be afforded the citizen, subject to such limitations as may be provided
by law.

This provision is self-executing. What may be provided by the legislature are reasonable
conditions and limitations upon the access to be afforded which must, of necessity, be
consistent with the declared state policy of full public disclosure of all transactions involving
public interest (Art.II, S28).

A petition for mandamus must be instituted by a party aggrieved by alleged inaction of any
tribunal, corporation, board, or person which unlawfully exclused such party from the enjoyment
of a legal right. He must be an “aggrieved party”.

1. Standing?
It is alleged that Legaspi has no actual interest in the eligibilities of Julian and Mariano, thus he
has no standing. But Legaspi anchored his case upon the right of the people to information, which
by its nature is a PUBLIC RIGHT. When the question is one of public right and the object of
mandamus is to procure the enforcement of a public duty, the people are regarded as the real
party in interest and the relator at whose instigation the proceedings are instituted need not show
that he has any legal or special interest in the result, it being sufficient to show that he is a
CITIZEN and as such interest in the execution of the laws.

“Public” embraces every person. Legaspi, being a citizen, has personality to seek redress for
alleged obstruction of the exercise of the public right.
2. For every fundamental right lies a corresponding duty on those who govern to respect and
protect that right. The guarantee under Art.III, S7 is further enhanced with the policy of full public
disclosure, this time “subject to reasonable conditions prescribed by law,” in Art. II, S28:
Subject to reasonable conditions prescribed by law, the State adopts and implements a
policy of full public disclosure of all its transactions involving public interest.

In Subido v. Ozaeta, we said: Except perhaps when it is clear that the purpose of the examination
is unlawful, or sheer idle curiosity, we do not believe it is the duty under the law of registration
officers to concern themselves with the motives, reasons, and objects of the person seeking access
to the records. If it be wrong to publish the contents of the records, it is the LEGISLATURE and
not the officials having custody thereof which is called upon to devise a remedy.

Thus, government agencies are without discretion in refusing disclosure of, or access to,
information of public concern. This is not to lose sight of the reasonable regulations which
may be imposed by said agencies in custody of public records on the manner in which the right
to info may be exercised by the public. In Subido, we recognized the authority of the register of
deeds to regulate the manner in which persons may inspect, examine or copy records relating to
registered lands. But such regulations are confined to manner and hours of examination to the
end that damage to or loss of the records may be avoided, that undue interference with the
duties of the custodian of the books, documents, and other employees may be prevented, that
the right of other persons entitled to make inspection may be insured.

But the authority to regulate the manner of examining public records does not carry with it the
power to prohibit. The prohibition is a limitation upon availability of access to the info sought,
which only the legislature may impose. Regulation pertains to the government agency charged
with the custody of public records. Its authority to regulate access is to be exercised solely to the
end that damage to or loss of xxx.

Thus, the manner of examining public records may be subject to reasonable regulation, but the
duty to disclose information cannot be discretionary on the part of said agencies. Thus, its
perfomrnace may be compelled by mandamus in a proper case. What is a proper case for
mandamus to issue? The question is whether the information sought is within the ambit of the
constitutional guarantee.

The right to info is not absolute. It does not open every door to all info. Access to official records,
etc. are “subject to limitations as may be provided by law. The law may exempt certain types
of info from public scrutiny like those affecting national security. Thus, it follows that the
availability of access to a public record must be circumscribed by the nature of the info sought, i.e.
1) being of public concern or one that involves public interest, and 2) NOT being exempted
by law from the operation of the constitutional guarantee. Is the info of public concern?

In case of denial, the agency has the burden of showing that the info is not of public concern,
or if it is, that the same has been exempted by law. The government is in an advantageous
position to marshall and interpret arguments against release. To safeguard the constitutional right,
every denial of access by the agency is subject to review by the courts and, in the proper case,
access may be compelled by mandamus.

There is no rigid test in determining whether an information is of public concern. “Public conern,”
like “public interest” is a term that eludes exact definition. Both terms embrace a broad spectrum
of subjects which the public may want to know either because these directly affect their lives, or
simply because such matters naturally arouse the interest of an ordinary citizen. In the final
analysis, it is for the COURTS to determine in a CASE BY CASE basis whether the matter
at issue is of interest or importance, as it relates to or affects the public.

In Tañada v. Tuvera, the public concern invoked was the need for adequate notice to the public of
various laws which regulate the conduct of citizens. In Subido, it was the knowledge of those real
estate transactions which some believed to have been registered in violation of the constitution.
The info sought in this case is the truth of the claim of certain government employees that they
are civil service eligible for the positions to which they were appointed. The constitution
expressly declares as a state policy that:
Appointments in the civil service shall be made only according to merit and fitness to be
determined, as far as practicable, and except as to positions which are policy determining,
primarily confidential or highly technical, by competitive examination. (Art. IX, B, Sec. 2.
[2]).
Public office being a public trust, it is the legitimate concern of citizens to ensure that
government positions requiring civil service eligibility are occupied only by persons who are
eligible.

The CSC also failed to cite and provision in the Civil Service Law which would limit the
petitioner’s right to know who are, and who are not, civil service eligible. We take judicial notice
of the fact that the names of those who pass the civil service exams are released to the public.
Thus, there is nothing secret about one’s civil service eligibility, if actually possessed. Legaspi’s
request is thus neither unusual nor unreasonable. And when government employees claim to be
eligible, the public, thru any citizen, has a right to verify their professed eligibilities from the CSC.
The eligibility of a sanitarian being of public concern, and without express limitation under the
law upon access to the register of eligible for said position, CSC has the duty to confirm or deny
eligibility of any person occupying the position. Mandamus thus lies.

138. Ricardo Valmonte v. Fliciano Belmonte, GR 74930, February 13, 1989 (Right to
Information; right to info is not right to compel custodian of official documents to make lists)
FACTS:
Petition for mandamus with preliminary injunction. The controversy arose when Valmonte, media
practitioner, wrote Belmonte a letter requesting the list of names of opposition members of the
Batasang Pambansa who were able to secure a clean loan of P2M each on guaranty of Imelda
Marcos and certified true copies evidencing their loan. It was denied on the ground that “a
confidential relationship exists between the GSIS and all those who borrow from it.” Hence this
suit, praying that they be 1) furnished the list of names of the BP members of UNIDO and PDP-
Laban who secured the loans before the Be.7 election thru the intercession of Imelda, 2) furnished
certified true copies of the documents evidencing the loans, and 3) allowed access to public records
for the subject information.
ISSUE:
Whether Valmonte, member of the media and citizen of the PH, has a right to be informed on the
list of names of opposition members of the defunct Batasang Pambansa who were able to secure a
clean loan of P2M each on guaranty of Imelda Marcos.
HELD: YES. But not to compel them to make a list.
As media practitioners, petitioners have the right to gather and the obligation to check the accuracy
of information they disseminate. The right to info is an essential premise of a meaningful right to
speech and expression (*and press). The right to info goes hand-in-hand with the constitutional
policies of full public disclosure(AII,S28) and honesty in the public service (AXI, S1).

1. But the right to info is not absolute. It is limited to “matters of public concern” and “subject to
such limitations as may be provided by law.” Before mandamus may issue, thus, the info sought
must be of “public interest” or “public concern” and is not exempted by law from the operation of
the constitutional guarantee.

The Court has always grappled with the meanings of “public interest” or “public concern” as
observed in Legaspi v. CSC (case by case basis). In Tañada, the public concern is the need for
adequate notice to the public of the laws which regulate the conduct of the citizens. In Legaspi, it
was the legitimate concern of citizens to ensure that government positions requiring civil service
eligibility are occupied only by persons who are eligible.

The info sought here is the truth of reports that certain BP opposition members were able to
secure clean loans from GSIS immediately before the Feb. 7, 1986 election through the
intercession of Imelda Marcos. GSIS is a trustee of contributions from the government and its
employees and the administrator of various insurance programs for the latter’s benefit. Its funds
assume a public character. The GSIS is expected to manage its resources with utmost prudence. It
is “not supposed to grant ‘clean loans’”. It is thus the legitimate concern of the public to ensure
that these funds are managed properly to maximize benefits that accrue to the insured government
employees. The public nature of the loanable funds of GSIS and the public office of the alleged
borrowers make the information sought a matter of public interest and concern.

2. The second requisite is that the info sought must not be among those excluded by law. Belmonte
argues that there is a confidential relationship between GSIS and its borrowers. But he fails to cite
any law granting GSIS the privilege of confidentiality as to documents subject of this petition. His
position is merely based on considerations of policy. The judiciary does not settle policy issues.

3. He then invokes right to privacy. When the info requested from the government intrudes into
the privacy of a citizen, a potential conflict between the rights to info and privacy may arise. But
these competing interests need not be resolved in this case. The right to privacy belongs to the
individual in his private capacity, and not to public and governmental agencies like GSIS. The
right also cannot be invoked by juridical entities like GSIS. A corporation has no right of
privacy in its name since the entire basis of such right is an injury to the feelings and sensibilities
of the party and a corporation would have no such ground for relief.

Neither can GSIS, through its general manager Belmonte, invoke the right to privacy of its
borrowers. The right is purely personal in nature and can be invoked only by those whose privacy
is claimed to be violated. Also, the concerned borrowers here may not succeed if they invoke their
right to privacy since they held public offices when the loans were allegedly granted.

4. Belmonte then asserts that the documents evidencing the loans of GSIS are private in nature
and are not covered by the right to info on matters of public concern. It is contended that the loan
function of GSIS is incidental to its insurance function, thus its loan transactions are not covered
by the policy of full public disclosure and the right to info which applies only to “official
transactions.”

The “constituent-ministrant” dichotomy of government function has long been repudiated. The
government, whether carrying out its sovereign attributes or running some business, discharges
the same function of service to the people. That the GSIS in granting loans was exercising a
proprietary function would not justify exclusion of the transactions from the right to info. The
intent of the framers to include GOCCs and their transactions within the coverage of the policy
of full public disclosure is manifest from the records of the proceedings. Considering the framers’
intent and that GOCCs, whether performing proprietary or governmental functions are accountable
to the people, GSIS’s transactions are within the ambit of the right to info pursuant to the policy
of transparency in government dealings.

Thus, petitioners are entitled access to the documents evidencing loans granted by GSIS subject to
reasonable regulations that GSIS may promulgate relating to the manner and hours of examination
etc. The petition as to the 2nd and 3rd alternative acts sought by petitioners is meritorious.

But as to the first act to furnish the list of BP members, although citizens have the right to info, the
constitution does not accord them a right to compel custodians of official records to prepare
lists, abstracts, summaries, and the like in their desire to acquire info on matters of public
concern. Thus, they have no clear legal right to the thing demanded as to this. Mandamus does not
lie as Belmonte has no duty to prepare the list requested.

139. Carmen Aquino-Sarmiento v. Manuel Morato, GR 92541, November 13, 1991 (Right to
Information)
FACTS:
In February 1989, Sarmiento, member of MTRCB, wrote its records officer requesting that she be
allowed to examine the board’s records pertaining to the voting slips accomplished by the
individual board members after a review of the movies and television productions. Based on the
slips, films are either banned, cut or classified accordingly. The records officer informed Sarmiento
that she has to get prior clearance from Morato, chairman of MTRCB, to gain access to the records
sought.

Morato denied the request on the ground that whenever MTRCB members sit in judgment over a
film, their decisions as reflected in the slips partake the nature of conscience votes and thus are
purely private and personal. The slips are exclusive property of the member concerned and
anybody who wants access must secure the member’s consent. Sarmiento argues that the
records/slips are public and, other than regulating the manner and hours of examination, Morato
and the MTRCB have no authority to deny any citizen seeking examination of the board’s records.
In an exclusive MTRCB meeting, 17 members voted to declare their individual voting records as
classified and inaccessible to the public without clearance from the chairman. Morato denied
Sarmiento’s request. MTRCB issued Resolution 10-89, declaring as confidential, private, and
personal the decision of the reviewing committee and the voting slips of the members.

ISSUE:
Whether the voting slips of MTRCB board members pertaining to their review of films, which
votes result in the films being banned, cut, or classified, may be disclosed under the right of the
people to information regarding matters of public concern.
HELD: YES.
Sarmiento seeks to nullify MTRCB Res.10-89. Morato’s refusal to allow Sarmiento to examine
the records of MTRCB pertaining to the decisions of the review committee and the individual
voting slips of its members violates Sarmiento’s right of access to public records.
"The right of the people to information on matters of public concern shall be recognized.
Access to official records, and to documents, and papers pertaining to official acts,
transactions, or decisions, as well as to government research data used as basis for policy
development, shall be afforded the citizen, subject to such limitations as may be provided
by law."

This is a self-executing provision. What the legislature may provide are reasonable conditions and
limitations upon the access to be afforded which must be consistent with the declared state policy
of full public disclosure of all transactions involving public interest. Respondents contend that
what is rendered by board members in reviewing films and in their voting slip is their individual
vote of conscience on the program and thus makes the slip purely private and personal, an
exclusive property of the member concerned.

“Private” means “belonging to or concerning, an individual person, company, or interest”; whereas


“public” means “pertaining to, or belonging to, or affecting a nation, state, or community at large.”
May the decisions of MTRCB members rendered in an official capacity be considered private?
Certainly not. As may be gleaned from PD1986 creating MTRCB, its existence is public in
character. It is created to serve public interest. Thus, respondents have no valid claim to privacy.
The right to privacy belongs to the individual acting in his private capacity and not to a
government agency or officers discharging public duties. There is no invasion of privacy here
since what is sought to be divulged is a product of action undertaken in the course of performing
official functions.

The slips are thus public records access to which is guaranteed to the citizenry. The constitutional
recognition of the citizen’s right of access to official records cannot be made dependent upon the
consent of the board members concerned, else, the right would be rendered nugatory. “If it be
wrong to publish the contents of the records, it is the legislature and not the officials having
custody thereof which is called upon to devise a remedy.”

140. Leo Echegaray v. SOJ, GR 132601, October 12, 1998, Per Curiam (Right to
Information)
FACTS:
Echegaray was convicted by the SC on June 25, 1996 of raping the 10 year-old daughter of his
common-law spouse and was imposed the death penalty. He moved for reconsideration and
assailed for the first time the constitutionality of RA 7659 (death penalty law). The MR was denied.

Meanwhile, RA 8177 was passed, changing the mode of execution of the death penalty from
electrocution to lethal injection. Echegaray filed a petition for prohibition seeking to enjoin the
SOJ and Director of the Bureau of Prisons from carrying out the execution by lethal injection. He
claims that RA 8177 and its IRR by the SOJ is unconstitutional for being cruel and inhuman
punishment, violative of PH’s obligations under international covenants, an undue delegation of
legislative power, and unlawful exercise of SOJ of the power to legislate.

Echegaray’s arguments are that RA 8177 and its IRR are void: 1) cruel, degrading, or inhuman
punishment, 2) violates international treaty obligations, 3) undue delegation of legislative power,
4) discriminatory.

ISSUE:
Whether the manual on the procedure of conducting lethal injection for convicts sentenced to the
death penaltymay be validly made confidential and not available to the public.
HELD:
1. Lethal injection is not cruel, degrading, or inhuman punishment. Punishments are cruel when
they involve torture or a lingering death. But the punishment of death is not cruel within the
meaning of the Constitution. It implies something inhuman and barbarous, more than the mere
extinguishment of life. Any infliction of pain in lethal injection is merely incidental in carrying
out the death penalty. The cruelty whih the constitution protects a convicted man is cruelty inherent
in the method of punishment, not the suffering involved in any method employed to extinguish life
humanely.

2. Reimposition of death penalty does not violate international treaty obligations. Art.6 of the
ICCPR explicitly recognizes that capital punishment is an allowable limitation on the right to life,
subject to the limitation that it be imposed for the “most serious crimes.” State parties are not
obliged to abolish the death penalty totally. They are obliged to limit its use and to abolish it for
other than the “most serious crimes.”

3. There is no undue delegation of legislative power in RA 8177 to the SOJ and director of Bureau
of Corrections. The rule is that what has been delegated cannot be delegated. Potestas delegata
non delegari potest. The exceptions are 1) delegation of tariff powers to the president under S28(2)
of Art.VI, 2) emergency powers to the president under S23(2) of Art.VI, 3) to the people at large,
4) to LGs, and 5) to administrative bodies. Empowering the SOJ win the Secretary of Health and
Director of Bureau of Corrections to promulgates rules on lethal injection is a form of delegation
to legislative authority to administrative bodies. Congress, to delegate to another branch of
government the power to fill in the details in the enforcement of a law, the law must 1) be complete
in itself- setting forth the policy to be executed or implemented, and 2) fix a standard- the limits
of which are sufficiently determinate or determinable. RA 8177 sufficiently describes what job
must be done, who is to do it, and what is the scope of his authority.
It also provides the legislative policy, requiring that the director of the bureau of corrections must
endeavor as far as possible to mitigate the sufferings of the person under the sentence during lethal
injection, etc.

But the SOJ’s IRR, Sec.19, is invalid:


"SEC. 19. EXECUTION PROCEDURE. — Details of the procedure prior to, during and
after administering the lethal injection shall be set forth in a manual to be prepared by the
Director. The manual shall contain details of, among others, the sequence of events before
and after execution; procedures in setting up the intravenous line; the administration of the
lethal drugs; the pronouncement of death; and the removal of the intravenous system.

Said manual shall be confidential and its distribution shall be limited to authorized
prison personnel."
The first paragraph shows that the SOJ has abdicated the power to promulgate the manual on
the execution procedure to the Director by not providing for a mode of review and approval
thereof. Being a mere constituent unit of the DOJ, the Bureau of Corrections could not promulgate
a manual that would not bear the imprimatur of the administrative superior, the SOJ, as the rule-
making authority under RA 8177.

As to the second paragraph, the requirement of confidentiality of the contents of the manual even
with respect to the convict is unduly suppressive. It sees no legal impediment for the convict,
should he desire, to obtain a copy of the manual. The contents of the manual are “matters of public
concern” which the public may want to know either because these directly affect their lives or
simply because such matters naturally arouse the interest of an ordinary citizen. Sec.7, Art.III of
the Constitution provides xxx.

The incorporation in the Constitution of a guarantee of access to information of public concern is


a recognition of the essentiality of the free flow of ideas and information in a democracy. In the
same way that free discussion enables members of society to cope with the exigencies of their
time, access to information of general interest aids the people in democratic decision-making by
giving them a better perspective of the vital issues confronting the nation.

141. Francisco Chavez v. PCGG, GR 130716, December 09, 1998, Panganiban, J. (Right to
Information)
FACTS:
Francisco Chavez, taxpayer citizen and former government official who initiated the prosecution
of the Marcoses, alleges that he saw news reports which impelled him to bring this action. The
news items referred to the 1) alleged discovery of billions of dollars of Marcos assets deposited in
various coded accounts in Swiss banks, and 2) the reported execution of a compromise between
the government thru PCGG and the Marcos heirs on how to split or share these assets. Chavez
invokes his right to information and the duty of the state to disclose publicly all transactions
involving national interest in demanding that PCGG to make public all negotiations and
agreements pertaining to PCGG’s task of recovering the Marcoses’ ill-gotten wealth. PCGG
does not deny forging a compromise agreement, but claims that the petition is premature as Chavez
has not asked PCGG to disclose the negotiations and even if he has, PCGG may not be compelled
any disclosure since the terms and conditions have not yet become effective and binding.
ISSUE:
Whether the right to information and the policy of full public disclosure includes the compromise
agreements crafted by Presidential Commission on Good Governance (PCGG) with the Marcoses
regarding ill-gotten wealth even if such agreements have not yet become operative.
HELD: YES.
1. Petitioner has legal standing since access to public documents and records is a public right, and
the real parties in interest are the people themselves. It is sufficient that petitioner is a citizen
and is thus interested in the execution of the laws. He need not show that he has any legal or special
interest in the result of the action. Petitioner, a former solgen, is a Filipino citizen. The two
requisites to sustain his standing is 1) the enforcement of a public right 2) espoused by a Filipino
citizen.

2. The Court has jurisdiction. The SC has original jurisdiction over petitions for mandamus, among
others. Respondent alleges that the petition should have first been filed with the Sandiganbayan
where the enforcement of the compromise agreements is pending resolution. But this petition is
not confined to the agreements but includes other ongoing or future undertaking towards any
settlement on the alleged Marcos loot. The issue boils down to the precise interpretation, in terms
of scope, of the twin constitutional provisions on “public transactions.”

3. Petitioner invokes S7, Art.III and S28, Art.II. Respondents counter that these provisions refer
to completed and operative official acts and not to those still being considered. The agreements
here have not yet been approved by the president and the Marcos heirs failed to fulfill their
undertaking. ("Sec. 28[Article II]. — Subject to reasonable conditions prescribed by law, the State
adopts and implements a policy of full public disclosure of all its transactions involving public
interest.")

The “information” and “transactions” referred to in the provisions have as yet no defined scope
and extent. There are no specific laws prescribing the exact limits within which the right may be
exercised or the state duty may be obliged. But the following are some of the recognized
restrictions: (1) national security matters and intelligence information, (2) trade secrets(IP Law)
and banking transactions (Secrecy of Bank Deposits Act), (3) criminal matters (law enforcement
matters like apprehension, prosecution, detention of criminals which courts may not inquire into
prior to such arrest.), and (4) other confidential information.

This jurisdiction recognizes the common law holding that there is a governmental privilege against
public disclosure of state secrets regarding military, diplomatic, and other national security
matters. But where there is no need to protect such state secrets, the privilege may not be invoked
provided that the documents and info are examined in strict confidence and given scrupulous
protection. Also, info on inter-government exchanges prior to conclusion of treaties and
executive agreements may be subject to reasonable safeguards for the sake of national interest.

The Ethical Standards Act prohibits public officials from using or divulging confidential info
officially known to them by reason of their office and not made available to the public (other
confidential info). Others include diplomatic correspondence, closed door cabinet meetings,
executive sessions of either house of Congress, and internal deliberations of the SC.
A. Scope: Matters of public concern and transactions involving public interest. (Valmonte: Eludes
exact definition, case by case basis)
RA 6713 mandates public officials to provide info on their policies and procedures in clear and
understandable language and ensure openness of info, public consultations, and hearings whenever
appropriate except when otherwise provided by law or required by public interest. Generally,
writings coming into the hands of public officers in connection with their official functions must
be accessible to the public, consistent with the policy of transparency of governmental affairs.

B. Nature of Marcoses’ ill-gotten wealth.


EO1, promulgated on Feb 28, 1986, only 2 days after the Marcoses fled the country, created the
PCGG primarily tasked to assist the president in recovering the vast government resources
allegedly amassed by Marcos and his family. EO14 gave PCGG more powers, taking into account
the overriding considerations of national interest and national survival. With these
pronouncements of our government whose authority emanates from the people, the recovery of
the ill-gotten wealth is a matter of public concern and imbued with public interest. Ill-gotten
wealth also assumed, by its very nature, a public character. The assets referred to supposedly
originated from the government itself. They belong to the people and upon reconveyance, they
will be returned to the public treasury. This settles the question of whether petitioner has a right to
respondents’ disclosure of any agreement that may be arrived at concerning Marcos’ ill-gotten
wealth.

C. Access to info on Negotiating Terms.


Does the constitution also guarantee access to info regarding ongoing negotiations or proposals
prior to the final agreement? This was clearly addressed by the framers in their deliberations.
Considering the intent of the framers, it is incumbent upon PCGG and its officers and other
government representatives to disclose sufficient public info on any proposed settlement they
decide to take up with the ostensible owners and holders of ill-gotten wealth. Such info,
though, must pertain to definite propositions of the government and not necessarily to intra-
agency or inter-agency recommendations during the stage when common assertions are still in the
process of being formulated or are in the “exploratory” stage. The same restrictions are to be
observed, however, like on matters of national security, etc.

142. Akbayan Citizens Action Party v. Thomas Aquino, GR 170516, July 16, 2008, Carpio-
Morales, J. (Right to Information)
FACTS:
Petitioners non-government organizations, Congresspersons, citizens and taxpayers seek
mandamus to obtain from respondents the full text of the Japan-PH Economic Partnership
Agreement (JPEPA) including the PH and Japanese offers submitted during the negotiation
process.

The HoR issued Res551 calling for an inquiry into the bilateral trade agreements being negotiated
by PH particularly the JPEPA. The House Special Committee on Globalization conducted an
inquiry into the negotiations of the JPEPA. The house committee requested Usec. Thomas Aquino,
chairman of the PH Coordinating Committee created by EO 213 to study and negotiate the
proposed JPEPA, and to furnish the committee with a copy of the latest draft of JPEPA. But
Usec.Aquino did not heed the request. Congressman Aguja later requested also, but Usec. Aquino
replied that he shall provide a copy once the negotiations are completed and as soon as a thorough
legal review of the proposed agreement has been conducted.

The house committee also requested ES Ermita to furnish it with all documents and the latest draft
of JPEPA. ES Ermita replied that it may be difficult to accomplish the request as the agreement
has been a work in progress for 3 years.

Amid speculations that JPEPA might be signed by PH within Dec. 2005, this petition was filed on
Dec.9, 2005. The agreement was signed on September 09, 2006 by PGMA and the Japan prime
minister in Finland. The president endorsed it to the Senate for concurrence. The full text of JPEPA
has been made accessible to the public since September 11, 2006.

ISSUE:
Whether petitioners-citizens and petitioners-congressmen may compel the disclosure of the PH
and JP offers during the negotiations leading up to the Japan-PH Economic Partnership Agreement
on the ground of right to information.
HELD: NO.
1. A petition anchored upon the right to information, which is a public right by its very nature,
petitioners need not show that they have any legal or special interest in the result, it being sufficient
to show that they are citizens and part of the general public who possesses the right.

2. Insofar as the petition seeks the full text of the JPEPA, it has become moot. But it is not moot
with respect to the PH and Japanese offers in the course of negotiations.

3. It is for the courts to determine on a case by case basis whether the matter at issue is of interest
or importance, as it relates to or affects the public. From the nature of JPEPA as an international
trade agreement, it is evident that the PH and JP offers submitted during the negotiations towards
its execution are matters of public concern.

4. Respondents claim executive privilege. Whether a claim of executive privilege is valid depends
on the ground invoked to justify it and the context it is made. here, they claim diplomatic
negotiations then in progress.

The privileged character of diplomatic negotiations has been recognized in this jurisdiction. The
Court declared in PMPF v. Manglapus, where petitioners sought info about on-going negotiations
of the RP-US military bases agreement, that secrecy of negotiations with foreign countries does
not violate free speech, free press, nor freedom of access to information. An essential characteristic
of diplomacy is its confidential nature. A complicated negotiation cannot be carried through
without many private talks and tentative suggestions and proposals.

The president is the sole organ of the nation in its negotiations with foreign countries. He makes
treaties with the consent of the senate, but he alone negotiates, in which field the senate cannot
intrude. He is the sole organ of the nation in its external relations and its sole representative with
foreign nations.
Thus, while the final text of the JPEPA may not be kept perpetually confidential since there should
be “ample opportunity for discussion before a treaty is approved,” the offers exchanged by the
parties during negotiations CONTINUE to be PRIVILEGED even after JPEPA is published.
It is reasonable to conclude that the Japanese representatives submitted their offers with the
understanding that “historic confidentiality” would govern the same. Disclosing these offers
could impair the ability of PH to deal not only with JP but with other foreign governments in future
negotiations.

Those involved in the practice of negotiations agree that publicity leads to grandstanding, tends to
freeze negotiating positions, and inhibits the give-and-take essential to successful negotiation.
Publicity would hamper the ability of our representatives to compromise which may jeopardize
higher national goals for the sake of securing less critical ones.

Thus, diplomatic NEGOTIATIONS are privileged, the JPEPA negotiations included. Such
privilege, however, is only presumptive. Recognizing a type of info as privileged does not mean
that it will be considered privileged in all instances. Only after considering the CONTEXT in
which the claim is made may it be determined if there is public interest that calls for disclosure of
the desired info, strong enough to overcome its traditionally privileged status.

5. Petitioners argue that PMPF v. Manglapus should not apply. The treaty therein involved is the
military bases agreement that pertained to national security whereas JPEPA involves an economic
treaty. But this erroneously assumes that information, to be privileged, must involve national
security. While there are privileges grounded on the need to safeguard national security, not all
are founded thereon. One example is Informer’s privilege, or the privilege of the government
not to disclose the identity of a person or persons who furnish information of violations of law to
officers charged with the enforcement of that law. The suspect need not be so notorious as to be a
threat to national security for this privilege to apply.

There is also privilege accorded to presidential communications, which are presumed


privileged without distinguishing between those involving national security or not. But this is not
absolute. One qualification is that the executive cannot invoke a general confidentiality privilege
to shield its officials and employees from investigations for possible criminal wrongdoing. This
applies whether the privilege is invoked in a judicial trial or congressional investigation.

There is also the deliberative process privilege which covers documents reflecting advisory
opinions, recommendations and deliberations comprising part of a process by which governmental
decisions and policies are formulated. The privileged status of such documents rests not on the
need to protect national security but on the realization that officials will not communicate candidly
among themselves if each remark is a potential item of discovery and front page news.

The diplomatic negotiations privilege has close resemblance to the deliberative process and
presidential communications privilege. The rationale is similar. The privilege for diplomatic
negotiations is meant to encourage a frank exchange of exploratory ideas between negotiating
parties by shielding such negotiations from public view. Like in presidential communications
privilege, diplomatic negotiations privilege seeks to protect the independence in decision-making
of the president in his capacity as the sole organ of the nation in its external relations. As drafts
of regulations are protected by deliberative process privilege, drafts of treaties should be
accorded the same protection. The privilege accorded to diplomatic negotiations follows as a
logical consequence from the privileged character of the deliberative process.

6. Petitioners claim that petitioners in PMPF were only mass media members while here,
petitioners include HoR members. But the privileged character of diplomatic negotiations does not
ipso facto lose all force and effect simply because the privilege is being claimed under different
circumstances. Presidential refusals to furnish info may be actuated by at least 3 distinct kinds of
considerations (state secrets, informer’s, generic privilege for internal deliberations) which may
be asserted with differing degrees of success in the context of either judicial or legislative
investigations. A privilege, once recognized, may be invoked under different procedural settings.
Thus, the privilege of diplomatic negotiations may be invoked not only against citizens’ demands
but also in the context of legislative investigations. Hence, PMPF cannot be considered irrelevant.

7. It is argued that the privilege applies to certain stages of the negotiation process only. When the
negotiations moved from the formulation and exploratory stage to the firming up of definite
propositions, there is duty to disclose. But while the right to info includes on-going negotiations
before a final contract, the info must constitute definite propositions by the government and
should not cover recognized exceptions like privileged info, etc. Even definite propositions may
not be disclosed if they fall under “recognized exceptions,” like diplomatic negotiations.

8. Is there sufficient public interest to overcome the claim of privilege?


There are at least 2 kinds of public interest that must be taken into account.: 1) presumed public
interest in favor of keeping the subject info confidential which is the reason for the privilege in
the first place, and 2) public interest in favor of disclosure, the existence of which must be shown
by the party asking for info. The criteria to determine if there is sufficient public interest for
disclosure may be gleaned from US cases. The US courts, in ruling against pres. Nixon’s claim of
privilege against a subpoena duces tecum of a court in a criminal proceeding, said that the claim
of privilege must be balanced against the constitutional duty of courts to ensure a fair
administration of criminal justice. When Nixon invoked the claim against a subpoena of a senate
committee, the US court spoke of the need to balance the claim with the duty of Congress to
perform its legislative functions. In In re sealed case involving a claim of deliberative process and
presidential communications privileges against a subpoena of a grand jury, the US court said that
the deliberative process privilege is a qualified privilege and can be overcome by a sufficient
showing of need. This need determination is made flexibly on a case by case ad hoc basis. Factors
to take into account include the relevance of the evidence, the availability of other evidence, the
seriousness of the litigation, the role of the government, and the possibility of future timidity
by government employees.

Here, petitioners, both petitioners-citizens and members of HoR, failed to present SUFFICIENT
SHOWING OF NEED referred to in the above US cases. Their arguments fall short of this
standard.

Petitioners claim right to participate in the JPEPA negotiation and the non-involvement of the
Filipinos therein. But various non-government sectors and private citizens have already publicly
expressed their views on JPEPA since this was already published to the public. Online articles
have been published criticizing JPEPA. Thus, there is no basis for petitioners’ claim that access to
the JP and PH offers is essential to the exercise of their right to participate in decision-making.

Petitioner members of HoR anchor their claim to Congress’ inherent power to regulate commerce.
They argue that the president cannot exclude Congress from the JPEPA negotiations. But by
constitutional fiat and by the intrinsic nature of his office, the president, as head of state, is the sole
organ and authority in the country’s external affairs. In the realm of treaty-making, the president
has sole authority to negotiate with other states. Into the field of negotiation, the senate cannot
intrude. Petitioners HoR members fail to show that the info sought is critical to the performance
of the functions of congress, functions which do not include treaty-negotiation.

The Court observes that the claim of privilege appearing in respondents' Comment to this petition
fails to satisfy in full the requirement laid down in Senate v. Ermita that the claim should be
invoked by the President or through the Executive Secretary "by order of the President".
Respondents' claim of privilege is being sustained, however, its flaw notwithstanding, because of
circumstances peculiar to the case. The decision in Senate v. Ermita was not yet final and executory
at the time respondents filed their comment to this petition. Thus, a strict application of this
requirement is unwarranted.

In executive privilege controversies, the requirement that parties present a “sufficient showing of
need” only means, in substance, that they should show a public interest in favor of disclosure
sufficient in degree to overcome the claim of privilege. The Court in such cases engaged in a
BALANCING of INTERESTS. The executive privilege is a limitation on the right to
information. A party cannot overcome this by merely asserting that the info demanded is a matter
of public concern without any further showing.

In determining if an information is covered by the right to information, a specific “showing of


need” for such information is NOT a relevant consideration, but only whether it is a matter of
PUBLIC CONCERN. BUT when the government claims executive privilege and it has
established that the info is indeed covered by the same, then the party demanding it, if it is to
overcome the privilege, must show that the info is vital not simply for satisfaction of curiosity,
but for its ability to effectively participate in social, political, and economic decision-making.
(**dito papasok si sufficient showing of need)

143. In re: Production of Court Records and Documents and the Attendance of Court
Officials and Employees as Witnesses under the Subpoenas of February 10, 2012 and the
Various Letters for the Impeachment Prosecution Panel Dated January 19 and 25, 2012,
February 14, 2012, Per Curiam.
FACTS: (Corona impeachment)
During the impeachment proceedings against CJ Corona, the prosecution panel manifested in a
compliance that it would present about 100 witneses and almost a thousand documents. The
proposed witnesses included SC justices and court employees who will testify on matters many of
which are internal to the Court.
Around this time, these letters were sent. Before us are the letters of Hon. Joseph Abaya,
congressman and impeachment prosecution panel manager, in behalf of the house impeachment
panel, requesting to be allowed
1. to examine the rollo of Flight attendants and stewards association of the PH (FASAP) v. PAL
and for certified true copies of the minutes and agenda of the deliberations of FASAP v. PAL.
2. to examine the rollo of Navarro v. Ermita, Merceditas Gutierrez v. HoR Committee on Justice,
and League of Cities(LCP) v. COMELEC.

The impeachment court then directed the attendance of witnesses clerk of court Vidal and Deputy
clerk Anama and the production of documents per the subpoena ad testificandum et duces tecum
in the case of FASAP v. PAL: the logbook of the raffle committee showing the assignment of the
FASAP case, etc.

Another subpoena ad testificandum directs Vidal in the case of foremer president GMA and Jose
Arroyo to bring with her for submission to the impeachment court various documents related to
the case.

The letters asked for the examination of records and issuance of certified copies of the rollos and
agenda and minutes of the deliberations. FASAP case (pending presently)- examination of the
rollo and agenda and minutes of the case. Navarro (pending)- rollo. Merceditas Gutierrez
(terminated case)- rollo. LCP- rollo. Atty. Vidal brought to our attention the subpoenas she
received commanding her to appear with the certified copies of these documents and testify.

ISSUE:
Whether the SC and its employees may be compelled to present to the Senate impeachment court
rollos of pending and terminated cases, minutes of its deliberations in cases, and the results of
raffle assignment to justices of certain cases.
HELD:
1. Each branch of government, under separation of powers, is considered co-equal, coordinate, and
supreme within its own sphere. The Court’s mandate is to keep the different branches within the
exercise of their assigned powers through the rule of law. The principle of comity is the practice
of voluntarily observing inter-departmental courtesy in undertaking each department’s assigned
constitutional duties for the harmonious working of government.

2. Underlying every request for info is the right to info under Sec.7, Art.III. This right is not
absolute by its very nature and the constitution’s own terms. On the part of private individuals, the
right to privacy also exists. Institutions enjoy their own right to confidentiality that, for
governmental departments and agencies, is expressed in terms of their need to protect the
integrity of their mandated tasks under the Constitution and laws.

In line with the public’s right to info, the Court has adopted a policy of transparency with respect
to documents in its custody. This is embodied in S11, Rule 136 of the RoC. The rule grants access
to court records to any person subject to fees and compliance with rules. But it is subject to the
limitations the laws and the Court’s own rules provide. A basic underlying limitation is the need
to preserve the integrity of their main adjudicative function.
The Internal Rules of the SC (IRSC) prohibits disclosure of (1) the result of the raffle of cases ,
(2) the actions taken by the Court on each case included in the agenda of the Court's session,
and (3) the deliberations of the Members in court sessions on cases and matters pending before
it. Results of raffle of cases is available only to the parties and their counsels under Rule 7 S3 of
IRSC, unless the cases involve bar matters, administrative cases, and criminal cases involving life
imprisonment, where the raffle results are not disclosed even to the parties themselves.

Only after the official release of the resolution embodying the Court action may that action be
made available to the public. A resolution is considered officially released once the envelope
containing its final copy addressed to the parties has been transmitted to the process server for
personal service or to the mailing section of the Judicial Records Office.

Court deliberations are traditionally recognized as privileged communication. This will enable
members of the Court to freely discuss the issues without fear of criticism for holding unpopular
positions or fear of humiliation for one’s comments. This is known as deliberative process
privilege involving the deliberative process of reaching a decision. The privilege is intended to
prevent the “chilling” of deliberative communications. All the branches enjoy this privilege. While
Rule 10, S2 of IRSC speaks only of confidentiality of court deliberations, it extends to documents
and other communications which are part of or are related to the deliberative process. This
privilege applies to court employees when asked to act on these documents and other
communications.

A senator may invoke legislative privilege when asked outside the senate for info gathered during
an executive session of inquiry in aid of legislation. A justice of the court or judge may invoke
judicial privilege in the senate sitting as impeachment court for proceedings in the performance
of his own judicial functions.

The Code of Conduct for Court Personnel also provides that access shall be denied with respect to
info or records relating to drafts of decisions, rulings, orders, or internal memoranda or internal
reports.

To qualify under the deliberative process privilege, the agency must show that the document is
both 1) PREDECISIONAL and 2) DELIBERATIVE. A document is “predecisional” if it
precedes, in temporal sequence, the decision to which it relates. Communications are predecisional
if made in the attempt to reach a final conclusion. A material is “deliberative” if it reflects the
give-and-take of the consultative process. The key question is whether the disclosure of the info
would discourage candid discussion within the agency. If it does, it is privileged. Thus, court
records that are predecisional and deliberative cannot be the subject of subpoena if judicial
privilege is to be preserved.

Two more grounds may be cited to deny access to court records and to prevent members of the
bench from being subjected to compulsory process: 1) disqualification by reason of privileged
communication and 2) the pendency of an action or matter (sub judice rule).

The rule of judicial ethics complements the rule of evidence that disqualifies public officials from
testifying on info they acquire in confidence in the course of their duties. Improper disclosure of
confidential info learned in official capacity is made criminally punishable under Art.229 of the
RPC, Sec.3(k) of RA 3019, and Sec.7, RA 6713. The impeachment court or senate does not have
the power to grant immunity from criminal prosecution for revealing confidential info.

Under the law thus, the members of the Court may not be compelled to testify in the
impeachment proceedings againt the CJ or other members about info they acquire in the
performance of their official function of adjudication, like info on how deliberations were
conducted or the material inputs that the justices used in decision-making, as disclosure might
subject them to criminal prosecution. Such act violates judicial privilege. Court members are
exempt from Congress’ power of inquiry on the basis not only of separation of powers but also on
fiscal autonomy and constitutional independence of the judiciary. Also, interdepartmental
courtesy (principle of comity) demands that the highest levels of each department be exempt from
compulsory process of other departments on matters related to the functions of their office. The
same rules on confidentiality apply to Court officials and employees.

These privileges belong to the judiciary and are for the Supreme Court and not for the
individual justice, judge, or court employee to waive. Every proposed waiver must be referred
to the SC for consideration and approval.

Differently stated, SC justices cannot be compelled to testify on matters relating to the internal
deliberations and actions of the Court in the exercise of their adjudicatory functions. This is
differentiated from where the testimony is on a matter external to such functions. (When the
impeachment ground is, say, bribery, a justice may be called to be a witness in the impeachment
of another as bribery is external and not connected with adjudicatory functions. But arguments of
the accused justice on internal debates are internal and are details of the deliberative process.)

Also, witnesses need not be summoned to testify on matters of public records. Entries in public or
official books may be proved by the production of the books or records themselves or by a copy
certified by the legal keeper thereof. These records may only be presented if not excluded by
reasons of privilege. A public officer is excused from appearing in court so that public business
may not be interrupted.

To summarize, the following are privileged:


(1) Court actions such as the result of the raffle of cases and the actions taken by the Court on each
case included in the agenda of the Court's session on acts done material to pending cases, except
where a party litigant requests information on the result of the raffle of the case, pursuant to Rule
7, Section 3 of the IRSC; (2) Court deliberations or the deliberations of the Members in court
sessions on cases and matters pending before the Court; (3) Court records which are
"predecisional" and "deliberative" in nature, in particular, documents and other communications
which are part of or related to the deliberative process, i.e., notes, drafts, research papers, internal
discussions, internal memoranda, records of internal deliberations, and similar papers. (4)
Confidential Information secured by justices, judges, court officials and employees in the course
of their official functions, mentioned in (2) and (3) above, are privileged even after their term of
office. (5) Records of cases that are still pending for decision are privileged materials that cannot
be disclosed, except only for pleadings, orders and resolutions that have been made available by
the court to the general public. (6) The principle of comity or inter-departmental courtesy demands
that the highest officials of each department be exempt from the compulsory processes of the other
departments. (7) These privileges belong to the Supreme Court as an institution, not to any justice
or judge in his or her individual capacity. Since the Court is higher than the individual justices or
judges, no sitting or retired justice or judge, not even the Chief Justice, may claim exception
without the consent of the Court.

RIGHT TO FORM ASSOCIATIONS


144. SSS Employees Association (SSSEA) v. CA, GR 85279, July 28, 1989, Cortes, J. (Right
to Form Associations)
FACTS:
SSS filed with the RTC a complaint for damages against petitioners SSSEA alleging that its
officers and members staged an illegal strike and barricaded the entrances to the SSS Building,
preventing non-striking employees from reporting for work and SSS members from transacting
business with SSS. The Public Sector Labor-Management Council ordered the strikers to return to
work, but they refused. SSS suffered damages. SSS also prayed that injunction be issued to enjoin
the strike and that the strikers be ordered to return to work and the strike be declared illegal.

SSSEA went on strike when SSS failed to act on the union’s demands, which included
implementation of the old SSS-SSSEA CBA on check-off of union dues; payment of accrued
overtime pay, night differential and holiday pay; conversion of contractual employees with 6
months or more of service into regular employees; allowances and benefits etc.

The RTC issued a TRO which was converted to an injunction, finding the strike illegal. The CA
dismissed SSSEA’s certiorari petition. Hence this petition for review.

SSSEA claims that the RTC has no jurisdiction as it is the DOLE or NLRC that has jurisdiction
since the case involves a labor dispute. SSS argues that the employees of SSS are covered by civil
service laws and rules, not the Labor Code and thus have no right to strike.

ISSUE:
Do the employees of the Social Security System have the right to strike despite being government
employees?
HELD: NO.
Art.XIII, S3 of the Constitution guarantees the rights of all workers to self-organization,
collective bargaining and negotiations, and peaceful concerted activities, including the right
to strike in accordance with law. While this provision seems to provide all employees including
public employees the right to strike, the constitution fails to expressly confirm this impression. In
the sub-article on the CSC, after defining the scope of the civil service as all branches, etc. of the
government, that “the right to self-organization shall not be denied to government employees.”
(Art.XI(B), Sec.2(1) and (5)) The BoR also provides that "[t]he right of the people, including those
employed in the public and private sectors, to form unions, associations, or societies for purposes
not contrary to law shall not abridged.” Thus, the constitution, while recognizing the right of
government employees to organize, is silent as to whether such recognition includes the right
to strike.
It appears that the framers in their proceedings intended to limit the right of government
employees to organize or form unions or associations only without including the right to
strike. (“xxx it does not mean that because they have the right to organize, they also have the right
to strike. That is a different matter.” “the moment we allow anybody in the government to strike,
then what will happen if the members of the Armed Forces will go on strike? What will happen to
those people trying to protect us?”)

CA 875, repealed by the Labor Code, expressly banned strikes by employees in the government,
including instrumentalities exercising governmental functions, but excluding entities entrusted
with proprietary functions. No similar provision is found in the Labor Code. The LC is silent as to
whether government employees may strike, for such are excluded from its coverage. The Civil
Service Decree (PD807) is equally silent on the matter.

On June 1, 1987, the President EO 180 which stated that the Civil Service law and rules governing
concerted activities and strikes in government service shall be observed subject to legislation that
may be enacted by Congress. She was referring to Memo circ. 6 s.1987 of the CSC which says
that before the enactment of applicable laws concerning strike by government employees…enjoins
under pain of administrative sanctions all government employees “from staging strikes,
demonstrations etc. and other forms of mass action which will result in temporary stoppage or
disruption of public service.” Thus, at present, without legislation allowing government
employees to strike and recognizing their right to do so, they are prohibited from striking by
express provision of Memo Circ. 6 (The validity of Memo Circ 6 is not at issue.)

SSS employees are covered by the prohibition against strikes. The civil service “embraces all
branches, subdivisions, instrumentalities, and agencies of the government, including GOCCs with
original charters” (Art.IX(B), S2(1)). SSS is a GOCC with original charter, having been created
by RA 1161, its employees are thus part of the civil service. They are covered by the CSC’s
memorandum prohibiting strikes. The SSSEA strikes are thus illegal.

The terms and conditions of employment in the government are governed by law. Government
workers thus cannot use the same weapons employed by workers in the private sector to secure
concessions from their employers. Relations between private employers and employees rest on a
voluntary basis and collective bargaining subject to minimum requirements of labor legislation. In
government employment, it is the legislature and to those administrative heads of government to
whom it delegates power that fix the terms of employment. By reason of the nature of the public
employer and the peculiar character of public service, the right to strike of unions in private
industry does not apply to public employees.

Government employees may, thru their unions or associations, either petition congress for better
terms of employment or negotiate with government agencies for improvement of those terms not
fixed by law. But they cannot resort to strikes and other temporary work stoppages.

145. In the matter of the IBP membership dues delinquency of Atty. Marcial Edillon, AC
1928, August 3, 1978, Castro, C.J. (Right to Form Associations)
FACTS:
The IBP board of governors adopted Res.75-65 ad an administrative case recommending the
removal of Atty. Edillon’s name from its roll of attorneys for stubborn refusal to pay his
membership dues to the IBP since its constitution notwithstanding due notice.

The all-encompassing scope of membership in the IBP is stated in Sec.1 of the Court Rule, creating
the IBP “composed of all persons whose names now appear or may hearafter be included in the
Roll of Attorneys of the SC.” Sec.9 obligates members to pay annual dues as the board of
governors shall determine with the SC’s approval. Sec.10 provides that non-payment of the annual
dues for 6 months shall warrant suspension of membership in the IBP, and default of payment for
1 year shall be ground for removal of the name from the roll of attorneys.
ISSUE:
Whether integrating the PH bar and making it mandatory for all lawyers to be members thereof
violates their rights to association.
HELD: NO.
Atty. Edillon argues that these provision (S9, S1, S10, Court Rule) are invasions of his
constitutional rights in that he is being compelled, as a pre-condition to maintaining his status as a
lawyer in good standing, to be a member of the IBP and to pay corresponding dues. Thus, he is
being deprived of the rights to liberty and property. Hence, the provisions are void.

An “Integrated Bar” is a State-organized Bar, to which every lawyer must belong, as distinguished
from bar associations organized by individual lawyers themselves, membership in which is
voluntary.an integrated bar is an official national body of which all lawyers are required to be
members. They are thus subject to all the rules prescribed for the governance of the bar, including
payment of reasonable annual fee for the effective discharge of the purposes of the bar. The
integration of the PH Bar was dictated by overriding considerations of public interest and
public welfare.

All legislation directing integration of the bar have been universally sustained as a valid exercise
of the police power over an important profession. The practice of law is not a vested right but a
privilege clothed with public interest. When Congress enacted RA 6397 authorizing the SC to
adopt rules of court to effect the integration of the PH bar under such conditions as it shall see fit,
it did so in the exercise of police power. The purpose is to raise the standards of the legal
profession and improve the administration of justice, etc. Even without RA6397, the constitution
vests the SC the power to promulgate rules concerning pleading, practice and procedure in all
courts and the admission to the practice of law. The SC thus has plenary power in all cases
regarding admission to and supervision of the practice of law.

Thus, when Edillon entered the legal profession, his practice of law is subject to the power of the
body politic to require him to conform to such regulations as might be established for the common
good even to the extent of interfering with some of his liberties.
1. He argues that the SC has no power to compel him to be a member of the IBP. But to compel a
lawyer to be a member of the IBP is not violative of his constitutional freedom to associate.

Integration does not make a lawyer a member of any group of which he is not already a
member. He became a member of the bar when he passed the bar exams. All that integration does
is to provide an official national organization for the well-defined but incohesive group of which
every lawyer is ALREADY A MEMBER.

Bar integration does not compel the lawyer to associate with anyone. He is free to attend or not
to attend IBP meetings or vote in its elections or not. The only compulsion is the payment of
annual dues. The SC, to further the legitimate interest of the state to elevate the quality of
professional legal services, may require that the cost of improving the profession be shared by the
beneficiaries of the regulatory program- the lawyers. But even if in a sense the provision does
compel a lawyer to be a member of the IBP, such compulsion is justified as an EXERCISE OF
POLICE POWER of the state.

2. Nothing in the constitution prohibits the Court under its power to promulgate rules from
requiring members of a privileged class like lawyers to pay a reasonable fee to defray expenses of
regulation of the profession to which they belong. The fee is imposed as a regulatory measure to
carry out the purpose of integration.

3. The right to practice law before the courts of this country is a matter subject to regulation and
inquiry. The power to impose the fee as a regulatory measure is recognized. Thus, a penalty
designed to enforce payment, which may be avoided by payment, it not void as unreasonable. But
practice of law is not a property right but a mere privilege and must bow to the inherent
regulatory power of the Court.

146. Benjamin Victoriano v. Elizalde Rope Worker’s Union, GR L-25246, September 12,
1974, Zaldivar, J. (Right to Form Associations)
FACTS:
Victoriano, member of INC, was employed in Elizalde Rope Factory, Inc. (Company) since 1958.
He was a member of Elizalde Rope Workers’ Union (Union) which had a CBA containing a closed
shop provision, requiring as a condition of employment for all permanent employees
membership in the union. RA 875, prior to amendment by RA 3350, allowed the employer to
make an agreement with a labor organization to require as condition of employment membership
therein if the labor organization represents the employees. RA 3350 amended this, adding “but
such agreement shall not cover members of any religious sects which prohibit affiliation of
their members in any such labor organization.

Being a member of ICN, religious sect, that prohibits affiliation of its members with any labor org,
Victoriano resigned from the Union and when no action was taken thereon, he reiterated his
resignation. The union wrote the company asking it to separate Victoriano from service since he
was resigning from the Union. Victoriano was informed by the Company that should he fail to
achieve a satisfactory arrangement with the Union, the company would dismiss him. Thus,
Victoriano filed an action for injunction with the CFI to enjoin the company and union from
dismissing him. The union assailed the constitutionality of RA 3350.

The CFI enjoined the Company from dismissing Victoriano and ordered the union to pay
attorney’s fees to Victoriano. The union appealed to this Court. The Union argues that RA 3350,
banning all those belonging to a religious sect prohibiting affiliation with any labor organization
from joining any labor union, deprives members of the sect of their right to form or join lawful
associations. They claim that RA 3350 impairs the obligation of contracts, is discriminatory in
favor of sects which ban members from unions, and violates “no religious test shall be required
for the exercise of a civil right”.

ISSUE:
Whether a law exempting employees of religious sects that prohibit affiliation with labor unions
from the coverage of union security clauses in the CBA is unconstitutional.
HELD: NO.
1. The contention that RA 3350 prohibits and bans members of such religious sects that forbid
affiliation with unions appears nowhere in the wording of RA 3350. The Constitution guarantees
the right to form or join associations. A “right” comprehends at least two broad notions: 1) liberty
or freedom i.e. absence of legal restraint, and 2) power, whereby an employee may, as he pleases,
join or REFRAIN from joining an association. It is thus for the employee to decide for himself
whether he should join or not an association and should he choose to join, he himself makes up his
mind as to which association to join. Even after he joins, he retains the liberty and power to leave
and cancel his membership at any time. The right to join a union includes the right to abstain
from joining any union. As what the Constitution (and the Industrial Peace Act) have recognized
is the “right” to join associations of his choice, it is absurd to say that the law also imposes upon
the employee the duty to join associations.

The right to refrain from joining is limited by the Industrial Peace Act in that it allows an employer
to agree with the union on a closed shop agreement where the employer may employ only members
of the union and employees must continue membership therein to be employed. To this all-
embracing coverage of the closed shop arrangement RA 3350 introduced an exception excepting
members of sects which prohibit affiliation of their members in any labor organization. It excludes
ipso jure from the coverage of the closed shop the employees belonging to such sects. It provides
that members of said sects cannot be COMPELLED or coerced to join labor unions even if
there is a closed shop agreement with employers. In spite of such close ship arrangement,
members of said sects cannot be refused employment or dismissed on the sole ground that
they are not members of the union. Thus, RA 3350 actually upholds and reinforces freedom
of association. It leaves to members the liberty and power to affiliate or not to affiliate. They can
refuse to join in deference to their faith or join notwithstanding such faith.

2. It canoe be denied that there was an impairment of the union security clause since the agreement
was made before enactment of RA 3350 and its provisions are thus not incorporated therein. The
law thus introduced a change into the terms of the clause and the Company was partly absolved
by law from its contractual obligation with the Union of employing only Union members in
permanent positions.

According to Black, any statute that introduces a change into the express terms of the contract, its
legal construction, its validity, its discharge, or the remedy for its enforcement, impairs the
contract. The extent of the change is immaterial. It is not a question of degree or manner or
cause, but on encroaching in any respect on its obligation or dispensing with any part of its
force.
But the prohibition to impair the obligation of contracts is not absolute. The prohibition is general
and not to be read with lieteral exactness like a mathematical formula for it prohibits
UNREASONABLE impairment only. The state still possesses authority to safeguard the vital
interests of its people. Laws appropriate to safeguard said interests may modify or abrogate
contracts already in effect. For not only existing laws read into contracts, but the reservation of
essential attributes of sovereign power is also read into contracts as a postulate of the legal
order. All contracts made with reference to any matter subject to regulation under police
power must be understood as made in reference to the possible exercise of that power.
Otherwise, valuable reforms may be precluded by simply entering into contracts.

The policy of protecting contracts against impairment presupposes the maintenance of a


government by virtue of which contractual relations are worthwhile- a government which retains
adequate authority to secure the peace and order of society. Thus, the contract clause of the
constitution must not only in harmony with, but also in subordination to, the reserved power of
the state to safeguard the vital interests of the people. Thus, not all legislations that impair a
contract violate the prohibition. A statute passed in the legitimate exercise of police power,
although it incidentally destroys existing contract rights, must be upheld by the courts. This
has special application to contracts regulating relations between capital and labor which are not
merely contractual, and said labor contracts, for being impressed with public interest, must yield
to the common good.

In determining whether legislation unconstitutionally impairs contract obligations, no unchanging


yardstick is applicable at all times and under all circumstances, but every case must be determined
upon its own circumstances. Legislation impairing the obligation of contracts can be sustained
when it is enacted to promote the general good of the people, and when the means adopted are
reasonable. The purpose of RA 3350 is to insure freedom of belief and religion and to promote
general welfare by preventing discrimination against religious sects which prohibit their
members from joining labor unions. The purpose is legitimate. It also protects said members
against two aggregates of group strength- collective labor (union) and collective capital
(management). The union is paradoxically both the champion of employee rights and a new source
of their frustration. When the union interacts with management, it produces a third aggregate of
group strength- the collective bargaining relationship.

The means adopted, exempting members of the sects from the coverage of union security
agreements, is reasonable. Also, the free exercise of religious profession or belief is superior to
contract rights. Religious freedom has a preferred position in the hierarchy of values.

3. It is also contended that RA 3350 discriminates in favor of members of the sects, violating the
non-establishment clause of the constitution. But if the state regulates conduct by enacting a
general law within its power which has for its purpose to advance secular goals, the statute is valid
despite its indirect burden on religious observance unless the state can accomplish the purpose
without imposing the burden. The statute must also have a secular legislative purpose and a
primary effect that neither advances nor inhibits religion. RA 3350 thus does not violate the
“no-establishment” of religion clause.
The purpose of RA 3350 is secular not spiritual or religious. It was intended to advance free
exercise of religion. To help citizens find gainful employment whereby they can make a living to
support themselves and their families is a valid objective of the state. The primary effect of the
exemption from closed shop agreements in favor of sect members is the protection of said
employees against the aggregate force of the CBA and relieving certain citizens of a burden on
their religious beliefs. The exemption does not directly advance or diminish the interests of any
particular religion. The benefit upon the religious sects is merely incidental and indirect. The
establishment clause does not ban regulation of conduct whose effect merely happens to coincide
or harmonize with the tenets of some or all religions. There is no compelling state interest to
withhold exemption.

4. Joining or withdrawing from a labor union requires a positive act. RA 3350 only exempts the
sect members from coverage of closed shop agreements. A religious objector is not required to do
a positive act- to exercise the right to join or to resign from the union. He is exempted without
need of a positive act. How then can there be a religious test for the exercise of a right when no
right need be exercised?

5. RA 3350 satisfies the condition for reasonable classification. It classifies employees, as to the
coverage of union security agreements, into those who by reason of their religious beliefs cannot
join a union and those whose religion does not prohibit membership in a union. The classification
is substantial and not merely imaginary. There is real distinction in the beliefs and sentiments of
employees. Religious beliefs are not mere belifs or ideas existing only in the mind, for they carry
with them practical conseuences.

The classification is germane to its purpose to avoid those who cannot join labor unions because
of religious belief from being deprived of their right to work and from being dismissed from work
because of union shop security agreements. RA 3350 is not limited to conditions existing at the
time of enactment as it is intended to apply for all times as long as the conditions to which the law
is applicable exist. As long as there are closed shop agreements and employees prohibited by their
religion from affiliating with unions, their exemption continues.

The law applies equally to all members of said religious sects evident from its provisions. The fact
that the law grants a privilege to members of said sects cannot by itself render the act
unconstitutional for the act only restores to them their freedom of association which closed
shop agreements have taken away and puts them in the same plane as other workers who are not
prohibited by their religion from joining labor unions.

Sec. 10- NON-IMPAIRMENT CLAUSE


147. Ortigas & Co., Limited Partnership v. Feati Bank & Trust Co., GR L-24670, December
14, 1979, Santos, J. (Non-Impairment Clause)
FACTS:
Plaintiff is engaged in real estate business, developing and selling lots to the public along EDSA
in Highway Hills Subdivision, Mandaluyong. Plaintiff, as vendor, and Padilla and Angeles, as
vendees, entered into separate agreements on sale on installments over two lands (lots 5 & 6) of
said subdivision. Vendees transferred their rights and interests over the lots to one Chavez. Upon
completion of payment of purchase price, plantiff executed deeds of sale in favor of Chavez. The
agreements of sale on installment and the deeds of sale contained the stipulations that:
"1. The parcel of land subject of this deed of sale shall be used by the Buyer exclusively
for residential purposes, and she shall not be entitled to take or remove soil, stones or
gravel from it or any other lots belonging to the Seller.

The restrictions were annotated in the titles issued to Chavez. Eventually, defendant Feati acquired
the lots, and the building restrictions were also annotated to its title. The lands were purchased
“free from all liens and encumbrances.” Plaintiff claims that the restrictions were imposed as part
of its general building scheme designed for the beautification and development of the Highway
Hills Subdivision.

Feati on the other hand claims that the area along the western part of EDSA has been declared a
commercial and industrial zone by Res.27 of the Municipal Council of Mandaluyong. Feati
began the construction of a building for banking purposes which Feati claims could also be
devoted for residential purposes. Plaintiff demanded in writing that the construction of the
commercial building be stopped. Feati refused to comply, contending that the building was being
constructed in accordance with the zoning regulations, Feati having filed building and planning
permit applications with Mandaluyong which was granted to proceed with the construction.

The trial court ruled for Feati, holding that the restrictions were subordinate to Res.27, predicating
its ruling on police power of the municipality. It upheld the classification of the EDSA area as
commercial and industrial zone and this rendered ineffective the restrictions. Hence this appeal to
this Court on questions of law.

Plaintiff argues that the municipal council did not have the power to nullify the contractual
obligations assumed by Feati.

ISSUE:
Whether lands along EDSA,which supports an endless stream of traffic, sold to Feati with the
restrictions in the contract stating that it may only be used for residential purposes may prevail and
be enforced as against Resolution 27 of the Municipality of Mandaluyong declaring said location
near EDSA to be a commercial and industrial zone.
HELD: NO.
While non-impairment of contracts is constitutionally guaranteed, it is not absolute since it has to
be reconciled with the legitimate exercise of police power. Res.27 was passed by the municipal
council in the exercise of its police power. Judicial notice may be taken of the conditions prevailing
in the area especially of lots 5 and 6. The lots not only front the highway, industrial and commercial
complexes have flourished about the place. EDSA supports an endless stream of traffic and the
resulting activity, noise, and pollution are hardly conducive to the health, safety, or welfare of the
residents in its route. Having been expressly granted the power to adopt zoning and subdivision
ordinances or regulations, the municipality of Mandaluyong, thru its council, was reasonably
and perfectly justified under the circumstances in passing the resolution. The state, to promote
general welfare, may interfere with personal liberty, property, business and occupations.
Since the motives behind the passage of Res.27 are reasonable, and it being a legitimate response
to a felt public need, not whimsical or oppressive, the non-impairment of contracts clause of the
constitution will not bar the municipality’s proper exercise of the power. Laws and reservation
of essential attributes of sovereign power are read into contracts agreed upon by the parties.
Since it would be a hazard to the health and comfort to use lots 5 and 6 for strictly residential
purposes, Feati should be permitted, on the strength of Res.27 promulgated under the police power
of the municipality, to use the same for commercial purposes.

Hence, the contractual obligations assumed cannot prevail over Res.27 of the Municipality of
Mandaluyong, which was validly exercised under police power. The restrictions cannot be
enforced.

Sec.11- FREE ACCESS and ADEQUATE


LEGAL ASSISTANCE
148.Sps. Algura v. City of Naga, GR 150135, October 30, 2006, Velasco, JR., J. (Free Access
and Adequate Legal Assistance)
FACTS:
Spouses Antonio and Lorencita Algura filed a complaint for damages against Naga City and its
officers from the alleged illegal demolition of their residence and boarding house and for payment
of lost income derived from fees paid by their boarders of P7k monthly. They also filed an ex-
parte motion to litigate as indigent litigants, to which Antonio’s payslip was attached, showing
a monthly income of P10474 and a net pay of P3616.99 for July 1999. There was also a
certification by the Office of the City Assessor of Naga stating that petitioners had no property
declared in their name for tax purposes.

Judge Atienza of the RTC granted petitioners’ plea for exemption from filing fees. Respondents
argued that the boarding house blocked the road right of way and the structure was a nuisance per
se. Respondents also moved to disqualify petitioners for non-payment of filing fees. They claim
that in addition to the more than P3k net income of Antonio, member of PNP, Lorencita also had
a mini-store and a computer shop on the ground floor of their residence. The second floor of their
residence was also a boarding house from which they earned more than P3k/mo. They had
additional income from their computer shop patronized by students and from several boarders who
paid rentals. Thus, petitioners, respondents aver, were not indigent litigants.

The RTC disqualified petitioners as indigent litigants on the ground that they failed to substantiate
their claim for exemption from payment of legal fees and to comply with payment of requisite
filing fees. Petitioners were given an opportunity to show that they were indigent litigants. They
claim that they did not own real property as certified and that the rentals from some boarders, her
husband’s salary, and the income from the small sari-sari store were not enough to pay for the
family’s basic necessities and 6 children. But still, the RTC denied reconsideration of the
disqualification.
ISSUE:
Whether spouses Algura, litigants, without real property as certified by the Office of the City
Assessor and whose gross monthly income total only P13k more or less may be considered
indigent litigants under the Rules of Court.
HELD: It depends on the discretion of the court under Rule 3, S21.
When the RoC took effect on January 01, 1964, the rule on pauper litigants was found in Rule 3,
S22, which allowed any court to authorize a litigant to litigate as pauper upon proper showing that
he has no means to that effect by affidavits, certificate of the city etc. treasurer. Rule 141 on legal
fees did not contain any provision on pauper litigants.

(*JANUARY 01, 1964- RULE3, S22- pauper litigant; RULE 141- NO provision on pauper
litigant)

In AM83-6-389-0, the Court approved the recommendation of the Committee on the Revision of
Rates and Charges of Court Fees to revise the fees in Rule 141 of the RoC. A provision on pauper
litigants was inserted. “Pauper litigants” included, in S16, wage earners whose gross income does
not exceed P2k/mo for those in Metro Manila, P1.5k/mo outside Manila, or those who do not own
real property with an assessed value of not more than P24k or P18k. The litigant shall execute an
affidavit that he does not earn the mentioned income. He will be exempt from payment of filing
fees, appeal bond, printed record and brief fees.

(*JULY 19, 1984- RULE 141, S16- 1) INCOME requirement (“wage earners whose gross
income”)= not exceeding P2k/P1.5k for those residing in/outside Metro Manila; 2) PROPERTY
requirement= P24k or P18k)

When the RoC on Civil Procedure was amended by the 1997 Rules of Civil Procedure(Rules 1-
71) in BM 803, Rule 3, S22 was superseded by Rule 3, S21 of the 1997 Rules. There was no
amendment on Rule 141, S16 on pauper litigants.

(*JULY 1, 1997- RULE 3, S21 superseded S22; NO AMENDMENT of RULE 141, S16)

Rule 141 was amended in AM00-2-01-SC, where certain fees were increased or adjusted. S16,
Rule 141, making it S18, was amended. The amounts were made P4k for those in Manila, P3k
outside, and those without real property of more than P50k. It can be seen that the rule on pauper
litigants was inserted in Rule 141 without revoking or amending S21 of Rule 3, exempting pauper
litigants from filing fees. There were then two existing rules on pauper litigants: Rule 3, S21 and
Rule 141, S18.

(*MARCH 1, 2000- RULE 141, S18 superseded S16; 1) INCOME requirement (“pauper litigants
whose gross income and that of their immediate family”) = not exceeding P4k/3k in/outside
Metro Manila, 2) PROPERTY requirement= P50k; RULE 3, S21 was NOT amended or revoked)

S18, Rule 141 was further amended in AM 04-2-04-SC, become S19:


SEC. 19. Indigent litigants exempt from payment of legal fees. — INDIGENT
LITIGANTS (A) WHOSE GROSS INCOME AND THAT OF THEIR IMMEDIATE
FAMILY DO NOT EXCEED AN AMOUNT DOUBLE THE MONTHLY MINIMUM
WAGE OF AN EMPLOYEE AND (B) WHO DO NOT OWN REAL PROPERTY
WITH A FAIR MARKET VALUE AS STATED IN THE CURRENT TAX
DECLARATION OF MORE THAN THREE HUNDRED THOUSAND (P300,000.00)
PESOS SHALL BE EXEMPT FROM PAYMENT OF LEGAL FEES.
The legal fees shall be a lien on any judgment rendered in the case favorable to the indigent
litigant unless the court otherwise provides.
To be entitled to the exemption herein provided, the litigant shall execute an affidavit that
he and his immediate family do not earn a gross income abovementioned, and they do not
own any real property with the fair value aforementioned, supported by an affidavit of a
disinterested person attesting to the truth of the litigant's affidavit. The current tax
declaration, if any, shall be attached to the litigant's affidavit.
Any falsity in the affidavit of litigant or disinterested person shall be sufficient cause to
dismiss the complaint or action or to strike out the pleading of that party, without prejudice
to whatever criminal liability may have been incurred.

(*AUGUST 16, 2004- RULE 141, S19 superseded S18; 1) INCOME requirement = not exceed
2x minimum wage of an employee, 2) PROPERTY requirement = P300k)

Amendments to Rule 141 were made to implement RA 9227, which increased filing fees. The
ceiling for the gross income of litigants was increased from P4k/P3k a month in/outside of Manila
to double the monthly minimum wage of an employee. The value of the property owned was
increased from P50k to P300k to accommodate more indigent litigants and promote easier access
to justice by the poor in the wake of the new increases in filing fees. There was no amendment or
recall of Rule 3, S21.

The RTC in its orders on April 14 and July 17, 2000, incorrectly applied Rule 141 S18 on Legal
fees when the applicable rules at that time were Rule 3, S21 on Indigent Party which took effect
on July 1, 1997 and Rule 141, S16 on Pauper litigants which became effective on July 19, 1984
up to Feb 28, 2000.

The old S16,Rule 141 has two requirements: 1) income requirement (monthly income of not
more than P2k/P1.5k) and 2) property requirement (not more than 18k). Here, petitioners
submitted the affidavits of Lorencita and neighbor Erlinda Bangate, the pay slip of Antonio
showing a monthly income of P10474, and a certification of Naga City stating that petitioners do
not have property declared in their names. The property requirement is thus met. But as to the
income requirement, the P10474 and P3k income of Lorencita, when combined, were above the
P1.5k monthly income threshold. The income requirement was not satisfied. Even if Rule 141,
S18 was applied, the income would still be beyond the P3k monthly income threshold.

Petitioners argue in their MR with the trial court that the rules have been relaxed by relying on
Rule 3, S21 of the 1997 Rules of Civil Procedure:
Section 21. Indigent party. — A party may be authorized to litigate his action, claim or
defense as an indigent if the court, upon an ex parte application and hearing, is satisfied
that the party is one who has no money or property sufficient and available for food, shelter
and basic necessities for himself and his family.
Such authority shall include an exemption from payment of docket and other lawful fees,
and of transcripts of stenographic notes which the court may order to be furnished him.
The amount of the docket and other lawful fees which the indigent was exempted from
paying shall be a lien on any judgment rendered in the case favorable to the indigent, unless
the court otherwise provides.
Any adverse party may contest the grant of such authority at any time before judgment is
rendered by the trial court. If the court should determine after hearing that the party
declared as an indigent is in fact a person with suFcient income or property, the proper
docket and other lawful fees shall be assessed and collected by the clerk of court. If
payment is not made within the time >xed by the court, execution shall issue for the
payment thereof, without prejudice to such other sanctions as the court may impose.

Does the trial court have to apply both Rule 141, S16 and Rule 3, S21? Or should it discard Rule
3, S21 as having been superseded by Rule 141,S16? The Court rules that Rule3,S21 and
Rule141,S16(later amended by Rule141,S19, which is the present rule) are still valid and
enforceable rules on indigent litigants.

The history of the two seemingly conflicting rules reveals that it was not the intent of the Court to
consider the old Rule 3,S22 as to have been amended and superseded by Rule 141, S16. If that
were the case, then the SC, upon recommendation of the Committee on Revision on Rules, could
have already deleted S22 from Rule 3 when it amended Rules 1-71 and approved the 1997
Rules of Civil Procedure. Also, Rule 141 was amended twice. Yet, despite these amendments,
there was no attempt to delete S21 from Rule 3. This shows the desire of the Court to maintain
the 2 rules on indigent litigants. It may not be said to have impliedly repealed Rule3,S21 as implied
repeals are frowned upon unless the repugnancy between the two is irreconcilable. The two rules
thus should be harmonized.

The two rules can stand together and are compatible. When an application to litigate as an indigent
litigant is filed, the court shall scrutinize the affidavits and supporting documents submitted by the
applicant to determine if the applicant complies with the income and property standards prescribed
in the present Section 19 of Rule 141 — that is, the applicant's gross income and that of the
applicant's immediate family do not exceed an amount double the monthly minimum wage of an
employee; and the applicant does not own real property with a fair market value of more than
Three Hundred Thousand Pesos (PhP 300,000.00). If the trial court finds that the applicant meets
the income and property requirements, the authority to litigate as indigent litigant is automatically
granted and the grant is a matter of right.

But if the trial court finds that one or both requirements have NOT been met, then it would set
a hearing to enable the applicant to prove that he has “no money or property sufficient and
available for food, shelter, and basic necessities for himself and his family.” The adverse party
may adduce counteravailing evidence. S21 Rule3 also provides that the adverse party may later
still contest the grant at any time before judgment is rendered by the trial court possibly based
on newly discovered evidence not obtained at the time the application was heard. If the court
determines after hearing that the party declared as indigent is in fact with sufficient
income/property, the proper docket and lawful fees shall be assessed and collected by the clerk of
court. If payment is not made within the time fixed, execution shall issue or the payment of
prescribed fees shall be made, without prejudice to such other sanctions as the court may impose.

It is conceded that Rule 141 S19 has specific standards while Rule 3 S21 does not. As litigants
may abuse the grant of authority, the trial court must use sound discretion and scrutinize
evidence strictly in granting exemptions, aware that the applicant has not hurdled the precise
standards in Rule 141.

Thus, the trial court should have applied Rule 3, S21 after the affidavits of the Alguras showed
that they did not satisfy the requirements in Rule 141 and called a hearing instead of disqualifying
them. The Court remands the case to the trial court to determine if petitioners can be considered
indigent litigants using Rule3, S21.

If the applicant meets the salary and property requirements in S19, Rule 141, the grant of the
application is MANDATORY. If the application does not satisfy one or both requirements, the
application should not be denied outright. The court must instead apply the “INDIGENCY
TEST” under S21, Rule 3 and use its sound discretion in determining the merits of the prayer
for exemption.

149. Re: Query of Mr. Roger Prioreschi re Exemption from Legal and Filing Fees of the
Good Shepherd Foundation, Inc., AM 09-6-9-SC, August 19, 2009, Bersamin,J. (Free Access
and Adequate Legal Assistance)
FACTS:
In a letter addressed to the CJ, Roger Prioreschi, administrator of the Good Shepherd Foundation,
Inc. wrote:
“The Hon. Court Administrator Jose Perez pointed out to the need of complying with OCA
Circular No. 42-2005 and Rule 141 that reserves this "privilege" to indigent persons. While
judges are appointed to interpret the law, this type of law seems to be extremely detailed
with requirements that do not leave much room for interpretations.

In addition, this law deals mainly with "individual indigent" and it does not include
Foundations or Associations that work with and for the most Indigent persons. Xxx
we have been working hard for the very Filipino people that the government and the society
cannot reach to, or have rejected or abandoned them. (**orphanage sila)

Can the Courts grant to our Foundation who works for indigent and underprivileged
people, the same option granted to indigent people? Xxx”

ISSUE:
Can the courts grant to foundations the same exemption from payment of legal fees granted to
indigent litigants even if the foundations are working for indigent and underprivileged people?
HELD: NO.
The basis for the exemption from legal and filing fees is the free access clause embodied in Sec.11,
Art.III of the Constitution:
Sec. 11. Free access to the courts and quasi judicial bodies and adequate legal assistance
shall not be denied to any person by reason of poverty.
Implementing this provision are Sec.21, Rule3 and Sec19, Rule 141 of the RoC. The clear intent
and precise language of these provisions indicate that only a natural party litigant may be
regarded as an indigent litigant. Since the Good Shepherd Foundation, Inc. is a corporation
invested by the state with a juridical personality separate and distinct from that of its members, it
is a juridical person. It has the power to acquire and possess property of all kinds and incur
obligations and bring civil and criminal actions in conformity with laws and regulations of their
organization. As a juridical person, it thus cannot be exempted from legal and filing fees granted
to indigent litigants.

That the Good Shepherd Foundation is working for indigent underprivileged people is of no
moment. The Constitution has explicitly premised the free access clause on a person’s poverty,
a condition that only a natural person can suffer. Also, extending the exemption to a juridical
person on such ground may be prone to abuse (even with rigid documentation requirements),
particularly by corporations bent on circumventing the rule on payment of the fees. The scrutiny
of compliance with the documentation requirements may also be too time-consuming and wasteful
for the courts.

150. Re: Letter dated April 18, 2011 of Chief Public Attorney Persida Reuda-Acosta
Requesting Exemption from the Payment of Sheriff’s Expenses, AM 11-10-03-O, July 30,
2013, Reyes, J. (Free Access and Adequate Legal Assistance)
FACTS:
PAO Chief Atty. Acosta sent a letter to the OCA in which she sought a clarification as to the
exemption of PAO’s clients from payment of sheriff’s expenses, alleging that PAO’s clients in ints
Regional office in region VII are being charged with sheriff’s expenses of P1k upon the filing of
a civil action in court. She claims that sheriff’s expenses should not be charged to PAO’s clients
since S6 of RA 9406 specifically exempts them from docket and other fees incidental to instituting
an action in court and other quasi-judicial bodies. The OCA replied that sheriff’s expenses are not,
strictly speaking, “legal fees” under Rule 141 of the RoC as they are not payable to the government
but to the sheriff to defray his travel expenses in serving court processes in relation to the litigant’s
case.

Atty. Acosta maintains that sheriff’s fees are nonetheless considered as a fee incidental to the filing
of an action in court and should not be exacted from PAO’s clients. The OCA, upon Atty. Acosta’s
request, referred the matter to the Court en banc with its recommendation maintaining its (OCA)
position. The SC adopted the recommendation of the OCA and denied Acosta’s request.

Acosta sought reconsideration of the denial. The SC denied this also thru a resolution. Acosta then
filed a motion for leave to file a second MR and a second MR of this denial. She claims that the
imposition of sheriff’s expenses on PAO’s clients is contrary to the language and spirit of RA
9406, S6 and would hinder the clients’ access to the courts contrary to S11, Art.III.

ISSUE:
Whether the clients of the Public Attorney’s Office may be exempted from paying sheriff’s fees
which is used to defray the transportation expenses of the sheriff in serving court processes.
HELD: NO.
What is involved here is the Court’s administrative power to determine its policy vis-à-vis the
exaction of legal fees from the litigants.

At the core is the proper interpretation of S6, RA 9406:


Sec. 6. New sections are hereby inserted in Chapter 5, Title III, Book IV of Executive Order
No. 292, to read as follows: xxx xxx xxx Sec. 16-D. Exemption from Fees and Costs of the
Suit. — The clients of PAO shall be exempt from payment of docket and other fees
incidental to instituting an action in court and other quasi-judicial bodies, as an original
proceeding or on appeal.

Words of a statute will be interpreted in their natural, plain, and ordinary acceptation and
signification unless the legislature intended a technical or special legal meaning to those words.
The intention of the lawmakers, who are ordinarily untrained philologists and lexicographers, to
use statutory phraseology in such a manner is always presumed. A plain reading of the provision
shows that the exemption granted to PAO’s clients cannot be extended to the payment of
sheriff’s expenses. The exemption is specifically limited to payment of fees- docket and other
fees incidental to instituting an action.

“Fees” is defined as a charge fixed by law or by an institution for certain privileges or services.
Viewed from this context, the phrase “docket and other fees incidental to instituting an action”
refers to the totality of the legal fees imposed under Rule 141 of the RoC. It includes filing or
docket fees, appeal fees, fees for issuance of provisional remedies, mediation fees, sheriff’s fees,
stenographer’s fees, and commissioner’s fees. These are fees exacted for the services rendered by
the court in connection with the action instituted before it.

But sheriff’s expenses cannot be classified as a “fee” within the exemption. Under Rule 141, S10,
it is “to defray the actual travel expenses of the sheriff, process server, or other court-
authorized persons in the service of summons, subpoena, and other court processes that
would be issued relative to the trial of the case.” The fee is thus not exacted for any service
rendered by the court. It is not the same as sheriff’s fees under S10 Rule 141 which refers to
those imposed by the court for services rendered to a party incident to the proceedings before it.

There is an intent to differentiate sheriff’s “fees” from sheriff’s “expenses” in S10 Rule 141, which
uses “fees” in delineating the enumeration in the first paragraph and “Expenses” in qualifying the
subsequent paragraphs. The first sentence of the second paragraph, “in addition to the fees
hereinabove fixed” shows that sheriff’s expenses are separate charges on top of sheriff’s fees.

However, the Court is not unmindful of the predicament of PAO’s clients. Free access to the courts
and adequate legal assistance are among the fundamental rights the constitution extends to the less
privileged. Thus, the Court hereby authorizes the officials and employees of PAO to serve
summons, subpoena, and other court processes pursuant to S3, Rule 14 of the RoC. This
authority is limited to cases involving their client. This would relieve their clients from the burden
of paying for the sheriff’s expenses despite their non-exemption. The amount to be defrayed in the
service of summons would thus have to be taken from PAO’s operating expenses. This amount
may be recovered from the amount recovered from the adversaries of PAO’s clients as costs of
suit, attorney’s fees, or contingent fees prior to the deposit thereof in the National Treasury.

Sec.12- CUSTODIAL INVESTIGATION.


151. People v. Hon. Ruben Ayson, GR 85215, July 07, 1989, Narvasa, J. (Custodial
Investigation)
FACTS:
Private respondent Felipe Ramos was a ticket freight clerk of PAL, assigned at its Baguio City
station. PAL management notified him of an investigation into his involvement in irregularities in
the sales of plane tickets. The investigation was scheduled in accordance with PAL’s Code of
Conduct and Discipline, and the CBA signed by it with the PAL Employees Association (PALEA).
One day before the investigation, Ramos gave to his superiors a handwritten note, saying that he
is willing to settle the irregularities allegedly charged against him in the amount of P76k subject
to conditions as may be imposed by PAL.

At the investigation, Ramos was informed of the “finding of the Audit Team”. Thereafter, Ramos’
answers in response to questions by branch manager Cruz were taken down. His answers
were to the effect that he had not made disclosure of the tickets mentioned in the Audit Team’s
findings and the proceeds were “misused” by him. He was prevented from paying back the money
“perhaps by shame” and that he was still willing to settle his obligation and proffered a compromise
to pay on staggered basis. But no compromise agreement was reached.

2 months later, an information against Ramos was filed, charging him of estafa allegedly
committed in Baguio City. The information alleged that Ramos, having been entrusted with and
received in trust fare tickets of passengers xxx in the total amount of P76,700.65 with the express
obligation to remit all the proceeds of the sale, xxx instead of complying with his obligation, xxx
misappropriate, xxx the value of the tickets xxx in spite of repeated demands failed and refused to
make good his obligation. He entered a plea of “not guilty” at trial, which was prosecuted by PAL
lawyers under the supervision of the fiscal.

The private prosecutors of PAL made a written offer of evidence which included the statement of
Ramos (Exhibit A) and his “handwritten admission” (Exhibit K). The defense objected as the
documents, which appears to be a confession, was taken without the accused being represented
by a lawyer. Judge Ayson admitted all the exhibits except Exhibits A and K, which it rejected. He
declared that Exhibits A and K are inadmissible as Ramos was not reminded of his constitutional
rights to remain silent and to have counsel and when he waived the same and gave his statement,
he was not with counsel.

He cited various cases, saying that in custodial investigations, the right to counsel may be waived
but only with assistance of counsel. The fact that Ramos was not detained at the time or that the
investigation was administrative in character does not except the case from the ambit of the
constitutional provision. Hence this petition for certiorari.

ISSUE:
Whether statements made by Felipe Ramos during an investigation by Philippine Airlines in its
Baguio City Ticket Office admitting that he indeed did not remit certain ticket sales to PAL and
that he was willing to pay the unremitted amount in staggered basis may be admissible in evidence
against him in a prosecution for Estafa.
HELD:
At core of the issue is S20, Article IV of the 1973 Constitution:
SEC. 20. No person shall be compelled to be a witness against himself. Any person under
investigation for the commission of an offense shall have the right to remain silent and to
counsel, and to be informed of such right. No force, violence, threat, intimidation, or any
other means which vitiates the free will shall be used against him. Any confession obtained
in violation of this section shall be inadmissible in evidence.
This section deals with 1) the right against self-incrimination and 2) the rights of a person in
custodial investigation- the rights of every suspect under investigation for the commission of an
offense.
The 1987 Constitution has placed these rights in separate sections and indicates more clearly the
disparateness of these right. The right against self-incrimination is now in S17 of Art.III while the
rights of a person in custodial interrogation are now in S12 of Art.III.

1. Right against self-incrimination.


This right applies to every person who gives evidence, voluntarily or under compulsion of
subpoena, in any civil, criminal, or administrative proceeding. It prescribes an option of refusal
to answer incriminating questions and not a prohibition of inquiry. It allows a witness to
refuse to answer any particular incriminatory question. The right can be claimed only when the
specific incriminatory question is actually put to the witness. It cannot be claimed at any other
time. It does not give the witness the right to disregard a subpoena, to decline to appear before the
court, or to refuse to testify altogether. It is only when a particular question is addressed to him,
the answer to which may incriminate him for some offense, that he may refuse to answer.

This does not impose on the judge or other presiding officer over a trial or investigation any
obligation to advise a witness of such right. The witness should know this right pursuant to the
axion that everyone is presumed to know the law and ignorance of the law excuses no one. The
judge nor witness cannot be expected to know in advance the character or effect of a question to
be put to the witness. This right is NOT self-executing. It must be claimed. Thus, it may be
waived, expressly or impliedly, as by a failure to claim it at the appropriate time.

2. Rights in custodial investigation.


These rights apply to persons “under investigation for the commission of an offense”- suspects
under investigation by police authorities. This is unlike the right against self-incrimination
which applies indiscriminately to any person testifying in any proceeding. This provision was
derived from the US SC decision in Miranda v. Arizona.

S20 states that when a person is under investigation for the commission of an offense: 1) he shall
have the right to remain silent and to counsel, and to be informed of such right, 2) nor force,
violence, threat, intimidation, or any other means which vitiates the free will shall be used against
him; and 3) any confession obtained in violation of . . . these rights shall be inadmissible in
evidence.

CJ Warren in Miranda summarized the procedural safeguards laid down for a person in police
custody, “in-custody interrogation” being regarded as the commencement of an adversary
proceeding against the suspect.
He must be warned prior to any questioning that he has the right to remain silent, that
anything he says can be used against him in a court of law, that he has the right to the
presence of an attorney, and that if he cannot afford an attorney one will be appointed for
him prior to any questioning if he so desires. Opportunity to exercise those rights must be
afforded to him throughout the interrogation. After such warnings have been given, such
opportunity afforded him, the individual may knowingly and intelligently waive these
rights and agree to answer or make a statement. But unless and until such warnings and
waivers are demonstrated by the prosecution at the trial, no evidence obtained as a result
of interrogation can be used against him.

The objective is to prohibit incommunicado interrogation of individuals in a police-dominated


atmosphere resulting in self-incriminating statement without full warnings of constitutional rights.
These rights exist only in “custodial interrogations”, which is meant “questioning initiated by
law enforcement officers after a person has been taken into custody or otherwise deprived of his
freedom of action in any significant way." After a person is arrested and his custodial investigation
begins, a confrontation arises which is unequal as he is examined by many investigators in
unfamiliar surroundings. S20 seeks to remedy this imbalance. If not made in custodial
interrogation, the statement is not protected.

3. Rights of defendant in criminal case as regards giving testimony.


Do the two rights above discussed apply to persons under preliminary investigation or already
charged in court for a crime? Such person is not under custodial investigation. Interrogation by
police would already have ended by the time of filing of the criminal case in court or with the
fiscal. Thus, this does NOT apply to a defendant in a criminal case already pending in court as
he is no longer under custodial interrogation.

But the accused in court or undergoing preliminary investigation with the fiscal possesses the right
against self-incrimination. Additionally, the accused has other rights under RoC like 1) to be
exempt from being a witness against himself and 2) to testify as witness in his own behalf but
subject to cross-examination as any other witness. His refusal to be a witness shall not prejudice
him.

Unlike an ordinary witness or party in a civil action who may be compelled to testify by subpoena,
having only the right to refuse to answer particular incriminatory questions, the accused can refuse
to testify altogether. But on cross-examination, he may not refuse to answer any question on the
ground that the answer would have the tendency to incriminate him.

If the accused be asked a question which might incriminate him, not for the crime with which he
is charged, but for some other crime, he may still decline to answer that specific queston under
S20, Art.IV of the 1973 Constitution (S17, Art. III, 1987 Constitution). Thus, in a prosecution for
murder, he may not on cross-examination refuse to answer any question that might implicate him
for that murder, but he may decline to answer a question which might implicate him for a different
offense, say, estafa.

Thus, a person suspected of committing a crime and subsequently charged has the following rights:
1) BEFORE THE CASE IS FILED IN COURT (or with the public prosecutor, for preliminary
investigation), but after having been taken into custody or otherwise deprived of his liberty in some
significant way, and on being interrogated by the police: the continuing right to remain silent and
to counsel, and to be informed thereof, not to be subjected to force, violence, threat, intimidation
or any other means which vitiates the free will; and to have evidence obtained in violation of these
rights rejected; and
2) AFTER THE CASE IS FILED IN COURT —a) to refuse to be a witness; b) not to have any
prejudice whatsoever result to him by such refusal; c) to testify to his own behalf, subject to
cross-examination by the prosecution; d) WHILE TESTIFYING, to refuse to answer a specific
question which tends to incriminate him for some time other offense than that for which he
is prosecuted.

Thus, Judge Ayson took these rights as applying to the same juridical situation, equating one with
the other. He has grossly erred. Ramos was not in any sense under custodial interrogation prior to
and during the administrative inquiry. The rights in S20 thus did not come into play. Ramos
voluntarily answered questions posed to him on the first day of the administrative investigation
and agreed that the proceedings should be recorded. His note offering to compromise his liability
was a free and spontaneous act that may not be precluded on the ground of “Miranda rights” not
being accorded.

Ayser adverts to a danger that the investigation being done by the complaining parties themselves
might be tainted with overeagerness on the hapless suspects. But the law requires due process
before disciplinary sanctions may be imposed. The employee must be informed of the offenses
and provided with adequate opportunity to explain his side. It is absurd to reject his statements, if
he opts to give statements in the investigation, on the ground that he had not been accorded his
“Miranda rights” which are relevant only in custodial investigations. The employee’s statements
are submitted by him precisely so that they may be considered by the investigating committee in
negation of his liability.

While Ayser’s fear may actually happen, that violence or undue pressure be brought upon the
employee under investigation, in such an event any admission wrung from the person under
interrogation would be inadmissible not because of S20, Art.IV of the 1973 constitution, but
simply on the general incontestable proposition that involuntary or coerced statements may not
in justice be received against the makers thereof. Exhibits A and K are thus admissible.

152.Miranda v. Arizona, 384 US 436, June 13, 1966, Chief Justice Warren (Custodial
Investigation)
FACTS:
The issue is the admissibility of statements obtained from a defendant questioned while in custody
or otherwise deprived of his freedom of action in any significant way. In each of the 4 cases
here, defendant was questioned by police officers, detectives, or a prosecuting attorney in a room
in which he was cut off from the outside world. In none of the cases was the defendant warned of
his rights at the outset of the interrogation process. In all cases, the questioning elicited oral
admissions and in 3 of them signed statements which were admitted at trial. They all thus share
salient features: incommunicado interrogation of individuals in a police-dominated atmosphere,
resulting in self-incriminating statements without full warnings of constitutional rights.
In No. 759, Miranda v. Arizona, the police arrested the defendant and took him to a special
interrogation room, where they secured a confession. In No. 760, Vignera v. New York, the
defendant made oral admissions to the police after interrogation in the afternoon, and then signed
an inculpatory statement upon being questioned by an assistant district attorney later the same
evening. In No. 761, Westover v. United States, the defendant was handed over to the Federal
Bureau of Investigation by local authorities after they had detained and interrogated him for a
lengthy period, both at night and the following morning. After some two hours of questioning, the
federal officers had obtained signed statements from the defendant. Lastly, in No. 584, California
v. Stewart, the local police held the defendant five days in the station and interrogated him on nine
separate occasions before they secured his inculpatory statement.

HELD:
The constitutional foundation underlying the privilege against self-incrimination is the respect a
government must accord to the dignity and integrity of its citizens. To maintain a “fair state-
individual balance”, to require the government “to shoulder the entire load,” our accusatory system
of criminal justice demands that the government seeking to punish an individual produce the
evidence against him by its own independent labors, rather than by the simple expedient of
compelling it from his mouth. The privilege is fulfilled only when the person is guaranted the
right to remain silent unless he chooses to speak in the unfettered exercise of his own will.

Is the privilege fully applicable during custodial interrogation? The principles embodied in the
privilege apply to informal compulsion by law enforcement officers during in-custody questioning
(*so yes). The compulsion to speak in the isolated setting of the police station may be greater than
in courts or other official investigations. In criminal trials, wherever a question arises whether a
confession is incompetent because not voluntary, the issue is controlled by that portion of the fifth
amendment commanding that no person shall be compelled in any criminal case to be a witness
against himself.

From the causes which the law treats as legally sufficient to engender in the mind of the accused
hope or fear in respect to the crime charged, the accused was not involuntarily impelled to make
a statement, when, but for the improper influences, he would have remained silent. The
requisite of voluntariness is not satisfied by establishing merely that the confession was not
induced by a promise or threat. A confession is voluntary in law if it was in fact voluntarily made.
judicial precedent clearly establishes the privilege’s application to incommunicado interrogation.

The voluntariness doctrine encompasses all interrogation practices which are likely to exert such
pressure upon an individual as to disable him from making a free and rational choice. The entire
thrust of police interrogation was to put the defendant in such an emotional state as to impair his
capacity for rational judgment. The abdication of the constitutional privilege- the choice on his
part to speak to the police (citing precedent in Escobedo v. Illinois), was not made knowingly
or competently because of the failure to apprise him of his rights. The compelling atmosphere
of the in-custody interrogation, and not an independent decision on his part, caused the defendant
to speak. The presence of counsel would be the adequate protective device to make the police
interrogation conform to the dictates of the privilege.

The fifth amendment today is available outside of criminal court proceedings and protects
persons in all settings in which their freedom of action is curtailed in any significant way from
being compelled to incriminate themselves. To combat the inherently compelling pressures in in-
custody interrogation, the accused must be adequately and effectively apprised of his rights,
and the exercise of those rights must be fully honored.
It is impossible for us to foresee the potential alternatives for protecting the privilege which
Congress might devise. Unless we are shown other procedures which are at least as effective in
apprising accused persons of their right of silence and in assuring a continuous opportunity
to exercise it, the following safeguards must be observed.

If a person in custody is to be subjected to interrogation, he must first be informed in clear and


unequivocal terms that he has the RIGHT TO REMAIN SILENT. For those unaware of the
privilege, the warning is needed simply to make them aware of it- the threshold requirement
for an intelligenct decision as to its exercise. This is a prerequisite to overcoming the inherent
pressures of the interrogation atmosphere. We will not inquire in individual cases whether the
defendant was aware of his rights, due to his age, education, intelligence, or prior contact with
authorities, without a warning being given as this can never be more than speculation.

The warning of the right to remain silent must be accompanied by the explanation that
ANYTHING SAID CAN AND WILL BE USED AGAINST THE INDIVIDUAL IN COURT.
This warning is needed to make him aware not only of the privilege, but also the consequences of
foregoing it. This may make the individual aware that he is not in the presence of persons acting
solely in his interest and that he is faced with a phase of the adversary system.

The circumstances of in-custody interrogation can very quickly overbear the will of one merely
made aware of his privilege by his interrogators. Thus, the RIGHT TO HAVE COUNSEL
PRESENT AT THE INTERROGATION is indispensable to the protection of the fifth
amendment. The aim is to assure that the individual’s right to choose between silence and speech
remains unfettered throughout the interrogation process. This comprehends not merely a right to
consult with counsel PRIOR to questioning, but to have counsel PRESENT during any
questioning if defendant so desires. With a lawyer present, the likelihood that police will use
coercion is reduced, and if it is nonetheless used, the lawyer can testify to it in court.

His failure to ask for a lawyer does NOT constitute a waiver. No effective waiver of the right to
counsel during interrogation can be recognized unless specifically made AFTER the warnings
have been given. The accused who does not know his rights and thus does not make a request may
be the person who most needs counsel. Where the assistance of counsel is a constitutional
requisite, the right to be furnished counsel does not depend not a request.

As with the warnings of the right to remain silent and anything stated can be used against him, this
warning is an absolute prerequisite to interrogation. No amount of circumstantial evidence that
the person is aware of this right will suffice to stand in its stead. Only thru such warning is there
ascertainable assurance that accused was aware of this right.

If an individual indicates that he wishes assistance of counsel before interrogation, the authorities
cannot ignore or deny the request on the basis that the individual cannot afford a retained attorney.
The financial ability of the individual has no relationship to the scope of the rights involved. Thus,
it is necessary to warn him not only that he has the right to consult with an attorney, but also
that, if he is an INDIGENT, a LAWYER WILL BE APPOINTED TO REPRESENT HIM.
Without this, the admonition of the right to consult with counsel would often be understood as
meaning only that he can consult with a lawyer if he has funds to obtain one.
Once the warnings are given, the subsequent procedure is clear. If the individual indicates in any
manner at any time prior to or during questioning that he wishes to remain silent, the
interrogation must cease. Any statement taken after the person invokes his privilege cannot be
other than the product of compulsion. If he states that he wants an attorney, the interrogation must
cease until an attorney is present. This does not mean that each police station must have a “station
house lawyer”. If authorities cannot provide counsel during a reasonable period, they may refrain
from doing so without violating the person’s fifth amendment privilege so long as they do not
question him during that time.

If the interrogation continues without an attorney and a statement is taken, a heavy burden rests on
the government to demonstrate that the defendant KNOWINGLY and INTELLIGENTLY
waived his privilege against self-incrimination and his right to retained or appointed counsel.
An express statement that the individual is willing to make a statement and does not want an
attorney, followed closely by the statement, could be a waiver. But a waiver will not be presumed
simply from the silence of the accused after warnings are given or simply because a confession
was eventually obtained. It must be shown that the accused was offered counsel but intelligently
and understandingly rejected the offer.

When a statement is made after lengthy interrogation or incommunicado incarceration, this is


strong evidence that accused did not validly waive his rights.

These warnings and the waiver necessary are, without a fully effective equivalent, prerequisites to
admissibility of any statement made by a defendant. No distinction can be made between direct
confessions and statements which amount to “admissions” of part or all of an offense. The privilege
against self-incrimination does not distinguish degrees of incrimination.

These principles deal with the protection of the privilege against self-incrimination when the
individual is first subjected to police interrogation while in custody at the station or otherwise
deprived of his freedom of action in any significant way. It is at this point that our adversary
system of criminal proceedings COMMENCES.

To summarize, we hold that, when an individual is taken into custody or otherwise deprived of his
freedom by the authorities in any significant way and is subjected to questioning, the privilege
against self-incrimination is jeopardized. Procedural safeguards must be employed to protect the
privilege, and unless other fully effective means are adopted to notify the person of his right of
silence and to assure that the exercise of the right will be scrupulously honored, the following
measures are required. He must be warned prior to any questioning that he has the right to remain
silent, that anything he says can be used against him in a court of law, that he has the right to the
presence of an attorney, and that, if he cannot afford an attorney one will be appointed for him
prior to any questioning if he so desires. Opportunity to exercise these rights must be afforded to
him throughout the interrogation. After such warnings have been given, and such opportunity
afforded him, the individual may knowingly and intelligently waive these rights and agree to
answer questions or make a statement. But unless and until such warnings and waiver are
demonstrated by the prosecution at trial, no evidence obtained as a result of interrogation can be
used against him.
- Miranda v. Arizona
Petitioner Ernesto Miranda was arrested at his home and taken in custody to a Phoenix police
station. He was identified by the complaining witness there. He was brought by police to
“Interrogation Room No.2” of the detective bureau. He was questioned by 2 police officers, who
admitted in trial that Miranda was not advised that he had a right to have an attorney present. 2
hours later, the officers emerged from the room with a written confession signed by Miranda. At
trial before a jury, the confession was admitted into evidence over objection of the defense.
Miranda was convicted of kidnapping and rape and sentenced to 20 to 30 years on each count. The
Arizona SC held that his rights were not violated and affirmed the conviction. It said that Miranda
did not request counsel

We reverse. Miranda was not apprised of his right to consult with attorney and to have one present
during the interrogation, nor was his right not to be compelled to incriminate himself effectively
protected. Without the warnings, the statements were inadmissible. The mere fact that he signed
a statement with a typed-in clause stating that he had “full knowledge” of his “legal rights” does
not approach the knowing and intelligent waiver required to relinquish constitutional rights.

Zacarias Villavicencio et al., v. Justo Lukban, et al., GR 14639, March 25, 1919 (Right to
Travel) *no need to digest.
FACTS:
Shall the judiciary permit a government of men instead of a government of laws to be set up in
PH?

Mayor of Manila Justo Lukban, to exterminate vice, ordered the segregated district for women of
ill repute, permitted for some years in Manila, closed. Between Oct.16-25, 1918, the women were
confined to their houses in the district by the police. During this period, the city authorities
perfected arrangements with the Bureau of Labor for sending the women to Davao, Mindanao,
as laborers. At midnight of Oct.25, the police, acting pursuant to orders from the chief of police,
Hohmann, and Mayor Lukban, descended upon the houses, hustled some 170 inmates into patrol
wagons, and placed them aboard the steamers that awaited their arrival. The women were given
no opportunity to collect their belongings and were under the impression that they were being
taken to a police station for investigation. They did not know that they were destined for a life in
Mindanao. They were not asked if they wished to depart from that region and neither given their
consent to the deportation. They were received on board the steamers by representatives of the
Bureau of Labor. The steamers sailed for Davao during the night of Oct. 25.

The 2 steamers reached Davao on Oct.29. The women were receipted for as laborers by Sales,
governor of Davao. He was not notified that the women were prostitutes expelled from Manila.
Some of the women eventually married, others assumed clandestine relations with men, others
went to work in different capacities, others assumed a life unknown and disappeared, and a goodly
portion found means to return to Manila.

The lawyers and relatives and friends of some of the women filed a petition for habeas corpus to
a member of the SC. The petition, through stipulation of the parties, was made to include all of the
women sent away. The petition alleged that the women were illegally restrained of their liberty by
Lukban and Hohmann. Respondents prayed that the writ should not be granted as petitioners are
not proper parties, the action should have been begun in Davao CFI, they did not have any of the
women under their custody, and because their jurisdiction did not extend beyond the boundaries
of Manila.

The Court awarded the writ in an order on Nov.4, directing respondents to bring before the court
the women on December 2. On this date, none of the women were produced in court by
respondents. They offered affidavits showing that the women were contented with their life in
Mindanao and did not wish to return to Manila. Respondents claim that they did not have custody
of the women, thus it was impossible to obey the habeas corpus. The SC still ordered (second
order) the women to be brought before it on January 13, 1919 unless they, in written statements
voluntarily made before a Davao CFI judge or clerk, renounce the right, or unless respondents
show other legal motives that make compliance impossible.

Respondents on January 13 technically presented the women who had returned to the city through
their own efforts and 8 others brought by respondents to Manila. respondents state that the other
81 women renounced the right to return to Manila even with free transpo thru sworn statements,
51 returned to Manila by other means, and 26 could not be located.

In its second order, the court promised to give reasons for granting the writ of HC. We now proceed
to do so.

HELD:
-Right to travel related
Despite the feeble attempt to prove that the women left voluntarily, that such was not the case is
shown by the mere fact that the presence of the police and constabulary was deemed necessary
and that these officers of the law chose the shades of night to cloak their stealthy acts.

By authority of what law did the Mayor and chief of police Hohmann act in deporting the women?
Alien prostitutes can be expelled from the PH in conformity with an Act of Congress. The
Governor-General can order the eviction of undesirable aliens after hearing from the PH. Act 519
and the Revised Ordinances of Manila, Sec.733, penalize a person who is a common prostitute.
Always a law! But one search in vain (*walang law or anything) for any law, order, or
regulation which even hints at the right of the Manila Mayor or Chief of Police to force
citizens of PH- and these women despite their being in a sense lepers of society are nevertheless
not chattels but PH citizens protected by the same constitutional guaranties- to change their
domicile from Manila to another locality. Contrarily, PH penal law punishes public officers
who, not being expressly authorized by law or regulation, compels any person to change his
residence.

Under the American constitutional system, liberty of abode is a principle deeply embedded in
jurisprudence and considered so elementary in nature as not even to require a constitutional
sanction. Even the PH Governor-General or US president has no such arbitrary prerogative. Much
less has the executive of a municipality who acts within a sphrere of delegated powers. If these
officials, including the presidents and police chiefs of one thousand other municipalities, can take
to themselves such power, then any other official can do the same. And if any official can, then all
persons would have just as much right to do so. And if a prostitute could be sent against her wishes
under no law from one locality to another within the country, then officialdom can hold the same
club over the head of any citizen.

Law defines power. No official, no matter how high, is above the law. The courts are the forum
which functions to safeguard liberty and to punish official transgressors. “The very idea that one
man may be compelled to hold his life, or the means of living, or any material right essential to
the enjoyment of life, at the mere will of another, seems to be intolerable in any country where
freedom prevails, as being the essence of slavery itself.” -Justice Matthews.

The remedies of citizens are 1)civil action, 2) criminal action, and 3) habeas corpus. The writ of
HC was devised as a speedy and effectual remedy to relieve persons from unlawful restraint, and
as the best and only sufficient defense of personal freedom. Any further rights of the parties are
left untouched by decision on the writ, whose principal purpose is to set the individual at liberty.

Respondents raise three objections to its issuance here.

1. The fiscal argues that there is a defect in the parties petitioners. But the petitioners were relatives
and friends of the deportees. The way the expulsion was conducted made it impossible for the
women to sign a petition for HC. Thus it was proper for the writ to be submitted by persons in
their behalf.

2. The fiscal argues that the SC should not assume jurisdiction and that the writ should have been
asked for in the Davao CFI. It is a general rule of good practice that, to avoid unnecessary expense
and inconvenience, petitions for HC should be presented to the nearest judge of CFI. But this is
not a hard and fast rule. The writ may be granted by the SC or any judge enforcible anywhere in
the PH. Whether the writ shall be made returnable before the SC or inferior court rests in the
discretion of the SC and is dependent on the particular circumstances.

Here, it was not shown that the Davao CFI was in session or that the women had any means by
which to advance their plea before that court. On the other hand, it was shown that petitioners and
two original respondents were in Manila. the case involved parties situated in different parts of
PH. The women might still be imprisoned or restrained of their liberty. And it was shown that if
the writ is to accomplish its purpose, it must be taken cognizance of and decided immediately by
the appellate court.

3. The fiscal argues that the women are not restrained of their liberty by respondents. When the
writ was prayed for, the women were free in Davao, and the jurisdiction of the mayor and chief
of police of Manila did not extend beyond city limits.

A prime specification of an application for writ of HC is restraint of liberty. The purpose is to


inquire into all manner of involuntary restraint as distinguished from voluntary, and to relieve a
person therefrom if such restraint is illegal. Any restraint which will preclude freedom of action is
sufficient. Placed in Davao without money or personal belongings, the women were prevented
from exercising the liberty of going when and where they pleased. The restraint of liberty,
which began in Manila, continued until they were returned to Manila and released or until
they freely and truly waived this right.

To agree with such a defense would mean that the mayor could illegally take a citizen and place
him beyond the boundaries of the municipality, and then, when called upon to defend his official
action, could claim that the person was under no restraint and that he had no jurisdiction over this
other municipality. If respondent is within the court’s jurisdiction and has it in his power to obey
the court’s order and thus undo the wrong he has inflicted, he should be compelled to do so. Even
if the party to whom the writ is addressed has illegally parted with the custody of a person before
the application for the writ is no reason why the writ should not issue. If the mayor and chief of
police, acting under no authority of law, could deport these women, these same officials must
necessarily have the same means to return them.

In a question as to whether a writ of HC can issue from the SC of the State of Michigan to bring
into the state a minor detained in another state, the opinion of Justice Cooley is quoted:
The writ is served not upon the person confined but his jailor. It does not reach the former
except through the latter. The court compels the oppressor to release his constraint. If he
fails to obey it, he may be fined and imprisoned. This is the ordinary mode of affording
relief, and if any other means are resorted to, they are only auxiliary to those which are
usual. The place of confinement is, thus, not important to the relief, if the guilty party is
within reach of process, so that by the power of the court he can be compelled to release
his grasp. The important question is, “where is the power exercised?”

The English courts also ruled on the subject. A child had been taken out of England by respondent.
A writ of HC was issued directing him to produce the child, but he did not do so, saying that he
handed the child to another and that it was no longer in his custody or control. It was impossible
for him to obey. He was found in contempt of court. On appeal, the court said:
A writ of HC is a command to bring the child before the judge and must be obeyed, unless
some lawful reason can be shown to excuse nonproduction of the child. If he lawfully
parted with the child before the issuing of the writ and he had no longer the power to
produce the child, this might be an answer. But without lawful reason, he is bound to
produce the child. If he does not, he is in contempt of court.

Thus, none of the defenses bar the granting of the HC.

4. Did respondents comply with the two orders of the SC awarding the writ of HC? If not, should
they be held in contempt and punished?

The first order directed Lukban, Hohmann, Sales, and Yñigo to produce the women on Dec.2. The
order was dated Nov. 04. They were given ample time. The Mayor waited until the 21 st of
November before sending a telegram to the governor of Davao. The Bureau of Labor responded
to the telegram that there were women in Davao who wanted to return but should not be allowed
to do so as they have contracted debts. The half-hearted effort resulted in none of the parties being
brought before the court on Dec.2.
To fulfill the order, the respondents had three optional courses: 1) produce the bodies, 2) show by
affidavit that on account of sickness or infirmity, those persons could not safely be brought before
the court, or 3) they could have presented affidavits to show that the parties in question or their
attorney waived the right to be present. Instead, a few stereotyped affidavits purporting to show
that the women were contented with their life in Davao, some of which have been repudiated by
the signers, were appended to the return of the writ. Through ordinary diligence, at least 60 women
could have been brought back to Manila as they were easily found in Davao. About this number
either returned also at their expense or were produced at the second hearing.

Respondents’ excuses were far from sufficient. “We thought that, having brought about that state
of things by his own illegal act, he must take the consequences.” “He was bound to write letters
for the purpose xxx, do everuthing that mortal man could do in the matter, and that the court would
only accept clear proof of an absolute impossibility by way of excuse.” Here, the return did not
show that every possible effort to produce the women was made by respondents.

In response to the second order, respondents became more zealous. There is substantial
compliance with the second order. Agents were dispatched to Mindanao, placards were posted,
police and constabulary joined in rounding up the women, and a steamer with free transpo to
Manila was provided.

The power to punish for contempt of court should be exercised on the preservative and not on the
vindictive principle. Of the respondents, it was Justo Lukban who was primarily responsible for
the unlawful deportation, who ordered the police to accomplish the same, made arrangements for
the steamers, who negotiated with the Bureau of Labor, and who later had it within his power to
facilitate the return of the unfortunate women to Manila. His intent to suppress the social evil was
commendable. His methods were unlawful. He was fined P100.

Zacarias Villavicencio et al., v. Justo Lukban, et al., GR 14639, March 25, 1919 (Right to
Travel) *no need to digest.
FACTS:
Shall the judiciary permit a government of men instead of a government of laws to be set up in
PH?

Mayor of Manila Justo Lukban, to exterminate vice, ordered the segregated district for women of
ill repute, permitted for some years in Manila, closed. Between Oct.16-25, 1918, the women were
confined to their houses in the district by the police. During this period, the city authorities
perfected arrangements with the Bureau of Labor for sending the women to Davao, Mindanao,
as laborers. At midnight of Oct.25, the police, acting pursuant to orders from the chief of police,
Hohmann, and Mayor Lukban, descended upon the houses, hustled some 170 inmates into patrol
wagons, and placed them aboard the steamers that awaited their arrival. The women were given
no opportunity to collect their belongings and were under the impression that they were being
taken to a police station for investigation. They did not know that they were destined for a life in
Mindanao. They were not asked if they wished to depart from that region and neither given their
consent to the deportation. They were received on board the steamers by representatives of the
Bureau of Labor. The steamers sailed for Davao during the night of Oct. 25.
The 2 steamers reached Davao on Oct.29. The women were receipted for as laborers by Sales,
governor of Davao. He was not notified that the women were prostitutes expelled from Manila.
Some of the women eventually married, others assumed clandestine relations with men, others
went to work in different capacities, others assumed a life unknown and disappeared, and a goodly
portion found means to return to Manila.

The lawyers and relatives and friends of some of the women filed a petition for habeas corpus to
a member of the SC. The petition, through stipulation of the parties, was made to include all of the
women sent away. The petition alleged that the women were illegally restrained of their liberty by
Lukban and Hohmann. Respondents prayed that the writ should not be granted as petitioners are
not proper parties, the action should have been begun in Davao CFI, they did not have any of the
women under their custody, and because their jurisdiction did not extend beyond the boundaries
of Manila.

The Court awarded the writ in an order on Nov.4, directing respondents to bring before the court
the women on December 2. On this date, none of the women were produced in court by
respondents. They offered affidavits showing that the women were contented with their life in
Mindanao and did not wish to return to Manila. Respondents claim that they did not have custody
of the women, thus it was impossible to obey the habeas corpus. The SC still ordered (second
order) the women to be brought before it on January 13, 1919 unless they, in written statements
voluntarily made before a Davao CFI judge or clerk, renounce the right, or unless respondents
show other legal motives that make compliance impossible.

Respondents on January 13 technically presented the women who had returned to the city through
their own efforts and 8 others brought by respondents to Manila. respondents state that the other
81 women renounced the right to return to Manila even with free transpo thru sworn statements,
51 returned to Manila by other means, and 26 could not be located.

In its second order, the court promised to give reasons for granting the writ of HC. We now proceed
to do so.

HELD:
-Right to travel related
Despite the feeble attempt to prove that the women left voluntarily, that such was not the case is
shown by the mere fact that the presence of the police and constabulary was deemed necessary
and that these officers of the law chose the shades of night to cloak their stealthy acts.

By authority of what law did the Mayor and chief of police Hohmann act in deporting the women?
Alien prostitutes can be expelled from the PH in conformity with an Act of Congress. The
Governor-General can order the eviction of undesirable aliens after hearing from the PH. Act 519
and the Revised Ordinances of Manila, Sec.733, penalize a person who is a common prostitute.
Always a law! But one search in vain (*walang law or anything) for any law, order, or
regulation which even hints at the right of the Manila Mayor or Chief of Police to force
citizens of PH- and these women despite their being in a sense lepers of society are nevertheless
not chattels but PH citizens protected by the same constitutional guaranties- to change their
domicile from Manila to another locality. Contrarily, PH penal law punishes public officers
who, not being expressly authorized by law or regulation, compels any person to change his
residence.

Under the American constitutional system, liberty of abode is a principle deeply embedded in
jurisprudence and considered so elementary in nature as not even to require a constitutional
sanction. Even the PH Governor-General or US president has no such arbitrary prerogative. Much
less has the executive of a municipality who acts within a sphrere of delegated powers. If these
officials, including the presidents and police chiefs of one thousand other municipalities, can take
to themselves such power, then any other official can do the same. And if any official can, then all
persons would have just as much right to do so. And if a prostitute could be sent against her wishes
under no law from one locality to another within the country, then officialdom can hold the same
club over the head of any citizen.

Law defines power. No official, no matter how high, is above the law. The courts are the forum
which functions to safeguard liberty and to punish official transgressors. “The very idea that one
man may be compelled to hold his life, or the means of living, or any material right essential to
the enjoyment of life, at the mere will of another, seems to be intolerable in any country where
freedom prevails, as being the essence of slavery itself.” -Justice Matthews.

The remedies of citizens are 1)civil action, 2) criminal action, and 3) habeas corpus. The writ of
HC was devised as a speedy and effectual remedy to relieve persons from unlawful restraint, and
as the best and only sufficient defense of personal freedom. Any further rights of the parties are
left untouched by decision on the writ, whose principal purpose is to set the individual at liberty.
Respondents’ excuses were far from sufficient. “We thought that, having brought about that state
of things by his own illegal act, he must take the consequences.” “He was bound to write letters
for the purpose xxx, do everuthing that mortal man could do in the matter, and that the court would
only accept clear proof of an absolute impossibility by way of excuse.” Here, the return did not
show that every possible effort to produce the women was made by respondents.

In response to the second order, respondents became more zealous. There is substantial
compliance with the second order. Agents were dispatched to Mindanao, placards were posted,
police and constabulary joined in rounding up the women, and a steamer with free transpo to
Manila was provided.

The power to punish for contempt of court should be exercised on the preservative and not on the
vindictive principle. Of the respondents, it was Justo Lukban who was primarily responsible for
the unlawful deportation, who ordered the police to accomplish the same, made arrangements for
the steamers, who negotiated with the Bureau of Labor, and who later had it within his power to
facilitate the return of the unfortunate women to Manila. His intent to suppress the social evil was
commendable. His methods were unlawful. He was fined P100.

153. People v. Ronilo Pinlac, GR 74123-24, September 26, 1988, Paras, J. (Custodial
Investigation)
FACTS:
Pinlac was charged of entering the house of Koji Sato and of Saeki Osamu with intent to gain by
means of force and violence. He was convicted by the RTC.

Two Japanese nationals were neighbors in San Lorenzo Village, Makati. Koji Sato rented a house
in the subdivision and living alone therein although he had a housemaid, Irene Jandayan and cook,
Delia Marcelino. Delia went on maternity leave as she was due to deliver a child with her husband,
Pinlac, who had frequently visited her in Sato’s place.

A low concrete fence separated the house rented by Sato from that rented by Saeki Osamu. April
7, 1984 was a Saturday. The next day was Jandayan’s day-off. 5pm of April 7, Sato went out of
his house. At 6:45pm, Jandayan also left the house to go home. Jandayan locked and closed all
windows and doors.

At 11:30pm, Sato, returning home, noticed that the front door was already unlocked. Upon
returning to his room, his Walkman transistor, placed beside his bed, was missing. He saw in
Jandayan’s room his transistor together with his two wrist watches, cigarette lighter, and eyeglass
case. Another watch, a gold necklace, and P180 were all missing and never recovered. Sato
reported the robbery to the police. It was when the investigators arrived at the residence that he
learned about Osamu’s death.

Osamu was rushed to Makati Medical Center, but he died upon arrival. Detective Viclar
photographed from different angles the crime scene. The death weapon, kitchen knife, was
recovered from the living room. Blood was scattered in the living room. Going around the house,
the investigators saw the slashed screen wall near the back door. Footprints were found in the
backyard corresponding to the impressions of the soles of Pinlac’s shoes. Osamu’s maid, Evelyn
Salomea, said that she saw Pinlac enter Sato’s house at 7pm although she did not see him leave.
She told police the address of Delia, and there the police found Pinlac and invited him to the police
station.

At 8:30pm of April 9, Sgt. Flores extracted the extrajudicial confession of Pinlac. Pinlac claims
that 3 police came to his house about 2pm of April 9 and arrested him without any warrant of
arrest. He was walked around the houses of Sato and Osamu and ordered to reenact according to
what the police theorized how the crime was committed. It was at this moment that the prints of
his shoes were all over the premises. During investigation at police headquarters, he was tortured
and forced to admit the crimes. As a result of the torture, his lips and mouth suffered cuts and
cracks to bleed furiously. Blood dripped into his clothings into his shoes, explaining the blood
stains on the shoes. Before and during the arrest, the officers never mentioned about the bloodstain
in his shoes which they could have easily detected during arrest. They got his shoes only after it
was stained with Pinlac’s blood. Thus, at around 9pm of April 9, where Pinlac could no longer
bear the torture starting from 2pm for 7 hours, he succumbed to the wishes of his torturers and
signed a prepared confession which he was not even allowed to read nor explained to him.

ISSUE:
Whether an extra-judicial confession obtained by the police from Pinlac without informing him of
his constitutional rights to remain silent and to have counsel and from physical torture is admissible
in evidence.
HELD: NO.
Pinlac claims that the RTC erred in admitting in evidence his extra-judicial confession allegedly
obtained thru torture without him having been apprised of his rights and without counsel.

The prosecution evidence leaves much to be desired. There is no direct evidence identifying Pinlac.
The only evidence were circumstantial regarding the fingerprints of Pinlac in the window stabs of
the maid’s quarters of Sato’s house. But Sato explained that this is because he was a frequent
visitor in said house where his wife works as a cook. He could have unknowingly left his
fingerprints, but most especially when he was arrested and ordered to reenact.

The correct procedure when making arrest and in conducting custodial investigation is as follows.
At the time of arrest, the arresting officer must inform him of the reason for the arrest and show
the warrant of arrest. He shall be informed of his constitutional rights to remain silent and to
counsel and that any statement he might make could be used against him. The arrested person
shall have the right to communicate with his lawyer, a relative, or anyone he chooses by the
most expedient means, by telephone if possible or by letter or messenger. No custodial
investigation shall be conducted unless it be in the presence of counsel engaged by the arrested
person, by any person on his behalf, or appointed by the court upon petition either of the detainee
or by anyone in his behalf. The right to counsel may be waived but this shall not be valid unless
made with the assistance of counsel. Any statement obtained in violation of this procedure,
whether exculpatory or inculpatory, in whole or in part shall be inadmissible in evidence.

When the constitution requires a person under investigation “to be informed” of his right to remain
silent and to counsel, it contemplates the transmission of meaningful information rather than
just the ceremonial and perfunctory recitation of an abstract constitutional principle. It is
not sufficient for the officer just to repeat the provisions of the constitution. He must explain the
rights in practical terms. Short of this, there is denial of the right as it cannot be trule said that the
person has been “informed” of his rights.

The prosecution evidence failed to prove compliance with these constitutional rights. Pinlac was
not assisted by counsel. The record is replete with evidence of the maltreatment and torture before
Pinlac signed the prepared extra-judicial confession which the prosecution failed to sufficiently
rebut.

154. People v. Ramon Bolanos, GR 101808, July 03, 1992, Paras, J. (Custodial Investigation)
FACTS:
The RTC convicted Bolanos of murder.

Patrolmen Rolando Alcantara and Francisco Dayao went to the crime scene of Marble Supply,
Bulacan. They saw the deceased Oscar Pagdalian lying on an improvised bed full of blood with
stab wounds. They were informed that Oscar was with 2 companions the previous night, one of
whom was Bolanos who had a drinking spree with Oscar until the wee hours of the morning. When
they apprehended Bolanos, they found the firearm of Oscar on the chair where Bolanos was seated.
They boarded Bolanos and the other companion, Claudio Magtibay, on the police vehicle and
brought them to the police station. In the vehicle, after he was asked by the police if he killed
Oscar, Bolanos admitted that he killed Oscar because he was abusive. The oral admission of
Bolanos was given without assistance of counsel. The RTC admitted the admission as it was given
freely and before investigation.

ISSUE:
Whether Bolanos’ oral admission that he killed Oscar, which admission was made while inside
the police vehicle on the way to the police station, is admissible in evidence.
HELD: NO.
Being already under custodial investigation while on board the police patrol jeep on the way
to the station where formal investigation may have been conducted, Bolanos should have been
informed of his constitutional rights under Art.III, Sec.12:
"(1) Any person under investigation for the commission of an offense shall have the right
to remain silent and to have competent and independent preferably of his own choice. If
the person cannot afford the service of counsel, he must be provided with one. These rights
cannot be waived except in writing and in the presence of counsel.
"(2) No torture, force, violence, threat, intimidation, or any other means which vitiate the
free will shall be used against him. Secret detention places, solitary, incommunicado, or
other similar forms of detention are prohibited.
"(3) Any confession or admission obtained in violation of this or the preceding section shall
be inadmissible in evidence against him.
"(4) The law shall provide for penal and civil sanctions for violation of this section as well
as compensation and rehabilitation of victims of torture or similar practices and their
families."

Since the oral admission was the only reason for the conviction of the RTC, Bolanos is acquitted.

155. Ernesto Navallo v. Sandiganbayan, GR 97214, July 18, 1994, Vitug, J. (Custodial
Investigation)
FACTS:
The provincial auditor of Surigao del Norte, Antonio Espino, made a preliminary audit
examination of cash and other accounts of Navallo, then collecting and disbursing officer of
Numancia National Vocational School. Navallo was found short of P16,483.62. Leopoldo
Dulguime made a final audit and wrote Navallo a letter demanding restitution of the missing
amount. Navallo neither complied nor offered any explanation for the shortage.

Navallo’s duties included collection of tuition fees and preparation of vouchers for salaries of
teachers and employees. The Sandiganbayan convicted Navallo of malversation under Art.217
par.4 of the RPC.

ISSUE:
Whether or not petitioner was under custodial investigation when he signed the certification
prepared by State Auditing Examiner Leopoldo Dulguime.
HELD: NO.
Navallo claims that he was deprived of his rights under S12, Art.III. Such rights are invocable only
when the accused is under “custodial investigation” or is “in custody investigation,” defined as
“any questioning initiated by law enforcement officers after a person has been taken into custody
or otherwise deprived of his freedom of action in any significant way. A person under a normal
audit examination is not under custodial investigation. An audit examiner can hardly be deemed
the law enforcement officer contemplated in the above rule.

156. People v. Rene Januario, GR 98252, February 7, 1997, Panganiban, J. (Custodial


Investigation)
FACTS:
Santiago Cid went to the house of Vicente Dilanco Pons, a farmer in the buy and sell business.
Cid, Pon’s cousin, asked Pons if he wanted to buy a jeepney. Pons replied that he did not have
money but that he could help Cid find a buyer for the jeepney for P50k. Pons offered to look for
a buyer of the jeep provided that Cid entrust the vehicle to him. Cid agreed. At the time, Pons did
not know who owned the jeep. He eventually offered it for sale to Myrna Temporas for P65k, but
Myrna paid only P48,500.

Myrna claims that Pons said the jeep was owned by his niece, Doris Wolf. Pons, acting on
instructions of Doris, borrowed from Myrna P48,500 and used the jeep as collateral. The amount
was given P10k in cash and the balance in check, which was encashed. Myrna asked Pons to secure
a SPA from Doris Wolf. Pons promised to comply. But Pons failed to pay the indebtedness. Myrna
filed a complaint for estafa against Pons with NBI. The NBI contacted the relatives of the jeep
owner and it was informed that its driver, Geronimo Malibago, and conductor, Andrew Patriarca,
Jr., had been killed by carnappers.

The NBI found that the carnapping and killing were done by Rene Januario, Efren Canape, Eliseo
Sarita (Toto), and Eduardo Sarinos (Digo) (with Cid, accused). The jeep was disposed of through
Cid. It learned that Sarita and Sarinos took Patriarca and Malibago inside a sugar plantation where
presumably they were killed.

At the Taft Ave. head NBI office, the team took the statements of appellants (Januario and Canape)
one at a time. They asked Atty. Saunar to assist appellants during the investigation. Agent Vela
took the statement of Januario while Agent Toribio took that of Canape. The statement of
Januario narrated how the carnapping took place. He signed and thumbmarked his statement
which was sworn before NBI executive director Ranin. It was also signed by Atty. Carlos Saunar
“as counsel.” Canape’s statement narrated the following:

Digo told Canape to find a buyer for a jeep. He and Januario found Cid interested to buy the jeep.
Thus, Januario and Canape went back to Digo and Toto and told them that there was a buyer
already. While drinking, Toto said that they will get a jeep. The four waited along Bulihan, Silang
Cavite and waited for a few minutes. When a jeep passed by without passengers, Digo haled it and
they rode the jeep. Toto and Digo got a sharp object and pointed it at the driver and conductor.
Toto, Digo, and Januario brought the driver and conductor down and brought them to the
“tubuhan.” Canape was left inside the jeep. The three went back to the jeep without the driver and
conductor. They went to Cid and gave him the jeep, who said that he will bring the jeep to Pons.

After the investigation, appellants went with NBI agents and searched for their companions. The
body of Andrew Patriarca, Jr. was found in a sugarcane plantation in Maguyam with his head
severed from the body. The driver, Malibago, stepfather of Doris Wolf, the jeep owner, was
recovered therein also.
Atty. Carlos Saunar testified that he joined the NBI in May or June 1988. In March 1988, while
still in private practice, he was at the NBI head office handling a client case when an NBI agent
approached him and introduced him to appellants and Cid. The agent Vela and Toribio told him
that the three had verbally confessed to participation in a crime and they needed his assistance as
they were a out to execute their sworn statements. Atty. Saunar agreed to assist them and allegedly
explained to them the consequences of their confession, like their rights to be silent and to counsel
and that whatever they would say could be used against them. He said that he was present in taking
Canape and Januario’s sworn statements.

ISSUE:
Whether the statements of Januario and Canape confessing to and narrating the crime of
carnapping committed by them and their co-accused in the presence of counsel, which counsel was
found by NBI agents in the NBI headquarters while he was there for a client’s case and also
applying for a job with the NBI, is admissible in evidence.
HELD: NO.
Section 12 (1), Art.III requires that a person under investigation shall have no less than “competent
and independent counsel preferable of his own choice.” “Competent” and “independent” were
terms absent in the constitutions previous to the 1987 constitution. Thus, the lawyer should be as
far as reasonably possible, the choice of the individual undergoing questioning. If the lawyer
were one furnished in the accused behalf, he should be COMPETENT and INDEPENDENT,
i.e. that he is willing to fully safeguard the constitutional rights of the accused, as distinguished
from one who would merely be giving a routine, peremptory, and meaningless recital of the
individual’s constitutional rights.

Ideally, a lawyer engaged for an individual facing custodial investigation, if he is unable to afford
one, should be engaged by the accused himself, or by the latter’s relative or person authorized
by him to engage an attorney or by the court, upon proper petition of the accused or authorized
person.

Saunar was not the choise of Januario as his investigation counsel. Agent Vela testified that Atty.
Saunar was “just somewhere around” and that they requested his services. Agent Toribio testified
that Atty. Saunar was present because he was applying for the position of NBI agent.

Under this circumstance, Atty. Saunar could NOT have been the independent counsel solemnly
spoken of by our constitution. He was an applicant for a position in the NBI. Thus, it cannot be
said that his loyalty was to the confessants. In fact, he was actually employed by the NBI a few
months after. As to Januario, Saunar might have really been around to properly apprise him of his
right as reflected in the sworn statement. But the same cannot be said of Canape. He was not
properly informed of his rights. Perfunctorily informing a confessant of his rights, asking him if
he wants to avail of the services of counsel, and telling him that he could ask for counsel if he so
desires or that one could be provided him at his request, are not in compliance with the
constitutional mandate. Canape was merely told of his rights and posthaste asked if he was willing
to confess. His affirmative answer may not, by any menas, be interpreted as a waiver of his right
to counsel of his own choice.
The right of a person under custodial investigation to be informed of his right to remain silent and
to counsel implies a correlative obligation of the police investigator to explain and to contemplate
an effective communication that results in an understanding of what is conveyed. Canape’s
sworn statement fails to reflect compliance with this requirement.

An admission which, under Section 26 of Rule 130 of the Rules of Court, is an "act, declaration
or omission of a party as to a relevant fact" is different from a confession which, in turn, is defined
in Section 33 of the same Rule as the "declaration of an accused acknowledging his guilt of the
offense charged, or of any offense necessarily included therein." In confession, there is an
acknowledgment of guilt while in admission, the statements of fact by the accused do not directly
involve an acknowledgment of guilt.

Also, according to Atty. Saunar, when he acceded to be the custodial investigation counsel of
appellants, the latter had already confessed verbally. Before the sworn statements, appellants
already made verbal admissions of complicity in the crime. Verbal admissions, however, should
be made also with assistance of counsel.
We have not only constitutionalized the Miranda warnings, but we have also adopted the
exclusionary rules known as the fruit of the poisonous tree, a phrase minted by Justice Felix
Frankfurter in the case of Nardone v. US. According to this rule, once the primary source (tree)
is shown to have been unlawfully obtained, any secondary or derivative evidence (fruit)
derived from it is also inadmissible. In other words, illegally seized evidence is obtained as a
direct result of the illegal act, while the “fruit of the poisonous tree” is the indirect result of the
same illegal act. This fruit is at least once removed from the illegally seized evidence, but is
equally inadmissible. Evidence illegally obtained by the state should not be used to gain other
evidence because the originally illegally obtained evidence taints all evidence subsequently
obtained.

Without the sworn statements, the remaining prosecution evidence, mostly hearsay testimony and
investigation reports, is inadequate. Appellants were acquitted.

157. People v. Pablito Andan, GR 116437, March 03, 1997, Per Curiam (Custodial
Investigation)
FACTS:
Marianne Guevarra, 20 yo and a 2nd year nursing student, left her home for her school dormitory
in Valenzuela, Manila. She was walking along the subdivision when Andan invited her inside his
house. He used the pretext that the blood pressure of his wife’s grandmother should be taken.
Marianne agreed to take her blood pressure as the old woman was her distant relative. She did not
know that nobody was inside the house. Andan punched her in the abdomen, brought her to the
kitchen and raped her. After, he dragged the unconscious girl to an old toilet at the back of the
house and left her there until dark. Night came, Andan pulled Marianne, still unconscious, to their
backyard. When the girl moved, he hit her head with a concrete block. He heard her moan and hit
her again on the face. After silence reigned, he dragged the body towards a shallow portion of the
lot and abandoned it.
11am the following day, February 20, 1994, the body was discovered. She was naked from the
chest down with her brassiere and T-shirt pulled toward her neck. Nearby was found a panty and
napkin. The autopsy revealed that Marianne died of traumatic injuries.

The police asked the occupants of the nearby house and learned from Calma, stepbrother of
Andan’s wife, that Andan also lived there but he, his wife and son left without a word. The police
tried to locate Andan and found him in his parents’ house. He was interrogated in the police
headquarters. He at first denied any knowledge of Marianne’s death. But when confronted with
the concrete block and the victim’s clothes, he relented and said that his neighbors, Gilbert Larin
and Reynaldo Dizon, killed Marianne and he was merely a lookout. He said he knew where Larin
and Dizon hid Marianne’s bag. Larin, Dizon (earlier rounded up by police), and Andan were
brought to Andan’s house. Andan went to an old toilet at the back of the house and retrieved from
a canal 2 bags of Marianne. Photographs of Andan and Larin and Dizon holding the bags were
taken.

The three suspects were brought to police headquarters. A physical examination was conducted on
them. Andan was found with multiple scratches on the neck, chest, and back. While there, media
were already gathered at the police headquarters. Mayor Trinidad went to the investigation room.
Andan whispered a request that they talk privately. The mayor led Andan to the office of the Chief
of Police and there, Andan broke down and said “Mayor, patawarin mo ako! I will tell you the
truth. I am the one who killed Marianne.” The mayor opened the door of the room to let the
public and media witness the confession. The mayor first asked for a lawyer to assist Andan but
since no lawyer was available, he ordered the proceedings photographed and videotaped. In the
presence of the mayor, the police, and media representatives, and Andan’s wife and son, Andan
confessed his guilt. He asked forgiveness from Larin and Dizon whom he false implicated saying
he did it because of ill-feelings against them. His confession was captured on videotape and
covered by media nationwide.

Andan was detained at police headquarters. The next two days, more media reporters came and
interviewed Andan. He affirmed his confession to the mayor and reenacted the crime. But he
pleaded not guilty on arraignment. He claims that police brought him to a hotel and covered his
face with a bedsheet and kicked him repeatedly to coerce him to confess that he raped Marianne.
When he refused, they pushed his head into a toilet bowl and injected something into his buttocks.
Weakened, Andan confessed to the crime.

The RTC convicted him of rape with homicide and sentenced him to death. The decision was based
on testimonies of the 3 investigating policemen, the mayor of Baliuag, and 4 news reporters to
whom Andan gave his extrajudicial oral confessions. It was also based on photos and videos of
Andan’s confessions and reenactments of the commission of the crime. Hence this automatic
review.

ISSUE:
Whether the spontaneous confessions made by Andan to the Mayor and to news reporters without
interrogation and prodding are admissible in evidence.
HELD: YES.
Andan assails the admission of the police testimonies, the mayor, and the news reporters as they
were made during custodial investigation without assistance of counsel.

Any person under investigation for the commission of an offense shall have the right (1) to remain
silent; (2) to have competent and independent counsel preferably of his own choice; and (3) to be
informed of such rights. These rights cannot be waived except in writing and in the presence of
counsel. The rights in Sec.12, Art.III are accorded to any person under investigation for the
commission of an offense. An investigation begins when it is no longer a general inquiry into an
unsolved crime but starts to focus on a particular person as a suspect, i.e. when the investigator
starts interrogating or exacting a confession from the suspect in connection with an alleged offense.
This includes investigations by the municipal police, the PC and the NBI and such other police
agencies in our government.

When the police arrested appellant, they were no longer engaged in a general inquiry about
Marianne’s death. Andan was already a prime suspect even before the police found him at his
parent’s house. He was already under custodial investigation when he confessed to police, who
failed to inform him of his rights. His confession is thus inadmissible. So too were the two bags
recovered from his house. The bags were the fruits of Andan’s uncounseled confession. They
are tainted evidence, hence also inadmissible.

Andan was detained after his initial confession. The following day, Mayor Trinidad visited him.
Andan requested for a private talk. They went inside a room and Andan confessed and pleaded for
forgiveness. Mayor Trinidad stopped him, then opened the door to let the media see the confession,
and he confessed again.

The confession before the mayor is not inadmissible. It is true that a municipal mayor has
“operational supervision and control” over local police and may arguably be deemed a law
enforcement officer for purposes of Sec.12 (1) and (3), Art.III. But Andan’s confession to the
mayor was not made in response to any interrogation by the latter. The mayor did not question
Andan at all. No police authority ordered him to talk to the mayor. It was Andan himself who
spontaneously, freely, and voluntarily sought the mayor for a private meeting. When Andan
talked with the mayor as confidant and not as a law enforcement officer, his uncounseled
confession did not violate his rights. The constitutional procedures on custodial investigation do
NOT apply to a SPONTANEOUS statement, not elicited thru questioning by the authorities,
but given in an ordinary manner whereby appellant orally admitted having committed the
crime. What the constitution bars is the compulsory disclosure of incriminating facts or
confessions. The rights in S12 are guaranteed to preclude the slightest use of coercion by the state
as would lead the accused to admit something false, not to prevent him from freely and voluntarily
telling the truth.

The confessions to the media were also properly admitted as they were made in response to
questions by NEWS REPORTERS, NOT by the POLICE or any other investigating officer.
Statements spontaneously made by a suspect to news reporters on a televised interview are deemed
voluntary and admissible. The news reporters were acting as news reporters when they interviewed
Andan. They were not acting under the direction and control of the police. They were there to
check Andan’s confession to the mayor. They did not force Andan to grant them an interview and
reenact the commission of the crime. They even asked his permission before interviewing him. He
repeatedly confessed and even supplied all the details in the commission of the crime and
consented to its reenactment.

His verbal confessions to the newsmen are not covered by S12 (1) and (3). The BoR does not
concern itself with the relation between a PRIVATE individual and another individual. It
governs the relationship between the individual and the state. Governmental power is not
unlimited and the BoR lays down limitations to protect the individual.

158. Juanita Aquino v. Teresita Paiste, GR 147782, June 25, 2008, Velasco, Jr., J. (Custodial
Investigation)
FACTS:
Aquino, Elizabeth Garganta, and Adeling went to the house of Paiste in Tondo. The children of
Aquino and Paiste were grade school classmates. Aquino convinced Paiste to buy a gold bar owned
by a certain Arnold, an Igorot. She was told it was genuine. But Paiste told the three that she had
no money. The following day, Aquino and Garganta went back to Paiste’s house and convinced
her to go with them to meet Arnold and see the gold bar. They met Arnold who showed them the
gold bar. He told Paiste that it was P60k. Paiste told them that she had no money. Aquino continued
to press her that buying the gold bar would be good investment. The following day again on March
16, 1991, Aquino, Garganta, and Adeling returned to Paiste’s house, but failed to convince her to
buy the gold bar.

The three returned again the next day, telling Paiste that the price was reduced to P10k. Paiste
agreed to go with them and meet Arnold. Arnold pretended to refuse the P10k and insisted on
P50k. On Aquino’s insistence, the two bought the gold bar for P50k. Paiste had the gold bar tested
and was informed that it was fake. Aquino said that they had to see Garganta and that she had
nothing to do with the transaction.

Paiste brought Aquino to the NBI-NCR in the presence of a certain Atty. Tolentino where Aquino
amicably promised Paiste that they would locate Garganta. They signed an amicable settlement
where Aquino promised to pay Paiste P25k in monthly installments of P1k. Aquino also waived
her right to counsel “despite recital of her constitutional rights made by NBI agent Ely
Tolentino.”

On April 6, 1991, Aquino brought Garganta to Paiste’s house. In the presence of the barangay
chairperson and a police officer, Paiste pointed to Garganta as the person who sold the fake gold
bar. Garganta was brought to the police station.

Paiste filed a criminal complaint for estafa against Garganta, Aquino, and 3 others. Only Aquino
was arrested as the others remained at large. The RTC convicted Aquino of estafa, finding that
Aquino conspired with Garganta, Adeling, and Arnold. It also gave credence to the amicable
settlement as additional proof of Aquino’s guilt as an amicable settlement in criminal cases is an
implied admission of guilt. The CA affirmed the conviction.

ISSUE:
Whether an amicable settlement signed by accused Aquino after being assigned a counsel and
conferring with said counsel, which settlement was an acceptance of fault in the estafa case filed
by Paiste, where Aquino agreed to pay the amount defrauded in installments, and where Aquino
waived right to counsel, is admissible in evidence.
HELD:
Aquino claims that the amicable settlement with waiver of right to counsel was executed by her
under threat and not freely, inviolation of S12 (1), Art.III of the constitution.

Custodial investigation involves any questioning initiated by law enforcement officers after a
person has been taken into custody or otherwise deprived of his freedom of action in any significant
way. It is only after the investigation ceases to be a general inquiry into an unsolved crime and
begins to focus on a particular suspect, the suspect is taken into custody, and the police carries out
a process of interrogations that lend itself to eliciting incriminating statements, that the rule begins
to operate. RA 7438 extended this constitutional guarantee to situations where the person is not
formally arrested but merely “invited” for questioning. It provides that “custodial investigation”
includes the practice of issuing an invitation to a person who is investigated in connection with an
offense he is suspected to have committed.

When Aquino was brought by Paiste before the NBI to be investigated, she was already under
custodial investigation and her rights under the Miranda rule has set in. she was provided with a
lawyer, Atty. Uy. But the custodial investigation on the inquiry or investigation for the crime was
aborted as the parties, Aquino and Paiste, agreed to amicably settle.

Her contention that her rights were breached as she signed under duress falls flat. She was provided
with counsel. The presumption that Atty. Uy is a competent and independent counsel whose
interests are not averse to Aquino has not been overturned. Second, Aquino never objected to
Atty. Uy’s appointment. When accused never objected against the lawyer’s appointment during
the course of the investigation and the accused thereafter subscribes to the veracity of his statement
before the swearing officer, he is deemed to have engaged such lawyer. Aquino is deemed to
have engaged Atty. Uy when she conferred with him and thereafter signed the amicable settlement
with waiver of right to counsel in his presence.

Third, when Aquino engaged Uy, she executed the amicable settlement. The right to counsel is
intended to preclude the slightest coercion as would lead the accused to admit something false.
The lawyer should never prevent an accused from freely and voluntarily telling the truth. An
amicable settlement is not an extrajudicial confession or admission but is a contract between the
parties. Atty. Uy’s presence safeguarded Aquino’s rights even if the custodial investigation did
not push through and precluded any violence, coercion, or intimidation. Miranda rights only render
inadmissible extrajudicial confession or admission made during custodial investigation.

Lastly, granting arguendo that the amicable settlement is an admission, the document would still
be admissible since none of her constitutional rights were violated. Aquino’s allegations of duress
are bare allegations without proof.

159. Josue Ladiana v. People, GR 144293, December 04, 2002, Panganiban, J. (Custodial
Investigation)
FACTS:
Caridad San Juan, wife of Francisco San Juan, victim, testified that Francisco was the brgy.
Captain until he was shot and killed by Ladiana. She was in her house on Dec.29, 1989 when an
unidentified woman told her that her husband was killed by Ladiana she rushed to Jacinto Street
where the incident happened. Many people were milling around and she saw Francisco’s body.
The woman told her that Ladiana killed Francisco. Ladiana also gave himself up to police. She
said that Francisco was with some persons who all went to Jacinto street to repair the steel humps
used to block the street during school days for safety of school children.

Before retired assistant prosecutor of Laguna Cortez testified, the defense admitted to the
voluntariness and authorship of the counter-affidavit of Ladiana, which was subscribed and
sworn to before Cortez in his preliminary investigation. In the affidavit, Ladiana admitted to
making the fatal shots on Francisco. But Ladiana allegedly did so in self-defense as Francisco
was purportedly attacking Ladiana and had already inflicted a stab wound on his arm.

The Sandiganbayan convicted Ladiana. It held that the counter-affidavit with the admission may
be used as evidence against him. But it found no evidence of treachery, thus it convicted him of
homicide only.

ISSUE:
Whether Ladiana, accused of murder, who filed a counter-affidavit without assistance of counsel
during his preliminary investigation and therein admitting his shooting of Francisco, may be
convicted based on such admission.
HELD: YES.
Ladiana argues that no counsel was present when his counter-affidavit was executed. He cites
Section 12(1) and (3), Art.III of the constitution.

S12 applies only to extra-judicial confessions or admissions during custodial investigations.


Here, Ladiana admits that the statements were made during preliminary investigation, not
during custodial investigation. He argues that right to counsel applies during preliminary
investigation.

We disagree. A preliminary investigation is an n inquiry or a proceeding to determine whether


there is sufficient ground to engender a well-founded belief that a crime has been committed, and
that the respondent is probably guilty thereof and should be held for trial. This is not custodial
investigation. An accused’s interrogation by police, if any, would have already ended at the time
of the filing of the criminal case in court or the prosecutor’s office. Thus, even without counsel,
the admissions in the counter-affidavit do not violate Ladiana’s constitutional rights. It was not
exacted by police while he was under custody or interrogation.

But the accused, whether in court or undergoing preliminary investigation, has 1) the right to
refuse to be made witnesses, 2) the right not to have any prejudice whatsoever imputed to them
by such refusal, 3) the right to testify on their own behalf, subject to cross-examination by the
prosecution, and 4) while testifying, the right to refuse to answer a specific question that tends
to incriminate them for some crime other than that for which they are being prosecuted.
The counter-affidavit is not an extrajudicial confession, only an admission. In confession, there is
acknowledgment of guilt. In admission, there is merely a statement of fact not directly involving
acknowledgment of guilt or of the criminal intent to commit the offense with which one is charged.
The counter-affidavit is an admission that Ladiana shot the victim when the latter was attacking
him. He denies criminal intent, claiming self-defense. Nonetheless, whether characterized as
admission or confession, it is admissible in evidence.

We do not doubt the voluntariness of the counter affidavit as Ladiana himself submitted it to the
prosecutor to justify his actions. The defense failed to prove self-defense. Thus, he is convicted.

160. Christopher Gamboa v. Hon. Alfredo Cruz, GR L-56291, June 27, 1988, Padilla, J.
(Custodial Investigation)
FACTS:
Gamboa was arrested for vagrancy without a warrant of arrest by patrolman Palencia. He was
brought to precint 2, Manila, where he was booked for vagrancy and therein detained with others.
The next day, during the lineup of 5 detainees including Gamboa, complainant Bernal pointed to
Gamboa and said “That one is a companion.” After the identification, the other detainees were
brought back to their cell but Gamboa was ordered to stay on. While Bernal was being interrogated
by the police investigator, Gamboa was told to sit in front of her.

An information for robbery was filed against Gamboa. Upon hearing, Gamboa filed a motion to
acquit or demurrer to evidence on the ground that the line-up, without notice to, and in the
absence of, his counsel violated his constitutional rights to counsel and due process. The trial court
denied the motion to acquit. Hence this petition.

ISSUE:
Whether Gamboa, who was arrested for vagrancy, detained, brought to a police line-up where
complainant Bernal pointed to him saying that he was a companion in a robbery, where Gamboa
was made to sit in front of Bernal while the police investigated Bernal, was denied his right to
counsel.
HELD: NO.
The right to counsel is found in Sec.20, Art.IV of the 1973 constitution or Sec.12 (1,2, &3) of
Art.III of the 1987 constitution. The right to counsel attaches upon the start of an investigation,
i.e. when the investigating officer starts to ask questions to elicit information and/or confessions
or admissions from the respondent/accused. No custodial investigation shall be conducted unless
it be in the presence of counsel, engaged by the person arrested, or by any person in his behalf, or
appointed by the court upon petition either of the detainee himself or by anyone in his behalf, and
the waiver of the right shall not be valid unless made in writing and in the presence of counsel.

But the police line-up (at least in this case) was NOT part of the custodial inquest, hence
Gamboa was not yet entitled to counsel.

In Kirby v. Illinois, petitioner and his companion were arrested and found to be in possession of
items stolen in a recent robbery. The robbery victim was brought to the police station and identified
the petitioner and companion as the robbers. No attorney was present nor were petitioner and his
companion advised of the right to counsel. Petitioner and the companion were indicted for robbery
and convicted. The US SC held that the right to counsel did not attach until judicial criminal
proceedings were initiated. The right attaches at the time of arraignment.

In PH, the 1973 and 1987 constitutions go beyond the rights under the 6th and 14th amendments.
Under our constitutions, the right to counsel attaches at the start of investigation against a
respondent and thus even before adversary judicial proceedings against the accused have begun.
While the Court finds no real need to afford a suspect the services of counsel during a police line-
up, the moment there is a move or even an urge of said investigators to ELICIT admissions or
confessions, or even plain information which may appear innocent or innocuous at the time,
from said suspect, he should then and there be assisted by counsel, unless he waives the right,
which must be made in writing and in the presence of counsel.

Gamboa was not deprived of due process as he was duly represented by a member of the Bar. He
had opportunities to be heard and to present evidence and substantiate his defense; only that he
chose not to and instead opted to move to acquit after the prosecution rested its case. What due
process abhors is the absolute lack of opportunity to be heard. This case is far from this situation.

161. People v. Valeriano Amestuzo, GR 104383, July 12, 2001, Kapunan, J. (Custodial
Investigations)
FACTS:
8 armed men wearing masks entered the house of Perlita Lacsamana and robbed it of valuables
worth P728k. 2 members of the gang raped Maria Catanyag and Estrella Rolago, niece and
employee of Perlita. Amestuzo and Bagas, among others, was charged with robbery in band with
double rape.

The RTC convicted Amestuzo and Bagas. He appealed to the SC, alleging 1) deprivation of his
constitutional right to be represented by counsel during his identification, 2) the trial court’s error
in giving due weight to the open court identification of him which was based on a suggestive and
irregular out-of-court identification, and 3) the trial court’s improper rejection of his alibi. He
narrates the circumstances of his arrest.

Four days after the alleged crime, a group of policemen, together with accused Ampatin who was
a suspect, went to the factory in Pasay where Bagas was working. They were looking for a certain
“Mario” and searched the first and second floors of the building. Failing to find Mario, the police
hit Ampatin and uttered, “Niloloko lang yata tayo ng taong ito” and “Magturo ka ng kahit sino.”
Thus, Ampatin pointed to Bagas as he was the first person Ampatin chanced to look upon. Bagas
was arrested. Ampatin, aboard the police vehicle, told Bagas that he (Ampatin) committed an error
in pointing him out to police. They were placed under detention.

When the complainants arrived, Bagas was brought out, instructed to turn to the left and right and
to talk. Complainant Lacsamana asked him if he knew accused Amestuzo and Viñas, he said no.
the police told the complainants that Bagas was one of the suspects, which incited complainants
to an emotional frenzy, kicking and hitting him.

ISSUE:
Whether the identification of Bagas by complainants Lacsamana et al. who were victims of robbery
in band with double rape after the police took Bagas out of his detention cell and presented him
alone to complainants who asked him if he knew the other accused, to which he replied that he did
not, but where the complainants were induced into an emotional frenzy and started kicking and
hitting him after the police told complainants that he was one of the suspects may be admissible in
evidence.
HELD: NO.
1. Right to counsel.
Bagas claims that he was deprived of his right to counsel during his investigation. His single
presentation to complainants for identification without counsel is a violation of his right to counsel.
His identification was a critical stage of prosecution at which he was entitled to counsel.

The guarantees of Section 12(1), Art.III of the Constition, so-called Miranda rights, may be
invoked only by a person while he is under custodial investigation. This starts when the police
investigation is no longer a general inquiry into an unsolved crime but has begun to focus on
a particular suspect taken into custody by the police who starts the interrogation of the
person to elicit incriminating statements. Police line-up is not part of custodial investigation.
Hence, the right to counsel cannot be invoked at this stage. This is because during a police line-
up, the process has not yet shifted from the investigatory to the accusatory and it is usually the
witness or complainant who is interrogated and who gives a statement in the course of the line-up.

There was also no showing that during his identification, the police sought to elicit any admission
or confession from him. Alleged infringement of the rights of accused under custodial
investigation is relevant only in cases in which an extrajudicial admission or confession
extracted from the accused becomes the basis of his conviction.

2. Identification irregularity
Bagas claims that the manner he was presented to complainants for identification was irregular as
he was not placed in a police line-up but made to stand before complainants alone.

There is no law requiring a police line-up as essential to proper identification. The fact that he was
brought out of the detention cell alone before the complainants does not detract from the validity
of the identification process.

But the out-of-court identification of Bagas was seriously flawed as to preclude its admissibility.
In admissibility and reliability of out-of-court identifications, we apply the totality of
circumstances test which lists the following factors:
(1) the witness' opportunity to view the criminal at the time of the crime; (2) the witness'
degree of attention at that time; (3) the accuracy of any prior description given by the
witness; (4) the level of certainty demonstrated by the witness at the identification; (5)
the length of time between the crime and the identification; and (6) the suggestiveness
of the identification process.

The out-of-court identification of Bagas by complainants is improperly suggestive. Even before


complainants had opportunity to view Bagas’ face, the police announced that he was one of the
suspects and he was pointed to by Ampatin. The identification was practically suggested by the
police. The fact that this information came to the knowledge of complainants prior to their
identification based on their own recall of the incident detracts from the spontaneity of their
subsequent identification and thus its objectivity.

3. Defense of alibi.
The trial court erroneously rejected Bagas’ alibi. Bagas clearly and positively testified that at the
time of the crime, he was working as a shell cutter in a factory in Pasay. Four days later, he was
arrested when Ampatin randomly pointed him out to police. This testimony of Bagas was
materially corroborated by two of his co-employees who were with him on the night of the crime
and his employer. The defense of alibi assumes strength when it is amply corroborated by a
credible witness. To be given weight, accused must prove not only that he was somewhere else
during the crime, but it was physically impossible for him to be present at the crime or its vicinity
during commission.

As impartial credible witnesses, Bagas’ co-employees and employer cannot be doubted without
showing of undue bias or prejudice. It was also testified that Bagas worked in the factory until
10pm and went to sleep after while the crime was committed 930pm. There was only one door in
the factory as the only means of entrance and exit and this was locked by the employer after 10pm.
There was no possible exit through the windows of the building. The crime took place in Kalookan
city, while Bagsa’ place of work was in Pasay.

Ampatin’s declaration in court that he does not know Bagas and merely pointed to him out of fear
of the police is corroborated. Thus, Bagas was acquitted.

162. People v. Anthony Escordial, GR 138934-35, January 16, 2002, Mendoza, J. (Custodial
Investigations)
FACTS:
Erma was awakened by the presence of a man who had his head covered with a t-shirt and carried
a knife. He warned Erma not to shout or he would kill her. He got the money of Erma and Michelle.

The man gave a t-shirt to Erma to blindfold Teresa and another to Michelle to blindfold Erma. He
blindfolded Michelle himself and began touching her in different parts of her body. He raped
Michelle.

PO3 Tancinco, one of the policemen who responded to the report after the crime, questioned
certain people about Escordial and found out that he was a helper of Hinolan, owner of Coffee
Break Corner. Hinolan said that Escordial went home on December 27, 1996, day of the crime, to
Brgy. Miranda, Pontevedra. Tancinco found Escordial at the basketball court and “invited” him to
go to the police station for questioning.

Michelle remained at the police station. When Escordial was brought there, he saw Michelle and
blushed. She recognized him as the man who robbed and raped her.

Erma, Teresa, Joniega, and Esmeral were asked to identify Escordial from four people inside a jail
cell. They picked Escordial.
As to the circumstances of arrest, police, led by PO3 Tancinco, went to Pontevedra police to ask
for help in locating Escordial, wanted for robbery with rape. Although Tancinco and company had
a mission order, they did not have a warrant of arrest. They found Escordial at the basketball court
watching a game. After informing him that he was a suspect, the group invited him to go with them
to the police headquarters.

Brgy. Captain Dojilo testified that he was at the police station and followed Tancinco and Escordial
to the investigating room. Michelle said that she could identify the attacker if she could see a lump
on his back. Michelle, after accused took off his shirt, said she was not sure because the attacker
was wearing a mask. Tancinco requested to be allowed to bring accused to Bacolod as they still
had some witnesses there who could identify the suspect. Accused was allowed to go with them,
but the Pontevedra police asked the Bacolod police Tancinco not to harm him.

In Bacolod, two of the complainants arrived and the police asked them to identify him, but they
just kept looking and they even asked the police if he was the suspect.

The RTC convicted Escordial. Hence this appeal.


ISSUE:
Whether Escordial may be convicted of robbery with rape based on the testimonies of witnesses
who have identified him in a show-up and line-up but claim 1) that at the time of rape, she was
blindfolded and could only feel keloids on the back of the assailant but Escordial has no such
keloid and 2) that she saw the appellant at the time of rape through her blindfold but without having
immediately reported his identity immediately and doing so only a week after the crime.
HELD: NO.
1. Warrantless arrest.
Escordial questions the legality of his arrest without warrant. The cases at bar do not fall under
Rule 113, Section 5(a) or (c). Escordial was watching a game when arrested in Brgy. Miranda. He
was not committing or attempting to commit a crime when arrested nor was he an escaped
prisoner.

Under Sec.5(b), “personal knowledge” must be based upon probable cause, which means “an
actual belief or reasonable grounds of suspicion.” The grounds of suspicion are reasonable
when, in the absence of actual belief of the arresting officers, the suspicion that the person to be
arrested is probably guilty is based on actual facts.

The crime took place on December 27, 1996, but Escordial was arrested only on January 3, 1997.
As the officers were not present when the crime was committed, they could not have “personal
knowledge.” The officers had no reason for not securing a warrant.

But Escordial pleaded not guilty to the crimes without questioning his arrest. Thus, he waived
objection to the legality of his arrest. Any defect in the arrest is deemed cured when he voluntarily
submitted to the jurisdiction of the court, for the legality of an arrest only affects jurisdiction of
the court over the person of the accused.

2. Escordial invokes Article II, Sec. 12(1). He claims that he was subjected to custodial
interrogation without being informed of his right to remain silent and have independent counsel.
While it cannot be denied that he was deprived of his right to be informed of his rights to remain
silent and to have competent and independent counsel, he has not shown that, as a result of his
custodial interrogation, the police obtained any statement from him, whether inculpatory or
exculpatory, which was used in evidence against him. No uncounseled statement was obtained
from him which should have been excluded as evidence against him.

3. Right to counsel.
An accused is not entitled to assistance of counsel in a police line-up since it is not part of
custodial inquest. But Escordial had already been under custodial investigation when the out-
of-court identifications were conducted by the police.

An out-of-court identification can be made in various ways. In a show-up, the accused alone is
brought face to face with the witness for identification. In a police line-up, the suspect is identified
by a witness from a group of persons. During custodial investigation, these identifications are
“critical confrontations of the accused by the prosecution” which necessitate presence of
counsel for the accused since the results of these pre-trial proceedings might settle the accused’s
fate and reduce trial to a mere formality. Any identification of an uncounseled accused made in a
police line-up or show-up after the start of custodial investigation is inadmissible in evidence.

Escordial was identified by Michelle in a show-up on January 3 1997 and by Erma, Teresa,
Joniega, and Esmeralda in a police line-up on various dates. Since accused had no counsel, these
out-of-court identifications are inadmissible for being the direct result of the illegal line-up.

But the inadmissibility of these out-of-court identifications does not render in-court identification
of accused inadmissible.

4. Credibility of prosecution witness.


Joniega and Esmeralda pointed to Escordial as the man they saw on the night of December 27 and
the person they identified inside a jail cell. Erma testified that she saw through her blindfold
Escordial raping Michelle and identified him in court.

The test is whether the prosecution was able to establish by clear and convincing evidence that
the in-court identifications were based upon observations of the suspect other than the line
up identification. This test considers various factors like prior opportunity to observe the alleged
criminal act, existence of any pre-line-up description and defendant’s actual description, any
identification prior to lineup of another person, identification by picture prior to lineup, failure to
identify the defendant on a prior occasion, and lapse of time between the alleged act and lineup
identification.

A. A show-up, as undertaken herein by police in identifying Escordial by Michelle, has been held
to be an underhanded mode of identification for “being pointedly suggestive, generating
confidence where there was none, activating visual imagination, and subverting their
reliability as an eyewitness.” Michelle knew she was going to identify a suspect. Upon seeing
Escordial escorted by police, she knew that he was the suspect she was supposed to identify. When
Escordial was shown to her, there was thus no doubt of what was expected of her. By arranging a
lineup, police evince their belief that they have caught the criminal. Witnesses, realizing this, will
feel foolish if they cannot identify anyone and may thus choose someone despite residual
uncertainty.

The failure of Michelle to see the face of the assailant and the apparent suggestiveness of the show-
up places in doubt her credibility. The possibility that her identification of Escordial was merely
planted in her mind cannot be disregarded. Michelle’s affidavit also indicated that she felt keloids
on the back of the assailant but testified that she did not see keloids on Escordial although she said
that his skin was rough.

B. Erma testified that she saw through her blindfold the assailant raping Michelle and identified
Escordial in open court. But the police blotter dated December 28, 1996 referred to an “unknown
suspect.” Also, when the police went to Pontevedra for Escordial, it was Michelle, who admitted
that she did not see the assailant’s face, and not Erma, who said she recognized Escordial, that was
taken along. Why did they take Michelle instead of Erma?

The affidavit of Erma was prepared on January 4, 1997, a day after the arrest of Escordial. This
delay belies her claim that she saw the assailant through her blindfold, for the normal reaction of
one who actually witnessed a crime and recognized the offender is to reveal it to authorities at the
earliest opportunity.

C. Escordial’s testimony that he was in Brgy. Miranda is corroborated by 3 people. Considering


the uncertainties of the testimonies of the prosecution witnesses, the defense of alibi by Escordial
deserves credence.

163. People v. Guillermo Samus, GR 135957-58, September 17, 2002, Panganiban, J.


(Custodial Investigation)
FACTS:
Samus was a farmer in Laguna. The murder victims, 62yo Dedicacion Balisi and her grandson 6yo
John Ardee Balisi were the neighbors of Samus’ father. SP Garcia went to investigate their muder
when the police received a telephone call about the victims’ deaths. After investigation and
autopsy, John was found to have suffered 3 contusions caused by a blunt instrument and
Dedicacion 4. A pair of earrings worn by Dedicacion was reported missing from her body by her
daughter.

On the same day as the investigation, September 2, 1996, Pontanos, acquaintance of Samus, met
Samus at Pacheca’s house where Samus asked Pontanos to accompany him to Pontanos’ wife to
pawn a pair of earrings. On September 10, Major Pante received information that Samus was the
principal suspect in the killing of the 2 victims and that he was in the house of spouses Rolly and
Josie Vallejo. Upon arrival at said house, the police team asked permission to enter the house,
which was granted. The team heard loud footsteps on the roof. Rushing outside, they saw Samus
crawling on the roof. They ordered him to stop, but he suddenly jumped from the roof and landed
hard on the ground, injuring his ankle and forearms. The team closed in on Samus who, while
trembling and shaking, admitted the killings upon a query from Rolly Vallejo.
Samus was brought to a PNP camp where he was informed of his rights. Assisted by Atty. Juliano,
he gave his statement admitting the killings. The pawned earrings were also recovered.

-Samus’ version
Samus denied the accusations and said that on September 2, from 6am-5pm, he never left his farm
and was harvesting palay. On September 10, Samus said that the police had no warrant. He was
brought to Camp Vicente Lim where he was tortured. He was brought to the hospital, given
medicine, then brought back to the cell. He was forced by the police to admit to the killing and the
sale of the earring thru torture and threat. He was forced to execute a document admitting the
killing and he did not know Atty. Juliano or talk to him.

The RTC found enough circumstantial evidence and convicted him of homicide for Dedicacion’s
death and murder with the aggravating circumstance of dwelling for John’s. Hence this automatic
review.

ISSUE:
Whether the admissions of Samus under custodial investigation without counsel may be admitted
despite not being objected to timely during trial.
HELD: NO.
1. The police’s version of the arrest is incredible and contrary to human experience. We find it
hard to believe that anyone would jump from the roof of a 2-story house to escape, and after
landing without broken bones, make a complete turnaround and just meekly surrender without
further ado. Even if true, jumping from a roof is not a crime to justify the warrantless arrest. None
of the circumstances for valid warrantless arrests was present. He was not a prisoner. The killing
was not done in the presence of the arresting officers. It took place on September 2 and cannot be
considered as “having just been committed.” The arrest was unlawful.

The police claim that it was Rolly Vallejo who had asked if Samus killed the victims and to this
question, Samus allegedly answered “Totoo nga pare, ako nga,” without further questions from
the officers. Thus they contend that Samus’ rights were not violated since they were not the ones
who had elicited evidentiary matters from him. But we are not persuaded. The events narrated by
the officers are too good to be true and their statements a day after the arrest contradict their
testimonies and raise doubts on their credibility. We find the version of Samus more credible that
he had been arrested inside Vallejo’s house with Rolly not being around.

2. After his illegal arrest, Samus was not informed of his rights to remain silent and to have
competent and independent counsel. Hence, any admission elicited from him by the officers during
custodial investigation are normally inadmissible in evidence. The officers’ testimonies reveal
their crude attempts to camouflage inadmissible evidence. Casis said that upon Samus’ being
brought to Camp Vicente Lim, he was turned over to SPO3 Malabanan. But SPO3 Bitos said that
the media interview could not be prevented because it was an ambush interview. SPO3 Malabanan
then claims that when he arrived, there were already reporters questioning Samus.

Without testimony from the media persons who allegedly interviewed Samus, the uncertainties
and vagueness about how they questioned Samus and led him to his confession lead us to believe
that they themselves investigated Samus and elicited uncounseled admissions.
Nonetheless, even if the uncounseled admission per se may be inadmissible, under the present
circumsstances, we cannot rule it out because of Samus’ failure to make TIMELY
OBJECTIONS. The evidence is inadmissible as it was given under custodial investigation and
made without assistance of counsel. But the defense failed to object to its presentation during
the trial, with the result that the defense is deemed to have WAIVED objection to its
admissibility. Having made no objection before the trial court, Samus cannot raise this question
for the first time on appeal. If only Samus made a timely objection, the prosecution could have
been warned of the need to present additional evidence to support its case. To disregard
unceremoniously a major portion of its case at this late stage when it can no longer present
additional evidence as substitute for that which is now claimed inadmissible goes against
fundamental fairness.

3. The following circumstantial evidence are presented: 1) finger and palm prints matching Samus’
found near the bloodstains at the scene of the crime, 2) the earrings of Dedicacion were missing
from her body which Samus pawned, and 3) Samus admitting the killing. The prints lead to no
other reasonable conclusion except that Samus was in the house in the afternoon when the victim
died. The pawning of the earrings the same afternoon of Dedicacion’s death is consistent with and
further supports the conclusion that he was at the crime scene at the time of killing. The only
rational hypothesis that can be drawn from these is that Samus is guilty of the killing.

4. For the death of Dedicacion- homicide only, since none of the qualifying circumstances alleged
was proven. For John’s death- murder since there was treachery (6yo). There is no aggravating
circumstance of dwelling since it was not alleged although proven.

Sec.13- Right to Bail


164. People v. Hon. Procoro Donato, Rodolfo Salas GR 79269, June 05, 1991, Davide, Jr., J.
(Right to Bail)
FACTS:
Private respondent Rodolfo Salas, “Commander Bilog”, and his co-accused were charged for
rebellion under Art.134 of the RPC. Salas and his co-accused were in military custody at the time
the information was filed following their arrest on Sept 29, 1986 at PGH Taft, Manila. a petition
for habeas corpus was filed, which was dismissed by this Court based on the parties’ agreement
that Salas will “remain in legal custody and will face trial before the court having custody over
his person” and the warrants for arrest of his co-accused are deemed recalled and they shall be
immediately released but shall submit themselves to the court having jurisdiction over their person.
Salas moved to quash information. Petitioner opposed, citing that in their joint manifestation, Salas
conceded that he will remain “in legal custody and face trial before the court having custody over
his person.”

The judge denied the motion to quash. Instead of filing an MR of the order, Salas filed a petition
for bail which petitioner opposed on the ground that rebellion became a capital offense under PDs
1996, 942, and 1834, which amended Art.135 of the RPC, imposing RP to death.

On June 5, 1987, EO187 repealed the said PDs and restored to full force and effect Art.135 as it
existed before the PDs. The original penalty of PM was restored. EO187 was published in the
Official Gazette in its June 15, 1987 issue and released for circulation June 26, 1987. Judge Donato,
taking into account EO 187, granted the petition for bail, fixed bail bond at P30k, and imposed the
condition that Salas shall report to the court once every 2 months within the first 10 days of every
period thereof. He said that under EO187, rebellion is penalized with PM, making it bailable.

As to the contention of the people that it would be dangerous to grant bail to Salas considering his
stature in the CPP-NPA hierarchy and would allow him to direct its armed struggle against the
government, Donato said there is a clash between Salas’ constitutional right to bail in a non-capital
offense guaranteed by the BoR and the paramount interest of the state. In case of such conflict, the
same should be resolved in favor of the individual who, in the eyes of the law, is alone in the
assertion of his rights under the BoR as against the state. The fear that Salas may jump bail and
rejoin his comrades to sow further anarchy is not reason to deny him bail as the accused, the law
is very explicit, is entitled as matter of right to bail.

In moving for reconsideration, the people says that 1) Salas evaded the authorities for 13 years and
was an escapee when arrested, 2) he was not arrested at his residence as he had no known address,
3) he was using a false name when arrested and presented a driver’s license to substantiate the
false identity, 4) he gave a false address, 5) he and his companions were on board a private vehicle
with a declared owner whose identity and address were also found to be false. These indicate that
Salas does not entertain the slightest intention to appear in court for trial if released. When the
interest of the state conflicts with that of the individual, the former prevails. Judge Donato denied
the MR. He only increased the bail bond from P30k to P50k. Hence, this petition.

ISSUE:
1. Whether Salas may be denied bail for a non-capital offense on the ground that he might rejoin
his communist rebel allies and would most likely jump bail.
2. Whether the right to bail may and was validly waived when Salas agreed with the government
to “remain in legal custody and face trial before the court having custody over his person” to allow
his co-accused Cruz and Concepcion to be released thru the arrest warrants against them being
revoked.
HELD:
1. NO. When the information was filed, the penalty for rebellion was RP to death. EO 187 restored
Article 135, which was governing at the time the trial court resolved the petition for bail. We agree
that bail cannot be denied to Salas since rebellion has a penalty of PM and a fine not exceeding
P20k. it is thus a bailable offense under S13, Art. III of the 1987 Constitution:
"Sec. 13. All persons, except those charged with offenses punishable by reclusion perpetua
when evidence of guilt is strong, shall, before conviction, be bailable by sufficient sureties,
or be released on recognizance as may be prescribed by law. The right to bail shall not be
impaired even when the privilege of the writ of habeas corpus is suspended. Excessive bail
shall not be required."

Section 3, Rule 114 of the RoC also provides:


"Bail, a matter of right: exception. — All persons in custody shall, before final conviction,
be entitled to bail as a matter of right, except those charged with a capital offense or an
offense which, under the law at the time of its commission and at the time of the application
for bail, is punishable by reclusion perpetua, when evidence of guilt is strong."
Thus, before conviction, bail is either a MATTER OF RIGHT or of DISCRETION. It is a matter
of right when the offense charged is punishable by any penalty lower than RP. To this extent the
right is absolute. Individual freedom is too basic, too transcendental and vital in a republican state
to be derived upon mere general principles and abstract consideration of public safety. The 1987
Constitution further strengthened the right to bail by explicitly providing that it shall not be
impaired even when the privilege of habeas corpus is suspended, overturning the ruling in Garcia-
Padilla v. Enrile, et al., which said that the suspension of HC must carry with it the suspension of
the right to bail if the government’s campaign to suppress rebellion is to be rendered effective,
since those arrested may rejoin their comrades.

If the offense charged is RP, bail becomes a matter of discretion, to be denied if evidence of
guilt is strong. The court’s discretion is limited to determining whether evidence of guilt is
strong. But if it is determined that such evidence is not strong, bail also becomes a matter of right.

The prosecution does not have the right to present evidence for denial of bail where it is a matter
of right. But if discretionary, due process requires that the prosecution must be given an
opportunity to present, within a reasonable time, all evidence that it may desire to introduce before
the court should resolve the motion for bail.

But we agree with petitioner that it was error for the trial court to fix bond at P30k then later at
P50k without hearing the prosecution. The guidelines for fixing bail bond provided in S10, Rule
114 are not matters left entirely to the court’s discretion. Certain guidelines in fixing bailbond, like
the nature and circumstances of the crime, character and reputation of the accused, etc., call for
the presentation of evidence and reasonable opportunity for the prosecution to refute it. Thus,
petitioner has sufficiently made out allegations necessitating opportunity to be heard for purposes
of determining amount of bail, but not for the denial thereof because S10, Rule 114 does not
authorize any court to deny bail.

Although now, RA 6968 made rebellion punishable by RP for persons who main or head it, it
cannot apply to Salas for acts allegedly committed prior to its effectivity as it is not favorable to
him.

2. YES. But Salas has waived his right to bail. A petition for habeas corpus was filed with this
Court for Salas and his co-accused Cruz and Concepcion against Juan Ponce Enrile, et al. We
issued the writ. The OSG filed a return of the writ alleging that Salas and co were apprehended at
the PGH. The parties, at the hearing on Oct.14, 1986, informed the Court of certain agreements
between them. Salas agreed to withdraw the petition for HC with him remaining under custody
whereas his co-detainees Cruz and Concepcion will be released immediately. The SOLGEN
confirmed this and declared that no objection will be made for the release of Cruz and Concepcion,
and that no bond will be required of them, but they will continue to face trial with Salas. The
parties submitted a joint manifestation and motion, agreeing to the withdrawal of the petition for
HC, Salas remaining in custody and to face trial, and the arrest warrant against Cruz and
Concepcion being deemed recalled. Thus, the SC dismissed the petition for HC.
Petitioner claims that Salas, in agreeing to remain in legal custody even during the pendency of
the trial of his criminal case, he has expressly waived his right to bail. Salas asserts that “legal
custody” only means that Salas agreed to continue to be in the custody of the law or in custodial
egis and nothing else; it is not to be interpreted as waiver.

When the parties stipulated that “b. Petitioner Rodolfo Salas will remain in legal custody and face
trial before the court having custody over his person," they simply meant that Salas will remain in
actual physical custody of the court or in actual confinement or detention, as distinguished from
the stipulation concerning his co-petitioners, who were to be released in view of the recall of the
arrest warrant. They agreed, however, to “submit themselves to the court having jurisdiction over
their persons.” There is deliberate distinction between “legal custody” and “court having custody
over the person” in respect to Salas and “court having jurisdiction over the persons” of his co-
accused. This was intended to emphasize the agreement that Salas will not be released, but should
remain in custody. Had the parties intended otherwise, they should have insisted on the use of
clearer language.

"Custody” has been held to mean nothing less than actual imprisonment. It is also defined as the
detainer of a person by virtue of a lawful authority, or the care and possession of a thing or person.
S1, Rule 114 defines bail as “the security given for the release of a person in custody of the law.”
It admits of no other meaning for “in custody of the law” than that as above indicated. The purpose
of bail is to relieve an accused from imprisonment until his conviction and yet secure his
appearance at trial. It presupposes that the person applying for it should be in the custody of the
law or otherwise deprived of liberty.

3. Is the waiver valid? Art.6 of the NCC says that "Art. 6. Rights may be waived, unless the waiver
is contrary to law, public order, public policy, morals, or good customs, or prejudicial to a third
person with a right recognized by law."

Waiver is defined as “a voluntary and intentional relinquishment or abandonment of a known


existing legal right, advantage, benefit, claim or privilege, which except for such waiver the party
would have enjoyed; or such conduct as warrants an inference of the relinquishment of such right;
or the intentional doing of an act inconsistent with claiming it."

As to what rights and privileges may be waived, the doctrine of waiver extends to rights of any
character. Since “waiver” covers every conceivable right, the general rule is that a person may
waive any matter which affects his property, and any alienable right or privilege of which he
is the owner or which belongs to him or to which he is legally entitled, whether secured by
contract, conferred with statute, or guaranteed by constitution, provided such rights and
privileges rest in the individual, are intended for his sole benefit, do not infringe on the rights
of others, and further provided that the waiver is not forbidden by law and does not contravene
public policy. Generally, all personal rights conferred by statute and guarantted by the constitution
may be waived. Constitutional provisions intended to protect property may be waived, even those
rights created to secure personal liberty.

Rights guaranteed to one accused of a crime fall naturally into two classes: 1) those in which the
state, as well as the accused, is interested, and 2) those which are personal to the accused, which
are in the nature of personal privileges. Those of the first class cannot be waived. Those of the
second may be.

This Court has recognized waivers of constitutional rights like the right against unreasonable
searches, to counsel and to remain silent, and to be heard. Even S12, (1) Art.III of the BoR
recognizes waiver of rights: “These rights cannot be waived except in writing and in the presence
of counsel.” This provision merely particularizes the form and manner of waiver. It clearly
suggests that the other rights may be waived in some other form or manner provided such
waiver will not offend Art.6 of the NCC.

Thus, the right to bail is another right which can be waived. Judge Donato acted with grave abuse
of discretion in granting bail to Salas.

165. People v. Agustin Fortes, GR 90643, June 25, 1993, Davide, Jr., J. (Right to Bail)
FACTS:
On Nov.26, 1983, Agripino Gine accompanied his 13 yo daughter, Merelyn, to the police station
to report rape by Fortes against Merelyn at 11am of that day. Fortes was apprehended. A complaint
was filed on behalf of Merelyn for rape against Fortes in the MCTC. The information stated that
Fortes, thru force and intimidation and armed with a bolo, threatened and dragged Merelyn inside
a nipa hut owned by Leobehildo Garra and raped her. The MCTC found probable cause and issued
an order for Fortes’ arrest. The bond for his temporary liberty was fixed at P30k but was later
reduced to P25k upon accused’s motion.

He waived his right to preliminary investigation. The court transmitted the records to the provincial
fiscal. The fiscal filed an information for rape with the RTC against Fortes. He pleaded not guilty.
The protracted trial began June 26, 1985 and finally submitted for decision 3 yrs later on Feb. 22,
1988. The RTC convicted Fortes on Jan.25, 1989.

He filed his notice of appeal on the same day where he requested the amount of appeal bond be
fixed by the RTC. The RTC gave due course to the appeal, but did not resolve the request to fix
amount of bail. Fortes filed an “Application for Bail on Appeal” reiterating his earlier request.
The RTC denied this, saying that “the accused has already been found guilty beyond reasonable
doubt of rape and sentenced to PR and his appeal from the decision already approved by the
court.” The RTC issued a Commitment of Final Sentence turning over the person of Fortes to the
director of prisons. Hence this petition for certiorari to set aside the RTC orders denying his
application for bail and MR thereof.

ISSUE:
Whether accused Fortes may be granted bail pending appeal of his conviction for rape in the trial
court.
HELD: NO.
CONTENTION: Before his conviction by final judgment, he enjoys the constitutional
presumption of innocence and is thus entitled to bail as a matter of right.

It is clear from S13, Art.III of the Constitution and S3, Rule 114 of the RoC that “before
conviction, bail is either a matter of right or of discretion. It is a matter of right when the offense
charged is punishable by any penalty lower than RP. To that extent the right is absolute.” But if
the offense is punishable by RP, bail becomes a matter of discretion. It shall be denied when
evidence of guilt is strong. The court’s discretion is limited to determining whether evidence of
guilt is strong. But once it is determined that evidence of guilt is not strong, bail also becomes a
matter of right.

The clear implication is thus that if an accused charged with a crime punishable by RP is convicted
by the trial court and sentenced to suffer such penalty, bail is neither a matter of right on the part
of accused nor of discretion on the part of the court. In such situation, the court would not have
only determined that evidence of guilt is strong, which would have been sufficient to deny bail
even before conviction, it would have likewise ruled that accused’s guilt has been proven beyond
reasonable doubt. Bail must not then be granted to accused during pendency of his appeal
from the judgment of conviction. Since rape under A335 of the RPC is punishable by RP, the
trial court correctly denied his application for bail during the pendency of the appeal.

166. Inocencio Basco v. Judge Rapatalo, AM RTJ-96-1335, March 5, 1997, Romero, J. (Right
to Bail)
FACTS:
Basco charged Judge Leo Rapatalo with gross ignorance or willful disregard of established rule of
law for granting bail to an accused in a murder case without receiving evidence and conducting
a hearing.

Basco, father of the victim, alleged that an information for murder was filed against one Roger
Morente, one of 3 accused. Morente filed a petition for bail. The hearing for the petition was set
for May 31, 1995 by petitioner but was not heard since Rapatalo was then on leave. It was reset to
June 8, 1995, but Rapatalo reset it to June 22, 1995. The hearing on June 22 did not materialize as
the prosecution’s witnesses in connection with said petition were not notified. Accused was instead
arraigned and trial was set. Another attempt was made to reset the hearing to July 17, 1995.

Meanwhile, Basco allegedly saw Morente in Rosario, La Union. He learned that Morente was out
on bail despite the fact that the petition had not been heard at all. Bail was actually granted on
June 29 based on a marginal note on the order stating “No Objection: P80,000,” signed and
approved by the prosecutor and Rapatalo. There was already a release order on June 29 when the
hearing was scheduled for July 17.

Rapatalo said he granted the petition based on the prosecutor’s option not to oppose the petition
and the latter’s recommendation setting the bail bond to P80k. Rapatalo avers that when the
prosecution chose not to oppose the petition for bail, he had the discretion on whether to approve
it or not. He said that when he approved the petition, he had a right to presume that the prosecutor
knew what he was doing since he was more familiar with the case, having conducted the
preliminary investigation. In any case, the bailbond was cancelled and a warrant for Morente’s
arrest was issued on account of Basco’s MR. Morente is, to date, confined at the provincial jail.

ISSUE:
Whether Judge Rapatalo may grant bail to an accused charged with a capital offense without
conducting a hearing since the prosecution interposed no objection.
HELD: NO.
"Bail" is the security required by the court and given by the accused to ensure that the accused
appears before the proper court at the scheduled time and place to answer the charges brought
against him or her. In theory, the only function of bail is to ensure appearance of defendant at
the time set for trial. The sole purpose of confining the accused in jail before conviction is to assure
his presence at trial. In other words, if the denial of bail is authorized in capital offenses, it is only
in theory that the proof being strong, the defendant would flee rather than face the verdict of the
court. Hence, the exception to the fundamental right to be bailed should be applied in direct ratio
to the extent of probability of evasion of the prosecution. In practice, bail has also been used to
prevent release of an accused who might be dangerous to society or whom the judges might not
want to release.

The determination of whether the evidence of guilt is strong, being a matter of judicial discretion,
remains with the judge. This discretion may rightly be exercised only after the evidence is
submitted to the court at the hearing. Since the discretion is directed to the weight of the
evidence and since evidence cannot properly be weighed if not duly exhibited or produced before
the court, a proper exercise of judicial discretion requires that the evidence of guilt be submitted
to the court, petitioner having the right of cross examination and to introduce his own evidence
in rebuttal. The discretion of the trial court is not absolute and must be exercised within reasonable
bounds.

Consequently, in the application for bail of a person charged with a capital offense punishable by
death, RP or life imprisonment, a hearing, whether summary or otherwise in the discretion of
the court, must actually be conducted to determine whether evidence of guilt against accused
is strong. A summary hearing means such brief and speedy method of receiving and considering
the evidence of guilt as is consistent with the purpose of hearing which is merely to determine the
weight of evidence for purposes of bail. On such hearing, the court does not sit to try the merits.
The course of inquiry may be left to the discretion of the court which may confine itself to
receiving such evidence as has reference to substantial matters, avoiding unnecessary
thoroughness in the examination and cross examination. If a party is denied opportunity to be
heard, there would be a violation of procedural due process.

Whether the motion for bail of a defendant in custody for a capital offense be resolved in a
summary proceeding or in the course of regular trial, the prosecution must be given opportunity to
present all evidence it may desire to introduce before the court should resolve the motion for bail.
Else, there would be violation of procedural due process, and the order of the court granting bail
should be considered void on that ground.

In People v. Dacudao, where the judge, without hearing, granted bail on the ground that there was
not enough evidence to warrant a case for murder since only affidavits of the prosecution witnesses
not eyewitnesses to the crime were filed, We held that these were only for prima facie determining
whether there was sufficient ground to engender a well-founded belief that the crime was
committed and pinpointing the persons who probably committed it. Whether evidence of guilt is
strong still has to be established unless the prosecution submits the issue on whatever it has already
presented. The prosecution must be consulted.
In People v. Calo, where the prosecution was scheduled to present 9 witnesses but after hearing
the fifth, the judge insisted on terminating the proceedings, we held that the prosecution was not
given adequate opportunity to prove that there is strong evidence of guilt. And to present within a
reasonable time all evidence it desired to present.

In Libarios v. Dabalo, the judge issued a warrant of arrest without prior hearing, fixing at the same
time bail at P50k since the evidence was merely circumstantial. We held that where the charge is
a capital offense, the trial court must conduct a hearing in a summary proceeding to let the
prosecution present, within a reasonable time, all evidence it may desire to produce to prove that
evidence of guilt is strong. Failure to conduct hearing before fixing bail violates due process.

In Borinaga v. Tamin, while the preliminary investigation was pending, a petition for bail was
filed. The judge ordered the prosecutor to appear at the hearing to present evidence that guilt of
accused is strong. The public prosecutor failed to appear, prompting the judge to grant application
for bail. We still held that the prosecution must be given opportunity within a reasonable time to
present all evidence it may desire to introduce before the court may resolve the motion for bail.
The judge was fined P20k.

All these cases are to the effect that when bail is discretionary, a HEARING, whether summary
or otherwise in the court’s discretion, should first be conducted to determine existence of
strong evidence or lack of it. Since the determination of whether evidence of guilt is strong is a
matter of judicial discretion, the judge is mandated to conduct a hearing even in cases where the
prosecution chooses to just file a comment or leave the application for bail to the discretion
of the court.

In Gimeno v. Arcueno, Sr., the judge granted bail to an accused in robbery with homicide without
letting the prosecution be heard. He said that he issued an order for motion to fix bail but the public
prosecutor filed a comment instead which the judge thought was adequate compliance with law.
he said that he granted bail as the evidence was not so strong and in fact the accused were later
dropped by the prosecutor for failure of the witnesses to positively identify them. We held that
there must still be hearing, but only imposed P5k fine.

In Concerned citizens v. Elma, the judge granted bail to a person charged with illegal recruitment
in large scale and estafa. The accused filed a motion to fix bail and the judge, instead of setting the
application for hearing, directed the prosecution to file its comment or opposition. The prosecution
submitted its comment, leaving the application for bail to the discretion of the court. The judge
said that he substantially complied with the requirement of a formal hearing by requiring the
prosecution to comment. He required the prosecution to present evidence but it refused, leaving
the determination of the motion to his discretion. We held that there must be hearing as the judge
could not have assessed the weight of evidence against the accused before granting bail.

In Baylon v. Sison, the judge granted bail to several accused in a double murder case. He claimed
that the prosecutor did not interpose an objection and never requested that it be allowed to show
evidence of guilt is strong but instead submitted the incident for resolution. We held that the
discretion lies not in determining whether there will be a hearing, but in appreciating the weight
of evidence of guilt against accused. The judge was fined P20k.
A hearing is also required if the prosecution refuses to adduce evidence in opposition to the
application to grant and fix bail. It is still mandatory.

In Tucay v. Domagas, the judge granted bail to an accused charged with murder without hearing.
The application for bail contained the annotation “No objection” of the prosecutor. We held that
the judge should nevertheless have set the petition for bail for hearing and diligently
ascertained from the prosecution whether the latter was not really contesting the bail
application. Only after satisfying himself that the prosecution did not wish to oppose the petition
for bail for justifiable cause (e.g. tactical reasons) and taking into account the factors enumerated
in Rule 114, S6 for fixing bail should the judge have ordered the petition for bail and release of
accused.

Another reason why hearing is required is for the court to take into consideration the guidelines
set forth in S6, Rule 114 of RoC in fixing the amount of bail. Even if the prosecutor fails to
adduce evidence in opposition to an application for bail, the court may still require that it answer
questions to ascertain not only the strength of the state’s evidence but also the adequacy of the
amount of bail.

After hearing, the court’s order granting or refusing bail must contain a SUMMARY of the
EVIDENCE for the prosecution. On the basis thereof, the judge should then formulate his own
conclusion as to whether the evidence so presented is strong enough as to indicate the guilt
of accused. otherwise, the order granting or denying bail may be invalidated as the summary of
evidence for the prosecution containing the judge’s evaluation of evidence may be considered
as an aspect of procedural due process for both the prosecution and the defense. In Carpio v.
Maglalang, the order of the judge granting bail was invalidated as it only stated the number of
prosecution witnesses but not their respective testimonies.

Rapatalo granted bail without hearing despite the offense charged being capital in disregard of the
procedure laid down in S8, Rule 114 as amended by Admin. Circular 12-94. His reasoning that he
has a right to presume that the prosecutor knows what he is doing is tantamount to ceding to the
prosecutor the duty of exercising judicial discretion to determining whether the guilt of accused
is strong.

The absence of objection from the prosecution is never a basis for granting bail. It is the court’s
determination after hearing that evidence of guilt is not strong that forms such basis. Rapatalo
should not have relied solely on the recommendation of the prosecutor but should have personally
ascertained whether evidence of guilt is strong. There will be violation of due process if the judge
grants bail without hearing since S8 of Rule 114 provides that whatever evidence presented for
or against accused’s provisional release will be determined at the hearing.

The practice of trial court judges of granting bail to accused when the prosecutor refuses or fails
to present evidence can be traced to Herras Teehankee v. Director of Prisons where this Court
gave the following instructions to the People’s Court:
1) In capital cases, when the prosecutor does not oppose the petition for release on bail, the court
should, as a general rule, in the proper exercise of discretion, grant the release. 2) But if the court
has reasons to believe that the prosecutor’s attitude is not justified, it may ask him questions to
ascertain the strength of the state’s evidence or to judge the adequacy of the amount of bail. 3) If
the prosecutor refuses to answer any particular question on the ground that the answer may involve
disclosure imperiling the success of the prosecution or jeopardizing public interest, the court may
not compel him to do so, if he exhibits a statement to that effect of the SOLGEN, proided that such
refusal will not prejudice the rights of the defendant or detainee.

But this case was decided fully half a century ago under a completely different factual milieu.
Teehankee was indicted under a law dealing with treason cases and collaboration with the enemy.
The “instructions” given in said case under the 1940 RoC no longer apply due to amendments
introduced in the 1985 RoC.

Under the 1940 RoC:


Sec. 7. Capital offense — Burden of proof . — On the hearing of an application for admission to
bail made by any person who is in custody for the commission of a capital offense, the burden of
showing that the evidence of guilt is strong is on the prosecution."

This has not been adopted in toto in the 1985 RoC as amended by AC 12-94 as some phrases have
been added:
Sec. 8. Burden of proof in bail application. — At the hearing of an application for
admission to bail filed by any person who is in custody for the commission of an offense
punishable by death, reclusion perpetua or life imprisonment, the prosecution has the
burden of showing that evidence of guilt is strong. The evidence presented during the bail
hearings shall be considered automatically reproduced at the trial, but upon motion of
either party, the court may recall any witness for additional examination unless the witness
is dead, outside of the Philippines or otherwise unable to testify."
The italicized sentence in S8, 1985 RoC was added to address a situation where in case the
prosecution does not choose to present evidence to oppose the application for bail, the judge may
feel duty-bound to grant the bail application. In such a case, the judge may well lose control of the
proceedings. In a sense, this undermines the authority of a judge since all that the prosecution has
to do to "force" the judge to grant the bail application is to refrain from presenting evidence
opposing the same. In the event that the prosecution fails or refuses to adduce evidence in the
scheduled hearing, then a hearing as in a regular trial should be scheduled. In this regard, a hearing
in the application for bail necessarily means presentation of evidence, and the filing of a comment
or a written opposition to the bail application by the prosecution will not suffice.

The prosecution under the revised provision is duty bound to present evidence in the bail
hearing to prove whether evidence of guilt is strong and not merely to oppose. This also
prevents the practice in the past where a petition for bail was used as a means to force the
prosecution into a premature revelation of evidence. The nature of the hearing in an application
for bail must be equated with its purpose, i.e. to determine bailability of accused.

The duties of the trial judge in case an application for bail is filed:
(1) Notify the prosecutor of the hearing of the application for bail or require him to submit his
recommendation (Section 18, Rule 114);
(2) Conduct a hearing of the application for bail regardless of whether the prosecution refuses to
present evidence to show that the guilt of the accused is strong for the purpose of enabling the
court to exercise its sound discretion (Sections 7 and 8, supra);
(3) Decide whether the evidence of guilt of the accused is strong based on the summary of evidence
of the prosecution;
(4) If the guilt of the accused is not strong, discharge the accused upon the approval of the bailbond.
(Section 19, supra). Otherwise, petition should be denied.

Judge Rapatalo was reprimanded.

167. Bgen. Jose Comendador v. Gen. Renato de Villa, GR 93177, August 2, 1991, Cruz, J.
(Right to Bail)
FACTS:
Petitioners in these 4 consolidated cases are officers of the AFP facing prosecution for their alleged
participation in the failed coup d’etat on Dec.1-9, 1989. They are charged with violations of
Articles of War (AW) 67 (mutiny), 96(conduct unbecoming an officer and gentleman), 94 (various
crimes), in relation to Art.248 of the RPC on murder.

A Pre-Trial Investigation (PTI) panel was constituted to investigate petitioners. At the first hearing,
they challenged the proceedings on various grounds. The PTI gave them 10 days to submit their
objections in writing. They filed a motion for summary dismissal, which was denied. the PTI gave
them 5 days to submit their counter-affidavits. They moved for reconsideration of the denial.

Petitioner in GR 95020, Ltc. Jacinto Ligot, applied for bail, but the application was denied by
General Court Martial (GCM) 14. He filed with the RTC a petition for certiorari. The RTC judge
granted provisional liberty to Ligot. Ligot filed a motion to enforce the order for his release and
declare in contempt the commanding officer of the PC/INP jail for disobeying the order. Generals
de Villa and Aguirre had refused to release him “pending final resolution of the appeal to be taken”
to the SC. The RTC reiterated its order for provisional liberty of Ligot, saying that S13, Art.III of
the Constitution grants the right to bail to all persons, setting aside the orders of GCM14 denying
bail on the “mistaken assumption that bail does not apply to military men facing court-martial
proceedings on the ground that there is no precedent.”

ISSUE:
Whether the right to bail may be granted to persons on trial with a general court martial for
violations of the Articles of War.
HELD: NO.
It is argued that since private respondents are officers of AFP accused of violations of AW, the
respondent courts (RTC, CA) have no authority to order their release and interfere with the court-
martial proceedings.

Civil courts as a rule exercise no supervision or correcting power over the proceedings of courts-
martial. But in the exercise of their undoubted discretion, courts-martial may commit such an abuse
of discretion as to give rise to a defect in their jurisdiction. The RTC has concurrent jurisdiction
with the CA and SC over petitions for certiorari, prohibition, or mandamus against inferior courts
and other bodies and on petitions for HC and quo warranto. Without a law providing that the
decisions, orders, and rulings of a court-martial or the office of the Chief of Staff can be questioned
only before the CA and SC, the RTC can exercise similar jurisdiction.

But the right to bail of private respondents in GR 95020 has traditionally not been recognized
and is not available in the military, as an exception to the general rule in the BoR. “The right
to a speedy trial is given more emphasis in the military where the right to bail does not exist.”

The unique structure of the military exempts military men from right to bail. Mutinous soldiers
operate within the framework of democratic system and are allowed fiduciary use of firearms by
the government paid out of revenues from the people. “National security considerations should
impress upon this Honorable Court that release on bail constitutes damaging precedent.” Imagine
if 1000 putschists were out on bail. They could freely resume their heinous activity which could
result in the overthrow of duly constituted authorities.

The argument that denial of right to bail from military would violate equal protection is not
acceptable. This does not apply where the subject is substantially different from others. They
could complain if they are denied bail and other military members are not. But they cannot
complain that they are not allowed the same right extended to civilians.

168. Miguel Paderanga v. CA, GR 115407, August 28, 1995, Regalado, J. (Right to Bail)
FACTS:
Paderanga was belatedly charged in an amended information as a co-conspirator in a multiple
murder for the killing of members of the Bucag family in 1984 in Gingoog City of which
Paderanga was mayor at the time. The trial of the case was all set to start with the issuance of an
arrest warrant for Paderanga’s apprehension but, before it could be served on him, he filed a motion
for admission to bail with the trial court which set the same for hearing on Nov. 05, 1992. The
hearing took place.

As Paderanga was then confined at a hospital due to acute costochondritis, his counsel manifested
that they were submitting the custody of their client to the local chapter president of the IBP and
that, for purposes of said hearing on his bail application, he be considered as being in the custody
of the law. The prosecution said that it was neither supporting nor opposing the application for bail
and submitting the same to the sound discretion of the trial judge. It also waived further
presentation of evidence.

The trial court granted bail in the amount of P200k. The next day, Paderanga, apparently still weak
but well enough to travel by then, managed to personally appear before the clerk of court and
posted bail. He was subsequently arraigned and trial also ensued. He personally attended all the
court hearings.

The prosecutor filed a mr of the resolution granting bail 20 days later, but this was denied. on
October 1, 1993 or more than 6 months later, the prosecutor elevated the matter to the CA thru
certiorari. The resolution of the trial court granting bail was annulled by the CA on the ground
that they were tainted with grave abuse of discretion. It said that when Paderanga filed his
application for bail, he was not yet “in the custody of the law” as he filed the same before he was
actually arrested or voluntarily surrendered.
ISSUE:
Whether an accused who applies for bail may be considered as under custody of law for such
purposes when he 1) files his application for bail with the trial court, 2) informs the court and
prosecution of his actual whereabouts (hospital recuperating), and 3) unequivocally recognizes the
jurisdiction of the court where he does not personally post his bail bond as he was in the hospital
recuperating from “acute costochondritis.”
HELD: YES.
1. Paderanga argues that his filing of the application for bail effectively conferred on the trial court
jurisdiction over his person. Thus he was in the custody of the law. There is no evidence of strong
guilt. The prosecution waived presentation of evidence and cannot claim denial of due process.

The main purpose of bail is to relieve an accused from the rigors of imprisonment until his
conviction and yet secure his appearance at the trial. As bail is intended to secure one’s provisional
liberty, it cannot be posted before custody over him has been acquired by judicial authorities,
either by his lawful arrest or voluntary surrender. It would be incongruous to grant bail to one who
is free. The rationale is to prevent resort to the pernicious practice whereby an accused could just
send another in his stead to post his bail, without recognizing the jurisdiction of the court by his
personal appearance. As a paramount requisite, only those who have either been arrested,
detained, or otherwise deprived of their freedom will have occasion to seek bail. The person
seeking provisional release need not even wiat for a formal complaint or information to be filed
against him as it is available to “all persons” where the offense is bailable. But this is subject to
the condition that the applicant is in the custody of the law.

A person is considerd in the custody of the law 1) when he is arrested either by virtue of a
warrant or by warrantless arrest, or 2) he has voluntarily submitted himself to the jurisdiction of
the court by surrendering to the proper authorities.

In Santiago v. Vasquez, petitioner was charged before the Sandiganbayan for violation of the Anti-
Graft and Corrupt Practices Act. She filed a motion for acceptance of cash bail bond. She was
confined in a hospital recuperating from serious injuries sustained in a major vehicular mishap.
She expressly sought leave that she be “considered as having placed herself under the jurisdiction
of the Sandiganbayan for purposes of the trial and other proceedings.” The Sandiganbayan
authorized her to post cash bail bond without need of her personal appearance in view of her
physical incapacity and as a matter of humane consideration. Later, when the Sandiganbayan
issued a hold departure order against her, she questioned its jurisdiction as she never voluntarily
surrendered or arrested and that she has not validly posted bail since she never personally appeared
before the court. The SC held that she was estopped by her earlier recognizing such jurisdiction.
By posting cash bond accepted by the court she effectively submitted to its jurisdiction over her
person.

Here, while Paderanga filed his motion for admission for admission to bail before he was actually
and physically placed under arrest, he may at that point and in the factual ambience thereof be
considered as being CONSTRUCTIVELY and legally under custody. He should be deemed to
have voluntarily submitted his person to the custody of the law and the jurisdiction of the
trial court. In fact, an arrest is made either by an actual restraint or merely by the arrestee’s
submission to the custody of the person making the arrest. The latter may be exemplified by
“house arrest” or, in case of military offenders, by being “confined to quarters” or restricted to the
military camp area.

Paderanga, thru counsel, emphatically made it known to the trial court and prosecution during the
hearing for bail that he could not personally appear as he was then confined at a hospital and
could not obtain medical clearance to leave the hospital. Despite their knowledge of
Paderanga’s whereabouts, the trial court and prosecution never lifted a finger to have the arrest
warrant served upon him. It would have taken the slightest effort to put Paderanga in the physical
custody of the authorities as he was incapacitated in a hospital bed just over a km away, by simple
ordering his confinement or placing him on guard. The trial court and prosecution agreed that
Paderanga was in the constructive custody of the law as they did not attempt to have him physically
restrained.

Thru his lawyers, Paderanga expressly submitted to physical and legal control of his person 1) by
filing the application for bail with the trial court, 2) by furnishing true information of his actual
whereabouts, and 3) by unequivocally recognizing the jurisdiction of said court. When he
knew that a warrant of arrest was issued, Paderanga never attempted to evade the clutches of the
law. Also, as manifestation of good faith and recognition of the authority of the trial court, his
counsel informed the court that they were surrendering custody of Paderanga to the IBP. Thus, the
motion for admission to bail was not filed for the purpose which the law proscribes for being
derogatory of the authority and jurisdiction of the courts as there was no intent or strategy
employed to obtain bail in absentia and thereby be able to avoid arrest should the application
therefor be denied.

2. S13, Art.III states that before conviction, all indictees shall be allowed bail except only those
charged with offenses punishable by RP when evidence of guilt is strong. Pursuant thereto, S4,
Rule 114 now provides that all persons in custody shall, before conviction by a RTC of an offense
not punishable by death, RP, or LI, be admitted to bail as a matter of right. The right to bail, which
may be waived considering its personal nature, springs from the presumption of innocence.
When it becomes a matter of judicial discretion under exceptions to the rule, a hearing is required
with the participation of both the defense and representative of the prosecution to ascertain if
evidence of guilt is strong. The burden of proof is on the prosecution. If the prosecution be denied
this opportunity, it is denied procedural due process. The court cannot rey on mere affidavits or
recitals of their contents, if timely objected to, for these represent only hearsay evidence, and thus
are insufficient to establish the quantum of evidence that the law requires.

The prosecution assails what it considers to be a violation of procedural due process when the
court allowed assistant prosecutor Abejo of the regional state prosecutor’s office to appear in
behalf of the prosecution, instead of state prosecutor Gingoyon who is claimed to be the sole
government prosecutor authorized to handle the case and who received his copy of the motion only
on the day after the hearing had been conducted. Thus, Abejo had no authority to waive the
presentation of any further evidence.
But the regional state prosecutor office acted as collaborating counsel with Gingoyon based on
authority from Chief state prosecutor Fernando de Leon. It was only withdrawn on July 12, 1993.
Thus, Abejo could validly represent the prosecution in the hearing on Nov.5, 1992.

3. It has been held that where the prosecutor interposes no objection to the motion of accused, the
trial court should nevertheless set it for hearing and ascertain from the prosecution whether the
latter is really not contesting the bail application. There is thus no irregularity in the context of
procedural due process here. The judge exhausted all means to convince itself of the propriety of
the waiver of evidence. The order also contained the required summary of evidence of both
prosecution and defense and only after sifting thru them did the court conclude that Paderanga
could be released on bail.

As to the contention that the prosecution was not given opportunity to present evidence within a
reasonable time, the records show that the regional prosecutor’s office received its copy of the
application for bail on the very same day it was filed on Oct. 28, 1992. From Nov. 2, 1992, the
prosecution had more than one week to muster such evidence as it would have wanted to adduce
to oppose the motion. That period, under the circumstances, was reasonable.

169. Government of Hongkong Special Administrative Region v. Hon. Olalia, GR 153675,


April 19, 2007, Sandoval-Gutierrez, J. (Right to Bail)
FACTS:
On January 30, 1995, PH and the then British Crown Colony of HK signed an “Agreement for the
Surrender of Accused and Convicted Persons.” It took effect June 20, 1997. On July 1, 1997, HK
reverted back to the People’s Republic of China and became the HK Administrative Region.
Private respondent Muñoz was charged before the HK court with 3 counts of “accepting an
advantage as agent.” He also faces 7 counts of the offense of conspiracy to defraud. If convicted,
he faces a jail term of 7 to 14 years for each charge.

The DOJ received from the HK DOJ a request for the provisional arrest of Muñoz. This ws
forwarded to the NBI which, in turn, filed with the RTC an application for Muñoz’s provisional
arrest. The RTC ordered his arrest. The NBI agents arrested and detained Muñoz. He filed a
petition for certiorari with the CA questioning the validity of the order of arrest. The CA declared
said order void. The DOJ filed with this Court for certiorari, praying reversal of the CA decision.
The SC granted the petition, sustaining the validity of the order of arrest against Muñoz. The
decision became final and executory on April 10, 2001.

Meanwhile, on Nov.22, 1999, petitioner HK Admin Region filed with the RTC a petition for the
extradition of Muñoz. Muñoz filed a petition for bail in said case. The RTC Judge Bernardo
denied the bail holding that there is no PH law granting bail in extradition cases and that Muñoz
is a high “flight risk.” Judge Bernardo inhibited himself. The case was raffled to Judge Olalia.
Muñoz moved for reconsideration. Olalia granted the motion, allowing Muñoz to post bail. Olalia
set bail at P750k and ordered him to surrender his valid passport to the Court. He was also required
to report to the government prosecutors handling the case and if they desire, they can manifest
before the RTC to require that all assets of Muñoz be filed with the RTC which will all be forfeited
if he flees.
HKAR moved to vacate this order, which was denied. Hence this petition.

ISSUE:
Whether Muñoz, a potential extraditee, may be granted bail upon application for his extradition by
the Hongkong Administrative Region to face criminal prosecution therein.
HELD: YES, upon showing of clear and convincing evidence that he is not a flight risk.
HKAR alleges that the RTC committed GADALEJ in admitting Muñoz to bail and there is nothing
in the constitution or law providing that a potential extradite has a right to bail, said right being
limited to criminal proceedings.

Art.III provides:
Sec. 13. All persons, except those charged with offenses punishable by reclusion perpetua
when evidence of guilt is strong, shall, before conviction, be bailable by suMcient sureties,
or be released on recognizance as may be provided by law. The right to bail shall not be
impaired even when the privilege of the writ of habeas corpus is suspended. Excessive bail
shall not be required.
It was held in Government of US of America v. Hon. Purganan that bail does not apply to
extradition proceedings and that it is available only in criminal proceedings. At first glance, the
ruling therein applies squarely here. But this Court cannot ignore the following trends in
international law: 1) growing importance of the individual person in public international law who,
in the 20th century, has gradually attained global recognition; 2) the higher value now being to
human rights in the international sphere, 3) the duty of countries to observe these universal human
rights in fulfilling treaty obligations; and 4) duty of this Court to balance the rights of the individual
under our fundamental law, on one hand, and the law on extradition, on the other. The modern
trend in public international law is the primacy placed on the worth of the individual person
and the sanctity of human rights. Slowly, the recognition that the individual person may be a
subject of international law is now taking root.

Thus, on Dec. 10, 1948, the UNGA adopted the UDHR. While not a treaty, the principles therein
were now recognized as customarily binding upon members of the international community.
It was held that the principles therein are part of the law of the land. The ICCPR was also adopted.

PH, along with other members of the family of nations, committed to uphold the fundamental
human rights. This commitment is enshrined in S2, Art.II of the constitution (The state values the
dignity of every human person and guarantees full respect for human rights.) thus, PH is under
obligation to make available to every person under detention such remedies which safeguard their
fundamental right to liberty, including the right to bail. In light of the various international treaties
giving recognition and protection to human rights, particularly to life and liberty, a reexamination
of this Court’s ruling in Purganan is in order.

First, we note that the state’s power to deprive an individual of his liberty is not limited to
criminal proceedings. Respondents in administrative proceedings, like deportation and
quarantine, have likewise been detained. Second, PH jurisprudence has not limited bail to
criminal proceedings only. Bail has been allowed to persons in detention during pendency of
administrative proceedings, taking cognizance of the obligation of PH under international
conventions to uphold human rights.
In the 1909 case of US v. Go-Sioco, a Chinese facing deportation was granted bail. The Court
opined while deportation is not a criminal proceeding, some of the machinery used is the
“machinery of criminal law.” In Mejoff v. Director of Prisons, the SC held that foreign nationals
against whom no formal criminal charges have been filed may be released on bail pending finality
of an order of deportation. The Court there relied on the UDHR.

If bail can be granted in deportation proceedings, we see no justification why it should not be
allowed in extradition cases. Since the UDHR applies to deportation, there is no reason why it
cannot be invoked in extradition. Both are administrative proceedings where the innocence or guilt
of the person detained is not in issue. The right of a prospective extradite to apply for bail in this
jurisdiction must be viewed in light of the various treaty obligations of PH.

Extradition has been characterized as the right of a foreign power, created by treaty, to demand the
surrender of one accused or convicted of a crime within its territorial jurisdiction, and the
correlative duty of the other state to surrender him to the demanding state. It is not a criminal
proceeding even if the potential extradite is a criminal, for it is not punishment for a crime even
though such punishment may follow extradition. It is sui generis, tracing its existence wholly to
treaty obligations between different nations. It is merely administrative in character. Its object
is to prevent the escape of a person accused or convicted of a crime and to secure his return to the
state from which he fled, for trial or punishment.

But while it is not a criminal proceeding, extradition is characterized by: 1) deprivation of liberty
and 2) the means employed to attain the purpose of extradition is also the “machinery of criminal
law. ”PD 1069, S6 (PH Extradition Law) mandates the “immediate arrest and temporary
detention of the accused” if such will “best serve the interest of justice.” Thus, while an
extradition proceeding is ostensibly administrative, it bears all earmarks of a criminal process. A
potential extradite may be subjected to arrest, prolonged restraint of liberty, and forced to
transfer to the demanding state following the proceedings.

While our extradition law does not provide for the grant of bail to an extradite, there is no
provision prohibiting him from filing a motion for bail, a right to due process under the
constitution.

However, the applicable standard of due process should not be the same as that in criminal
proceedings. The standard in the latter is premised on the presumption of innocence. It is from
this major premise that the ancillary presumption in favor of admitting to bail arises. Bearing in
mind the purpose of extradition proceedings, the premise behind issuance of the arrest awarrant
and temporary detention is the possibility of flight of the potential extradite. This is based on
the assumption that he is a fugitive from justice. Thus, the extraditee bears the ONUS
PROBANDI of showing that he is not a flight risk and should be granted bail.

Pacta sunt servanda demands that PH honoes its obligations under the extradition treaty with
HKAR. The standard of proof in an extradition proceeding, being sui generis, cannot be beyond
reasonable doubt or preponderance of evidence. The standard of substantial evidence in
administrative proceedings cannot likewise apply even if it is administrative in character given the
object of extradition to prevent the extraditee from fleeing our jurisdiction. The new standard
proposed in CJ Puno’s separate opinion in Purganan, CLEAR AND CONVINCING
EVIDENCE, should be used in granting bail in extradition cases. This should be lower than
proof beyond reasonable doubt but higher than preponderance of evidence.

Since there is no showing that Muñoz presented evidence to show that he is not a flight risk, this
case should be remanded to the trial court to determine if he may be granted bail based on clear
and convincing evidence.

170. Government of the United States of America v. Hon. Purganan, GR 148571, September
24, 2002, Panganiban, J. (Right to Bail)
FACTS:
Pursuant to the RP-US Extradition Treaty, the US sent to PH note verbale 0522 requesting the
extradition of Mark Jimenez aka Mario Batacan Crespo. The Secretary of Foreign Affairs (SFA)
transmitted the documents to SOJ for appropriate action pursuant to S5 of PD 1069, Extradition
Law. Kimenez sought and was granted a TRO by the RTC, prohibiting the DOJ from filing with
the RTC a petition for his extradition. The validity of the TRO was assailed by SOJ before the SC.
The SC initially dismissed the SOJ’s petition. The SOJ was ordered to furnish Jimenez copies of
the extradition request and to grant him a reasonable period to file comment and evidence.

Upon MR by SOJ, the SC reversed its earlier decision, holding that Jimenez was bereft of the right
to notice and hearing during the evaluation stage of the extradition process. Finding no more legal
obstacle, US, represented by PH DOJ, filed with RTC a petition for extradition. The petition
alleged that Jimenez was the subject of an arrest warrant issued by the US district court in Florida
in connection with offenses like tax evasion, wire fraud, illegal campaign contributions, etc. The
petition prayed for Jimenez’s immediate arrest pursuant to S6, PD 1069 to prevent his flight.

Jimenez prayed that US’s application for arrest warrant be set for hearing. The RTC granted this
motion and set the case for hearing. The petitioner US manifested its reservations on the procedure
adopted by the RTC allowing the accused in an extradition case to be heard before issuing an arrest
warrant. Jimenez also sought an alternative prayer that in case the warrant be issued, he be allowed
to post bail of P100k. The alternative prayer was also set for hearing. After, the RTC issued its
order, issuing a warrant of arrest and fixing bail at P1M in cash. After he surrendered his passport
and posted the cash bond, Jimenez was granted provisional liberty. Hence this petition.

ISSUE:
1. Whether Jimenez, as a potential extradite, is entitled to notice and hearing before a warrant for
his arrest can be issued.
2. Whether Jimenez is also entitled to bail and provisional liberty while extradition proceedings
against him are pending.
HELD:
- Five Postulates of Extradition
1. Extradition is a major instrument for suppression of crime. Extradition treaties are entered into
to suppress crime by facilitating arrest and custodial transfer of a fugitive from a state to another.
A majority of nations have come to look upon extradition as the major effective instrument of
international cooperation in the suppression of crime. In this era of faster international travel, we
cannot afford to be an isolationist state.
2. The requesting state will accord due process to the accused. An extradition treaty presupposes
that both parties have examined, and that both accept and trust each other’s legal system.
3. The proceedings are sui generis. Extradition proceedings are not criminal in nature. They are
not criminal proceedings which will call into operation the rights of an accused as guaranteed by
the BoR. It does not involve determination of guilt or innocence, which will be adjudged in the
court of the state where he will be extradited. Hence, as a rule, rights that are only relevant to
determine the guilt or innocence of an accused cannot be invoked by an extraditee. An
extradition proceeding is summary in nature unlike criminal proceedings which involve a full-
blown trial. It only requires showing of “prima facie case” to order the extradition of a fugitive.
Also, while in a criminal case judgment becomes executory upon being rendered final, in
extradition cases while the courts may adjudge an individual extraditable, the president has the
final discretion to extradite him.

The ultimate purpose of extradition proceedings is only to determine whether the extradition
request complies with the extradition treaty and if the person sought is extraditable.

4. Compliance shall be in good faith. We are bound by pacta sunt servanda to comply in good
faith with our obligations under the treaty. The demanding government, where it has done all that
the treaty and the law require it to do, is entitled to the delivery of the accused and the other
government is under obligation to make the surrender.
5. There is an underlying risk of flight. Potential extraditees are presumed to be flight risks. This
presumption finds reinforcement in the experience of the executive branch. Jimenez’s 1) leaving
the requesting state before the conclusion of his indictment proceedings there and 2) remaining
here in the requested state despite learning that the requesting state is seeking his return and that
the crimes he is charged with are bailable, speak of his aversion to the processes of the requesting
state and his predisposition to avoid them.

FIRST ISSUE- NO.


US contends that informing accused of the extradition petition was filed and that US is seeking his
arrest gives him notice to escape. Jimenez claims that there is no law setting forth the procedure
prior to issuance of arrest warrant after the filing of petition for extradition. Thus, the formulation
of that procedure is within the discretion of the judge.

PD 1069 reads:
SEC. 6. Issuance of Summons; Temporary Arrest, Hearing, Service of Notices. — (1)
Immediately upon receipt of the petition, the presiding judge of the court shall, as soon as
practicable, summon the accused to appear and to answer the petition on the day and hour
fixed in the order. [H]e may issue a warrant for the immediate arrest of the accused
which may be served anywhere within the Philippines if it appears to the presiding judge
that the immediate arrest and temporary detention of the accused will best serve the ends
of justice. Xxx

This does not sanction RTC Judge Purganan’s act of immediately setting the hearing for issuance
of warrant of arrest. The qualifying word “immediate” in S6 would be rendered nugatory by setting
for hearing the issuance of arrest warrant. Hearing entails sending notice to the opposing parties,
receiving facts and arguments, and giving them time to prepare and present such facts and
arguments.

“If it appears” further conveys that accuracy is not as important as speed at such early stage. The
trial court is not expected to make an exhaustive determination immediately upon filing of the
petition. The court is merely expected to get a good first impression, a prima facie finding,
sufficient to make a speedy initial determination as to the arrest and detention of accused. Here,
there were appendices and annexes attached to the petition for extradition that Purganan could
have already gotten an impression adequate to make an initial determination of whether Jimenez
was someone who should be immediately arrested to “best serve the ends of justice.” The
extradition request was already prima facie meritorious. Thus, Purganan gravely abused his
discretion when he set the matter for hearing. Sending to persons sought to be extradited a notice
of request for their arrest and setting it for hearing would give them ample opportunity to escape.

Even Art.III, S2 of the constitution only requires the examination under oath or affirmation of
complainants and the witnesses they may produce. There is no requirement to notify and hear the
accused before issuing arrest warrants. All that is required was that the judge must have sufficient
supporting documents upon which to make his independent judgment or upon which to verify the
findings of the prosecutor as to existence of probable cause. At most, in cases of clear insufficiency
of evidence, judges merely further examine complainants and their witnesses. Requiring such
hearing could convert the determination of prima facie case into a full-blown trial and may make
trial of the main case superfluous.

The proper procedure is that upon receipt of a petition for extradition and its supporting documents,
the judge must study them and make, as soon as possible, a prima facie finding whether 1) they
are sufficient in form and substance, 2) they show compliance with the Extradition Treaty and
the Law, and 3) the person sought is extraditable. The judge may require submission of further
documentation or may personally examine the affiants and witnesses of petitioner. If despite this
study and examination, no prima facie finding is possible, the petition may be dismissed at the
discretion of the judge. If there is a prima facie case, the judge must immediately issue an arrest
warrant of the extraditee who is at the same time summoned to answer the petition and to appear
at scheduled summary hearings. Prior to issuance of warrant, the judge must not inform or
notify the potentitial extraditee of the pendency of the petition lest he be given opportunity to
escape.

SECOND ISSUE- NO.


Jimenez claims that Art.III, S13 secures right to bail of all persons, including potential extraditees.
The only exceptions are those charged with offenses punishable by RP when evidence of guilt is
strong. US claims that there is no provision in the Constitution granting bail to potential
extraditees.

"Art. III, Sec. 13. All persons, except those charged with offenses punishable by reclusion
perpetua when evidence of guilt is strong, shall, before conviction, be bailable by sufficient
sureties, or be released on recognizance as may be provided by law. The right to bail shall
not be impaired even when the privilege of the writ of habeas corpus is suspended.
Excessive bail shall not be required."

We agree with US. As suggested by “conviction,” bail in S13 and S4, Rule 114 applies only when
a person has been arrested and detained for violation of PH CRIMINAL laws. It does NOT
apply to extradition proceedings because extradition courts do not render judgments of
conviction or acquittal.

Right to bail “flows from the presumption of innocence in favor of every accused who should
not be subjected to loss of freedom as thereafter he would be entitled to acquittal, unless his guilt
be proved beyond reasonable doubt.” It thus follows that right to bail does not apply to a case like
extradition, where the presumption of innocence is NOT at issue.

The second sentence does not detract from the rule that right to bail is available only in criminal
proceedings. Suspension of the privilege of the writ of HC applies only to persons charged for
rebellion or offenses inherent in or connected with invasion. Thus, the second sentence merely
emphasizes the right to bail in criminal proceedings in these offenses.

That Jimenez’s offenses are bailable in the US is not an argument as extradition proceedings are
separate and distinct from the trial for which he is charged. He should apply for bail before the
courts trying the criminal cases against him, not before the extradition court.

- Due Process
There is no violation of due process in the detention prior to the conclusion of the extradition
proceedings. Its essence is the opportunity to be heard but does not always call for prior
opportunity to be heard. When the circumstances, like those present in extradition cases, call for
it, a subsequent opportunity to be heard is enough. Jimenez will be given full opportunity ot be
heard when the extradition court hears the petition for extradition.

There is no arbitrariness. 1) The DOJ’s filing in court the petition with its supporting documents
after a determination that the extradition request meets the requirements of the law and relevant
treaty, 2) the judge’s independent prima facie determination that his arrest will best serve the ends
of justice before issuance of warrant for his arrest, and 3) his opportunity, once under the court’s
custody, to apply for bail as an exception to the no-initial-bail rule all ensure that the arrest and
detention will not be arbitrary.

Also, would it be proper for the government to increase the risk of violating its treaty obligations
to accord Jimenez his personal liberty? His supposed immediate deprivation of liberty without due
process that he had previously shunned (*in the US courts indictments) pales against the
government’s interest in fulfilling its treaty obligations in cooperating with the world community
in suppressing crime.

- Exceptions to the “No Bail” rule.


Bail is not a matter of right in extradition cases. We believe that the right to due process is broad
enough to include the grant of basic fairness to extraditees. Thus, AFTER a potential extraditee is
arrested or placed under the custody of the law, bail may be applied for and granted as an
exception, only upon a clear and convincing showing 1) that, once granted bail, the applicant
will not be a flight risk or a danger to the community and 2) there exist special, humanitarian,
and compelling circumstances including, as a matter of reciprocity, those cited by the highest
court in the requesting state when it grants provisional liberty in extradition cases therein.

Since this exception has no express statutory basis and is derived from general principles of justice
and fairness, the applicant bears the burden of proving the above two-tiered requirement with
clarity, precision, and emphatic forcefulness. The Court realizes that extradition is an executive,
not a judicial, responsibility arising from the presidential power to conduct foreign relations.
Hence, any intrusion by the courts into the exercise of this power should be characterized by
caution. While this Court is protective of the sporting idea of fair play, it recognizes the limits of
its own prerogatives and the need to fulfill international obligations.

The following are the special circumstances cited by Jimenez for the Court to grant his request:
1. Alleged disenfranchisement of his manila district of 600k residents as he was elected a member
of the House of Representatives. We are not persuaded. The voters elected him with full awareness
of the limitations of his freedom of action. As to equal protection, does being an elective official
result in a substantial distinction allowing different treatment? The performance of duties by public
officers has never been an excuse to free a person from prison. A congressman is only one of 250
members of the HoR. Election to congressman is not a reasonable classification in criminal law
enforcement. Before Jimenez ran for public office and won, it was already of public knowledge
that the US was requesting his extradition. His constituents should have been prepared for the
consequences.

2. Anticipated delay since extradition proceedings are lengthy, it would be unfair to confine him
during pendency thereof. Extradition cases are summary in nature.

3. Not a flight risk? That he has not yet fled from PH cannot be taken to mean that he will stand
his ground and still be within reach of our government if and when it matters, that is, upon
resolution of the petition for extradition. In any event, bail may be applied for after the applicant
has been taken into custody.

171. Government of the US of America v. Hon. Purganan, GR 148571, December 17, 2002,
Motion for Reconsideration (Right to Bail)
Jimenez filed a MR.
1. He insists that the extradition court acted properly in granting bail to him. But his repeated
invocation of the extradition court’s grant of bail has not convinced us that he deserves bail under
the exception in our decision that 1) once granted bail, the applicant will not be a flight risk or a
danger to the community; and (2) that there exists special, humanitarian and compelling
circumstances including, as matter of reciprocity, those cited by the highest court in the requesting
state when it grants provisional liberty in extradition cases therein.

There is no clear and convincing showing of these two.

2. He argues that our Decision violates his due process rights. We have discussed this matter by
saying in our decision that, in its simplest concept, due process is merely opportunity to be heard,
which opportunity need not always be a prior one. Jimenez has been given more than enough
opportunity to be heard in this court and the extradition court. Even his MR has been allowed
“additional arguments,” which are normally not allowed.

3. Jimenez contends that as member of Congress, he is immune from arrest “arising from
offenses punishable by not more than six years imprisonment,” saying that he cannot be prevented
from performing his legislative duties as his constituents would be disenfranchised. A
congressman may be suspended or removed from office only by 2/3 vote of the HoR.

Our decision, citing People v. Jalosjos, has already debunked the disenfranchisement argument.
Further, it does not in any manner suspend or remove him from office. neither his arrest or
detention from the extradition proceeding constitutes suspension or removal from office. His
election to congressman with the duty to discharge legislative functions is not a substantial
differentiation which warrants placing him in a classification apart from other persons confined
and deprived of liberty pending resolution of their extradition cases. We reiterate that lawful
arrest and temporary confinement of a potential extraditee are germane to the purposes of the law
and apply to all those belonging to the same class.

As stated, the procedure adopted by the extradition court of first notifying and hearing a
prospective extraditee before issuance of arrest warrant is tantamount to giving notice to flee and
avoid extradition. Whether a potential extraditee does in fact go into hiding or not is beside the
point. The method by the lower court is at loggerheads with the purpose, object, and rational of
the law and overlooked the evils to be remedied.

As already stated, Jimene can avoid arrest and detention which are the consequences of the
extradition proceeding simply by applying for bail before the courts trying the criminal case
against him in the US as the indictments there are bailable. He is financially capable of producing
the necessary bail in the US.

172. OSG v. Judge Antonio de Castro, AM RTJ-06-2018, August 3, 2007, Tinga, J. (Right to
Bail)
FACTS:
The OSG filed an administrative complaint against executive judge Eugenio and Judge Antonio.
Judge Antonio issued an order in a habeas corpus proceeding temporarily restraining the
deportation of a Chinese national, Gao Yuan, for 17 days. OSG alleges that the order was in blatant
disregard of CA 613 or the PH Immigration Act of 1940, the pertinent provisions of the RoC on
habeas corpus, and prevailing jurisprudence. Judge Eugenio earlier restrained Yuan’s deportation
for 72 hours.

Gao Yuan is a national of the People’s Republic of China (PROC) and holder of a special non-
immigrant visa to PH and an immigrant visa to Canada. Gao Yuan, her husband James who is a
US national, and their two young children were on their way to a vacation in Canada when PH
immigration officers arrested Gao and prevented her from boarding her flight. The arrest was by
virtue of an order issued by BI Commissioner Fernandez, Jr. which was in turn a response to a
letter from the Consul General of the PROC, alleging that Gao Yuan was a fugitive from justice
and charged with embezzlement by Chinese police and requested her arrest and deportation to
China. Gao was detained at BI detention center.

James filed a petition for HC with application for TRO with the RTC, alleging that Gao was
illegally detained since she is not a fugitive from justice and no case was filed against her when
she left China in 2001. It was also alleged that Gao filed with the DOJ a petition for asylum as
political refugee. Judge Eugenio issued a 72-hour TRO. The case was then raffled to Judge
Antonio.

Commissioner Fernandez, thru OSG, filed a return of the writ, saying that Gao’s passport had been
cancelled by PROC and her apprehension had been requested by the embassy of PROC. Her
summary deportation was already ordered for being an undocumented and undesirable alien by the
BI Board of Commissioners.

Judge Antonio extended the 72-hour TRO by 17 days. He promulgated an order of release,
directing Fernandez to discharge the custody of Gao, she having filed cash bond of P250k. BI
refused to release Gao as there was no BI clearance. Fernandez filed an urgent MR. Judge Antonio
insisted on releasing Gao from custody. The RTC took custody of Gao. The RTC clarified that it
was only a provisional release for the duration of the TRO subject to conditions like posting of
additional P100k bond, monitored movements by the RTC, arrest warrant to be issued should she
try to hide, and signing by Gao and her husband of an undertaking that she will come to court if
called. Antonio based the provisional release on humanitarian reasons, considering that Gao was
merely wanted as a witness in a case in PROC and she is a nursing mother to a 17-month old
child.

Fernandez filed notice to appeal. Antonio said the notice was premature since the release is only
temporary. Antonio issued a warrant of arrest against Gao for failing to appear before the RTC
after having been directed to do so.

OSG contends that Gao’s release on bail is illegal since such falls within the exclusive jurisdiction
of BI and not the regular courts pursuant to S37(e) of CA 613.

ISSUE:
Whether an alien detained by the BI due to a deportation order may be granted bail for
humanitarian reasons by the courts.
HELD: NO.
A petition for the issuance of a writ of habeas corpus is a special proceeding governed by Rule 102
of the Revised Rules of Court. The objective of the writ is to determine whether the confinement
or detention is valid or lawful. If it is, the writ cannot be issued. What is to be inquired into is the
legality of his detention as of, at the earliest, the filing of application for the writ of HC since the
detention, if at inception illegal, may, by reason of supervening events, be no longer illegal at thee
time of filing of application.

Thus, once a person detained is duly charged in court, he may no longer question his detention
thru petition for HC. His remedy would be to quash the information or arrest warrant. “Court”
includes quasi-judicial bodies or governmental agencies authorized to order the person’s
confinement.

Here, the petition for HC was filed Sept.8, 2005. The return was filed Sept. 9 accompanied by the
charge sheet and summary deportation order dated both dated Sept. 7. When the petition was filed
by James Mahshi, a charge sheet and deportation order had already been filed against Gao Yuan.
The restraint of Gao’s liberty was already by virtue of a lawful process. Thus, the RTC no longer
had jurisdiction over the petition for HC.

Antonio also disregarded the rule on burden of proof after the writ has been returned as stated in
Rule 102, S13. If the detention is by reason or in pursuance of law, the return is considered prima
facie evidence of the validity of the restraint and the petitioner has the burden of proof to show
that the restraint is illegal. The reason is the presumption that official duty has been regularly
performed. The transcript of the hearing on Sept 16 shows that the allegations in the return that
Gao had a pending deportation case with the BI Board and that a deportation order was already
issued against her and that she used an expired Chinese passport in her attempt to leave PH, were
not controverted by James.

The provisional or temporary release of Gao also effectively granted granted the HC petition since
the main prayer in a HC petition relates to the release or discharge of the detainee. The general
rule is that the release, permanent or temporary, renders the HC petition moot. Such release must
be free from involuntary restraints. Gao’s release, while subject to conditions, did not unduly
restrain her movements or deprive her of her constitutional freedoms. The conditions did not
restrict her freedom of movement as she was able to elude the authorities.

Antoniio may also have been under the impression that the case was one for extradition since
Gao’s arrest and detention were pursuant to a request from PROC. If that were so, his acts of
releasing Gao upon filing of bond would have been sanctioned by the ruling in Government of
HKSAR v. Olalia, which allows the prospective extraditee to apply for bail provided he presents
clear and convincing evidence that he is not a flight risk and will abide with the orders and
processes of the extradition court. But the petition was one for habeas corpus which raised the
simple issue of whether Gao was held under lawful authority. The return sufficiently established
the basis of Gao’s detention. Her confinement was not illegal. Thus, it was error to continue with
the proceeding and thereafter order her release upon posting of cash bond.

It may be argued that in Government of HKSAR, the Court relied on 3 deportation cases which
allowed bail to persons in detention during pendency of their cases, stating that if bail can be
granted in deportation cases, there is no justification why it should not be allowed in extradition
cases. But circumstances peculiar to the 3 deportation cases existed that warranted admission to
bail. In US v. Go-Sioco, where a Chinese facing deportation for failure to secure the necessary
certificate of registration was granted bail pending his appeal, it was noted that said Chinese had
committed no crime, was born in the Philippines and lived here for more than 35 years, and at the
time of the case was living here with his mother, a Filipina. Said case was also brought under Act
No. 702 which falls, by provision of said law, under the jurisdiction of the courts of justice. In
Mejoff v. Director of Prisons and Chirskoff v. Commissioner of Immigration, the proposed
deportees were stateless foreign nationals, not enemies, against whom no criminal charges had
been formally made and who had been under detention for over two years after attempts at having
them deported failed. No such extraordinary circumstances appear in the case at bar.
S37(9)(e) of the PH Immigration Act of 1940, “Any alien under arrest in a deportation proceeding
may be released under bond or under such other conditions as may be imposed by the
Commissioner of Immigration," also confers upon the BI Commissioner the power and
discretion to grant bail in deportation proceedings, but does not grant to aliens the right to be
released on bail. The exercise of such power by the Commissioner is discretionary. So too the
determination of the propriety of allowing the temporary release on bail of the alien subject
to deportation and the conditions of such release falls under his jurisdiction, NOT the courts
of justice.

The reason is that courts do not administer immigration laws. The power of the commissioner to
grant bail should be exercised when the alien is still under investigation and not after the order of
deportation has been issued by the BI. When an alien is detained by the BI for deporation pursuant
to an order of deportation by the Deportation Board, the RTCs have no power to release such
alien on bail even on HC proceedings because there is NO LAW authorizing it.

Thus, Judge Antonio is administratively liable for gross ignorance of the law. When the law is so
elementary, not to be aware of it is gross ignorance of the law. He was suspended for 3m1d.

173. Juan Ponce Enrile v. Sandiganbayan, GR 213847, August 18, 2015, Bersamin, J. (Right
to Bail)
FACTS:
The Ombudsman charged Enrile and others with plunder in the Sandiganbayan based on their
purported involvement in the misuse of their PDAF. Enrile moved to be allowed to post bail should
probable cause be found against him. The Sandiganbayan heard the motions and denied it on the
ground of prematurity since Enrile had not yet then voluntarily surrendered or been placed under
custody of the law. The Sandiganbayan then ordered his arrest. On the same day that his arrest
warrant was issued, Enrile voluntarily surrendered. He moved for detention at the PNP General
Hospital and to fix bail which were heard by the Sandiganbayan.

Enrile argued that he should be allowed to post bail because 1) the prosecution had not yet
established that evidence of his guilt was strong, 2) although he was charged with plunder, the
penalty would only be RT, not RP, and 3) he was not a flight risk, and his age and physical
condition must further be seriously considered. The Sandiganbayan denied the motion to fix bail.

He claims that the prosecution failed to establish that Enrile, if convicted of plunder, is punishable
by RP since there are 2 mitigating circumstances- his age and voluntary surrender. He should not
be considered a flight risk considering his age of 90, his medical condition, and his social standing.
ISSUE:
Whether Enrile, charged with plunder punishable by RP to death, may be released on bail based
on his allegations that he is not a flight risk considering his numerous health ailments.
HELD: YES.
1. Bail protects the right of accused to due process and to be presumed innocent. The presumption
of innocence is rooted in due process and safeguarded by right to bail. Bail is not granted to prevent
the accused from committing additional crimes. Its purpose is to guarantee the appearance of the
accused at trial or whenever required by the trial court. The amount of bail should be high enough
to assure the presence of accused when required, but it should be no higher than is reasonably
calculated to fulfill this purpose. Bail reconciles accused’s interest in provisional liberty before
or during trial and society’s interest in assuring the accused’s presence at trial.

2. Bail may be granted as a matter of right or of discretion. Bail is afforded by S13, Art.III and
repeated in S7, Rule 114 of the RoC. The general rule is, before conviction, any person is bailable
unless he is charged with a capital offense (death penalty) or with an offense punishable with RP
or life imprisonment, and the evidence of guilt is strong. Thus, from the moment he is placed
under arrest, or is detained or restrained by officers, he can claim his right to bail and retains
this right unless charged with the said offenses punishable by said penalties and evidence of guilt
is strong.

As a result, all criminal cases within the jurisdiction of MeTC, MTC or MCTC are bailable as they
do not have jurisdiction to try capital offenses or offenses punishable by RP or LI. Bail is
discretionary 1) upon conviction by RTC of an offense not punishable by death, RP, or LI, or 2) if
the RTC has imposed a penalty of imprisonment exceeding 6 years, provided none of the
circumstances under par.3, S5, Rule 114 is present:
(a) That he is a recidivist, quasi-recidivist, or habitual delinquent, or has committed the
crime aggravated by the circumstance of reiteration; (b) That he has previously escaped
from legal confinement, evaded sentence, or violated the conditions of his bail without
valid justification; (c) That he committed the offense while under probation, parole, or
conditional pardon; (d) That the circumstances of his case indicate the probability of flight
if released on bail; or (e) That there is undue risk that he may commit another crime during
the pendency of the appeal.

3. Bail in RP, LI, or death offenses is subject to judicial discretion. Determining if guilt is strong
in criminal cases involving such offenses lies within the discretion of the trial court. But such
discretion may be exercised only after the hearing called to ascertain the degree of guilt for the
purpose of whether he should be granted bail. Bail cannot be allowed unless there is hearing
with notice to the prosecution. The hearing, which may be summary or otherwise in the
discretion of the court, should primarily determine if evidence of guilt is strong. A summary
hearing means such brief and speedy method of receiving and considering the evidence of guilt as
is practicable and consistent with the purpose of hearing which is merely to determine the weight
of evidence for purposes of bail. The court does not try the merits and will confine itself to
receiving evidence which has reference to substantial matters, avoiding unnecessary thoroughness
in the examination and cross examination.

4. Enrile’s poor health justifies his admission to bail.


Enrile averred that since there are 2 mitigating circumstances (over 70 yo and voluntary surrender),
the maximum possible penalty is only RT. The Prosecution, in its opposition, said that the
constitution deems as relevant only that he is “charged with an offense punishable by.” It is thus
the maximum penalty that has bearing and not the possibility of mitigating circumstances being
appreciated.
But We do not determine whether Enrile’s averment could entitle him to bail since this
determination, being primarily factual in context, is ideally to be made by the trial court.

Nonetheless, the Court is guided by the principal purpose of bail, which is to guarantee appearance
of the accused at trial or whenever required by the court. The Court is also mindful of the PH’s
responsibility in the international community arsing from the national commitment under the
UDHR to “uphold fundamental human rights. “The PH has the responsibility of protecting and
promoting the right of every person to liberty and due process, xxx.” PH is under obligation to
“make available to every person under detention such remedies which safeguard their fundamental
right to liberty” including right to bail. This national commitment has authorized bali not only to
those charged in criminal proceedings but also to extraditees upon a clear and convincing showing
that 1) the detainee will not be a flight risk or danger to the community, and 2) there are special,
humanitarian, and compelling circumstances.

Enrile’s social and political standing and his having immediately surrendered upon being
charged indicate that his risk of flight or escape is highly unlikely. His personal disposition from
the onset of his indictment has demonstrated his utter respect for the legal processes of this
country. At an earlier time many years ago when he had been charged with rebellion with murder
and multiple frustrated murder, he already evinced a similar personal disposition of respect for the
legal processes and was granted bail. With his solid reputation in both public and private lives, his
long years of public service, and history’s judgment of him being at stake, he should be granted
bail.

His fragile state of health also presents another compelling justification for his admission to bail.
He is suffering from hypertension, atherosclerotic cardiovascular disease, arrhythmia, etc. Dr.
Gonzales attested that his medical conditions could pose significant risks to Enrile’s life.

Bail for the provisional liberty of accused, REGARDLESS OF THE CRIME CHARGED,
should be allowed independently of the merits of the charge, provided his continued
incarceration is clearly shown to be INJURIOUS to his HEALTH or to ENDANGER his
LIFE. Denying him bail despite imperiling his health and life would not serve the true objective
of preventive incarceration during trial.

Granting bail to Enrile for these reasons is not unprecedented. As already held in Dela Rama v.
The People’s Court: unless allowance of bail is forbidden by law, the illness of the prisoner,
independently of the merits of the case, is a circumstance, and the humanity of the law makes it
a consideration which should, regardless of the charge and stage of the proceeding, influence
the court to exercise its discretion to admit the prisoner to bail.

Granting bail to Enrile will let him have his medical condition properly addressed. This will not
only aid in his adequate preparation of his defense, but, more importantly, will guarantee his
appearance in court for trial. Thus, there was GADALEJ on the Sandiganbayan’s part. Bail was
set at P1M.

LEONEN, J. Dissenting
Enrile’s release for medical and humanitarian reasons was not the basis of his prayer in his motion
to fix bail filed with the Sandiganbayan. Neither did he base his prayer for bail in this petition on
his medical condition. The grant of bail, therefore, by the majority is a special accommodation for
Enrile. It is based on a ground never raised before the Sandiganbayan or in the pleadings filed
before this court. It is not the duty of an impartial court to find what it deems a better argument for
the accused at the expense of the prosecution and the people they represent.

The allegation that Enrile suffers from medical conditions requiring special treatment is a question
of fact. We cannot take judicial notice of the truth contained in a certification coming from one
doctor. This doctor has to be presented as an expert witness who will be subjected to direct and
cross-examination. Rebutting evidence presented by the prosecution should also be considered.
All this would be proper before the sandiganbayan, which did not consider any of this as Enrile
insisted that he was entitled to bail as a matter of right on grounds other than his medical condition.

Also, the majority opinion, other than the invocation of a general human rights principle, does
not provide clear legal basis for the grant of bail on humanitarian grounds. Bail for
humanitarian considerations is neither presently provided in our RoC nor found in any statute or
the Constitution. This case would put pressure on all trial courts and the sandiganbayan that will
predictably be deluged with motions to fix bail based on humanitarian considerations. They will
have to decide whether this is applicable, without guidance, whether bail should be granted
because of advanced age or other diseases, whether this is applicable only to senators and former
presidents charged with plunder and not to those accused of drug trafficking, etc. and other crimes
punishable by RP or LI. They will decide based on personal discretion causing petitions for
certiorari to be filed before this Court. This will usher an era of selective justice not based on clear
legal provisions, but one that is unpredictable, partial, and solely grounded on human compassion.

1. The provisions on bail provide a balance between the presumption of innocence and due process
rights of the state to be able to effect the accused’s prosecution. That balance is not exclusively
judicially determined. The constitution frames that judicial discretion.

Enrile insists that this Court justify that bial be granted without hearing before the sandiganbayan
on whether evidence of guilt is strong. Building on consistent precedent, the sandiganbayan
correctly denied the motion to fix bail for being premature. This could not have been the basis for
declaring that sandiganbayan gravely abused its discretion when it denied bail.

2. The sandiganbayan did not commit gadalej when it failed to release Enrile on bail for medical
or humanitarian reasons. He did not ask for bail because of his medical condition or humanitarian
reasons. Neither Enrile nor the prosecution developed their arguments on this point. Yet, it now
becomes the very basis for Enrile’s grant of bail.

The ponencia does away with petitioner’s entire argument, stating that “we do not now determine
the question of whether or not Enrile’s averment on the presence of the two mitigating
circumstances could entitle him to bail xxx.” Despite brushing aside all of petitioner’s argument,
the majority, instead of denying the certiorari, grants it on some other ground not even argued nor
prayed for by petitioner. The majority grants bail merely on the basis of the certification by Dr.
Gonzales stating that Enrile is suffering from numerous debilitating conditions. This certification
was submitted as an annex to a manifestation regarding the remoteness of the possibility of flight
of the accused not for purposes of asking for bail due to such ailments.

Nowhere in the rules of procedure do we allow bail based on judicial notice of a doctor’s
certification. In doing so, we effectively suspend our rules on evidence by doing away with cross-
examination and authentication of Dr. Gonzales’ findings. Enrile’s ailments are not matters of
public knowledge or are capable of unquestionable demonstration. His illness is not a matter of
general notoriety (such that we may take judicial notice thereof).

Assuming the medical ailments of Enrile are relevant issues for bail, the prosecution is now
deprived of a fair opportunity to present any evidence that may rebut the findings of Dr.
Gonzales. Due process requires that we remand this matter for a bail hearing to verify
Gonzales’ findings and to ensure that that is still the condition that prevails at present. The better
part of prudence is that we follow strictly our procedures for bail. The matter to determine is
whether evidence of guilt is strong. This is to be examined when a hearing is granted as a
mandatory manner after a petition for bail is filed by accused. the medical condition of accused
should be pleaded and heard.

3. Assuming without conceding that Enrile suffers from illnesses requiring immediate medical
attention, this court has not established clear guidelines for such releases. The closest that the
majority reaches for a standard is bail for the provisional liberty of the accused, “regardless of the
crime charged, should be allowed independently of the merits of the charge, provided his continued
incarceration is clearly shown to be injurious to his health xxx.” To see the logical fallacy, we
break the argument down to its premises:
Premise: There are those whose continued incarceration is clearly shown to be injurious to
their health OR whose lives are endangered due to incarceration.
Premise: Petitioner is suffering from some ailments.
Therefore: Petitioner should be released.

Alternative detention is granted upon a clear showing before the trial court or sandiganbayan that
the physical condition of the accused, proven thru evidence in court, is absolutely requiring
medical attention that could not be accommodated within the current custodial arrangements. The
sandiganbayan already issued resolutions allowing Enrile to remain at PNP General Hospital and
continue medical examinations until further orders. It authorized Enrile to be brought to any
hospital immediately if he exhibits symptoms that cannot be treated at the PNPGH subject only to
reportorial requirements to the court. In granting bail to Enrile, we are in effect declaring that
sandiganbayan’s decisions in relation to its supervision of the accused’s detention were tainted
with grave abuse. But these orders were not subject of this certiorari.

To the sandiganbayan, Enrile does not seem to suffer from a unique debilitating disease whose
treatment cannot be provided for by our detention facilities and temporary hospital arrest in
accordance with their order. How the majority arrived at a different conclusion has not been
thoroughly explained. It is unclear if this privilege would apply to all who have similar
conditions and also undergoing trial for plunder. It is unclear if Enrile’s incarceration aggravates
his medical conditions or if his conditions are simply that which come with advanced age.
The majority has not set specific bases for finding that the medical condition of petitioner entitles
him to treatment different from all those who are now under detention and undergoing trial for
plunder. There is no showing as to how grave his conditions are in relation to the facilities that are
made available to him. There is also no showing as to whether any of his medical ailments is
actually aggravating in spite of the best care available. If his health is deteriorating, there is no
showing that it is his detention that is the most significant factor or cause for such deterioration.

Usually, when there is a medical emergency that would make detention in the hospital necessary,
courts do not grant bail. They merely modify the conditions for the accused’s detention. There is
now no clarity as to when special bail based on medical conditions and modified arrest should
be imposed.

There is also no guidance as to whether this special bail based on medical condition is applicable
only to those of advanced age and if that advanced age is beyond 90 or 91 yo etc. The majority has
perilously set an unstated if not ambiguous standard for the special grant of bail on the ground
of medical conditions. Neither is there clarity in the majority opinion as to the conditions for this
special kind of bail.

The crime charged in petitioner's case is one where the imposable penalty is reclusion perpetua.
The Constitution and our rules require that bail can only be granted after granting the prosecution
the opportunity to prove that evidence of guilt is strong. The special grant of bail, due to medical
conditions, is unique, extraordinary, and exceptional. There is no rule on whether the grant of
provisional liberty based on humanitarian considerations extends even after the medical
emergency has passed.

4. There is no evidentiary basis for the P1M as the amount of bail.

5. There is no specific binding international law provision that compels this court to release Enrile
given his medical condition. The UDHR is a general declaration to uphod the value and dignity of
every person. It does not prohibit the arrest of any accused based on lawful causes. It only implies
that any arrest or detention must be carried out in a dignified and humane manner.

It is true that the constitution is replete with provisions on both respect for human dignity and
protection of human rights. But the mere invocation of the broadest concept of human rights is not
shibboleth. It should not be cause for us to be nonchalant about other constitutional and statutory
provisions and the norms in the RoC. The mere invocation of human rights does not mean that the
Rule of Law is suspended. It is not a shortcut to arrive at the conclusion or result that we want.

6. Plunder is not the only crime punished with RP or LI. There is parricide etc. under the RPC and
carnapping with homicide or rape etc. under special laws.

Sec.14- CRIMINAL DUE PROCESS


174. Bayani Alonte v. Hon. Savellano, GR 131652, March 9, 1998, Vitug, J.(Criminal Due
Process)
FACTS:
An information for rape was filed against Bayani Alonte, a mayor of Biñan, Laguna, and
Buenaventura Concepcion on the complaint of Juvielyn Punongbayan. Juvielyn, thru counsel and
Assistant Chief State Prosecutor (ACSP) Guiyab, filed with the OCA a petition for change of
venue to transfer the case to the Manila RTC. During pendency of this petition, Juvielyn, assisted
by her parents and counsel, executed an affidavit of desistance. The SC granted the change of
venue.

In the Manila RTC, Juvielyn submitted a “compliance” where she reiterated her decision to abide
by her affidavit of desistance. Judge Savellano found probable cause and issued arrest warrants for
petitioners Alonte and Concepcion. Both petitioners pleaded not guilty. The parties waived pre-
trial. Per Judge Savellano, both parties agreed to proceed with the trial on the merits. But according
to Alonte, Savellano allowed the prosecution to present evidence relative only to the question of
voluntariness and validity of the affidavit of desistance.

After arraignment, the prosecution presented Juvielyn, who affirmed the voluntariness of her
affidavit of desistance. She said that she was compelled to desist as she was being harassed by
media, that no pressures nor influences were exerted upon her, and that she did not receive a single
centavo from anybody to secure the affidavit of desistance. Juvielyn’s parents affirmed their
signatures on the affidavit of desistance and their consent to their daughter’s decision to desist.
ACSP Campomanes manifested that in light of the decision of private complainant and her parents
not to pursue the case, the state had no further evidence against the accused to prove their guilt.
She moved for the dismissal of the case against both Alonte and Concepcion.

Judge Savellano said that the case was submitted for decision. Alonte filed an urgent motion to
admit to bail and a plea to resolve the motion for bail. Both were unacted upon by Savellano. He
filed a fifth motion for early resolution of the application for bail which were also unacted upon.
Savellano convicted Alonte and Concepcion of Rape and sentenced them to RP. Alonte moved
for reconsideration. Without waiting for its resolution, Alonte filed this case for certiorari, habeas
corpus, bail, recusation of Savellano, and for disciplinary action against him. Concepcion also filed
his own certiorari and mandamus.

ISSUE:
Whether a conviction without the prosecution being given opportunity to present evidence due to
the affidavit of desistance of the private complainant in the criminal case for rape against Alonte
and Concepcion may be upheld.
HELD: NO.
According to Judge Savellano, the 2 accused did not present counteravailing evidence during trial.
They did not take the witness stand to refute under oath the truth of the contents of the desistance
but instead rested and submitted the case for decision merely based on the affidavit of desistance
which, to them, was sufficient enough for their purposes. But according to petitioners, there was
no such trial as Savellano, aside from the arraignment. only determined the validity of the affidavit
of desistance.

S14 (1) (2) of Art. III provides the fundamentals:


"(1) No person shall be held to answer for a criminal offense without due process of law.
"(2) In all criminal prosecutions, the accused shall be presumed innocent until the contrary
is proved, and shall enjoy the right to be heard by himself and counsel, to be informed of
the nature and cause of the accusation against him, to have a speedy, impartial, and public
trial, to meet the witnesses face to face, and to have compulsory process to secure the
attendance of witnesses and the production of evidence in his behalf. However, after
arraignment, trial may proceed notwithstanding the absence of the accused provided that
he has been duly notified and his failure to appear is unjustifiable."

Due process in criminal proceedings, in particular, require: (a) that the court or tribunal trying the
case is properly clothed with judicial power to hear and determine the matter before it; (b) that
jurisdiction is lawfully acquired by it over the person of the accused; (c) that the accused is given
an opportunity to be heard; and (d) that judgment is rendered only upon lawful hearing. These
are mandatory and indispensable.

The order of trial in criminal cases is clearly spelled out in S3, Rule 119:
"(a) The prosecution shall present evidence to prove the charge and, in the proper case,
the civil liability. "(b) The accused may present evidence to prove his defense, and
damages, if any, arising from the issuance of any provisional remedy in the case. "(c) The
parties may then respectively present rebutting evidence only, unless the court, in
furtherance of justice, permits them to present additional evidence bearing upon the main
issue. "(d) Upon admission of the evidence, the case shall be deemed submitted for
decision unless the court directs the parties to argue orally or to submit memoranda. "(e)
However, when the accused admits the act or omission charged in the complaint or
information but interposes a lawful defense, the order of trial may be modified
accordingly."

Each step in the trial process serves a specific purpose. The presumption of innocence requires
that an accused be given sufficient opportunity to present his defense. Any deviation from the
regular course of trial should always take into consideration the rights of all the parties to the case.

Savellano claims that none of the defense counsel interposed an intention to cross-examine
Juvielyn even after she attested to the truth of her two affidavits- one detailing the rape and the
other attempts to buy her desistance. Since the opportunity was missed/not used, it is waived. But
waiver must be positively demonstrated. The standard of waiver requires that it not only be
voluntary, but must be knowing, intelligent, and done with sufficient awareness of the relevant
circumstances and likely consequences. Mere silence should not be construed as waiver, and the
courts must indulge every reasonable presumption against waiver.

These were a few of the deviations from what otherwise should have been the regular course of
trial: 1) petitioners have not been directed to present evidence to prove their defenses nor dates
scheduled for that purpose, 2) the parties have not been given opportunity to present rebutting
evidence, and 3) petitioners have not admitted the act charged in the information to justify any
modification in the order of trial. There can be no short-cut to the legal process, and there can be
no excuse for not affording an accused his full day in court.
This case must be remanded for further proceedings. All related proceedings therein, including
the petition for bail, should be subject to the proper disposition of the trial court.

175. Saturnina Galman et al. v. Sandiganbayan, GR 72670, September 12, 1986, Teehankee,
C.J. (Criminal Due Process)
FACTS:
Ninoy Aquino was allowed in May 1980 to leave the country to undergo successful heart surgery.
After 3 years of exile and despite the regime’s refusal to give him a passport, he sought to return
home. He was to be killed while under escort away by soldiers from his plane that had just landed
at the MIA. He was shot point-blank into the back of his head notwithstanding that the airport was
ringed by airtight security of close to 2000 soldiers. The military investigators reported within 3
hours that the man who shot Aquino was a communist-hired gunman, revealed days later as
Rolando Galman, and that the military escorts gunned him down in turn. The military later filmed
a re-enactment of the killing scripted according to this version and replayed it on all TV channels.
Marcos instantly accepted the military version. To induece disbelief that the military had a hand
in the killing, the president said in a nationally televised press conference that “if the purpose was
to eliminate Aquino, this was not the way to do it.”

Marcos was constrained to creat a Fact Finding Board 3 to investigate the “treacherous and
vicious assassination of former Sen. Benigno Aquino, Jr.” After 125 hearing days, with the
testimonies of 194 witnesses recorded in 20,377 pages of transcripts,the minority and majority
reports were submitted to Marcos. Both reports were one in rejecting the military version of events
that Rolando was a NPA-hired assassin stating that “the evidence shows that Rolando Galman had
no subversive affiliations.” “Only the soldiers in the staircase with Sen. Aquino could have shot
him.” “Galman, the military’s fall guy, was not the assassin of Sen. Aquino” and that the “SWAT
troopers who shot Galman and Aquino’s escorts deliberately and in conspiracy gave a perjured
story to us regarding the alleged shooting.” The only difference between the two reports is that the
majority report found all 26 respondents in this case involved in the military conspiracy while the
minority report only found 6 of them who were on the service stairs while Aquino was descending.

Petitioners Saturnina Galman and Reynaldo Galman, mother and son respectively, of Rolando
Galman, and 29 other petitioners, filed the present action alleging that respondents Tanodbayan
and Sandiganbayan committed serious irregularities constituting mistrial and resulting in
miscarriage of justice and gross violation of the constitutional rights of petitioners and the
sovereign people of PH to due process of law. They claim that the Tanodbayan did not represent
the interest of the people when he failed to exert genuine and earnest efforts to present important
evidence for the prosecution and that the Sandiganbayan justices were biased in favor of
accused. petitioners prayed for a TRO restraining Sandiganbayan from rendering a decision on
the merits in the cases and that judgment be rendered declaring a mistrial and nullifying the
Sandiganbayan proceedings and ordering a retrial by an impartial tribunal by an unbiased
prosecutor.

The SC issued the TRO. It required the Tanodbayan to submit a copy of his 84-page memorandum
for the prosecution as filed in the Sandiganbayan, the signature page of which alone was submitted.
10 days later, the SC dismissed the petition and lifted the TRO. Petitioners filed an MR. the
Sandiganbayan acquitted all accused and declaring them innocent and absolving them of any
civil liability. Sandiganbayan in effect convicted the very victim Rolando Galman who was not on
trial as the assassin of Ninoy contrary to the very information and evidence submitted by the
prosecution. The SC then denied the MR.

Petitioners filed a second MR based on the revelations of Deputy Tanodbayan Herrera as reported
in the March 6, 1986 issue of Manila Times “Aquino Trial a Sham” that Marcos, then president,
had ordered Sandiganbayan and Tanodbayan Fernandez and the prosecution to whitewash the
criminal cases and acquit them. The SC admitted the second MR.

The office of the tanodbayan wanted to file against 26 respondnets in the Agrava Board majority
report the crime of double murder for the death of Aquino and Rolando Galman. When
Malacañang learned of the impending filing of the charge, he summoned Justice Fernandez, the 3
members of the investigating panel, and justice Pamaran of the Sandiganbayan to a conference in
Malacañang. Marcos initially expressed his disagreement with the recommendation of the
investigating panel. But later, Marcos was convinced of the advisability of filing the murder charge
so that, after being acquitted as planned, the accused may no longer be prosecuted due to double
jeopardy. To ensure that not all accused would be denied bail, Marcos directed that several accused
be charged as accomplices and accessories. Marcos ordered that the case be handled personally by
Justice Pamaran who should dispose of it in the earliest possible time. Marcos’ instructions were
followed to the letter. The Court-appointed three-member commission thus recommended that a
mistrial be declared as the proceedings in the sandiganbayan was vitiated by lack of due process.

ISSUE:
Whether a trial where the judge and prosecutor are ordered by the president Marcos to eventually
acquit all accused, where the trial is monitored constantly and pervasively by Marcos who
suppressed witnesses for the prosecution, and where the decision of the sandiganbayan after trial
followed to the letter Marcos’ instructions and acquitted the 26 respondents who thus neither incur
“criminal nor civil liability” is a valid trial.
HELD: NO.
The Court adopts the Report that Marcos had stage-managed from Malacañang a “scripted and
predetermined manner of handling and disposing of the Aquino-Galman murder case and that the
prosecution and justices who tried and decided the case acted under compulsion of pressure beyond
their capacity to resist. The report is duly substantiated by the evidence.

The following are brief excerpts.


1. The changing of the original Herrera panel draft resolution charging all 26 accused as principals
by charging only 17 as principals, 8 others as accessories, and the civilian as accomplice, and
recommending bail for the latter 2 categories.

2. Suppression of vital evidence and harassment of witnesses.


3. The discarding of the affidavits executed by US airmen.
4. 9 proposed rebuttal witnesses not presented.
5. failure to exhaust available remedies against adverse developments.
6. The assignment of the case to Justice Pamaran. Justice Herrera testified that Marcos ordered
Pamaran point-blank to handle the case.
7. The custody of the accused, their confinement in a military camp, instead of in a civilian jail.
8. The monitoring of proceedings and developments from Malacañang and by Malacañang
personnel. Cameras boldly carrying the label of OP were installed in the courtroom to monitor the
proceedings and take remedial measures as may be necessary. There was a “war room” in the
sandiganbayan where military and Malacañang personnel stayed to keep track of proceedings.
9. Partiality of sandiganbayan betrayed by its decision. Marcos wanted all 26 accused acquitted.
The evidence presented by the prosecution was totally ignored and disregarded. The decision had
to pronounce them innocent and thus incur “neither criminal nor civil liability.” It is a rare
phenomenon to see a person accused of a crime to be favored with such total absolution.

The record shows that from beginning to end, Marcos used the overwhelming resources of the
government and his authoritarian powers to corrupt and make a mockery of the judicial process in
the Aquino-Galman murder cases. the acquittal accomplished the two principal objectives of
satisfaction of the public clamor for the suspected killers to be charged in court and of giving
them through their acquittal the legal shield of double jeopardy.

His obsession to acquit the respondents led to several “first” which would otherwise be
inexplicable.
1. He repudiated the findings of the very fact finding board he himself appointed to investigate the
Ninoy assassination.
2. He received the minority report one day ahead of the four majority members.
3. He totally disregarded the board’s majority and minority findings and publicly insisted that the
military’s fall guy Rolando was the killer of Ninoy.
4. The sandiganbayan decision in effect convicted Rolando as Ninoy’s assassin notwithstanding
that he was not on trial but the victim according to the very information filed and evidence to the
contrary submitted.
5. Justice Pamaran’s ponencia acquitted all 26 accused and that they incur neither criminal nor
civil liability despite evidence based on the fact finding board’s declaration that the soldier’s
version of Galman being Aquino’s killer being a perjured story.

The SC cannot permit such a sham trial and verdict and travesty of justice to stand unrectified.
The Court is constrained to declare the sham trial a mock trial- the non-trial of the century- and
that the predetermined judgment of acquittal was unlawful and void ab initio.

1. No Double Jeopardy. Double jeopardy cannot be invoked against this Court’s setting aside of
the trial court’s judgment of dismissal or acquittal where the prosecution which represents the
sovereign people in criminal cases is denied due process. Where the prosecution is deprived of a
fair opportunity to prosecute and prove its case, its right to due process is violated. Where there is
violation of basic constitutional rights, courts are ousted of jurisdiction. Where the denial of
the fundamental right of due process is apparent, a decision rendered in disregard of that right
is void for lack of jurisdiction.

Legal jeopardy attaches only 1) upon a valid indictment, 2) before a competent court, 3) after
arraignment, 4) a valid plea having been entered, 5) the case was dismissed or terminated
without the express consent of the accused. “The lower court was not competent as it was ousted
of jurisdiction when it violated the right of the prosecution to due process.” In effect, the first
jeopardy was never terminated, and the remand of the criminal case for further hearing merely
continues the first jeopardy and does not expose the accused to a second jeopardy.

Here, the prosecution and sovereign people were denied due process of law with a partial court
and biased tanodbayan under the constant monitoring and pressure exerted by the authoritarian
president to assure the carrying out of his instructions. A dictated, coerced, and scripted verdict
of acquittal like in the case at bar is a VOID judgment. In legal contemplation, it is no judgment
at all.

The general rule that an acquittal ends the case which cannot be appealed or reopened without
being put in double jeopardy presupposes a valid judgment. Since the judgment here is void for
lack of jurisdiction, no double jeopardy attaches. For justice to prevail, the scales must balance. It
is not to be dispensed for the accused alone.

2. With the declaration of nullity of the proceedings, the cases must no be tried before an impartial
court with an unbiased prosecutor.

PRESUMPTION of INNOCENCE
176. Elvira Agullo v. Sandiganbayan, GR 132926, July 20, 2001, Buena, J. (Presumption of
Innocence)
FACTS:
Agullo was charged with malversation of public funds. She was the disbursing officer of the then
Ministry of Public Works and Highways (MPWH). The charge germinated from an audit which
found P26,404,26 shortage was discovered on Agullo’s accountability. On the same day, Agullo
was informed of the cash shortage and required, thru letter of demand, to produce immediately the
missing funds. Agullo complied with the directive, explaining that the cash shortage was due to a
“fortuitous event” where the amount could have been stolen by somebody on the day she suffered
a stroke in Tacloban. Agullo professed her innocence and denied having malversed or converted
the public funds for her own personal use or benefit. She admitted the cash shortage.

The defense, to overturn the presumption of malversation, presented various witnesses. The
defense also presented various documentary evidence. Agullo, while testifying, maintained her
innocence. She said that in the morning of Oct. 21, 1985, she reported for work and prepared an
inventory of her cash accountability as disbursing officer of MPWH regional office. She received
13 checks in the form of cash advances in her name totaling P26,076.87, representing salaries of
MPWH officials and employees. At 1:30 pm, Agullo went to the PNB bank to encash the checks.
She put the money inside a PNB envelope which she placed inside her bag. From PNB, Agullo,
who boarded the official vehicle driven by Veridiano to go back to the office, felt dizziness and
nausea. She thus requested Veridiano to drop her off at her residence.

The next day, she strove to report for work, bringing the bag with the money. She was able to reach
a distance of 50m away from her residence when she was stricken with deep chest pain and
experienced dizziness. Her vision blurred and the right part of her body became heavy. She lost
consciousness. In the afternoon, she found herself in the hospital bed.

The sandiganbayan convicted her, striking down the defense as incredible and without basis.
ISSUE:
Whether a judgment of conviction for malversation may be sustained if it relies only on the
presumption of malversation under Art.217 (4) despite evidence to the contrary by the defense
where the prosecution’s evidence only comprise
HELD:
The pieces of evidence against Agullo do not fulfill the test of moral certainty and is not sufficient
to support a conviction. The prosecution’s evidence consisted solely of the Report of Cash
examination to prove the cash shortage of P26k. It also presented the Letter of Demand signed
by Auditing Examiner Gerez. Aside from these documents, the prosecution opted not to present a
single witness to buttress its bid for conviction and merely relied on the prima facie evidence of
malversation in Art.217, par.4 of the RPC. The judgment of guilt was anchored solely on the
presumption in Art. 217 which prima facie evidence, in turn, was rooted loosely on the
documentary evidence presented by the prosecution. These documents do not suffice to convict
petitioner beyond reasonable doubt.

The presumption in Art.217 is rebuttable, disputable by satisfactory evidence to the effect that
accused did not utilize the public funds or property for his personal use, gain, or benefit. If accused
can present evidence that can nullify any likelihood that he put the funds to personal use, the prima
facie case is negated.

In this case, the prosecution, whose burden was the task of establishing by proof beyond
reasonable doubt that Agullo committed the offense charged, mainly relied on the statutory
presumption and failed to present any substantial evidence to indicate that Agullo used the funds
for personal gain.

While the Sandiganbayan admitted that the conversion of the funds to personal use has indeed not
been proven, it leaped into the wrong conclusion that Agullo was guilty invoking the prima facie
evidence in Art.217 (4). While factual findings of the sandiganbayan are conclusive on this court,
the exceptions are, among others: : (1) the conclusion is a finding grounded entirely on speculation,
surmise and conjectures; (2) the inference made is manifestly mistaken; (3) there is grave abuse of
discretion; (4) the judgment is based on misapprehension of facts; and (5) the findings of fact of
the Sandiganbayan are premised on a want of evidence and are contradicted by evidence on record

The sandiganbayan’s conclusion that “there is no evidence to show that Agullo was carrying the
P26k when she collapsed” is without factual basis and not supported by evidence. The records
show, and as Agullo testified, that she had the money with her when she collapsed and suffered a
stroke. The sandiganbayan overlooked certain evidence of substance.

Agullo has satisfactorily overcome and rebutted by competent proof the prima facie evidence of
conversion. She presented evidence that satisfactorily prove that not a single centavo of the missing
funds was used for her own personal gain.

The records reveal that the prosecution admitted that Agullo suffered a stroke on the streets of
Tacloban. The allegation that no evidence exists as to the loss of public funds, this is belied by the
records as Agullo herself testified on the stand that she had the money subject of inquiry when she
collapsed. To us, this circumstance, coupled with the other peculiarities in this case and also
considering the failure of the prosecution to adduce other evidence to establish conversion, suffice
to make the mind uneasy as to Agullo’s guilt, notwithstanding the prima facie evidence
established by law against her, which by no means dispenses the need of proving guilt beyond
reasonable doubt.

A finding of prima facie evidence of accountability does not shatter the presumptive
innocence the accused enjoys because, before prima facie evidence arises, "certain facts [have
still to be] proved"; the trial court cannot depend alone on such an evidence, because precisely, it
is merely prima facie. It must still satisfy that the accused is guilty — beyond reasonable doubt —
of the offense charged. Neither can it rely on the weak defense the latter may adduce.

The sandiganbayan relied more on the flaws of the defense evidence, not on the strength of the
prosecution evidence. This is imperssible as the prosecution’s evidence cannot sustain a conviction
“in an unprejudiced mind.”

The purpose of the presumption of innocence is to balance the scales in what would otherwise be
an uneven contest between the lone individual pitted against the people of the PH and all the
resources at their command.

177. People v. Florentino del Mundo, GR 138929, October 2, 2001, Ynares-Santiago, J.


(Presumption of Innocence)
FACTS:
Del Mundo was charged with violation of the Dangerous Drugs Act of 1972 for selling,
distributing, and transporting marijuana. Since del Mundo was among those listed in their order of
battle against illegal drugs, a team of 4 policemen were instructed to go to said barangay upon info
received that he was selling drugs and to apprehend him. They saw del Mundo conversing with
another person. From a distance, they observed him hand something to the other person. When
the policemen approached, he hurriedly boarded his tricycle and sped away while his companion
fled on foot. Del Mundo was caught.

Del Mundo was asked to alight from his vehicle and then subjected to a body search. Finding no
illegal drugs or weapons, the officers searched his tricycle. They found a packaged wrapped in
newspaper inside a plastic bag and detected the scent of marijuana. They opened the package and
found marijuana. Del Mundo was arrested. The two policemen were presented as witnesses.

On the other hand, the defense presented only 1 witness, del Mundo himself. He vehemently denied
the allegations. He said that while plying a route on his tricycle, a man boarded his vehicle carrying
a plastic bag asking to be ferried somewhere. While the passenger was about to pay the fare, two
men aboard a motorcycle approached with firearms aimed at them. Fearing for their lives, del
Mundo drove away while his passenger fled. He was chased by the two policemen who frisked
him and searched his vehicle. They found the plastic bag left by the passenger. He denied that he
owned the bag and its contents. He was nonetheless arrested and incarcerated at the police station.

The RTC found that the apprehending police positively testified that the marijuana was confiscated
from the tricycle, that the police had no ill-motive for them to fabricate such a serious charge, and
the police are presumed to have regularly performed their duties. Del Mundo was convicted. Hence
this appeal.

ISSUE:
Whether the defense of del Mundo, accused of transporting and selling marijuana, that he did not
own the marijuana found in his tricycle and that he ran away from the approaching police officers
because they had their firearms pointed at him as they approached, may prevail over the testimony
of the apprehending officers that they saw him talking and handing something over to someone.
HELD: NO.
Del Mundo claims that the prosecution failed to prove that he was the owner of the marijuana and
was selling, distributing, or transporting the same with full knowledge that he was transporting a
prohibited drug. The fact that the officers chased the passengers showed that they were also in
doubt as to the real owner and where ownership of the drug is in doubt, the doubt must be resolved
in favor of accused. He also claims illegal warrantless arrest.

We affirm the conviction. The trial court did not err when it ruled that it was immaterial whether
del Mundo was the owner of the marijuana. Proof of ownership of the marijuana is not necessary
in prosecuting illegal drug cases. the defense that the marijuana belonged to the unidentified
passenger is too convenient an alibi to instill belief.

The DD Act punishes the acts of selling etc. Thus, when an accused is charged with illegal
possession or transportation, the ownership thereof is immaterial. It is sufficient that the
prohibited substance was found in del Mundo’s tricycle when he was apprehended. His bare denial
is an intrinsically weak defense. It is self-serving without weight in law. his lone testimony was
not substantiated by clear and convincing evidence and cannot prevail over the positive testimony
of the prosecution witnesses and the physical evidence. Apart from his solitary testimony, there is
nothing by way of credible evidence that the courts can rely on to even consider his defense.

Lack of knowledge or criminal intent and good faith are not exempting circumstances where the
crime charged is malum prohibitum. To warrant acquittal, del Mundo must show that his act of
transporting the package of marijuana in his tricycle was without intent to possess a prohibited
drug. Despite his protestations to the contrary, his reaction to the arrival of the policemen belied
his claim of innocence. If he did not know the package contained marijuana, he would not have
attempted to flee from the policemen. His story that the policemen had guns drawn and he fled out
of fear is contrary to human experience. An innocent person caught in such situation would more
likely stay and profess his innocence rather than further endanger his life by fleeing.

Flight is an indication of guilt. The flight of an accused, without credible explanation, would
be a circumstance from which an inference of guilt may be established “for a truly innocent person
would normally grasp the first available opportunity to defend himself and assert his innocence.”
An explanation, standing by itself, which is too trite and hackneyed to be accepted at face value,
since it is obviously contrary to human experience is insufficient to overcome the prima facie
evidence that accused had knowledge of his possession of prohibited drugs.

Del Mundo claims that his testimony should have raised reasonable doubt in the minds of the court.
But bare denials unsubstantiated by convincing evidence are not enough to engender reasonable
doubt particularly where the prosecution presents sufficiently telling proof of guilt. Absolute
certainty of guilt is not required for conviction. Moral certainty as to the presence of the elements
of the offense is sufficient as long as the identity of the offender is established.

That the marijuana was found in his tricycle is prima facie evidence of knowledge or animus
possidendi enough to convict in the absence of satisfactory explanation. The presumption of
innocence will NOT apply as long as there is some logical connection between the fact proved
and the ultimate fact presumed, and the inference of one fact from proof of another shall not
be so unreasonable as to be a purely arbitrary mandate. Thus, the onus of evidence to prove
absence of animus possidendi thus shifted to the possessor of the drugs.

Here, the evidence established beyond any doubt that del Mundo was in possession of the package
with marijuana. The possession gives rise to the disputable presumption under S3(j), Rule 131 that
he is the owner of the package and its contents. His mere denial is insufficient to overcome the
presumption. His defense, without convincing evidence, is viewed with disfavor as it can easily be
concocted and is the most common defense tactic employed in most drug cases.

The warrantless arrest and seizure was legal as he was caught attempting to flee from pursuing
policemen who were ordered to the specific place where del Mundo was suspected to be dealing
drugs. They found him in the act. The attempt to escape is indicative of guilt and bolstered the
policemen’s suspicion that he had either committed, was actually committing, or was attempting
to commit an offense when they found him talking to an unidentified person beside his tricycle.
The warrantless arrest was valid as the officers had reasonable grounds to believe that he was
dealing or transporting drugs having been informed by a reliable police informer and acting upon
orders of their superior. Even if it was invalid, having failed to move to quash the information
before arraignment, del Mundo is estopped from questioning the legality of his arrest.

178. People v. Cesar Galvez, GR 157221, March 30, 2007, Austria-Martinez, J. (Presumption
of Innocence)
FACTS:
Around 11pm of July 27, 1991, Danilo Perez, Rosalio Enojarda, Noel Cugal, Ricardo Francisco,
and Wilfredo Rellios took a break from making copra to eat leftover dinner inside the copra kiln
in the farm of Perez. When Enojarda stood up from the circle where they were eating to drink
water, shots rang out and Enojarda fell to the ground shouting “Dan, I am hit.” The rest of the
group took cover, crawling to different directions. After the attack, Rellios reported the incident to
the barangay captain and they brought Enojarda’s dead body to his family.

An information was filed against Cesar Galvez, PNP member, for murder. The prosecution
evidence showed that after Enojarda fell, Rellios, while in a crawling position, saw Galvez 5 meters
away holding an armalite and firing at their direction. Rellios did not recognize the companions of
Galvez as they were 9 meters away. When the firing stopped, one of the attackers passed by about
2 meters from where Perez was hiding and recognized Galvez, his cousin with the armalite.

Galvez put up alibi as defense, saying that he was at his father-in-law’s house.
The RTC found that while Enojarda was shot by 4 men, how many gunshot wounds he suffered
and what part of his body was hit was not shown. The day before the incident and on the day itself,
Galvez has not fired any firearms. The 5 empty shells of armalite found in the crime scene did not
crome from the M16 with serial number 117460, the gun issued to Galvez. The RTC concluded
that since Galvez did not fire the gun and the shells were not from his armalite, then the bullet that
killed Enojarda came from the gun fired by any of the 3 unidentified persons who were Galvez’
companions.

But the RTC found prosecution witnesses Rellios and Peres credible and the defense witness’
testimony full of loopholes. It convicted Galvez of murder based on conspiracy. His offer to have
the case settled out of court is also an indication of his guilt. The CA affirmed.

ISSUE:
Whether Galvez may be convicted of murder when it was not proved that he was the one who shot
the bullet that killed the victim and where the information failed to allege conspiracy among him
and his 3 other unidentified companions.
HELD: NO.
The Court resolves to acquit Galvez. Conspiracy must be alleged in the information to hold an
accused liable for the acts of his co-accused. Without such averment of conspiracy, an accused can
only be made liable for the acts committed by him alone and such criminal responsibility is
individual and not collective. Since conspiracy was not alleged, the prosecution must prove
Galvez’s direct participation in the killing. The prosecution failed to do so.

The prosecution witnesses never saw Galvez actually shoot the victim. They did not see Galvez
actually shoot but only presumed it was him after multiple shots came from different directions
and, after crawling away, saw Galvez. Based on the testimonies, the following circumstances are
established: 1) around 11pm, they were eating merienda when they were sprayed with gunfire. 2)
Enojarda was hit and fell. 3) The others ducked and crawled to seek cover. 4) 5 minutes after the
first burst of gunfire, Galvez, armed with an M16, was seen firing at Rellios, Perez, and their two
companions in the direction of the kiln. 5) About 20-25 minutes after the first burst of gunfire,
Galvez was again seen carrying an M16 with 3 armed companions who then left the crime scene.

These circumstances are not sufficient to establish the guilt of Galvez beyond reasonable doubt.
Other circumstances from the testimonies of the prosecution witnesses must be appreciated in
favor of Galvez. Both Perez and Rellios testified that they saw Galvez and 3 others minutes after
Enojarda was shot but did not see him before the shooting. Perez testified that only one shot hit
Enojarda. He did not see Galvez shoot at Enojarda and merely assumed that Galvez shot him .
Rellios only presumed that Galvez shot Enojarda. Perez testified that he had no misunderstanding
with Galvez and he does not know any motive why Enojarda was killed.

In considering both favorable and incriminating circumstances, it must borne in mind that the
information charged Galvez as the sole perpetrator of murder and there was no allegation of
conspiracy. The 3 armed companions were not included s John Does. Thus, the prosecution must
prove that Galvez was the sole author of the shot that killed Enojarda. The incriminating
circumstances do not point to Galvez as the sole perpetrator. The presence of the three armed men
raises the probability that any one of those men inflicted the fatal shot. There is no evidence that
Galvez was seen together with the three other armed men when Enojarda was hit. There is a
missing link precluding the Court from concluding that it was Galvez who shot Enojarda.

If the circumstances are capable of two or more explanations, one of which is consistent with the
innocence of the accused and the other consistent with his guilt, then the evidence does not fulfill
the test of moral certainty and is not sufficient to support a conviction and, thus, that which
is favorable to the accused should be considered.

Motive also becomes important when the evidence of the crime is purely circumstantial or
inconclusive. Here, Perez testified that he did not know of any motive on the part of Galvez to
kill Enojarda. This is a circumstances that should be taken in favor of Galvez.

The facts, as established by the circumstantial evidence, failed to exclude the possibility that
another person shot Enojarda. There were three other armed men, any one of whom could be
the culprit. The State, aside from showing the existence of a crime, has the burden of correctly
identifying the author of such crime. Both must be proved beyond reasonable doubt on the
strength of its evidence and without solace from the weakness of the defense.

The offer to compromise, while generally admissible as evidence against the party making it, was
not admitted in evidence. The only basis by the RTC in concluding that Galvez made an offer of
compromise is when the wife of the victim and Galvez with his counsel appeared together in court
once. Where the evidence on the alleged offer of compromise is amorphous, it shall not benefit the
prosecution. The accused is permitted to show that the offer was not made under consciousness of
guilt, but merely to avoid the inconvenience of imprisonment or for some other reason to justify a
claim by the accused that the offer was not an admission of guilt or an attempt to avoid the legal
consequences which would ordinarily ensue therefrom. The compromise was also not shown to
have been offered under consciousness of guilt. He was not given the opportunity to explain that
it was given for some other reason to justify that it was not an admission of guilt or just an attempt
to avoid its legal consequences.

Thus, the presumption of innocence prevails over the alleged implied admission of guilt. In
case two or more presumptions are involved, one tending to demonstrate guilt and the other
innocence, it is necessary to examine the basis for each presumption and determine what logical
or social basis exists for each presumption, and then determine which should prevail over the other.
The existence of presumption of guilt does not in itself destroy the presumption of innocence
unless the inculpating presumption with all the evidence is sufficient to overcome the
presumption of innocence by proving guilt beyond reasonable doubt. The presumption of
innocence is not overcome by mere suspicion or conjecture, a probability that defendant
committed the crime, nor by the fact that he had opportunity to do so.

The prosecution has not proven guilt beyond reasonable doubt.

The prosecution argues that the negative paraffin test do not prove that Galvez did not fire a gun.
The argument that the negative result of the paraffin test does not conclusively prove that Galvez
did not shoot the victim, the same negative results cannot be used as circumstantial evidence
against Galvez to prove that he shot Enojarda. To do otherwise would violate presumption of
innocence.

That Galvez, a police officer who could have justified his presence in the crime scene with a lawful
purpose yet put up alibi, and that he did not present his wife and father-in-law as witnesses to
corroborate his alibi, pertain to the weakness of Galvez’s alibi which may cast doubt on his
innocence. But these do not prove guilt beyond reasonable doubt. Although an accused must
satisfactorily prove his alibi, the burden in criminal cases still rests on the prosecution to prove
guilt. The prosecution evidence must stand or fall ON ITS OWN WEIGHT and cannot draw
strength from the weakness of the defense. Unless the prosecution overturns the presumption of
innocence, it remains. Courts must judge guilt or innocence based on facts and not on mere
conjectures, presumptions, or suspicions. Even if the defense is weak, it is inconsequential if, in
the first place, the prosecution failed to discharge the onus of his identity and culpability.

While it is lamentable that because of the lapses of the prosecution, justice could not be rendered
for the untimely death of Enojarda, justice would also not be served with Galvez’s conviction. The
protection by the BoR is bestowed upon all individuals without exception, regardless of race,
color, creed, gender, or political persuasion, whether privileged or less privileged. The accused is
not called to disprove what the prosecution has not proved.

178. People v. Sergio Bato and Abraham Bato, GR 113804, January 16, 1998, Panganiban,
J. (Presumption of Innocence)
FACTS:
Sergio and Abraham Bato, brothers, were charged with murder. They pleaded not guilty. The RTC
convicted them. The CA affirmed their guilt and certified the case to the SC. During pendency of
the appeal, Sergio died, extinguishing criminal liability. Hence, this decision only pertains to
Abraham.

Ernesto Flores Jr., son of the victim, testified for the prosecution. His testimony narrated the
following facts: he and his father Ernesto Flores Sr. were going home from a barangay. While
passing by brgy. Hibucawan, they were called by Abraham and Sergio to join them in a drinking
spree. Ernesto Sr. accepted while Ernesto Jr. sat 2 meters away from his father. When his father
was drunk, appellants tied him (Ernesto Sr.) with his hands placed at the back. Later, he saw
appellants bring his father somewhere else. Seeing his father being held, he ran away, as he was
afraid to be taken also. The next day, his father was found dead at the Binaha-an River 5km away.
He reported the incident.

The defense raised denial. It presented Pfc. Montanejos who affirmed that the entry he made in the
police blotter did not mention the accused as suspects. He also said that it was the barangay captain
who reporte the incident, contradicting Ernesto Jr. who claimed that he did so.

The RTC said that Ernesto Jr. positively identified both accused. The CA affirmed, adding that the
prosecution evidence constituted more than sufficient incriminatory circumstances. Ernesto Jr.’s
testimony is clear and positively identified both accused. He could not have been mistaken as he
had known them long before the commission of the offense.
ISSUE:
Whether Abraham Bato may be convicted based on the sole testimony of the victim’s son, Ernesto
Jr., that Abraham and Sergio drank with Ernesto Sr. for two hours, then tied the hands of Ernesto
Sr. and took him somewhere, and the next day he was found dead 5 km away.
HELD: NO.
The circumstantial evidence adduced by the prosecution fails to evoke moral certainty of guilt.
While in the absence of direct proof, a conviction may be based on circumstantial evidence, to
convict, these requisites must concur: 1) There is more than one circumstance, 2) the facts from
which the inferences are derived are proven, and 3) The combination of all the circumstances
is such as to produce a conviction beyond reasonable doubt. A conviction based on
circumstantial evidence can be upheld only if the circumstances proven constitute an unbroken
chain leading to one fair and reasonable conclusion, to the exclusion of any other, that the
accused are guilty. The circumstances proved must be consistent with the hypothesis that the
accused is guilty and, at the same time, inconsistent with any hypothesis other than that of guilt.

Aside from the doctor who conducted the post mortem examination, the only other witness of the
prosecution was Ernesto Jr. He established the following circumstances: (1) that the Bato brothers
invited the victim and his son for a drink; (2) that after two hours of drinking, said brothers
suddenly tied the hands of the older Flores and took him away, and (3) that the following day, the
body of the victim, which sustained several hack and stab wounds, was recovered at the Binaha-
an River, about five kilometers away from where he was last seen by the witness.

The authorship of the crime was not established beyond reasonable doubt. Ernesto Jr. admitted
that there was no bad blood between his father and accused. He asserted that there was no
altercation during the drinking spree. He made no statement that the Bato brothers at the time
carried any bladed weapon which could have been used to kill his father. He did not see where his
father was brought after they tied his hands. He failed to testify how his father was killed, who
killed him, or even when he was killed. These lacunae spawn doubts in the mind of a
reasonable person. Because the appellants tied the victim’s hands, can it be inferred that they
intended to kill him and actually killed him? What happened between the time the victim was tied
and the following morning when his body was found? The prosecution, in effect, asked the courts
merely to guess or to surmise that the accused must have killed the victim during the
interregnum. Conjectures, surmises, and suspicions cannot take the place of evidence particularly
where, as here, contrary suspicions, surmises, and queries can also be floated and believed.

It is also noteworthy that Ernesto Jr. did not attempt to attract the attention of other people nearby
or seek their aid. Instead, he ran home and related the events to his mother. Oddly, his mother
reacted not by reporting to police or even to the brgy. Chairman. They simply slept the night away!

Notwithstanding the presence of other persons nearby when appellants tied the hands of the victim,
the prosecution failed to present any other witnesses to corroborate Ernesto’s testimony. His
testimony was grossly insufficient and sorely in need of corroboration.

The totality of the prosecution evidence does not constitute an unbroken chain leading, beyond
reasonable doubt, to guilt. An accused shall be presumed innocent until the contrary is proven
beyond reasonable doubt. Where the state fails to meet the quantum of proof to overcome the
constitutional presumption, accused is entitled to acquittal regardless of the weakness or even
absence of his defense. By constitutional fiat, the burden of proof is vested in the prosecution.
In acquitting Abraham Bato, this Court is not decreeing that he did not participate in the
killing. It is merely ruling that the state failed to present sufficient evidence to overturn the
constitutional presumption of innocence.

179.Generoso Corpuz v. People, GR 74259, February 14, 1991, Cruz, J. (Presumption of


Innocence)
FACTS:
As supervising accounting clerk in the office of the provincial treasurer, Corpuz was designated
as acting supervising cashier in said office. He received collections, disbursed funds, and made
bank deposits and withdrawals pertaining to government accounts. His designation as acting
cashier was then terminated. A transfer of accountabilities was effected between Corpuz and his
successor. The certificate of turnover revealed a shortage of P72,823.08. a demand letter required
Corpuz to produce the amount but he was only able to pay P10k. Another demand letter was sent.
The amount was reduced by P12k thru payment to Corpuz of temporarily disallowed cash items
and deductions from salary before his dismissal from service.

A final demand letter was sent for P50,596.07. Since the demand was not met, an information for
malversation was filed against him. Corpuz insists that he is innocent since the shortage was
malversed by other persons. He claims that the P50k, bulk of the shortage, represented the
unliquidated withdrawal made by paymaster Pineda thru one of four checks encashed while
Corpuz was on official leave of absence. He avers he was later made to post the amount in his
cash book by acting deputy provincial treasurer Alunin and had no choice but to comply although
he had not actually received the amount.

Testifying for the prosecution, Pineda insists that he had liquidated all four checks after the
amounts thereof were disbursed, turning over to Corpuz the corresponding withdrawal vouchers,
paid vouchers, and payrolls. He said that Corpuz was not really absent on the dates in question but
was in fact the one who prepared the said checks. Acting provincial treasurer Martinez
corroborated Pineda’s testimony that Corpuz was not on official leave on the dates in question.
He said that while the check was encashed, the encashment was not immediately recorded in
Corpuz’s cashbook which was “one way of temporarily hiding the early detection of a shortage.”
Corpuz, when confronted of the shortage, had no explanation to offer.

Aluning denied he exerted pressure on Corpuz to post the shortage in Corpuz’s cash book. He said
that although the P50k check was already encashed, he discovered that it was not reflected in
Corpuz’s cash book. As superior, he required Corpuz to make the proper entry.

The Sandiganbayan said that there was no explanation of the P595.87 from the defense so Corpuz
must be held answerable therefor. As to the P50k, the Sandiganbayan convicted Corpuz. The P50k
check (958525) was only one of four checks issued and encashed for the same purpose. Except for
said check, all other 3 checks were duly entered. One check which was issued and encashed on the
same day as check 958525 was duly entered in his cashbook. Non-entry of the latter check on time
was a subtle way of camouflaging the embezzlement. Pineda, prosecution witness, swore that
he duly liquidated the proceeds of the 4 checks. Pineda also declared that he was not absent.
ISSUE:
Whether Corpuz may be convicted of malversation based on the testimony of his superior that he
did not enter the encashment of a check on time in Corpuz’s cashbook, of Pineda that he encashed
all 4 checks (one of which was the one malversed) but only 3 of which was timely entered in
Corpuz’s cashbook, and of Martinez who corroborated Pineda’s testimony that, contrary to
Corpuz’s claim that he was on official leave when the checks were encashed, he was actually
present.
HELD: YES.
There is no reason to disturb the factual findings based on substantial evidence absent any
exceptional circumstance to justify their reversal. Contrarily, the Court is convinced that the facts
point unmistakably to Corpuz’s guilt. The absence of a post-audit is not, as he contends, a fatal
omission. That is not a requirement to file an information for malversation as long as the prima
facie guilt is already established.

Corpuz’s claim that he is the victim of a “sinister design” to hold him responsible for a crime he
has not committed is less than convincing. His attempt to throw the blame on others only shows it
is he who is looking for a scapegoat.

The equipoise rule invoked by the petitioner is applicable only where the evidence of the parties
is evenly balanced, in which case the constitutional presumption of innocence should tilt the
scales in favor of the accused. There is no such equipoise here. The evidence of the prosecution
is overwhelming and has not been overcome by the petitioner with his nebulous claims of
persecution and conspiracy. The presumed innocence of the accused must yield to the positive
finding that he malversed the sum of P50,310.87 to the prejudice of the public whose confidence
he has breached. His conviction must be affirmed.

180. Bienvenido Marquez, Jr. v. COMELEC, GR 112889, April 18, 1995, Vitug, J.
(Presumption of Innocence)
FACTS:
Bienvenido Marquez, a defeated candidate for Governor in Quezon Province in the May 11, 1992
elections, filed this certiorari praying for the reversal of the COMELEC resolution which
dismissed his quo warranto against the winning candidate, private respondent Eduardo Rodriguez,
for being allegedly a fugitive from justice. He claims that at the time Rodriguez filed his
certificate of candidacy (COC), a criminal charge against him for 10 counts of insurance fraud or
grand theft of personal property was still pending before the municipal court of Los Angeles in
US. A warrant was issued for his arrest by said court. It has yet to be served on Rodriguez on
account of his alleged “flight” from that country.

After Rodriguez was proclaimed Governor-elect, Marquez filed quo warranto proceedings against
him before the COMELEC. The COMELEC dismissed the petition. Hence this certiorari. The core
issue is if Rodriguez is a “fugitive from justice” as disqualified by S40(e) of the LGC:

Sec. 40. Disqualifications. — The following persons are disquali8ed from running for any
elective local position: "xxx xxx xxx
"(e) Fugitive from justice in criminal or non-political cases here or abroad(.)
ISSUE:
What does “fugitive from justice” mean when S40(e) of the LGC disqualifies such persons in
criminal or non-political cases “here or abroad” from running for any elective position?
HELD:
Marquez claims that S40(e) is clear and needs no interpretation. “Fugitive from justice” includes
not only those who flee after conviction to avoid punishment but also those who, after being
charged, flee to avoid prosecution. This definition truly finds support from jurisprudence and may
be so conceded as expressing the general and ordinary connotation of the term.

Rodriguez would have the Court respect the conclusions of the Oversight Committee which was
convened by the president to formulate the rules and regulations to implement the LGC pursuant
to S533 thereof. They concluded that “fugitive from justice” means one who has been convicted
by final judgment.

The Oversight Committee entertained serious apprehensions on the possible constitutional


infirmity of S40(e) of RA 7160 if the disqualification therein meant those who were merely facing
criminal charges. The Committee finally came out with Article 73 of the LGC IRR, defining
“fugitive from justice” as a person “who has been convicted by final judgment.”

The Court agrees with Rodriguez that the construction placed upon by a law by the officials in
charge of its enforcement deserves great weight. However, when there is no obscurity and
ambiguity in an enabling law, it must merely be made to apply as it is so written. An
administrative rule cannot expand or constrict the law but must be congruent to it. Thus, the Court
believes and holds that A73 of the IRR of the LGC, defining “fugitive from justice”, is an
inordinate and undue circumscription of the law.

COMELEC did not make any definite finding on whether Rodriguez was in fact a “fugitive from
justice” as such term is applied in the light of the Court’s opinion. The COMELEC dismissed
outrightly the quo warranto based on A73 of the IRR of the Oversight Committee. The Court,
being not a trier of facts, is thus constrained to remand the case to the COMELEC for a
determination of this unresolved factual matter. (Despite the reservations of “the ponente” *on the
constitutionality of S40(e) based on presumption of innocence)

Davide Jr., Separate opinion


I do not share the doubt of Justice Vitug on the constitutionality of the disqualification based on
the presumption of innocence clause.S1, Art. V of the constitution recognizes the authority of
Congress to determine who are disqualified from exercising the right of suffrage. Since the
minimum requirement of a candidate for public office is that he must be a qualified voter, it
logically follows that Congress has the plenary power to determine who are disqualified to seek
election for a public office.

A public office is a public trust. Stricter qualifications for public office may be required by law.

The disqualification in question does not involve presumption of innocence. One is not
disqualified because he is presumed guilty by the filing of an information against him. He is
disqualified because he is a “fugitive from justice”- he was not brought within the jurisdiction of
the court because he had successfully evaded arrest, or if brought within the jurisdiction of the
court, tried and convicted, he has evaded service of sentence because he had jumped bail or
escaped. The disqualification is based on his flight from justice. In the face of the doctrine that
flight is an indication of guilt, it may even be said that it is not the disqualifying provision which
overcomes the presumption of innocence but rather the disqualified person himself who has
proven his guilt.

RIGHT to be HEARD by HIMSELF and


COUNSEL
181. People v. Frisco Holgado, GR L-2809, March 22, 1950, Moran, C.J. (Right to be heard
by himself and counsel)
FACTS:
Frisco Holgado was charged in the CFI with slight illegal detention because, being a private
person, he (acc to the information filed) kidnapped and detained one Artemia in the house of
Antero for 8 hours, depriving Artemia of her personal liberty. On the day of the trial, the judge
asked Holgado:
"Court: — to the accused. "Q. Do you have an attorney or are you going to plead guilty? — A. I
have no lawyer and I will plead guilty.”
“Court- Arraign the accused.”

Two days later, the trial court convicted Holgado. It sentenced him to kidnapping and serious
illegal detention- PMmax to RTmed. The title of the decision was “SLIGHT ILLEGAL
DETENTION”.

ISSUE:
Whether Holgado, after being asked by the trial court judge if he has an attorney or “are you going
to plead guilty?”, replied that he does not have a lawyer and he will plead guilty, and did in fact
plead guilty, may be convicted without evidence and hearing.
HELD: NO.
It must be noticed that in the caption of the case judgment, the offense charged is named “SLIGHT
ILLEGAL DETENTION” while in the body of the judgment it is said that the accused “stands
charged with the crime of kidnapping and serious illegal detention.” In the information filed, it
is said that “of the crime of slight illegal detention.” The facts alleged in the information are not
clear as to whether the offense charged is merely “slight illegal detention” as the offense is named
therein or the capital offense of kidnapping and serious illegal detention as found by the trial judge.
Since Holgado pleaded guilty and no evidence was presented by either party, the trial judge
must have deduced the capital offense from the facts pleaded in the information.

Under the circumstances, particularly the qualified plea given by Holgado who was unaided by
counsel, it was not prudent for the trial court to render such a serious judgment without absolutely
any evidence to determine and clarify the true facts of the case.

Under Rule 112, S3, when a defendant appears without attorney, the court has four important
duties to comply with: 1 — It must inform the defendant that it is his right to have attorney before
being arraigned; 2 — After giving him such information the court must ask him if he desires the
aid of an attorney; 3 — If he desires and is unable to employ attorney, the court must assign
attorney de officio to defend him; and 4 — If the accused desires to procure an attorney of his
own the court must grant him a reasonable time therefor. Not one of these duties was complied
with.

The question asked by the court only was “Do you have an attorney or are you going to plead
guilty?” The question not only failed to inform the accused that it was his right to have an
attorney before arraignment, but worse it was framed that it could have been construed by
the accused as a suggestion from the court that he plead guilty if he had no attorney. This is
a denial of fair hearing in violation of due process clause in the constitution.

The constitution guarantees that “no person shall be held to answer for a criminal offense without
due process of law,” and that all accused “shall enjoy the right to be heard by himself and counsel.”
In criminal cases, there can be no fair hearing unless the accused is given opportunity to be
heard by counsel. The right to be heard would be of little avail if it does not include the right to
be heard by counsel. Even the most intelligent or educated man may have no skills in the science
of law, particularly in the rules of procedure, and, without counsel, he may be convicted not
because he is guilty but because he does not know how to establish his innocence.

182. People v. Mario Serzo, Jr., GR 118435, June 20, 1997, Panganiban, J. (Right to be Heard
by himself and counsel)
FACTS:
Alfredo Alcantara, with his wife Adelaida, was staying inside their house watching TV. Around
11:30pm, Susana Serzo, mother of accused, and one Epifania came knocking at their door pleading
for help to bring out her grandchildren who were being held inside their house by her son, accused.
The couple went to the house across their house and were able to rescue the grandchildren. When
returning to their house, Alfredo was walking an armslength ahead of his wife when he was
attacked by Mario Serzo from behind.

Serzo claims denial of right to counsel. Arraignment was set by the RTC on January 8, 1991 where
Serzo appeared without counsel. The court appointed Atty. Lina-ac as counsel de officio for the
arraignment only. But Serzo moved to reset the arraignment and he be given time to engage a
counsel of his own choice, which was granted.

Feb.11- Serzo appeared without a counsel de parte. He was still arraigned with assistance of Atty.
Lina-ac as counsel de officio.

The hearings scheduled on April 22 and May 6 were cancelled on motion of prosecutor Tobia.
Serzo appeared with Atty. Lina-ac on both dates. On May 13 and June 3, trial proceeded with the
testimonies of prosecution witnesses. Lina-ac cross-examined the witnesses. On June 17, trial was
again cancelled as Serzo appeared without counsel. On August 13, the prosecution rested.

On November 4 and 11, the presentation of evidence for the defense was reset as appellant was
not ready to testify and he manifested intention to secure counsel de parte. On March 3, 1992,
Atty. Lina-ac was relieved as counsel de officio in view of Serzo’s refusal to cooperate with said
counsel. On April 6, Serzo appeared without counsel, forcing the trial court to appoint another
counsel de officio, Atty. Antonano. Counsels for both parties agreed to reset the trial, but Serzo
refused to sign the minutes of the proceedings.

On April 27, over the objection of the prosecution, hearing was reset for the last time as Serzo was
still looking for counsel de parte. On August 25, Serzo appeared without counsel. The trial court
appointed Atty. Garcia of PAO as counsel de officio. Trial was again postponed. On September
1 and October 19, trial was postponed on motion of Atty. Garcia. Serzo again refused to sign the
minutes of the proceedings for both dates. On November 5, Serzo refused to cooperate with Atty.
Garcia by declining to take the witness stand, forcing the defense to rest its case. Both parties
were ordered to submit their memoranda after which the case would be submitted for decision.
Atty. Garcia was also ordered to manifest within 10 days if Serzo would change his mind and
cooperate with her. No such memorandum or manifestation was filed.

The RTC convicted Serzo of murder and noted that he refused to secure the services of counsel de
parte and to present evidence in his defense despite ample opportunity accorded to him. Serzo,
thru counsel Arcilla, appealed to the SC.

ISSUE:
Whether the accused Serzo had his right to counsel violated when he was convicted after he refused
to cooperate with 3 counsels de officio appointed for him after he moved to be given time to find
counsel de parte but appeared in court without counsel resulting in the postponement several times
of the hearings.
HELD: NO.
Serzo mainly alleges that he was denied effective legal representation.

The right of an accused to counsel is guaranteed to minimize the imbalance in the adversarial
system where the accused is pitted against the awesome prosecutor machinery of the state. Without
counsel, though he be not guilty, the accused faces the danger of conviction because he does not
know how to establish his innocence. The right covers the period beginning from custodial
investigation into the rendition of judgment and even on appeal. (S12(1), S14)

With these precepts, the RoC grants an accused the right to counsel under Rule112, S7
(Preliminary Investigation), Rule 113, S14(Arrest), Rule 115, S1 (Rights of Accused, S1- at the
trial).Rule 116 makes it compulsory that the trial court inform the accused of his right to counsel
prior to arraignment. Even on appeal, the accused is afforded right to counsel (Rule 122, S13).
This is buttressed by Rule 124, S2. RA 7438 was enacted providing that any person arrested,
detained, or under custodial investigation shall at all times be assisted by counsel.

But right to counsel de parte is not absolute. An accused may exercise his right to counsel by
electing to be represented by a court-appointed lawyer or by one of his own choice. While his
right to be represented by counsel is immutable, his option to secure services of counsel de parte
is not absolute. The court is obliged to balance the privilege to retain a counsel of choice against
the state’s and offended party’s equally important right to speedy and adequate justice. Thus, the
court may restrict the accused's option to retain a counsel de parte if the accused insists on an
attorney he cannot afford, or the chosen counsel is not a member of the bar, or the attorney declines
to represent the accused for a valid reason, e. g. conflict of interest and the like.

The right to counsel de parte is also, like other personal rights, waivable so long as 1) the waiver
is not contrary to law etc. or prejudicial to a third person with a right recognized by law, and 2) the
waiver is unequivocally, knowingly, and intelligently made. in Sayson v. People, the Court held
that the duty of the court to appoint a counsel de officio is not mandatory where the accused
proceeded with arraignment and the trial with a counsel of his choice but, when the time for
presentation of the evidence for the defense was due, he appears by himself alone because of the
inexcusable absence of his counsel.

Here, he claims that he was not given sufficient time to engage counsel de parte and that his
counsels de officio did not exert their “utmost efforts” in representing him. The SOLGEN rebuts,
saying Serzo showed a “lackadaisical stance on his own defense.” Serzo was given ample time to
secure counsel de parte, but his subsequent appearances in court without such counsel and his act
of allowing this situation to continue until presentation of his evidence betrays his lack of
intention to do so. It even appears that he was merely delaying his own presentation of evidence
on purpose to the prejudice of the offended party, the trial court, and the orderly administration of
justice.

He did not demonstrate in what way the services of counsels de officio were unsatisfactory. He
was afforded a chance to be heard by counsel of his own choice, but by his own neglect or mischief,
he effectively waived such right. He was not deprived of his right to counsel as he was adequately
represented by 3 court-appointed lawyers. Courts are not required to await indefinitely the
pleasure and convenience of the accused as they are also mandated to promote the speedy and
orderly administration of justice. PUBLIC POLICY requires that the trial continue as
scheduled, considering that Serzo was adequately represented by counsels who were not shown to
be negligent or unable to represent him.

183. People v. Eduardo Agbayani, GR 122770, January 16, 1998, Per Curiam (Right to be
Heard by Himself and Counsel)
FACTS:
The PNP endorsed to the prosecution office of QC the complaint of Eden Agbayani for rape against
her father Eduardo Agbayani. After preliminary investigation, a complaint for rape signed by Eden
was filed against Eduardo with the RTC. At arraignment, Eduardo, assisted by Attys. Baldado and
de la Cruz as counsel de officio, pleaded not guilty. The first prosecution witness, Dr. Baltazar,
was cross examined by Atty. Balbado. On the succeeding dates of trial, the prosecution presented
Eden and SPO1 Buenviaje. During these hearings, however, Eduardo was represented by Atty.
Temanil of PAO.
Eduardo put up denial and alibi. Adoracion corroborated the alibi. The RTC convicted Eduardo of
rape qualified by under 18 and the offender is a parent, giving credence to the testimony of Eden
who appeared candid, coherent, and responsive. The alibi of Eduardo is self-serving. The penalty
imposed was death. The case was forwarded to the SC for automatic review.
Eduardo, thru his new counsel de parte Attys. Siobal and Floresta, filed a motion for new trial on
the ground that serious irregularities prejudicial to his substantial rights were committed during
the trial. The trial court denied the motion.

Hence this appeal. He claims that the lower court failed to apprise him of his right to have counsel
of his own choice.

ISSUE:
Whether Eduardo Agbayani’s, who claims that he was not apprised of his right to counsel by the
trial court for his charge of raping his daughter Eden, where two counsels de officio appear on
record to have been appointed for him and cross-examined the prosecution’s first witness, but
where the transcript does not show that Agbayani was indeed informed of such rights, right to
counsel was violated.
HELD: NO.
It is true that the transcript only state that the court appointed de officio counsel with the consent
of the accused. They do not categorically disclose that the RTC informed Eduardo of his right to
counsel of his own choice. But this does not mean that the RTC failed to inform him of such right.
The trial court’s order states that the de officio counsel were “duly appointed by the court with the
consent of the accused.” Since Eduardo miserably failed to show that he was not informed of his
right to counsel, the presumptions that the law has been obeyed and official duty has been
regularly performed by the RTC stand. The RTC is presumed to have complied with its four-
fold duties under S6, Rule 116: , (1) to inform the accused that he has the right to have his own
counsel before being arraigned; (2) after giving such information, to ask accused whether he
desires the aid of counsel; (3) if he so desires to procure the services of counsel, the court must
grant him reasonable time to do so; and (4) if he so desires to have counsel but is unable to employ
one, the court must assign counsel de oficio to defend him.

The failure of the record to disclose affirmatively that the trial judge advised the accused of his
right to counsel is not sufficient ground to reverse conviction since the trial court must be
presumed to have complied with the procedure prescribed by law for the hearing and trial
of cases, and such presumption can only be overcome by an affirmative showing to the contrary.

Here, the RTC appointed two de officio counsel who assisted Eduardo at his arraignment, one of
whom extensively cross-examined the first witness for the prosecution, Dr. Baltazar. Besides, it is
only in this appeal that Eduardo raised this issue of failure to be informed of right to counsel. He
did not raise this in the trial court despite ample opportunity to do so. His consent to be assisted
by counsel de officio, coupled with said counsel’s extensive cross-examination , may be
considered a waiver of his right to question the alleged failure of the RTC to inform him of his
right to counsel.

Nevertheless, we take this opportunity to admonish trial courts to ensure their compliance with
pre-arraignment duties to inform accused os hif right to counsel etc. must be on record.

184. Baltazar Amion v. Judge Chiongson, AM RTJ-97-1371, January 22, 1999, Martinez, J.
(Right to be Heard by Himself and Counsel)
FACTS:
Amion charges Judge Chiongson on his appointment of a counsel de officio for Amion despite
Amion’s objection on the ground that he had his own retained counsel, Atty. Depasucat. There
was a scheduled hearing on March 28 and 29, 1996. Amion avers that his retained counsel was
ready for the hearing but on March 27, he was informed that Atty. Depasucat was ill. So Amion
was not represented by his defense counsel, which prompted Judge Chiongson to appoint Free
Legal Aid lawyer Atty. Ong. Notwithstanding Amion’s opposition, Chiongson proceeded with the
trial on March 28 with Atty. Ong representing Amion. Amion claims that Atty. Ong had no
sufficient knowledge of the case and no prior conference was held between Ong and him. Amion
thus claims that he was deprived of his right to be defended by counsel of his own choice.

Judge Chiongson explained that Amion is a police officer charged for killing a fellow policeman.
From the time he assumed office as presiding judge on November 27, 1995, other than the
arraignment on September 25 before Judge Labayen, the case has not moved. When he set the
hearing on January 9, 1996, trial was not held as Atty. Depasucat was not feeling well. The
hearing was reset to January 19 where Atty. Depasucat also failed to appear. To avoid further
delay, Chiongson tried to appoint Atty. Jacildo of PAO as de officio, but Jacildo explained that
PAO’s policy is not to represent a party with counsel of his own choice.

At the next scheduled hearing of Feb. 21, Amion’s counsel de parte still did not show up,
prompting private complainant Vaflor, victim’s wife, to speak in open court and pour out all her
frustration about the long delay of the case. Since Vaflor and another witness reside about 70 to
80 km from Bacolod City, and the appearance of Atty. Depasucat remained uncertain, Chiongson
appointed Atty. Ong from the Free Legal Aid Office to represent Amion without prejudice to
appearance of counsel de parte. Due to the continued absence of Depasucat, Atty. Ong represented
Amion at the March 28 hearing. The hearings were rescheduled on May 13 and 17. Amion filed
for inhibition of Chiongson.

Depasucat moved for postponement, alleging that the motion for inhibition should be resolved and
he would not be available on the rescheduled dates as he would be out of the country. Chiongson
denied the motion for inhibition and to set aside the proceedings of March 28. At the scheduled
hearing on Aug. 1, Depasucat asked to be allowed to withdraw as counsel de parte, causing further
delay. The trial was reset to September 2, 5, and 6. Thereafter, Amion engaged the services of
different counsels who continued to adopt the dilatory tactics utilized by previous counsel de
parte.

ISSUE:
Whether appointing counsel de officio to represent Amion when he has counsel de parte where his
counsels de parte did not appear in several hearings causing their rescheduling and where they file
dilatory motions for inhibitions etc. violates Amion’s right to counsel.
HELD: NO.
The facts point to the pervasive procrastination of the proceedings undertaken by Amion and his
counsel. Contrary to what he wants to impress upon this Court, it seems that Amion has been the
oppressor while Judge Chiongson the oppressed. Through the course of the proceedings in the
criminal case, Amion had filed several motions for inhibition, certiorari and mandamus, and this
administrative complaint with a view of delaying the disposition of the case. His claim that the
appointment of counsel de officio violates his right to due process and right to be defended by
counsel of his own choise cannot be countenanced.

The “preference in the choice of counsel” pertains more specifically to a person under
investigation rather than one accused in a criminal prosecution. Even if we extend the concept
of preference in the choice of counsel to criminal prosecution, such cannot partake of a discretion
so absolute and arbitrary as would make the choice of counsel refer exclusively to the predilection
of the accused. “Preferably” under S12(1), Art.III of the Constitution does not mean that the choice
of a lawyer of a person under investigation is exclusive as to preclude other equally competent
and independent attorneys from handling his case. If this were the rule, the tempo of custodial
investigation will be solely in the hands of accused who can obstruct the progress of the
interrogation by simply selecting a lawyer who, for one reason or another, is not available to
protect his interest. Applying this, we may likewise say that the accused’s discretion in a criminal
prosecution as to his choice of counsel is not so much as to grant him a plenary prerogative
which would preclude other equally competent and independent counsels from representing
him. Otherwise, the pace of a criminal prosecution will entirely be dictated by accused.

Amion was not denied due process as he was accorded all the opportunities to be heard and to
present evidence to substantiate his defense but forfeited this right for not appearing in court with
his counsel at the scheduled hearings.

There is no denial of right to counsel where a counsel de officio was appointed during absence of
counsel de parte pursuant to the court’s desire to finish the case as early as practicable under
the continuous trial system. It was Amion’s own strategic machinations which brought about the
need to appoint counsel de officio.

RIGHT to be INFORMED of the NATURE and


CAUSE of ACCUSATION
185. Odon Pecho v. People, GR 111399, September 27, 1996, Davide, Jr., J. (Right to be
informed of nature and cause of accusation)
FACTS:
This is the MR of the SC’s decision on November 14, 1994, holding Pecho guilty of the complex
crime of attempted estafa thru falsification of official and commercial documents. We held that
although he could not be convicted of the crime charged- violation of S3(e) of RA 3019- because
this only penalizes consummated offenses and the offense charged was not consummated, he
could, nonetheless, be convicted of attempted estafa thru falsification of official and commercial
documents, which is necessarily included in the crime charged.

Pecho argues that after being acquitted of S3(e), a special law, he could not be convicted anymore
of attempted estafa thru falsification, punishable under RPC. Otherwise, there is double jeopardy.
Both offenses arose from the same overt act. He also claims that conviction for said attempted
crime under an information charging S3(e) of RA 3019 violates his right to be informed of the
nature and cause of the accusation against him.
ISSUE:
Whether Pecho, charged with violation of S3(e) of RA 3019, may be convicted of the crime of
attempted estafa thru falsification of official and commercial documents instead, the latter crime
being necessarily included in the charge.
HELD: YES.
There is no merit in the claim of violation of the right to be informed of the nature and cause of
accusation. This originated in the BoR which the people of Great Britain demanded and received
from the prince and princess of Orange on Feb. 13, 1688. It was adopted by the US constitution
and extended to PH by Act 235 or the PH Bill of 1902. It was later carried into the Jones Law and
then enshrined in the 1935, 1973, and 1987 Constitutions. It has the following objectives:
1) To furnish accused with such a description of the charge against him as will enable him
to make his defense; 2) to avail himself of his conviction or acquittal for protection against
a further prosecution for the same cause; and 3) to inform the court of the facts alleged, so
that it may decide whether they are suf8cient in law to support a conviction, if one should
be had.

To satisfy the requirement, facts must be stated, not conclusions of law. Every crime is made up
of certain acts and intent. These must be set forth in the complaint with reasonable particularity
of time, place, names (plaintiff or defendant), and circumstances. The complaint must contain a
specific allegation of every fact and circumstance necessary to constitute the crime charged.

What determines the real nature and cause of accusation is the actual recital of facts in the
information and NOT the caption or preamble of the information or complaint nor the
specification of the provision of law alleged to have been violated, they being conclusions of
law. An incorrect caption is not a fatal mistake. Thus, an accused may be convicted of a crime
which, although not the one charged, is necessarily included in the latter.

We held in the decision that the information can also be considered as charging two offenses: S3(e)
and the complex crime of attempted estafa thru falsification. Since Pecho failed to object before
trial to such duplicity, he could be validly convicted of both or either of the offenses charged and
proved.

186. People v. Ronnie Quitlong, GR 121562, July 10, 1998, Vitug, J. (Right to be Informed
of the Nature and Cause of Accusation)
FACTS:
Around 6pm, Lito Adjaro, who had just come from work, went to a game parlor where he saw
19yo medtech student Jonathan Calpito playing billiards with Jonathan Gosil. Adjaro was
Calpito’s barkada. At 8pm, they decided to go home. Calpito and Gosil first approached a fishball
vendor and bought fishballs worth 15 pesos. When Calpito counted the change for his 100-peso
bill, he saw that he was handed back only 35 pesos. Confronted by Calpito and Gosil, the fishball
vendor would not admit that he had short-changed Calpito.

Eight men rushed towards Gosil and Calpito. Some of the men backed out but four pursued Calpito
who had started to retreat from the group. The four cornered Calpito. Adjaro saw appellant Senoto
embracing Calpito from behind and Salvador and Ronnie Quitlong holding Calpito’s right and left
hand. Calpito unsuccessfully tried to free himself. Ronnie stabbed Calpito at the left side of the
body below the nipple. Calpito fell to the ground. His assailants hit him with their feet. Police on
foot-patrol went to the commotion and brought accused to the police station. Calpito died at
hospital.

The defense admitted the presence of accused-appellants at the vicinity of the crime scene but
denied participation in the crime. The RTC convicted Ronnie, Salvador, Senoto guilty of murder.
The RTC found that there was conspiracy between them. The information failed to explicitly allege
conspiracy, but it said that “it is fortunate that in the case at bench conspiracy may readily be
inferred from the way the allegation of superior strength has been phrased xxx taking advantage
of their numerical superiority and combined strength xxx.”

ISSUE:
Whether conspiracy may be inferred from the allegation in the information that the several accused
“taking advantage of their numerical superiority and combined strength” without conspiracy being
expressly alleged.
HELD: NO.
Overwhelming evidence of conspiracy is not enough for an accused to respond to all its grave
legal consequences. The accused must be apprised when the charge is made conformably with
prevailing substantive and procedural requirements. Art.III, S14 mandates that criminal due
process and that the accused must be informed of the nature and cause of the accusation. The right
to be informed of such indictment is also explicit in procedural rules.

An information, to ensure that the right of accused to be informed of the nature and cause of his
accusation is not violated, must state the name of the accused, the designation given to the
offense by the statute, a statement of the acts or omissions complained of as constituting the
offense, the name of the offended party, the approximate time and date of commission of the
offense, and the place where the offense was committed. The information must set forth facts
and circumstances that have a bearing on the culpability of accused so that he can properly
prepare for and undertake his defense.

One such fact or circumstance against two or more accused is conspiracy. Unlike the omission of
an ordinary recital of fact which, if not objected to during trial, may be corrected by proof, an
allegation of conspiracy is indispensable to hold such person equally guilty with others in the
commission of the crime. An accused must know from the information if he faces a criminal
responsibility not only for his acts but also for the acts of his co-accused.

A conspiracy indictment need not aver all the components of conspiracy or allege all the details
thereof like the evidence proving the common design etc. It is enough that the indictment contains
a statement of the facts relied upon to be constitutive of the offense in ordinary and concise
language with as much certainty as the nature of the case will admit, in a manner that can enable
a person of common understanding to know what is intended, and with such precision that the
accused may plead his acquittal or conviction to a subsequent indictment based on the same facts.
An indictment may be held sufficient if “it follows the words of the statute and reasonably informs
the accused of the character of the offense he is charged with conspiring to commit, or, following
the language of the statute, contains a sufficient statement of an overt act to effect the object of the
conspiracy, or alleges both the conspiracy and the contemplated crime in the language of the
respective statutes defining them."

The RTC’s opinion that conspiracy may be inferred from the allegation of abuse of superior
strength is difficult to accept. Conspiracy arises when two or more persons come to an agreement
concerning the commission of a felony and decide to commit it. Verily, the information must state
that the accused confederated to commit the crime etc. In the absence of the usual words
“conspired” or “confederated”, the allegation must appear in the information in the form of
definitive acts constituting conspiracy. Conspiracy must be alleged, not merely inferred, in the
information on which basis an accused can enter his plea, a matter different from the adequacy of
evidence that may be required to prove it. In establishing conspiracy when properly alleged, the
evidence to support it need not necessarily be shown by direct proof but may be inferred from the
conduct of accused.

Without the allegation of conspiracy, an accused can be made liable only for the acts committed
by him alone and the criminal responsibility is individual and not collective. It was established that
Ronnie Quitlong was a principal by his own act of stabbing Calpito, and Salvador Quitlong and
Senoto were holding the hands of Calpito when Ronnie stabbed him. Simultaneity itself would not
demonstrate conspiracy. The incident appears to have occurred at the spur of the moment. Salvador
and Senoto are accomplices. There was abuse of superior strength. There is no treachery as there
is no proof that accused consciously adopted the mode of attack. They were convicted of murder.

187. People v. Radel Gallarde, GR 133025, February 17, 2000, Davide, Jr., C.J. (Right to be
Informed of the Nature and Cause of Accusation; rape with homicide, rape not proved)
FACTS:
Gallarde was charged with the special complex crime of rape with homicide.

At the house of spouses Talan, their neighbors converged. Among them was Radel Gallarde,
Francisco, Renato, and Edwin Fernandez, Hernandez, Cabinta, etc. Idling by was Editha, 10yo
daughter of spouses Talan. They drank beer. They dined in the kitchen. Gallarde went out of the
house and talked to Editha. Gallarde then sprinted towards his house.

After, Editha entered the kitchen and took hold of a kerosene lamp. Jaime asked her where she was
going. She said she would look for Gallarde and left enroute to where Gallarde went. Jaime,
Francisco, Edwin, and Rose regrouped at another place where they relaxed. Roger arrived and said
that Editha was missing. They searched the houses. When Jaime mentioned that Gallarde was the
last person he saw talking with Editha, the searchers went to Gallarde’s house. They saw Gallarde
in the toilet, his hands and knees covered with soil. He said he was relieving himself.

He said he did not know where Editha was and that he let her go home. He was asked where he
came from before he went to the toilet and he said he was with Kiko. One searcher got angry and
said that this was impossible as Kiko was with them drinking. Gallarde was brought to the brgy.
Captain.

The searchers found Editha buried near Gallarde’s house. The Brgy captain gave Gallarde’s
custody to policemen. The police went to where Editha was found, shoved soil aside, and found
Editha completely naked. The cause of death was from suffocation as a result of powerful covering
of the nose and mouth.

Gallarde interposed denial and alibi. The RTC convicted Gallarde of murder only, not of rape with
homicide because of lack of proof of carnal knowledge.

ISSUE:
Whether a charge of committing the special complex crime of rape with homicide where the rape
is not proved may result in a conviction of murder where the information does not state any
qualifying circumstance of murder.
HELD: NO.
The trial court erred in convicting Gallarde of murder in an information charging rape with
homicide. There was no allegation in the information of any qualifying circumstance. Although
the term “homicide” in rape with homicide is used in its generic sense, where a complex crime is
charged and the evidence fails to support the charge as to one of the component offense, the
accused can be convicted of the other. In rape with homicide, to be convicted of murder in case
the evidence fails to support the charge of rape, the qualifying circumstance must be sufficiently
alleged and proved. Otherwise, it would be a denial of the right of accused to be informed of
the nature of the accusation. Every element of the offense must be alleged in the information to
allow the accused to suitably prepare his defense. He is presumed to have no independent
knowledge of the facts that constitute the offense.

An accused cannot be convicted of an offense higher than that with which he is charged in the
information under which he is tried. It does not matter how convincing the evidence of guilt may
be, but an accused cannot be convicted of any offense unless it is charged in the information or is
necessarily included in that which is charged.

But there is enough circumstantial evidence to prove Gallarde’s guilt for the death of Editha. But
as to the rape, the evidence is lacking. There is no convincing proof that the laceration of the vagina
and rupture of the hymen of Editha were caused by coitus or a male organ. The doctor testifying
was not asked if the laceration could have been caused by a penis.

The allegation of the place of commission in the information is sufficient if it can be understood
therefrom that the offense was committed or some of the essential ingredients thereof occurred at
some place within the jurisdiction of the court. The rule merely requires that the information
shows that the crime was committed within the territorial jurisdiction of the court. The court may
even take judicial notice that said place is within its jurisdiction.

As to the time, the phrase “on or about” in the information does not require the prosecution to
prove any precise date or time, but may prove any date or time which is not so remote as to
surprise and prejudice defendant. The prosecution, contrary to Gallarde’s claim, established the
proximate time of commission of the crime between 9pm when Gallarde left the house of Talan
followed by Editha and 10:30pm, when Editha’s body was found.

Gallarde was convicted of homicide only.


188. People v. Renato Dadulla, GR 172321, February 09, 2011, Bersamin, J. (Right to be
Informed of the Nature and Cause of Accusation; relationship, qualifying circ, not alleged)
FACTS:
Dadulla was charged with rape and attempted rape.

In one evening on January 15, 1998, AAA, sleeping in the bedroom that she and her five youger
siblings shared with their father, was roused from sleep by Dadulla, her father, undressing her.
AAA resisted, but Dadulla, wielding a bladed weapon, threatened to kill her if she shouted. Dadulla
then raped her. On January 22, AAA was again awoken by her father touching her body. She went
under the bed and firmly held the bed to resist the attempts of his father to pull her out. She told
him that she would not get out from the bed because what he was doing to her was bad. Upon
hearing her, he stopped, told her to leave the house, then went to sleep. Meanwhile, BBB, AAA’s
sister, was awakened. She heard Dadulla tell AAA: “Tumigil ka nan ang kaiiyak, wala ka nang
pakinabang.” AAA just cried under the bed and did not say anything.

In the morning, BBB heard Dadulla tell AAA to leave the house. When Dadulla had left, BBB
approached the crying AAA and asked what happened. AAA told BBB her ordeal. They both went
to their uncle, CCC, to report the incident. CCC requested his wife to accompany AAA to the
barangay to file a complaint. AAA and CCC’s wife went to camp Crame for genital examination,
which established that AAA had a deep healed hymenal laceration.

Dadulla denied molesting AAA. The RTC convicted Dadulla of rape and imposed the death
penalty and of attempted rape with PCmin-max as penalty. The CA held that the penalty is only
RP since the information did not allege any qualifying circumstance, so it is only simple rape. As
to the attempted rape, it held that it was only acts of lasciviousness.

ISSUE:
Whether a Dadullo may be convicted of qualified rape, qualified by relationship, for raping his
daughter when the relationship was not alleged in the information.
HELD: NO.
The CA is correct. The failure to allege the qualifying circumstance of relationship in the
information precluded a finding of qualified rape. Rule 110 of the RoC expressly requires that
qualifying and aggravating circumstances be specifically alleged in the information. Due to such
requirement being pro reo, the Court has authorized its retroactive application in favor of even
those charged with felonies committed prior Dec.1, 2000.

“Aggravating circumstance” is strictly construed when the appreciation of the modifying


circumstance can lead to imposing the maximum penalty of death. Consequently, the qualifying
circ of relationship, even if established during trial, could not affect the penalty since it was not
alleged. Relationship was neither alleged nor necessarily included. The proper penalty is thus RP.

RIGHT to SPEEDY, IMPARTIAL, and PUBLIC


TRIAL
189. Sps. Henry and Rosario Uy v. Judge Adriano, GR 159098, October 27, 2006, Callejo,
Sr., J. (Right to SPEEDY, Impartial, and Public Trial)
FACTS:
Henry Uy had been engaged in selling fake Marca Piña soy sauce. Bundoc, intelligence officer,
applied for a search warrant for unfair competition which was granted. When it was implemented,
the NBI seized 55 bottles of label Marca Pina soy sauce. Henry was charged with violation of Art.
189, unfair competition, of the RPC. Private respondent Piñakamasarap corp. moved to amend the
charge by including Henry’s spouse, Rosario Uy. It was granted. They were charged with selling
Pinakamasarap brand bottles but substituting the contents thereof with those the accused
manufactured and passing to the public that said products were the corporation’s.

On July 10, 1995, petitioners were arraigned and pleaded not guilty. They waived pre-trial
conference. Initial trial was set on November 27, 1995. But it was only on February 26, 1996 that
the first witness of the prosecution testified. Meanwhile, the SC issued AO 104-96 providing that
the RTC shall have exclusive jurisdiction over violations of Art. 188-189 of the RPC. But the
MTC continued with the trial. On August 25, 1997, Tomboc, analyst of BFAD, testified.
Meanwhile, Arts. 188-189 were amended by RA 8293. On August 30, 1999, or two years later,
Lomboy, supervisor of Pinakamasarap corp, testified. On October 22, 1999, counsel for petitioners
moved to withdraw. New counsel for petitioners entered its appearance on November 24, 1999.
On December 12, 1999, the prosecution filed its formal offer of evidence.

On February 15, 2000, the court admitted the documentary evidence of the prosecution. The
prosecution rested. On March 20, the defense filed a demurrer to evidence, arguing that no
sufficient evidence was presented to prove beyond reasonable doubt of guilt. Also, the RTC had
jurisdiction over the crime charged.

The MTC ruled that the RTC was vested by law with jurisdiction to try charges for violation of
Art.189 as amended by RA 166. Thus, the court denied demurrer to evidence and ordered the
records of the case forwarded to the provincial prosecutor for appropriate action. The
prosecutor forwarded the case records to the RTC clerk of court. The RTC ordered the prosecutor
to conduct preliminary investigation and to file the necessary information if there is probable
cause.

The prosecutor found probable cause based on the MTC findings that there was a prima facie case
against petitioners. He filed an information in the RTC on July 18, 2000 for violation of Art.189.
petitioners moved to quash information, alleging that their rights to due process and speedy trial
had been violated. There was already a delay of 6 years from when the initial complaint was filed
and that they had already been prejudiced. The RTC denied the motion to quash. It granted the
motion of prosecution to amend the information to show that the violation is of RA 8293, not Art.
189 of the RPC.

Petitioners filed with the CA a certiorari petition, saying that the trial court gravely abused its
discretion in denying their motion to quash. The CA dismissed the petition.

ISSUE:
Whether delays in the trial in the MTC of 4 years may be a ground to dismiss the case for violation
of the right to speedy trial of the accused.
HELD: It depends.
S1(h), Rule 115 of the Revised Rules of Criminal Procedure provides that the accused is entitled
to a speedy, impartial, and public trial. S2, Rule 119 provides that trial, once commenced, shall be
continuous until terminated (In no case shall the entire trial period exceed 180 days from the first
day of trial except as authorized by the SC). The right to speedy etc. trial is also in the constitution.
“Speedy trial” is a relative term and a flexible concept. In determining whether the right of the
accused to a speedy trial was violated, the delay should be considered, in view of the entirety of
the proceedings. Mere mathematical reckoning of the time involved would not suffice as the
realities of everyday life must be regarded in judicial proceedings.

AO 113-95 also provides that violation of intellectual property rights shall be immediately
commenced and continue from day to day to be terminated as far as practicable within 60 days
from initial trial. The Court, in Martin v. Ver, adopted the “balancing test” to determine if a
defendant’s right to a speedy trial was violated. This test compels the courts to approach speedy
trial cases on an ad hoc basis, the conduct of both prosecution and defendant are weighed apropos
the four-fold factors: 1) length of delay, 2) reason for the delay, 3) defendant;s assertion or
non-assertion of his right, and 4) prejudice to defendant resulting from the delay. None of these
elements, however, is either a necessary or sufficient condition. They are related and must be
considered together with other relevant circumstances.

1. Length of delay is to some extent a triggering mechanism. Until there is some delay, which is
presumptively prejudicial, there is no necessity to inquire into the other three factors.
2. Reason for the delay.
The accused has the burden to prove the factual basis of the motion to quash the information on
the ground of denial of their right to a speedy trial. They must demonstrate that the delay is
vexatious, capricious, and oppressive, or is caused by unjustified postponements asked for and
secured, or that without cause or justifiable motive, a long period of time is allowed to elapse
without the case being tried. The prosecution is required to establish that the delay was
reasonably attributed to the ordinary processes of justice, and that petitioners suffered no
serious prejudice.

Here, there had been a considerable delay in the trial in the MTC. Upon motion/agreement of
petitioners and prosecution, or because of joint absences, the trial was delayed for more than 11
months. In its own instance, the MTC reset some trial dates to correct mistakes in scheduling or
because the witnesses were not duly notified, delaying trial by 7 months. Peittioners contributed
to the delay of more than 5 months- they or their former counsel were either absent or moved for
postponements to attend another pending case or due to health concerns. 21 months delay can be
attributed to the prosecution.

Not only petitioners but the state also was prejudiced by the inordinate delay. The prosecution took
more than 4 years to rest its case after presenting only 3 witnesses. But the mistake of the
prosecutor and failure of MTC to dismiss the case motu proprio (despite AO104-96 declaring that
the RTC has exclusive jurisdiction) should not prejudice the interest of the state to prosecute
criminal offenses and defeat the right of the offended party to redress for grievance. Petitioners do
not attribute to the prosecution or MTC any malice nor prove the same by clear and convincing
evidence. Hence, without showing of BF or gross negligence, delay caused by the lapse of the
prosecution is not in itself violative of the right to speedy trial.
As significant as the right to speedy trial is the right of the state to prosecute people who violate
its penal laws. This right is violated only when the proceeding is attended by vexatious, capricious,
and oppressive delays. The right cannot be invoked where to sustain it would deny due process to
the prosecution. The offended party is also entitled to due process in criminal cases.

3. Petitioners’ assertion of the right.


The assertion of the right is entitled to strong evidentiary weight in determining whether defendant
is deprived thereof. Here, except in only one instance, the records do not show that petitioners
have bothered to raise their objection to the several re-setting of the trial dates. They actively
participated in the trial when the prosecution presented its evidence and cross examined the
witnesses. Until the motion to quash with the RTC, they never contested the prosecutorial
proceedings nor timely challenged the pendency of the case in the MTC.

Lack of jurisdiction must be seasonably raised at the earliest possible opportunity. Otherwise,
active participation in the trial would estop a party from later challenging it. Likewise, one’s failure
to timely question the delay of the trial would be an implied acceptance of such delay and a
waiver of the right to question the same. The right may be waived when not positively
ASSERTED. A party’s silence may amount to laches. RA 8493, Speedy Trial Act of 1998, is am
means to enforce S14(2) of Art.III. Its spirit is that the accused must go on record in the attitude
of demanding a trial or RESISTING DELAY. Otherwise, he waives the privilege.

4. Prejudice to petitioners. (Barker v. Wingo)


Prejudice should be assessed in light of the interests of a defendant which the speedy trial right
was designed to protect, namely: (1) to prevent oppressive pretrial incarceration; (2) to
minimize anxiety and concern of the accused; and (3) to limit the possibility that the defense
will be impaired. The most serious is (3), because the inability of a defendant to adequately
prepare his case skews the fairness of the entire system. There is prejudice if witnesses die or
disappear during a delay or if defense witnesses are unable to recall accurately events of the distant
past.

The records do not show that the delay caused them any prejudice tantamount to a deprivation of
their right to speedy trial. Petitioners were not subjected to pretrial incarceration. There is no
showing that they suffered undue pressures (anxiety and concern). Mere reference to a general
asseveration that life, liberty, property, and reputation were prejudiced is not enough. There must
be conclusive factual basis. The standard here is minimization, not necessarily elimination of
the natural consequences of an indictment. There is also no basis for the claim that evidence or
witnesses may become unavailable due to the delays. The claim of impairment of defense due to
delay must be SPECIFIC and not mere conjecture.

Since none of the types of prejudice mentioned in Barker v. Wingo or any others is shown, the
petition is dismissed. The right to a speedy trial is not primarily intended to prevent prejudice to
the defense caused by the passage of time. That interest is protected primarily by the due process
clause and statute of limitations.
190. Imelda Marcos v. Sandiganbayan, GR 126995, October 6, 1998, Purisima, J. (Right to
SPEEDY, Impartial, and Public Trial)
FACTS:
Imelda Marcos and Jose Dans were charged for violation of S3(g) of RA 3019. “Being then
Chairman and Vice-Chairman of the LRT Authority (LRTA)” conspired with one another and
entered on behalf of the government corporation LRTA a lease agreement covering LRTA
property in Pasay with the PGH Foundation, Inc. (PGHFI), a private enterprise, under terms
grossly disadvantageous to the government.

The case was raffled to the Sandiganbayan first division which, on September 15, 1993, failed to
comply with the legal requirement of unanimity of its 3 members due to the dissent of Justice
Atienza. A special division of 5 was made with Justices Amores and del Rosario as additional
members. Justice Amores wrote presiding Justice Garchitorena on September 21 requesting 15
days to send his manifestation. But on the same day, when Justice Balajadia and Garchitorena
agreed with the opinion of del Rosario, Garchitorena dissolved the special division of five without
waiting for Justice Amores’ manifestation, saying that the request was pointless because of the
agreement of Balajadia and him to the conclusion of Justice Atienza. Thus, the Sandiganbayan on
September 24 handed down the now assailed decision of the first sandiganbayan division.

ISSUE:
Whether the judgment of the special division of 5 sandiganbayan justices convicting Imelda of
violation of S3(g) of RA 3019 based on the discussion of only 3 justices on a QC restaurant to
convict Imelda with a non-sandiganbayan justice and without waiting for the manifestation of one
of the justices is valid.
HELD: NO.
The charge of S3(g) of RA 3019 has the following elements: 1) the accused acted as a public
officer, 2) the contract or transaction entered into by the latter is manifestly and grossly
disadvantageous to the government. Imelda, in her capacity as chairman of PGHFI and Dans as
vice chairman of LRTA, signed the lease agreement on June 8, 1984 by virtue of which LRTA
leased to PGHFI a lot of 7,340 m2 at a monthly rental of P102,760 for 25 years. PGHFI, represented
by Imelda, and Transnational Construction Corporation signed the sublease agreement where
lessee rented the same area for P734000 a month for 25 years.

Imelda and Dans were thus indicted for executing the lease agreement, conspiring with each other.
The Sandiganbayan convicted Imelda and Dans. On June 29, 1998, the third division of this court
affirmed the judgment against Imelda but reversing the conviction against Dans. Hence this MR.

Imelda signed the lease in her capacity as chairman of PGHFI, not as chairman of LRTA. It was
Dans who signed the contract as vice chairman of LRTA. Thus, Imelda did not sign the lease
agreement as a public officer within the contemplation of RA 3019 and, thus, the first element
is wanting.

As to the second element, the sandiganbayan compared the Lease agreement (Exhibit B) of rental
P102,760 a month and the sublease agreement with P734,000. It concluded that the P102,760 is
disadvantageous to the government. But Exhibit B by itself does not prove any offense or that the
contract was grossly disadvantageous to the government. At most, it creates only a doubt in the
mind of the objective readers as to which between the lease and sublease rental rates is the fair
and reasonable one. It could happen that in both contracts, neither LRTA nor the government
suffered any injury. There is thus insufficient evidence to prove Imelda’s guilt beyond reasonable
doubt.

The only basis of sandiganbayan was the comparison of the rental with the very much higher rental
price of the sublease. The comparison is purely speculative and violative of due process. The
disparity in rental does not necessarily mean that the rental of P102,760 is unreasonable and grossly
disadvantageous to the government. There are many factors to consider in determining what is
reasonable rental. Dans stressed that the lease rental rate was based on a study of generally
accepted rules of rental computation. Mr. Cuervo, real estate appraiser, testified as expert witness
and whose impartially was never impugned, assured the court that the rental was adequate and fair.

Other than the lease and sublease agreement, the prosecution offered no other evidence to prove
the accusation.

There is also violation of her right to substantive and procedural due process by sandiganbayan.
The first division of sandiganbayan, composed of Garchitorena, Balajadia, and Atienza could not
agree on whether to convict or acquit Imelda. As there was no unanimity of votes, Garchitorena
made a special division of 5 justices. Justice Amores requested that he be given 15 days to submit
his manifestation, but Garchitorena, Balajadia, and del Rosario, after attending a hearing in HoR,
lunched together where they discussed Imelda’s cases in the absence of Justices Atienza and
Amores and in the presence of a non-member of the special division. There, they agreed with
Atienza to acquit Imelda and to convict her in the other cases. When the justices returned to the
sandiganbayan, Garchitorena dissolved the special division. The procedural flaws are fatal to the
validity of its decision convicting Imelda.
1) Rule VI, S4 provides that sandiganbayan session shall be held in its principal office where it
shall try and determine all cases. 2) The sandiganbayan rules do not allow unscheduled
discussion of cases. 3) The rules also do not allow informal discussion of cases. There is no way
to know how the discussion was conducted in the QC resto as it was not minuted. 4) The rules do
not allow the presence of a non-member in the deliberation of cases. 5) The rules do not allow
the exclusion of a member of a division in the deliberation of cases.

These irregularities violated the right of Imelda to be tried by a collegial court. Under PD 1606
and pursuant to the sandiganbayan rules, Imelda cannot be convicted except upon the vote of 3
justices or special division of 5 justices. But more important is the process by which they arrive at
their vote. It is indispensable that their vote be preceded by discussion and deliberation by all
members of the division.

Here, Garchitorena had already created the special division of 5 justices in view of the lack of
unanimity of the 3 justices. At that stage, Imelda had a vested right to be heard by the 5 justices.
Imelda was denied the opinion of Justice Amores for before it could be given, Garchitorena
dissolved the special division. We reject the rationalization that the opinion of Justice Amores was
not important as it cannot overturn the votes of the three justices convicting Imelda. This is mere
guesswork. The more reasonable supposition is that said opinion could have changed the
opinions of the other justices if based on an unbiased appreciation of facts and undistorted
interpretation of pertinent laws.

If it were an opinion for acquittal, that opinion will have an added value when Imelda appeals
her conviction to the SC. That minority opinion could sway the opinion of the SC towards
acquittal. Thus, the sandiganbayan decision is void for violating due process.

It is opined that this case should be remanded to sandiganbayan for re-decision by a special division
of 5. Generally, a void decision will not result in acquittal and does not expose accused to double
jeopardy. But here, there is a great length of time that the case has been pending with our courts.
Imelda was first indicted in January 1992. 6 years has passed but her prosecution is far from over.
To remand the case will not sit well with her right to speedy disposition in S16, Art.III. This
right expands the right to have speedy, impartial, and public trial in S14(2). It has a broadening
effect because S16 covers periods before, during, and after trial whereas S14(2) covers only the
TRIAL PERIOD. (**”disposition” vs. “trial”) We have held that an accused should be acquitted
when his right to speedy trial has been violated.

There is also Garchitorena’s undue interference in the examination of witness Cuervo, revealing
his BIAS and prejudice against Imelda. The court questions were so numerous totaling 179
compared to the prosecutor’s 73. The court also propounded leading, misleading, and baseless
hypothetical questions.

191. Hon. Garcia v. Hon. Domingo, GR L-30104, July 25, 1973, Fernando, J. (Right to
Speedy, Impartial, and PUBLIC Trial)
FACTS:
In branch 1 of the city court of Manila presided by petitioner Judge Garcia, there commenced 8
criminal actions against Respondents Calo, Carbonnel, and petitioner Lorenzana. The trial of these
cases was jointly held on 14 trial dates. This was arranged by the parties and the court upon
insistence of respondents Calo and Carbonnel who, as police officers under suspension because of
the cases, desired the same to be terminated as soon as possible and since there were many cases
scheduled for trial on the usual trial days (MWF), Saturday was agreed upon for the trial dates.
The trial of the cases in question was held, with the conformity of the accused and their counsel,
in the chambers of Judge Garcia.

It is alleged that during the 14 trial days spanning several months, the accused were represented
by counsel who acted not only in defense of their clients, but as prosecutors of the accusations
filed at their clients’ instance. There was only one day when Atty. Consengco, representing
respondent Calo and Carbonnel, was absent. But on the insistence of Carbonnel, the trial
proceeded, and said respondent cross-examined one of the adverse witnesses. Respondents did not
aver that this constituted an irregularity.

When the hearings ended, the accused, thru counsel Atty. Consengco, submitted a 14 page
memorandum. Up to this date, respondents Calo and Carbonnel had not objected to or pointed out
any supposed irregularity in the proceedings thus far.
A petition for certiorari was filed with Judge Domingo of CFI by Calo and Carbonnel alleging
jurisdictional defects. Judge Domingo issued a restraining order, causing deferment of the
promulgation of judgment. After proceedings, Domingo declared that the rights of accused had
been violated, adversely affecting their right to a free and impartial trial. It noted that the trial
was held exclusively in chambers and not in the court room open to the public. It ordered city
court judge Garcia to desist from promulgating the decisions.

Petitioners appealed to the SC thru certiorari.

ISSUE:
Whether the holding of trial in the judge’s chambers instead of the courtroom and only during
Saturdays violates the right to public trial.
HELD: NO.
The 1935 constitution in force at the time of the petition explicitly enumerated the right to a public
trial to which an accused was entitled. Justice Jose Laurel, a delegate in the committee of the BoR,
stressed “Trial should also be public in order to offset any danger of conducting it in an illegal
and unjust manner.” This right is merely a reiteration of what appeared in the PH Bill of 1902
then the PH Autonomy Act of 1916 (Jones Law). the right to public trial has its roots in the English
common law heritage. Its exact date of origin is obscure.

What does the right to public trial signify? Offhand, it appears that there is no ambiguity in the
words and it must be given a literal application. The trial must be public. Anyone interested in
observing the manner a judge conducts the proceedings in his courtroom may do so. His being a
stranger to the litigants is of no moment. The thought behind this safeguard is the belief that the
accused is thereby afforded further protection that his trial is likely to be conducted with
regularity. It is embraced in procedural due process. The recognized exception warranting
exclusion of the public is where the evidence may be characterized as “offensive to decency or
public morals.”

Did the trial in the city court judge’s air-conditioned chambers vitiate the proceedings as violative
of this right? No. There is no showing that the public was thereby excluded. It is admitted that
the size of the judge’s room would reduce the number of those who could be present. But such
a fact is not indicative of any transgression of this right. Some courtrooms are smaller than others.
It sufficies to satisfy the requirement of trial being public if the accused could “have his
FRIENDS, RELATIVES, and COUNSEL present, no matter with what offense he may be
charged. Also, 14 hearings were held in the chambers without objection by the policemen.

Judge Domingo argues that there is abdication by the fiscal of his control over the prosecution.
But if any party could complain at all, it is the people of the PH for whom a fiscal speaks and
acts.

192. Re Request for Live Radio-TV Coverage of the Trial in the Sandiganbayan of the
Plunder Cases Against Former President Joseph Estrada, AM 01-4-03-SC, June 29, 2001,
Vitug, J. (Right to Speedy, Impartial, and Public Trial)
FACTS:
SOJ Hernando Perez filed this petition. He avers that the criminal cases involve a matter of public
concern and interest, right to information on matters of public concern, andensuring transparency
in the administration of justice. The petition in effect seeks a re-examination of the Oct.23, 1991
resolution of the SC in a case for libel filed by Cory, where it was said that:
“A trial of any kind or in any court is a matter of serious importance to all concerned and should
not be treated as a means of entertainment.” “Witnesses might be frightened, play to the camera,
or become nervous. They are subject to extraordinary out-of-court influences which might affect
their testimony.” “The TV camera is a powerful weapon which intentionally or inadvertently can
destroy an accused and his case in the eyes of the public.” “Massive intrusion of media
representatives into trial can destroy the constitutionally necessary judicial atmosphere and
decorum that the requirements of impartiality imposed by due process of law are denied a
defendant xxx.” “Considering the prejudice it poses to the defendant’s right to due process and to
the fair and orderly administration of justice, and considering further that the freedom of the press
and right to info may be served by less distracting, degrading, and prejudicial means, live radio
and TV coverage of court proceedings shall not be allowed.”

ISSUE:
Whether the the trial of Erap, former president of PH, for plunder in the sandiganbayan may be
broadcasted live thru radio and television.
HELD: NO.
The propriety of granting or denying this petition involve the weighing out of the constitutional
guarantees of freedom of the press and the right to public information, on the one hand, and the
fundamental rights of the accused, on the other hand, along with the constitutional power of a court
to control its proceedings in ensuring a fair and impartial trial. Jurisprudence tells us that the right
of the accused must be preferred. With the possibility of losing not only the liberty but also the
life of an accused, it behooves all to make absolutely certain that an accused received verdict solely
on the basis of a just and dispassionate judgment, a verdict that would come only after presentation
of credible evidence testified to by unbiased witnesses unswayed by any kind of pressure. Due
process guarantees the accused a presumption of innocence until the contrary is proved in a
trial that is not lifted above its individual settings nor made an object of public’s attention and
where the conclusions reached are induced not by any outside force or influence, but only by
evidence and argument given in court.

TV can work profound changes in the behavior of the people it focuses on. It is not unlikely
for a vote of guilt or innocence to yield to it. The effect of TV may escape the ordinary means
of proof, but it is not far-fetched for it to gradually erode our basal conception of a trial such as
we know it now.

An accused has a right to public trial but is is a right that BELONGS to HIM, more than anyone
else, where his life or liberty can be held critically in balance. A public trial is not synonymous
with publicized trial. It only implies that the court doors must be open to those who wish to
come, sit in the available seats, conduct themselves with decorum and observe the trial
process. A courtroom should have enough facilities for a reasonable number of the public to
observe the proceedings, not too small as to render the openness negligible and not too large
as to distract the trial participants from their proper functions, who shall then be totally free
to report what they have observed during the proceedings.
Within the courthouse, the overriding consideration is the paramount right of the accused to due
process. While a maximum freedom must be allowed the press in carrying out the important
function of informing the public in a democratic society, its exercise must be subject to the
maintenance of absolute fairness in the judicial process.

Other possible prejudices are that the quality of testimony in criminal trials will be impaired as the
witness may be demoralized, frightened, some cocky and given to overstatement, memories may
falter, and accuracy undermined, the additional responsibilities of the presence of TV places on
the trial judge, and the impact of courtroom TV on the defendant. (Estes v. Texas)

MR to Request for live TV-Radio Coverage of Erap’s trial. September 13, 2001
FACTS:
This is a MR of the SC decision denying SOJ’s request for permission to televise and broadcast
live the trial of Erap before the Sandiganbayan. SOJ argues that there is no conflict between the
right of the people to public information and freedom of the press on one hand and on the other
the right of accused to a fair trial. Even if there was such conflict, it must be resolved in favor of
the people and press as the people is the repository of sovereignty and are entitled to information.
Live media coverage is a safeguard against attempts by any party to use the courts as instruments
for pursuit of selfish purposes.

Erap objects, claiming that the live coverage will violate the sub judice rule. It would trigger mass
demonstrations to pressure the sandiganbayan to decide one way or the other.

ISSUE:
Whether the trial of Erap, former president of PH, for plunder in the sandiganbayan may be
broadcasted live thru radio and television.
HELD: NO, only recorded and broadcasted after final judgment.
By a vote of 9-6, the Court denies the MR. In lieu of live TV and radio coverage of the trial, the
Court, by vote of 8 justices, resolved to order the audio-visual recording of the trial for
documentary purposes. 7 justices voted against such recording.

Considering the significance of the trial of Erap and the importance of preserving the records
thereof, the Court believes that there should be an audio-visual recording of the proceedings. The
recording will not be live or real time but for documentary purposes. Only after the
sandiganbayan promulgates its decision will they be available to the public. The master film shall
be deposited in the National Museum for historical preservation and exhibition.

Cameras will be inconspicuously installed in the courtroom and the movement of TV crews will
be regulated consistent with the solemnity of the proceedings. The trial shall be recorded in its
entirety, except such portions as the sandiganbayan may decide should not be held public. No
comment shall be included in the documentary except annotations which may be necessary to
explain certain scenes depicted. The recordings shall be under the supervision and control of the
sandiganbayan.
The hearings are of historic significance. They affirm our commitment to the rule that “the king
is under no man, but he is under God and the law.” Second, the Erap cases involve matters of
vital concern to our people who have a fundamental right to KNOW how their government
is conducted. Third, audio-visual presentation is essential for the edication and civic training of
the people.

There is a need to keep the records of the hearings for documentary purposes. The recordings
will be useful in preserving the essence of the proceedings. They will be primarily for the use of
appellate courts in the event a REVIEW of the proceedings or decisions of the sandiganbayan
is sought. The accuracy of the transcripts of stenographic notes can be referenced to the tapes.

By delaying the release of the tapes, concerns that those in the proceedings will be playing to the
cameras and be distracted from their roles, whether as counsel, witnesses, court personnel, or
judges, will be allayed. The possibility that judgment will be rendered by the popular tribunal
before the court of justice can render its own will be avoided. The concerns on the fairness of the
trial will be addressed.

The right of privacy is not a bar to the production of such documentary. In Ayer Productions v.
Capulong, the SC held that a limited intrusion into a person’s privacy is permissible if that person
is a public figure and the info sought from him or to be published about him are matters of a
public character. No one can prevent the making of a movie based on the trial. But at least any
movie can be checked for accuracy against such documentary and any attempt to distort the
truth can be averted.

Thus, the recording of Erap’s trial is ordered under the following conditions:
(a) the trial shall be recorded in its entirety, excepting such portions thereof as the Sandiganbayan
determine should not be held public under Rule 119, §21 of the Rules of Criminal Procedure; (b)
cameras shall be installed inconspicuously inside the courtroom and the movement of TV crews
shall be regulated consistent with the dignity and solemnity of the proceedings; (c) the audio-visual
recordings shall be made for documentary purposes only and shall be made without comment
except such annotations of scenes depicted therein as may be necessary to explain them; (d) the
live broadcast of the recordings before the Sandiganbayan shall have rendered its decision in all
the cases against the former President shall be prohibited under pain of contempt of court and other
sanctions in case of violations of the prohibition; (e) to ensure that the conditions are observed, the
audio-visual recording of the proceedings shall be made under the supervision and control of the
Sandiganbayan or its Division concerned and shall be made pursuant to rules promulgated by it;
and (f) simultaneously with the release of the audio-visual recordings for public broadcast, the
original thereof shall be deposited in the National Museum and the Records Management and
Archives Office for preservation and exhibition in accordance with law.

193. Re: Petition for Radio and TV Coverage of the Multiple Murder Cases against
Maguindanao Governor Zaldy Ampatuan, Et Al., AM 10-11-5-SC, June 14, 2011, Carpio-
Morales, J. (Right to Speedy, Impartial, and Public Trial)
FACTS:
On November 23, 2009, 57 people, including 32 journalists and media practitioners, were killed
while on their way to Shariff Aguak in Maguindanao. This spawned 57 charges of murder and
additional charge of rebellion against 197 accused. The cases are being tried in the QC RTC inside
the Camp Bagong Diwa, Taguig.

The National Union of Journalists of the PH (NUJP), ABSCBN, GMA Network, relatives of the
victims, journalists, and academe members, petitioned the SC praying that live TV and radio
coverage of the trial in these criminal cases be allowed, recording devices be permitted in the
courtroom, and guidelines formulated to govern the broadcast coverage and use of devices.

Petitioners seek the lifting of the ban on live tv and radio coverage of court proceedings, urging
the Court to revisit the 1991 ruling in Re live tv and radio coverage of the hearing of Cory’s Libel
Case and the 2001 ruling in Re request Radio-TV coverage of Erap’s plunder trial in
Sandiganbayan. They assert right to info, freedom of the press, right to a fair and public trial, right
to petition the government for redress of grievances, free access to courts, freedom of association,
subject to regulations by the SC.

ISSUE:
Whether live broadcast of the Maguindanao Massacre proceedings involving 57 victims and 197
accused may be allowed.
HELD: YES, subject to guidelines.
The SC partially grants pro hac vice the prayer for a live broadcast of the trial court proceedings,
subject to guidelines. The indication of “serious risks” posed by live media coverage to the accused
right to due process, left unexplained in Aquino and Estrada, has left a blow to the exercise of
press freedom and right to public information. The rationale for an outright total prohibition was
shrouded inside the cocoon of feared speculation, which fear, if any, may be dealt with by
safeguards under existing rules and exacting regulations.

The basic principle in Aquino is firm- a trial of any kind in any court is a matter of serious
importance to all concerned and should not be treated as a means of entertainment as to deprive
the court of the dignity and departs from the orderly quest for truth which our judicial proceedings
are formulated. The observation that massive intrusion of news media representatives into the trial
can destroy the constitutionally necessary atmosphere and decorum stands.

In Estrada (not the MR), the SC held that the propriety of granting or denying the petition involves
the weighing out of the guarantees of freedom of the press and the right to public information, on
one hand, and the right of the accused, on the other, along with the power of a court to control its
proceedings in ensuring a fair and impartial trial. The SC denied the petition. In the MR, the Court
ordered audio-visual recording for documentary purposes under certain conditions.

Other jurisdictions welcome the idea of media coverage. Almost all proceedings of UK’s SC are
filmed and sometimes broadcast. The ICC broadcasts its proceedings thru video streaming in the
internet.

As to the possible influence of media coverage on the impartiality of trial court judges, prejudicial
publicity insofar as it undermines the right to a fair trial must pass the TOTALITY OF
CIRCUMSTANCES TEST that the right of an accused to a fair trial is not incompatible to a
free press, that pervasive publicity is not per se prejudicial to the right of an accused to a fair
trial, and that there must be allegation and proof of the impaired capacity of a judge to render a
bias-free decision. Mere fear of possible undue influence is not tantamount to actual prejudice
resulting in the deprivation of the right to a fair trial.

Also, an aggrieved party has ample legal remedies. He may challenge the validity of an adverse
judgment arising from a proceeding that transgressed a constitutional right. He may move for a
change of venue, for disqualification of the judge, and for closure of portions of the trial when
necessary. The trial court may exercise its powers of contempt and issue gag orders.

One circumstance that sets the Maguindanao Massacre cases apart from the earlier cases is the
impossibility of accommodating even the parties to the cases- complainants, families of
victims, witnesses- inside the courtroom. The families of the 57 victims and 197 accused have as
much interest beyond curiosity to attend the proceedings as those impleaded parties. In Estrada, it
was said that a courtroom should have enough facilities for a reasonable number of the public
to observe the proceedings, not too small to render the openness negligible or too large as to distract
the trial participants from their proper functions. But the impossibility of holding all parties in a
courtroom in this case is unfortunate enough, what more if the right itself commands that a
reasonable number of the general public be allowed to witness the proceeding. TECHNOLOGY
tends to provide the only solution to break the inherent limitations of the courtroom to satisfy the
imperative of a transparent, open, and public trial.

In allowing pro hac vice the live broadcasting of the Maguindanao Massacre cases, the Court lays
down the ff guidelines towards the concerns mentioned in Aquino and Estrada:
1) An audio-visual recording of the Maguindanao massacre cases may be made both for
documentary purposes and for transmittal to live radio and television broadcasting.
2) Media entities must file with the trial court a letter of application, manifesting that they intend
to broadcast the audio-visual recording of the proceedings and that they have the necessary
technological equipment and technical plan to carry out the same.
3) A single fixed compact camera shall be installed inconscpicuously inside the courtroom. No
panning and zooming shall be allowed to avoid unduly highlighting or downplaying incidents in
the proceedings. The necessary equipment shall be operated only by a duly designated official of
the SC. The least wiring must be unobtrusively located in places indicated by the trial court.
4) The transmittal of the recording from inside the courtroom to media entities shall be done with
the least physical disturbance.
5) The broadcasting of the proceedings for a particular day must be continuous and in its entirety,
except where the trial court excludes, upon motion, prospective witnesses from the courtroom in
instances like unresolved identification issues or security of witnesses, etc.
6) To provide a faithful and complete broadcast, no commercial break or gap shall be allowed until
the day’s proceedings are adjourned except during recess and where the public is ordered excluded.
7) The proceedings shall be broadcast without any voice-overs except brief annotations of scenes
depicted therein as necessary to explain them. Any commentary shall observe the sub judice rule.
8) No repeat airing is allowed until after finality of judgment except brief footages and still images.
9) The original recording shall be deposited in the National Museum.
10) The recording shall be made under the supervision and control of the trial court.
11) The Court shall create a special committee which shall forthwith study, design, and
recommend appropriate arrangements etc. concerning live broadcast of the proceedings pro hac
vice in accordance with the above guidelines.

Partial MR of the Zaldy Ampatuan live coverage case.


It is alleged that the provisos 1) prohibiting voice-overs except brief annotations, 2) prohibiting
repeat airing except until final judgment, 3) requiring continuous broadcast without commercial
break except during recess or where the public is excluded, all substitute the Court’s editorial
judgment for media’s own and constitute prior restraint infringing the right to free expression.
Provisos 6,7, and 8 (*same lang) of the guidelines are also alleged to be prior restraint as they
dictate what media can and cannot report about the trial.

Ampatuan also filed a MR alleging that the June 14, 2011 resolution deprives him of due process,
presumption of innocence, and to be shielded from degrading psychological punishment. He
contends that public opinion which live media coverage can produe would affect everyone,
including the judge, witnesses, and relatives of all parties.

HELD:
The Court partially grants reconsideration. It is now disallowing live media broadcast of the
Maguindanao massacre cases but is still allowing filming for 1) real-time transmission to specified
viewing areas, and 2) documentation. The Court in the June 14 resolution recognized the
impossibility of accommodating even the parties to the cases and made use of technology to
provide the “only solution to break the inherent limitations of the courtroom, to satisfy the
imperative of a transparent, open, and public trial.” But upon review, the Court has sought a
way to provide a public trial as required, a right granted to accused, without inviting media frenzy
that affect due process rights of the accused in this high-profile case.

While the Court recognizes the freedom of the press and right to public info which are rights of
non-direct parties to the case, the rights of the direct parties should not be forgotten. In such a
clash, jurisprudence is clear that the balance should be weighed in favor of the accused.

The constitutional rights specific to the accused under Section 14, Article III of the Constitution
such as the right to due process of law, to be presumed innocent until the contrary is proved, and
to an impartial and public trial and the requirement of the highest quantum of proof to justify
deprivation of his liberty (or even of his life) provide more than ample justification to take a second
look at the view that a camera that broadcasts the proceedings live on television has no place in a
criminal trial because of its prejudicial effects on the rights of accused individuals.

Aquino found that live coverage of judicial proceedings involve an inherent denial of due
process. The case here requires a greater degree of care as it is a sensational case. To be in the best
position to weigh the conflicting testimonies of witnesses, the judge must not be affected by any
outside influence. Like any human being, a judge is not immune from the pervasive effects of
media. The witnesses must also be shielded from the pressure of being aware that their testimony
is broadcasted live to be judged by the court of public opinion. This might compromise the
reliability of the fact-finding process.
Public trial is not publicized trial. The right to public trial belongs to the accused. The
requirement of a public trial is satisfied by the opportunity of the public and press to attend
the trial and to REPORT what they observed. The right to public trial of accused should not
be confused with freedom of the press and the public’s right to know as justification for
allowing the live broadcast.

To address the physical impossibility of accommodating the large number of interested parties
inside the courtroom in Camp Bagong Diwa, it is not necessary to allow the press to broadcast the
proceedings. But the Court may allow the opening of closed-circuit viewing areas OUTSIDE
THE COURTROOM where those who may be so minded can come and watch the
proceedings. This out-of-court, real-time viewing grants to a larger audience the opportunity to
monitor the proceedings as if they were inside the trial court but at the same time obviates the
massive publicity entailed in media broadcasting. Viewing areas may also be opened in selected
trial courts in Maguindanao where most of the relatives of the accused and victims reside, letting
them watch the trial without having to come to Camp Bagong Diwa. These viewing areas will, at
all times, be under the control of the trial court judges involved, subject to this Court’s supervision.

The live broadcast was disallowed. The streaming of the video coverage is subject to these
guidelines:
a. An audio-visual recording of the trial may be made both for documentary purposes and for
transmittal to specified closed-circuit viewing areas 1) outside the courtroom within Camp
Bagong Diwa, and 2) selected trial courts in Maguindanao where relatives of accused and the
victims reside.
b. The viewing areas will be installed to accommodate the public who want to observe the
proceedings and relatives. They can watch the proceedings in real time.
c. A single fixed compact camera shall be installed inconspicuously inside the courtroom. No
panning and zooming shall be allowed to avoid unduly highlighting or downplaying incidents in
the proceedings. The Camera shall be operated only by a designated employee of the SC.
d. The transmittal of the audio-visual recording from inside the courtroom to the closed-circuit
viewing areas shall be conducted in such a way that the least physical disturbance shall be ensured
in keeping with the dignity and solemnity of the proceedings.
e. The Public Information Office and OCA shall assist the trial courts involved on the physical set-
up of the camera and equipment.
f. The original recording shall be deposited in the National Museum.
g. The recording and transmittal shall be made under the control of the trial court which may issue
supplementary directives subject to this Court’s supervision.
h. In all cases, the witnesses should be excluded from watching the proceedings whether inside
the courtroom or in the viewing areas.

RIGHT to CONFRONTATION and


COMPULSORY PROCESS
194.Agustin Talino v. Sandiganbayan, GR L-75511-14, March 16, 1987, Cruz, J.
(Confrontation and Compulsory Process)
FACTS:
Talino, with others, were charged in four separate informations with estafa thru falsification of
public documents for having allegedly conspired to defraud the government of P26,523,
representing cost of repairs claimed to have been undertaken, but not actually needed and never
made, on four government vehicles, thru falsification of the supporting papers to authorize the
illegal payments. These cases were tried jointly for all accused until the prosecution had rested,
when Basilio, Macadangdang, and Talino asked for separate trials, which were allowed.

They presented their evidence in such trials, while the other accused continued defending
themselves in the original proceedings, at which one of them, Ulat, gave damaging testimony
against Talino, detailing his participation in the transactions. The sandiganbayan convicted
Talino, Basilio, Macadangdang, Ulat, and Valdez. Talino challenges this decision on the ground
that it violates his right of confrontation. The Sandiganbayan said in the decision that since the
trials were separate and the other accused did not cross examine Ulat, Ulat’s testimonies cannot
be used against the other accused.

ISSUE:
Whether the testimony of a co-accused damaging to Talino but which testimony was made in a
separate trial may be used against Talino.
HELD: NO.
The grant of separate trial rests in the sound discretion of the court and is not a matter of right of
accused. The right of confrontation is one of the rights guaranteed by the Constitution to persons
facing criminal prosecution who should know who his accusers are and must be given a chance
to cross-examine them on their charges. No accusation is permitted to be made against his back
or in his absence nor is any derogatory information accepted if it is made anonymously. This is the
reason why ex parte affidavits are not permitted unless the affiant is presented in court, and
hearsay is barred except on those cases allowed in the RoC, like dying declaration.

This was intended to prevent the conviction of accused upon depositions or ex parte affidavits and
to preserve the right of accused to test the recollection of witnesses in the exercise of the right of
cross examination. Confrontation is essential because cross-examination is essential. A second
reason is that a tribunal may have before it the deportment and appearance of the witnesses
while testifying.

We studied the Sandiganbayan decision and find that it did not consider the testimony of Ulat. The
part of the decision convicting Talino did not mention Ulat but confined itself to Talino’s own acts
in approving the vouchers as proof of his complicity to swindle the government.

195. People v. Hon. Seneris, GR L-48883, August 6, 1980, Makasiar, J. (Confrontation and
Compulsory Process)
FACTS:
Prosecutor Tamin filed with the CFI an amended information for parricide charging private
respondent Pimentel as principal by inducement, Santos and Salim Doe as principals by direct
participation, and Julkanain as accomplice in the fatal stabbing of Orario, husband of Pimentel.
Pimentel was granted a separate trial.
Accused Santos pleaded guilty and was convicted. After judgment, Santos offered to testify
against Pimentel in her separate trial. He was allowed and thus testified. After the prosecution
terminated the direct examination of Santos, Pimentel moved to hold in abeyance the cross-
examination of Santos until Pimentel’s counsel shall have been furnished with the transcripts of
the direct examination of Santos. This was granted. Cross-examination of Santos by Pimentel’s
counsel was not completed for lack of material time. The counsel reserved his right to cross-
examine which was granted.

However, prosecution witness Santos was shot dead by police while allegedly escaping from
prison where he was serving sentence. The completion of the cross-examination became
impossible. Petitioner moved for a ruling on the admissibility of deceased Santos’ testimony
without a motion from the defense. Judge Seneris declared the entire testimony inadmissible as
the cross-examination was incomplete.

ISSUE:
Whether the testimony of a co-accused against Pimentel may be admitted in evidence against him
after the co-accused, Santos, was shot dead by police before the defense could complete its cross-
examination.
HELD: Only insofar as the cross-examination was completed.
The right of confrontation is part of due process not only in criminal proceedings but also in civil
proceedings and in proceedings in administrative tribunals with quasi-judicial powers. It ensures
that the witness will give his testimony under oath, deterring lying by the threat of perjury charge.
It forces the witness to submit to cross-examination, a valuable instrument in exposing falsehood
and bringing out the truth. It enables the court to observe the demeanor of witnesses and assess
his credibility.

But this right can be waived expressly or impliedly by conduct amounting to a renunciation of
right to cross-examination. There is implied waiver when the party was given opportunity to
confront and cross-examine an opposing witness but failed to take advantage of it for reasons
attributable to himself alone. The testimony given on direct examination will be received or
allowed to remain in record. But if the cross-examination is not and cannot be done or completed
due to causes attributable to the party offering the witness, the uncompleted testimony is
rendered inadmissible.

The express recognition of the right to cross-examine the witness of his adversary is invaluable as
it is inviolable in civil cases, no less than the right of the accused in criminal cases. The recognition
of the right in the constitution does not render the right of parties in civil cases less constitutionally
based for it is an indispensable part of due process.

The direct testimony of a witness who dies before conclusion of the cross-examination can be
stricken only INSOFAR as not covered by the cross-examination, and absence of a witness is
not enough to warrant striking his testimony for failure to appear for further cross-examination
where the witness has already been SUFFICIENTLY cross-examined, or the matter on which
further cross-examination is sought is not in controversy.
Ortigas Jr. v. Lufthansa is not applicable in this case as in that case, it was the unjustified failure
of Lufthansa to present its witness which had already been preceded by several postponements
initiated by Lufthansa itself, depriving the other party the opportunity to complete cross-
examination, that made the Court rule that if cross-examination cannot be done due to causes
attributable to the party offering the witness, the uncompleted testimony is inadmissible. The ruling
there applies only if the cause for non-completion of cross-examination was attributable to the very
party offering the said witness. Here, the cause for non-completion of cross-examination was a
fortuitous event as he was killed by law enforcers after his prison escape.

The prosecution did not order the shooting of Santos. It is probably its cause that will suffer from
said death. While the death may be attributable to the negligence of petitioner’s agents, the state
is not bound by the negligence or tortious acts of its agents. The cause of non-completion is neither
the fault of petitioner nor private respondent Pimentel. Thus, the admissibility or inadmissibility
of the testimony of Santos cannot be resolved based on the Lufthansa case.

But the cross-examination was completed insofar as the elements of parricide, fact of killing, is
concerned. What remained was merely the cross-examination regarding the price or reward,
which is not an element of parricide, but only an aggravating circumstance. Because the cross
examination made by Pimentel’s counsel of deceased witness was extensive and already covered
the subject matter of his direct testimony as state witness relating to the essential elements of
parricide, and what remained for further cross-examination is the matter of price or reward, which
is merely an aggravating circumstance and does not affect the existence of the offense, Judge
Seneris gravely abused his discretion in declaring as entirely inadmissible the testimony who died
thru no fault of any of the parties before his cross-examination could be finished.

Thus, the testimony only as to the portion concerning the aggravating circumstance of price or
reward which was not covered by cross-examination was the only part excluded.

196. Domingo Roco v. Hon. Contreras, GR 158275, June 28, 2005, Garcia, J. (Confrontation
and Compulsory Process)
FACTS:
Roco was engaged in the business of buying and selling dressed chicken. Sometime in 1993, he
purchased his supply of dressed chicken from private respondent Cal’s Poultry Supply Corporation
(Cal’s). As payment, Roco drew five checks payable to Cal’s corporation against his PCIB
account. The bank dishonored the checks due to being drawn against a closed account. Cal’s filed
a complaint for BP22 against Roco. Five informations for violation of BP 22 were filed against
Roco.

Roco filed with the MTCC a request for subpoena, requiring Vivian or Danilo of Cal’s to appear
and testify in court and to bring with them certain documents, records, and books of accounts for
1993-1999. The private prosecutor manifested that it was improper to issue the requested
subpoenas. It claims that the documents are immaterial and irrelevant to the crimes for which Roco
was being prosecuted. The MTCC denied the request since the documents were immaterial and
the subpoenas will only delay the hearing of the case. The RTC dismissed the certiorari of this
denial. The CA also denied the certiorari.
ISSUE:
Whether Roco may be granted his request for the subpoena of certain records and books of
accounts of Cal’s who charged him with violations of BP 22.
HELD: NO.
Subpoena is a process directed to a person requiring him to attend and to testify at the hearing or
trial of an action or investigation conducted under the laws of PH or for the taking of his deposition.
There are 2 kinds of subpoena in this jurisdiction: 1) subpoena ad testificandum and 2) subpoena
duces tecum. The first is used to compel a person to testify, while the second is used to compel
the production of books, records, things, or documents therein specified. The subpoena duces
tecum may issue only if the court is satisfied that the following requisites are present: : (1) the
books, documents or other things requested must appear prima facie relevant to the issue subject
of the controversy (test of relevancy); and (2) such books must be reasonably described by the
parties to be readily identified (test of definiteness).

Petitioner has to prove to the satisfaction of the court the RELEVANCY and the
DEFINITENESS of the documents he seeks to be brought before it. The documents Roco
requested to be subpoenaed are designated and described in the request with definiteness and
readily identifiable. The test of definiteness is satisfied. But as to relevancy, Roco fails to
discharge his burden.

The gravamen of the offense in BP 22 is the act of issuing a worthless check. It is consummated
from the very moment a person issues the said check. Roco wants to show that the documents he
requested are indispensable or, at least, relevant to prove his innocence. The Court disagrees. Roco
had been issued temporary receipts in the form of yellow pad slips of paper evidencing his
payments, which slips had been validated by Cal’s. Thus, the production of the books and
documents requested are not indispensable to prove his defense of payment. Thus, the subpoena
would only delay the criminal proceedings.

TRIAL in ABSENTIA
197. Mary Estrada v. People, GR 162371, August 25, 2005, Austria-Martinez, J. (Trial in
Absentia)
FACTS:
A criminal case was filed against Estrada. An information charging her with estafa was filed with
the RTC. Since Estrada jumped bail, the RTC declared her to have waived her right to present
evidence. The RTC rendered judgment based only on the prosecution evidence.

Junimar applied for employment in japan with Estrada. Estrada collected money from Junimar and
his wife of P68,700. After the payment, Estrada told Junimar to go to the JP Embassy to claim the
plane tickets. But when they went to the embassy, nothing was filed. Estrada went with them the
second time and that was the only time when she filed the necessary documents. The embassy
required Junimar to submit documents but Estrada failed to produce said documents. So Junimar
abandoned his plan of going to Japan and just get the money from Estrada who failed to return the
money despite demand. The RTC convicted Estrada.

Estrada assailed the RTC decision for violating her rights. The proceedings were as follows:
An information for estafa was filed against Estrada. She signed an undertaking that in case of her
failure to appear during trial despite due notice, her absence would constitute an express
waiver of her right to be present during trial and promulgation of judgment, and the RTC would
proceed with the hearing in absentia. During hearing, Atty. Quicho, Estrada’s counsel, failed to
appear. Since Estrada jumped bail, the RTC considered her to have waived presentation of her
evidence and declared the case submitted for decision. After, she filed a notice of appearance thru
counsel and argued that, despite trial in absentia, Estrada could still present evidence. But the RTC
rendered its decision. 2 years after the conviction judgment, she was arrested and detained.

Estrada’s appeal was denied due course for being filed beyond the reglementary period. She filed
a certiorari with the CA. The CA denied the petition. Hence this petition for certiorari.

ISSUE:
Whether an accused who jumped bail may complain that her right to due process was violated
when the trial court held the trial and promulgated judgment in absentia.
HELD: NO.
Estrada claims that the RTC denied her right to be heard and to be assisted by counsel by failing
to furnish her counsel copies of the order setting the date for reception of defense evidence, and
the order considering her to have waived her right to present evidence.

Estrada jumped bail. The address she furnished the trial court from the beginning was incorrect.
From such fact alone, Estrada’s arguments regarding the validity of the proceedings and
promulgation of judgment in absentia for violating her due process rights are doomed to fail. Trial
in absentia is authorized in S14(2), Art.III of the 1987 Constitution.

In People v. Tabag, we held that the accused’s escape from preventive detention therein should
have been considered a waiver of their right to be present at their trial, and the inability of the
court to notify them of the subsequent hearings did not prevent it from continuing with their
trial. They were deemed to have received notice. Their escape made their failure to appear
unjustified as, by escaping, they placed themselves beyond the pale and protection of the law. The
trial court should not wait for the fugitives’ re-arrest. They were deemed to have waived their right
to present evidence on their own behalf and to confront and cross-examine the witnesses who
testified against them.

Thus, Estrada’s protestations that she was denied due process as she and her counsel did not receive
notices of the trial court’s orders are all to naught by the mere fact that she jumped bail and could
no longer be found. She is considered to have waived her right to be present at trial, and she and
her counsel were to be deemed to have received notice. Nevertheless, the CA found that Estrada
and her counsel were actually served with the copies of the RTC decision at the addresses they
submitted to the RTC. Thus, the argument that the RTC decision was void for violating due process
must fail.

The RTC decision in absentia is also valid. Rule 120, S6 is applicable:


Section 6. Promulgation of judgment — The judgment is promulgated by reading the same
in the presence of the accused and any judge of the court in which it was rendered.xxx
The proper clerk of court shall give notice to the accused personally or through his
bondsman or warden and counsel, requiring him to be present at the promulgation of the
decision. In case the accused fails to appear thereat the promulgation shall consist in the
recording of the judgment in the criminal docket and a copy thereof shall be served upon
the accused or counsel. If the judgment is for conviction and the accused's failure to appear
was without justifiable cause, the court shall further order the arrest of the accused, who
may appeal within fifteen (15) days from notice of the decision to him or his counsel.

Thus, promulgation of judgment in absentia is ALLOWED under the rules. Such promulgation
is valid if 1) the judgement be recorded in the criminal docket, and 2) a copy thereof be served
upon the accused or counsel. It is presumed that official duties are regularly performed and that
the proceedings are made of record. This serves as substantial compliance with the procedural
requirement of the recording of the judgment in the criminal c=docket of the court.

Here, since the records show that copies of the decision were sent by registered mail to the given
addresses of Estrada and counsel, and the judgment was recorded in the criminal docket of the
court, the promulgation was valid. The purpose of recording in the criminal docket is to GIVE
NOTICE thereof to all persons. Thus, Estrada is deemed notified of the decision upon its
recording in the criminal docket and she had only 15 days therefrom to file an appeal. The notice
of appeal was filed out of time.

198. Fiscal Gimenez v. Hon. Nazareno, GR L-37933, April 15, 1988, Gancayco, J. (Trial in
Absentia)
FACTS:
Samson, Alex, Rogelio, Fernando, Rogelio Baguio, and private respondent Teodoro de la Vega
were charged with murder. All were arraigned and pleaded not guilty. Hearing was set. All accused
were informed. But before the date of hearing, Teodoro escaped from his detention center and
failed to appear in court at the hearing. The fiscals moved to try all accused and Teodoro be tried
in absentia invoking S19, Art.IV of the 1973 Constitution. (However, after arraignment, trial may
proceed notwithstanding the absence of the accused provided that he has been duly notified and
his failure to appear is unjustified.)

The lower court proceeded with trial. The lower court dismissed the case against five accused
while holding in abeyance the proceedings against Teodoro.

ISSUE:
Whether judgment of conviction or acquittal must await the appearance in court of an escapee to
let him cross-examine witnesses against him and to present his evidence.
HELD: NO.
The lower court acquired jurisdiction over Teodoro when he appeared during arraignment and
pleaded not guilty. Jurisdiction is acquired either by arrest or voluntary appearance in court.
Teodoro voluntarily appeared for arraignment. But was jurisdiction lost when he escaped from the
custody of the law and failed to appear during trial? No. Jurisdiction once acquired is not lost
upon the instance of the parties but continues until the case is terminated. Jurisdiction of
Teodoro, once he pleaded not guilty, is acquired and continues until the case is terminated
notwithstanding his escape from the custody of the law.
As to S19, Art.IV of the 1973 Constitution, “trial in absentia” may be had when all the ff are
present: 1) arraignment, 2) the accused has been notified, and 3) he fails to appear and his failure
to do so is unjustified. All these were present here. Teodoro was informed of the scheduled
hearings. No explanation for his failure to appear in court in any scheduled hearing was given.
Thus, the trial court correctly proceeded with the reception of the evidence of the prosecution and
other accused in the absence of Teodoro, but it erred when it suspended the proceedings as to
Teodoro and rendered a decision as to the other accused only.

Upon termination of a trial in absentia, the court has the duty to rule upon the evidence presented.
It need not wait for when the accused who escaped from custody finally decides to appear in
court to present his evidence and cross-examine witnesses. To allow the delay of proceedings for
this purpose is to render ineffective the provision on trial in absentia.

The contention of judge Nazareno that the right to be presumed innocent will be violated if
judgment is rendered as to Teodoro is untenable. He is still presumed innocent. A conviction must
still be based on evidence presented that prove his guilt beyond reasonable doubt. There is also no
violation of due process since accused was given opportunity to be heard.

An escapee does not retain his rights to cross-examine and to present evidence on his behalf. He
virtually waived these rights by his failure to appear during trial of which he had notice. Thus, an
escapee who has been duly tried in absentia WAIVES his right to present evidence on his own
behalf and to confront and cross-examine witnesses who testified against him.

SUSPENSION of the PRIVILEGE of the WRIT of


HABEAS CORPUS
199. Rep. Edcel Lagman v. Medialdea, GR 231658, July 4, 2017, del Castillo, J. (Suspension
of the Privilege of the Writ of HC)
FACTS:
On May 23, 2017, for a period not exceeding 60 days, President Duterte issued Proc. 216 declaring
a state of ML and suspending HC in the whole of Mindanao. He submitted a written report to
Congress on the factual basis of Proc. 216. It explained that a government operation to capture
high-ranking Abu Sayyaf (ASG) officers and the Maute group was conducted. These groups
confronted the government by intensifying their efforts at sowing violence. The report said that
the Maute and ASG aimed to establish an Islamic State.

Maute and ASG members attacked government and private facilities in the Mindanao City of
Marawi. 50 armed criminals assaulted Marawi City Jail who forcibly entered its gates. Power
supply into Marawi City was interrupted. They ambushed and burned the Marawi Police Station.
The Maute occupied several areas in Marawi and set up road blockades and checkpoints. They
attacked hospitals and educational institutions and held hostage the people therein. The president
concluded that their aim is to establish the groups’ seat of power in Marawi for their planned
establishment of a DAESH wilayat or province.
These are various petitions. Lagman claims that the martial law declaration has no sufficient
factual basis as there is no rebellion or invasion in Marawi or in any part of Mindanao. Acts of
terrorism in Mindanao do not constitute rebellion as there is no proof that its purpose is to remove
Mindanao or any part thereof from its allegiance to PH. It prays that the SC declare as void Proc.
216.

HELD:
1. Locus Standi- transcendental issues.
2. “Appropriate proceeding”?
“The Supreme Court may review, in an appropriate proceeding filed by any citizen, the
sufficiency of the factual basis of the proclamation of martial law or the suspension of the
privilege of the writ or the extension thereof, and must promulgate its decision thereon
within thirty days from its filing.”

Jurisdiction is conferred only by the constitution or by law. unless it has been specifically
conferred, no body or tribunal has the power to act or pass upon a matter brought before it. Without
clear legislative intent, jurisdiction cannot be implied from the language of the constitution or law.
S18, Art. VII specifically grants authority to the SC to determine the factual basis of ML or
suspension of HC.

“Appropriate proceeding” does not refer to certiorari. The standard of review there is if there is
GADALEJ. It is not the proper tool to review factual sufficiency of ML or the suspension.

S18, Art.VII constitutionalized the pre-Marcos ruling in In the Matter of the Petition for HC of
Lansang that the factual basis of ML or suspension of HC is not a political question but within
the ambit of judicial review. The Lansang case reversed Montenegro v. Castañeda saying that the
authority to determine a state of rebellion requiring suspension of HC is lodged with the president
whose decision is conclusive upon the courts. But Garcia-Padilla v. Enrile, during Marcos ML,
reverted to Montenegro. Thus, in effect, the framers constitutionalized Lansang. The purpose of
S18 is to provide additional safeguard against possible abuse of the president of the exercise of his
extraordinary powers. It also relaxed the rule on standing by allowing any citien to question the
factual sufficiency of ML or suspension of HC. Its purpose it to curtail the powers of the president.
Thus, to interpret “appropriate proceeding” as filed under S1, Art.VIII would be contrary to the
intent of the Constitutionas S18, third par., Art. VII would be subsumed under S1, Art.VIII.

The unique features of the third par. of S18, Art. VII indicates that it should be treated as sui
generis different from those in Art.VIII.

3. The Court’s review of the factual sufficiency of ML or suspension of HC is independent of the


actions taken by Congress. The Court may strike down the proclamation in an appropriate
proceeding filed by any citizen on the ground of lack of sufficient factual basis. Congress may
revoke the proclamation or suspension, which revocation shall not be set aside by the President.
In reviewing the factual sufficiency, the Court considered only the info and data available to the
president prior to or at the time of the declaration. It is not allowed to undertake an independent
investigation beyond the pleadings. On the other hand, Congress may take into consideration not
only data available prior to, but also events supervening the declaration. The SC’s power to
review is passive initiated by a petition while Congress’ review is automatic- it may be activated
by Congress at any time after the proclamation or suspension. Thus, the two are different and
independent although they have the same trajectory to nullify the proclamation. The declaration in
Fortun v. President Macapagal-Arroyo that the Court must review only if Congress defaults in its
duty is set aside.

4. The judicial power to review the factual sufficiency does not extend to the calibration of the
president’s decision of which among his graduated powers he will avail of in a given situation.

Among his 3 extraordinary powers, the calling out power is the most benign and involves ordinary
police action. He may resort to this whenever it becomes necessary to prevent or suppress lawless
violence, invasion, or rebellion. The only limitation is that he should not act with grave abuse of
discretion. The actual use to which the president puts the armed forces is not subject to judicial
review.

The power to declare ML or suspend HC may be exercised only when there is actual invasion or
rebellion, and public safety requires it. The 1987 Constitution limits these powers: 1) time limit
of 60 days, 2) review and possible revocation by Congress and 3) by the SC. These powers
involve suppression of civil rights and individual freedom. ML serves as a warning to citizens
that the Executive department has called upon the military to assist in the maintenance of
law and order and, while the emergency remains, the citizens must, under pain of arrest and
punishment, not act in a manner that will render it more difficult to restore order and
enforce the law.

What powers could the president exercise during ML that he could not without ML? ML does not
suspend the operation of the Constitution nor supplant the functioning of civil courts or legislative
assemblies. Fr. Bernas in the deliberations says that the president, during ML, may have the powers
of a commanding general in a theatre of way. He, as commanding general, has authority to issue
orders which have the effect of law but strictly in a theater of war (traditional concept of ML
which has reference to the theater of war). Thus, the president exercises police power, normally a
function of the legislature, with military assistance to ensure public safety and in place of
government agencies which for the time being are unable to cope with the condition in a locality.

In David v. President Macapagal-Arroyo, the SC said that under a valid ML, the president as
commander-in-chief may order the 1) arrests and seizures without warrants, 2) ban on public
assemblies, 3) takeover of news media and agencies and press censorship, and 4) issue presidential
decrees. But these do not let the president infringe on the rights of civilians during ML as the
constitution and its BoR remain in place. And where the privilege of HC is also suspended, such
suspension applies only to those judicially charged with REBELLION or offenses connected
with INVASION.

From the least benign, the constitution gives the president: calling out power, power to suspend
HC, and to declare ML. But the graduation refers to the hierarchy based on scope and effect and
not to a sequence or order which the president must follow. The determination, at least initially, of
which power to use in a given situation lies with the president. Thus, to calibrate the president’s
decision would be an incursion into the exclusive domain of the executive and infringe the
prerogative that solely, at least initially, lies with the president. Also, the framers intended
Congress not to interfere a priori in the decision-making process of the president by eliminating
the requirement of prior concurrence of Congress in the initial imposition of ML or suspension of
HC. Thus, the Court must similarly refrain from calibrating the president’s decision which among
his extraordinary powers to avail given a situation.

Ther recommendation of the defense secretary is not a condition to declare ML or suspend HC. A
plain reading of S18, Art.VII shows that the power therein is not subject to any condition except
for the requirements of actual invasion or rebellion and that public safety requires it. It would be
contrary to common sense if the decision of the president is dependent on the recommendation of
his mere alter ego.

5. Proc. 216 cannot be facially challenged using the vagueness doctrine as it does not regulate
speech, religious freedom, and other fundamental rights. It seeks to penalize conduct, not speech.
There is also no need to determine the constitutionality of the implementing or operational
guidelines etc. issued after the proclamation as these are irrelevant to the review. Any act
committed under these orders must be resolved in a separate proceeding.

6. The calling out power is in a different category from the power to declare ML or to suspend HC.
The nullification of Proc. 216 will not affect Proc. 55, declaring a state of national emergency and
calling upon the AFP and PNP to suppress all forms of lawless violence in Mindanao. The
president may exercise the power to call out the AFP independently of the power to suspend HC
or declare ML. Under the “operative fact” doctrine, the unconstitutional statute is recognized as
an operative fact before it is declared unconstitutional.

7. The scope of the power to review only refers to determining factual sufficiency. The
“arbitrariness test” in Lansang has been done away with. The phrase “sufficiency of factual basis”
in S18, Art.VII is the only test for judicial review of the power to declare ML or suspend HC.
The Court does not need to satisfy itself that the president’s decision is correct. It only needs to
determine if the decision had sufficient factual bases. As said, the Court’s review is limited to
facts known by or available to the president at the time he made the declaration or suspension. As
to how far the past events are from the present depends on if they may be considered as
jusitification for the declaration or suspension and connected or related to the current situation
existing at the time of declaration. Events after the proclamation cannot be considered.

The Court should look into the full complement or totality of the factual basis and not piecemeal.
Neither should the Court expect absolute correctness of the facts stated in the proclamation or
report to Congress as he could not be expected to verify the accuracy of all facts reported to him
due to the urgency of the situation. It is irrelevant to the Court’s review if subsequent events prove
that the situation had not been accurately reported to him. After all, the review is limited to the
sufficiency, not accuracy, of his info. Thus, our review is limited to an examination of whether the
president acted within the bounds set by the Constitution: whether the facts in his possession prior
to and at the time of the declaration or suspension are sufficient for him to declare martial law or
suspend the privilege of the writ of habeas corpus.
8. S18, Art.VII sets the parameters for determining the sufficiency of factual basis for declaring
ML or suspending HC, namely: 1) ACTUAL INVASION or REBELLION, and 2) PUBLIC
SAFETY REQUIRES the exercise of such power. Without the concurrence of these two, the
proclamation is invalid. And, following infra, 3) there is PROBABLE CAUSE for the president
to believe that there is actual rebellion or invasion.

A word in a statute which has a technical or legal meaning is construed to have the same technical
or legal meaning. Since the Constitution did not define “rebellion”, it must be understood to
havethe same meaning as the crime of “Rebellion” in the RPC. To give it a different definition
would create confusion and give the president wide latitude of discretion which may be abused. In
determining existence of rebellion, the president only needs to convince himself that there is
PROBABLE CAUSE or evidence showing that more likely than not a rebellion was committed
or is being committed. A higher standard of proof would restrict the exercise of emergency
powers. This merely necessitates an “average man to weigh the facts and circumstances without
resorting to the calibration of the rules of evidence of which he has no technical knowledge. He
merely relies on common sense and needs only to rest on evidence showing that, more likely than
not, a crime has been committed by the accused.”

9. There is sufficient factual basis. The elements of rebellion are:


1. That there be (a) public uprising, and (b) taking up arms against the Government; and 2.
That the purpose of the uprising or movement is either: (a) to remove from the allegiance
to said Government or its laws the territory of the Philippines or any part thereof, or any
body of land, naval or other armed forces or (b) to deprive the Chief Executive or Congress,
wholly or partially, of any of their powers or prerogatives.

It is conceded that there is an armed public uprising in Marawi. The contention that the armed
hostilities is not for the culpable political purpose lacks merit. The president considered various
facts like the attacks in Marawi and the bombing in other places and occasions and the takeover of
major social, economic, and political foundations which paralyzed Marawi. The president
concluded that the Maute were attempting to remove from allegiance to PH Marawi City. From
the facts available to him, he concluded that the culpable purpose of the armed public uprising was
to remove from the allegiance to PH a portion of its territory and to deprive the Chief Executive
of any of his powers and prerogatives, leading Duterte to believe that there was probable cause
that rebellion was and is being committed and that public safety requires ML and suspending HC.

A review of said facts similarly leads the Court to conclude that the president had sufficient factual
bases tending to show that actual rebellion exists. The president’s conclusion was reached after a
tactical consideration of the facts. The allegations that the facts stated in Proc. 216 and the report
to Congress are false does not persuade. The Court is not concerned about the absolute correctness
or accuracy of facts because to do so would unduly tie the hands of the president in responding to
an urgent situation.

10. Public Safety requires ML or suspension of HC in the whole of Mindanao. ML and


suspending HC is for the protection of the security of the nation and the good and safety of the
public. Suspension of HC is “precautionary, and although it might curtail certain rights of
individuals, it is for the purpose of denfending and protecting the security of the state.”
The president also receives vital classified and live information which assist him in making
decisions. The facts enumerated in the proclamation and report are not exhaustive as some info are
classified as confidential and cannot be divulged for reasons of national security. These documents
may contain info about position of government troops and that of the rebels, etc. The president can
rely on intelligence reports and classified documents. The Court has no machinery equal to that of
the Commander-in-Chief to ably assess the ground conditions. As mentioned, the Court’s review
is limited to the info mentioned in the report and proclamation. Thus, the Court will have to rely
on the fact-finding capabilities of the executive department. In turn, the executive will have to
open its findings to the Court, which it did during the closed door session.

As to the territorial coverage, the 1987 Constitution grants to the president as Commander-in-chief
the discretion to determine the territorial coverage of ML or suspension of HC. S18, Art.VII
states that the president may suspend HC or place “the PH or any part thereof under ML.” This
is a recognition that the president, as repository of vital classified and live information necessary
for calibrating the territorial application of ML and suspension of HC, has a more informed
understanding of what is happening on the ground. While there is a limit on the period, which is
60 days unless sooner nullified, there is none on the territorial scope or area of coverage. It is
merely stated “the PH or any part thereof” depending on the assessment of the president.

The Constitution has sufficient safeguards against possible abuses. It limited the grounds to actual
invasion or rebellion and the duration was fixed at 60 days unless sooner revoked. It is subject to
the veto powers of the Court and Congress. The Constitution recognizes that any further
curtailment of the presidential power would not generate any good among the 3 co-equal branches,
to the country, and its citizens. The president may just declare a revolutionary government that
sets him free to deal with the invasion or insurrection.

Rebellion is a crime of masses or multitudes. It cannot be confined a priori within predetermined


bounds. The precise extent or range of rebellion could not be measured by exact metes and
bounds. If the rebels take over the SC compound in Padre Faura, we cannot say that the rebellion
is limited to such compound as there might be other rebels positioned in nearby buildings like
PGH. There is no way of knowing that all rebels had stayed inside the Court compound. If he
imposes ML on the entire Manila, is the territorial coverage too expansive? To answer, we go back
to the premise that the discretion to determine territorial coverage lies with the president. This is
not only practical but also logical. The president has to respond quickly and need not wait for
another rebellion to be mounted in QC. Thus, he is not required to impose ML only in the Court
compound nor precluded to from expanding the coverage beyond said compound. It is difficult if
not impossible to fix the territorial scope of ML in direct proportion to the range of actual
rebellion and public safety because rebellion and public safety have no fixed physical dimensions.

The determination of the territorial scope belongs to the president. The Court cannot overreach
and must stay within its power.

To determine which of rebellion or terrorism is committed, we look into the main objective of the
malefactors. If it is political, it is rebellion. If it is to sow and create a condition of widespread and
extraordinary fear and panic among the populace to coerce the government to give in to an
unlawful demand, the crime is terrorism. Even if the insurgency in Marawi can also be
characterized as terrorism, it will not affect Proc.216. S2, RA 9372 states that nothing “in this act
shall be interpreted as a curtailment” of constitutionally recognized powers of the executive branch
of government.

The petition was dismissed.

200. In the matter of the petition for Habeas Corpus of Datukan Malang Salibo, GR 197597,
April 08, 2015, Leonen, J. (Suspension of the Privilege of the Writ of HC; mistaken identity)
FACTS:
From Nov.7-Dec.19, 2009, Datukan Malang Salibo and other Filipinos were allegedly in Saudi
Arabia for the Hajj Pilgrimage. While in Saudi, Salibo visited and prayed in the cities of Medina
etc. He returned to PH on Dec. 20. On August 3, 2010, Salibo learned that officers of Datu Hofer
police station suspected him to be Butukan Malang, who was one of the 197 accused of 57 counts
of murder in the Maguindanao Massacre of Nov.23, 2009. He had a pending warrant of arrest.

Salibo presented himself before the police officers to clear his name and explained that he was not
Butukan and he could not have participated in the massacre as he was in Saudi Arabia on Nov. 23.
He presented pertinent portions of his passport, boarding passes, etc. tending to prove that a certain
Malang Salibo was in Saudi from Nov.7-Dec.19. The police officers apprehended Salibo and tore
off page 2 of his passport evidencing his departure for Saudi on Nov.7. they detained him at the
police station for 3 days then transferred him to the criminal investigation group in Cotabato City
where he was detained for another 10 days. he was made to sign and thumbmark documents. Then
he was transferred to the QC jail, BJMP bldg. camp Bagong Diwa where he is currently detained.

Salibo filed with the CA an urgent petition for HC questioning the legality of his detention. It
issued the writ and ordered that Salibo be brought before the trial court. Solicitors, appearing on
behalf of the warden of the QC jail, argued that the HC petition should be dismissed as Salibo was
charged under a valid information. Thus, HC is no longer availing. Salibo countered that the
information, arrest warrant etc. referred by the warden all point to Butukan Malang, not Datukan
Malang Salibo. The trial court was convinced that Salibo was not Butukan. It thus ordered the
release of Salibo from detention.

The CA reversed the trial court’s decision on appeal. The CA found that Salibo’s arrest was made
under a valid information and arrest warrant. It said that the proper remedy was a motion to quash
even if Salibo was not Butukan. Salibo filed this petition for review.

ISSUE:
Whether Datukan Malang Salibo, detained by virtue of a warrant of arrest for and information in
court charging “Butukan S. Malang”, and who was arrested and detained without preliminary
investigation, may be granted his petition for Habeas Corpus.
HELD: YES.
The writ of HC exists as a speedy and effectual remedy to relieve persons from unlawful restraint.
It is summary in nature. It extends to all cases of illegal confinement or detention by which any
person is deprived of his liberty, or by which the rightful custody of any person is withheld from
the person entitled thereto. The restraint of liberty need not be related to any offense to entitle a
person to the efficient remedy of HC. It may be availed of as a post-convition remedy or when
there is an alleged violation of the liberty of abode.

In Gumabon, et al. v. Director of the Bureau of Prisons, Gumabon et al. were convicted of the
complex crime of rebellion with murder and commenced serving their sentence. Then this Court
promulgated People v. Hernandez in 1956, ruling that the complex crime of rebellion with murder
does not exist. Gumabon et al; then filed a petition for HC, praying for release and arguing that the
Hernandez ruling be applied retroactively to them. This Court ruled that they properly availed of
HC.

The writ of habeas corpus is different from the final decision on the petition for issuance of
the writ. The writ commands the production of the body of the person allegedly restrained of his
liberty. It is the final decision where a court determines the legality of the restraint. Between the
issuance of the writ and the final decision, it is the issuance that is essential. It sets in motion the
speedy judicial inquiry on the legality of any deprivation of liberty. Courts shall liberally issue
writs of HC even if the petition for its issuance is on its face devoid of merit. Although the
privilege of the writ of HC may be suspended in cases of invasion, rebellion, or when public
safety requires it, the writ itself may NOT be suspended.

It is true that a writ of HC may no longer be issued if the person deprived of liberty is restrained
under a lawful process or order of the court. The restraint then has become legal and the remedy
of HC is rendered moot.

In Ilagan v. Hon. Ponce Enrile, this court dismissed the petition for HC, ruling that it became moot
with the filing of the information for rebellion against the detained lawyers and warrants of arrest.
The function of HC is to inquire into the legality of one’s detention. Now that the incarceration is
by virtue of a judicial order in relation to criminal cases subsequently filed against them, the
remedy of HC no longer lies.

In such cases, instead of availing of HC, persons restrained under a lawful process or order of
the court must pursue the orderly course of trial and exhaust the usual remedies. This
ordinary remedy is to file a motion to quash the information or warrant of arrest. At any time
before a plea is entered, the accused may file a motion to quash complaint or information based on
any of the grounds in Rule 117, S3. If the motion to quash is based on an alleged defect of the
information or complaint curable by amendment, the court shall order the amendment. If based on
the ground that the facts alleged do not constitute an offense, the trial court shall give the
prosecution an opportunity to correct the defect. If after amendment the complaint or information
still suffers from the same defect, the trial court shall quash it.

But here, Salibo was not arrested by virtue of any warrant charging him of an offense nor restrained
under lawful process or order of a court. He was illegally deprived of liberty and correctly availed
of HC. The information and arrest warrant charged Butukan S. Malang, not Datukan Malang
Salibo. Further, Salibo was not validly arrested without warrant under Rule 113, S5. He was
neither committing nor attempting to commit an offense when in the presence of the police officers
of Datu Hofer police station etc. The officers thus deprived him of liberty without due process of
law, for which a petition for HC may be issued.
The arrest of Salibo is similar to the arrest of Atty. Risonar in Ilagan. They both went to verify and
contest any arrest papers against them and were then and there arrested. It was stated by Justice
Teehankee’s dissent that the lack of preliminary investigation deprived Atty. Risonar of his right
to due process of law- a ground for the grant of a petition for HC.

Salibo’s remedy is not a motion to quash information or arrest warrant. None of the grounds for
filing a motion to quash information napply to him. Changing the name in the information from
“Butukan” to “Datukan Malang Salibo” will not cure the lack of preliminary investigation in
this case. A motion for reinvestigation will not cure the defect of lack of preliminary investigation.
The information and arrest warrant were issued on the premise that Butukan and Datukan are
the same person. There is evidence, however, that the person detained by virtue of these processes
is not Butukan but another person named Datukan. Salibo presented his passport, identification
card, TIN card, and NBI clearance all bearing his picture and indicating his name as “Datukan”.
None of these showed that he used “Butukan.” There is also evidence that Salibo was not in the
country on November 23 when the massacre occurred.

In ordering Salibo’s release, we are not prejudging his guilt or innocence. Should the government
choose to prosecute him, it must pursue the proper remedies against him as provided in our
rules. Until then, we rule that Salibo is illegally deprived of his liberty and grant the petition.

WRIT of AMPARO
201. The Secretary of National Defense v. Raymond Manalo, GR 180906, October 07, 2008,
Puno, C.J. (Writ of Amparo)
FACTS:
Manalo recounted that one or two weeks before Feb. 14, 2006, several uniformed and armed
soldiers and CAFGU summoned to a meeting all the residents of their barangay in Bulacan. He
saw some soldiers when he passed by the barangay hall. On Feb. 14, Manalo was sleeping in their
house. At past noon, several armed soldiers entered their house and roused him. They asked him
if he was Bestre, but his mother, Ester Manalo, replied that he was Raymond, not Bestre. The
soldier slapped him on both cheeks and nudged him in the stomach. He was then handcuffed,
brought to the rear of his house, and forced to the ground face down. He was kicked on the hip,
ordered to stand and face up to the light, then forcibly brought near the road.

The men forced Manalo into a while L300 van. Once inside, he was blindfolded. The van drove
off then stopped. A person was brought inside and made to sit beside him. Both of them were
beaten up. He recognized the voice of the person beside him as his brother Reynaldo’s. They
reached a house. Raymond and Reynaldo were each brought to a different room. He was
questioned if he was a member of the NPA and each time he said no, he was hit with the butt of
their guns.

He was blindfolded while interrogated by higher officials, but he saw their faces. When the
interrogator left, his guards beat him up. Raymond was fed only at night usually with left-over and
rotten food. On the third week of detention, two men beat Raymond up, doused him with urine
and hot water, hit his stomach with a piece of wood, slapped his forehead twice with a .45 pistol,
and burnt some parts of his body with burning wood. They also subjected Reynaldo to the same
ordeal in another room. They warned him that they would kill him the next day. The following
night, Raymond tried to escape, but soldiers caught him and beat him up.

From April until May 2006, he was detained in a room by steel bars measuring 1x2 meters. He did
everything there including urinating, defecating, bathing, eating, and sleeping. One day,
respondents were fetched and went to a detachment in Bulacan with Hilario. They were detained
in a big two-storey house where Hilario’s men beat Ramond. They were then brought to Sapang
Bulacan and detained in a big unfinished house. Gen. Palparan talked to them. Manalo narrated
that Palparan said that “bibigyan ko kayo ng isang pagkakataon na mabuhay basta’t sundin niyo
ang lahat ng sasabihin ko” to tell his “magulang” that “huwag na pumunta sa mga rali, sa hearing,
sa karapatan at sa human right dahil niloloko lang kayo.”

Manalo agreed as they felt they could not do otherwise. At 3am, the same group that abducted
them brought them to their parents’ house. Hilario threatened Raymond’s parents that if they
continued to join human rights rallies, they would never see their children again. Respondents were
then brought back to Sapang. Gen. Palparan told Raymond to gain back strength and be healthy.
He said that they should prove that they are on the side of the military and warned that they would
not be given another chance.

After 3 months in Sapang, Manalo was brought to Camp Tecson and made to clean barracks. He
met Sherlyn, who told him that she was a UP student abducted from Bulacan. She said she was
tortured and raped. After a week, Reynaldo was also brought to Camp Tecson. 2 days after, two
other captives arrived. They were threatened that if they escaped, their families would be killed.
Before the hearing on November 6 or 8, 2006, respondents were brought to their parents to instruct
them not to attend the hearing, but their parents already left for manila. On November 22, 2006,
respondents were transferred to a camp in Limay Bataan. They stayed there until May 8, 2007.
They were beaten by soldiers. They were made to clean, cook, and help in raising livestock.

Manalo said that when Operation Lubog was launched, he was brought with soldiers to kill all
NPA sympathizers. He saw the killing of an old man doing kaingin because he had a son who was
an NPA member and he coddled NPA members in his house.

They were brought to a safehouse near the sea and stayed there from May 8 until June 2007. In
June, they were brought back to the camp in Limay. He narrated what he saw: 2 bodies being
brought from a truck, 2 Ita beaten up and killed etc. On June 13, they were brought to Pangasinan.
They started to plan their escape. In the evening of Aug.13, 2007, when the guards had a drinking
session and did not wake up to Raymond’s loud radio, they went towards the highway and boarded
a bus bound for manila. they were detained for 18 months. Dr. Molino confirmed by his findings
the physical injuries allegedly inflicted upon them.

This case was originally a petition for prohibition, injunction, and TRO filed before this Court to
stop petitioners herein from depriving respondents herein of their right to liberty and other basic
rights. While the petition was pending, the rule on the Writ of Amparo took effect on October 24,
2007. Thus, it was prayed that the petition be treated as an Amparo petition. The Court resolved
to treat the petition as one under the Amparo rule. A writ of Amparo was issued to petitioners
requiring them to file with the CA a return of the writ. The CA was directed to conduct a summary
hearing on the petition.

Petitioners dispute respondents’ account of their alleged abduction and torture. They filed a return
of the writ of amparo admitting the abduction but denying any involvement therein. The secretary
of National Defense said that he assumed office only on August 8, 2007 and was thus unaware of
the Manalo brothers’ alleged abduction.

The CA rendered a decision in favor of respondents and granted the writ of Amparo. Hence this
appeal.

ISSUE:
Whether a writ of amparo may issue in favor of the Manalo brothers who were detained, transferred
to various detention places, threatened, and tortured for 18 months by the military.
HELD: YES.
This is the first decision on the application of the Rule on the Writ of Amparo. The Court on
Oct.24, 2007 promulgated the Amparo rule in light of the prevalence of extralegal killing and
enforced disappearances. It was an exercise for the first time of the Court’s expanded power to
promulgate rules to protect our people’s constitutional rights. The Amparo rule was intended to
address the problem of “extralegal killings” and “enforced disappearances”. Its coverage
presently is confined to these two. “Extralegal killings” are “killings committed without due
process of law, i.e. without legal safeguards or judicial proceedings.” “Enforced disappearances”
are attended by the following characteristics: an arrest, detention or abduction of a person by a
government official or organized groups or private individuals acting with the direct or indirect
acquiescence of the government; the refusal of the State to disclose the fate or whereabouts of the
person concerned or a refusal to acknowledge the deprivation of liberty which places such persons
outside the protection of law."

The writ of Amparo originated in Mexico. “Amparo” literally means “protection” in Spanish. It
spread throughout the western hemisphere, gradually evolving into various forms in response to
the particular needs of each country. In Latin American countries except Cuba, amparo was
constitutionally adopted to protect against human rights abuses committed under military juntas.
Others like Colombia, Chile, Germany, and Spain have chosen to limit the protection only to some
constitutional guarantees or fundamental rights.

In the PH, while the 1987 Constitution does not explicitly provide for the writ of amparo, several
of the amparo protections of other countries are guaranteed by our charter. The 2 nd par. of Art.
VIII, S1 provides that judicial power can determine if there is GADALEJ of any branch of
government. We also have habeas corpus similar to Amparo libertad. But these remedies may not
be adequate to address the problem of extralegal killings and enforced disappearances. With the
swiftness required to resolve a petition for a writ of amparo thru summary proceedings and
the availability of appropriate interim and permanent reliefs under the amparo rule, thus offers
a better remedy. It provides a rapid judicial relief as it partakes of a summary proceeding requiring
only substantial evidence to make appropriate reliefs available to petitioner.
The writ is preventive in that it breaks the expectation of impunity in the commission of these
offenses. It is curative in that it facilitates subsequent punishment of perpetrators as it will
inevitably yield leads to subsequent investigation and action. The goal of both preventive and
curative roles is to deter the further commission of extralegal killings and enforced
disappearances.

Petitioners claim that the CA erred in believing fully the uncorroborated and self-serving testimony
of Raymond Manalo. To determine the if the evidence presented is metal-strong to satisfy the
degree of proof, we determine the cause of action. S1 of the Rule on the writ of Amparo provides
for the following causes of action:
Section 1. Petition. — The petition for a writ of amparo is a remedy available to any person
whose right to life, liberty and security is violated or threatened with violation by an
unlawful act or omission of a public official or employee, or of a private individual or
entity. The writ shall cover extralegal killings and enforced disappearances or threats
thereof.

S17 and 18 provide that the parties must establish their claims by substantial evidence. This is
defined as such relevant evidence as a reasonable mind might accept as adequate to support a
conclusion. After perusing the evidence, we affirm the CA findings that respondents were abducted
from their house on Feb. 14, 2006 and were detained until they escaped on aug. 13, 2007. The
abduction, detention, torture, and escape were narrated by Raymond in a clear and convincing
manner with countless candid details of respondents’ harrowing experience captured through
different senses etched in his memory. We are convinced that the reason for the abduction was the
suspicion that they were either members or sympathizers of NPA since they were looking for Ka
Bestre, who turned out to be Rolando. The efforts exerted by the military command to look into
the abduction were, at best, merely superficial.

Also, Gen. Palparan, at the very least, was aware of their captivity. In fact, he and his men did not
controvert Raymond’s claim that Palparan met them. Raymond’s affidavit and testimony were
corroborated by Reynaldo’s affidavit. Raymon’s familiarity with the facilities in Fort Magsaysay
like the “DTU” or Division Training Unit firms up his story that they were detained for some time
in said military facility. With the secret nature of an enforced disappearance and the torture
perpetrated on the victim during detention, it logically holds that much of the information and
evidence of the ordeal will come from the victims themselves, and the veracity of their
account will depend on their credibility and candidness of their written/oral statements. Their
statements can be corroborated by other evidence like the physical evidence left by the torture or
landmarks they can identify in the places where they were detained.

Do respondents have the right to the privilege of the writ of amparo? Raymond and Reynaldo are
now physically free and no longer in detention as they are now free. But they assert that they are
not free “in every sense of the word” as their movements continue to be restricted for fear
that people they have named in their affidavits and testified against are still at large and have
not been held accountable. These people are connected with the AFP and in a position to threaten
their life, liberty, and security. They are under threat of being abducted again or even killed,
violating their right to security of person. They assert that their cause of action consists in the
threat to their life and liberty, and a violation of their right to security.
The right to security finds a textual hook in Art. III, S2. It also protects privacy. The right to
security of persons is a guarantee of the secure quality of his right to be alive in S1, Art.III. “The
right to which each person has a right is not a life lived in fear that his person and property may
be unreasonably violated by a powerful ruler. Rather, it is a life lived with the assurance that
the government he established and consented to will protect the security of his person and
property. Security includes the right to enjoyment of life while existing.

-Various permutations of this right are:


First, right to security is freedom from fear. The UDHR provides for the right to security of
person. This is also found in the ICCPR. The PH is a signatory to both. In the context of S1 of the
Amparo rule, “freedom from fear” is the right and any threat to the rights to life, liberty or security
is the actionable wrong. Fear is a state of mind, a reaction; threat is a stimulus, a cause of action.
Fear can vary from one person to another. Thus, in the amparo context, it is more correct to say
that “right to security” is actually “freedom from threat.

Second, the right to security of person is a guarantee of bodily and psychological integrity and
security.

Third, it is a guarantee of protection of one’s right by the government. Protection includes


conducting effective investigations, organization of the government apparatus to protect victims
of extralegal killings or enforced disappearances and their families, and bringing offenders to
justice. The European Court of Human Rights (EHCR) has interpreted right to security not only
as prohibiting the state from arbitrarily depriving liberty, but imposing a positive duty on the
state to protect right to liberty.

We now apply. First, violation of right to security as freedom from threat to life, liberty, and
security. Respondents were threatened that if they escaped, them and their families would be killed.
They were told that it was still being decided if they would be executed. Now that they escaped,
this continuing threat to their life is apparent, moreso now that they implicated military officers
not only in their own abduction and torture, but also of other persons like Sherlyn etc. The threat
vitiates their free will as they are forced to limit their movements or activities. The
circumstances of their abduction, detention, torture, and escape reasonably support a conclusion
that there is an apparent threat that they will again be abducted and even executed. These are threats
to liberty, security, and life actionable thru amparo.

Next, violation of security as protection by government. The military failed to protect respondents
by themselves perpetrating the abduction etc. They also miserably failed to conduct an
effective investigation of respondents’ abduction. The one-day investigation was superficial and
limited. Jimenez relied on sworn statements of the six implicated CAFGU. He did not call for other
witnesses to test the alibis of the six. The secretary of National Defense also attested that he
directed the AFP Chief to investigate, who in turn directed to AFP units to establish the
circumstances of the alleged disappearance and reappearance of the respondents. But to this day,
almost a year after the directive was issued by the secretary, respondents have not been furnished
the results of the investigation which they now seek in this petition for writ of amparo. There is
thus substantial evidence to conclude that there is violation of respondents’ right to security as
guarantee of protection by the government.

As to the reliefs, the CA ordered 1) that petitioners furnish respondents of all official and unofficial
reports of the investigation of the case, 2) confirm in writing the present places of official
assignment of Sgt. Hilaro and Donald Caigas, and 3) produce to the CA all medical reports, records
and charts, and reports of any medicines and treatment prescribed to the Manalo brothers,
including a list of medical personnel who attended to them from Feb. 14, 2006 to Aug. 12, 2007.
Petitioners argue that reliefs 1 and 2 partake of the nature of search warrant.

The production order under the Amparo rule should not be confused with a search warrant for law
enforcement under S2, Art.III. Instead, the amparo production may be likened to the production
of documents or things under S1, Rule 27 of the RoC:

Section 1. Motion for production or inspection order. — Upon motion of any party showing
good cause therefor, the court in which an action is pending may (a) order any party to
produce and permit the inspection and copying or photographing, by or on behalf of the
moving party, of any designated documents, papers, books of accounts, letters,
photographs, objects or tangible things, not privileged, which constitute or contain
evidence material to any matter involved in the action and which are in his possession,
custody or control. . .
This subpoena pertains to a civil procedure that cannot be confused with unreasonable searches
prohibited by the Constitution.

Petitioners assert that reliefs 2 and 3 are irrelevant and unnecessarily compromise official functions
of the military officers and expose them to threats. But contrarily, the places of assignment are
relevant in ensuring the safety of respondents by avoiding their areas of territorial
jurisdiction. This also ensures that such officers can be served with notices and court processes.
The list of medical personnel is also relevant in securing information to create medical history of
respondents and make appropriate medical interventions. The CA decision was affirmed.

202. Secretary Leila de Lima v. Magtanggol Gatdula, GR 204528, February 29, 2013,
Leonen, J. (Writ of Amparo)
FACTS:
Gatdula filed a petition for writ of amparo in the RTC. It was directed against petitioners de Lima,
Director Rojas and deputy director Esmeralda of the NBI. Gatdula wanted de Lima et al. to cease
from framing up Gatdula for the fake ambush incident by filing bogus charges of frustrated
murder against him in relation to the alleged ambush. Instead of deciding whether to issue a writ
of amparo, the judge issued summons and ordered de Lima et al. to file an answer. The judge set
the case for hearing.

In the hearing, counsel for De Lima et al. manifested that return, not answer, is appropriate for
Amparo cases. Judge Pampilo insisted that since no writ was issued, return os not required but
answer. He said that the RoC apply suppletorily in Amparo cases and thus required an answer.
Judge Pampilo conducted a hearing on the main case. Even without return nor answer, he ordered
the parties to file their memoranda. The RTC rendered a “Decision” granting the issuance of the
writ of Amparo. It also granted the interim reliefs namely temporary protection, production, and
inspection orders involving the investigation of the attempted assassination of Esmeralda.

Petitioners assailed the RTC “Decision” thru certiorari as enunciated by S19 of the Amparo rule:
SEC. 19. Appeal. — Any party may appeal from the final judgment or order to the
Supreme Court under Rule 45. The appeal may raise questions of fact or law or both. . . .

ISSUE:
Whether the “Decision” granting the privilege of the writ and the interim reliefs is the “final
judgment or order” contemplated under S19 of the Rule on the Writ of Amparo that may be
appealed under Rule 45.
HELD: NO.
The “Decision” granting the writ of Amparo is not the judgment or final order contemplated
in S19. Hence, a petition for review under Rule 45 may not yet be the proper remedy at this time.

The remedy of amparo is an equitable and extraordinary remedy to safeguard the right of the people
to life, liberty, and security as enshrined in the 1987 Constitution. The amparo rule was issued as
an exercise of the SC’s power to promulgate rules concerning the protection and enforcement of
constitutional rights. It aims to address extrajudicial killings and enforced disappearances, among
others. Due to the urgent and delicate nature of these controversies, the procedure was made to
afford swift but decisive relief.

It is initiated thru a petition to be filed in a RTC, Sandiganbayan, CA, or SC. The judge or
justice then makes an “immediate” evaluation of the facts as alleged in the petition and affidavits
submitted with the “attendant circumstances detailed.” After evaluation, the judge has the option
to issue the Writ of Amparo or immediately dismiss the case. Dismissal is proper if the petition
and the supporting affidavits do not show that the petitioner's right to life, liberty or security
is under threat or the acts complained of are not unlawful. On the other hand, the issuance of
the writ itself sets in motion presumptive judicial protection for the petitioner. The court
compels the respondents to appear before a court of law to show whether the grounds for more
permanent protection and interim reliefs are necessary.

The respondents are required to file a return after issuance of the writ thru the clerk of court. It
serves as the responsive pleading to the petition. Unlike an answer, the return has other purposes
aside from identifying the issues in the case. Respondents are also required to detail the actions
they had taken to determine the fate or whereabouts of the aggrieved party.

If respondents are public officials or employees, they are also required to state the actions they had
taken to (i) verify the identity of the aggrieved party; (ii) recover and preserve evidence related
to the death or disappearance of the person identified in the petition; (iii) identify witnesses and
obtain statements concerning the death or disappearance; (iv) determine the cause, manner,
location, and time of death or disappearance as well as any pattern or practice that may have
brought about the death or disappearance; and (vi) bring the suspected offenders before a
competent court. Clearly these matters are important to the judge so that s/he can calibrate the
means and methods that will be required to further the protections, if any, that will be due to the
petitioner

There will be a summary hearing only after the return is filed to determine the merits of the
petition and whether interim reliefs are warranted. If return is not filed, the hearing will be ex
parte. After the hearing, the court will render judgment within 10 days from the time the petition
is submitted for decision.

If the allegations are proven with substantial evidence, the court shall grant the privilege of the
writ and reliefs it may find proper. The judgment should contain measures which the judge views
as essential for the CONTINUED PROTECTION of the petitioner. These must be detailed
enough so that the judge may monitor the actions taken by respondents. It is this judgment that is
subject to appeal to the SC thru Rule 45. After the measures have served their purpose, judgment
will be satisfied. In amparo cases, this is when the threats to petitioner’s life, liberty, and security
cease to exist as evaluated by the court. The case may also be terminated thru consolidation should
a subsequent case be filed, either criminal or civil. Until the full satisfaction of the judgment,
amparo allows vigilant judicial monitoring to ensure protection of constitutional rights.

The “Decision could not be the final order appealable under S19 of the Amparo Rule. It pertained
to issuance of the writ under S6 of the rule, not the judgment under S18. The “Decision is thus
an interlocutory order as suggested by the fact that temporary protection, production, and
inspection orders were given together with the decision. These are interim reliefs granted upon
filing of the petition but before final judgment.

The filing of an answer was inappropriate. It is the return that is the responsive pleading. The
answer is contrary to the intent of the court to provide a speedy remedy. Judge Pampilo applied
the Rules of Summary Procedure as S25 of the Amparo rule states that the RoC applies
suppletorily. But the Rules on summary procedure is a special rule that the court devised for
specific circumstances like civil cases- forcible entry etc.- and criminal cases- violation of traffic
laws, rental law, etc. Thus, this type of summary procedure only applies to MTC/MTCC/MCTCs.
It is mind boggling how this rule could possibly apply to proceedings in an RTC. Also, that rule is
limited to certain civil/criminal cases. a writ of amparo is a special proceeding. It is a remedy by
which a party seeks to establish a status, a right or particular fact. It is NOT a civil nor
criminal action.

The second irregularity is the hearing on the main case before the issuance of the writ and filing
of return. Without a return, the issues could not have been properly joined. Worse is the third
irregularity when Pampilo required a memorandum in lieu of a responsive pleading (answer)
of De Lima et al.

The return allows the respondents to frame the issues subject to a hearing. It should thus be done
prior to the hearing, not after. A memorandum is a synthesis of the claims of the party litigants
and is a final pleading usually required before the case is submitted for decision. These have
different functions in facilitating the suit. More importantly, a memorandum is a prohibited
pleading under the Amparo Rule.
The fourth irregularity was the “Decision” itself. The body states “this court GRANTS the
privilege of the writ and the interim reliefs”. This gives the impression that it was the “judgment”
since the phraseology is similar to S18 of the Amparo Rule (grant the privilege of the writ and
such reliefs). The PRIVILEGE of the writ of amparo should be distinguished from the ACTUAL
ORDER called the writ of amparo. The privilege includes availment of the entire procedure
outlined in AM07-9-12-SC, Amparo Rule. After examining the petition, the affidavits, the return,
and evidence presented in the summary hearing, the judgment should detail the required acts from
respondents that will mitigate, if not totally eradicate, the violation of or the threat to petitioner’s
life, liberty, or security.

The petition for review is not the proper remedy to assail the interlocutory order, “Decision”. A
certiorari is prohibited. Simply dismissing the present petition will cause grave injustice to the
parties. Thus, in the interest of justice, the SC nullified the orders of Pampilo and directed him
within 48 hours to determine if the issuance of the writ of amparo is proper based on the petition
and its attached affidavits.

203. Edgardo Navia v. Virginia Pardico, GR 184467, June 19, 2012, Del Castillo, J. (Writ of
Amparo)
FACTS:
On March 31, 2008 around 830 pm, a vehicle of Asian Land Strategies Corp (ALSC) arrived at
the house of Lolita in Malolos. The arrival awakened Lolita’s son, Bong and Ben. Lolita saw two
uniformed guards disembarking from the vehicle. One asked Lolita where they could find Bong.
Before she could answer, the guard saw Bong and told him that he and Ben should go with them
to the security office of ALSC because a complaint was lodged against them for theft of electric
wires and lamps in the subdivision. Bong, Lolita, and Ben were then in the office of the security
department of ALSC located also inside Grand Royale Subdivision where Lolita lived.

-Version of Respondent.
Bong and Ben were unlawfully arrested, shoved into ALSC’s vehicle, and brought to the security
office. Navia slapped him. Ben begged for mercy but he was punched by Navia who hit him on
different parts of his body. Navia held his gun, looked at Bong, and said “Wala kang nakita at
narinig, papatayin ko na si Ben.” Bong admitted that he and Ben attempted to take the lamp to
transfer it near their house. But it was not working anymore so he reinstalled it on the post. Lolita
was made to sign the guard’s logbook to prove that they released Bong unharmed but that Ben had
to stay as his case will be forwarded to the barangay. Lolita signed since she has poor eyesight.
Thus, Lolita and Bong, afraid of Navia, left Ben behind in the office. Lolita was made to sign the
logbook again after being assured that it only pertains to Bong’s release.

The next morning, Virginia went to ALSC security office to visit Ben, but she was told that
petitioners had already released him with Bong the night before. She could not find her husband,
so she reported to the police. She filed a writ of amparo before the RTC. The amparo court
directed the issuance of a writ of amparo and the production of the body of Ben before it on June
30, 2008. Petitioners filed their compliance, praying for denial of the petition for amparo. After
summary hearing, the RTC issued the challenged decision granting the amparo.
The RTC directed the NBI to investigate petitioners, protect witnesses who testified, and the
prosecutor to investigate the legality of the arrest of Ben. Hence this petition.

ISSUE:
Whether a writ of amparo may issue against ALSC, a private entity.
HELD: NO.
Virginia’s petition for writ of amparo is fatally defective. The Amparo rule was made to provide
an expeditious and effective relief to any person whose right to life, liberty, and security is violated
or threatened with violation by an unlawful act or omission of a public official or employee, or of
a private individual or entity.

Here, Ben’s right to life, liberty, and security is settled as his identity as the person questioned at
petitioners’ security office is not disputed. This fact ipso facto established Ben’s inherent right to
life, liberty, and security. But Ben’s disappearance does not fall within the ambit of the Amparo
rule. S1 of AM 07-9-12-SC reads:
SECTION 1. Petition. — The petition for a writ of amparo is a remedy available to any
person whose right to life, liberty and security is violated or threatened with violation by
an unlawful act or omission of a public official or employee, or of a private individual or
entity. The writ shall cover extralegal killings and enforced disappearances or threats
thereof.
It does not define extralegal killings and enforced disappearances. This was intentional to let it
evolve through time and jurisprudence and thru substantive laws. Then the Court in Razon Jr. v.
Tagitis defined enforced disappearances. It said therein that the principles of international law
and adopted by the International Convention for the Protection of All Persons from Enforced
Disappearance’s definition thereof as “the arrest, detention, abduction, or any other form of
deprivation of liberty by AGENTS of the STATE or by persons or groups of persons acting with
the authorization, support, or acquiescence of the STATE, followed by a refusal to acknowledge
the deprivation of liberty or by concealment of the fate or whereabouts of the disappeared person,
which place such a person outside the protection of the law.”

Another development affected AM07-9-12-SC after Congress enacted RA 9851. S3(g) defined
enforced or involuntary disappearances:
(g) "Enforced or involuntary disappearance of persons" means the arrest, detention, or
abduction of persons by, or with the authorization, support or acquiescence of, a State or a
political organization followed by a refusal to acknowledge that deprivation of freedom or
to give information on the fate or whereabouts of those persons, with the intention of
removing from the protection of the law for a prolonged period of time.”
Thus, in probing disappearance cases, courts should read AM07-9-12-SC in relation to RA 9851.
From this definition, we can derive the following elements: (a) that there be an arrest, detention,
abduction or any form of deprivation of liberty; (b) that it be carried out by, or with the
authorization, support or acquiescence of, the State or a political organization; (c) that it be
followed by the State or political organization's refusal to acknowledge or give information on the
fate or whereabouts of the person subject of the amparo petition; and, (d) that the intention for
such refusal is to remove subject person from the protection of the law for a prolonged period of
time.
Thus, it is not enough to prove that the persons subject of the writ are missing. It must also be
proved by substantial evidence that the disappearance was carried out by or with authorization etc.
of the state or political organization. The GOVERNMENT PARTICIPATION must be
proved by substantial evidence. Here, we do not doubt Bong’s testimony that Navia had a
menacing attitude towards Ben and slapped and beat him. But the indispensable element of State
participation is not present here. While a writ of amparo may lie against a private individual
or entity, there must still be government involvement in the disappearance. Petitioners are
mere security guards in ALSC, private entity, and do not work for the government. Nothing was
presented to link or connect them to some government operation.

204. Arthur Balao v. Gloria Macapagal Arroyo, GR 186050, December 13, 2011, Villarama,
Jr., J. (Writ of Amparo)
FACTS:
James Balao is a psychology and Economics graduate of UP-Baguio. In 1984, he was among those
who founded the Cordillera People’s Alliance (CPA), a coalition of NGOs working for the cause
of indigenous peoples in Cordillera. James actively helped in training and the organization of
farmers. While working for CPA in 1988, he was arrested on the charge of violation of the Anti-
Subversion Law, but the case was dismissed for lack of evidence.

On September 17, 2008 around 830am, a white van arrived in front of a store in Benguet. 5 men
in civilian clothes carrying firearms alighted from the van and approached the James who was
standing in front of the store. One of the men addressed the witnesses, saying they were policemen.
Another said that the man was being arrested for drugs. They pushed James inside the van. The
driver was told to go to Camp Dangwa, PNP headquarters. James had told his family in May 2008
that he and his family were being surveilled.

The Balao family, with assistance of CPA and other NGOs, tried to locate James to no avail.
Petitioners also enumerated several incidents of harassments and human rights violations against
CPA officers.

Arthur, Winston, Nonette, and Jonilyn Balao, siblings of James Balao, and Beverly Longid
(petitioners) filed with the RTC a petition for issuance of writ of amparo in favor of James who
was abducted by unidentified armed men in Benguet. Named respondents were then president
GMA, ES Ermita, Defense Secretary Teodoro, PNP Director General Verzosa, etc. and several
John Does. They prayed that respondents be ordered to disclose where James is confined, to release
him, and to cease from further harming him. The writ of amparo was issued. Respondents, in their
return, contending that the petition failed to meet the requirement in the Amparo rule that claims
be established by substantial evidence.

The RTC issued a writ of amparo, ordering respondents to disclose where James is detained, to
release him, and to cease from further harming hi,. It denied the issuance of inspection order,
production order, and witness protection order for failure of petitioners to comply with the
provisions of the amparo rule. It found respondents’ investigation as very superficial and one-
sided. Hence, both parties appealed to this Court.

ISSUE:
Whether a writ of amparo may issue from evidence based on similar past abductions to show that
the current abduction of James Balao is most likely perpetrated by the government.
HELD: NO.
S18 of the Amparo Rule provides that the allegations in the petition must be proven by substantial
evidence. The issue here is whether the totality of evidence satisfies the degree of proof to establish
enforced disappearance. In granting the writ, the trial court considered James’ activitist leanings
as the most likely motive for his disappearance shown by several harassment incidents of activities
testified by Beverly longid. It considered these enough circumstances to establish substantial
evidence of enforced disappearance. The RTC gave considerable weight to the discussion in the
petition of briefing papers supposedly obtained from the AFP indicating that the anti-insurgency
campaign of the military under PGME included targeting identified legal organizations including
the CPA as “enemies of the state.” The petition enumerated previously documented cases of
extralegal killings of activities including CPA leaders and workers, almost all of which have been
preceded by military surveillance.

But we hold that the documented practice of targeting activists in the military’s counter-insurgency
program by itself does not fulfill the evidentiary standard in the Amparo rule to establish an
enforced disappearance. The similarity between circumstances attending a particular case of
abduction with those of previous instances of enforced disappearances does not necessarily
carry sufficient weight to prove that the government orchestrated the abduction. The RTC
cannot simply infer government involvement in James’ abduction from past similar incidents.

The petition also premised government complicity, stating that respondents have command
responsibility of all acts of their subordinates, thus the abduction can only be attributed to them.
Command responsibility, acc to Fr. Bernas, means the “responsibility of commanders for crimes
committed by subordinate members of the armed forces or other persons subject to their control in
international wars or domestic conflict." It is in this sense a form of criminal complicity. It may
be contended that command responsibility is legal basis to hold military/police commanders liable
for extra-legal killings or enforced disappearances. But it would be inappropriate to apply it as a
form of criminal complicity thru omission for individual respondents’ criminal liability is
beyond the reach of amparo. The Court does NOT rule in such proceedings the issue of
criminal culpability, even if incidentally a crime of infraction of an administrative rule may
have been committed. It does not determine guilt. It determines RESPONSIBILITY, or at least
ACCOUNTABILITY, for the enforced disappearance, threats thereof, or extrajudicial
killings for the purpose of imposing appropriate remedies.

But commanders may be impleaded, not on the basis of command responsibility, but rather on
their responsibility, or at least accountability. “Responsibility” refers to the extent the actors have
been established by substantial evidence to have participated in whatever way, by action or
omission, in an enforced disappearance, as a measure of the remedies this Court shall craft, among
them, the directive to file the appropriate criminal and civil cases against the responsible parties in
the proper courts. “Accountability” refers to the measure of remedies that should be addressed to
those who exhibited involvement in the enforced disappearance without bringing the level of
complicity to the level of responsibility defined above, or those who carry, but have failed to
discharge, the burden of extraordinary diligence in the investigation of the enforced
disappearance.
The participation in any manner of military and police authorities here has not been adequately
proven. The identities of the abductors have not been established, much less their link to any
military or police unit. There is also no evidence indicating that James is being detained upon
orders or acquiescence of government agents. But we agree with the RTC that the actions of
respondents were “very limited, superficial, and one-sided.” The investigation did not investigate
the military officials believed to be behind the abduction and did not lead to Camp Dangwa where
the abductors were supposed to have proceeded.

They claim that petitioners did not cooperate with the military. But such non-cooperation provides
no excuse. Petitioners do not trust the government agencies to protect them. As police officers,
their duty is to thoroughly investigate the abduction, a duty that includes looking into the cause,
manner and like details of the disappearance; identifying witnesses and obtaining statements
from them; and following evidentiary leads; and securing and preserving evidence related
to the abduction and the threats that may aid in the prosecution of the persons responsibility.

Thus, respondents failed to discharge their burden of extraordinary diligence in the investigation
of James’ abduction. The SC cannot thus completely exonerate respondents from allegations of
accountability for James’ disappearance. The SC remanded the case back to the RTC for further
investigation by PNP and CIDG.

As to the denial of the interim reliefs under the amparo rule, an inspection order is an interim
relief designed to support the claim of a petitioner in an amparo petition, in order to aid the court
before making a decision. A basic requirement before an inspection order may be granted is that
the place to be inspected is reasonably determinable from the allegations of the party seeking the
order. Here, the order was properly denied since the petitioners specified several military and
police establishments based on testimonies of victims in previous incidents of similar abductions
involving activitists that disclosed those premises as detention centers.

The SC reversed the grant of the writ of amparo. It ordered the AFP Chief etc. to continue the
investigations.

SECTION 16- RIGHT to SPEEDY


DISPOSITION of CASES
205. Rodrigo Duterte v. Sandiganbayan, GR 130191, April 27, 1998, Kapunan, J. (Speedy
Disposition of Cases)
FACTS:
Davao City Local Automation Project was launched by Davao City. Its goal was to make Davao a
leading center for computer systems and technology development. It aimed to provide consultancy
and training services and to assist all LGUs in Mindanao to set up their computer systems. A
computerization committee was made. The committee recommended the acquisition of Goldstar
computers made by Goldstar corp, SoKor, exclusively distributed in PH by Systems Plus, Inc.
(SPI). The Committee negotiated with SPI for acquisition and installation of the computer
hardware and personnel training. The Sanggunian of Davao authorized Davao mayor Duterte to
sign the contract in behalf of Davao.

The Ombudsman-Mindanao received a letter on November 27, 1990 from a concerned citizen
stating that some city officials are going to “make a killing” in the transaction. In February 1991,
a complaint was instituted in the RTC by Dean Braga etc. against petitioners, the city council etc.
for the nullity of the computer contract (Civil Case 20,550-91). Goldstar sent a proposal to Duterte
to cancel the computerization contract. The Sanggunian resolved to accept the offer to cancel. The
contract was mutually rescinded on May 6, 1991.

An audit team was made to determine if the contract conformed to government laws and
regulations. On May 31, the team recommended the rescission of the contract.

Davao, intent on pursuing its computerization plan, sought the assistance of National Computer
Center (NCC), which recommended acquisition of Philips computers. Davao obtained the
computers needed. On August 1, 1991, the Anti-Graft League filed an unverified complaint with
the Ombudsman against petitioners and Davao city and SPI. It alleged that the computerization
contract violated RA 3019 and other laws (OMB-3).

On Oct. 14, 1991, Judge Arcangel dismissed Civil case 20,550-91 since the mutual cancellation
of the contract mooted the case. In OMB-3,on Nov. 12, 1991, graft investigator Manriquez ordered
petitioners to file their comment upon every allegation in Civil Case 20,550-91.

Four years later on February 22, 1996, petitioners received a memorandum of prosecutor
Guzman addressed to Ombudsman Desierto regarding OMB-3. Guzman recommended petitioners
be charged under S3(g) of RA 3019 for entering into a contract grossly disadvantageous to the
government. Thus, an information was filed against petitioners before the sandiganbayan.
Petitioners filed an MR, stating that they were deprived of the right to preliminary investigation,
due process, and the speedy disposition of their case. The Ombudsman denied the MR. Their
motions to quash were also denied. Hence this petition.

ISSUE:
Whether a preliminary investigation by the ombudsman against Duterte et al. where they were
only required to submit their comment and where the preliminary investigation took 4 years to
finish, violated Duterte et al.’s right to speedy disposition of cases.
HELD: YES.
The preliminary investigation was not conducted in the manner laid down in AO 7. Petitioners
were merely directed to submit a point-by-point comment of the allegations in Civil Case 20,550-
91. The order was not accompanied by a single affidavit of any person charging petitioners of any
offense as required by law. Petitioners had no inkling that they were being subjected to a
preliminary investigation as there was no indication in the order that a preliminary investigation
was being conducted. While the right is statutory rather than constitutional, it is a component part
of due process in criminal justice. The right to have preliminary investigation is not merely a
formal or technical right. It is a substantive right. It is mandatory for the complainant to submit his
affidavit and those of his witnesses before the respondent can be compelled to submit his counter-
affidavits and other supporting documents.
There was also the undue and unreasonable delay in the termination of the irregularly
conducted preliminary investigation. Petitioners’ manifestation adopting the comments of their
co-respondents was filed on Feb. 18, 1992. But it was only on Feb.22, 1996, 4 years later, that
petitioners received a memorandum by Prosecutor Guzman recommending the filing of
information for S3(g) of RA 3019. The inordinate delay in the conduct of the “preliminary
investigation” infringed their right to a speedy disposition of their case.

It has been suggested that the long delay in terminating the preliminary investigation should not
be deemed fatal, for even the complete absence of a preliminary investigation does not warrant
dismissal of the information. True, but the absence of a PI can be corrected by giving the accused
such investigation. But undue delay in the conduct of PI cannot be corrected for man has not yet
invented a device for setting back time. (Tatad v. Sandiganbayan)

Ombudsman claims that Tatad does not apply as the charges there are politically motivated. But
the right to speedy disposition of cases does not come into play only when political considerations
are involved. The Constitution makes no such distinction. While political motivation in Tatad may
have been a factor in the undue delay in the termination of the PI to justify their invocation of the
right to speedy disposition, the particular facts of each case must be taken into consideration.

The concept of speedy disposition is a relative term and must be a flexible concept. The factors
that may be considered are the “length of the delay, the assertion or failure to assert such right
by the accused, and the prejudice caused by the delay. But petitioners here could not have urged
the speedy resolution of their case as they were completely unaware that the investigation
against them was still on-going. Peculiar to this case is that petitioners were merely asked to
comment, and not file counter-affidavits which is the proper procedure in PI. After giving their
explanation and after 4 years of being in the dark, petitioners had reason to assume that the charges
against them had already been dismissed. The Ombudsman failed to present any special or novel
reason to justify the 4-year delay. Its excuse that there are many layers of review and the case had
to undergo meticulous scrutiny has lost its novelty and is no longer appealing. The incident here
does not involve complicated factual and legal issues.

206. Francisco Tatad v. Sandiganbayan, GR 73225-39, March 21, 1988, Yap, J. (Speedy
Disposition of Cases)
FACTS:
Antonio de los Reyes, former head executive assistant of the then Department of Public
Information (DPI), filed a formal report with the Presidential Security Command (PSC) charging
Tatad, then Secretary of DPI, with alleged violation of RA 3019. No action was taken on the report.
5 years later, it became publicly known that Tatad resigned as minister of public information and
2 months after, Antonio filed a complaint with the Tanodbayan against Tatad, accusing him of
graft and corrupt practices in the conduct of his office as then secretary.

The Tanodbayan referred the complaint of Antonio on January 26, 1980 to the Criminal
Investigation Service (CIS) for fact-finding investigation. On June 16, 1980, CIS Investigator
Dizon submitted his report and concluded that from the evidence gathered, Tatad violated S3(e)
and S7 of RA 3019. Tatad moved to dismiss the complaint against him but this was denied on July
26, 1982. His MR was denied on Oct.5. On Oct. 25, 1982, all affidavits and counter-affidavits
were with the Tanodbayan for final disposition. On July 5, 1985, approved a resolution dated April
1, 1985 recommending that informations be filed against Tatad. Thus, on June 12,
1985,informations for violation of S3(b), S7, & S3(e) of RA 3019, were filed against Tatad.

Tatad moved to quash information, alleging deprivation of due process and right to speedy
disposition of cases. The Sandiganbayan rendered its challenged resolution denied the motion to
quash.

ISSUE:
Whether the period of 3 years for the termination of a preliminary investigation for the charge of
failing to file SALN, bribery, and giving unwarranted benefits to a relative while in government
office violates right to speedy disposition of cases.
HELD: YES.
Tatad claims that the Tanodbayan violated his right to due process and speedy disposition in
unduly prolonging the termination of the preliminary investigation and in filing the informations
only after more than a decade from the alleged offenses.

In a number of cases, this Court has not hesitated to grant the so-called “radical relief” and to
spare the accused from undergoing the rigors and expense of a full-blown trial where it is clear
that he has been deprived of due process or other rights. Particular regard must be taken of the
facts and circumstances peculiar to each case.

Here, Antonio originally filed a report with PSC on Oct.1974. This was made to “sleep” in the
PSC office until the end of 1979 when it became widely known that Tatad had a falling out with
Pres. Marcos and had resigned from the cabinet. On Dec. 12, 1979, the 1974 complaint was
resurrected thru formal complaint with the Tanodbayan. It acted on the complaint in April 1, 1980,
2 months after Tatad’s resignation was accepted by Marcos. On June 16, 1980, CIS report was
submitted to Tanodbayan, recommending filing of charges. By Oct. 25, 1982, all affidavit and
counter-affidavits were in and the case was ready for disposition. But it was only on July 5, 1985
that a resolution was approved by Tanodbayan recommending filing of informations. 5 criminal
informations were filed on June 12, 1985 against Tatad alone.

A review of the facts leaves the impression that political motivations played a vital role in
propelling the prosecutorial process in this case. The complaint came to life only after Tatad had
a falling out with Marcos. Departing from established procedures by law for PI, requiring
submission of affidavits and counter-affidavits, the Tanodbayan referred the complaint to PSC
for fact-finding investigation and report. This departure from established procedure is dubious.

Also, the long delay in resoling PI cannot be justified. PD911 prescribes a 10-day period for a
prosecutor to resolve a case under PI by him from its termination. While this period is merely
“directory”, yet it cannot be disregarded completely, with absolute impunity. The long delay
thus violated Tatad’s right to due process.

Substantial adherence to the requirements of the law governing the conduct of PI, including
substantial compliance with the time limitation prescribed by law for resolution of cases by
the prosecutor, is part of procedural due process. Not only under the broad umbrella of the due
process clause, but under the guarantee of “speedy disposition of cases in S16, BoR. A delay of
close to 3 years cannot be deemed reasonable or justifiable in light of the circumstances in this
case. We are not impressed by Sandiganbayan’s claim that the delay may be “due to a painstaking
and grueling scrutiny by Tanodbayan as to whether the evidence presented during PI merited
prosecution of a former high-ranking government official.” Such a statement suggests a double
standard of treatment. 3 out of the 5 charges against Tatad were for his alleged failure to file his
SALN which do not involve complicated legal and factual issues needing such “painstaking
and grueling scrutiny” to justify a 3 year delay in terminating PI. The other two charges relating
to alleged bribery and giving unwarranted benefits to a relative, while more substantial, do not
warrant the period of 3 years.

It is suggested that the long delay in terminating PI should not be deemed fatal, for even the
complete absence of PI does not warrant dismissal of information. True, but the absence of PI may
be corrected by giving such investigation. But an undue delay in PI cannot be corrected for, until
now, no man has yet invented a device for setting back time. Thus, the complaints were dismissed.

207. Abelardo Licaros v. Sandiganbayan, GR 145851, November 22, 2001, Panganiban, J.


(Right to Speedy Disposition of Cases)
FACTS:
The Legaspi City Branch of Central Bank was robbed of P19M. Modesto, one of principal accused,
with 4 companions, delivered in sacks a substantial portion of the stolen money to Concepcion
Building in Intramuros where Home Savings Bank had its offices to be deposited. Licaros was
then its vice chairman. Abelardo tried to report the incident to General Ver but could not get in
touch with him. He reported to CB Governor Laya his suspicion that the money may have been
stolen. The Tanodbayan, after preliminary investigation, filed an information for robbery with
sandiganbayan against, among others, Abelardo Licaros as accessory.

On Nov. 26, 1982, Tanodbayan filed an amended information. Nov. 29- Licaros was arraigned.
The prosecution rested on Oct. 1, 1984. On January 14, 1986, Licaros filed a motion for separate
trial. On Aug. 26, 1986, the case was deemed submitted for decision. In a resolution dated Oct 8,
1986, the sandiganbayan deferred the decision of the case as to Licaros until after submission
of the case as to the other accused. Licaros filed an MR, but it was denied.

The case was submitted for decision on June 20, 1990. As of this writing and more than 10 years
after the case was submitted for decision, the Sandiganbayan has not rendered the decision. It did
not render a decision even while the trial of Licaros as accessory were terminated as early as Oct.
8, 1986, while those against all the principal accused was deemed submitted for decision on June
20, 1990. As early as Oct. 16, 1986, Licaros already invoked his right to speedy justice when he
filed a motion praying the reconsideration of the sandiganbayan’s resolution deferring the decision
of his case until the submission for decision of the case of other accused. Licaros filed an omnibus
motion to dismiss on March 23, 2000. He filed a motion to resolve on Aug. 15 and motion for
early resolution on Sept. 21, 2000. These were unacted upon. Hence this petition.

ISSUE:
Whether the delay of more than 10 years of the promulgation of judgment from when the case was
submitted for decision in a criminal case entitles accused to a dismissal of said case on the ground
of violation of speedy disposition of cases where the accused has persistently moved for the
resolution of said case.
HELD: YES.
Under PD1606, S6, the Sandiganbayan has only 90 days to decide a case from the time it is deemed
submitted for decision. Even if we consider the period provided under S15(1), Art.III of the 1987
constitution, which is 12 months from submission of the case for decision, the Sandiganbayan
would still have miserably failed to perform its duty to render a decision on the case within the
period prescribed by law. Even more important than the period is the right against an
unreasonable delay in the disposition of one’s case before any judicial, quasi-judicial, or
administrative body. This right finds greater significance in a criminal case where any delay in
disposition may result in denial of justice.

Licaros’ right to speedy disposition was violated. The Sandiganbayan admitted that the case was
“one of those cases that got buried” in the archives during the reorganization in that court.

It is argued that the rulings in Tatad, Angchangco v. Ombudsman, and Roque et al. v. Office of the
Ombudsman should not apply here as what was involved there was a delay of the Ombudsman or
special prosecutor with respect to the holding of a preliminary investigation. The case here was
already proven by the ombudsman by the filing of the information.

We cannot accept this constrained interpretation. In the ordinary course of a criminal proceeding,
a court is responsible for the ultimate disposition of the case. This is true irrespective of the
prosecution’s punctual performance of its duty. Hence, notwithstanding the filing of
information, the presentation of evidence, and completion of the trial proper, the eventual
disposition of the case will still depend largely on the timely rendition of judgment by a court. If
it does not act promptly on the adjudication of a case, the accused’s right to speedy disposition is
just as prejudiced as when the prosecution is deferred indefinitely.

A breach of the right to speedy disposition may justify dismissal of a criminal case if the
proceedings have been marred by vexatious, capricious, and oppressive delays. Here, the 10-
year delay involved more than just mere procrastination in the proceedings. The case was kept in
idle slumber. Licaros had been assertively and assiduously invoking his right to speedy
disposition even before the case was submitted for decision on June 20, 1990. He filed an omnibus
motion to dismiss etc., all of which fell on deaf ears of the Sandiganbayan. Thus, Licaros cannot
be said to have slept on his rights. He was persistent in his demand for disposition of the case.

Licaros was kept in the dark as to the final outcome of the case. Although the failure is not
deliberately intended by Sandiganbayan, its unjustified delay has nonetheless caused as much
vexation and oppression. Hence, the case may be dismissed not so much on the basis of right to
speedy trial, but on the right to a speedy disposition of his case, which is of broader and more
appropriate application under the circumstances.

In determining if the right to speedy disposition was violated, the factors to consider are: (1) the
length of the delay; (2) the reasons for the delay; (3) the assertion or failure to assert such right by
the accused; and (4) the prejudice caused by the delay." The delay cannot be attributed to Licaros
who has been pushing for the resolution of his case. The delay caused much prejudice, distress,
and anxiety to Licaros, whose career as bank executive and businessman has suffered the stigma
of being shackled to an unresolved criminal prosecution. Thus, the dismissal is justified.

208. Capt. Roquero v. Chancellor of UP-Manila, GR 181851, March 9, 2010, Perez, J. (Right
to Speedy Disposition of Cases)
FACTS:
Petitioner Wilfredo Roquero is an employee of UPM assigned at PGH security division as special
police captain. Private respondent Imelda Abutal is a lady guard applying for a position in the
security force assigned at UP-PGH. Abutal filed a complaint for grave misconduct against Roquero
with then UPM Chancellor Perla. Abutal claimed that Roquero proposed to her to be his mistress
for which in return he would facilitate her application. Roquero was placed under preventive
suspension. The Administrative Disciplinary Tribunal (ADT) was organized to hear the case. Atty.
Flor was university prosecutor. Roquero was represented by Atty. Inovejas of PAO.

The prosecution presented its only witness, Abutal. After cross-examination, the prosecution
agreed to submit its formal offer of evidence on or before July 16, 1999. But it failed to submit
said formal offer within the period. When the case was called on August 10, only Roquero and his
counsel appeared. Atty. Flor merely called by telephone and requested that the case be reset to
another date. On August 11, only petitioner and his counsel appeared. Atty. Flor called again
asking for postponement of the hearing. Atty. Docena set the hearing on September 29. But the
prosecution failed to appear again.

Roquero prayed that Abutal be deemed to have waived her rights to formally offer her exhibits.
The ADT was not able to act on said motion for 5 years. Roquero filed on May 19, 2004 another
motion asking for the dismissal of the administrative case against him. On May 26, 2004, the
prosecution, represented now by Atty. Felicen, alleged that a formal offer of documentary exhibits
had been filed on January 24, 2004. On June 8, 2004, Atty. Docena denied Roquero’s motion to
dismiss.

Roquero then filed with the CA a certiorari. The CA denied the petition. Hence this petition.

ISSUE:
Whether the failure of ADT to resolve Roquero’s motion to declare complainant Abutal to have
waived her right to submit her formal offer of exhibit which he seasonably filed on Oct. 22, 1999
and the assailed order of ADT dated June 8, 2004 admitting the Formal Offer of Exhibit of
complainant Imelda Abutal despite having been filed after almost five years violated the
constitutional right of Roquero to a speedy disposition of cases.
HELD: YES.
The prosecution tried to explain that the resignation of Atty. Flor in August 1999, who had by then
been on leave since mid-July 1999, contributed to the delay of the filing of the formal offer and
the offer could not be prepared by another counsel until all the transcripts had been given to him.
The ADT admitted this explanation without asking why it took him almost 5 years to explain. The
prosecution failed to explain why it took them so long to find a replacement prosecutor. While
ADMINISTRATIVE investigations should not be bound by strict adherence to technical rules
of procedure and evidence applicable to judicial proceedings, the same should not violate the
right to speedy disposition of cases.

The right to speedy disposition is not limited to criminal proceedings but extends to ALL
PARTIES in ALL CASES, including civil and administrative cases, and in all proceedings,
including judicial and quasi-judicial hearings. Hence, under the constitution, any party to a case
may demand expeditious action by all officials who are tasked with the administration of justice.

The right to a speedy disposition, like the right to speedy trial, is deemed violated only when the
proceedings are attended by vexatious, capricious, and oppressive delays, or when unjustified
postponements of the trial are asked for and secured, or even without cause or justifiable
motive, a long period of time is allowed to elapse without the party having his case tried. Also
applicable is the balancing test to determine if a defendant has been denied his right to a speedy
trial or disposition of a case in which the conduct of both the prosecution and defendant is
weighed, and such factors as the length of delay, reasons, assertion, and prejudice.

Here, 1) the delay of almost 5 years of ADT in resolving the motion of Roquero, 2) the
unreasonableness of the delay, and 3) the timely assertions by Roquero of his right thru his
motion to dismiss. The delay was prejudicial to Roquero as he was under preventive suspension
for 90 days and during the interregnum of 5 years, the trial of the accusation against him remained
stagnant.

Sec. 17- RIGHT AGAINST SELF-


INCRIMINATION
209. US v. Tan Teng, GR 7081, September 07, 1912, Johnson, J. (Right Against Self-
Incrimination)
FACTS:
Defendant Teng was charged with rape. After hearing the evidence, Judge Lobingier convicted
him. Teng appealed, claiming that it was error to admit the testimony of physicians about having
taken a certain substance from the body of accused while he was confined in jail and regarding the
chemical analysis of the substance to demonstrate his physical condition as to a venereal disease.

Oliva was staying in her sister’s house. A number of chinamen were gambling near the house.
Some visited the house. Oliva, after taking a bath, went to her room. Teng followed her intoher
room and asked for some face powder. After using the face powder on his private parts, he threw
Oliva on the floor and placed his private parts upon hers and remained in that position for some
time. After a week or two, Oliva was discovered as suffering from a venereal disease known as
gonorrhea. Oliva told her sister what happened and pointed to Teng as the one who attempted to
violate her. Teng was then examined in the police station who swore that his body bore every sign
of gonorrhea. It took a portion of the substance emitting from his body. The analysis showed that
Teng was suffering from gonorrhea.

The mdical experts testified said that the disease can be communicated by the contact described
(resting penis on vagina).
ISSUE:
Whether the substance taken from Tan Teng’s body, used as evidence against him to prove that
the gonorrhea suffered by Oliva, a rape victim, was obtained from him, violates his right against
self-incrimination.
HELD: NO.
We believe that the gonorrhea suffered by Oliva was due to the illegal conduct of Teng. But it is
only corroborative of the truth of Oliva’s declaration. The defense contended that the result of the
scientific examination of the substance from Teng’s body is not admissible to prove that he was
suffering from gonorrhea, that to admit the evidence would be to compel Teng to testify against
himself.

The substance was taken from Teng’s body without his objection. Had the defendant been found
with stolen property, there certainly could have been no question had the stolen property been
taken for the purpose of using the same as evidence against him. So also if the clothing he wore,
by reason of blood stains or otherwise, had furnished evidence of a crime, there certainly could
have been no objection to taking such to use it as proof.

The prohibition of compelling a man in a criminal court to be a witness against himself is a


prohibition of the use of physical or moral compulsion to extort communications from him,
NOT an exclusion of his body as evidence when it may be material. To admit the contention
might exclude the testimony of a physician or medical expert who had been appointed to make
observations of a person who plead insanity as a defense, where such testimony was against the
contention of defendant. The medical expert must necessarily use the person of defendant to make
the examination.

The prohibition contained in section 5 of the Philippine Bill that a person shall not be compelled
to be a witness against himself, is simply a prohibition against legal process to extract from the
defendant's own lips, against his will, an admission of his guilt. It is not merely compulsion that
is the kernel of the privilege, but TESTIMONIAL compulsion. Else, if the inviolability also
includes physical control in whatever form exercised, it would be possible for a guilty person to
shut humself up in his house with all the indicia and tools of his crime and defy the authority of
law to employ in evidence anything that might be obtained by forcibly overthrowing his possession
of the evidential articles.

The main purpose is to prohibit compulsory oral examination of prisoners before trial, or upon
trial, for the purpose of extorting unwilling confessions or declarations implicating them in a
crime. An inspection of the bodily features cannot violate this privilege because it does not call
upon the accused as a witness or for his testimonial responsibility. Evidence obtained in this way
is not testimony by his body but his body itself. He was convicted of “abusos deshonestos.”

210. Emeteria Villaflor v. Ricardo Summers, GR 16444, September 08, 1920, Malcolm, J.
(Right Against Self-Incrimination; determine if pregnant, ocular)
FACTS:
Villaflor prays for a writ of habeas corpus. In a criminal case pending before CFI, Villaflor and
Souingco are charged with adultery. The CFI ordered Villaflor to submit her body to the
examination of doctors to determine if she was pregnant or not. She refused to obey on the ground
that such examination was a violation of the constitutional provision in contempt of court and was
ordered to be committed to Bilibid prison until she permits the examination required.

ISSUE:
Whether the compelling of Emetria Villaflor to let her body be examined by physicians to
determine if she is pregnant, in relation to a charge of adultery, violates her right against self-
incrimination.

HELD:
This Court has not the right to compel the prisoner to submit to an examination by physicians and
then have them testify on such examination. Evidence obtained thus would be inadmissible.

The prime purpose of criminal trial is to purge the community of persons who violate laws to the
prejudice of their fellow men. Criminal procedure, the rules of evidence, and constitutional
provisions, are then provided, not to protect the guilty but to protect the innocent. No accused
person should be afraid of the use of any method which will tend to establish the truth. Under the
constitution, the due process of law clause, every person has a natural and inherent right to the
possession and control of his own body. To compel any one, especially a woman, to lay bare the
body, or to submit to the touch of a stranger, without lawful authority, is an indignity, an assault,
and a trespass. But it is conceded that superior to the immunity of a person to be let alone is the
interest of the public in orderly administration of justice.

Once again, we lay down the rule that the constitutional guaranty that no person shall be compelled
in any criminal case to be a witness against himself is limited to a prohibition against compulsory
testimonial self-incrimination. The corollary to the proposition is that, on a proper showing and
under an order of the trial court, an OCULAR inspection of the body of accused is permissible.
The proviso is that torture or force shall be avoided. Whether facts fall within or without the rule
with its corollary and proviso is to be decided as cases arise. (*limited to ocular only?)

It is reasonable to presume that in an examination by disinterested physicians, due care will be


taken not to use violence and not to embarrass the patient any more than is absolutely
necessary. Thus, no objection to the physical examination being made by the family doctor of
the accused or by doctor of the same sex can be seen. The habeas corpus is denied.

211. People v. Gallarde (See case 187 for facts; photographs, mechanical act)
HELD:
We cannot agree with the trial court’s rejection of the photographs taken of Gallarde immediately
after the incident on the ground that the same were taken while Gallarde was “already under the
mercy of the police.” The taking of the pictures even without assistance of counsel, being a
purely MECHANICAL ACT, is not a violation of his right against self-incrimination. This right
proscribes the use of physical or moral compulsion to extor communications from accused and not
the inclusion of his body in evidence when it may be material. Purely mechanical acts are
NOT included in the prohibition as the accused does not thereby speak his guilt. Hence, the
assistance of counsel is not required. The essence of this right is testimonial compulsion, that is,
the giving of evidence against himself thru a TESTIMONIAL ACT. Hence, it has been held
that a woman charged with adultery may be compelled to submit to physical examination to
determine pregnancy, and an accused may be compelled to submit to physical examination and to
have a substance taken from his body for medical determination as to whether he was suffering
from gonorrhea contracted by his victim; to expel morphine from his mouth; and to be
photographed or measured, or his garments or shoes removed or replaced, or to move his body to
enable the foregoing acts to be done.

212.Social Justice Society (SJS) v. Dangerous Drugs Board, GR 157870, November 03, 2008,
Velasco, Jr., J. (Right Against Self-Incrimination)
SEC. 36. Authorized Drug Testing.—Authorized drug testing shall be done by any
government forensic laboratories or by any of the drug testing laboratories accredited and
monitored by the DOH to safeguard the quality of the test results. . . . The drug testing shall
employ, among others, two (2) testing methods, the screening test which will determine
the positive result as well as the type of drug used and the confirmatory test which will
confirm a positive screening test. . . . The following shall be subjected to undergo drug
testing:
(f) All persons charged before the prosecutor's office with a criminal offense having an
imposable penalty of imprisonment of not less than six (6) years and one (1) day shall
undergo a mandatory drug test;

HELD:
There is no valid justification for mandatory drug testing for persons accused of crimes. The
operative concepts in the mandatory drug testing are “randomness” and “suspicionless”. In case of
persons charged with a crime before the prosecutor’s office, a mandatory drug testing can never
be random or suspicionless. The ideas of randomness and being suspicionless are antithetical to
their being made defendants in a criminal complaint. They are not randomly picked nor are they
beyond suspicion. When persons suspected of committing a crime are charged, they are singled
out and are impleaded against their will. The persons thus charged, by the bare fact of being haled
before the prosecutor's office and peaceably submitting themselves to drug testing, if that be the
case, do not necessarily consent to the procedure, let alone waive their right to privacy. To impose
mandatory drug testing on the accused is a blatant attempt to harness a medical test as a tool for
criminal prosecution, contrary to the stated objectives of RA 9165. Drug testing in this case would
violate a persons' right to privacy guaranteed under Sec. 2, Art. III of the Constitution. Worse
still, the accused persons are veritably forced to incriminate themselves.

213. Jaime dela Cruz v. People, GR 200748, July 23, 2014, Sereno, C.J. (Right Against Self-
incrimination; urine, positive for drugs)
FACTS:
Jaime was charged with violation of S15 of RA 9165.

The NBI received a complaint from Corazon and Charito that Ariel, live-in partner of Corazon and
son of Charito, was picked up by several unknown male persons believed to be police officers for
allegedly selling drugs. An errand boy gave a number to them and when they called it, they were
instructed to go to Gorordo Police Office. Therein, they met Jaime who demanded P40k in
exchange for Ariel’s release. After the meeting, complainants reported to the NBI. A team was
formed to implement an entrapment operation which took place inside a Jollibee branch. Jaime
was caught with a pre-marked P500 bill, which was made part of the amount demanded.

Jaime was brought to the forensic laboratory of the NBI for forensic examination. Jaime was
required to submit his urine for drug testing. It was positive for dangerous drugs. He was
convicted by the RTC and sentenced to compulsory rehabilitation of at least 6months. The CA
affirmed.

ISSUE:
Whether Jaime, arrested for trying to extort money from Corazon and Charito in exchange for the
release of Ariel to them, can be convicted of violation of S15 of RA 9165 for being positive for
use of dangerous drugs after a confirmatory testing was done on him after he was required to
submit his urine sample after he was arrested.

HELD: NO.
The RTC said that extracting urine is merely a mechanical act, falling outside custodial
investigation.
Section 15. Use of Dangerous Drugs. — A person apprehended or arrested, who is found
to be positive for use of any dangerous drug, after a confirmatory test, shall be imposed a
penalty of a minimum of six (6) months rehabilitation in a government center for the first
offense, subject to the provisions of Article VIII of this Act. If apprehended using any
dangerous drug for the second time, he/she shall suffer the penalty of imprisonment ranging
from six (6) years and one (1) day to twelve (12) years and a fine ranging from Fifty
thousand pesos (PhP50,000.00) to Two hundred thousand pesos (PhP200,000.00):
Provided, That this Section shall not be applicable where the person tested is also found to
have in his/her possession such quantity of any dangerous drug provided for under Section
11 of this Act, in which case the provisions stated therein shall apply

The drug test in S15 does not cover persons arrested for any unlawful act, but only for unlawful
acts listed under Art.II of RA 9165. A person “apprehended or arrested” cannot literally mean
any person arrested for any crime. The phrase must be understood in consonance with RA 9165.
To make the S15 applicable to all persons arrested for any crime not listed in Art.II of RA 9165
will unduly expand its meaning. Jaime was arrested for the alleged act of extortion.

Also, to make the said phrase applicable to all persons arrested for any unlawful act is tantamount
to a mandatory drug testing of all persons arrested for any crime. As we said in SJS v.
Dangerous Drugs Board, “to impose a mandatory drug testing on accused is a blatant attempt to
harness a medical test as a tool for criminal prosecution, contrary to the objectives of RA 9165.
Drug testing in this case would violate a person’s right to privacy guaranteed under S2, Art.III of
the Constitution. Worse still, the accused persons are forced to incriminate themselves.”

The drug test is not covered by allowable non-testimonial compulsion. Cases where non-
testimonial compulsion was allowed reveal that the pieces of evidence obtained were all
MATERIAL to the principal cause of the arrest. The right against self-incrimination proscribes
the use of physical or moral compulsion to extort communications from the accused and not the
inclusion of his body in evidence when it may be material.
Here, we fail to see how a urine sample could be material to the charge of extortion. Thus, the
RTC and CA erred when they held that the extraction of urine for drug testing was merely a
mechanical act.

In Gutang v. People, petitioners were arrested in connection with a drug case. They were asked to
give urine samples, which yielded positive results. They were convicted of illegal possession and
use of prohibited drugs. The Court said that the urine samples were admissible for being a
mechanical act the accused was made to undergo not meant to unearth undisclosed facts but to
ascertain physical attributes determinable by simple observation. The circumstances therein are
different from this case as Gutang volunteered to give his urine, there were other pieces of
evidence for his conviction, and he was arrested in connection with a drug case. Here, Jaime
resisted having his urine sample taken, Jaime’s conviction was based only on the sample, and he
was arrested for extortion.

The drug test violated Jaime’s right to privacy and against self-incrimination. He refused to have
his urine taken and asked for a lawyer prior to his urine test. He was still compelled to submit his
urine. Thus, Jaime was acquitted.

214. Francisco Beltran v. Felix Samson, GR 32025, September 23, 1929, Romualdez, J. (Right
Against Self-incrimination; handwriting)
FACTS:
This is a petition for prohibition. Beltran complains that Judge Samson ordered him to appear
before the fiscal to take dictation in his own handwriting. The handwriting will be compared with
those in certain documents supposed to be falsified to determine if Beltran was the one who
falsified them.

ISSUE:
Whether compelling Beltran to write the dictation of the fiscal so that such handwriting may be
compared with that found in a falsified document to determine if Beltran was the one who falsified
the same violates his right against self-incrimination.
HELD: YES.
S1687 of the Admin Code lets the fiscal compel witnesses to be present at the investigation of any
crime. But this power must be exercised without prejudice to the rights of persons cited to appear.

The privilege not to be compelled to be a witness against himself in a criminal case extends to all
giving or furnishing of evidence. Is the writing from the fiscal’s dictation constitute evidence
against himself within the scope of the constitutional provision? It has been held that an accused,
in testifying in his own behalf, may on cross-examination be compelled to write in open court to
let the jury compare his handwriting with the one in question inasmuch as the defendant, in
offering himself as witness in his own behalf, waived his personal privileges. Of like character
is the case where the judge asked defendant to write his name during hearing, and he did so
voluntarily.

But these cases are different. We are not concerned here with a defendant for it does not appear
that any information was filed against Beltran for the supposed falsification, and still less is it a
question of a defendant testifying in trial and under cross-examination. This is only an
investigation prior to the information with a view to filing it. Beltran also does not voluntarily
want to write.

Writing is something more than moving the body, hand, or fingers. Writing is not a purely
mechanical act. It requires the application of intelligence and attention. And in this case, writing
means that Beltran is to furnish a means to determine if he is the falsifier. This case is similar
to that of producing documents of chattels in one’s possession, only more serious. In both cases,
the witness is required to furnish evidence against himself. This case is more serious because the
witness is compelled to write and create, by means of writing, evidence which does not exist,
and which may identify him as the falsifier.

It cannot be contended that if permission to obtain the handwriting is not granted, the crime would
go unpunished. Beltran is a municipal treasurer. It should not be a difficult matter for the fiscal to
obtain genuine specimens of his handwriting. Even if this was impossible, it would not be reason
for trampling upon a personal right guaranteed by the constitution.

In People v. Badilla, the defendants were not questioned by the fiscal against their will, and if they
did not refuse to answer, they must be understood to have waived their constitutional privilege.
This also holds in US v. Tan Teng, where defendant did not oppose the extraction from his body
of the substance later used as evidence against him. In Villaflor v. Summers, the case was that of
examination of the body by physicians, which was said not to compel petitioner to furnish evidence
by means of a testimonial act. She was not compelled to execute any positive act; she was only
enjoined from something, preventing the examination. This is different here, where Beltran is
sought to be compelled to perform a positive testimonial act.

215. People v. Romeo Policarpio, GR 69844, February 23, 1988, Gancayco, J. (Right Against
Self-incrimination)
FACTS:
Operatives of the Narcotics command swooped down at Bagong Bayan, Rizal, after receiving info
that Policarpio is selling and distributing marijuana. Pat Mangila served as poseur buyer. He was
introduced by the informant to Policarpio as needing marijuana. Policarpio gave the marked P20
bill. He gave the signal and the team arrested Policarpio. After the arrest, Policarpio led the
operatives to his house and the operatives seized another 6 small bags of dried marijuana and
rolling papers. Policarpio was brought to Camp Crame and the marijuana was examined, giving a
positive result for marijuana. Policarpio signed a bond paper acknowledging that the 6 bags of
marijuana were confiscated from him. He also signed a document acknowledging that the marked
P20 bill was confiscated from him.

Policarpio was convicted of violation of RA 6425. Hence this appeal.

ISSUE:
Whether receipts signed by Policarpio acknowledging that 6 small plastic bags of marijuana and a
marked P20 bill used by the poseur-buyer to buy the bags from him was confiscated from him,
where the officers informed him of his right to be silent and to counsel etc., are admissible in
evidence.
HELD: NO.
Policarpio claims that the receipt he signed acknowledging that the 6 bags of marijuana and the
marked P20 bill were confiscated from him are not admissible as they were taken in violation of
his right. The records show that he was informed of his right to be silent and that he may refuse to
give a statement which may be used against him, that is why he refused to give such written
statement unless with his lawyer as shown by the paper he signed to this effect.

But Policarpio was the victim of a clever ruse to make him sign these alleged receipts which in
effect are extra-judicial confessions. It is the police officers who confiscated the objects that
should have signed the receipts. This is a violation of Policarpio’s right to remain silent where
he was made to admit the commission of the offense without informing him of his right.

But even if the 6 marijuana bags and P20 bill are inadmissible, the records have ample evidence
other than the receipts to establish the commission of the offense.

216. Roger Chavez v. CA, GR L-29169, August 19, 1968, Sanchez, J. (Right Against Self-
incrimination; Nemo tenetur seipsum accusare)
FACTS:
Chavez saw Lee, a Chinese, driving a thunderbird car. Chavez asked Lee if his car was for sale.
Lee said yes and left his address with Chavez. Chavez met Sumilang and informed him about the
car. But Sumilang said that he had changed his mind about buying a new car. Instead, he told
Chavez that he wanted to mortgage his Buick car for P10k to cover a debt in Pasay. They went to
Asistio who told them that he had a better idea on how to raise the money. His plan was to capitalize
on Sumilang’s reputation as a wealthy movie star, introduce him as buyer to someone selling a
car, and after the deed of sale is signed, by trickery to run away with the car. Asistio would then
register it, sell it to a third person for profit. Chavez furnished the name of Lee.

Chavez and Sumilang met Lee in his Thunderbird car. Sumilang was introduced as the interested
buyer. After Sumilang and Lee agreed on the price of P21k, they went to a notary public to draft
the deed of sale. It was signed by Sumilang as vendee, Dy Sun Hiok (registered owner-cousin of
Lee) as vendor, and Sumilang’s driver and Lee as witnesses.

They met at Eugene’s restaurant for payment. Sumilang left the table, never to return. Chavez also
disappeared after he left on the pretext of buying cigarettes. The two Chinese could not find
Sumilang and Chavez. They went to where the Thunderbird was parked and found that it was gone.
Sumilang and Chavez, after the meeting at Eugene’s, registered the car in Asistio’s name. They
reported to the NBI, who recovered the already repainted car and impounded it.

Sumilang testifies that he borrowed money from various people to pay for the car. Thereafter,
Asistio saw his car, liked it, and bought it. But before he could be paid in full, the car was
impounded.

The fiscal presented Chavez as “ordinary witness”, not as state witness. His counsel objected
and was given 15 minutes to confer with Chavez. Chavez said he will not testify. But the judge
said that the prosecution has the right to ask anybody to act as witness. Thus, trial began with the
“direct examination” of Chavez by “Fiscal Grecia.”
The CFI gave credence to Sumilang’s averment, strengthened by the loaners’ corroborations. So
Sumilang was cleared. Asistio was cleared whom the trial court believed to be a mere buyer of the
car. But as to Roger Chavez, the trial court said that he “does not offer any defense. His testimony
as witness for the prosecution establishes his guilt beyond reasonable doubt.” The trial court
branded him a “self-confessed culprit.” If Lee was not paid, the trial court said that he only had
Chavez to blame as agent of Sumilang. So it convicted Chavez of qualified theft.

The CA dismissed the appeal for non-filing of appellant’s brief despite lapse of time to file. Hence
this petition for habeas corpus. Chavez claims that he was compelled to testify against himself.

ISSUE:
Whether the prosecution may compel Chavez to testify on behalf of the People when he is one of
the accused in the criminal case and to have such testimony admitted in evidence to convict
Chavez.
HELD: NO.
A court may not resort to compulsory disclosure of facts usable against an accused as a confession
of the crime. Compulsion as understood here does not necessarily connote use of violence. It may
be the product of unintentional statements. Pressure which operates to overbear his will,
disable him from making a rational and free choice, or impair his capacity for rational judgment,
would be sufficient. So is moral coercion “tending to force testimony from the unwilling lips of
the defendant.”

Chavez is a defendant in a criminal case. He was called by the prosecution as the first witness in
that case to testify for the people during the first day of trial. Chavez objected and invoked the
privilege of self-incrimination. He stated that he will not testify. But the judge stated that “it is
the right of the prosecution to ask anybody to act as witness on the witness-stand including the
accused,” and that defense counsel “could not object to have the accused called on the witness
stand.” Thus, Chavez was required to take the stand and asked to create evidence against himself.

Chavez, as accused, occupies a different tier of protection from an ordinary witness. Whereas an
ordinary witness may be compelled to take the witness stand and claim the privilege as each
question requiring an incriminating answer is shot at him, an accused may altogether refuse
to take the witness stand and refuse to answer any and all questions. For, in reality, the purpose
of calling an accused as witness for the people would be to incriminate him. This rule may apply
even to a co-defendant in a joint trial. The guide in interpreting the constitutional precept is not
the probability of the evidence, but it is the capability of abuse.

When the judge said “But surely, counsel could not object to have the accused called on the
witness-stand”, Chavez was enveloped by a coercive force depriving him of his will to resist. As
he took his oath to tell the truth and nothing but the truth, no genuine consent underlay submission
to take the witness stand. Constitutionally sound consent was absent.

Thus, Chavez was prejudiced. The record shows that by leading questions, Chavez was made to
affirm his statement given to the NBI agents detailing his plan and its execution by Sumilang,
Asistio, and himself to deprive Lee of his car. And he himself narrated the same in open court. The
trial court even said that his testimony for the prosecution proved his guilt beyond reasonable doubt
and that he was a self-confessed culprit.

Chavez was forced to testify to incriminate himself, violating his right to remain silent. He did
not waive his right. He did not volunteer to take the stand in his own defense. Contrarily, he
claimed the right upon being called to testify. If Chavez answered the questions in spite of his fear
of being accused of perjury or being put in contempt, this cannot be counted against him. His
testimony is not of his own choice. It does not matter that after taking the stand, no objections to
questions asked to Chavez were made. What is involved here is not merely self-incrimination,
but a defendant’s immunity from being called to testify against himself. And the objection he
made at the beginning is a continuing one. There is thus no waiver. To be effective, a waiver must
be certain, unequivocally, intelligently, understandably and willingly made.

A void judgment is in legal effect no judgment. The grant of habeas corpus to Chavez whose case
present a clear disregard of constitutional right is absolutely proper. But since he is serving for a
conviction of another offense, we take the position that he is entitled to liberty only with respect
to Criminal Case 5311.

217. Alfredo Rosete v. Juliano Lim, GR 136051, June 08, 2006, Chico-Nazario, J. (Right
Against Self-incrimination; Civil case with same allegations as criminal case)
FACTS:
Juliano and Lilia Lim filed with RTC a complaint for annulment, specific performance with
damages against the AFP Retirement and Separation Benefits System (AFP-RSBS) etc. It asked
that the deed of sale executed by AFP-RSBS covering lands in favor of Espreme Realty and the
titles thereof under Espreme’s name be annulled, and that AFP-RSBS and Espreme be ordered to
execute the necessary documents to restore ownership of the lands to Juliano and Lila Lim.

Petitioners moved to dismiss on the ground that the court has no jurisdiction. This was denied.
respondents filed a notice to take deposition upon oral examination, giving notice that they will
cause the deposition of petitioners Oscar Mapalo and Chito Rosete. Petitioners objected, which
objection was denied. The RTC scheduled the taking of the deposition. The RTC declared Mapalo
and Chito Rosete in default for continued refusal to be sworn.

The CA, on certiorari, denied the petition questioning the denial of the objection to the order of
taking oral deposition. Hence this petition for certiorari.

ISSUE:
Whether parties in a civil case for annulment, specific performance with damages relating to a
deed of sale may refuse to take the witness stand and give oral deposition on the ground that 2
criminal cases are pending against them in another court with the same allegations and issues.
HELD: NO.
The CA held that the trial court did not abuse its discretion when it refused to recognize Malapo
and Chito Rosete’s right against self-incrimination when it allowed the taking of their oral
depositions. They claim that they refused the oral depositions due to two pending criminal cases
against them for BP22 and estafa because their answers would expose them to criminal liability.
They claim that the questions during deposition would revolve around the allegations in the
complaint in the civil case which are identical to the allegations in the complaint-affidavits in the
two criminal cases, thus there is tendency to incriminate them.

The right against self-incrimination is accorded to every person who gives evidence, whether
voluntary or under compulsion of subpoena, in any civil, criminal, or administrative proceeding.
The right is not to be compelled to be a witness against himself. The right can be claimed only
when the specific question, incriminatory in character, is actually put to the witness. It does
not give the witness the right to disregard a subpoena, to decline to appear before the court at the
time appointed, or to refuse to testify altogether. As to an accused in a criminal case, it is settled
that he can refuse outright to take the stand as a witness.

Unlike an ordinary witness (or a party in a civil action), who may be compelled to testify by
subpoena, having only the right to refuse to answer a particular incriminatory question at the
time it is put to him, the defendant in a criminal action can refuse to testify altogether. Only an
accused in a criminal case can refuse to take the witness stand. The right to refuse to take the
stand does not generally apply to parties in administrative cases or proceedings. The parties
thereto can only refuse to answer if incriminating questions are propounded. The exception-
a party not an accused in a criminal case is allowed not to take the witness stand- was applied in
administrative and civil cases that partook of the nature of a criminal proceeding or analogous
to a criminal proceeding. As long as the suit is criminal in nature, the party can altogether
decline to take the witness stand. It is not the character of the suit but the nature of the proceedings
that controls.

Here, the case is civil, being a suit for annulment, specific performance with damages. For
petitioners to exercise the right to refuse to take the witness stand and give their depositions, the
case must partake of the nature of a criminal proceeding. This case cannot be categorized as such.
The fact that there are two criminal cases pending allegedly based on the same set of facts as
the civil case will not give them the right to refuse to give their depositions. They are not facing
criminal charges in the civil case. Like an ordinary witness, they can invoke the right against
self-incrimination only when the incriminating question is actually asked of them.

218. Arsenio Pascual, Jr. v. Board of Medical Examiners, GR L-25018, May 26, 1969,
Fernando, J. (Right Against Self-incrimination; administrative case)
FACTS:
Pascual filed with CFI an action for prohibition against the Board of Medical Examiners. It was
alleged that at the initial hearing of an administrative case for alleged immorality, counsel for
complainants announced that he would present as his first witness Pascual, who was the respondent
in the malpractice charge. Pascual objected, relying on the right from being a witness against
himself. The Board noted the objection, stating that unless he could get a restraining order from a
competent authority, he would be called upon to testify as such witness. The CFI issued a writ of
preliminary injunction against the Board.

The Board claims that the remedy is to object once he is in the witness stand. The CFI ruled in
favor of Pascual.

ISSUE:
Whether the Board of Medical Examiners hearing the administrative case against Pascual for
alleged immorality in which he is at risk of being penalized with revocation of medical license and
the like may compel Pascual to take the witness stand.
HELD: NO.
In Cabal v. Kapunan, there was an administrative charge for unexplained wealth against petitioner
under the Anti-Graft Act to be penalized by forfeiture of whatever property a public officer may
acquire manifestly out of proportion to his salary. It was said that there was a penalty although the
case was administrative in character. Thus, the petitioner could not be compelled to take the
witness stand. This case is similar as Pascual could suffer revocation of his license as medical
practitioner.

This appeal proceeds on the Board’s mistaken assumption that the guarantee against self-
incrimination should be limited to allowing a witness to object to questions the answers to which
could lead to penal liability. But the constitutional guarantee protects as well the right to silence
and his silence cannot be used as a presumption of his guilt. Thus, we hold that in an
administrative hearing against a medical practitioner for alleged malpractice, respondent Board
cannot, consistently with the self-incrimination clause, compel the person proceeded against to
take the witness stand without his consent.

219. Manuel Cabal v. Hon. Ruperto Kapunan, GR L-19052, December 29, 1962, Concepcion,
J. (Right Against Self-incrimination)
FACTS:
Col. Maristela of the PH Army filed with the Sec. of National Defense a letter complaint charging
Cabal, then Chief of Staff of AFP, with graft, corrupt practices, etc. The PH president created a
committee to investigate the charge of unexplained wealth. At the beginning of the investigation,
the committee, upon request of Col. Maristela, ordered Cabal to take the witness stand and be
sworn to as witness for Maristela in support of his charge of unexplained wealth. Cabal objected,
invoking right against self-incrimination. The committee insisted that Cabal take the witness stand
and be sworn to, subject to his right to refuse to answer such questions as may be incriminatory.
Cabal still refused. The committee referred the matter to the city fiscal for such action as may be
deemed proper. The fiscal charged Cabal of contempt of the presidential committee.

Cabal moved to quash the charge. Judge Kapunan denied the motion to quash. Hence this petition.
Respondents allege that the investigation is administrative, not criminal, in nature.

ISSUE:
Whether Cabal, accused of corrupt practices punishable by forfeiture of the properties manifestly
out of proportion to his salary as a public officer, may be compelled to take the witness stand for
complainant Col. Maristela.
HELD: NO.
It is not disputed that the accused in a criminal case may refuse not only to answer incriminatory
questions, but also to take the witness stand. Are the committee proceedings civil or criminal? The
purpose of the charge against Cabal is to apply RA 1379, Anti-Graft Law, authorizing forfeiture
to the state of property of a public officer which is manifestly out of proportion to his salary as
such. The forfeiture has been held to partake of the nature of a penalty. It is a method deemed
necessary by the legislature to restrain the commission of an offense and to aid in the prevention
of such an offense. As a consequence, proceedings for forfeiture of property are deemed criminal
or penal, thus the exemption of defendants in criminal case from the obligation to be witnesses
against themselves are applicable.

Informations for forfeiture, generally, that seek no judgment of fine or imprisonment are deemed
to be civil proceedings in rem. Such are criminal in nature to the extent that where the person
using the res illegally is the owner of it, the forfeiture proceeding is in the nature of a punishment.
It has been held that a proceeding to declare forfeiture of certain property, though technically a
civil proceeding, is in substance and effect a criminal one. Similarly, a proceeding for the
removal of an officer was held to be in substance criminal. Thus, the constitutional provision
applies whenever the proceeding is not “purely remedial” or intended as a redress for a private
grievance, but primarily to punish a violation of duty or a public wrong and to deter others
from offending in like manner.

Sec.18- RIGHT AGAINST INVOLUNTARY


SERVITUDE
220. In the Matter of the Petition for Habeas Corpus. Segifredo Aclaracion v. Hon.
Gatmaitan, GR L-39115, May 26, 1975, Aquino, J. (Right Against Involuntary Servitude)
FACTS:
Aclaracion was a temporary stenographer in the Gapan branch of the CFI. His appointment expired
on Nov. 21, 1972. After, he was employed as stenographer in the Public Assistance and Claims
Adjudication Division of the Insurance Commission, where he is now working. After he ceased to
be a court stenographer, the CA required him to transcribe his stenographic notes in two cases
decided by the Gapan court which had been appealed. He failed to comply and was declared in
contempt of court. He was thus arrested and incarcerated in the municipal jail. He asked the CA
that he be not required to transcribe his notes in all the cases tried in the Gapan court, suggesting
that the testimonies in the said cases be retaken.

Aclaracion filed with this Court a petition for habeas corpus, saying that to compel him to
transcribe his stenographic notes, after he ceased to be a stenographer, would violate the rule
that “no involuntary servitude in any form shall exist except as a punishment for a crime whereof
the party shall have been duly convicted.” (S14, Art.IV, BoR, 1972 Constitution). He was ordered
by the SC released on the condition that he complete the stenographic notes within 20 days. he
completed the notes.

ISSUE:
Whether a former court stenographer of the CFI, now working in the Insurance Commission, may
be compelled by the CA to transcribe his stenographic notes in the CFI cases appealed to the CA
which he handled while stenographer of CFI.
HELD: YES.
The habeas corpus petition has become moot in view of his release from jail during the pendency
of his case. We hold that an appellate court may compel a former court stenographer to transcribe
his stenographic notes. That prerogative is incidental to its appellate jurisdiction and is part of its
inherent powers which are necessary to the ordinary and efficient exercise of its jurisdiction
and essential to the due administration of justice. The provision of S12, Rule 41 of the RoC
that “upon approval of the record on appeal the clerk shall direct the stenographer to attach to the
case record 5 copies of the transcript” includes stenographers who are no longer in the judiciary.

The traditional mode of exercising the court’s coercive power is to hold the recalcitrant
stenographer in contempt of court if he does not comply with the order for the transcription of his
notes and imprison him until he obeys the order. Another sanction is to hold in abeyance the
transfer, promotion, resignation, or clearance of a stenographer until he completes the
transcription.

Here, Aclaracion transcribed his notes while he was an employee of the Insurance Commission.
During the time he made the transcription, he received his salary as such employee. We hold that
he could be required to transcribe his notes in other cases, particularly in CA-GR 49687 which
the CA required him to transcribe. The CA also fined Aclaracion in CA-GR 51330 for his failure
to transcribe his notes and warned him that he would be arrested if he failed to submit the transcript
within 10 days from notice. The same arrangement should be made by the clerk of court of the SC
with the Insurance Commissioner that Aclaracion should be allowed to receive his salary while
making the transcription.

The contention on involuntary servitude is not tenable. Involuntary servitude denotes a condition
of enforced, compulsory service of one to another or the condition of one who is compelled
by force, coercion, or imprisonment, and against his will, to labor for another, whether he is
paid or not. This is not the case here.

Sec.19- PROHIBITED PUNISHMENT


221. Leo Echegaray v. SOJ, GR 132601, October 12, 1998, Per Curiam (Prohibited
Punishment)
(See case 140 for facts)
ISSUE:
Whether death penalty by lethal injection is cruel, degrading, and inhuman punishment and thus
unconstitutional.
HELD: NO.
Lethal injection is not cruel, degrading, or inhuman punishment under S19, Art.III of the 1987
Constitution. The 1935 Constitution reads: “Excessive fines shall not be imposed, nor cruel and
inhuman punishment inflicted.” In the 1973 Constitution, the phrase became “cruel or unusual
punishment.” the 1986 concom read the 1973 modification as prohibiting unusual punishment even
if not cruel. It was thus seen as an obstacle to experimentation in penology. Thus, the present text
prohibits “cruel, degrading, or inhuman punishment.”

It is contended that death by lethal injection is cruel, degrading, and inhuman punishment because
of possible botched executions and the uncertainties as to the date of execution, time of
notification, etc. which cause the greatest suffering for the convict.

Death penalty per se is not cruel, degrading, or inhuman punishment. Punishments are cruel when
they involve torture or a lingering death. It implies something inhuman and barbarous,
something more than the mere extinguishment of life. The lack of particularity as to the details
in the execution does not render RA 8177 cruel, degrading, or inhuman. The implementing details
of RA 8177 are matters properly left to the expertise of administrative officials.

As petitioner already knows, the “court” which designates the date of execution is the trial court
which convicted the accused after the SC has reviewed the entire records of the case and affirmed
the judgment of the lower court. Thereafter, judgment is entered 15 days after its promulgation,
and 10 days after the records are remanded to the court below with a certified copy of the judgment
for execution. There is no uncertainty as to the date of execution or time of notification. S15 of the
IRR, read in conjunction with the last sentence of S1, RA 8177, provides that the death sentence
shall be carried out not earlier than 1 year nor later than 18 months after judgment has become
final. Thus, the convict is assured of 18 months from the judgment’s finality wherein he can seek
executive clemency and attend to all his spiritual affairs.

Echegaray contends that the possible “wanton pain” in cases of complications renders the lethal
injection a cruel, degrading, and inhuman punishment. but this is highly speculative. S1 of RA
8177 requires that all personnel involved in the execution should be trained prior to the
performance of the task. We must presume that the public officials entrusted with implementing
death penalty will carefully avoid inflicting cruel punishment. Also, any infliction of pain in lethal
injection is merely incidental in carrying out the execution. Since punishment imports pain or
suffering to the convict, it may be said that all punishments are cruel. But the constitution does not
mean that crime, for this reason, is to go unpunished. The cruelty against which the constitution
protects a convicted man is cruelty inherent in the method of punishment, not the necessary
suffering involved in any method employed to extinguish life humanely.

What is cruel and unusual must draw its meaning from the evolving standards of decency that
mark the progress of a maturing society. The primary indicator of society’s standard of decency
as to capital punishment is the response of the country’s legislatures to the sanction. Hence, for as
long as death penalty remains in our statute books and meets the most stringent requirements of
the constitution, we must confine our inquiry to the legality of RA 8177 whose constitutionality
we sustain.

Death penalty does not violate international treaty obligations. Art.6 of ICCPR enshrines the
individual’s right to life. Art.6(2) recognizes that capital punishment is an allowable limitation on
the right to life, only subject to the limitation that it be imposed for the “most serious crimes.”
State parties are not obliged to abolish the death penalty, but only to limit its use and to abolish
it for other than the most serious crimes. The second optional protocol to the ICCPR aiming at the
abolition of the death penalty was not signed nor ratified by PH.

Sec.20- NON-IMPRISONMENT for DEBT


222. Florentina Lozano v. Hon. Martinez, GR L-63419, December 18, 1986, Yap, J. (Non-
imprisonment for Debt)
FACTS:
The constitutionality of BP22 is at issue. These petitions arose from cases involving prosecution
of offenses under BP22. The defendants moved to quash information on the ground that the acts
charged do not constitute an offense, the statute being unconstitutional. In one case, the trial court
declared the law unconstitutional and dismissed the case. The parties adversely affected appealed
to the SC.

ISSUE:
Whether BP22, punishing the making or drawing and issuing of a worthless check that is
subsequently dishonored, violates the prohibition against imprisonment for debt.
HELD: NO.
BP22 is aimed at putting a stop to the practice of issuing worthless checks because of the injury
caused to the public interests. Since estafa under Art.315 2d did not punish checks issued for
payment of a pre-existing obligation, the then interim Batasan took a bold step and enacted BP22.
Those who question BP22 claim that it violates the prohibition for imprisonment for debt. It is
claimed that BP22 is a veiled device to coerce payment of a debt under threat of penal sanction
since the offense therein is consummated only upon the dishonor of the check.

Historically, the constitutional prohibition evolved during the early part of the 19th century in
various states in the American Union as a result of the people’s revulsion at the cruel practice,
sanctioned by common law, permitting creditors to cause the incarceration of debtors who could
not pay their debts thru a writ of capias ad satisfaciendum. Later, this provision was transported
to PH by the Americans at the turn of the century. Later, our constitution also outlawed not only
imprisonment for debt, but also the infamous practice native to PH of jailing people for non-
payment of the cedula or poll tax.

Justice Malcolm in Ganaway v. Quillen stated that the “debt” intended to be covered by the
prohibition apples to liabilities arising from actions ex contractu. The prohibition was never meant
to include damages arising in actions ex delicto, since damages recoverable therein are imposed
for the wrong the defendant has done and are considered as punishment, nor to fines and penalties
imposed by the courts in criminal proceedings as punishments for crime.

In People v. Vera Reyes, a statute punishing the refusal of an employer to pay, when he can do so,
the salaries of his employees was upheld by the Court, it being within the authority of the
legislature to enact such a law in the exercise of the police power. It was held that one of the
purpose of the law was to suppress possible abuses of the employers who hire employees without
paying salaries. The law was viewed not as a measure to coerce payment of an obligation, although
such could be its effect, but to banish a practice considered harmful to public welfare.

Here, the gravamen of the offense punished by BP22 is the act of making and issuing a worthless
check or a check that is dishonored or putting such into circulation because of its deleterious
effects on the public interest. It is not the non-payment of an obligation which is punished.
The law is not intended to coerce a debtor to pay his debt. The law punishes the act not as an
offense against property, but an offense against public order.

It may be unconstitutional for the legislature to penalize a person for non-payment of a debt ex
contractu, but it is within their prerogative to proscribe certain acts deemed inimical to public
welfare. The state can punish not only mala in se, but also malum prohibitum, in the exercise of
its police power. The enactment of BP22 is a declaration by the legislature that, as a matter of
public policy, the making and issuance of a worthless check is deemed a public nuisance to be
abated by penal sanctions. It is not for us to question the wisdom of the law. It is sufficient that a
reasonable nexus exists between means and end.

It is not difficult to understand the public concern prompting its enactment. The approximate value
of bouncing checks per day was close to P200M. the effects of issuing a worthless check transcends
the private interests of the parties involved in the transaction and touches the interests of the public.
The harmful practice of putting worthless commercial papers in circulation can pollute the
channels of trade and commerce and injure the banking system. Thus, BP22 is a valid exercise of
police power and does not violate the prohibition of imprisonment for debt.

223. People v. Hon. Nitafan, GR 81559-60, April 06, 1992, Gutierrez, Jr., J. (Non-
Imprisonment for Debt)
FACTS:
Petitioner and private petitioner Allied Banking Corp. raise the issue of whether an entrustee in a
trust receipt agreement who fails to deliver the proceeds of the sale or to return the goods if not
sold to the entruster-bank is liable for estafa.

Allied charged Betty Ang with estafa. It alleged that Betty received in trust from Allied plastics
worth P398k under the obligation to sell it and to account for the proceeds of the sale if sold and
to return them if unsold or upon demand. But Betty, notwithstanding demands, paid only
P283,115.78, leaving unaccounted P114,884.22 which she misappropriated. Betty moved to quash
on the ground that the facts do not constitute an offense. Judge Nitafan granted the motion to quash.
Betty claims that PD 115, Trust Receipts Law, violates non-imprisonment for debt.

ISSUE:
Whether PD115, penalizing the failure of an entrustee of goods to turn over the proceeds of the
sale of the goods covered by a trust receipt or to return the goods if unsold violates the prohibition
against imprisonment for debt.
HELD: NO.
PD115, S13 provides: The failure of an entrustee to turn over the proceeds of the sale of the goods,
documents or instruments covered by a trust receipt to the extent of the amount owing to the
entruster or as appears in the trust receipt or to return said goods, documents or instruments if they
were not sold or disposed of in accordance with the terms of the trust receipt shall constitute the
crime of estafa, punishable under the provisions of Article 315, paragraph 1(b) of xxx the Revised
Penal Code. xxx

The violation here was committed during the effectivity of PD115. The failure to account for the
P114k balance is what makes Betty criminally liable for estafa. A trust receipt arrangement does
not involve a simple loan transaction between creditor and debtor-importer. Apart from the loan
feature, the trust receipt arrangement has a security feature covered by the trust receipt itself. This
is what provides the much needed financial assistance to our traders in the importation or purchase
of goods thru the use of those goods as collateral for the advancements made a bank. The title of
the bank to the security is the one sought to be protected and not the loan which is a separate
and distinct agreement.
PD115 punishes the dishonesty and abuse of confidence in the handling of money or goods to
the prejudice of another regardless if the latter is the owner or not. The law does not seek to
enforce payment of the loan. There is no violation of non-imprisonment for debt. PD 115, like
BP22, punishes the act not as an offense against property, but as an offense against public
order. It is in the context of upholding public interest that the law penalizes a breach of a trust
receipt agreement.

The offense is punished as malum prohibitum. A mere failure to deliver the proceeds or the goods
if unsold constitutes a crime that causes prejudice not only to another, but more to public interest.
Trust receipts are indispensable contracts in international and domestic business transactions.

Sec.21- DOUBLE JEOPARDY


224. Wilfred Chiok v. People, GR 179814, December 07, 2015, Jardeleza, J. (Double
Jeopardy)
FACTS:
According to the prosecution, petitioner Chua met Chiok, during which Chiok offered to be her
investment adviser. Convinced by Chiok’s representations and the fact that he is Chinese, Chua
made an initial investment of P200k. She did not receive any document evidencing the stock
transactions and relied only on the assurances of Chiok. In 1995, she accepted his proposal to buy
shares in bulk for P9.5M. Chua became suspicious when Chiok later avoided her calls and when
he failed to show any document of the sale. Chiok gave Chua 2 checks totaling P9.5M which were
dishonored.

Chiok was charged with estafa under Art.315 1b of the RPC. The RTC convicted Chiok of estafa.
The MR was denied. The RTC also cancelled his bail. On appeal, the CA reversed and acquitted
Chiok for failure of the prosecution to prove his guilt beyond reasonable doubt. The RTC
findings, CA said, did not contain findings of fact but merely recited the prosecution’s evidence
as if such was already proof of the facts constituting estafa. The RTC relied on the weakness of
the defense. The CA ruled that only civil liability for P9.5M must be paid.

The OSG did not file an MR on the ground of double jeopardy. Chua filed an MR. The CA denied
the MR on the ground that acquittal is immediately final and re-examination of the case record
would violate double jeopardy. Hence this certiorari petition.

ISSUE:
Whether Chua may appeal the acquittal by the CA of Chiok whom she charged with estafa and
BP22 on the ground that there were tamperings in the evidence and that evidence of Chiok’s offer
of settlement was secretly removed.
HELD: NO.
Chua lacks the legal standing to question the CA decision because it is only the OSG, on behalf
of the state, which can bring actions in criminal proceedings before the CA and SC. The private
complainant may question the acquittal or dismissal only insofar as the civil liability of accused
is concerned. But even if we disregard this procedural rule, the merits still call for dismissal.

The appeal from the judgment of acquittal will place Chiok in double jeopardy. S7, Rule 117 of
the 1985 and 2000 Rules on Criminal Procedure provide for the requisites for double jeopardy to
attach: 1) a valid information sufficient in form and substance to sustain conviction of the crime
charged, 2) a court of competent jurisdiction, 3) the accused has been arraigned and had
pleaded, and 4) the accused was convicted or acquitted or the case was dismissed without his
express consent.

To give life to the rule on double jeopardy, our rules on criminal proceedings require that a
judgment of acquittal, whether ordered by the trial or appellate court, is final, unappealable, and
immediately executory upon its promulgation. This is the “finality-of-acquittal” rule. The
rationale is that the state, with all its resources and power, should not be allowed to make
repeated attempts to convict an individual for an alleged offense, subjecting him to
embarrassment, expense, and ordeal and compelling him to live in a continuing state of anxiety as
well as enhancing the possibility that even though innocent, he may be found guilty. There is a
need for “repose”, a desire to know the exact extent of one’s liability. Because the innocence of
the accused has been confined by a final judgment, the Constitution conclusively presumes that a
second trial would be unfair.

There were cases, however, where we recognized certain exceptions to the finality-of-acquittal
and the rule against double jeopardy. In Galman v. Sandiganbayan, we remanded a judgment of
acquittal due to mistrial as the prosecution was denied due process due to the pervasive
monitoring by the authoritarian president Marcos to carry out his instructions. In People v. Uy, we
held that an acquittal in a criminal case may be assailed by certiorari under Rule 65 of the RoC
upon clear showing that the lower court, in acquitting the accused, committed not merely reversible
errors of judgment but GADALEJ or a denial of due process, rendering the judgment void.

Here, the exceptions do not apply. Chua claims that 1) the CA proceeding is a mock proceeding,
2) the people, thru OSG, was deprived of opportunity to be heard, and 3) the result is a void
judgment of acquittal. She claims that a transcript of stenographic records, where Chiok made an
implied admission of guilt by an offer of settlement, was secretly removed and there were
tamperings in the evidence.

But the factual milieu of Galman is starkly different. There was there an overwhelming evidence
of collusion and undue pressure by Marcos on the prosecution and justices. Here, Chua presents a
report by Judge Panganiban showing irregularities in the BP 22 case against Chiok. We do not see
the same evils in Galman when the alleged anomalies pointed out by Chua were in a different case
and when the main basis of acquittal is not on the credibility of the physical evidence but on the
testimony of Chua herself. Also, the CA considered the settlement offer. The state was not
deprived of due process as the OSG actively participated in prosecuting the case before the CA.
Thus, there is no grave abuse of discretion.

225. People v. Hon. Laguio, GR 128587, March 16, 2007, Garcia, J. (Double Jeopardy)
FACTS:
Police arrested SPO2 Vergel, Rogelio, and Arellano for unlawful possession of shabu. In the
investigation, Frank and Joseph were identified as the drug’s source. An entrapment operation was
set after the three were prevailed upon to call their source and pretend to order another supply of
shabu. Redentor and Joseph were arrested while they were about to hand over another bag of shabu
to SPO2 Vergel et al. Redentor and Joseph said that they worked as talent manager and gymnast
instructor respectively of Glamour Modeling Agency owned by Lawrence Wang. The police
surveilled Maria Orosa apartment where Wang was said to be.

Officers approached Wang when he came out of the apartment towards his parked BMW. When
he answered that he was Wang, the officers frisked him and asked him to open the back of the
BMW car. The officers found an unlicenced pistol with ammunitions. Shabu was found in the
BMW and another unlicenced pistol. Wang resisted the warrantless arrest and search. 3
informations were filed against Lawrence Wang for violation of the Dangerous Drugs Act, Illegal
Possession of Firearms, and Violation of Comelec Gun Ban.

Wang filed a demurrer to evidence, praying for acquittal and dismissal of the 3 cases for lack of a
valid arrest and search warrant and the inadmissibility of the prosecution’s evidence against him.
Judge Laguio granted the demurrer to evidence, acquitting Wang. Hence this petition for
certiorari by the people.

ISSUE:
Whether the dismissal of a criminal case by the trial court on demurrer to evidence on the ground
of inadmissibility of evidence of the prosecution due to a search incidental to unlawful arrest is
reviewable by the SC on appeal by way of petition for review on certiorari.
HELD: NO.
This case is an appeal filed directly with SC via certiorari. The right to appeal is neither a natural
right nor part of due process, being merely a statutory privilege which may be exercised only in
the manner provided by law. The right to appeal in S2, Rule 122 is subject to the prohibition against
putting accused in double jeopardy. In effect, S2, Rule 122 disallows appeal by the People from
judgments of acquittal. An order granting demurrer to evidence is a resolution on the merits and
amounts to an acquittal. Generally, any further prosecution of accused after acquittal would
violate the right to double jeopardy. But there are exceptions.

Galman v. Sandiganbayan- when the prosecution is denied due process of law.


People v. Uy- Another is when the trial court commits grave abuse of discretion in dismissing a
criminal case by granting the accused’s demurrer to evidence. An acquittal in a criminal case may
be assailed thru certiorari upon a clear showing that the lower court, in acquitting the accused,
committed not merely reversible errors of judgment, but GADALEJ, rendering the assailed
judgment void.

In resolving demurrer to evidence, the court is merely required to ascertain whether there is
sufficient evidence to sustain a verdict of guilt. Its grant or denial is left to the sound discretion
of the trial court and its ruling shall not be disturbed in the absence of grave abuse of discretion.
Even an appeal based on an alleged misappreciation of evidence will not lie. The only instance
when double jeopardy will not attach is when the trial court acted with GADALEJ. But while
certiorari may be availed of to correct an erroneous acquittal, the petitioner in such an extraordinary
proceeding must clearly demonstrate that the trial court blatantly abused its authority to a point so
grave as to deprive it of its very power to dispense justice.
It is settled that the appellate court may review dismissals of trial courts granting demurrers to
evidence thru CERTIORARI under Rule 65 based on gadalej. Such dismissal order, being
considered a void judgment, does not result in jeopardy.

Unfortunately, what the People thru SOJ and SOLGEN filed here is an appeal by way of petition
for review on certiorari under Rule 45 raising a pure question of law, which is DIFFERENT
from a petition for certiorari under Rule 65. The following are the distinctions between appeal
and certiorari:
PURPOSE: Certiorari is designed to correct errors of jurisdiction, not errors of judgment.
Even if the findings of the court are incorrect, as long as it has jurisdiction over the case, such
correction is normally beyond the province of certiorari. If the error is not of jurisdiction but an
error of law or fact- a mistake of judgment- appeal is the remedy.
MANNER OF FILING: Appeal- the CA exercises its appellate jurisdiction and power of review.
Certiorari- the higher court uses its original jurisdiction in accordance with its power of control
and supervision over the proceedings of lower courts. An appeal is a continuation of the original
suit, while certiorari is an original and independent action. The parties to appeal are the original
parties in the action while the parties in certiorari are the aggrieved party, who becomes petitioner,
and the prevailing parties, respondents.
SUBJECT MATTER: Only judgments or final orders and those that the RoC so declares are
appealable. Certiorari- may be directed against an interlocutory order prior to appeal since the
issue is jurisdiction.
PERIOD OF FILING: Appeal- 15 days from notice of judgment or final order appealed from
Appeal by Certiorari-15 days from notice of judgment or final order also, or denial of MR or
motion for new trial.
Certiorari- 60 days from notice of judgment, order, or resolution.
NEED FOR MR: Certiorari- MR is generally required before filing certiorari. Appeal- not
required before appealing a judgment or final order.

Where appeal is available, certiorari will not prosper. In dismissal of a criminal case upon demurrer
to evidence, appeal is not available as such an appeal will put the accused in double jeopardy.
Certiorari is however allowed. For being the wrong remedy by the People here, the petition is
outrightly dismissible. The SC cannot reverse the decision of the trial court by appeal without
violating Wang’s right against double jeopardy. But even assuming that the Court may treat an
“appeal” as a special civil action of certiorari, which it can when there is clear showing of grave
abuse, the petition still fails.

Finding that the warrantless arrest preceded the warrantless search, the trial court granted Wang’s
demurrer and acquitted him because of the unlawful arrest resulting in the inadmissibility of the
evidence. It found that none of the circumstances under Rule 113, S5 was present for a valid
warrantless arrest as Wang was merely walking from his apartment and was about to enter the
parked BMW car. He was not committing any offense etc. The officers had no info that Wang was
carrying an unlicenced handgun nor did they see him in possession thereof before the arrest. Thus,
the trial court found that since there was no valid warrantless arrest, the incidental search was also
unlawful. All evidence acquired as a consequence are inadmissible. The trial court thus dismissed
the case for lack of evidence.
The People contend that the warrantless search preceded the warrantless arrest. But the conflicting
versions as to whether the arrest preceded the search or vice versa is a matter of credibility of
evidence. It entails appreciation of evidence, which may be done in an appeal of a criminal case
because the entire case is thrown open for review, but not in the case of a petition for
certiorari where the factual findings of the trial court are binding upon the Court. Since a
dismissal order due to demurrer is not subject to appeal and reviewable only by certiorari, the
factual finding that the arrest preceded the search is conclusive upon this Court. The only legal
basis for this Court to reverse the dismissal upon demurrer would be if the trial court committed
GADALEJ when it ruled that there was no legal basis to effect a warrantless arrest.

But since Wang was merely walking from his apartment to the parked BMW and did not manifest
any suspicious behavior, there is no valid warrantless arrest in flagrante delicto under Rule 113,
S5(a). Reliable info alone without any overt act indicative of a felonious enterprise in the presence
of the officers is not sufficient for probable cause in S5(a). S5(b) also does not apply as Teck and
Junio did not even categorically identify Wang to be their source of shabu. Thus, the warrantless
arrest was really illegal, and the incidental search to the illegal arrest is likewise unlawful.

226. SPO2 Rolando Jamaca v. People, GR 183681, July 27, 2015, Peralta, J. (Double
Jeopardy)
FACTS:
Atty. Bangot filed a complaint for grave threats against Jamaca with the Office of the Deputy
Ombudsman for the Military. He also filed a similar complaint with the Office of the City
Prosecutor. The ombudsman dismissed the complaint saying that the accusation was unfounded
based solely on the statement of one Rustom that there were no threatening words uttered by
Jamaca. A certiorari petition was filed with this Court but this was dismissed.

The complaint with the City Prosecutor prospered and led to the filing of an information against
Jamaca. He was charged with grave threats. (“If I will lose my work, I will break the head of Atty.
Bangot”). Three prosecution witnesses, including Bangot’s son, testified that while at the house of
Rustom, they all heard Jamaca utter words threatening to cause Atty. Bangot grave bodily harm.
The RTC convicted Jamaca. The CA affirmed. Hence this petition for review on certiorari. He
claims that the CA should have dismissed the petition and the RTC had no jurisdiction as Bangot
was guilty of forum shopping, having filed similar complaints with the Office of the Deputy
Ombudsman and the Office of the City Prosecutor. He also claims that there was res judicata as
the grave threats case was dismissed by the military ombudsman already.

ISSUE:
Whether a dismissal by the military ombudsman of the complaint for grave threats by Atty. Bangot
against SPO2 Jamaca during the preliminary investigation stage for lack of probable cause
precludes the city prosecutor from filing an information against Jamaca for the same case.
HELD: NO.
For a claim of double jeopardy to prosper, petitioner has to prove that first jeopardy has attached
prior to the second. The first jeopardy attaches only 1) after a valid indictment, 2) before a
competent court, 3) after arraignment, 4) when a valid plea has been entered, and 5) when the
accused was acquitted or convicted, or the case was dismissed or otherwise terminated without his
express consent. Here, the complaint before the military ombudsman was dismissed as early as
the preliminary investigation stage. Thus, there was as yet, no indictment to speak of. No
complaint or information has been brought before a competent court. None of the aforementioned
events transpired for first jeopardy to attach.

It is settled that the dismissal of a case during its preliminary investigation does not constitute
double jeopardy since a preliminary investigation is not part of the trial and is not the occasion
for the full and exhaustive display of the parties’ evidence but only such as may engender a
well-grounded belief that an offense has been committed and accused is probably guilty thereof.
It cannot be considered equivalent to a judicial pronouncement of acquittal.

The ombudsman’s jurisdiction to investigate public officers under S15, RA 6770 is not exclusive
but is shared with other similarly authorized government agencies. Thus, the prosecutor can still
file the information even if the ombudsman has already dismissed the case.

227. People v. Aurelio Balisacan, GR L-26376, August 31, 1966, Bengzon, J.P., J. (Double
Jeopardy)
FACTS:
Appeal by the prosecution of an acquittal. Balisacan was charged with homicide in the CFI. He
pleaded guilty. At his de oficio counsel’s petition, he was allowed to present evidence to prove
mitigating circumstances. Balisacan testified that he stabbed the deceased in self-defense as the
latter was strangling him. He also said that after the incident, he surrendered voluntarily to police.
Based on the testimony, the CFI acquitted him. The prosecution in this appeal claims that the trial
court erred in acquitting Balisacan despite his plea of guilty.

ISSUE:
Whether the acquittal of Balisacan by the CFI based on his testimony that he acted in self-defense
when the defense was allowed to present evidence only to show mitigating circumstances and
where the prosecution was not given opportunity to present its own evidence can still be appealed
without violating the prohibition against double jeopardy.
HELD: YES.
A plea of guilty is an unconditional admission of guilt. It forecloses the right to defend oneself
from the charge and leaves the court with no alternative but to impose the penalty fixed by law
under the circumstances. Here, Balisacan was allowed to testify to establish mitigating
circumstances for purposes of fixing the penalty. The testimony could not thus be taken as a
trial on the merits to determine the guilt or innocence of the accused.

In view of the assertion of self-defense, the proper course should have been for the court to take
defendant’s plea anew and then proceed to the trial in the order under S3, Rule 119. In deciding
the case upon the merits without the requisite trial, the CFI not only erred in procedure but
deprived the prosecution of its day in court and right to be heard.

Does this appeal place Balisacan in double jeopardy? While he entered a plea of guilty, he testified
that he acted in self-defense. This had the effect of vacating his plea of guilty and the CFI should
have required him to plead anew on the charge, or at least direct that a new plea of not guilty
be entered for him. This was not done. It follows that in effect, there having been no standing plea
at the time the CFI rendered its judgment of acquittal, there can be no double jeopardy.
Also, the CFI decided the case upon the merits without giving the prosecution the opportunity to
present its evidence or to rebut the testimony of Balisacan. It acted without due process of law.
For lack of this fundamental pre-requisite, its action is null and void. The acquittal is thus void
for want of due process. It is no acquittal at all and cannot be the basis for a claim of double
jeopardy. The case was remanded.

228. Ramon Paulin v. Hon. Gimenez, GR 103323, January 21, 1993, Melo, J. (Double
Jeopardy)
FACTS:
The jeep ridden by private respondent and brgy. Captain Belme was overtaken by the Nissan Patrol
ridden by petitioners, Spouses Paulin, smothering the former with dust. Irked, Belme followed the
Nissan until it entered the back gate of Rattan Originals in Cebu. Inquiring with the security guard
as to who owns the Nissan, he was informed that it was driven by Ramon Paulin.

Later, while Belme was investigating some problems of his constituents, Ramon and his wife
Angie allegedly pointed their guns at Belme while Bacho, their companion, acted as back-up.
Belme instructed one tanod to call police and the rest to block the exit of the spouses. Sensing they
were outnumbered, the spouses put their guns down upon arrival of police. They were brought to
the police station. A complaint for grave threats was filed against spouses Paulin and Bacho,
petitioners. The station commander also filed a complaint for grave threats and oral defamation
against Belme.

The cases were tried jointly. Acting on a motion of the spouses Paulin and Bacho, the case
against them were dismissed. Belme filed a MR, which was granted. Petitioners sought the setting
aside of the resolution granting the MR, but this was denied. they filed a certiorari petition with
the RTC presided by Judge Gimenez, who dismissed the petition. Hence this petition, arguing that
the judgment dismissing their case was an acquittal and may no longer be set aside without
violating petitioners’ right against double jeopardy.

ISSUE:
Whether the dismissal of a criminal case upon motion of the accused counts as first jeopardy for
purposes of the right against double jeopardy.
HELD: NO.
For double jeopardy to be validly invoked, the following are the requisites: 1) a valid complaint or
information, 2) a competent court, 3) the defendant had pleaded to the charge, and 4) the defendant
was acquitted or convicted, or the case against him was dismissed or terminated without his express
consent.

But there are exceptions. An appeal by the prosecution from the dismissal order of the criminal
case by the trial court shall not constitute double jeopardy if 1) the dismissal is made upon motion
or with the express consent of the defendant, 2) the dismissal is not an acquittal or based upon
consideration of the evidence or merits of the case, and 3) the question to be passed upon by the
appellate court is purely legal so that should the dismissal be found incorrect, the case would have
to be remanded to the court of origin to determine guilt or innocence.
Where the dismissal was ordered upon motion or with the express assent of the accused, he is
deemed to have waived his protection against double jeopardy. Here, the dismissal was granted
upon motion of petitioners. Double jeopardy did not attach.

It is insisted that the MTC decision is an acquittal as it was issued after the merits of the
prosecution’s evidence were considered. There are distinctions between an acquittal and dismissal:
Acquittal is based on the merits- the defendant is acquitted because the evidence does not show
that defendant’s guilt is beyond reasonable doubt; but dismissal does not decide the case on the
merits or that the defendant is not guilty. Dismissals terminate the proceedings either because
the court is not a court of competent jurisdiction, or the evidence does not show that the offense
was committed within the territorial jurisdiction of the court, or the complaint or information is
not sufficient in form and substance, etc.

The MTC decision dismissing the case is not an acquittal considering that no finding was made
as to the guilt or innocence of the petitioners. The case was dismissed without the proper
information having been filed, it appearing that the proper charge should have been “disturbance
of public performance” under Art. 153, not grave threats under Art. 282, of the RPC.

There are exceptional instances when dismissal may be held to be final even if made on motion
of accused: 1) where the dismissal is based on a demurrer to evidence filed by accused after the
prosecution rests, which has the effect of a judgment on the merits and operates as an acquittal,
and 2) where the dismissal is made, also on motion of accused, because of denial of his right to a
speedy trial which is in effect a failure to prosecute. Petitioners’ motion to dismiss premised on
procedural grounds is not one of the exceptions, especially since the prosecution still had to
present several witnesses when the case was dismissed. The prosecution would be denied due
process if it never had the chance to offer its evidence formally in view of the trial court’s
dismissal. The trial court would be ousted from jurisdiction by aborting the prosecution’s right to
complete presentation of its evidence, and thus first jeopardy would not be terminated.

229. People v. City Court of Silay, GR L-43790, December 09, 1976, Muñoz Palma, J. (Double
Jeopardy)
FACTS:
Private respondent Ernesto et al. were charged with falsification by private individuals and use of
falsified document under par.2, Art.172 of the RPC. After the prosecution rested its case, private
respondents moved to dismiss the charge on the ground that the evidence presented was not
sufficient to establish their guilt beyond reasonable doubt. The City Court, acting on this motion,
dismissed the case on the ground that the acts committed as narrated do not constitute the crime of
falsification as charged. Hence this petition for review by the city fiscal et al.

Private respondents claim that there was no error by the city court and that for this Court to grant
this petition would place them in double jeopardy. The People assert that the plea of double
jeopardy is not tenable as the case was dismissed upon motion of the accused.

ISSUE:
Whether a motion to dismiss by the accused in a criminal case after the prosecution has rested its
case, which was granted, may be reversed by the trial court.
HELD: NO.
We disagree with the SOLGEN. It is true that the criminal case of falsification was dismissed on
motion of accused. But this motion was filed AFTER the prosecution had RESTED its case,
calling for an appreciation of the evidence adduced and its sufficiency to warrant conviction
beyond reasonable doubt, resulting in a dismissal of the case ON THE MERITS, tantamount
to an acquittal.

In Catilo v. Hon. Abaya, the accused moved to dismiss the case due to insufficiency of evidence
after the prosecution rested its case. Abaya granted it, but later he set aside his order motu proprio.
The Court said that it was an acquittal, regardless of whether it was due to some misrepresentation
of facts as stated in Abaya’s order of reconsideration or to a misapprehension of the law or of the
evidence of the prosecution. The fact is that it was a valid order of acquittal.

All the elements of double jeopardy are present: 1) a valid information sufficient in form and
substance to sustain a conviction of the crime charged, (2) a court of competent jurisdiction, and
(3) an unconditional dismissal of the complaint after the prosecution had rested its case, amounting
to the acquittal of the accused. The dismissal being one on the merits, the doctrine of waiver of
the accused to a plea of double jeopardy cannot be invoked.

While it is clear that the dismissal of the case was erroneous, however erroneous the order of the
city court was, and although a miscarriage of justice resulted from the order, such error cannot
now be righted because of the timely plea of double jeopardy.

230. Generoso Esmeña v. Judge Pogoy, GR L-54110, February 20, 1981, Aquino, J. (Double
Jeopardy)
FACTS:
Petitioners Esmeña and Alba and their co-accused were charged with grave coercion for allegedly
having forced Rev. Fr. Tibudan to withdraw P5k from the bank and give the amount to the accused
because the priest lost it in a game of cards. The case was calendared on Oct. 4, 1978 for
arraignment and trial. Upon the request of Fr. Tibudan, the case was reset to Dec. 13. Because
Esmeña and Alba were not notified, they did not appear. They pleaded guilty at their arraignment
on January 23, 1979. No trial was held after arraignment as complainant Fr. Tibudan requested the
transfer of the hearing to another date.

The fiscal then lost his record of the case. So the hearing scheduled on June 18 was cancelled at
his instance. The Judge ordered setting the trial “for the last time on August 16, 1979.” When the
case was called on that date, the fiscal informed the court that the private prosecutor received from
Fr. Tibudan a telegram stating that he was sick. Counsel for petitioners opposed the cancellation
of the hearing, invoking their right to have speedy trial. They insisted that the case be heard on
that day, otherwise it should be dismissed.

Judge Pogoy provisionally dismissed the case as to the accused who were present as it has been
“dragging all along and the accused are ready for hearing” but the fiscal was not ready with his
witness. 27 days later on Sept. 12, the fiscal moved to revive the case. The judge granted it.
Petitioners moved to dismiss on the ground of double jeopardy. This was denied. hence this
petition for certiorari.
ISSUE:
Whether the dismissal of the criminal case against petitioners on the part of the judge alone due to
their invocation of their right to speedy trial after the trial of the case was delayed multiple times
because the private complainant cannot attend the trial constitutes first jeopardy for purposes of
double jeopardy.
HELD: YES.
For double jeopardy to exist, there should be 1) a valid complaint or information 2) before a court
of competent jurisdiction and 3) the accused has been arraigned and pleaded. When all these are
present, the acquittal or conviction of accused or dismissal of the case without his express consent
constitutes res judicata, barring another prosecution for the offense or for any attempt to commit
the same or frustration thereof, or for any offense which necessarily includes or is included therein.

Here, petitioners were placed in jeopardy by the provisional dismissal of the case. That
provisional dismissal would not have placed them in jeopardy if Judge Pogoy took the precaution
of making sure that the dismissal was with their consent. Here, it is not very clear that
petitioners consented to the dismissal of the case.

It is the practice of some judges before issuing an order of provisional dismissal in a case where
accused is already arraigned to require accused and counsel to sign the minutes of the session or
any available part of the record to show conformity of accused to the provisional dismissal. The
judge specifies in the order of provisional dismissal that the accused and his counsel assented
thereto. This procedure leaves no doubt as to the consent and precludes double jeopardy.

The petitioners here were insisting on a trial relying on their right to have speedy trial. The fiscal
was not ready as his witness was not in court. Judge Pogoy of his own volition provisionally
dismissed the case. The petitioners did not expressly manifest their conformity to the
provisional dismissal. Hence, the dismissal placed them in jeopardy.

231. People v. Hon. Pineda, GR L-44205, February 16, 1993, Melo, J. (Double Jeopardy)
FACTS:
When Consolacion Naval, private respondent, was separately accused of having committed estafa
in Crim Case 15795 and of falsification in Crim Case 15796 both in the CFI, she sought the
quashal of the latter charge on the ground that she is in danger of being convicted for the same
felony. Judge Pineda was persuaded, believing that the alleged falsification was a necessary
means of committing.

The two informations disclose that Naval sold a land to Edilberto who made a partial payment of
P130k. 2 years later, an application for registration under the Land Registration Act was submitted
by Naval where she stated that she owned the lot and that it was unencumbered. The title was
issued in her name but she allegedly disclosed the half portion to nine other persons.

Judge Pineda at first denied Naval’s motion to quash, but then reconsidered. Hence this petition.

ISSUE:
Whether Naval’s right against double jeopardy is violated when she is charged by two separate
informations charging estafa and falsification on the ground that these are the same offenses or the
falsification was a necessary means to the estafa.
HELD: NO.
It is unacceptable to say that Naval concocted the scheme of falsification in 1971 to facilitate the
estafa in 1973 such that both crimes emanated from a single criminal impulse. They were
committed on different dates and places. The hiatus of 2 years precludes the submission that the
first is an integral part of or intimately interwoven with the second felony.

The mere filing of two informations charging the same offense is not an appropriate basis for
invoking double jeopardy since the first jeopardy has not yet set in by a previous conviction,
acquittal, or termination of the case without consent of the accused. The mere filing of two
informations or complaints charging the same offense does not yet afford accused the occasion to
complain that he is being placed in jeopardy twice for the same offense for the primary basis of
double jeopardy is that the accused has already been convicted or acquitted in the first case or
that the same was terminated without his consent.

Legal jeopardy attaches only upon a 1) valid indictment 2) before a competent court 3) after
arraignment, 4) a valid plea having been entered, and 5) the case was dismissed or otherwise
terminated without the express consent of the accused. Where there is no conviction, acquittal, or
dismissal, no jeopardy attaches.

232. People v. Hon. Adil, GRL-41863, April 22, 1977, Barredo, J. (Double Jeopardy)
FACTS:
The first criminal complaint filed against respondent Fama (Case 3335) was for physical injuries
requiring “medical attendance for a period from 5 to 9 days barring complications.”(Slight
physical injuries) Fama pleaded not guilty. Meanwhile, complainant Viajar filed a letter-
complainant with the fiscal charging Fama with serious physical injuries arising from the same
incident in Case 3335. After preliminary investigation, the fiscal filed an information (Case 5241)
for serious physical injuries, “leaving a permanent scar and deforming on the right face.”

Fama moved to defer proceedings in Case 5241 claiming that since he was already charged and
pleaded not guilty in Case 3335, he would be in double jeopardy if Case 5241 were to be
prosecuted. Case 3335 was set for hearing, but the fiscal sought postponements to await the
resolution of the issue of double jeopardy in Case 5241. As a result, the trial court dismissed Case
3335.

Fama filed an addendum to memorandum in Case 5241, inviting attention to the dismissal of Case
3335 and reiterating his theory of double jeopardy. The trial court then issued the impugned order
dismissing Case 5241.

ISSUE:
Whether the initial charge of slight physical injuries had put Fama in jeopardy such that when the
injuries of the victim healed and it became apparent that his wounds would scar his face causing
deformity, where such deformity could not be predetermined until the wounds healed, Fama could
no longer be prosecuted for serious physical injuries for the deformity as there would be double
jeopardy.
HELD: NO.
What happened here was that when Case 3335 was filed, the charge was only slight physical
injuries only because the certification of the attending physician stated that the injuries of offended
party Viajar would require medical attendance from 5-9 days only “barring complications.” Only
3 days passed since the incident when the complaint was filed and there no indications of a graver
injury to be suffered by Viajar. It was only after Case 3335 was filed already and the wound on
Viajar’s face already healed, that the deformity became apparent. The scar or deformity left by
a wound on the face of a person cannot be predetermined.

In Peo v. Silva, there was no question that the extent of the damage to property and physical injuries
suffered by the victims therein were already existing and known when the prior minor case was
prosecuted. What is controlling here is Melo v. People, in which it was held that the rule of identity
does not apply when the second offense was not in existence at the time of the first prosecution,
for in such case, there is no possibility for the accused during the first prosecution to be
convicted for an offense that was then inexistent. Thus, where accused was charged with
physical injuries and after conviction the injured dies, the charge of homicide against the same
accused does not put him twice in jeopardy.

If after the first prosecution a new fact supervenes on which defendant may be held liable, altering
the character of the crime and giving rise to a new offense, the accused cannot be said to be in
second jeopardy if indicted for the new offense. Thus, Fama’s plea of double jeopardy cannot hold.

233. Jason Ivler v. Hon. Modesto-San Pedro, GR 172716, November 17, 2010, Carpio, J.
(Double Jeopardy)
FACTS:
After a vehicular collision, Jason Ivler was charged before the MeTC with 2 separate offenses: 1)
Reckless Imprudence Resulting in Slight Physical Injuries (Case 82367) and 2) Reckless
Imprudence Resulting in Homicide and Damage to Property (Case 82366) for the death of
respondent Ponce’s husband Nestor Ponce and damage to spouses Ponce’s vehicle.

Ivler pleaded guilty to the charge of Reckless Imprudence Resulting in Slight Physical Injuries
(Case 82367) and was meted the penalty of public censure. Invoking this conviction, Ivler moved
to quash the information in Case 82366 for placing him in jeopardy of second punishment for
the same offense of reckless imprudence.
DEFENSE: Double Jeopardy.
MeTC: Refused quashal, finding no identity of offenses in the two cases.
He filed certiorari with the RTC. He sought suspension of proceedings of Case 82366 including
arraignment, invoking as prejudicial question the RTC case. MeTC proceeded with arraignment,
cancelled his bail because of his absence, ordered his arrest, and denied his motion to suspend and
postpone arraignment. Relying on the arrest order, respondent Ponce sought dismissal of the RTC
case for Ivler’s loss of standing to maintain suit.
RTC: Dismissed. Forfeiture of standing due to order of arrest. Effectively affirmed MeTC.
Hence this petition.
CONTENTION of PONCE: Light offenses cannot be complexed under Art.48 with grave or less
grave felonies. Hence, the prosecution was obliged to separate the charge in Case 82366 for slight
physical injuries from Case 82367 for homicide and damage to property.
HELD:
The right against double jeopardy protects an accused from, among others, post-conviction
prosecution for the same offense, with the prior verdict rendered by a court of competent
jurisdiction upon a valid information. Case 82367 was rendered by a court of competent
jurisdiction upon a valid charge. Thus, the question is whether Cases 82366 and 82367 involve the
“same offense.” It does.

Reckless imprudence is a single crime. Its consequences on persons and property are material only
to determine the penalty. The two charges arising from the same facts were prosecuted under the
same Art.365 of RPC. Structurally, these 9 paragraphs are collapsible into four sub-groupings
relating to 1) the penaltiesattached to the quasi-offenses of "imprudence" and "negligence"
(paragraphs 1-2); (2) a modified penalty scheme for either or both quasi offenses (paragraphs 3-4,
6 and 9); (3) a generic rule for trial courts in imposing penalties (paragraph 5); and (4) the de9nition
of "reckless imprudence" and "simple imprudence" (paragraphs 7-8). Conceptually, quasi-offenses
penalize the mental attitude or condition behind the act, the dangerous recklessness, lack of
care or foresight, unlike willful offenses which punish the intentional criminal act.

Criminal negligence is not a mere modality of committing felonies, else it would be absorbed in
the mitigating circ of lack of intent to commit so grave a wrong. Also, the actual penalty for
criminal negligence has no relation to the individual willful crime (resulting in xxx) but is set in
relation to a whole class or series of crimes. This explains why the technically correct way to allege
quasi-crimes is to state that their commission results in damage, either to person or property.

The essence of the quasi offense of criminal negligence under Art.365 lies in the execution of an
imprudent or negligent act that, if intentionally done, would be punishable as a felony. The
law penalizes the negligent or careless act, not the result thereof. The gravity of the consequence
is only taken into account to determine the penalty, it does not qualify the substance of the offense.
As the careless act is single, whether the injurious result should affect one or several persons,
the offense (criminal negligence) remains one and the same, and cannot be split into different
crimes and prosecutions. Hence, the lower courts erred in refusing to extend in his favor the
protection of the double jeopardy clause.

Article 48 does not apply to acts penalized under Art.365 of RPC. Art.48 is a procedural device
allowing single prosecution of multiple felonies falling under either of 2 categories: 1) when a
single act constitutes two or more grave or less grave felonies (excluding light felonies), and 2)
when an offense is a necessary means for committing the other. In contrast, Art.365 is a
substantive rule, penalizing not an act defined as a felony but the mental attitude behind the
act, the dangerous recklessness, lack of care or foresight.” Thus, Art. 365 was crafted as one
quasi-crime resulting in one or more consequences.

Ordinarily, these two provisions operate smoothly. Art.48 combines in a single prosecution
intentional crimes in Titles 1-13 of RPC, Book II. Art.365 governs prosecution of imprudent acts
and their consequences. But the complexities of human interaction can produce a hybrid quasi-
offense not falling under either models- that of a single criminal negligence resulting in multiple
non-crime damage to persons and property with varying penalties corresponding to light, less
grave or grave offenses. Should Art.48’s framework apply to “complex” the single quasi-offense
with its multiple (non-crime) consequences (excluding light offenses separately tried)? OR should
the prosecution proceed under a single charge, collectively alleging all the consequences of the
single quasi-crime, to be penalized separately following the scheme of penalties in Art.365?
Jurisprudence adopts both approaches.

One line of rulings (none of which involved double jeopardy) applied Art.48 by “complexing” one
quasi crime with its multiple consequences unless one consequence amounts to a light felony in
which case charges were split by grouping, on the one hand, resulting acts amounting to grave or
less grave felonies and filing the charge with the second level courts and, on the other hand,
resulting acts amounting to light felonies and filing the charge with the first level courts. This is
the approach the MeTC impliedly sanctioned.

Under this approach, the issue of double jeopardy will not arise if the “complexing” of acts
penalized under Art.365 involves only resulting acts penalized as grave or less grave felonies as
there will be a single prosecution of all the resulting acts. The issue arises if one resulting act is
penalized as a light offense and the other as grave or less grave, in which case, Art.48 is not
deemed to apply and the act penalized as a light offense is tried separately from the resulting
acts penalized as grave or less grave offenses.

The second jurisprudential path nixes Art.48 and sanctions a single prosecution of all the effects
of the quasi-crime collectively alleged in one charge, regardless of their number or severity,
penalizing each consequence separately. Thus, par.3 of Art.365 was interpreted as meaning that if
there is only damage to property, the amount fixed therein shall be imposed, but if there are also
physical injuries, there should be an additional penalty (penalty scheme in Art.365) for the latter.
The information cannot be split into two, one for physical injuries and another for damage to
property.

These approaches are irreconcilable. It is conceptually impossible for a quasi-offense to stand for
a single act constituting two or more grave or less grave felonies, or an offense which is a
necessary means for committing another. The contention that double jeopardy does not bar a
second prosecution for slight PI thru reckless imprudence because the charge could not be joined
with the other for serious PI thru reckless imprudence following Art.48 was rejected. “Having first
prosecuted defendant for the lesser offense, the prosecuting attorney is not in a position to press in
this case the more serious charge arising from the same alleged reckless imprudence.”

Thus, we hold that prosecutions under Art.365 should proceed from a single charge, regardless
of the number or severity of the consequences. In imposing penalties, the judge will do no more
than apply the penalties under Art.365 for each consequence alleged and proven. There shall
be no splitting of charges and only one information shall be filed in the same first level court.

234. People v. Hon. Relova, GR L-45129, March 06, 1987, Feliciano, J. (Double Jeopardy)
FACTS:
Batangas police, with a search warrant, searched the premises of Opulencia Capena Ice Plant
owned by private respondent Manuel Opulencia. The police found that electric wiring and
contraptions had been installed without the necessary authority from the city government. These
contraptions were designed purposely to lower the readings of electric current consumption in the
electric meter of the plant. Opulencia admitted in a written statement that he caused the installation
of the electric devices to lower the readings of his electric meter.

The fiscal charged Opulencia of violating Ordinance 1 s.1974, Batangas City. this was punishable
by a fine of P5 to P50 or imprisonment not exceeding 30 days or both at the discretion of the court.
Opulencia pleaded not guilty. He moved to dismiss on the ground of prescription and lack of
jurisdiction, which was granted since the offense is a light felony prescribing in 2 months from
discovery, it appearing that the charge was filed 9 months after discovery.

14 days later, the fiscal filed another information against Opulencia for theft of electric power
under Art. 308 in relation to Art.309 par.1. Opulencia moved to quash on the ground of previous
acquittal and double jeopardy. The judge granted the motion to quash. Hence this certiorari
petition.

ISSUE:
Whether the act of installing a device to reduce the electric meter reading in Opulencia’s plant may
be charged as theft under the RPC after the dismissal of the first charge of violation of Ordinance
1 of Batangas for the same act on the ground of prescription.
HELD: NO.
Petitioner argues that Ordinance 1 punishes installing electric wiring without authority. It is not
necessary to establish mens rea. In contrast, theft under Art.308 has different elements. Thus, it
concludes that the unauthorized installation punished by the ordinance of Batangas is not the same
as theft of electricity under the RTC and the second offense is not an attempt to commit the first
or a frustration thereof and the second offense is not necessarily included in the first offense.

S22, Art.IV of the 1973 Constitution reads:


"No person shall be twice put in jeopardy of punishment for the same offense. If an act is
punished by a law and an ordinance, conviction or acquittal under either shall constitute a
bar to another prosecution for the same act."
The first sentence sets forth the general rule: the protection against double jeopardy is not
available where the second prosecution is for an offense that is different from the offense charged
in the first or prior prosecution, although the first and second offenses may be based upon the
same act or set of acts. The second sentence embodies an exception to the general proposition:
the protection is available although the prior offense charged under an ordinance be different from
the offense charged subsequently under a national statute like the RPC, provided both offenses
spring from the same act or set of acts.

This was made clear in Yap v. Lutero, where Yap was charged with violating an ordinance on
reckless driving and 3 months later again charged with serious physical injuries thru reckless
imprudence, charging a violation of the Revised Motor Vehicle Law by recklessly driving and as
a result inflicting injuries upon a pedestrian. The second charge was dismissed by the CFI, which
dismissal was affirmed by the SC, holding that there was double jeopardy. We held that the first
sentence prohibits double jeopardy for the same offense whereas the second contemplates double
jeopardy for the same act. Under the first, one may be put twice in jeopardy of the same act,
provided he is charged with different offenses or the offense charged in one case is not included
or does not include the crime charged in the other case. The second sentence applies even if the
offenses charged are not the same, owing to the fact that one is a violation of an ordinance and
the other of a statute. Incidentally, such conviction or acquittal is not indispensable provided
jeopardy has attached under one of the informations charging said offense.

Put differently, where the offenses charged are penalized either by different sections of the same
or different statutes, the important inquiry relates to the identity of OFFENSES charged. The
protection against dj is available only where an identity is shown between the offenses. In contrast,
where one offense is charged under a municipal ordinance while the other is penalized by a
statute, the inquiry is to the identity of the ACTS which the accused is said to have committed
and which are alleged to have given rise to the two offenses. The protection is available if the act
giving rise to the offense under the ordinance is the same act giving rise to the offense in the statute.

As to why this is so, if the second sentence of the double jeopardy provision was not written into
the constitution, conviction or acquittal under a municipal ordinance would never bar another
prosecution for the same act under a national statute. An offense penalized by a municipal
ordinance is, by definition, different from an offense under a statute. The two offenses would
never constitute the same offense having been promulgated by different rule-making
authorities and the plea of double jeopardy would never be. The second sentence was inserted
precisely to extend the constitutional protection to a situation which would otherwise not be
covered by the first sentence.

The question of identity or lack of identity of offenses is addressed by examining the essential
elements of each of the two offenses charged as set out by the legislative definitions of the
offenses. The question of identity of acts claimed to generate liability both under a municipal
ordinance and a national statute must be addressed, in the first instance, by examining the location
of such acts in time and space. When the acts are so related in time and space as to be reasonably
regarded as having taken place on the same occasion and where those acts have been moved by
one and the same, or continuing, intent or voluntary design or negligence, such acts may be
characterized as an integral whole capable of giving rise to penal liability simultaneously under an
ordinance and statute.

Here, the acts took place within the same time frame, from Nov. 1974 to Feb. 1975. During this
period, Opulencia installed electrical devices in his plant without authority. The corrupt intent to
reduce the electric power bill was present from the very moment the unauthorized installation
began. The immediate physical effect was the flow of electric current into the plant without its
recording in the meter. The “taking” of electric current was integral with the unauthorized
installation of the devices.

The identity of the first and second offense need not be absolute identity. The 2 offenses may be
regarded as the same offense where the second offense necessarily includes the first or where the
second offense is an attempt to commit the first or a frustration thereof. Not all the technical
elements constituting the first offense need be present in the technical definition of the second
offense. The law seeks to prevent harassment of an accused by multiple prosecutions for offenses
which though different are nonetheless constituted by a common set or overlapping sets of
technical elements.

Likewise, acts of a person which physically occur on the same occasion and are infused by a
common intent or design or negligence and thus form a moral unity should not be segmented and
sliced to produce as many different acts as there are offenses under ordinances or statutes.

Thus, it is not without reluctance that we deny the people’s petition for certiorari, for the fiscal by
inadvertence chose to file an information for an offense which he should have known already
prescribed. Since prescription is one of the grounds for total extinction of criminal liability under
Art.89 of the RPC, the dismissal by the City Court on prescription was an acquittal. But the
extinction of criminal liability does not carry with it extinction of civil liability. The determination
of civil liability was remanded to the CFI.

235. William Tan v. Hernani Barrios, GR 85481-82, October 18, 1990, Griño-Aquino, J.
(Double Jeopardy)
FACTS:
Based on Proc. 1081 thru General Order 8, Marcos authorized the AFP chief to create military
tribunals to try and decide cases of military personnel. General Order 21 vested the military
tribunals, “exclusive of the civil courts”, with jurisdiction over violations of the law on firearms,
among others, and other crimes directly related to the quelling of rebellion and preservation of the
safety of PH. General Order 12-b added crimes against persons in RPC to the jurisdiction of
military tribunals. General Order 49 redefined the jurisdiction of the military tribunals, excluding
crimes against persons. But although civil courts have exclusive jurisdiction over offenses not
mentioned, the order stated that the president may, in the public interest, refer to a military tribunal
a case falling within the jurisdiction of the civil courts and vice versa.

The three petitioners herein and 12 others were arrested and charged with murder thru use of an
unlienced firearm and unlawful possession of a pistol. The case was retained in the military court.
They all pleaded not guilty. 5 were convicted of murder. One was convicted of murder and illegal
possession. 8, including the 3 petitioners, were acquitted (Criminal Case 1-67).

Proc. 2045 ended martial law and abolished the military tribunals. On May 22, 1987, the SC, in a
decision in Olaguer v. Military Commission 34, vacated a sentence by military commission 34
against Olaguer et al. declaring that military commissions have no jurisdiction, even during
martial law, over civilians charged with criminal offenses cognizable by civil courts as long as
those courts are functioning which they were during martial law. the creation of military
commissions to try civilians was found unconstitutional. All their proceedings were annulled. It
was held that “the trial contemplated by the due process clause is a trial by judicial process, not by
executive or military process. Military tribunals are not courts.” “They pertain to the executive
department.” “The power of interpreting the laws is primarily a function of the judiciary. It cannot
be the function of the executive department thru the military authorities.”

Several habeas corpus petitions were filed by 217 prisoners in the national penitentiary who were
convicted by the military commissions for common crimes. The SC granted the petition for HC
and released some who had fully served their sentences, were acquitted, or granted amnesty;
dismissed the petitions of those who were military personnel; and nullified the proceedings against
those convicted and still serving the sentences but, without ordering their release, directed the DOJ
to file the necessary informations against them (Cruz v. Enrile).

SOJ designated state prosecutor Barrios to investigate and prosecute criminal case 1-67. Without
investigating, Barrios filed two informations for illegal possession and murder, incorrectly
certifying that the case was filed in accordance with the SC order in Cruz v. Enrile as petitioners
here are not parties in Cruz. The RTC judge ordered Barrios to submit copies of the supporting
affidavits of the previous cases “wherever they are now.” He did not comply.

Petitioners William, Joaquin, and Vicente Tan filed this petition for certiorari praying that the
informations against them be annulled and that the fiscal and public respondents be permanently
enjoined from prosecuting them anew for the offenses charged because they had already been
acquitted of the same by the military commission in Crim Cas 1-67. The RTC denied.

Petitioners claim that the decision in Cruz did not direct the filing of informations by SOJ against
those who were acquitted by court martial proceedings and that they were not parties in Cruz.
Their reprosecution would violate their right against double jeopardy.

The SOLGEN argues that since the proceedings in the military commission were void due to
Olaguer for being without jurisdiction over civilians, their decisions, whether of conviction or
acquittal, do not bar reprosecution before a civil court.
ISSUE:
Whether the SC’s ruling in Olaguer, stating that military tribunals had no jurisdiction over
common crimes cognizable by functioning civil courts, should nullify such tribunal’s acquittal of
petitioners herein for murder and illegal possession of firearm and allow their reprosecution for
the same offenses.
HELD: NO.
Public respondents gravely abused their discretion in misconstruing the dispositive portion of the
decision in Cruz as their authority to refile in the civil court the criminal actions against petitioners
who were acquitted by military commission no.1. There is nothing in Cruz for such authority to
reprosecute every civilian who had ever faced a court martial, much less those who had been
acquitted by such bodies like petitioners. Cruz would be an instrument of oppression unless given
a limited application only to the parties therein. Only they are affected. “Matters adjudged in a
cause do not prejudice those who were not parties to it.” The reprosecution of those who were not
parties in Cruz would violate their right to due process.

But we rectify the inconsistency of application of the Olaguer doctrine in Cruz. Although the Court
nullified the proceedings against civilians-petitioners who were still serving sentence after
conviction by the military courts, we did not nullify the court martial proceedings against other
civilians-petitioners who 1) finished serving sentence, 2) were granted amnesty, 3) were acquitted
by the courts martial. We merely set them free. In effect, the Court applied a rule of retroactive
invalidity to the first group and of prospective invalidity for the second group who were ordered
released.
We now hold that Olaguer should be applied prospectively only to future cases and cases still
ongoing or not yet final when that decision was promulgated. There should be no retroactive
nullification of final judgments, whether of conviction or acquittal, by courts martial against
civilians before the promulgation of Olaguer. Such final sentences should not be disturbed. Only
in cases where the convicted person shows that there was serious denial of his constitutional
rights should the nullity of the sentence be declared and a retrial be ordered based on such
violation, and not on the Olaguer doctrine. Olaguer was sentenced to death by the court martial
without receiving his evidence. It would be a cruel distortion of Olaguer to use it as authority for
reprosecution regardless if the accused was afforded a fair trial and regardless of whether they
have already been acquitted or released. Not everybody convicted by a military court desires to
undergo the ordeal of a second trial for the same offense, albeit in a civil court. Why should one
who has accepted the justness of the court martial’s verdict and is satisfied that he had a fair hearing
be dragged through another hearing? The possible acquittal in the second hearing would be of
small comfort if he is held without bail pending completion of the second trial which may take
longer than his original sentence.

The trial of civilians for common crimes before military tribunals during 1971-1981 is an
operative fact which may not be justly ignored. The doctrine of operative facts applies to
petitioners and their co-accused. The principle of absolute invalidity of the jurisdiction of military
courts over civilians should not be allowed to obliterate the operative facts that, in the particular
case of petitioners, the proceedings were fair, there was no serious violation of their right to
due process, and the jurisdiction of the military commission that heard their charges was affirmed
by the SC in Aquino v. Military Commission 2 years before Olaguer.

Because of these established operative facts, the refiling of the information against petitioners
would place them in double jeopardy. Also, depriving petitioners of the protection of the
judgment of acquittal by the military commission by retroactively divesting such commission of
jurisdiction it had exercised over them would amount to an ex-post facto law or ruling.

Sec.22- EX-POST FACTO LAW and BILL OF


ATTAINDER
236. People v. Hon. Ferrer, GR L-32613-14, December 27, 1972, Castro, J. (Ex-post Facto
Law and Bill of Attainder)
FACTS:
A criminal complaint for violation of S4 of the Anti-Subversion Act was filed against respondent
Feliciano Co in the CFI. Judge Guzman conducted a preliminary investigation and, finding a prima
facie case against Co, directed government prosecutors to file the corresponding information. Co
moved to quash on the ground that the Anti-Subversion Act is a bill of attainder. Tayag and five
others were also charged with subversion. Tayag moved to quash alleging that the act is a bill of
attainder and violates equal protection. The trial court declared that the Anti-Subversion Act is
void. Hence this appeal.

ISSUE:
Whether the Anti-Subversion Act, punishing those who, after its enactment, knowingly join the
Communist Party of the Philippines with the specific intent of furthering its objective of
overthrowing the Philippine government, is a bill of attainder.
HELD: NO.
A bill of attainder is a legislative act which inflicts punishment without trial. Its essence is the
substitution of a legislative for a judicial determination of guilt. The ban on bills of attainder serves
to implement separation of powers by confining legislatures to rule-making and forestalling
legislative usurpation of the judicial function. The singling out of a definite class, the imposition
of a burden on it, and a legislative intent, suffice to stigmatize a statute as a bill of attainder.

Here, the Anti-Subversion Act was condemned as a bill of attainder because it “tars the CPP as a
continuing menace to the security of the country.” The trial court said that Congress usurped the
powers of the judge.

1. The act does not specify the CPP or its members for the purpose of punishment. It simply
declares the Party to be an organized conspiracy for the overthrow of the Government for the
purposes of the prohibition. The term “Communist Party of the PH” is used solely for definitional
purposes. The Act does not apply only to the CPP, but also to “any other organization having
the same purpose and their successors.” Its focus is not on individuals but on conduct. This
distinguishes it from the US Federal Labor-Management Reporting and Disclosure Act of 1959
which was held to be a bill of attainder in US v. Brown.

S504 of the US law provides that no person who is or was a member of the communist party shall
serve as officer etc. of any labor organization within 5 years of termination of membership in the
communist party and penalizes those who violate its provisions. This statute specifies the
Communist Party and imposes disability and penalties on its members. Membership in the
party, without more, ipso facto disqualifies a person from becoming an officer of any labor
organization. The law did not set forth a rule decreeing that any person who commits certain acts
or possesses certain characteristics making them likely to initiate political strikes shall not hold
union office and leaves to courts the job of deciding what persons have committed the specified
acts. Instead, it designates the persons who possess the feared characteristics and therefore
cannot hold union office.

Were the Anti-Subversion Act a bill of attainder, it would be totally unnecessary to charge
communists in court as the law alone, without more, would suffice to secure their
punishment. But the fact is that their guilt still has to be judicially established. The government
has yet to prove at trial that the accused joined the party knowingly and by overt acts, and that they
joined the CPP knowing its subversive character with the specific intent to further its basic
objective to overthrow the PH government.

As to the claim that organizational guilt is imputed despite requiring proof of knowing
membership, suffice it to say that this is precisely the nature of conspiracy. The contention would
be correct if the statute punished mere membership devoid of any specific intent to further the
party’s unlawful goals. But the law specifically requires that membership must be knowing or
active with intent to further the party’s illegal objectives. The specific intent to pursue the unlawful
goals must be shown by “overt acts”. This is an element of membership.
2. But even assuming that the Act SPECIFIES INDIVIDUALS and not activities, this is not
enough to render it a bill of attainder. A law prohibiting partners of securities underwriting firms
from serving as officers or employees of national banks based on a legislative finding that the
persons mentioned would be tempted to commit acts inimical to the national economy was
declared not a bill of attainder. Also, a law requiring every secret society having membership of at
least 20 to register and punishing members of such societies who fails to register was declared
valid even if in operation it was shown to apply only to the Ku Klux Klan members.

It is only when a statute applies either to named individuals or to easily ascertainable members
of a group in such a way as to inflict punishment on them without judicial trial does it become
a bill of attainder. It is upon this ground that laws disqualifying those who took part in the rebellion
against US during the Civil war from holding office, exercising their profession, etc. based on a
finding that they engaged in subversive activities have been invalidated as bills of attainder.

But when the judgment expressed in legislation is so universally acknowledged to be certain as


to be “judicially noticeable, the legislature may apply its own rules and judicial hearing is not
needed to make such determination. In upholding the aforementioned law against the Ku Klux
Klan although it did not apply to other secret societies like masonic societies, the US relied on
common knowledge of the nature and activities of the Ku Klux Klan, saying that there was a
difference between the Ku Klux Klan and other societies in that the former was commonly known
to have used its secrecy for acts inimical to public welfare.

Here, the CPP has been the object of continuing scrutiny by the SC. In 1932, we found it to be an
illegal association. In 1969, we found that its objective was the overthrow of the PH government
by armed struggle and to establish a communist form of government. Thus, we entertain no doubts
about the existence of a sizeable group of men who have publicly risen in arms to overthrow the
government.

3. It is also not enough that the law specify persons or groups to be a bill of attainder. It is also
necessary that it must APPLY RETROACTIVELY and reach past conduct. This requirement
follows from the nature of a bill of attainder as a legislative adjudication of guilt. Justice
Frankfurter observed that a bill of attainder was doubly objectionable because of its ex post facto
features. This is the historic explanation for uniting the two mischiefs in one clause (No bill of
attainder or ex post facto law shall be passed). If a statute is a bill of attainder, it is also an ex post
facto law. but if it is not an ex post facto law, it cannot be a bill of attainder. Indeed, if one
objection to the bill of attainder is that Congress thereby assumes judicial magistracy, then it must
reach past conduct and that the penalties it imposes are inescapable. It would be valid if the person
can escape regulation merely by altering the course of their own present activities.

S4 of the Anti-Subversion Act provides that it only applies to acts committed “After the approval
of this Act.” Those who were members of the party or of any subversive association at the time
of enactment of the law were given the opportunity of purging themselves of liability by
renouncing in writing and under oath their membership in the party. The law provides that
such renunciation will exempt them from penal liability. The penalties are thus not inescapable.
4. The Act and the requirements of due process.
Congress stated the findings of its investigations in the preamble of the law that the CPP is in fact
an organized conspiracy to overthrow the PH government. Its continued existence is a grave danger
to the security of PH. In truth, the constitutionality of the Act would be open to question if Congress
omitted to make these findings.

In saying that thru the Act, Congress assumed judicial magistracy, the trial court failed to take
proper account of the distinction between legislative fact and adjudicative fact. A law forbidding
the sale of beverages with more than 3.2% alcohol would raise a question of legislative fact-
whether this standard has a reasonable relation to public health, etc. A law forbidding the sale of
intoxicating beverages would raise a question of adjudicative fact- whether a beverage is
intoxicating within the meaning of the statute and the limits on governmental action imposed by
the Constitution.

If laws are seen to have a reasonable relation to a proper legislative purpose, and are neither
arbitrary nor discriminatory, the requirements of due process are satisfied, and judicial
determination to that effect renders a court functus officio. The recital of legislative findings
implements this test. It is not for the courts to reexamine the validity of these legislative findings
and reject them.

Here, the government has a right to protect itself against subversion.

ARTICLE IV- CITIZENSHIP


237. Rizalito David v. Senate Electoral Tribunal, GR 221538, September 20, 2016, Leonen,
J. (Citizenship)
FACTS:
This is a petition for certiorari filed by David praying for the nullification of the decision of public
respondent Senate Electoral Tribunal (SET). The SET decision dismissed the quo warranto
petition of David seeking to unseat private respondent Grace Poe as senator for allegedly not being
a natural-born citizen and thus not qualified to hold such office under Art.VI, S3.

Poe is a foundling whose biological parents are unknown. As an infant, she was abandoned at
the Parish Church of Jaro, Iloilo. Edgardo Militar found her outside the church. He later turned her
over to Mr. and Mrs. Emiliano Militar. Emiliano reported to the office of the local civil registrar
that the infant was found. She was given the name Mary Grace Natividad Contreras Militar. The
registrar issued her a foundling certificate.

The municipal court of San Juan granted the petition for adoption of Grace Poe by spouses Ronald
Poe (Fernando Poe, Jr.) and Jesusa Poe (Susan Roces). She became a registered voter in Greenhills,
San Juan when she turned 18yo. The COMELEC issued her a voter’s identification card on Dec.13,
1986. The DFA issued her a passport on April 4, 1988. Having become a senator, she was also
issued a PH diplomatic passport on December 19, 2013.
She obtained her college degree in the US in 1988 in political science from Boston College,
Massachusetts. She married Misael Llamanzares on July 27, 1991, an American and Filipino
national. On July 29, she returned to US and lived there for some time. She was naturalized and
granted American citizenship on Oct. 18, 2001 and thereafter given a US passport.

Her father died on December 14, 2004. She stayed in the country until Feb. 3, 2005 to attend her
father’s funeral and settle his estate. In 2004, she resigned from work in the US. She returned to
PH on May 24, 2005. She was then issued by the BIR a TIN on July 22, 2005. On July 7, 2006,
she took the Oath of Allegiance to PH. On July 10, 2006, she filed a petition for re-acquisition
of PH Citizenship thru RA 9225. This was granted by the BID, which stated that “she is deemed
to have re-acquired her PH citizenship.”

Poe made several trips to US between 2006-2009 using her US passport. She used the US passport
“after having taken her oath of allegiance on July 7, 2006, but not after she formally renounced
her American citizenship on Oct. 20, 2010.” She was appointed by Pres.Noynoy as MTRCB
chairperson. On Oct. 20, 2010, she renounced her US citizenship, executing an affidavit of
renunciation of allegiance to US therefor. On Oct. 21, she took her oath of office as MTRCB
chairperson and assumed office.

She decided to run as senator in the 2013 elections and won. David, a losing candidate, filed a
petition for quo warranto with SET on August 6, 2015, contesting her election for failing to comply
with the citizen requirement. The SET held oral arguments. The parties were required to submit
their memoranda without prejudice to DNA evidence by Poe. Poe submitted a manifestation
regarding the results of DNA testing, which stated that “none of the tests that Poe took provided
results that would shed light to the real identity of her biological parents. The SET found that Poe
is a natural-born citizen. The SET filed denied David’s MR. hence this petition for certiorari.

ISSUE:
Whether a foundling, Grace Poe, may be considered a natural-born citizen despite her biological
parents being unknown for purposes of election to public office.
HELD: YES.
1. The SET, with the HRET, is created by Art.VI, S17 of the 1987 Constitution:
SECTION 17. The Senate and the House of Representatives shall each have an Electoral
Tribunal which shall be the sole judge of all contests relating to the election, returns, and
qualifications of their respective Members. xxx

The power to resolve contests relating to election, returns, and qualifications is exclusive to any
other body. The resolution of such contests is its only task. It performs no other function. The use
of the word “sole” emphasizes the exclusive character of the jurisdiction conferred. They are
vested with exclusive original jurisdiction over such contests. But the judgments of these tribunals
are not beyond the scope of any review. S17’s “sole” judge must be read in harmony with Art.
VIII, S1’s express statement that judicial power includes the power to determine if there has been
GADALEJ on the part of any branch or instrumentality of the government. Judicial review is
thus still possible, but only in the exercise of the Court’s so-called extraordinary jurisdiction. The
review is limited to determining if there was an error in jurisdiction, not an error in judgment.
An appeal is a continuation of the proceedings in the tribunal from which the appeal is taken. A
petition for certiorari is an independent civil action. There is grave abuse of discretion when a
constitutional organ like the SET makes manifestly gross errors in its factual inferences. It must
be so patent and gross so as to amount to an evasion of a positive duty xxx.

We find no basis to conclude that SET acted without or in excess of jurisdiction or with gadalej.
Its conclusions are in keeping with a faithful and exhaustive reading of the Constitution.
Confronted with a novel legal question in the quo warranto petition, the citizenship of a foundling,
without judicial precedent, the SET was only asked to make a reasonable interpretation of the law
while heedfully considering the established personal circumstances of Poe. It did not insist on Poe
proving her biological parentage which she has never been in a position to know throughout her
lifetime. It instead appreciated the implications of all other facts known about her. It arrived at
conclusions in a manner in keeping with the degree of proof required in quasi-judicial proceedings-
substantial evidence.

2. David claims that as a foundling whose parents are unknown, Poe fails to satisfy the jus
sanguinis principle, failing to establish her Filipino bloodline. At the heart of the controversy is a
constitutional ambiguity. Resolving the issue hinges on constitutional interpretation in discovering
the sovereign’s purpose so as to identify which among competing interpretations of the same text
is the more contemporarily viable construction.

To the extent possible, words must be given their ordinary meaning (verba legis). The constitution
is a public document in that it was ratified by a direct act of the people thru plebiscite. The
preeminent consideration in reading the constitution therefore is the people’s consciousness, that
is, popular rather than technical-legal understanding. The constitution should also be read as
a whole- ut magis valeat quam pereat. Art.IV, S1 may also be compared with counterpart
provisions not only in earlier provisions but even in organic laws and in similar mechanisms
introduced by colonial rulers. Contemporaneous construction and aids external to the text may be
resorted to when the text is capable of multiple meanings. One may also consider analogous
jurisprudence, that is, judicial decisions on similar, but not the very same, matters or concerns, and
similar international norms.

In the hierarchy of the means for constitutional interpretation, inferring meaning from the supposed
intent of the framers is the weakest approach. This leaves the greatest room for subjective
interpretation and allows for the greatest errors. The alleged intent of the framers is not necessarily
exhaustively articulated in the records of deliberations. Those that were silent may have reasons
entirely of their own. It is also possible that the beliefs that motivated them were based on
erroneous premises. Also, the original intent of the framers is not always uniform with the original
understanding of the people who ratified it.

3. Though her parents are unknown, Poe is a PH citizen without the need for an express statement
in the Constitution making her so. Her status as such is but the logical consequence of a
reasonable reading of the Constitution within its plain text. There is not even a need to delve
into the deliberations of its framers and the implications of international legal instruments.
On an initial level reading, a plain textual reading readily identifies the specific provision which
principally governs: the Constitution’s actual definition in Art.IV, S2 of “natural-born citizens.”
This must be harmonized with S1’s enumeration, which includes a reference to parentage. The
pieces of evidence before the SET, admitted facts and uncontroverted circumstances
adequately justify the conclusion of Poe’s Filipino parentage.

On another level of reading, the assumption should be that foundlings are natural-born unless
there is substantial evidence to the contrary. This is engendered by a complete consideration
of the whole constitution, not just its provisions on citizenship. This includes the mandate of
defending the well-being of children, guaranteeing equal protection, equal access to opportunities
for public service, and respecting human rights, as well as reasons for requiring natural-born status
for some public offices. This is also a reading validated by contemporaneous construction.

4. Citizenship is a legal device denoting political affiliation. It is the right to have rights. It is
one’s personal and permanent membership in a political community. Its core is the capacity
to enjoy political rights (to participate in government thru the right to vote, to hold public office,
and to petition the government for redress of grievance). It also entails obligations, particularly
loyalty.

After the Treaty of Paris, Philippine Bill of 1902, and Philippine Autonomy Act of 1916, the 1935
Constitution was the first to make reference to parentage in determining citizenship. Art.III, S1
thereof read:
SECTION 1. The following are citizens of the Philippines: (1) Those who are citizens of
the Philippine Islands at the time of the adoption of this Constitution (2) Those born in the
Philippines Islands of foreign parents who, before the adoption of this Constitution, had
been elected to public office in the Philippine Islands. (3) Those whose fathers are citizens
of the Philippines. (4) Those whose mothers are citizens of the Philippines and upon
reaching the age of majority, elect Philippine citizenship. (5) Those who are naturalized in
accordance with law.
The term “natural-born citizen” first appeared in the 1935 constitution, ArtVII, S3 in stating the
qualifications for president and vice president, but the constitution did not define the term. Art.III,
S1(4) of the 1935 constitution, read with the then civil law provision on the automatic loss of
Filipino citizenship by women who marry alien husbands, was discriminatory towards women.
The 1973 Constitution rectified this:
SECTION 1. The following are citizens of the Philippines: (1) Those who are citizens of
the Philippines at the time of the adoption of this Constitution. (2) Those whose fathers or
mothers are citizens of the Philippines. (3) Those who elect Philippine citizenship pursuant
to the provisions of the Constitution of nineteen hundred and thirty-five. (4) Those who are
naturalized in accordance with law.
SECTION 2. A female citizen of the Philippines who marries an alien shall retain her
Philippine citizenship, unless by her act or omission she is deemed, under the law, to have
renounced her citizenship.
It was the first document to define “natural-born citizen” in Art.III, S4. “SECTION 4. A natural-
born citizen is one who is a citizen of the Philippines from birth without having to perform any
act to acquire or perfect his Philippine citizenship.”
The 1987 Constitution adopted most of the provisions of the 1973 Constitution on citizenship
except (3) thereof to correct the irregular situation generated by the questionable proviso in the
1935 constitution.

Art. IV, S1 enumerates who are citizens of PH; S2 defines natural-born citizens, calibrating the
1973 constitution’s previous definition:
Section 1. The following are citizens of the Philippines: (1) Those who are citizens of the
Philippines at the time of the adoption of this Constitution; (2) Those whose fathers or
mothers are citizens of the Philippines; (3) Those born before January 17, 1973, of Filipino
mothers, who elect Philippine citizenship upon reaching the age of majority; and (4) Those
who are naturalized in accordance with law.
Sec. 2. Natural-born citizens are those who are citizens of the Philippines from birth
without having to perform any act to acquire or perfect their Philippine citizenship.
Those who elect Philippine citizenship in accordance with paragraph (3), Section 1 hereof
shall be deemed natural-born citizens.

It has been suggested that the requirement of being natural-born was introduced as a safeguard
against foreign infiltration in the administration of national government.

Now, there are only two categories of Filipino citizens- natural-born and naturalized. By
implication of the definition in Art.IV, S2 of natural-born, naturalized citizens are those who are
not natural-born. Justice Panganiban in his concurring opinion in Bengson v. HRET that
naturalized citizens are former aliens who had to undergo a rigid procedure in which they had to
adduce sufficient evidence to prove that they possessed all the qualifications and none of the
disqualifications provided by law in order to become Filipino citizens.

David’s reliance on S1 and the need to establish bloodline is misplaced, inordinately selective, and
myopic. It turns a blind eye to the succeeding section’s definition. Between Art. IV, S1(2), which
he harps on, and S2, it is S2 that is on point. To determine if Poe is a natural-born, we must look
into whether she had to do anything to perfect her citizenship. This calls for an inquiry into
whether she underwent the naturalization process to become Filipino. She did not.

Natural-born citizenship is not concerned with being a human thoroughbred. S1(2) stipulates
that to be a citizen, either one’s father or mother must be a Filipino citizen. That’s all there is to it.
Physical features, genetics, pedigree, and ethnicity are not determinative of citizenship. It does
not require one’s parents to be natural-born Filipinos. It does not even require them to conform
to traditional conceptions of what is indigenously or ethnically Filipino. One or both parents
can thus be ethnically foreign. It requires only one ascendant degree: parentage. The citizenship
of everyone else in one’s ancestry is irrelevant. There is no need for a pure Filipino bloodline.
S1(2) requires citizenship, not identity. A conclusion of Filipino citizenship may be sustained by
evidence adduced in a proper proceeding which substantially proves that either of one’s parents is
a Filipino citizen. Poe has done this. Her evidence show that at least one, if not both, of her
biological parents were Filipino citizens.

Proving Poe’s biological parentage is practically impossible now. She was abandoned as a
newborn infant almost half a century ago. But our evidentiary rules admit of circumstantial
evidence in lieu of direct evidence. Although Rule 133, S4 mentions the circumstantial evidence
requirements (more than 1 circ, etc.) in reference to criminal proceedings, this Court has sustained
the use of circumstantial evidence in other proceedings. There is no rational basis to exclusively
use circumstantial evidence for criminal proceedings only and not in civil or administrative ones
as valid means of proof. If circumstantial evidence suffices for such a high standard of beyond
reasonable doubt, so too may it suffice to satisfy the less stringent standard of proof in
administrative and quasi-judicial proceedings like those in SET- substantial evidence.

Poe was found as a newborn infant outside a parish church in Iloilo. In 1968, Iloilo, as most PH
provinces, had a predominantly Filipino population. Poe is described as having “brown almond-
shaped eyes, a low nasal bridge, straight black hair, and an oval-shaped face.” She stands 5ft. 2in
tall. Also, in 1968, there was no international airport in Jaro, Iloilo. These circumstances are
substantial evidence justifying an inference that her biological parents were Filipino. Her
abandonment at a Catholic Church is consistent with how a Filipino who, in 1968, lived in a
predominantly religious and catholic environment, would have behaved. Though proof of ethnicity
is unnecessary, her physical features nonetheless attest to it. Also, out of the 900165 recorded
births in the PH in 1968, only 1595 or 0,.18% newborns were foreigners. There is 99.8%
probability that Poe was born a Filipino citizen.

5. David’s claim that the burden to prove natural-born status shifted to Poe upon showing that she
is a foundling is a serious error. This would make a presumption that all newborns abandoned in
rural areas in PH are not Filipinos and places an impossible condition. To require proof from Poe
borders on the absurd where there is no dispute that the identity of her biological parents is simply
not known. In a petition for quo warranto, the burden of proof necessarily falls on the party who
brings the action. Once he makes a prima facie case, the burden of evidence shifts to the
respondent. Poe’s admitted status as foundling does not establish a prima facie case in favor of
David. It merely establishes that the identities of her biological parents are unknown, not that
neither her father nor mother is a Filipino. The most that David had in his favor was a taint of
doubt, which is not substantial evidence establishing a prima facie case to shift the burden of
evidence.

6. Apart from how Poe is a natural-born Filipino consistent with a reading harmonizing Art. IV,
S2 and S1(2)’s reference to parentage, the Constitution sustains a presumption that all
foundlings found in PH are born to at least a Filipino father or mother and are thus natural-
born, unless there is substantial proof otherwise. Such proof must show that both, not just one, of
the foundling’s biological parents are not Filipinos consistent with S1(2).

The presumption that all foundlings found in PH are born to at least either a Filipino mother or
father arises when one reads the constitution as a whole. Art.II, S13 and Art.XV, S3 require the
state to enhance children’s well-being and to protect them from conditions prejudicial to or
that may undermine their development. Fulfilling this mandate includes preventing
discriminatory conditions.

Certain crucial government offices are exclusive to natural-born citizens of PH: (1) President; (2)
Vice-President; (3) Senator; (4) Member of the House of Representatives; (5) Member of the
Supreme Court or any lower collegiate court; (6) Chairperson and Commissioners of the Civil
Service Commission; (7) Chairperson and Commissioners of the Commission on Elections; (8)
Chairperson and Commissioners of the Commission on Audit; (9) Ombudsman and his or her
deputies; (10) Board of Governors of the Bangko Sentral ng Pilipinas; and (11) Chairperson and
Members of the Commission on Human Rights. Others include city fiscals, justices of
Sandiganbayan, etc. Certain professions are also limited to natural-born citizens.

Concluding that foundlings are not natural-born Filipinos is tantamount to permanently


discriminating against our foundling citizens. They can then never be of service to the country
in the highest possible capabilities. It also excludes them from certain means like professions and
state scholarships. These cannot be tolerated by the Constitution. The constitution also guarantees
equal protection (AIII, S1) and equal access to opportunities for public service (AII, S26, AXIII,
S1).

Other than the anonymity of their biological parents, no substantial distinction differentiates
foundlings from children with known Filipino parents. They are both entitled to the full extent of
the state’s protection.

The presumption is also validated by a parallel consideration or contemporaneous construction of


the constitution with acts of Congress, international instruments in force in PH, and acts of the
executive organs like BI and the president of PH. Congress has enacted laws founded on the
premise that foundlings are Filipino at birth (RA 9344, Juvenile Justice and Welfare Act).
Consistent with RA 9344 is our ratification of the UN Convention on the Rights of the Child,
requiring states-parties’ protection of children’s rights to immediate registration and nationality
after birth, against statelessness, and against discrimination on account of their birth status. The
ICCPR also requires that children be allowed immediate registration after birth and to acquire a
nationality. It also defends them against discrimination.

After ratification by the Senate of a treaty, no further action, legislative or otherwise, is necessary.
The government is bound to abide by the treaty consistent with pacta sunt servanda. Thus,
foundlings cannot be the object of discrimination.

RA 8552, Domestic Adoption Act, refers to foundlings as among the “Filipino children” covered
therein. RA 8043, Inter-Country Adoption Act, expressly includes among “Filipino children” who
may be adopted those with “foundling certificate.” Foundling certificates may be presented in lieu
of authenticated birth certificates for issuance of passports to facilitate adoption by foreigners. Our
statutes on adoption allow for recognition of foundlings’ Filipino citizen on account of their
birth.

Her status as natural-born was never questioned until she was a viable candidate for president. She
was issued a foundling certificate and benefited from the domestic adoption process, granted an
order of reacquisition of natural-born citizenship under RA 9225 by BI, and the president
appointed her as MTRCB chairperson- an office requiring natural-born citizenship.

7. “PH citizenship may be lost or reqacquired in the manner provided by law.” CA 63, in effect
when Poe was naturalized as American citizen on Oct. 18, 2001, provided in S1(1) that a Filipino
may lose citizenship by naturalization in a foreign country. Thus, Poe then lost her PH citizenship
when naturalized as American. But on July 7, 2006, she took her oath of allegiance to PH under
S3 of RA 9225. She then filed with BID a petition for reacquisition of PH citizenship, which was
granted.

RA 9225 superseded CA 63 and RA 8171 “to do away with CA 63 which takes away PH
citizenship from natural-born Filipinos who become naturalized citizens of other countries.” The
citizenship regime in RA 9225 is designed, in its own words, to ensure that “PH citizens who
become citizens of another country shall be deemed not to have lost their PH citizenship.” What
RA 9225 does is allow dual citizenship to natural-born Filipinos who lost PH citizen due to
naturalization as citizens of a foreign country. It made natural-born status permanent and
immutable despite naturalization as citizens of other countries.

S3 thereof reads:
SEC. 3. Retention of Philippine Citizenship. — Any provision of law to the contrary
notwithstanding, natural-born citizens of the Philippines who have lost their Philippine
citizenship by reason of their naturalization as citizens of a foreign country are hereby
deemed to have reacquired Philippine citizenship upon taking the following oath of
allegiance to the Republic: xxx
Natural-born citizens of the Philippines who, after the effectivity of this Act, become
citizens of a foreign country shall retain their Philippine citizenship upon taking the
aforesaid oath.

Natural-borns who, after RA 9225 took effect, are naturalized in foreign countries retain, that is,
keep, their PH citizenship although subject to certain solemnities. On the other hand, those who
became citizens of another country before the effectivity of RA 9225 reacquire their PH
citizenship upon compliance with certain solemnities. This reacquisition works to restore natural-
born status as though it was never lost at all.

Taking the oath of allegiance effects the retention or reacquisition of natural-born citizenship. It
also facilitates enjoyment of civil and political rights. But other conditions must be met for the
exercise of other faculties like in S5. Any natural-born Filipino naturalized elsewhere and wish to
run for elective public office must comply with the ff: 1) taking the oath of allegiance to the
Republic. 2) Compliance with Art.V, S1, RA 9189, Overseas Absentee Voting Act of 2003, to
facilitate exercise of right of suffrage. 3) Making a personal and sworn renunciation of any and
all foreign citizenship before any public officer authorized to administer an oath. Along with
satisfying other qualification requirements under relevant laws, makes one eligible for elective
public office.

Poe has complied with all these. She took her oath, she became a registered voter of Barangay
Santa Lucia, San Juan (complying with Art.V, S1). Since she was to vote within the country, this
dispensed with the need to comply with RA 9189. Lastly, she executed an affidavit of
renunciation of allegiance to US and renunciation of American Citizenship. Thus, she has fully
reacquired natural-born citizenship and complied with all other requirements for eligibility
to elective public office as stipulated in RA 9225.

238. Antonio Bengson III v. HRET, GR 142840, May 07, 2001, Kapunan, J. (Citizenship)
FACTS:
The citizenship of Teodoro Cruz is in issue due to the constitutional requirement that HoR
members must be a natural-born citizen. Cruz was a natural-born citizen of PH born in Tarlac of
Filipino parents on April 27, 1960, during the 1935 Constitution. On Nov. 5, 1985, he enlisted in
the US Marine Corps and without the consent of PH, took an oath of allegiance to US. He thus
lost his PH citizenship under CA 63 by rendering service in the armed forces of a foreign country.
He was naturalized as a US citizen on June 5, 1990.

On March 17, 1994, Cruz reaquired his PH citizenship thru repatriation under RA 2630. He was
elected representative of the 2nd district of Pangasinan. He won by 26,671 votes against Antonio
Bengson. Bengson thus filed a quo warranto with HRET against Cruz, claiming that Cruz is not
qualified to be a HoR member since he is not a natural-born citizen.

The HRET dismissed the petition. Hence this certiorari.

ISSUE:
Whether Teodoro, who was born in PH to Filipino parents, lost his citizenship because he enlisted
in the US Marine Corps without PH’s consent and took the oath of allegiance to US, and reacquired
his citizenship thru repatriation, is a natural-born citizen.
HELD: YES.
There are two ways of acquiring citizenship: 1) by birth and 2) by naturalization. These
correspond to 2 kinds of citizens: natural-born and naturalized. Natural-born citizens are those
“citizens of the PH from birth without having to perform any act to acquire or perfect the PH
citizenship.” Naturalized citizens are those who become Filipino thru naturalization, generally
under CA 473, Revised Naturalization Law, which repealed Act 2927. To be naturalized, an
applicant has to prove that he possesses all the qualifications and none of the disqualifications
provided by law to become a Filipino citizen. The decision granting PH citizenship becomes
executory only after 2 years from its promulgation when the court is satisfied that during the
intervening period, the applicant has 1) not left the PH, 2) has dedicated himself to a lawful
calling or profession, 3) has not been convicted of any offense or violation of government
promulgated rules, or 4) committed any act prejudicial to the interest of the nation or
contrary to any government announced policies.

Filipino citizens who lost citizenship may reacquire the same in the manner provided by law. CA
63 enumerates the three modes by which PH citizenship may be reacquired: 1) by
naturalization, 2) by repatriation, and 3) by direct act of Congress.

Naturalization is a mode of both acquisition and reacquisition of PH citizenship. As a mode of


initially acquiring PH citizenship, naturalization is governed by CA 473. Naturalization as a mode
of reacquiring PH citizenship is governed by CA 63.

Repatriation may be had under various statutes by those who lost their citizenship due to 1)
desertion of the armed forces, 2) service in the armed forces of the allied forces in WW2, 3)
service in the armed forces of the US at any other time, 4) marriage of a Filipino woman to
an alien, and 5) political and economic necessity. Repatriation simply consists of taking an oath
of allegiance to PH and registering said oath in the local civil registry where the person resides
or last resided. He would not even need to file a petition in court.

Repatriation results in the recovery of the original nationality. Here, Cruz lost his Filipino
citizenship when he rendered service in the US armed forces. But he reacquired PH citizenship
under RA 2630. Having taken the required oath and registering the same in the civil registry in
Pangasinan, Cruz is deemed to have recovered his original status as a natural-born citizen.

The contention that Cruz is no longer natural-born as he had to perform an act to regain his
citizenship is untenable. There are two requisites in S4, Art.III of the 1973 Constitution for a person
to be natural-born: 1) a person must be a Filipino citizen from birth and 2) he does not have to
perform any act to obtain or perfect his PH citizenship. In the 1973 constitution definition,
those naturalized and those born before January 17, 1973 of Filipino mothers who elect PH
citizenship upon reaching the age of majority are not natural-born. The latter, because they had to
perform an act, electing PH citizenship, to perfect their PH citizenship.

But the 1987 Constitution now considers those born of Filipino mothers before the 1973
Constitution and who elected PH citizenship as natural-born. S2, Art. IV adds a sentence: “Those
who elect PH citizenship in accordance with par.(3), S1 hereof shall be deemed natural-born
citizens.” Thus, only naturalized Filipinos are not natural-born. There are now only two classes
of citizens: natural-born or naturalized.

The absence in the enumeration of a separate category for persons who reacquire citizenship is
because such person would either be natural-born or naturalized depending on the reasons for the
loss of citizenship and the mode prescribed by law for reacquisition. As Cruz was not required
by law to go thru naturalization proceedings to reacquire his citizenship, he is natural-born.

239. Grace Poe-Llamanzares v. COMELEC, GR 221697, March 08, 2016, Perez, J.


(Citizenship)
FACTS:
(Life story ulet ni Grace Poe)
On July 7, 2006, Poe took her oath of allegiance to PH pursuant to RA 9225 or the Citizenship
Retention and Reacquisition Act of 2003. She registered as voter in Brgy. Santa Lucia, San Juan
City and acquired a new PH passport from DFA. Pres. Noynoy appointed her as MTRCB
chairperson. Before assuming her post, she executed an Affidavit of Renunciation of Allegiance
to the US of America and Renunciation of American Citizenship in satisfaction of the legal
requisites in S5, RA 9225. She took her oath of office as MTRCB chairperson. From then on, Poe
stopped using her American passport.

She executed before the vice consul of the US embassy an Affirmation of Renunciation of
Nationality of US and accomplished a sworn questionnaire before the vice consul. The vice consul
issued her a Certificate of Loss of Nationality of US effective October 21, 2010.

On October 2, 2012, Poe filed with COMELEC her COC for senator for the 2013 elections wherein
she answered “6years and 6months” to the question “Period of residence in PH before May 13,
2013.” She won as senator and obtained a PH diplomatic passport.
On October 15, 2015, Poe filed her COC for the presidency for the May 2016 elections. In her
COC, she declared that she is a natural-born citizen and that her residence in PH up to the day
before May 9, 2016 would be “10years 11months counted from May 24, 2005.” This filing of
COC triggered the filing of several COMELEC cases against her subject of these cases.

It is contended that Poe is not a natural-born Filipino as she was a foundling. Also, she is bound
by her declaration in her 2012 COC for senator where she said that she resided in the country for
only 6y6m as of May 2013. It is claimed that assuming arguendo that Poe is qualified to regain
natural-born status under RA 9225, she still fell short of the 10 year residency requirement as her
residence could only be counted at the earliest from July 2006, when she reacquired PH
citizenship under RA 9225. She also failed to reestablish her domicile in PH.

Poe counters that she could reestablish residence even before she reacquired natural-born
citizenship under RA 9225 and foundlings are presumed under international law to be born of
citizens of the place where they are found.

The COMELEC cancelled Poe’s COC, finding that her COC for the 2016 elections had material
representations which are false. Hence this certiorari petition.

ISSUE:
Whether Grace Poe’s declaration in her 2012 COC for Senator that her residence in PH was only
6y6m may bind her in her 2015 COC when she falsely believed that what was asked to be stated
was residence up to the date of filing and not to the date of election.
HELD: NO.
The issue before COMELEC is whether Poe’s COC should be cancelled on the exclusive ground
that she made a false material representation. The exclusivity of this ground should restrain
COMELEC from going into the issue of the qualifications of the candidate for the position if such
issue is yet undecided by the proper authority. The COMELEC cannot itself, in the same
cancellation case, decide the qualification or lack thereof of the candidate.

Not any one of COMELEC’s enumerated powers in Art. IX-C, S2 approximate the exactitude of
Art.VI, S17 on the powers of HRET and SET or the last paragraph of Art.VII, S4 on the PET.
Disqualification proceedings are for the purpose of barring an individual from becoming a
candidate or from continuing as a candidate for public office- to eliminate a candidate from the
race. “Ineligibility”, on the other hand, refers to the lack of qualifications prescribed in the
Constitution for holding public office and the purpose of the proceedings for declaration of
ineligibility is to remove an incumbent from office. Unless a candidate wins and is elected, there
is no necessity for determining his eligibility for office. In contrast, whether an individual should
be disqualified as a candidate for acts constituting election offenses (vote buying etc.) is a
prejudicial question which should be determined lest he wins because of the very acts for which
his disqualification is being sought.

Poe’s blood relationship with a Filipino citizen is demonstrable. There is more than sufficient
evidence that Poe has Filipino parents and is a natural-born Filipino. The burden of proof was on
private respondents. Her admission that she is a foundling did not shift the burden to her as this
did not exclude the possibility that her parents were Filipinos, especially here where there is a high
probability, if not certainty, that her parents are Filipinos.

The factual issue is not who Poe’s parents are, as their identities are unknown, but whether such
parents are Filipinos. SOLGEN offered statistics that from 1965-1975, the number of aliens born
in PH was 15,986 while the number of Filipinos born in the country was 10,558,278. When Poe
was found in 1968, the majority of the population in Iloilo was Filipino. Other circumstantial
evidence of Poe’s nationality are the fact that she was abandoned as an infant in a Catholic Church
in Iloilo. She has typical Filipino features. Thus, there would be more than a 99% chance that a
child born in the province would be a Filipino. To assume otherwise would be to accept the
virtually impossible.

As a matter of law, foundlings are as a class natural-born citizens. The deliberations of the 1934
Constitutional Convention show that the framers intended foundlings to be covered by the
enumeration. The amendment covering this was not put into the Constitution because the
convention believed that the cases of foundlings, being too few to warrant the inclusion of a
provision in the Constitution to apply to them, should be governed by statutory legislation. The
“Rafols amendment” was rejected not because it was declined but because it was felt that there is
no more need to expressly declare foundlings as Filipinos.

There is nothing in the records of the 1935, 1973, and 1987 Constitution showing that the framers
permitted discrimination against foundlings. Contrarily, all 3 constitutions guarantee the basic
right to equal protection. In the present charter, Art.II, S11, Art. XIII, S1, and Art. XV, S3
contradict such intent. Also, RA 8043 (inter-country adoption), RA 8552 (domestic adoption), and
the SC’s Rule on Adoption all expressly refer to “Filipino children” and include foundlings as
among Filipino children who may be adopted.

It is argued that the process to determine that the child is a foundling leading to issuance of a
foundling certificate under these laws are acts to acquire PH citizenship which make the foundling
naturalized at best. This is wrong. Under Art. IV, S2, Natural-born citizens are those who are
citizens from birth without having to perform any act to acquire their PH citizenship. “Having
to perform an act” means that the act must be personally done by the citizen. Here, the
determination of foundling status is done not by the child but by authorities. Second, the object of
the process is the determination of the whereabouts of the parents, not the citizenship of the child.
Lastly, the process is not analogous to naturalization proceedings to acquire PH citizenship or the
election of such citizenship by one born of an alien father and Filipino mother under the 1935
Constitution, which is an act to perfect it. Here, the issue is moot as there is no dispute that Poe
is a foundling as evidenced by her foundling certificate.

Foundlings are also citizens under international law. Art. 15 of UDHR, part of the generally
accepted principles of international law, provides that everyone has the right to a nationality. The
UN Convention on the Rights of the Child (UNCRC), Art. 7, also states that the child shall have
the right to acquire a nationality. The ICCPR also provides for the right of every child to acquire
a nationality. The common thread of these is to obligate PH to grant nationality from birth and
ensure that no child is stateless.
At least 60 countries in Asia, North and South America, and Europe have passed legislation
recognizing foundlings as its citizen. 42 of those follow jus sanguinis. These show that it is a
generally accepted principle of international law to presume foundlings as having been born of
nationals of the country in which the foundling is found. Current legislation adheres to this
generally accepted principle. RA 8552, RA 8042, and the SC’s Rule on Adoption refer to “Filipino
children.” Foundlings are among the Filipino children who could be adopted. The DFA also issues
passports to foundlings. Passports are by law issued only to citizens. Thus, even the executive
department, acting through the DFA, considers foundlings as PH citizens.

The COMELEC also ruled that Poe’s repatriation in July 2006 under RA 9225 did not result in
reacquisition of natural-born citizenship since she must perform an act. This disregards
jurisprudence and RA 9225. In Bengson III v. HRET, repatriation was said to result in recovery of
the original nationality.

RA 9225 is a repatriation statute. Congress saw it fit to decree that natural-born citizenship may
be reacquired even if once lost. It is not for the COMELEC to disagree with Congress’
determination. In Bengson, “from birth” was clarified to mean at the time of birth, and not that
natural-born status must be continuous. Neither is repatriation an act to acquire or perfect one’s
citizenship. There are only 2 types of citizens and there is no third category for repatriated citizens.

As to the falsehood Poe allegedly committed when she put in the spaces for “born to” in her
application for repatriation under RA 9225 the names of her adoptive parents, misleading BI to
presume that she was natural-born, it is contended that the date required were the names of her
biological parents which are unknown.

This disregards the fact that Poe was legally adopted. One of the effects of adoption is to sever
all legal ties between the biological parents and the adoptee except when the biological parent
is the spouse of the adoptee. RA 8552 entitles Poe to an amended birth cert attesting to the fact
that the adoptee is the child of the adopters without notation that it is an amended issue. The law
thus allows Poe to state that her adoptive parents were her birth parents as that was what would be
stated in her birth cert anyway.

- As to the alleged false material representation when she stated in her COC that she has
before and until May 9, 2015 been a resident of PH for 10y11m, Poe’s claim is true. The
Constitution requires presidential candidates to have 10 years residence in PH before the day of
elections. She states that she had been a resident since May 25, 2005 when she returned for good
from the US.

When Poe immigrated to US in 1991, she lost her original domicile in PH. There are 3 requisites
to acquire a new domicile: 1) residence or bodily presence in a new locality, 2) an intention to
remain there, and 3) an intention to abandon the old domicile. To effect a change of domicile,
one must demonstrate an actual removal or change of domicile; a bona fide intention of
abandoning the former place of residence and establishing a new one and definite acts which
correspond with the purpose. There must basically be animus manendi coupled with animus non
revertendi.
Poe showed voluminous evidence showing that she and her family abandoned their US domicile
and relocated to PH for good. These evidence include Poe’s former US passport showing her
arrival on May 24, 2005 and her return to PH every time she travelled abroad; email
correspondence from March 2005 to September 2006 with a freight company to arrange for the
shipment of their household items to PH; documents showing her children’s enrollment in PH
schools, etc.

The COMELEC contended that there was no animus non-revertendi, disregarding the import of
all of Poe’s evidence based on the supposition that the earliest date that Poe could start residence
in PH was in July 2006 when her application under RA 9225 was approved by BI. It contended
that the stay of an alien former Filipinos cannot be counted until he obtains a permanent resident
visa or reacquires PH citizenship.

The cases cited by COMELEC had sparse evidence, leaving the Court no choice to hold that
residence could be counted only from acquisition of a permanent resident visa or from
reacquisition of PH citizenship. In contrast, Poe’s evidence is overwhelming and taken together
leads to no other conclusion that she decided to permanently abandon her US residence.
Coupled with her eventual application to reacquire PH citizenship and her family’s actual
continuous stay in PH over the years, it is clear that when Poe returned on May 24, 2005, it was
for good.

Petitioner’s evidence of residence is unprecedented by judicial precedent. There is no case yet that
comes close to the facts of residence of Poe.

The COMELEC ruled that Poe’s claim of residence of 10y11m in her 2015 COC was false as she
put 6y6m as period of residence in her 2012 COC. COMELEC automatically assumed as true
the statement in the 2012 COC and the 2015 COC as false. As explained by Poe, she misunderstood
the date required in the 2013 COC as the period of residence as of the day she submitted the 2012
COC. She reckoned residency from April-May 2006 when her husband returned to PH. It was in
2015 that her lawyers advised her that residence could be counted from May 25, 2005.

It is the fact of residence, not a statement in a COC which ought to be decisive in determining
if an individual has satisfied the constitution’s residency qualification requirement.

COMELEC disregarded the evidence that Poe actually and physically returned on May 24, 2005
because it took the position that domicile could be established only from Poe’s repatriation under
RA 9225 in July 2006. But it does not take away the fact that in reality, Poe had returned from
the US and was here to stay permanently, on May 24, 2005.

Thus, COMELEC gravely abused its discretion.

240. Maria Tecson v. COMELEC, GR 161434, March 3, 2004, Vitug, J. (Citizenship)


FACTS:
FPJ filed his COC for president under the KNP Party. He represented himself in the COC as
natural-born PH citizen. A petition was filed by Victorino Fornier to disqualify FPJ and to cancel
his COC on the ground that FPJ made a material representation in his COC by claiming to be
natural-born when his parents are foreigners. His mother, Bessie Poe, was an American, and his
father, Allan Poe, was Spanish. The COMELEC denied this, hence this petition under Rule 65.
Other petitions consolidated with his petition include Maria Tecson v. COMELEC, etc.

ISSUE:
Whether FPJ is a natural-born citizen.
HELD: YES.
1. Under Art. VII, S4, par7, the SC shall be the sole judge of all contests relating to the election
etc. of the president or vice-president. “Contest” refers to a post-election scenario. Election
contests consist of either an election protest or quo warranto which, though distinct remedies, have
one objective- to dislodge the winning candidate from office. The rules of the PET speak of the
jurisdiction of PET over contests relating to the election, returns, and qualifications of the
“president” or “vice-president,” not of “candidates” for such. The election contest can only
contemplate a post-election scenario. Thus, the petitions of Tecson and Velez would have to be
dismissed for lack of jurisdiction.

As to Fornier’s petition, he invoked S78 of the OEC to cancel or deny due course to a COC in
consonance with COMELEC’s general powers in S52 of OEC. Decisions of the COMELEC on
disqualification cases may be reviewed in an action for certiorari under Rule 65. Also, Art. VIII,
S1 provides that the SC can review GADALEJ. Thus, the SC has jurisdiction.

Treaty of Paris- Citizenship of PH islands subject to US legislation.


April 11, 1899-July 1, 1902- Jus Soli.
PH Bill of 1902- “Philippine citizens” was first used. A citizen of the PH was one who was an
inhabitant of PH and a Spanish subject on April 11, 1899. “Inhabitant” included 1) a native-born
inhabitant, 2) an inhabitant who was a native of peninsular Spain, and 3) an inhabitant who
obtained Spanish papers on or before April 11, 1899.
PH Autonomy Act/Jones Law (1916)- Native-born inhabitant is a citizen of PH if 1) subject of
Spain on April 11, 1899, 2) Residing in PH on said date, 3) since that date, not a citizen of some
other country.
1935 Consti, 1973, 1987.

FPJ was born on August 20, 1939 during the 1935 Constitution. Through its history, four modes
of acquiring citizenship- naturalization, jus soli, res judicata, and jus sanguinis- had been in
vogue. Only two- jus soli and jus sanguinis, could qualify a person to be natural-born. Jus soli did
not last long with the adoption of the 1935 constitution and Roa v. Collector of Customs in 1912.
Jus sanguinis or blood relationship is now the primary basis of citizenship by birth.

2. FPJ’s earliest established direct ascent was his paternal grandfather Lorenzo Pou. While his
birth record was not presented, his death cert identified him to be a Filipino and 84 years old at the
time of death on September 11, 1954. Allan, FPJ’s father, in his birth cert is shown to be born on
May 17, 1915 to an español father Lorenzo and mestiza mother Marta. The marriage certificate,
dated September 16, 1940, of Allan and Bessie showed that Allan, stated to be 25 yo, was a Filipino
citizen, and Bessie an American citizen. FPJ’s birth cert shows that he was born on August 20,
1939 to Allan, a Filipino, 24 yo, married to Bessie.
Considering some reservations made by the parties on the veracity of some entries on the birth
certs of FPJ and marriage cert of his parents, the only conclusions that may be drawn with some
certainty is that 1) FPJ’s parents were Allan Poe and Bessie Kelley; 2) FPJ was born on August
20, 1939; 3) Allan and Bessie married on September 16, 1940; 4) the father of Allan was Lorenzo;
and 5) Lorenzo was 84yo at the time of his death on September 11, 1954. Are these sufficient to
establish that FPJ is natural-born? Being public documents, the death, birth, and marriage certs
constitute prima facie proof of their contents.

Thus, it could be assumed that Lorenzo was born sometime in 1870 when PH was still a colony of
Spain. It is argued that Lorenzo was not in PH during the crucial period from 1898 to 1902 since
there is no record of such fact, but petitioner failed to show that Lorenzo was at any other place
during the same period. The death cert of Lorenzo stated his residence to be in Pangasinan. Without
contrary evidence, it should be sound to presume that the place of residence of a person at the
time of his death was also his residence before death. Thus, Lorenzo had benefited from the
“en masse Filipinization” that the Philippine Bill had effected in 1902.

3. Petitioner claims that in establishing filiation or paternity of an illegitimate child, FPJ being
evidently illegitimate, the rules under civil law must be used.

Under Spain’s civil code in force from December 8, 1889 until August 29, 1950 when the PH civil
code took effect, acknowledgment was required to establish filiation or paternity.
Acknowledgment was either judicial (compulsory) or voluntary. Judicial acknowledgment was
possible only if done during the lifetime of the putative parent. Voluntary acknowledgment could
only be done in a record of birth, a will, or public document. To let the birth cert be used to prove
voluntary acknowledgment, the cert must be signed or sworn to by the father. FPJ’s birth cert has
no signature of Allan. The only other proof was voluntary recognition in some other public
document.

The 1950 Civil Code categorized acknowledgment of illegitimate children into voluntary, legal,
or compulsory.

The FC has further liberalized the rules in Arts. 172, 173, and 175. The FC is retroactively applied
as stated in Art.256 insofar as it does not prejudice vested rights.

The growing trend to liberalize acknowledgment or recognition of illegitimate children is an


attempt to break away from the traditional idea of keeping apart legitimate and illegitimate
relationships within the family in favor of the greater interest of the child. The provisions are
intended to merely govern the private and personal affairs of the family. There is little to
indicate that the legitimate/illegitimate civil status of the individual would also affect his political
rights or relationship to the state.

The proof of filiation/paternity to determine citizenship should thus be deemed independent from
that prescribed for civil law purposes. The relevance of citizenship to civil law is best exemplified
in Art. 15- “"Laws relating to family rights and duties, or to the status, condition and legal capacity
of persons are binding upon citizens of the Philippines, even though living abroad." The NCC or
FC provisions on proof of filiation do not have preclusive effects on matters alien to personal
and family relations. The ordinary rules on evidence should govern.

Thus, the duly notarized declaration made by Ruby, sister of Bessie, submitted before COMELEC
may be accepted to prove the facts of Allan recognizing his own paternal relationship with
FPJ, i.e. living together with Bessie and his children, including FPJ, in one house and as one
family.

4. Petitioner argues that even if Allan was Filipino, he could not have transmitted his citizenship
to FPJ as FPJ was illegitimate. He bases his stand on Morano v. Vivo, citing Chiongbian v. de Leon
and Serra v. Republic. But the statements there supporting his thesis hinge solely on pure obiter
dicta and should fail.

When jurisprudence regarded an illegitimate child as taking after the citizenship of its mother, it
did so for the benefit of the child. It was to ensure a Filipino nationality for the illegitimate child
of an alien father in line with the assumption that the mother had custody, would exercise parental
authority, and had the duty to support her illegitimate child. It was to help the child, not to prejudice
or discriminate against him. The 1935 constitution, prevailing during FPJ’s birth, cannot be more
explicit. It provided neither conditions nor distinctions, stating that among PH citizens are “those
whose fathers are citizens of PH.” There is no justification to prescribe distinctions where there
are none.

Lorenzo’s citizenship extended to his son Allan, father of FPJ. The 1935 constitution confers
citizenship to all persons whose fathers are Filipinos regardless of whether they are legitimate or
illegitimate. While the evidence at hand may not establish conclusively that FPJ is natural-born,
the evidence would still preponderate in his favor enough to hold that he cannot be held guilty
of making a material misrepresentation in his COC.

241. Ernesto Mercado v. Eduardo Manzano, GR 135083, May 26, 1999, Mendoza, J.
(Citizenship)
FACTS:
Mercado and Manzano were candidates for Makati vice mayor in the May 11, 1998 elections.
Manzano won. The proclamation of Manzano was suspended in view of a pending petition for
disqualification filed by one Mamaril who alleged that Manzano was not a PH citizen but of the
US.

The COMELEC had cancelled the COC of Manzano stating that he is a dual citizen, and under
S40(d) of the LGC, persons with dual citizenship are disqualified for running for any elective
position. The MR remained pending even after the election on May 11 and his proclamation was
thus suspended. Mercado sought to intervene in the case for disqualification. This was not
resolved. But the COMELEC reversed its ruling and declared Manzano qualified, stating that
Manzano renounced his US citizenship when he registered as voter and voted in the 1992, 1995,
and 1998 elections. Manzano was proclaimed as winner. Hence this certiorari assailing the
COMELEC resolution declaring Manzano qualified.

ISSUE:
Whether Manzano, a dual citizen of US and PH, can hold the post of Makati vice mayor despite
S40 of the LGC disqualifying “dual citizens” from running for any elective local position.
HELD: YES.
S40 of the LGC disqualifies dual citizens from running for any elective local position. Dual
citizenship is different from dual allegiance. Dual citizenship arises when, as a result of the
concurrent application of the different laws of two or more states, a person is simultaneously
considered a national by the said states. Considering our Art. IV, it is possible for 1) those born of
Filipino fathers or mothers in foreign countries which follow jus soli, 2) those born in PH of
Filipino mothers and alien fathers if by the laws of their father’s country such children are citizens
of that country, and 3) those who marry aliens if by the laws of the alien’s country the former are
considered citizens, unless by their act or omission they are deemed to have renounced PH
citizenship to possess dual citizenship. There may be other situations.

Dual allegiance refers to a situation in which a person simultaneously owes, by some positive act,
loyalty to two or more states. While dual citizenship is involuntary, dual allegiance is the result
of an individual’s volition. Art. IV, S5 provides that “Dual allegiance of citizens is inimical to the
national interest and shall be dealt with by law.” This provision was included at the instance of
Commissioner Blas Ople who said that dual allegiance is more threatening than mere double
citizenship.

In including S5 in Art.IV on citizenship, the concern of the Constitutional commission was not
with dual citizens per se but with naturalized citizens who maintain their allegiance to their
countries of origin even after their naturalization. Thus, the phrase “dual citizenship” in RA
7160 (LGC), S40(d) and in RA 7854, S20 must be understood as referring to “DUAL
ALLEGIANCE”. Persons with mere dual citizenship do not fall under this disqualification.

Unlike those with dual allegiance, who must be subject to strict process as to the termination of
their status, for candidates with dual citizenship, it should suffice if, upon the filing of their COCs,
they elect PH citizenship to terminate their status as persons with dual citizenship considering
that their condition is the unavoidable consequence of conflicting laws of different states. By
electing PH citizenship, such candidates at the same time forswear allegiance to the other country
of which they are also citizens and thereby terminate their status as dual citizens. While it may
be that from the point of view of the foreign state and its laws such an individual has not renounced
his foreign citizenship, this is of no moment.

Manzano was born in California of Filipino parents. Since PH adheres to jus sanguinis, while US
follows jus soli, Manzano was both, at birth at least, a national of PH and US. Petitioner challenges
COMELEC’s ruling that by voting in the 1992, 1995, and 1998 elections, Manzano renounced his
US citizenship. Petitioner argues that merely taking part in the elections is not enough evidence of
renunciation and it was made when Manzano was already 37 yo when it should have been made
when he reached the age of majority.

The COMELEC must have had in mind S349 of the Immigration and Nationality Act of the US
which provided that a person who is a national of US shall lose his nationality by voting in a
political election in a foreign state. This was declared unconstitutional by the US SC as beyond
the power of Congress to regulate foreign relations. However, by filing a COC when he ran for
his present post, Manzano elected PH citizenship and in effect renounced his American
citizenship (as far as the laws of this country are concerned). (COC had “I will support and
defend the constitution of the PH and will maintain true faith and allegiance thereto xxx.”- oath of
allegiance)

There is no merit in the contention that the oath of allegiance in the COC is insufficient to constitute
renunciation of US citizenship and that the renunciation should be made upon reaching the age of
majority. No law requires election of PH citizenship upon reaching majority age.

Much is made of the fact that Manzano registered as an American citizen in the BID and that he
holds an American passport which he used in his last travel to US on April 22, 1997. There is no
merit in this. Until the filing of his COC on March 21, 1998, he had dual citizenship. His acts can
be considered simply as the assertion of his American nationality before its termination. “The
certification that he is an American does not mean that he is not still a Filipino.” There can be no
loss of PH citizenship when there is no renunciation, either express or implied.”

Manzano’s oath of allegiance, considered with the fact that he spent his youth and adulthood,
received his education, practiced his profession as an artist, and took part in past elections in this
country, leaves no doubt of his election of PH citizenship. His declarations will be taken upon the
faith that he will fulfill his undertaking made under oath. Should he betray that trust, there are
enough sanctions for declaring the loss of PH citizenship thru expatriation.

242. AASJS member Hector Calilung v. SOJ, GR 160869, May 11, 2007, Quisumbing, J.
(Citizenship)
FACTS:
Calilung filed this petition for prohibition against SOJ Datumanong, the official tasked to
implement laws governing citizenship. Calilung prays that SOJ be stopped from implementing RA
9225, “An Act Making the Citizenship of Philippine Citizens Who Acquire Foreign Citizenship
Permanent, Amending for the Purpose Commonwealth Act No. 63, As Amended, and for Other
Purposes." Calilung claims that RA 9225 is unconstitutional as it violates Art. IV, S5 on dual
allegiance of the Constitution.

ISSUE:
Whether RA 9225 is unconstitutional.
HELD: NO.
Calilung claims that S2 and 3 together allow dual allegiance and not dual citizenship. S2 allows
Filipinos who become foreign citizens to retain PH citizenship without losing their foreign
citizenship. S3 allows natural-born citizens of PH to regain PH citizenship by simply taking an
oath of allegiance without forfeiting their foreign allegiance.

From the legislative deliberations, the intent in RA 9225 is to do away with the provision in CA
63 which takes away PH citizenship from natural-born Filipinos who become naturalized
citizens of other countries. What RA 9225 does is allow dual citizenship to natural-born
Filipinos who have lost PH citizenship by reason of their naturalization as citizens of a
foreign country. On its face, it does not recognize dual allegiance. By swearing to the supreme
authority of PH, the person implicitly renounces his foreign citizenship. RA 9225, S3, stayed
clear of the problem of dual allegiance and shifted the burden of confronting the issue of whether
there is dual allegiance to the concerned foreign country. The other citizenship was not made a
concern of RA 9225.

SEC. 3. Retention of Philippine Citizenship. — Any provision of law to the contrary


notwithstanding, natural-born citizens of the Philippines who have lost their Philippine
citizenship by reason of their naturalization as citizens of a foreign country are hereby
deemed to have reacquired Philippine citizenship upon taking the following oath of
allegiance to the Republic: xxx
Natural-born citizens of the Philippines who, after the effectivity of this Act, become
citizens of a foreign country shall retain their Philippine citizenship upon taking the
aforesaid oath.

Petitioner also claims that although there is no law yet on dual allegiance, the SC could still rule
on the issue as Mercado v. Manzano already drawn up guidelines on how to distinguish dual
allegiance from dual citizenship.

S5, Art.IV of the Constitution is a declaration of policy and is not a self-executing provision.
The legislature still has to enact the law on dual allegiance. In S2 and 3 of RA 9225, the framers
were not concerned with dual citizenship per se, but with the status of naturalized citizens who
maintain their allegiance to their countries of origin even after their naturalization. Until Congress
drafts a law setting parameters for what really constitutes dual allegiance, it would be
premature for the judicial department to rule on issues pertaining to dual allegiance. Mercado did
not set parameters on dual allegiance and dual citizenship, but merely made a distinction between
them. We cannot arrogate the duty of setting the parameters of what constitutes dual allegiance
when the Constitution has delegated such duty to Congress.

243. Nestor Jacot v. Rogen Dal, GR 179848, November 27, 2008, Chico-Nazario, J.
(Citizenship)
FACTS:
Jacot was a natural-born citizen of PH who became a naturalized citizen of US on December 13,
1989. He sought to reacquire his PH citizenship under RA 9225. He filed a request for
administration of his oath of allegiance to PH with the PH Consulate General (PCG) of Los
Angeles. The PCG approved and Jacot took his oath. The BI issued a certificate recognizing Jacot
as a citizen of PH.

6 months later, Jacot filed his COC for vice mayor of Catarman, Camiguin. Dal filed for his
disqualification with COMELEC, arguing that Jacot failed to renounce his US citizenship as
required under S5(2) of RA 9225:
Section 5. Civil and Political Rights and Liabilities. — Those who retain or reacquire
Philippine citizenship under this Act shall enjoy full civil and political rights and be subject
to all attendant liabilities and responsibilities under existing laws of the Philippines and the
following conditions: xxx xxx xxx
(2) Those seeking elective public office in the Philippines shall meet the qualifications for
holding such public office as required by the Constitution and existing laws and, at the time
of the filing of the certificate of candidacy, make a personal and sworn renunciation of
any and all foreign citizenship before any public officer authorized to administer an oath.

Jacot won as vice mayor. But the COMELEC issued its resolution disqualifying him from running
for vice mayor for failure to renounce US citizenship. Hence this certiorari.

ISSUE:
Whether Jacot, a natural-born Filipino citizen who became naturalized as citizen of US and who
reacquired his Filipino citizenship under RA 9225 by taking the oath of allegiance to the PH, is
qualified to run for vice-mayor.
HELD: NO.
Jacot’s oath of allegiance to PH made before the LA PCG and his COC do not substantially comply
with the requirement of a personal and sworn renunciation of foreign citizenship because these are
distinct requirements to be complied with for different purposes.

S3 of RA 9225 requires that natural-born citizens of PH who are already naturalized citizens of a
foreign country must take the oath therein to reacquire or retain PH citizenship. By this oath, the
Filipino swears allegiance to PH, but there is nothing therein on his renunciation of foreign
citizenship. The oath in S3 is substantially similar to the one contained in the COC which must be
executed by any person wishing to run for public office in PH.

S5(2) compels natural-born Filipinos naturalized in a foreign country but who reacquired or
retained PH citizenship 1) to take the oath of allegiance under S3, RA 9225 and 2) for those
seeking elective public offices in PH, to additionally execute a personal and sworn
renunciation of any and all foreign citizenship before an authorized public officer before or
simultaneous to the filing of COCs, to qualify as candidates in PH elections. S5(2) requires
Filipinos availing themselves of the benefits in RA 9225 to accomplish an undertaking other than
that which they have presumably complied with under S3. The oath is different from the
renunciation of foreign citizenship. The intent of the legislators was not only for Filipinos
reacquiring or retaining PH citizenship to take their oath, but also to explicitly renounce foreign
citizenship if they wish to run for public office. To qualify as a candidate in PH elections, Filipinos
must only have one citizenship, PH citizenship.

Thus, the oath contained in the COC is not the personal and sworn renunciation sought. The
oath is a general requirement for all those who wish to run as candidates while renunciation of
foreign citizenship is an additional requisite only for those who have retained or reacquired PH
citizenship under RA 9225 and who seek elective public posts.

The invocation of Mercado wherein the filing of a person with dual citizenship of a COC
containing an oath was already considered as renunciation of foreign citizenship. In Mercado, the
disqualification was sought under another law, S40(d) of LGC. When the Court decided this case
on August 9, 2000, the more explicitly worded requirements of S5(2) of RA 9225 were not yet
enacted.

Jacot presents for the first time before this Court an affidavit of renunciation of allegiance to US
and any and all foreign citizenship which he allegedly executed on February 7, 2007 even before
he filed his COC on March 26, 2007. He asserts now a new theory- that he complied with the
requirement of personal and sworn renunciation. But as a rule, no question will be entertained on
appeal unless it has been raised in the proceedings below. The Court cannot countenance late
submission of evidence. Since the affidavit was not formally offered before COMELEC, and
COMELEC had no opportunity to examine and controvert it, to admit it would be contrary to due
process.

The fact that a candidate received the highest number of votes for an elective position does not
dispense with the election requirements. The will of the people thru ballot cannot cure the vice of
ineligibility, especially if they mistakenly believed that the candidate was qualified.

244. Casan Maquiling v. COMELEC, GR 195649, April 16, 2013, Sereno, C.J. (Citizenship)
FACTS:
Respondent Arnado is a natural born Filipino. He was naturalized as US citizen and lost his
Filipino citizenship. Arnado applied for repatriation under RA 9225 before the consul general of
PH in USA San Francisco and took the oath of allegiance to PH. He also executed an affidavit of
renunciation of his foreign citizenship.

Arnado filed his COC for mayor of Kauswagan, Lanao del Norte. Balua, another mayor candidate,
filed a petition to disqualify Arnado and to cancel his COC. Balua contends that Arnado is not a
resident of Kauswagan and that he is a US citizen. Balua also presented a computer-generated
travel record indicating that Arnado had been using his US Passport in entering and departing PH.
He left on April 14, 2009, returned on June 25, departed July 29, and arrived Nov. 24.

Arnado garnered the highest number of votes and was proclaimed as the winning candidate. The
COMELEC ruled that Arnado’s continued use of his US passport is a strong indication that he had
no real intention to renounce his US citizenship and that he only executed his affidavit of
renunciation to enable him to run for office. So COMELEC annulled the proclamation of Arnado
as winning candidate.

Arnado sought reconsideration. Maquiling, another candidate for mayor garnering the second
highest number of votes, intervened. The COMELEC reversed and granted Arnado’s MR, saying
that the use of US passport does not operate to “un-renounce” what he has earlier renounced. Use
of foreign passport is not one of the grounds under S1, CA 63 thru which PH citizenship may be
lost.

Hence this petition by Maquiling questioning Arnado’s qualification, praying that he be declared
the winner.

ISSUE:
Whether Arnado’s use of his US passport after executing an affidavit of renunciation of his US
citizenship, required for his application to be a candidate for public office, in effect recants such
affidavit.
HELD: YES.
The COMELEC correctly treated the petition as one for disqualification. If a candidate is not
declared by final judgment before an election to be disqualified and he is voted for and wins,
COMELEC shall continue with his trial and may during pendency thereof order the suspension of
the proclamation of such candidate when evidence of guilt is strong. Intervention may be allowed
in disqualification proceedings even after election if there has yet been no final judgment rendered.
Thus, Maquiling has the right to intervene.

The use of foreign passport after renouncing one’s foreign citizenship is a positive and
voluntary act of representation as to one’s nationality and citizenship. It does not divest
Filipino citizenship regained by repatriation but it RECANTS the oath of renunciation
required to qualify one to run for an elective public position.

Arnado took the necessary steps to qualify to run for public office under S5(2) of RA 9225. By
taking the oath of allegiance to PH, Arnado re-acquired his PH citizenship. But he was at the time
also an American citnzen. Arnado had thus become a dual citizen. He renounced his American
citizenship by executing an affidavit of renunciation. He was deemed to be solely a Filipino citizen
regardless of the effect of such renunciation under the laws of the foreign country.

But this legal presumption does not operate permanently and is open to attack when, after
renouncing the foreign citizenship, the citizen performs positive acts showing his continued
possession of a foreign citizenship. Arnado subjected the issue of his citizenship open to attack
when, after renouncing his foreign citizenship, he continued to use his US passport to travel in and
out of the country before filing his COC.

Between April 3, 2009 when he renounced the foreign citizenship and Nov. 30, 2009 when he filed
his COC, he used his US passport four times, actions running counter to his affidavit of
renunciation. By using the US passport, he positively and voluntarily represented himself as an
American, declaring before immigration authorities of both countries that he is an American
citizen with all attendant rights and privileges granted by the US.

The renunciation of foreign citizenship is not a hollow oath that can be professed at any time only
to be violated the next day. It requires absolute and perpetual renunciation of foreign citizenship
and a full divestment of all civil and political rights granted by the foreign country. While the use
of foreign passport is not one of the acts enumerated in CA 63 constituting renunciation and
loss of PH citizenship, it is nevertheless an act which repudiates the very oath of renunciation
required. When Arnado used his US passport on April 14, 2009, 11 days after he renounced US
citizenship, he recanted his oath of renunciation that he “absolutely and perpetually renounce
all allegiance and fidelity to US” and that he “divests himself of full employment of all civil and
political rights and privileges of US.”

The act of using a foreign passport does not divest Arnado of his Filipino citizenship which he
acquired by repatriation. But by representing himself as American, he effectively reverted to his
earlier status as dual citizen. Such reversion was not retroactive. It took place the instant Arnado
used his US passport. Arnado’s category of dual citizenship is that by which foreign citizenship
is acquired thru a positive act of applying for naturalization. This is distinct from those
considered dual citizens by virtue of birth, who are not required by law to take the oath of
renunciation as the mere filing of COC already carries with it an implied renunciation of
foreign citizenship. Dual citizens by naturalization are required to take not only the oath but also
to personally renounce foreign citizenship. In effect, Arnado was exclusively a Filipino citizen
only for a period of 11 days from April 3-April 14, 2009.

Petitioner justifies the use of US passport saying that he was not notified of the issuance of his PH
passport on June 18, 2009 and obtained his PH passport only 3 months later. But Arnado used his
US passport again on Nov. 24, 2009, after he claimed to have acquired his PH passport in
September. Besides, the subsequent use of PH passport does not correct the fact that after his
renunciation of foreign citizenship and prior to filing COC, he used his US passport.

As to the issue of whether Maquiling can be declared the winner garnering the second highest
number of votes, it was held that he can. The popular vote does not cure the ineligibility of a
candidate. Maquiling is not a second-placer as he obtained the highest number of votes from
among the qualified candidates with Arnado’s disqualification. A void COC cannot produce any
legal effect. The votes cast are not considered in determining the winner. Even when the votes for
the ineligible candidate are disregarded, the will of the electorate is still respected. That the
disqualified candidate has already been proclaimed and has assumed office is of no moment.
Subsequent disqualification based on a substantive ground that existed prior to filing of COC voids
not only the COC but also the proclamation.

The oft-quoted phrase in Topacio v. Paredes that “the wreath of victory cannot be transferred from
an ineligible candidate to any other candidate when the sole question is the eligibility of the one
receiving a plurality of the legally cast ballots” is a mere obiter where the Court was comparing
the “effect of a decision that a candidate is not entitled to the office because of fraud or
irregularities in the elections … with that produced by declaring a person ineligible to hold such
an office.

The disqualifying circumstance affecting Arnado’s candidacy involves his citizenship and does
not involve election offenses in S68 of OEC, the effect of which is to disqualify the candidate or,
if already elected, from holding the office. the disqualifying circumstance affecting Arnado is his
citizenship. He was both a Filipino and American when he filed his COC. S40(d) of LGC
disqualifies dual citizens (*dual allegiance?) from running for any elective local position.

245. Renato David v. Editha Agbay, GR 199113, March 18, 2015, Villarama, Jr., J.
(Citizenship)
FACTS:
In 1974, David migrated to Canada where he became a Canadian citizen by naturalization. Upon
retirement, he and his wife returned to PH. In 2000, they purchased a 600m2 lot in Oriental
Mindoro where they constructed a residential house. But in 2004, they came to the knowledge that
the portion where they built their house was public land. In 2007, David filed a miscellaneous lease
application (MLA) over the land with DENR at CENRO. He indicated in MLA that he is a Filipino
citizen.

Private respondent Editha opposed the application on the ground that David, a Canadian citizen,
is disqualified to own land. Meanwhile, David re-acquired his Filipino citizenship under RA
9225.
David claims that at the time of his application, he had intended to re-acquire PH citizenship and
he had been assured by a CENRO officer that he could declare himself Filipino. The CENRO, in
2008, rejected David’s MLA, ruling that the subsequent re-acquisition of PH citizenship did not
cure the defect in the MLA which was void ab initio.

An information for falsification of public document was also filed against David. The MTC denied
his motion to redetermine probable cause. The RTC also denied David’s certiorari petition. Hence
this petition.

ISSUE:
Whether David, a former PH citizen who was naturalized as a Canadian citizen, who, after
returning to the PH, filed a MLA stating that he is a Filipino citizen 6 months before he reacquired
his Filipino citizenship under RA 9225, may be charged with falsification of public documents.
HELD: YES.
S2 of RA 9225 declares the general policy that Filipinos who became citizens of another country
shall be deemed not to have lost their PH citizenship “under the conditions of this Act.” S3 lays
down such conditions for 2 categories of natural born Filipinos- those who lost citizenship by
naturalization in a foreign country who shall reacquire PH citizenship upon taking the oath of
allegiance to PH and those who became foreign citizens after RA 9225 took effect, who shall
retain their PH citizenship upon taking the same oath.

The law makes a distinction between natural born Filipinos who became foreign citizens before
and after the effectivity of RA 9225. The authors of the law intentionally employed the words
“reacquire” and “retain” to describe the legal effect of taking the oath of allegiance to PH. Thus,
for those who were naturalized in a foreign country, they are deemed to have reacquired their
PH citizenship which was lost pursuant to CA 63, under which naturalization in a foreign country
is one of the ways by which PH citizenship may be lost. RA 9225 amends CA 63 by doing away
with the provision in CA 63 that takes away PH citizenship from natural borns who become
naturalized citizens of other countries and allowing dual citizenship. In the case of those who
became foreign citizens after RA 9225, they shall retain PH citizenship despite having acquired
foreign citizenship provided they took the oath of allegiance.

The law distinguishes between reacquisition and retention of PH citizenship. Sen. Drilon in the
deliberations said that reacquisition will apply to those who lost their PH citizenship by virtue of
CA 63. The retention of PH citizenship applies to future instances.

Since David was naturalized Canadian before RA 9225, he lost PH citizenship and belongs to the
first category. As the new law allows dual citizenship, he was able to reacquire his PH citizenship
by taking the required oath. RA 9225 treats those of David’s category as having already lost PH
citizenship in contrast to those natural borns who became foreign citizens after RA 9225. S2,
declaring the policy that considers Filipinos who became foreign citizens as not having lost PH
citizenship, should be read together with S3, the second paragraph of which clarifies that such
policy governs all cases after the new law’s effectivity. (*before=lost + reacquire, after=
retained)
David’s plea to adopt the interpretation most favorable to accused is misplaced. Penal statutes are
construed strictly against the state and liberally in favor of accused. but RA 9225 is not a penal
law. David made the untruthful statement in MLA, a public document, that he is a Filipino citizen
at the time of its filing when in fact he was still Canadian. The falsification was already
consummated when he reacquired his PH citizenship under RA 9225 6 months later. Hence, the
MTC did not err when it found probable cause against David.

246. Balgamelo Cabiling v. Commissioner Fernandez, GR 183133, July 26, 2010, Perez, J.
(Citizenship; elected PH citizenship under 1935, failed to register)
FACTS:
Balgamelo, Felix, Valeriano, Lechi, Arceli, Nicolas, and Isidro are the children of Felix Yao Kong
Ma, Taiwanese, and Dolores Cabiling, Filipina. Felix, Balgamelo, and Valeriano were born under
the 1935 Constitution in 1948, 1951, and 1957 respectively. They were all raised in PH and resided
here for almost 60 years. They spent their whole lives, studied, and received their education here.
They do not speak nor understand Chinese, have set foot in Taiwan, and do not know any relative
of their father. They have not traveled abroad and have raised their respective families in PH.
During their age of minority, they secured from BI their Alien Certificates of Registration (ACRs).

Upon reaching 21 yo, they claimed PH citizenship in accordance with S1(4), Art.IV of the 1935
Constitution (“those whose mothers are citizens of PH and, upon reaching the age of majority,
elect PH citizenship). Thus, they all executed affidavits of election of PH citizenship and took their
oaths of allegiance. However, they failed to have the necessary documents registered in the
civil registry as required under S1 of CA 625. It was only on July 27, 2005, 30 years after the
election, that Balgamelo and Felix did so. There is no showing that Valeriano complied with this.

They are registered voters in their barangay. Balgamelo is a barangay kagawad.

Lechi and Arceli were born in Surigao City in 1953 and 1959 respectively. The documents showing
that Arceli elected PH citizenship were registered. But no other supporting documents show that
Lechi initially obtained an ACR nor that she subsequently elected PH citizenship upon reaching
the age of majority. No document exists to provide info on the citizenship of Nicolas and Isidro.

The BI received a complaint from one Catral alleging that Felix Yao Kong Ma and his 7 children
are undesirable and overstaying aliens. The Ma family believes that the complaint was politically
motivated as they supported a candidate in Surigao in the 2004 national and local elections. The
BI charged them of violating CA 613, Philippine Immigration Act of 1940. The Board of
Commissioners of BI, composed of public respondents, ruled that they all violated CA 613. For
failing to comply with the procedure to prove a valid claim to PH citizenship, it concluded that
Felix, Balgamelo, Arceli, Valeriano, and Lechi are undocumented and improperly documented
aliens. Nicolas and Isidro, for not submitting any document to support their claim to PH
citizenship, were deemed likewise. They were all subject to deportation. They reconsidered their
judgment as to Arceli, convinced that she is an immigrant under CA 613, but denied the MR as to
the others.

The CA, upon certiorari, dismissed the certiorari.


ISSUE:
Whether persons born under the 1935 Constitution of a Filipino mother and alien Taiwanese father
who lived their lives in PH and elected upon reaching age of majority PH citizenship but failed to
register such election as required by CA 625 must still be considered aliens.
HELD: NO.
The 1935 constitution declares as PH citizens those whose mothers are PH citizens and elect PH
citizenship upon reaching the age of majority. In 1941, CA 625 was enacted, laying down the
manner of electing PH citizenship:
Section 1. The option to elect Philippine citizenship in accordance with subsection (4),
Section 1, Article IV, of the Constitution shall be expressed in a statement to be signed
and sworn to by the party concerned before any officer authorized to administer oaths, and
shall be filed with the nearest civil registry. The said party shall accompany the aforesaid
statement with the oath of allegiance to the Constitution and the Government of the
Philippines
The statutory formalities of electing PH citizenship are: 1) a statement of election under oath,
2) an oath of allegiance to the Constitution and PH government, and 3) registration of the
statement of election and of the oath with the nearest civil registry.

The 1935 Constitution and CA 625 did not prescribe a time period within which the election of
PH citizenship should be made. In Re: Application for Admission to PH Bar, Vicente Ching, we
interpreted “upon reaching the age of majority” by referring to the NCC, SOJ opinions, and Cueco
v. SOJ. The 1935 Constitution only provides “upon reaching the age of majority.” This
commenced then upon reaching 21 yo. The SOJ opinion based the period on SC decisions before
the 1935 Constitution’s effectivity. These decisions were in turn based on the pronouncements of
the Department of State of US that the election should be made within a reasonable time after
attaining the age of majority. “Reasonable time” has been interpreted to mean within 3 years
from reaching the age of majority. But in Cuenco v. SOJ, we held that this 3 year period is not an
inflexible rule. But the extention of the period to elect was not indefinite also.

In two cases, we ruled against petitioners because they belatedly complied with the requirements.
Their election and registration were all done beyond 3 years upon reaching the age of majority.
But here, petitioners complied with the first and second requirements upon reaching the age
of majority. It was only the registration of the documents of election with the civil registry that
was belatedly done. We rule that under the peculiar facts, the right to elect has not been lost and
they should be allowed to complete the statutory requirements for such election.

We are not prepared to state that the mere exercise of suffrage, being elected public official,
continuous and uninterrupted stay in PH, and other similar acts showing exercise of PH
citizenship can take the place of election of citizenship. What we now say is that where, in
petitioners’ case, the election was in fact done and documented within the constitutional and
statutory time frame, the registration of the documents of election beyond the frame should be
allowed if in the meanwhile positive acts of citizenship have publicly, consistently, and
continuously been done. The ACTUAL EXERCISE of PH citizenship for over half a century
by petitioners is actual notice to the PH public which is equivalent to formal registration of
election of PH citizenship.
To register is to record or annotate. It is made for the purpose of notification. Actual knowledge
may even have the effect of registration as to the person who has knowledge. Its purpose is to give
notice to all persons. Registration neither adds to validity nor converts an invalid instrument into
a valid one. Registration is not a mode of acquiring a right. Thus, registration is the confirmation
of the existence of a fact. Here, registration is the confirmation of election as such election. it
is NOT the registration of the act of election, although a valid requirement in CA 625, that
confers PH citizenship on petitioners. It is only a MENAS of CONFIRMING the fact that
citizenship has been claimed.

Petitioners timely took their oath. They have passed decades of their lives in PH as Filipinos. They
can have no other national identity except that which they chose upon reaching the age of reason.

We are guided by the evolvement from election of PH citizenship upon reaching age of majority
under the 1935 Constitution to dispensing with the election requirement under the 1973
Constitution to express classification of these children as natural-born under the 1987 constitution
towards the conclusion that the omission of the 1941 statutory requirement of registration of the
documents of election should not result in the obliteration of the right to PH citizenship. The
registration may still be complied with subject to administrative penalties if any.

247. Re: Application for Admission to the PH Bar, Vicente Ching, BM 914, October 1, 1999,
Kapunan, J. (Citizenship)
FACTS:
Vicente Ching is the legitimate son of Spouses Tat Ching, Chinese, and Prescila Dulay, Filipino.
He was born in La Union on April 11, 1964 and has since resided in PH. On July 17, 1998, Ching,
after having completed a Bachoelor of Laws course in St. Louis University in Baguio City, filed
an application to take the 1998 bar Exams. He was allowed to take the Bar subject to the condition
that he submit proof of PH citizenship. Ching passed the Bar.

But because of the questionable status of Ching’s citizenship, he was not allowed to take his oath.
Ching claims that he always considered himself a Filipino, registered himself as Filipino, is a CPA,
participated in electoral processes, served as Sangguniang Bayan member, elected PH citizenship
on July 15, 1999, etc.

ISSUE:
Whether Ching may be considered a Filipino citizen, having been born on April 11, 1964 under
the 1935 Constitution of a Filipino mother and Chinese father, after electing Filipino citizenship
14 years after reaching the age of majority.
HELD: NO.
The 1973 and 1987 Constitutional provisions on the election of PH citizenship should not be
understood as having a curative effect on any irregularity in the acquisition of citizenship for those
covered by the 1935 Constitution. CA 625 prescribes the procedure to be followed to make a valid
election of PH citizenship. S1 thereof states that legitimate children born of Filipino mothers may
elect PH citizenship by expressing such intention in a statement to be signed and sworn to by
the party concerned before any officer authorized to administer oath, and shall be filed with
the nearest civil registry. The party shall accompany the statement with the oath of allegiance.
The 1935 Constitution and CA 625 did not prescribe a time period within which the election of
PH citizenship should be made. The 1935 Charter only provides that the election should be made
“upon reaching the age of majority,” which commenced then upon reaching 21 yo. The SOJ
opinions based the time period on SC decisions prior to the effectivity of the 1935 Constitution. In
these decisions, the proper period for electing PH citizenship was, in turn, based on the
pronouncements of the Department of State of US to the effect that the election should be made
within a “REASONABLE TIME” after attaining the age of majority. “Reasonable time” has
been interpreted to mean that the election should be made WITHIN THREE YEARS from
reaching the age of majority. But in Cuenco v. SOJ, we held that the 3 year period is not an
inflexible rule. It may be extended under certain circumstances. But we cautioned in Cuenco
that the extention is not indefinite.

Here, Ching was already 35 yo when he complied with CA 625 on June 15, 1999, or over 14 years
after he had reached the age of majority. Based on the interpretation of “upon reaching the age
of majority,” Ching’s election was beyond, by any reasonable yardstick, the allowable period
within which to exercise the privilege. The special circumstances invoked by Ching- his
continuous uninterrupted stay in PH and being a CPA, etc.- cannot vest in him PH citizenship as
the law specifically lays down the requirements for acquisition of PH citizenship by election.
these cannot constitute what Ching erroneously labels as “informal election of citizenship.”

248. Republic v. Azucena Batuigas, GR 183110, October 07, 2013, Del Castillo, J.
(Citizenship)
FACTS:
Azucena filed a petition for naturalization before the RTC. She alleged that she believes in the
principles underlying the PH constitution, that she has conducted herself in a proper manner during
her stay in PH and in her relations with its government and the community in which she is living,
that she has mingled socially with the Filipinos and has evinced a sincere desire to learn and
embrace their customs; that she has all the qualifications in S2 and none of the disqualifications in
S4 of CA 473 etc. The OSG moved to dismiss on the ground that Azucena failed to allege that she
is engaged in a lawful occupation or in some known lucrative trade. Since the OSG did not appear
on the hearings set, Azucena was allowed to present evidence ex parte.

Azucena, born in Zamboanga del Sur on September 28, 1941 to Chinese parents, never departed
PH since birth. She can speak English, Tagalog, Visayan, and Chavacano. Her primary, secondary,
and tertiary education were taken in PH schools. She then practiced her teaching profession. She
married Santiago batuigas, a natural born Filipino citizen. They have 5 children, all of whom
studied in PH schools and are all professionals.

Azucena and her husband, as conjugal partners, engaged in the retail business of
milling/distributing corn, rice, and copra. Azucena submitted their joint tax returns as proof of
income. The business name and permits issued to them, “Azucena’s General Merchandising,” are
registered in Santiago’s name. She also submitted NBI learances to prove that she has no criminal
records.

The RTC found that Azucena has amply supported the allegations in her petition. It thus granted
the petition. In its omnibus motion, the OSG argued that the ex-parte presentation of evidence
violates S10 of CA 473 as the law mandates pubic hearing in naturalization cases. The RTC held
that the public has been fully apprised of the naturalization proceedings and was free to intervene.
The CA also dismissed OSG’s appeal. It held that the RTC complied with the law giving notice to
the OSG of the schedule of hearing. Hence this petition.

ISSUE:
Whether a Chinese wife who married a Filipino husband and started teaching in PH and
undertaking a business therein may be granted her petition for naturalization as PH citizen.
HELD: YES.
Under existing laws, an alien may acquire PH citizenship thru either judicial naturalization
under CA 473 or administrative naturalization under RA 9139 (Administrative Naturalization
Law of 2000). A third option, derivative naturalization, is available to alien women married to
Filipino husbands under S15, CA 473. This reads:
"[a]ny woman who is now or may hereafter be married to a citizen of the Philippines and
who might herself be lawfully naturalized shall be deemed a citizen of the Philippines."

Under this provision, foreign women married to PH citizens may be deemed ipso facto PH citizens
and it is neither necessary for them to prove that they possess other qualifications for naturalization
at the time of marriage nor do they have to submit themselves to judicial naturalization. The Court
declared in Moy Ya Lim Yao v. Commissioner of Immigration that an alien woman marrying a
Filipino becomes ipso facto Filipina provided she is not disqualified to be a PH citizen under S4
of CA 473. Also, if an alien woman marries an alien man subsequently naturalized, she follows
the PH citizenship of her husband the moment he takes his oath as Filipino citizen.

In Moy, the procedure for an alien wife to formalize conferment of Filipino citizenship was
explained: The alien must file a petition for cancellation of her alien certificate of registration
(ACR) alleging that she is married to a Filipino citizen and she is not disqualified from acquiring
her husband’s citizenship in S4, CA 473. Upon filing of said petition, which should be
accompanied by the joint affidavit of petitioner and her Filipino husband to the effect that
petitioner does not belong to any disqualified groups, the BI conducts an investigation and
thereafter promulgates its decision granting or denying the petition.

Records show that in Feb. 1980, Azucena applied before the then Commissioner on Immigration
and Deportation (CID) for the cancellation of her ACR by reason of her marriage to a Filipino.
The CID granted the application. But the ministry of justice set aside the CID ruling. Having been
denied of the process in CID, she was constrained to file a petition for judicial naturalization
based on CA 473. While this would not have been necessary if the process at CID was granted in
her favor, there is nothing that prevents her from seeking acquisition of PH citizenship thru
the regular naturalization proceedings available to all qualified foreign nationals. The choice of
what option to take rests in applicant. The fact that her application for derivative naturalization
under S15 was denied should not prevent her from seeking judicial naturalization under the same
law. her application at CID was denied not because she was disqualified, but because her husband’s
citizenship was not proven as only the marriage certificate was presented as evidence. Even if the
denial was based on other grounds, it is proper, in a judicial naturalization proceeding, for the
courts to determine if there are grounds to deny her of PH citizenship.
Santiago’s Filipino citizenship in the records here has been proven. His birth cert was submitted,
voter’s registration, land titles, and business registrations. He has always comported himself as
Filipino. Nothing shows that Azucena suffers from any disqualification under S4.

The OSG claims that Azucena does not have the qualification in #4, S2 of CA 473 as she does not
have any lucrative income and the proceeding in the lower court was not in the nature of a public
hearing. But the proceedings complied with the public hearing requirement as the OSG was duly
notified of the hearings and he failed to appear. Since the government who has an interest in, and
the only one who can contest, the citizenship of a person, was duly notified thru OSG, the public
hearing requirement in CA 473 is complied with.

#4, S2, CA 473 provides as qualification to become a PH citizen:


4. He must own real estate in the Philippines worth not less than five thousand pesos,
Philippine currency, or must have known lucrative trade, profession, or lawful
occupation.

Azucena is a teacher by profession and exercised her profession before she had to quit her
teaching job to assume her family duties and take her role as joint provider with her husband.
Together, the couple raised all 5 children, provided them with education, and have all become
professionals and responsible citizens of PH. Certainly, this is proof enough of both husband
and wife’s lucrative trade. Azucena is a professional and can resume teaching at any time. This
is more than sufficient guarantee that she will not be a charge to the only country she has known
since birth.

The main objective of extending the citizenship privilege to an alien wife is to maintain a unity
of allegiance among family members.

While there is no proceeding authorized by law or the RoC for judicial declaration of citizenship
of an individual, this case is not a petition for judicial declaration but for judicial naturalization. In
the first, the petitioner believes that he is a Filipino citizen and asks a court to declare or confirm
his status as a PH citizen. In the second, petitioner acknowledges he is an alien and seeks judicial
approval to acquire the privilege of becoming a PH citizen based on requirements under CA 473.

249. Moy Ya Lim Yao v. Commissioner of Immigration, GR L-21289, October 04, 1971,
Barredo, J. (Citizenship)
FACTS:
Petitioners sought injunction with CFI against Commissioner of Immigration (CI) to restrain him
from ordering plaintiff Lau Yuen Yeung to leave PH and causing her arrest and deportation. Lau
had applied for a passport visa to enter PH as non-immigrant. Lau stated that she was a Chinese
residing at HK and she desired to take a pleasure trip to PH to visit her grand uncle Ching for 1
month. She was permitted to come on March 13, 1961 and to stay for 1 month until April 13, 1961.
After repeated extensions, Lau was allowed to stay up to February 13, 1962. On January 25, 1962,
she married Moy Ya Lim alias Edilberto Lim, a Filipino. Because of the contemplated action of
CI to order her immediate deportation after the expiration of her authorized stay, she filed the
petition for injunction. It was admitted in the hearing that Lau could not write English or Tagalog
and, except for a few words, could not speak them. She could not name any Filipino neighbor and
did not know the names of her brothers and sisters-in-law.

The CFI denied injunction. It said that S15 of the Revised Naturalization Law requires an alien
woman to possess all the qualifications and none of the disqualifications in the law before she can
be considered a Filipino citizen for marrying a Filipino. The marriage was made merely for
convenience to avoid the deportation.

ISSUE:
Whether an alien woman, Lau Yeung, who marries a Filipino citizen, Moy Ya Lim, may be
considered ipso facto a Filipino citizen upon showing only that he is not disqualified under CA473.
HELD: YES.
S15 of CA 473 reads: 'Any woman who is now or may hereafter be married to a citizen of the
Philippines, and who might herself be lawfully naturalized shall be deemed a citizen of the
Philippines.' Thus, it excludes those disqualified from being naturalized citizens under S4.
Evidence to the effect that the woman is not disqualified may be presented in the action to recover
her bond confiscated by CI and upon proof of such fact, she may be recognized as Filipina. This
was modified in Lee Suan Ay, where it was held instead that Lee must possess the qualifications
required by law to become Filipino by naturalization and none of the disqualifications therefor.
This is a significant modification of the construction of the law since in Ly Giok Ha, the case relied
on the SOJ opinion stating that the phrase “who might herself be lawfully naturalized” should be
construed as not requiring the woman to have the qualifications of residence, good character
etc. as in naturalization by judicial proceedings, but merely that she is of the race by persons
who may be naturalized. (*no disqualification)

In La San Tuang v. Galang, the issue was squarely whether in addition to not having any
disqualification in S4, whether the woman should also possess all the qualificatiosn of residence,
moral character, knowledge of a native principal dialect, etc. provided by law. It was held there
that both qualifications and none possession of any disqualification must be shown. A person who
is not disqualified is not necessarily qualified to become a PH citizen, because the law treats
qualifications and disqualifications in separate sections. It was held in Sun Peck Yong v. CI that
the naturalization of the husband does not automatically make the wife Filipino also as it must be
shown that she also possesses all the qualifications and none of the disqualifications to become a
PH citizen.

Here, the Court is called upon to rule on the same issue. S15, CA 473 reads:
"SEC. 15. Effect of the naturalization on wife and children. — Any woman, who is now or
may hereafter be married to a citizen of the Philippines, and who might herself be lawfully
naturalized shall be deemed a citizen of the Philippines.
"Minor children of persons naturalized under this law who have been born in the
Philippines shall be considered citizens thereof. xxx”

Is it necessary so that an alien woman who marries a Filipino or who is married to a man who
subsequently becomes Filipino, may become Filipino herself, that, aside from not suffering from
any disqualification, she must also possess all the qualifications? While the cited jurisprudence
would make inevitable an affirmative answer. But there are other circumstance which make it
necessary that the Court take up the matter anew.

S15 of CA 473, Naturalization Law, has been taken directly, copied, and adopted from its
American counterpart. It is nothing less than a reenactment of the American provision. The
reasoning in Ly Giok Ha requiring both qualifications and no disqualifications was that otherwise,
the policy of the law, CA 473, might be violated. The wife might be a believer in racial supremacy,
in government by certain selected classes, yet she would not be disqualified under S4 as long as
she is not “opposed to organized government.” But a closer study reveals relevant considerations
adversely affecting the premises on which the reasoning is premised.

1. The legislators, in reenacting thru S15 of CA 473 S13(a) of Act 2927 (first naturalization law)
which was copied from S1994 of the Revised Statutes of US before its repeal in 1922, in effect
incorporated into the provision the construction given to it by the American courts of “who might
herself be lawfully naturalized”. The constructions in US stated that “it was not necessary that it
should appear affirmatively that she possessed the other qualifications at the time of her marriage,”
“upon her marriage, she became ipso facto a citizen of the US as fully as if she had complied with
all of the provisions of the statutes upon the subject of naturalization.”

We have examined all leading American decisions on the subject and found no warrant for the
proposition that “who might herself be lawfully naturalized” was meant solely as a racial bar
(*which was the premise nung SC decisions saying na both qualification and disqualification
dapat).

2. In Lo San Tuang, Choy King Tee, and Ly Giok Ha, the Court inferred that because S1 of Act
2927 was eliminated by CA 473, it follows that in place thereof particularly its subdivision (c),
being the criterion of whether an alien wife may be lawfully naturalized, what should be required
is that not only she be not disqualified but also must possess all the qualifications in S2 like age,
residence, etc. thereby implying that S2 was purposely intended to take the place of S1, Act 2927.
But such inference is not sufficiently justified.
"SECTION 1. Who may become Philippines citizens. — Philippine citizenship may be
acquired by: (a) natives of the Philippines who are not citizens thereof under the Jones
Law; (b) natives of the other Insular possessions of the United States; (c) citizens of the
United States, or foreigners who under the laws of the United States may become citizens
of said country if residing therein." (S1, Act 2927)

Nothing in the legislative history indicates that such was the clear intent of the legislature. What
is clear is that S15 is an exact copy of S1994 of the Revised Statutes of US. Pertinent American
decisions agree that in the construction of S1994, the qualifications of residence, good moral
character etc. are not supposed to be considered and that the only eligibility to be taken into
accouns is that of the race or class to which the subject belongs. The views sustaining the contrary
conclusion appear to be based on inaccurate factual premises related to the real legislative
background of the framing of our naturalization law presently.

When CA 473 copied S1994, it had already been repealed by the Americans opting instead to
compel alien wives to undergo judicial naturalization. The lawmakers had two choices- to adopt
S1994 with its settled construction and the other to follow the new posture of the Americans
requiring judicial naturalization. They opted for the first, thus our law still follows the old
American law on the subject. It would thus violate STATCON principles to state that S15 has a
nationalistic and selective orientation and that it should be construed independently of the previous
American posture because of the difference of the circumstances here and in US.

3. The qualifications have always been considered disqualifications in the sense that those who
did not possess them were the ones who could not “be lawfully naturalized” just as if they were
suffering from any of the disqualifications. Why should their elimination then be not viewed as a
subtraction from or a lessening of the disqualifications?

4. Practical considerations militate towards the same conclusions. Requiring the alien wife of a
Filipino to prove that she possesses the qualifications is unreasonably onerous. For example, one
requirement is 10 year residence. If she married a Filipino, she cannot acquire the citizenship of
her husband for 10 years. If her country’s law declares that upon her marriage to a foreigner she
automatically loses her citizenship, she would be stateless for 10 years. It would be more difficult
for an alien wife related by marriage to a Filipino to become a citizen than a foreigner who is not
so related. But the purpose of S15 par.1 was to accord an alien woman, by reason of marriage to a
Filipino, a privilege not similarly granted to other aliens. If the qualifications were also required,
the privilege granted to alien wives would become illusory. Such construction is contrary ot the
manifested object of the statute.

Thus, we now hold that all previous decisions of this Court indicating otherwise notwithstanding,
under S15 of CA 473, an alien woman marrying a Filipino, native born or naturalized, becomes
IPSO FACTO a Filipina provided she is NOT DISQUALIFIED to be a citizen of the PH
under S4 of CA 473.

5. May Lau be deemed a Filipina without submitting to a naturalization proceeding? No doubt


whatever is entertained as to the point that minor children falling within the conditions of place
and time of birth and residence in the provision are vested with PH citizenship directly by
legislative fiat without need for any judicial declaration.
Section 15- xxx
"A foreign-born minor child, if dwelling in the Philippines at the time of naturalization of
the parents, shall automatically become a Philippine citizen, and a foreign-born minor
child, who is not in the Philippines at the time the parent is naturalized, shall be deemed a
Philippines citizen only during his minority, unless he begins to reside permanently in the
Philippines when still a minor, in which case, he will continue to be a Philippine citizen
even after becoming of age.
"A child born outside of the Philippines after the naturalization of his parent, shall be
considered a Philippine citizen, unless within one year after reaching the age of minority,
he fails to register himself as a Philippine citizen at the American Consulate of the country
where he resides, and to take the necessary oath of allegiance."
But it is claimed that the same expression “shall be deemed a citizen of PH” in reference to the
wife does not connote vesting of citizenship by legislative fiat as the antecedent phrase requiring
that she must be one :who might herself be lawfully naturalized” implies that such status is
intended to attach only after judicial naturalization.
It is within the power of Congress to vest citizenship by legislative fiat. The issue is whether the
legislature has done so in S15. One of the most respected authorities on political law, Dr. Sinco,
observes that in PH, a foreign woman married to a Filipino becomes ipso facto naturalized, if she
belongs to any of the classes who may apply for naturalization under PH laws.

S1994 of the Revised Statutes of US from where S15 was as aforediscussed copied was constructed
as to make an alien woman who married a citizen also a citizen by force of law and without
having to undergo any naturalization proceedings provided that she was not disqualified. When
it repealed this, it explicit required such alien wives to submit to judicial naturalization albeit under
more liberal terms than for other applicants for citizenship. And as mentioned, the PH legislature,
instead of following suit, copied S1994.

250. Edison So v. Republic, GR 170603, January 29, 2007, Callejo, Sr., J. (Citizenship)
FACTS:
Edison So filed with RTC a petition for naturalization under CA473, Revised Naturalization Law.
He alleges that he was born in Manila. He is a Chinese citizen and lived in Binondo since birth.
As an employee, he derives an average of P100k income annual with free board, lodging, and other
benefits. He is single, able to speak and write English, Chinese, and Tagalog. He studied in a PH
school recognized by PH government where PH history, government, and culture are taught. He
is of good moral character. He believes in the principles of the PH Constitution. He has mingled
socially with the Filipinos and evinced a sincere desire to embrace the Filipino customs and
traditions. He has all the qualifications in S2 and none of the disqualifications in S4 etc. etc.

The petition was published once a week for 3 consecutive weeks and posted in public and
conspicuous places in the Manila City Hall as ordered by RTC. No one opposed. Two witnesses
and So himself testified for So. The RTC granted the petition granting naturalization. Republic
thru OSG appealed to CA.

The OSG insists that his 2 witnesses did not know him well enough to vouch for his fitness to
become Filipino and gave only general statements about his character. He thus failed to prove that
he had all qualifications and none of the disqualifications in CA 473. The CA set aside the RTC
ruling and dismissed the petition for naturalization without prejudice, saying that the two witnesses
were not credible and merely “parroted” the provisions of the Naturalization Act without
explaining their applicability to So’s case. His age was also only 20y9m25d, short of the 21y
requirement.

Hence this petition.

ISSUE:
Whether Edison So’s application for judicial naturalization may be granted based on the testimony
of his witnesses consisting of general statements on his good moral character and enumerating the
requirements in the law.
HELD: NO.
Naturalization signifies the act of formally adopting a foreigner into the political body of a nation
by clothing him or her with the privileges of a citizen. Currently, there are 3 ways by which an
alien may become a citizen by naturalization: 1) administrative naturalization pursuant to RA
9139, 2) judicial naturalization pursuant to CA 473, and 3) legislative naturalization in the form
of a law enacted by Congress bestowing PH citizenship to an alien.

So’s contention that the qualifications in RA 9139 should apply and not CA 473 is barren of merit.
The qualifications and disqualifications of an applicant for naturalization by judicial act are set
forth in S2 and 4 of CA 473. On the other hand, S3 and 4 of RA 9139 apply to naturalization by
administrative act.

RA 9139 was enacted as a remedial measure intended to make the process of acquiring PH
citizenship less tedious, less technical, and more encouraging. It addresses the concerns of degree
holders who, due to lack of citizenship, cannot practice their profession, promoting “brain gain”
for PH.

CA 473 and RA 9139 are separate and distinct laws. CA 473 covers all aliens regardless of class
while RA 9139 covers native-born aliens who lived in PH all their lives, who never saw any
other country and all along thought they were Filipinos, who have demonstrated love and
loyalty to PH and affinity to the customs and traditions. There is nothing in RA 9139 signifying
an intent to amend or repeal CA 473. The legislature merely prescribed another mode of acquiring
PH citizenship by native born aliens, who can thus apply for judicial or administrative
naturalization.

Here, So applied for naturalization by judicial act, though at the time of filing the petition
administrative naturalization in RA 9139 was already available. Thus, his application is governed
by CA 473. If RA 9139 would be made applicable to judicial naturalization, the coverage would
be broadened as it would apply to aliens not native born. It would also be contrary to the intention
of the legislature to liberalize naturalization in the country as an alien not qualified under RA 9139
may be naturalized under CA 473.

In any event, So failed to prove that his witnesses were competent to vouch for his good moral
character or are themselves possessed of such. Character witnesses in naturalization proceedings
stand as insurers of the applicant’s character. Thus, they ought to testify on specific facts and
events justifying inference that applicant has all qualifications and no disqualifications. His
witnesses’ testimonies consisted mainly of general statements in answer to leading questions by
his counsel. They enumerated the qualifications in the law without specific details. The witnesses
clearly did not personally know him well enough.

In naturalization proceedings, the applicant must prove not only his own good moral character but
also that of his witnesses, who must be credible persons. Within the purview of naturalization, a
“credible person” is not only one not previously convicted of a crime etc. What must be credible
is not the declaration but the person making it. The records do not show that his witnesses are
persons of good standing in the community, honest and upright, or reputed to be trustworthy and
reliable.

The failure of OSG to oppose the petition does not preclude it from questioning the RTC decision.
A naturalization proceeding is not a judicial adversary proceeding and the decision rendered
therein does not constitute res judicata. A certificate of naturalization may be cancelled if it is
subsequently discovered that applicant obtained it by misleading the court upon any material fact
or based on conditions arising subsequent to the granting of the certificate.

251. Ciceron Altarejos v. COMELEC, GR 163256, November 10, 2004, Azcuna, J.


(Citizenship)
FACTS:
Altajeros was a candidate for Mayor in Masbate in the May 10, 2004 elections. Private respondents
Jose and Vernon, registered voters of Masbate, filed with COMELEC a petition to disqualify and
to cancel Altarejos’ COC on the ground that he is not Filipino and that he made a false
representation in his COC when he said that he was not a permanent resident of or immigrant to a
foreign country. He is a holder of a permanent US resident visa, an alien cert of registration, and
an immigration cert.

Altarejos claims that he was already issued a cert of repatriation by the Special Committee on
Naturalization after he filed a petition for repatriation pursuant to RA 8171. He thus claims that
his Filipino citizenship was already restored. The COMELEC disqualified Altarejos and denied
his MR. On May 10, 2004, the election day itself, Altarejos filed this petition.

The OSG says that Altarejos’ name as mayor candidate was retained, but his opponent was already
proclaimed as duly elected mayor. The petition is thus moot. Altarejos claims that COMELEC’s
resolutions disqualifying him adversely affected his candidacy as his supporters were made to
believe that his votes would not be counted and that the resolutions cast a doubt on his PH
citizenship. He says that he took his oath of allegiance and was elected mayor during the 1998
elections. If there was delay in the registration of his cert of repatriation with BI and the civil
registry, the same was brought about by the inaction of said offices as the records of the Special
Committee on Naturalization show that his cert was already transmitted to those office.
ISSUE:
Is the registration of Altarejos’ repatriation cert with the proper civil registry and with BI a
prerequisite in effecting repatriation?
HELD: YES.
S2 of RA 8171 provides:
SEC. 2. Repatriation shall be effected by taking the necessary oath of allegiance to the
Republic of the Philippines and registration in the proper civil registry and in the Bureau
of Immigration. The Bureau of Immigration shall thereupon cancel the pertinent alien
certificate of registration and issue the certificate of identification as Filipino citizen to the
repatriated citizen.

The law is clear that repatriation is effected by taking the oath and registration. Thus in addition
to taking the oath, the registration of the repatriation cert is a prerequisite in effecting repatriation.

Here, Altarejos took his oath on Dec.17, 1997, but his cert of repatriation was registered with the
civil registry of Makati only after 6 years or on Feb. 18, 2004 and with the BI on March 1, 2004.
Thus, he complied with the requirements of repatriation only after he filed his COC, but before
elections.
The citizenship qualification for elective office must be construed as “applying to the time of
proclamation of the elected official and at the start of his term.” Under S39 of LGC, an elective
local official must be a citizen, registered voter in the locality where he intends to be elected, a
resident therein for 1 year preceding the day of election, able to read and write Filipino or other
local language, and candidates for governor must be at least 23yo no election day. The law does
not specify any particular date or time when the candidate must possess citizenship. The
purpose of the citizenship qualification is to ensure that no alien shall govern our people. An
official begins to govern only upon proclamation and on the day the law mandates his term of
office to begin. The purpose is achieved by construing the citizenship qualification as applying to
the time of proclamation and at the start of his term.

It has also been held that repatriation retroacts to the time of filing of the application. RA 8171
has impliedly repealed PD 725 as they cover the same subject matter- repatriation of Filipino
women who lost their citizenship by marriage to aliens and of natural born Filipinos. Thus,
Altarejo’s repatriation retroacted to the date he filed his application in 1997. He was thus
qualified to run for a mayoralty position in the May 10, 2004 elections.

252. Joevanie Tabasa v. CA, GR 125793, August 29, 2006, Velasco, Jr., J. (Citizenship)
FACTS:
Joevanie Tabasa was a natural born citizen of PH. In 1968, when he was 7yo, his father Rodolfo
became a naturalized citizen of US. By derivative naturalization, Tabasa also acquired American
citizenship. Tabasa arrived in PH on August 3, 1995 and was admitted as balikbayan for 1 year.
Thereafter, Tabasa was arrested and detained by BIR on May 23, 1996. Tabasa was investigated
and accused of violating S8, Chapter 3, Title 1, Book 3 of the 1987 Admin Code. He was subject
to deportation to US as the BI was informed that his US passport was revoked by US for “unlawful
flight to avoid prosecution” in violation of the US Code, S1073.

Tabasa filed with the CA a petition for habeas corpus alleging that he was not afforded due process.
At the time of filing, he was already 35 yo. On June 13, 1996, he filed a supplemental petition
alleging that he had acquired Filipino citizenship by repatriation in accordance with RA 8171
and thus cannot be deported anymore. The CA denied Tabasa’s petition on the ground that he had
not legally acquired his Filipino citizenship under RA 8171 as although he became an American
citizen by derivative naturalization, there was no evidence to show that he lost his PH citizenship
“on account of political or economic necessity” as provided in RA 8171 S1. The affidavit does not
state that political or economic necessity was the compelling reason for his parents to give up their
Filipino citizenship in 1968.

ISSUE:

HELD:
RA 8171, “An Act Providing for the Repatriation of Filipino Women Who Have Lost Their PH
Citizenship by Marriage to Aliens and of Natural Born Filipinos” was enacted on Oct. 23, 1995. It
only provides for the repatriation of two classes of persons: 1) Filipino women who lost their PH
citizenship by marriage to aliens and 2) Natural born Filipinos including minor children who
lost their PH citizenship on account of political or economic necessity. Tabasa claims that he
could be repatriated under RA 8171 because he is a child of a natural born and he lost his PH
citizenship by derivative naturalization when he was a minor.

He overlooks the fact that RA 8171’s repatriation is available only to natural borns who lost
citizenship on account of political or economic necessity and to their minor children. This
includes a situation where a former Filipino who subsequently has children while naturalized as
citizen of a foreign country, and his repatriation will automatically vest PH citizenship on his
children of jus sanguinis or blood relationship. The children acquire the citizenship of their parents
who are natural born. The children must be minor at the time the petition for repatriation is filed
by the parent because the child does not have the legal capacity for all acts of civil life much less
to undertake a political act like election of citizenship.

Here, Tabasa was Filipino at birth. While still a minor, he acquired thru derivative naturalization
US citizenship. He lost his PH citizenship by operation of law and not due to political or
economic exigencies. It was his father who could have been motivated by economic or political
reasons in deciding to apply for naturalization. It was his parents’ decision and not his. The
privilege of repatriation under RA 8171 is extended to natural borns who could prove that they
acquired foreign citizenship due to political and economic reasons and extended indirectly to the
minor children at the time of repatriation. Thus, he is not qualified to avail of repatriation under
RA 8171 but can possibly reacquire PH citizenship under RA 9225 by simply taking an oath of
allegiance to PH.

Even if we concede that he can avail of RA 8171, he failed to follow the procedure for reacquisition
of PH citizenship. He has to file his petition for repatriation with the Special Committee on
Naturalization (SCN) designated to process petitions for repatriation. In the SCN rules, applicants
are required to submit documents like birth cert and other evidence proving their claim to
Filipino citizenship. Tabasa merely took his oath then executed an affidavit of repatriation which
he registered with his birth cert with the civil registrar. The registry issued him a certificate when
the SCN was already in place. He should have filed his petition before SCN.

He also failed to prove that his parents relinquished PH citizenship on account of political or
economic necessity as provided in the law. He claims that “economic and political reasons” is
merely descriptive and not restrictive. But the legislature’s intent is shown in S1 of RA 8171 to
limit the benefit of repatriation only to natural borns who lost PH citizenship due to such reasons.

While it is true that renunciation of allegiance to one’s native country is necessarily a political act,
it does not follow that the act is inevitably politically or economically motivated. There are other
reasons why Filipinos relinquish PH citizenship like to escape punishment for crime or those who
really feel that they are not Filipinos and that they deserve a better nationality. Thus, Tabasa must
prove the reason for the loss of citizenship.

Tabasa, whose passport was cancelled after admission to PH, became an undocumented alien
who can be summarily deported.

Das könnte Ihnen auch gefallen